You are on page 1of 716

17 chapters aligned

as per class 11 & 12


Syllabus

Er. Deepak Agarwal


• Head Office : B-32, Shivalik Main Road, Malviya Nagar, New Delhi-110017

• Sales Office : B-48, Shivalik Main Road, Malviya Nagar, New Delhi-110017
Tel. : 011-26691021 / 26691713

How to access the ebook(s)?


Educore

1 . Mail your Order ID at ebooks.support@aiets.co.in to get the 16 digit Access Code.

2 . Go to www.educoreonline.com/register.htm

3 . Enter your details along with your 16 digit Access Code.

4 . Click Register & you would be successfully redirected to the Login Page.

Note: If you are already registered with us, you just have to login

(http://www.educoreonline.com/login.htm) & enter your new 16 Character


Unique Code under your Account Section.

5 . Login with your registered email ID & password.


Typeset by Disha DTP Team 6 . You can now view you e-book(s) under your Library.

7 . You can read your e-books either Online or Offline. For offline, simply download
our Educore App once & download the e-books inside the app. Educore App is
available for Windows Desktop, IOS & Android.

8 . Educore works best in Desktop, Laptop, 7" & 10" tablets.

9 . Contact us at support@educoreonline.com for any further assistance.

Note: This app is not accessible on Mobiles.

DISHA PUBLICATION
ALL RIGHTS RESERVED

© Copyright Publisher
No part of this publication may be reproduced in any form without prior permission of the publisher. The author and the
publisher do not take any legal responsibility for any errors or misrepresentations that might have crept in. We have tried
and made our best efforts to provide accurate up-to-date information in this book.

For further information about books from DISHA,


Log on to www.dishapublication.com or email to info@dishapublication.com
PREFACE TO THE NEW EDITION
THE NEW CHALLENGER FOR THE NEW JEE
It gives me immense pleasure in introducing the revised and updated edition of the Physics Challenger.
This challenger book has evolved over a period of time and adopted itself to the changing IIT-JEE
pattern. The book is enriched with the introduction of a lot of new and innovative questions in each
variety of questions in each of the chapters.
The book has been reoriented and aligned as per the latest CBSE board pattern - Class XI and XII. I have
tried to rename the chapters as per the latest NCERT books, such that the class XI & XII students can
easily attempt their respective chapters. The book provides response bubbles/grids to mark answers,
of the questions on respective pages of each chapter. This will make the book more useful for the
students as they can mark their responses and need not check the answers one by one.
The book contains 17 chapters in all. Each chapter covers exhaustive collection of challenging problems
of all the varieties being asked in the latest JEE Examinations.
• MCQ one option correct
• MCQ more than one correct option(s)
• Passage based question
• Reasoning / Assertion type questions
• Column Matching Type Questions
• Numeric / Integer Answer Based Subjective Problems
This new book contains the collection of the best possible problems for IIT-JEE. I have really worked
very hard to create new problems on Comprehension based questions, Matching Type Questions,
Assertion Reasoning and Integer answer type Questions.
I can assure all the aspirants that I have made an attempt to provide the best possible material for JEE.
I hope that the users will find this book to stand upto their expectations. In the end I would like to
thank family, friends, publishers and colleagues, who have helped me in various forms in completing
this project. I would specially like to thank the Editorial team of Disha Publication comprising of
Mr. Sunil Kumar, Mr. Sanjay Kumar for helping in bringing the book to the present shape. I would also
like to thank Mr. Sandeep and Mr. Nitin for helping me in the proof reading work. And last but not the
least I would like to specially thanks Prof. Ragvindra of Narayana (Hyderabad) without whose help this
work would not have been possible. However, all these hardships will look small if readers find this
effort useful to them. I shall be highly thankful to all those readers who let me know, their views about
the book, any kind of error(s) crept inadvertently and any new problems if he/she wants to see it/
them included in forthcoming editions.

Author
CH. 1 GENERAL PHYSICS (Units, Dimensions, Errors & General Experiments) 1-20

CH. 2 MOTION IN ONE AND TWO DIMENSIONS 21-78

CH. 3 LAWS OF MOTION 79-134

CH. 4 WORK, POWER, ENERGY & CONSERVATION LAWS 135-182

CH. 5 CIRCULAR & ROTATIONAL DYNAMICS 183-252

CH. 6 GRAVITATION 253-282

CH. 7 PROPERTIES OF MATTER & FLUID MECHANICS 283-324

CH. 8 SIMPLE HARMONIC MOTION 325-356

CH. 9 WAVES 357-394

CH. 10 HEAT AND THERMODYNAMICS 395-438

CH. 11 ELECTROSTATICS 439-492

CH. 12 CURRENT ELECTRICITY 493-528

CH. 13 MAGNETIC EFFECT OF CURRENT 529-564

CH. 14 ELECTROMAGNETIC INDUCTION & AC CURRENT 565-610

CH. 15 RAY OPTICS 611-646

CH. 16 WAVE OPTICS 647-674

CH. 17 MODERN PHYSICS 675-712


1. A physical quantity Q is related to four observables x, y, z 6. The length and breadth of a rectangle are (5.7 ± 0.1) cm and
and t by the relation (3.4 ± 0.2) cm. The area of the rectangle with error limits is
(a) (19.0 ± 1.5) sq.cm. (b) (19.0 ± 2.5) sq.cm.
x2 / 5 z3
Q= (c) (19.0 ± 3.5) sq.cm. (d) (19.0 ± 4.5) sq.cm.
y t
The percentage errors of measurement in x, y, z and t are V
7. A quantity X is given by 0L where 0 is the
2.5%, 2%, 0.5% and 1% respectively. The percentage error t
in Q will be permittivity of the free space, L is a length, V is a potential
(a) 5% (b) 4.5% difference and t is a time interval. The dimensional formula
(c) 8% (d) 7.75% for X is the same as that of
2. If area (A), velocity (v) and density ( ) are base units, then (a) resistance (b) charge
the dimensional formula of force can be represented as (c) voltage (d) current
(a) Av (b) Av² 8. When a small sphere moves at low speed through a fluid,
(c) Av ² (d) A²v the viscous force F, opposing the motion is experimentally
1 found to depend upon the radius r, the velocity v of the
2
3. The dimension of ÷ 0E ( 0 : permittivity of free space, sphere and the viscosity of the fluid. Expression for force
2
is
E electric field) is
(a) 4 rv2 (b) 5 r2v
(a) MLT–1 (b) ML2T–2
–1 –2 (c) 2 r2v (d) 6 rv
(c) ML T (d) ML2T–1
9. A gas bubble from an explosion under water, oscillates
4. The resistance R of a wire is given by the relation
with a period T proportional to pa db Ec where p is the
R= . Percentage error in the measurement of , and static pressure, d is the density of water and E is the total
r2 energy of explosion. Find the values of a, b and c.
r is 1%, 2% and 3% respectively. Then the percentage
(a) –5/6, 1/2, 1/3 (b) –5/6, 1/3, 1/2
error in the measurement of R is
(a) 6 (b) 9 (c) –1/6, 1/3, 1/4 (d) –1/3, 1/3, 1/4
3
(c) 8 (d) 10
10. Let y 2
where = 2.0 ± 0.1, z = 1.0 ± 0.1 then
5. If force, acceleration and time are taken as fundamental z
quantities, then the dimensions of length will be the value of y is given by
(a) FT2 (b) F–1 A2 T–1 (a) + 2 ± 0.8 (b) – 4 ± 1.6
(c) FA2T (d) AT2 (c) – 4 ± 0.8 (d) None of these

MARK YOUR 1. 2. 3. 4. 5.
RESPONSE 6. 7. 8. 9. 10.
2 IIT-JEE PHYSICS Challenger
11. A cube has a side of length 1.2 × 10–2m. Calculate its volume. 18. To find the value of g using simple pendulum, T = 2.00
(a) 1.7 × 10–6 m3 (b) 1.73 × 10–6 m3 second and = 50cm was measured. The maximum
(c) 1.70 × 10 m –6 3 (d) 1.732 × 10–6 m3 permissible error in g is :
12. Given that ( /p ) = az/KB where p is pressure, z is
distance, KB is Boltzmann constant and is temperature, ( T = ± 0.01, = ± 0.1 )
the dimensions of are (a) 1.4% (b) 1.1%
(a) [L0M 0 T 0] (b) [L1M –1T 2]
2
(c) [L M T ]0 0
(d) [L–1 M 1T –2] (c) 1.5% (d) 1.2%
13. If E is the electric field intensity and 0 is the permeability 19. A wire has a length = 6 ± 0.06 cm, radius r = 0.5 ± 0.005 cm
of free space, then the quantity E2/ 0 has the dimensions and mass m = 0.3 ± 0.003 gm. Maximum percentage error in
of density is
(a) [M 0L1T –2] (b) [M 1L1T –4] (a) 4 (b) 2
(c) [M1L0T –4] (d) [M 2L2T 0]
(c) 1 (d) 6.8
14. The speed of light in vacuum, c, depends on two
fundamental constants, the permeability of free space, 0 20. The frequency ( f ) of a wire oscillating with a length , in p
and the permittivity of free space, 0. The speed of light is
p T
1 loops, under a tension T is given by f = where
given by c = . The units of are N–1C2m–2. The 2
0
0 0
units for 0 are µ = linear density of the wire. If the error made in determining
(a) kg–1m–1C2 (b) kg mC –2 length, tension and linear density be 1%, –2% and 4%,
(c) kg ms–4C–2 (d) kg–1s–3C–2 then find the percentage error in the calculated frequency
is
z
15. Pressure depends on distance as, P exp ÷ , (a) – 4% (b) – 2%
k
where , are constants, z is distance, k is Boltzmann’s (c) –1% (d) –5%
constant and is temperature. The dimensions of are 21. The density of a sphere is measured by measuring its mass
(a) M0L0T0 (b) M–1L–1T –1 and diameter. If, it is known that the maximum percentage
0
(c) M L T 2 0 (d) M–1L1T 2 errors in the measurement are 2% and 3%, then find the
16. The velocity v of surface waves on a liquid may be related maximum percentage error in the measurement of density?
to their wavelength , the surface tension of liquid and (a) 15% (b) 18%
its density by the following equation : v = k
where k is a dimensionless constant. The values of , (c) 9% (d) 11%
and should respectively be given as 22. Which of the following set have different dimensions?
1 1 1 (a) Pressure, Young’s modulus , Stress
(a) , , (b) – 1, +1 – 1
2 2 2 (b) EMF, Potential difference, Electric potential
1 1 1 (c) Heat, Work done, Energy
(c) + , ,+ (d) + 1 –1 + 1
2 2 2 (d) Dipole moment, Electric flux, Electric field
17. A physical quantity is calulated using the formula 23. While measuring length of an object it was observed that
1 2 1/ 3 the zero of the vernier lies between 1.4 and 1.5 of the main
= xy / z , where x, y and z are experimentally scale and the fifth vernier division coincides with a main
10
measured quantities. If the fractional error in the scale division. If the length of the object measured is ,
measurement of x, y and z are 2 %, 1% and 3% respectively, then the value of ( – 1.4) in terms of the least count C of
then the fractional error in will be the instrument is
(a) 0.5% (b) 5% (a) C (b) 1.45 C
(c) 6% (d) 7%
(c) 4 C (d) 5 C

11. 12. 13. 14. 15.


MARK YOUR
16. 17. 18. 19. 20.
RESPONSE
21. 22. 23.
GENERAL PHYSICS 3

24. In a screw gauge, the zero of mainscale coincides with fifth (b) Spherometer and screw gauge
division of circular scale in figure (i). The circular division (c) Slide callipers and spherometer
of screw gauge are 50. It moves 0.5 mm on main scale in (d) Slide callipers, screw gauge and spherometer
one rotation. The diameter of the ball in figure (ii) is 29. Intensity observed in an interference pattern is
0 10 I I 0 sin 2 . At = 30° intensity I = 5 ± 0.0020 W/m2.
5
0 Find percentage error in angle if I0 = 20 W/m2.
4 2
(a) 3 10 2 % (b) 3 10 2 %

1 3
(c) 3 10 2 % (d) 3 10 2 %
Figure (i)
30. The momentum of an electron in an orbit is h/ where h is
0 30 a constant and is wavelength associated with it. The
25
20 nuclear magneton of electron of charge e and mass me is
eh
given as µn = . The dimensions of µn is
3672 me
(A current)
Figure (ii) (a) [ML2A] (b) [ML3A]
2
(c) [L A] (d) [ML2]
(a) 2.25 mm (b) 2.20 mm
(c) 1.20 mm (d) 1.25 mm 31. In an experiment to determine the inertial mass of an object
25. The force experienced by a particle is F0 when its speed using Newton’s second law, following graph is obtained
is v = v0. If the speed varies with displacement as v = kx, between net force on the object and the acceleration
then the force acting on the particle when its speed produced in it. The mass of the object within error limits is
becomes 2v0 is a(m/s2)
(a) 4F0 (b) F0/2
(c) 2F0 F02
(d)
3
26. 49 divisions on the vernier scale coincides with 50 divisions
on the main scale of a vernier calliper. The least count of
the instrument is, if graduation on the main scale is 2 mm 2
1 1
(a) mm (b) mm 1
25 50
2 1 F(N)
(c) mm (d) mm
49 49 1 2 3
27. A formula is given as (a) 1.0 kg (b) 1 kg
(c) (1.0 ± 0.1) kg (d) (1.0 ± 0.2) kg
b k . .t 3 32. A physical quantity A is dependent on other four physical
P= 1+
a m.a
where P = pressure; k = Boltzmann’s constant; pq
quantities p, q, r and s as given below A = . The
= temperature; t = time; ‘a’ and ‘b’ are constants. 2 3
r s
Dimensional formula of ‘b’ is same as
percentage error of measurement in p, q, r and s are 1%,
(a) Formula (b) Linear momentum
(c) Angular momentum (d) Torque 3%, 0.5% and 0.33% respectively, then the maximum
28. Backlash error may occur in which of the following percentage error in A is
instrument? (a) 2% (b) 0%
(a) Slide callipers and screw gauge (c) 4% (d) 3%

MARK YOUR 24. 25. 26. 27. 28.


RESPONSE 29. 30. 31. 32.
4 IIT-JEE PHYSICS Challenger
33. The dimensional equation for electric flux is (symbols have of the spring k is measured in kg/s2 and the chair’s
usual meaning, I current) frequency f is 0.50 s–1 for a 62 kg astronaut, what is the
(a) ML3T–3I–1 (b) ML–3 T3I–1 chair’s frequency for a 75kg astronaut ? The chair itself
(c) ML3T3I–1 (d) ML–3T3I has a mass of 10.0kg.
34. Dimensionally wavelength is equivalent to (a) 0.46 s–1 (b) 0.12 s–1
(c) 0.78 s–1 (d) 0.92 s–1
E LC E 39. Vernier callipers has 20 divisions on its vernier scale which
(a) (b)
B B LC coincide with 19 divisions on the main scale. Least count
of the instrument is 0.1mm. The main scale division is
B LC B
(c) (d) (a) 1 mm (b) 4 mm
E E LC
(c) 2 mm (d) – 2 mm
35. A student performs an experiment for determination of
40. A student performs an experiment to determine the Young's
2 modulus of a wire, exactly 2 m long, by Searle's method. In
4
g = ÷ . The error in length is and in time T is T a particular reading, the student measures the extension in
T2
the length of the wire to be 0.8 mm with an uncertainty
and n is number of times the reading is taken. The of ± 0.05 mm at a load of exactly 1.0 kg. The student also
measurement of g is most accurate for measures the diameter of the wire to be 0.4 mm with an
T n uncertainty of ± 0.01 mm. Take g = 9.8 m/s2 (exact). The
(a) 5 mm 0.2 sec 10 Young's modulus obtained from the reading is

(b) 5 mm 0.2 sec 20 (a) (2.0 ± 0.3) × 1011 N/m2 (b) (2.0 ± 0.2) × 1011 N/m2

(c) 5 mm 0.1 sec. 10 (c) (2.0 ± 0.1) × 1011 N/m2 (d) (2.0 ± 0.05) × 1011 N/m2

(d) 1 mm 0.1 sec 50 41. In a new system of units, the fundamental quantities mass,
length and time are replaced by acceleration ‘a’, density
36. The length of the string of a simple pendulum is measured ‘ ’ and frequency ‘f’. The dimensional formula for force in
with a metre scale to be 90.0 cm. The radius of the bob plus this system is
the length of the hook is calculated to be 2.13 cm using
measurements with a slide callipers. What is the effective (a) [ a4 f ] (b) [ a4 f –6]
length of the pendulum ? (This effective length is defined (c) [ –1a–4f 6] (d) [ –1a–4 f –1]
as the distance between the point of suspension and the 42. The pitch of a screw gauge is 1mm and there are 100
centre of the bob) division on its circular scale. When nothing is put in
(a) 87.87cm (b) 92.1 cm between its jaws, the zero of the circular scale lies 4
(c) 91.2 cm (d) 90.2 cm divisions below the reference line. When a steel wire is
placed between the jaws, two main scale divisions are
37. The length of a cylinder is measured with the help of a clearly visible and 67 divisions on the circular scale are
vernier callipers whose smallest division on the main scale observed. The diameter of the wire is
is 0.5mm and nine divisions of the main scale are equal to
(a) 2.71 mm (b) 2.67 mm
ten divisions of the vernier scale. It is observed that 78th
divisions of the main scale coincides with the sixth division (c) 2.63 mm (d) 2.65 mm
of the vernier scale. Find the length of the cylinder. 43. When the callipers are completely closed, zero of vernier
scale lies to the right of zero of main scale and coinciding
(a) 3.13 cm (b) 3.33 cm
vernier division is 7. If L.C. is (1/20) cm., the zero error is
(c) 3.63 cm (d) 3.93 cm
7 7
38. The space shuttle astronauts use a massing chair to (a) + cm (b) cm
measure their mass. The chair is attached to a spring and is 20 20
free to oscillate back and forth. The frequency of the 7 7
oscillation is measured and that is used to calculate the (c) + cm (d) cm
10 10
total mass m attached to the spring. If the spring constant

33. 34. 35. 36. 37.


MARK YOUR
38. 39. 40. 41. 42.
RESPONSE
43.
GENERAL PHYSICS 5

44. The formula for the period of a simple pendulum is


d d
(a) (b)
T= 2 /g . (m 1) ( m + 1)
The fractional error in the measurement of the time period
d d
T is ± x and that in the measurement of the length is ± y. (c) (d)
The maximum fractional error in the calculated value of g is m ( m + 1)
(a) ± (x + y) (b) ± (x – y) 46. The circular scale of screw gauge has 100 equal divisions.
(c) ± (2x + y) (d) ± (2x – y) When it is given 4 complete rotations, it moves through
45. In vernier callipers, m divisions of main scale coincide with 2mm. The L.C. of screw gauge is
(m + 1) divisions of vernier scale. If each division of main (a) 0.005 cm (b) 0.0005 cm
scale is d units, the least count of the instrument is (c) 0.001 cm (d) 0.0001 cm

MARK YOUR 44. 45. 46.


RESPONSE

PASSAGE-1 In this sketch, which is not to scale, imagine an observer looking


at objects A and B, standing at the pole of the Earth with his head
towards us. Now he sees object A to be to the right of B. Six
Parallax refers to the different views that you see from two different
positions. Try this experiment. Hold the index finger of your left months ago, he saw it to be to the left of B. Now most stars are so
hand vertical, 20 cm in front of you. Hold the index finger of your far away from us that we cannot observe any relative motion in
right hand vertical, 40 cm in front of you. Now close your left eye this way. However, for close stars it is possible. The next sketch
and, using just your right eye, move the two fingers sideways shows the path of light from a close object and from a very distant
until they line up. Now close your right eye and open the left. The star.
closer finger has ‘jumped’ to the right of the further finger. Repeat
a few times. Compared to a distant background, both fingers have
jumped to the right, but the closer one jumps farther. If you measure
Earth light from a very distant star
the angles through which they jump and the distance between
now
your eyes, you can work out how far away the fingers are. Sun
D
For distant objects, the distance between our viewing positions
must be greater than the distance between your eyes. Fortunately R
light from same star, six months ago
for astronomers, the Earth shifts our telescopes round the sun, so
we can get a separation equal to the diameter of the orbit of the Earth six months ago
Earth (16 light minutes) if we wait six months, as shown in this
diagram.
R R
From trigonometry, D = =
Earth tan
now
A B where we have used the small angle approximation for measured
Sun
in radians. A parsec is defined as the distance to an object that
‘moves’ (w.r.t. to the distant stars) by an angle of 1 second (1/3600
of a degree) when the Earth moves by the mean radius of its orbit.
In terms of this sketch, if = one second, D = 1 parsec. Now all
stars except the sun are more than one parsec distant, so to measure
Sun their distance by parallax, we need to be able to resolve angles of
A B
about 1 second or better.

Earth six months ago


6 IIT-JEE PHYSICS Challenger
1. If the first experiment if we have two point objects A and B in
PASSAGE-2
place of your left and right fingers respectively then
(a) your two eyes and A and B will lie on a circle
(b) your two eyes and A and B will always lie in a plane Three of the fundamental constants of physics are the universal
(c) your two eye and A will form a triangle area of which is gravitational constant, G = 6.7 × 10–11m3kg–1s–2, the speed of light,
equal to the area of the triangle formed by A, B and c = 3.0 × 108 m/s, and Planck’s constant, h = 6.6 × 10–34 kg m2s–1.
your left eye
4. Find a combination of these three constants that has the
(d) Both (b) and (c)
2. In above if we have three objects A, B and C such that you dimensions of time. This time is called the Planck time and
see only A & B from your left eye and A & C from your right represents the age of the universe before which the laws of
eye. Angle formed by AB on your left eye equals that formed physics as presently understood cannot be applied.
by AC on your right eye. Then AB will
(a) always be equal to AC hG hG
(a) (b)
(b) be equal to AC if your two eye and A form an isosceles c 4
c3
triangle
(c) be equal to AC if your two eye and A form a right
angled triangle hG hG
(c) (d)
(d) none of these c c5
3. Parallax method is useful only if the star whose distance is
to be measured should be closer to the earth and the reference 5. Find the value of Planck time in seconds
star should be (a) 1.3 × 10– 33 s (b) 1.3 × 10– 43 s
(a) very close (b) very far away (c) 2.3 × 10– 13 s (d) 0.3 × 10– 23 s
(c) any other star (d) only pole star

MARK YOUR
1. 2. 3. 4. 5.
RESPONSE

1. Statement-1 : In Searle’s experiment after every step 2. Statement-1 : A screw gauge having a smaller value of
of loading we wait for some time pitch has greater accuracy.
(generally two to three minutes) before Statement-2 : The least count of screw gauge is
taking the readings. directly proportional to the number of
Statement-2 : In this time the wire gets free from kinks. divisions on circular scale.

MARK YOUR
1. 2.
RESPONSE
GENERAL PHYSICS 7

3. Statement - 1 : The dimensional formula for relative 4. Statement-1 : Specific gravity of a fluid is a
velocity is same as that of the change in dimensionless quantity.
velocity. Statement-2 : It is the ratio of density of fluid to the
Statement - 2 : Relative velocity of P w.r.t. Q is the ratio density of water.
of velocity of P to that of Q.

MARK YOUR 3. 4.
RESPONSE

1. Let 0 be the absolute permittivity, r be the relative 5. Which of the following are dimensionless quantities?
permittivity, µ 0 and µ r be the absolute and relative 2
L 0 0E
permeability respectively. If M represents mass, L represents (a) (b) 2
CR 2 B
length, T represents time and A represents current, then
2
(a) [ 0] = [M–1L–3A2T4] (b) [( 0µ0)2] = [M0L–4T4] E
(c) (d) mLi2
2
(c) [ 0µr] = [M0L–2T–2] (d) [ r] = [µr] = [M0L0T0A0] B
(Here symbols have their usual meanings)
2. Let [ 0] denote the dimensional formula of the permittivity 6. The velocity, acceleration and force in two systems of units
are related as under :
of the vacuum, and [ 0] that of the permeability of the
2 1
vacuum. If M = mass, L = length, T = time and I = electric (i) v = v (ii) a' = ( )a (iii) F = F
current,
All the primed symbols belong to one system and unprimed
(a) [ 0] =M 1
L 3
T2 (b) [ 0 ] = M 1 L 3 T 4 2 ones belong to the other system. and are dimensionless
constants. Which of the following are correct
(c) [ 0] = M LT (d) [ 0 ] = M L2T 1
2 2
(a) Length standards of the two systems are related by :
3
3. The quantity/quantities that does/do not have mass in its/
L = 3 L
their dimensions is/are
(a) specific heat (b) Mass standards of the two systems are related by :
(b) latent heat
1
(c) electric potential difference M = 2 2÷
M
(d) electrical resistance
(c) Time standards of the two systems are related by :
4. The dimensions of the quantities in one (or more) of the
following pairs are the same. Identify the pair (s)
T =

T
(a) Torque and Work
(b) Angular momentum and Work (d) Momentum standards of the two systems are related
(c) Energy and Young’s modulus
1
(d) Light year and Wavelength by : p = 3÷
p

MARK YOUR 1. 2. 3. 4. 5.
RESPONSE 6.
8 IIT-JEE PHYSICS Challenger
7. The SI unit of inductance, the henry can be written as adopted as the fundamental units, choose the correct
(a) weber/ampere (b) volt-sec/amp option (s)
(a) unit of mass is 1.05 × 1052 kg
(c) Joule/(ampere)2 (d) ohm-second
(b) unit of length is 9.17 × 1015 m
8. If velocity of light in vacuum (3 × 108 m/s), acceleration due to (c) unit of time is 3.06 × 10 7 s
gravity (9.81 m/s2) and density of mercury (13600 kgm–3) be (d) unit of time is 3.06 × 10 4 s

MARK YOUR
7. 8.
RESPONSE

1. Column -I lists different type of methods to use different physical quantities while Column-II lists name for instrument that can
be used in these methods.
Column-I Column -II
(A) Determination of g using simple pendulum (p) Vernier callipers
(B) Determination of Young’s modulus of steel by Searle’s method (q) Stop clock
(C) Focal length of a concave mirror using u – v method (r) Screw gauge
(D) Determination of specific resistivity of material using (s) Optical bench
meter bridge method
2. Some physical quantities are given in Column I and some possible SI unit in which these quantities may be expressed are given
in Column II. Match the physical quantities in Column I with the units in Column II.
Column I Column II
1
(A) 0E
2 (q) (kilogram) (metre)2 (second) –1
2
0 : Permittivity of free space; E : Electric field
(B) h (q) (joule) (metre)–3
h : Planck’s constant
(C) P (r) (joule) (second)
P : Pressure
(D) Angular momentum (s) (kilogram) (metre)–1 (second) –2
3. Match the quantities in Column I with the correct units in Column II.
Column I Column II
(A) Electric field (p) volt
(B) Electrostatic potential (q) weber/ second
(C) Magnetic field (r) volt/meter
(D) Rate of change of magnetic flux w.r.t. time (s) weber/ metre2

1. 2. 3.

MARK YOUR
RESPONSE
GENERAL PHYSICS 9

4. Some of the quantities in Column I have the same dimension as quantities in Column II.
Column I Column II

ms 1
(A) (p)
KA 0 0
m = mass 0 = permeability of free space

s = specific heat capacity; = length; 0 = permittivity of free space


K = thermal conductivity; A = area
(B) E / B (q) gh
E = strength of electric field = density of fluid
B = strength of magnetic field g = acceleration due to gravity; h = height of column
1 2 GM
(C) 0E (r)
2 r
E = electric field G = Newton’s gravitational constant
0= permittivity of free space M = mass; r = radius

1 2
(D) v (s) LC
2
= density of fluid; v = speed of flow L = inductance; C = capacitance.
5. For the question given below : M = Mass; L = Length; T = Time; A = Ampere
Column I Column II
(A) Torque (p) ML–1T2
(B) Pressure (q) ML2T–2
(C) Electric field intensity (r) MLT– 3
(D) Intensity of a wave (s) MT–3
6. For the question given below : M = Mass; L = Length; T = Time; A = Ampere
Column I Column II
(A) Energy × Time (p) Gravitational
(B) Energy per unit length × Time (q) Angular momentum

Energy per unit electric current


(C) (r) Inductance
Electric current

Energy per unit mass


(D) (s) Linear momentum
length

4. 5. 6.

MARK YOUR
RESPONSE
10 IIT-JEE PHYSICS Challenger
7. Column I Column II

Viscosity ( ) J
(A) (p)
density( ) m3

(B) Velocity per unit electric field (q) m3


s2

cm 2
(C) Amount of heat supplied per unit volume (r)
s

cm 2
(D) Newton’s Gravitational constant × Mass (s)
V s

8. Considering force (F), Velocity (V) and energy (E) as fundamental quantities, match the correct dimensions of following quantities.
Column I Column II
(A) Mass (p) [F –1V 0E 1]
(B) Light year (q) [F 1V 1E –1]
(C) Frequency (r) [F 3V 0E –2]
(D) Pressure (s) [F 0V –2E 1]

7. 8.

MARK YOUR
RESPONSE
GENERAL PHYSICS 11

1. In Searle’s experiment, which is used to find Young’s 3. Each side of a cube is measured to be 7.2 m. What are the
Modulus of elasticity, the diameter of experimental wire is total surface area (in m 2 )of the cube to appropriate
D = 0.05 cm (measured by a scale of least count 0.001 cm) significant figures?
and length is L = 110 cm (measured by a scale of least count 4. 5.74 g of a substance occupies 1.2 cm3. Express its density
0.1 cm). A weight of 50 N causes an extension of ( in g cm–3) by keeping the significant figures in view.
X = 0.125 cm (measured by a micrometer of least count 5. The mass of a box measured by a grocer’s balance is 2.300
0.001cm). The maximum possible error in the values of kg. Two gold pieces of masses 20.15 g and 20.17 g are added
to the box. The difference in the masses (in g) of the pieces
Young’s modulus is x 109 N/m 2 . Find the value of x .
to correct significant figures ?
Screw gauge and meter scale are free from error.
2. A gas bubble from an explosion under water, oscillates with 6. The period of oscillation of a simple pendulum is 2 L/g .
a period T proportional to pa db Ec where p is the static Measured value of L is 20.0 cm known to 1 mm accuracy and
pressure, d is the density of water and E is the total energy time for 100 oscillations of the pendulum is found to be 90 s
of explosion. Find the value of (b + c – a) using a wrist watch of 1 s resolution. What is the percentage
error in the determination of g ?

1. 2. 3. 4. 5. 6.

MARK
YOUR
RESPONSE
12 IIT-JEE PHYSICS Challenger

1 (a) 9 (a) 17 (b) 25 (c) 33 (a) 41 (b)


2 (b) 10 (b) 18 (d) 26 (c) 34 (a) 42 (c)
3 (c) 11 (a) 19 (a) 27 (b) 35 (d) 43 (a)
4 (b) 12 (c) 20 (a) 28 (b) 36 (b) 44 (c)
5 (d) 13 (b) 21 (d) 29 (a) 37 (c) 45 (b)
6 (a) 14 (b) 22 (d) 30 (c) 38 (a) 46 (b)
7 (d) 15 (c) 23 (d) 31 (c) 39 (c)
8 (d) 16 (a) 24 (c) 32 (c) 40 (b)

1 (d) 2 (d) 3 (b) 4 (d) 5 (b)

1 (c) 2 (c) 3 (c) 4 (a)

1 (a, b, d) 3 (a, b) 5 (a, b) 7 (a, b, c, d)


2 (b, c) 4 (a, d) 6 (a, b, c, d) 8 (a, b, c)

1. A-p, q; B-r; C-s; D-r 2. A-q, s; B-p, r; C-q, s; D-p, r


3. A-r; B-p, q; C-s; D-q, p 4. A-s; B-p, r; C-q; D-q
5. A-q; B-p; C-r; D-s 6. A-q; B-s; C-r; D-r
7. A-r; B-s; C-p; D-q 8. A-s; B-p; C-q; D-r

1 11 2 1.67 3 31 4 4.8 5 0.02 6 3


GENERAL PHYSICS 13

1. (a) Given:
A b 0.1 0.2
1/ 2 =± =±
Q = x 2 / 5 y 1t z3 A b 5.7 3.4

0.34 1.14
Q 2 x y =±
100 = 100 100 5.7 3.4
Q 5 x y
1 t z A 1.48 1.48
100 3 100 =± A=± A
2 t z A 19.38 19.38
1.48
2 1 =± 19.38 1.48
= 2.5 + 2 + 1 3 0.5 = 5% 19.38
5 2 A = ± 1.5 (rounding off to two significant figures)
2. (b) Let force be F Aa vb c Area = (19.0 ± 1.5) sq.cm.
7. (d) Dimensionally 0 L = C
MLT 2
= [ L2 ]a [ LT 1 ]b [ ML 3 ]c where C = capacitance
a = 1, b = 2, c = 1 Now the given expression reduces to C V t.
F = Av²
Dimensionally C V t =q
1 2 where q is charge.
3. (c) Here ÷ 0E represents energy per unit volume.
2 Again the given expression reduces to q t.
q
[ 2
]=
[ Energy] = ML2T 2
= ML 1T 2
Dimensionally
t
=I
0 ] [E
[ Volume] L3 where I is current
8. (d) We can thus say that the viscous force (F) is the
4. (b) Given R = , then function of radius (r), velocity (v) and viscosity ( ).
r2 or F = f ( , r, v) or F = k x ry vz ...... (1)
Where k is a constant.
R r Now, dimensions of the constituents are
100 = 100 100 2 100 Force = [MLT–2] ; viscosity = [ML–1 T –1]
R r
Radius = [L] ; velocity = [LT –1]
= 1% + 2% + 2 × 3% = 9% Since dimensions on both sides have to be the same.
[MLT–2] = [ML–1T–1]x [L]y [LT–1]z
5. (d) L F x A yT z = [MxL–x+y+z T–x–z]
M 0 L1T 0 [ MLT 2 x
] [ LT 2 y
] Tz Equating the exponents of similar quantities of both
sides we get, x = 1 ; – x + y + z = 1 and – x – z = – 2
= M x Lx y T 2 x 2 y + z Solving for x, y & z, we get x = y = z = 1
Equation (1) becomes F = k rv
x = 0, x + y = 1, – 2x – 2y + z = 0
Experimentally, it was found that
x = 0, y = 1, z = 2 k = 6 or F = 6 rv,
Hence, L AT 2 which is the famous Stokes' law.
9. (a) Given that, T µ pa db Ec
6. (a) Here, = (5.7 ± 0.1) cm, b = (3.4 ± 0.2) cm
Equating both sides dimensionally,
Area A = × b = 5.7 × 3.4 = 19.38 cm²
[T] = [ML–1 T –2]a [ML–3]b [ML2T –2]c
= 19.0 cm² [M0L0T] = [Ma+b+c L–a–3b+2c T–2a–2c]
(rounding off to two significant fig.) Equating the exponents of similar quantities,
a + b + c = 0, – a – 3b + 2c = 0 and
– 2a – 2c = 1
14 IIT-JEE PHYSICS Challenger
Solving these equations, we get 14. (b) We have,
5 1 1 1
a=– ; b = and c = c=
6 2 3
0 0
3
2
10. (b) y 1
z 0
0 c2
2 3 2 3
z.3 d dz 3
dy 2 d 2
2 ÷ d + 2 dz 1
z z z
units of 0 =
units of 0 c 2
3 22 8
(± 0.1) + ( ± 0.1)
1 ÷
= 2 2 1
1
= 1 2 2
N C m (ms 1 ) 2
= – 8 (± 0.1) + 8 (± 0.1) = ± 1.6
= Ns2 C–2
3
2 23 = (kgms–2) s2 C–2 = kgm C–2
y = 22 4 8 4
z 1 15. (c) Unit of k is joules per kelvin or dimensional formula of
y = – 4 ± 1.6
k is [ML2T–2 1]
3
11. (a) V = = (1.2 × 10–2 m)3 = 1.728 × 10–6 m3 The power of an exponent is a number.
V = 1.7 × 10–6 m3.
z
has two significant figures. Hence V will also have Therefore, dimensionally = M ° L°T °
two significant figures. k
12. (c) Use the concept that the expression ( /p ) and
k
(az/KB ) are dimensionless. = × a dimensionless quantity
z
az dimensional formula of
p = K
[ ML2T 2 1 ][ ]
= = MLT 2
az [ L]
p =
K Also, dimensional formula of P = [ML–1 T–2]

K and dimensionally P = =
[ ]= P
paz

MLT 2
ML2T 2 1
. [ ]= = M 0 L2T 0
= 1 2 = L 2 M 0T 0 ML 1T 2
ML T .L
13. (b) Write E 2 / 0 as (E 2 0 )/( 0 0 ) and note that the Alternatively,
z
numerator has the dimensions of energy per unit = M 0 L0T 0
volume whereas the denominator has the dimensions k
of square of reciprocal of speed.
k
= × a dimensionless quantity
E2 [E 2 ] z
Also, dimensionally
0 0

We have, P=

[E] = MLT–3 I–1 (using E V )


d 1 k
= = ....(i)
P P z
0 Id sin
[ 0] =MLT–2 I–2 (using dB = . ) Also for one molecule of a gas
4 r2
R
E2 M 2 L2T 6 I 2 k=
NA ÷
PV = k
= = MLT–4
0 MLT 2 I 2
GENERAL PHYSICS 15

k p 1 1
V=
P
....(ii) log f log ÷ log log T log
2 2 2
From (i) & (ii) Differentiating partially on both sides,
V L3 df d 1 dT 1d
= = = L2 = M 0 L2T 0 0
z L f 2 T 2
16. (a) v = k
M 0 L1 T –1 = L [MT –2] [MT –3] df d
or 100 100÷
or M 0 L 1T – 1 = M + L – 3 T –2 f
Comparing dimension on both sides of equation
1 dT 1d
1 100÷ 100÷
–2 =–1 = 2 T 2
2
1 1
1 = ( 1) ( 2) (4) = –1 – 1 – 2 = – 4%
and –2 = – 1 2 2
2 21. (d) Let m and d be the mass and diameter of the sphere,
1 then the density of the sphere is given by
–3 =1
2
mass m 6m
= = 3
= 3
x 2 y 1 z volume 4 d d
17. (b) 100 = + + 100 ÷
x y 3 z ÷ 3 2

1 Taking log and differentiating partially we get


=2 2 1 3 = 5%
3 d dm 3d ( d )
2 m d
4
18. (d) T 2 g=
g T2 maximum % error = 2 + (3 × 3) = 11
22. (d) Electric flux E = E .S
g T
= +2
g ÷ max T Dimensionally E E
23. (d) Let x = – 1.4
0.1 0.01
= +2 ÷ 100% = 1.2% Then 1.4 + x + 5V = 1.4 + 5 S, where S denotes the
50.0 2.00 length of one main scale division, and V denotes the
length of one vernier scale division.
m
19. (a) = Hence, x = 5 S – 5V = 5 (1S – 1V) = 5C
r2
0.5
24. (c) Least count = = 0.01mm
m 2 r 50
= + +
m r Zero error = 5 × L.C
Putting the values = 5 × 0.01 mm
= 0.06 cm, = 6 cm; r = 0.005 cm; r = 0.5 cm, = 0.05 mm
Diameter of ball = [Reading on main scale] + [Reading
m = 0.3 gm; m = 0.003 gm
on circular scale × L . C] – Zero error
we get
= 0.5 × 2 + 25 × 0.01 – 0.05
4 = 1.20 mm
=
100 25. (c) Since, v = kx
dv d
100 = 4%. a=v = v. (kx) = kv
dx dx
Hence force acting on particle when its velocity is v.
p T F = ma = kmv
20. (a) Given f =
2
F µv
Taking log on both sides So, when v is doubled F is doubled.
16 IIT-JEE PHYSICS Challenger
26. (c) Least count of vernier scale
g T
1 35. (d) = +2
= 1 + ÷ – 1 × least count of main scale g T
n
1 2 and T are least and number of readings are
2 mm = mm maximum in option (d), therefore the measurement of g
49 49
is most accurate with data used in this option.
k .t 3
27. (b) = dimensionless 36. (b) L = 90.0 cm, r = 2.13 cm.
ma Effective length = 90.0 + 2.13 = 92.1 cm.
ML2T 2
K 1.K .T 3 0.5
= M 0 L0 T 0 37. (c) Vernier constant = = 0.05mm
M .a 10
a = L2T Since 78th division of the main scale coincides with 6th
Dimension of P = Dimension of
b division of the vernier scale, therefore, the zero of the
a vernier scale may lie close to (78 – 6 = 72) division of
[b] the main scale.
ML–1T–2 = Therefore, main scale reading is 72 × 0.5 = 36mm
L2T
Vernier scale reading = 6 × 0.05 = 0.3 mm
[b] = MLT–1 = [Linear momentum]
28. (b) Backlash error occurs only in instruments using screws. Length of cylinder = 36 + 0.3 = 36.3 mm = 3.63cm.

I
38. (a) f µ ma k b
29. (a) I I 0 sin 2
= sin 1
I0 T 1 = M a M bT 2b
a + b = 0 ; – 2b = – 1 b = 1/2, a = – 1/2
1 I0 dI
d =
2 I0 I k 0.50 85 1 72
I0 I f µ ; = f = = 0.46 s–1
m f 72 2 85
d 1 dI
= 1 (main scale division)
2 I ( I0 1
I ) sin I / I0 39. (c) Least count =
N
3
0.0020 2 10
= = 1 (main scale division)
2 5 15 10 3 0.1 mm = ;
20
6 6
1 main scale division = 2 mm.
12 4 4 4
10 3 10
3 mg L
40. (b) We know that Y = 2
4 D
% error = 3 10 2 % 4

ATML2T 1 4mgL 4 1 9.8 2


30. (c) n = = L2 A Y= =
(0.4 ) (0.8 )
M 2 3 2
D 10 10 3
31. (c) Least count of measuring device = 0.1
Using F = ma ; m = (1 ± 0.1) kg
1 1 1 1 2.0 1011 N/m 2
32. (c) % error in A = 1 3 2 3 = 4%
2 2 2 3
2 2
Now Y = 2 D +
F [ MLT ][ L ] Y D
33. (a) = E. A. = A=
q [ IT ] [ the value of m, g and L are exact]

[ ] = [ ML3T 3
I 1
] 0.01 0.05
=2 + = 2 × 0.025 + 0.0625
0.4 0.8
E
34. (a) = velocity = m / s and LC = time period = s = 0.05 + 0.0625 = 0.1125
B
Y = 2 × 1011 × 0.1125 = 0.225 × 1011
E LC
= m = wavelength 0.2 1011 N/m 2
B
Y = ( 2 ± 0.2 ) 1011 N/m 2
GENERAL PHYSICS 17

41. (b) F = ma = volume ‘a’


2
To write volume in terms of ‘a’ and ‘f ’ 44. (c) g 4 .
T2
3 g T
L = +2
Volume = L3 = 2÷
T 6 = a3 f 6 Therefore, = ± ( y + 2x)
T g T
46. (b) md = (m + 1)v (v = length of one vernier scale division)
F = a4 f –6
md
42. (c) Diameter = 2 + reading corresponding to 67 division. v=
m +1
100 1mm
md d
least count = d =
67
67 m +1 m +1
100 [Since m divisions of main scale are equivalent to
due to +ve zero error reading corresponding to (m + 1) divisions of vernier scale, one division of
vernier scale is equivalent to [m / (m + 1)] divisions
63 of main scale. Now, use the definition : least count
67 division = mm = 0.63mm of vernier = one division of main scale – one
100
division of vernier scale.]
43. (a) As zero of vernier scale lies to the right of zero of main
2 1 1
scale, therefore, zero correction is negative and zero 46. (b) Pitch = mm = mm = cm.
4 2 20
+7
error is (+ 7 × L.C.) = cm. Pitch 1
20 L.C. = = cm = 0.0005 cm
100 2000

1. (d) If two point objects A and B are in front of left and right 4. (d), 5. (b).
fingers respectively then A and B and eyes will be on
same plane and two eyes and A will form a triangle area hG kgm 2s 1 m3kg 1s 2
=
of which is equal to the area of the triangle formed by c5 m 5 / s5
A, B and left eye.
2. (d) Nothing can be said as size of AB will depend on = s2 = s
position of observation.
Putting the values of h, G and c in above relation
3. (b) To reduce effect of relative motion star should be as far
Planck time = 1.3 × 10– 43 s.
as possible.

1. (c) The correct explanation for statement-1 is that we have Relative velocity is measured not by calculating ratio
to give some time to wire to acquire whole extension but by calculating difference.
according to load. 4. (a) Specific gravity of fluid
2. (c) No. of division Least count Pitch accuracy
density of fluid
3. (c) Relative velocity which is vector subtraction of two =
density of water
velocities will also be a vector of the form of velocity
so, its dimensional formula will remain unchanged. It is a ratio.
18 IIT-JEE PHYSICS Challenger

1 [ AT ]2 [eq. (1)]2
1. (a, b, d) F= Now, gives
4 0 L2 eq.(2)
1 4 1 2
A2T 2 [L T ] 1 [ LT ]
0= 2 2
= M 1L 3 A2T 4 2 2 2
L MLT [L T ] [ LT ]

1 3
= c (speed of light) [ L ] = 3 [ L]
0 0

1 1
( 0 0)
2
= = = L 4T 4 p [ M L ] [T ] 1 3 2
1
c 4
[ LT 1 4
] = = 2 2 3
= 3
p [T ] [ ML ]
r and r = dimensionless
dI 2U
Q1Q2 F µ0 I1 I 2 7. (a, b, c, d) L = ; L= e ÷; L= 2 ; L=R×t
2. (b, c) By definition F = and = I dt I
2 2 L
(4 0 )r
weber
L= =
[Q]2 I 2T 2 I ampere
Hence, [ 0] = =
[ F ][r 2 ] MLT 2 .L2
e volt volt-sec
= M–1L–3 T 4 I 2 L= = =
dI / dt ampere/sec ampere
[F ] MLT 2
[µ0] = = = MLT 2 I 2 L=
2U
=
joule
[ I ]2 I2 I 2
(ampere) 2
3. (a, b) The quantities specific heat and latent heat both
contain a term ‘energy per unit mass’. However energy L = R t ohm sec
itself contains mass and hence the dimensions of both 8. (a, b, c) Let the dependence of mass (M) on velocity (v),
these quantities do not contain mass. acc. due to gravity (g) and density ( ) be as
4. (a, d) = F × r × sin ; W = F × d × cos M vx g y z
Dimensionally, light year = wavelength = [L] Substituting the dimensions of v, g and in R.H.S.
L
= [CR ] = [Time] ML T = [ LT
0 0 1 x
] [ LT 2 y
] [ ML ] 3 z
5. (a, b)
R
or ML0T 0 = M z Lx y 3 z T x 2 y
Comparing the dimensions on both sides (i.e. the
1 E
= = [velocity] equating the powers of M, L and T), we get
0 0 B z = 1, x + y – 3z = 0 and – x – 2y = 0
Solving, we get, x = 6, y = – 3 and z = 1
2
(a, b, c, d) [ L T ] =
1
6. [ LT 1 ] ..... (1) Mass [M] = v 6 g 3
Unit of mass
2 2 ..... (2)
[L T ] [ LT ]
3 (108 )6
= 13600 1.05 1052 kg
1 (9.81)3
[M L T 2
]= [MLT 2
] ..... (3)
Similarly, it can be shown that, the length (L) will be
given by L = v2/g.
1
Dividing eq. (3) by (2), we get, [ M ] = [M ]
2 2 (3 108 )2
Unit of length = 9.17 1015 m
Dividing eq. (1) by (2), we get 9.81
and time T is given by T = v/g
2
1
[T ] [T ] = 2 [T ] (3 108 )
Unit of time = 3.06 107 s
9.81
GENERAL PHYSICS 19

8. A-s; B-p; C-q; D-r


1. A-p, q; B-r; C-s; D-r
2. A-q, s; B-p, r; C-q, s; D-p, r Energy
(A) Mass = = EV 2
F0 ;
(velocity)2
3. A-r; B-p, q; C-s; D-q, p
Energy E
4. A-s; B-p, r; C-q; D-q (B) Length = = = E1 F 1V 0
force F
5. A-q; B-p; C-r; D-s
Length E1 F 1V 0 ;
6. A-q; B-s; C-r; D-r Time = = = E1F 1V 1
Velocity V1
7. A-r; B-s; C-p; D-q 1
(C) Frequency = = E 1FV
Time
Force F
(D) Pressure = = = F 3V 0 E 2
(length) 2 ( E1 F 1V 0 ) 2

1. 11 2. 1.67
Maximum percentage error in Y is given by Given that, T µ pa db Ec
Equating both sides dimensionally,
W L [T] = [ML–1 T–2]a [ML–3]b [ML2T–2]c
Y= 2
D X [M0L0T] = [Ma+b+c L–a–3b+2c T–2a–2c]
4 Equating the exponents of similar quantities,
a + b + c = 0, – a – 3b + 2c = 0 and – 2a – 2c = 1
Y D X L Solving these equations, we get
=2 ÷+ +
Y max D X L
5 1 1
a=– ; b= and c =
6 2 3
0.001 0.001 0.1
=2 + + ÷ = 0.0489
0.05 0.125 110 1 1 5 10
b c a= + + = = 1.67
2 3 6 6
So, maximum percentage error = 4.89%
3. 31
It is given that
The number of significant figures in the measured length is
W = 50 N; D = 0.05 cm; = 0.05 × 10–2m;
2. The calculated area should, therefore, be rounded off to 2
X = 0.125 cm = 0.125 × 10–2m;
significant figures.
L = 110 cm = 110 × 10–2m Surface area of the cube = 6(7.2)2 m2
= 311.04 m2 = 31 m2
50 4 110 10 2
Y= 4. 4.8
3.14 (0.05 10 2 ) (0.125 10 2 ) There are 3 significant figures in the measured mass whereas
= 2.24 × 1011N/m2 there are only 2 significant figures in the measured volume.
Hence, the density should be expressed to only 2 significant
Y = 0.0489 × 2.24 × 1011 = 1.09 × 1010 N/m2
figures.
11 109 N/m 2
5.74 3
Density = gcm = 4.8 g cm–3 .
x = 11 1.2
20 IIT-JEE PHYSICS Challenger
5. 0.02 The errors in both L and t are the least count errors. Therefore,
Difference = 20.17 g – 20.15 g = 0.02 g ( g/g) = ( L/L) + 2 ( T/T )
6. 3
0.1 1
= +2 ÷ = 0.027
2 2 20.0 90
g 4 L/T ;
Thus, the percentage error in g is
t t T t
Here, T = and T= . Therefore, = . 100 ( ) = 100 ( L/L) + 2 × 100 ( T/T ) = 3%.
n n T t
1. A particle P is projected from a point on the surface of smooth 4. Two particles A and B separated by a distance 2R are moving
inclined plane (see figure). Simultaneously another particle counter clockwise along the same circular path of radius R
Q is released on the smooth inclined plane from the same each with uniform speed v. At time t = 0, A is given a tangential
position. P and Q collide on the inclined plane after t = 4 72v 2
second. The speed of projection of P is acceleration of magnitude a
25 R
P 6 R
Q (a) the time lapse for the two bodies to collide is
5v
(b) the angle covered by A is 11 /6
11v
(c) angular velocity of A is
5R
60°
(d) radial acceleration of A is 289v2/5R
(a) 5 m/s (b) 10 m/s 5. A cannon ball has the same range R on a horizontal plane
(c) 15 m/s (d) 20 m/s for two angles of projection. If h1 and h2 are the greatest
2. The figure shows the velocity and acceleration of a point heights in the two paths for which this is possible, then
like body at the initial moment of its motion. The acceleration
vector of the body remains constant. The minimum radius (a) R = h1 h2 (b) R 4 h1 h2
of curvature of trajectory of the body is
1
(c) R = 3 h1 h2 (d) R = h1 h2 4
=150° v=8m/s
0

6. A passenger in an open car travelling at 30 m/s throws a ball


out over the bonnet. Relative to the car the initial velocity of
the ball is 20 m/s at 600 to the horizontal. The angle of
a=2m/s² projection of the ball with respect to the horizontal road will
(a) 2 m (b) 4 m be
(c) 8 m (d) 16 m
2 3
3. A stone projected at an angle with horizontal from the roof (a) tan–1 (b) tan–1
of a tall building falls on the ground after three second. Two 3 4
second after the projection it was again at the level of
projection. Then the height of the building is 4 3
(a) 15 m (b) 5 m (c) tan–1 (d) tan–1
3 4
(c) 25 m (d) 20 m

MARK YOUR 1. 2. 3. 4. 5.
RESPONSE 6.
22 IIT-JEE P HYSICS Challenger
7. The height y and the distance x along the horizontal plane
13. A body is thrown horizontally with a velocity 2gh from
of a projectile on a certain planet (with no surrounding
atmosphere) are given by y = (8t – 5t2) meter and x = 6t the top of a tower of height h. It strikes the level ground
meter, where t is in second. The velocity with which the through the foot of the tower at a distance x from the tower.
projectile is projected is The value of x is
(a) 8 m/sec (a) h (b) h/2
(b) 6 m/sec (c) 2h (d) 2h/3
(c) 10 m/sec 14. A particle is projected at angle 37° with the incline plane in
(d) Not obtainable from the data upward direction with speed 10 m/s. The angle of incline
8. A particle is projected at an angle of elevation and after t plane is given 53°. Then the maximum height attained by the
seconds it appears to have an angle of elevation as seen particle from the incline plane will be
from point of projection. The initial velocity will be (a) 3 m (b) 4 m
(c) 5 m (d) zero
gt gt cos
(a) (b) 15. From the given position, as shown in the figure, the plank
2 sin ( ) 2sin ( )
starts moving towards left with initial velocity zero and
sin ( ) 2 sin ( ) acceleration 8 m/s2. The rod flies in the air and falls back on
(c) (d) the plank. With all surfaces smooth, what should be the
2gt gt cos
least possible length , so that the rod doesn’t fall on the
9. It was calculated that a shell when fired from a gun
plank?
with a certain velocity and at an angle of elevation
5 /36 rad should strike a given target. In actual
practice, it was found that a hill just prevented the
trajectory. At what angle of elevation should the gun
be fired to hit the target
5 11
(a) rad (b) rad 37° 0.75 m
36 36
7 13 l
(c) rad (d) rad (a) 4.84 m (b) 3.84 m
36 36
(c) 5.62 m (d) 4.62 m
10. A cricket ball is hit at 30º with the horizontal with kinetic
energy K. The kinetic energy at the highest point is 16. For a stone thrown from a lower of unknown height, the
(a) 0 (b) K/4 maximum range for a projection speed of 10 m/s is obtained
(c) K/2 (d) 3K/4 for a projection angle of 30°. The corresponding distance
11. A body of mass m is projected at an angle of 45º with the between the foot of the lower and the point of landing of the
horizontal with velocity u. If air resistance is negligible, then stone is
total change in momentum when it strikes the ground is (a) 10 m (b) 20 m
(a) 2 mu (b) 2 mu
(c) 20 / 3 m (d) 10 / 3 m
(c) mu (d) mu/ 2
12. A ball is projected horizontally with a speed v from the top 17. A particle moves with a constant speed u along the curve
of a plane inclined at an angle 45° with the horizontal. How y = sin x. The magnitude of its acceleration at the point
far from the point of projection will the ball strike the plane corresponding to x = /2 is
v2 v2
(a) (b) 2 u2 u2
g g (a) (b)
2 2
2v 2 2v 2
(c) (d) 2 (c) u 2 (d) 2 u2
g g

7. 8. 9. 10. 11.
MARK YOUR
12. 13. 14. 15. 16.
RESPONSE
17.
MOTION IN ONE AND TWO DIMENSIONS 23

18. A skier travels with a constant speed of 6 m/s along a 23. A boat B is moving upstream with velocity 3 m/s with respect
to ground. An observer standing on boat observes that a
x2
parabolic path y . Find the acceleration of the skier swimmer S is crossing the river perpendicular to the
20
direction of motion of boat. If river flow velocity is 4 m/s and
when he is at (10, 5). Neglect the size of skier.
swimmer crosses the river of width 100 m in 50 sec, then
9 9 2
(a) units (b) units
10 2 10
(c) 1 unit (d) none of these vb
B
19. A swimmer can swim in still water with a speed of 5 m/s.
j
While crossing a river his average speed is 3 m/s. If he
crosses the river in the shortest possible time, what is the oS v i
speed of flow of water?
(a) 2m/s (b) 4 m/s
(c) 6 m/s (d) 8 m/s
20. A particle starts from rest and moves with an acceleration of (a) velocity of swimmer w.r.t ground is 13 m/s
a = {2 + |t – 2|} m/s2, the velocity of the particle at t = 4 sec is (b) drift of swimmer along river is zero
(a) 2 m/s (b) 4 m/s
(c) drift of swimmer along river will be 50 m
(c) zero (d) 12 m/s
21. A car accelerates from rest with a constant acceleration (d) velocity of swimmer w.r.t ground is 2 m/s
on a straight road. After gaining a velocity v1 the car moves 24. For an observer on trolley direction of projection of particle
with the same velocity for some-time. Then the car is shown in the figure, while for observer on ground ball rise
decelerated to rest with a retardation . If the total distance
vertically. The maximum height reached by ball from trolley
covered by the car is equal to S, the total time taken for its
motion is is

S v 1 1 S v v v (w. r. t. trolley)
(a) (b)
v 2 v

v v S v v v 10 m/s
(c) (d) –
v 2
22. A disc having plane parallel to the horizontal is moving (a) 10 m (b) 15 m
such that velocity of point P with respect to ground on its
(c) 20 m (d) 5 m
periphery is 2 m/s j as shown in the figure. If radius of disc
25. A particle is projected with a velocity u at an angle with
is R = 1 m and angular speed of disc about vertical axis the horizontal. After some time velocity of particle becomes
passing through disc is = 2 rad/s, the velocity of centre of
perpendicular to initial velocity when the particle is still
disc in m/s is
above the horizontal. The angle of projection may be
2 ˆj
(choose the most appropriate option)
P u
j

C i
(a) 30°
(b) 60°
(a) 2 j (b) 2 i + 2 j (c) any value except = 0°
(d) possible for no value of
(c) – 2 i + 2 j (d) none of these

MARK YOUR 18. 19. 20. 21. 22.


RESPONSE 23. 24. 25.
24 IIT-JEE P HYSICS Challenger
26. AB is an inclined plane of inclination 30° with horizontal.
Point O is 20 m above point A. A particle is projected Y
horizontally and it collides with the plane AB,
perpendicularly. Speed of the particle must be (g = 10 m/s2)

O
20m
B X
20m
25m
30°
Horizontal
A 75m

(a) 13 m/s (b) 8 3m / s


1 4 1 3
(c)
4 5m / s (d) 2 5m / s (a) tan (b) tan
3 4
27. A boy is standing on a cart moving along x-axis with the
speed of 10 m/s. When the cart reaches the origin he throws 3 1 12
(c) tan (d) tan
a stone in the horizontal x-y plane with the speed of 5 m/s 2 3
with respect to himself at an angle with the x-axis. It is 31. A person throws vertically n balls per second with the same
found that the stone hits a ball lying at rest at a point whose velocity. He throws a ball whenever the previous one is at
its highest point. The height to which the balls rise is
co-ordinates are ( 3m,1m ). The value of is (gravitational
(a) g/n 2 (b) 2gn
effect is to be ignored)
(c) g/2n2 (d) 2gn2
(a) 30° (b) 60°
32. The velocity-displacement graph of a particle moving
(c) 90° (d) 120° along a straight line is shown
28. A particle is moving with a constant speed of m/s on a
circular track of radius 12 m. The magnitude of its average
acceleration for the time interval of 6s is
2
(a) m / s2 (b) m / s2
12 3 2
2
(c) m/s (d) 0 m/s2
6
29. A particle, of mass m, is falling under the influence of gravity The most suitable acceleration-displacement graph will be
through a medium whose resistance equals µ times the
velocity. If the particle were released from rest, the distance
fallen through in time t is
(a) (b)
m2 t/m t m2 t/m t
(a) g e 1 (b) g 2
e 1
2 m m

m2 t/m 2 t m2 t/m t
(c) g e 1 (d) g 2
e 1
2 m 2 m
30. A ball thrown down the incline strikes at a point on the incline
(c) (d)
25m below the horizontal as shown in the figure. If the ball
rises to a maximum height of 20m above the point of projection,
the angle of projection (with horizontal x-axis) is

MARK YOUR 26. 27. 28. 29. 30.


RESPONSE 31. 32.
MOTION IN ONE AND TWO DIMENSIONS 25

33. A point moves in x-y plane according to the law x = 3 cos 4t


and y = 3 (1 – sin 4t). The distance travelled by the particle 14.0

Velocity (m/s) [west]


in 2 sec is (where x and y are in meters) 12.0
Car A
10.0
(a) 48 m (b) 24 m 8.0
(c) 48 2 m (d) 24 2 m 6.0
4.0
34. Two particles 1 and 2 are projected with same speed v as Car B
2.0
shown in figure. Particle 2 is on the ground and particle 1 0.0
0.0 2.0 4.0 6.0 8.0 10.0 12.0 14.0
is at a height h from the ground and at a horizontal distance
Time (s)
s from particle 2. If a graph is plotted between v and s for
the condition of collision of the two then (v on y-axis and
s on x-axis) Which of the following is correct?
(a) Only statement 1 is true
v
1 (b) Only statement 2 is true
(c) Only statement 3 is true
h (d) Only statements 1 and 2 are true
v 37. Two drag racers accelerate from rest down a drag strip. The
2 engine of each car produces a constant forward force of
s 1200 N on the car. Car A has a mass of 1.25× 103 kg, while
(a) It will be a parabola passing through the origin car B has a mass of 1.20× 103 kg. When car A has gone
1.00 × 102 m, car B will be
(b) It will be a straight line passing through the origin and
(a) 2m behind car A (b) 2m behind car A
g
having a slope of (c) 2 m ahead of car A (d) 4 m ahead of car A
8h
(c) It will be a straight line passing through the origin and 38. A particle starts from rest at time t = 0 and moves on a
straight line with acceleration as plotted in figure. The speed
8h of the particle will be maximum at time
having a slope of
g
(d) None of these
35. A particle of unit mass is projected with velocity u at an 10
a (in m/s2 )

inclination above the horizon in a medium whose


resistance is k times the velocity. Its direction will again 0 t
2 4
make an angle with the horizon after a time (in seconds)
–10
1 2ku 1 2ku
(a) log 1 sin (b) log 1 sin
k g k g

(a) 1s (b) 2s
1 ku 1 2ku
(c) log 1 sin (d) log 1 sin (c) 3s (d) 4s
k g k 3g
39. A car moves with a speed of 60 km/hr from point A to point
36. The graph shown illustrates velocity versus time for two B and then with the speed of 40 km/hr from point B to point
cars A and B constrained to move in a straight line. Both C. Further it moves to a point D with a speed equal to its
cars were at the same position at t = 0s. Consider the average speed between A and C. Points A, B, C and D are
following statements. collinear and equidistant. The average speed of the car
(1) Car A is travelling west and Car B is travelling east. between A and D is
(2) Car A overtakes Car B at t = 5 s. (a) 30 km/hr (b) 50 km/hr
(3) Car A overtakes Car B at t = 10 s. (c) 48 km/hr (d) 60 km/hr

MARK YOUR 33. 34. 35. 36. 37.


RESPONSE 38. 39.
26 IIT-JEE P HYSICS Challenger
40. Two particles start moving from rest from the same point 2ab ba
along the same straight line. The first moves with constant (a) (b)
c c
velocity v and the second with constant acceleration a.
During the time that elapse before the second catches the 3ba
(c) (d) None of these
first, the greatest distance between the particles is c
45. The greatest range of a particle, projected with a given
v2 v2 2v 2 v2 velocity on an inclined plane, is x times the greatest vertical
(a) (b) (c) (d) altitude above the inclined plane. Find the value of x.
a 2a a 4a (a) 2 (b) 4 (c) 3 (d) 1/2
41. If a particle is projected with speed u from ground at an
46. An object has velocity v1 relative to the ground. An
angle with horizontal ,then radius of curvature of a point
where velocity vector is perpendicular to initial velocity observer moving with a constant velocity v0 relative to the
vector is given by ground measures the velocity of the object to v2 be (relative
to the observer). The magnitudes of these velocities are
u 2 cos 2 u 2 cot 2 related by
(a) (b)
g g sin (a) v0 v1 v 2 (b) v1 v2 v0
u2 u 2 tan 2 (c) v2 v0 v1 (d) All the above are true
(c) (d)
g g cos 47. A particle is projected with speed u at an angle from
42. Rishabh of Raxaul skated the 10,000m race in Salt Lake City horizontal at t = 0. Its horizontal component of velocity (vx)
in 12min, 58.92 seconds. The oval track is made up of two varies with time as following graph :
straight 112.00m sections and two essentially identical vx
semicircular curves. There are two lanes, each 5.00m wide.
The 400m lap starts at A on the inner straightway, rounds
the inner curve, crosses over in the next straight section in
the shortest diagonal path to the outside lane (the other
skater crosses over the other way), and rounds the outer (a)
curve, ending up on the adjacent lane at B (see dotted line).
The measurement is made 5cm out from the inner edge of
the lane, and is exactly 400m for one lap. What is the radius T/2 T
t
of the inner curve, R, in m? vx

R R
112m
5m (b)
A
5cm t
5cm B
T/2 T
5m vx
(a) 24.23 (b) 25.47
(c) 25.31 (d) 25.44
43. A particle starts sliding down a frictionless inclined plane.
If Sn is the distance travelled by it from time t = n – 1 sec to
t = n sec, the ratio Sn/Sn+1 is (c)
2n 1 2n 1
(a) (b) t
2n 1 2n T/2 T
vx
2n 2n 1
(c) (d)
2n 1 2n 1
44. A particle is projected from a horizontal plane (x-z plane)
such that its velocity vector at time t is given by
v aiˆ (b ct ) ˆj . Its range on the horizontal plane is given (d)
by
t
T/2 T

MARK YOUR 40. 41. 42. 43. 44.


RESPONSE 45. 46. 47.
MOTION IN ONE AND TWO DIMENSIONS 27

48. An airplane flies from a town A to a town B when there is no


wind and takes a total time T0 for a return trip. When there is 30m/s 40m/s
A
a wind blowing in a direction from town A to town B, the B
10km
plane’s time for a similar return trip, Tw, would satisfy
(a) T0 < Tw (b) T0 > Tw (a) 80 sec (b) 120 sec
(c) T0 = Tw (c) 160 sec (d) None of these
(d) the result depends on the wind velocity between the 53. A particle is projected from a tower as shown in figure, then
towns the distance from the foot of the tower where it will strike
49. A bird flies with a speed of 10 km/h and a car moves the ground will be
with uniform speed of 8 km/h. Both start from B towards
A (BA = 40km) at the same instant. The bird having reached 37°
A, flies back immediately to meet the approaching car. As
soon as it reaches the car, it flies back to A. The bird repeats
500 m/s

1500m
this till both the car and the bird reach A simultaneously. 3
The total distance flown by the bird is
(a) 80 km (b) 40 km
(c) 50 km (d) cannot be determined.
50. A block B moves with a velocity u relative to the wedge A. If
the velocity of the wedge is v as shown in figure, what is the ///////////////////////////////////////////////////
value of so that the block B moves vertically as seen from
(a) 4000/3 m (b) 2000/m
ground ?
(c) 1000/3 m (d) 2500/3 m
u 54. If a boat can travel with a speed of v in still water, which of
B the following trips will take the least amount of time ?
A
v (a) travelling a distance of 2d in still water
(b) travelling a distance of 2d across (perpendicular to)
the current in a stream
1 u 1 v (c) travelling a distance d downstream and returning a
(a) cos (b) cos
v u distance d upstream
(d) travelling a distance d upstream and returning a
u1 1 v distance d downstream
(c) sin (d) sin
v u 55. A body A is thrown vertically upward with the initial velocity
51. A hunter tries to hunt a monkey with a small, very poisonous v1. Another body B is dropped from a height h. Find how
arrow, blown from a pipe with initial speed v0. The monkey the distance x between the bodies depends on the time t if
is hanging on a branch of a tree at height H above the the bodies begin to move simultaneously.
ground. The hunter is at a distance L from the bottom of the (a) x = h – v1t (b) x = (h – v1) t
tree. The monkey sees the arrow leaving the blow pipe and
immediately loses the grip on the tree, falling freely down v1 h
(c) x = h – (d) x = – v1
with zero initial velocity. The minimum initial speed v0 of the t t
arrow for hunter to succeed while monkey is in air is 56. Two swimmers start from point A on one bank of a river to
reach a point B on the other bank lying directly opposite to
g ( H 2 L2 ) gH 2 point A. One of them crosses the river along the straight line
(a) (b)
2H H2 L2 AB while the other swims at right angles to the stream and
then walks the distance which he has been carried away by
2gH 2 the stream to get to point B. What was the velocity (assumed
g H 2 L2 uniform) of his walking if both the swimmers reached point
(c) (d)
H H2 L2 B simultaneously ? Velocity of each swimmer in still water is
52. Ship A is moving with velocity 30m/s due east and ship B 2.5 km/hr and the stream velocity is 2 km/hr.
with velocity 40m/s due north. Initial separation between (a) 4 km/hr (b) 3 km/hr.
the ships is 10km as shown in figure. After what time ships (c) 5 km/hr (d) 8 km/hr
are closest to each other ?

MARK YOUR 48. 49. 50. 51. 52.


RESPONSE 53. 54. 55. 56.
28 IIT-JEE PHYSICS Challenger
57. A body A begins to move with initial velocity 2 m/sec and 63. A man travelling in a car with a maximum constant speed of
continues to move at a constant acceleration a. t = 10 20m/s watches the friend start off at a distance 100m ahead
seconds after the body A begins to move a body B departs on a motor cycle with constant acceleration ‘a’. The maximum
from the same point with an initial velocity 12 m/sec and value of ‘a’ for which the man in the car can reach his friend
moves with the same acceleration a. What is the maximum is
acceleration a at which the body B can overtake A ?
(a) 1 m/s2 (b) 2 m/s2
(c) 1/2 m/s 2 (d) 3 m/s2 v a
58. A person travelling eastward finds the wind to blow from
north. On doubling his speed he finds it to come from north-
east. If he travel his speed, the wind would appear to him to 100m
come from a direction making an angle (a) 2 m/s2 (b) 1 m/s2
(a) = tan–1 (1/2) north of east (c) 4 m/s 2 (d) None of these
(b) = tan–1 (1/2) south of east 64. Three ships A, B and C are sailing at constant speeds on
(c) = tan–1 (1/2) north of west steady courses. B is sailing due east at 12 km/hr and C is
(d) None of these sailing in a direction N 30° E at 8 km/hr. To an observer on C,
59. A train moving at 30 m/sec reduces its speed to 10 m/sec in the direction of A appears to be S 30° E and to an observer on
a distance of 240m. At what distance will the train come to a B, A appears to be sailing due south. Find the speed of A.
stop? If the brake power is increased by 12½%, the train will (a) 8 3 km / hr (b) 4 3 km / hr
stop in a total distance of
(a) 120 m (b) 240 m (c) 2 3 km / hr (d) 3 3 km / hr
(c) 360 m (d) 420 m 65. A ball is dropped vertically from a height d above the ground.
60. A truck has to carry a load in the shortest time from one
It hits the ground and bounces up vertically to a height d/2.
station to another station situated at a distance L from the
Neglecting subsequent motion and air resistance, its velocity
first. It can start up or slowdown at the same acceleration or
v varies with the height h above the ground as
deceleration a. What maximum velocity must the truck attain
to satisfy this condition?
(a) La (b) 2La
(c) 3La (d) none of these (a) (b)
61. A body starts from rest at time t = 0, the acceleration time
graph is shown in the figure. The maximum velocity attained
by the body will be

(c) (d)

66. A body is projected vertically upwards with a velocity u,


after time t another body is projected vertically upwards
from the same point with a velocity v, where v < u. If they
meet as soon as possible, then choose the correct option
(a) 110 m/s (b) 55 m/s
(c) 650 m/s (d) 550 m/s u v + u 2 + v2 u v u2 v2
62. The velocity of an object moving rectilinearly is given as a (a) t= (b) t=
g g
function of time by v = 4t – 3t2, where v is in m/s and t is in
seconds. The average velocity of particle between t = 0 to t = 2
u + v + u2 v2 u v u2 v2
seconds is (c) t= (d) t=
(a) 0 (b) –2m/s g 2g
(c) – 4m/s (d) None of these

MARK YOUR 57. 58. 59. 60. 61.


RESPONSE 62. 63. 64. 65. 66.
MOTION IN ONE AND TWO DIMENSIONS 29

67. A man in a row boat must get from point A to point B on the 71. When a shot is projected from a gun at any angle of elevation,
opposite bank of the river (figure). The distance BC = a. the shot as seen from the point of projection will appear to
The width of the river AC = b. At what minimum speed u descend past a vertical target with
relative to still water should the boat travel to reach the (a) uniform velocity (b) uniform acceleration
point B ? The velocity of flow of the river is v0. (c) non-uniform velocity (d) None of these
C B 72. The drawing shows velocity (v) versus time (t) graphs for
two cyclists moving along the same straight segment of a
b highway from the same point. The first cyclist starts at t = 0
v0
min and the second cyclist starts moving at t = 3.0 min. The
A time at which the two cyclists meet is (Both velocity-time
curves intersect at t = 4 min)
v0b 2v0b
(a) (b)
2 2
a b a 2 b2 v

v0b v0b
(c) (d)
a 2 b2 a2 2b2
68. A particle has initial velocity 10m/s. It moves due to a
constant force along the line of velocity which initially
produces retardation of 5m/s2. Then
(a) the distance travelled in first 3 seconds is 10.0m
(b) the distance travelled in first 3 seconds is 7.5m 0 3.0 4.0 t
(c) the distance travelled in first 3 seconds is 12.5m
(d) the distance travelled in first 3 seconds is 17.5m (a) 4.0 min (b) 6.0 min
69. A particle is moving in a plane with velocity given by (c) 8 min (d) 12 min
73. A sandbag ballast is dropped from a balloon that is
u u0iˆ ( a cos t ) ˆj , where iˆ and ˆj are unit vectors ascending with a velocity of 40 ft/s. If the sandbag reaches
along x and y axis respectively. If the particle is at the origin the ground in 20s, how high was the balloon when the bag
3 was dropped? Neglect air resistance.
at t = 0, find its distance from the origin at time . (a) 5200 ft (b) 6000 ft
2
(c) 5000 ft (d) 5600 ft
9 2 2
u0 9 2 2
u0 74. A particle is projected with speed 10m/s at an angle 60° with
(a) a2 (b) 2
a2 the horizontal. Then the time after which its speed becomes
2
4 2 half of initial is

3 2u02 7 2 2
u0 1 3
a2 a2 (a) sec (b) sec
(c) 2 (d) 2 2 2
4 4

70. A shot is fired with velocity 2gh from the top of a 3


(c) 1 sec (d) sec
2
mountain which is in the form of hemisphere of radius r. The
farthest points which can be reached by the shot are at a 75. An arrow is shot into the air on a parabolic path to a target.
distance (measured in a straight line) from the point of Neglecting air resistance, at its highest point
projection is (a) both velocity and acceleration vectors are horizontal
(b) the acceleration vector is zero but not the velocity
(a) (r r 2 4rh ) (b) (r r 2 4rh ) (c) the velocity and acceleration vectors are both zero
(d) the upward component of velocity is zero but not the
(c) (r r 2 4rh ) (d) (r r 2 2rh ) acceleration

MARK YOUR 67. 68. 69. 70. 71.


RESPONSE 72. 73. 74. 75.
30 IIT-JEE P HYSICS Challenger
76. For a particle moving along x-axis, which of the velocity 80. The velocity-time graph of a particle moving along a straight
versus position graphs given in options below is possible line in a given time interval is as shown in figure. Then the
(position is represented by x-coordinate of the particle) particle (with increase in time starting from t = 0 sec)
v
v (m/s)
v

(a) (b)
x x

v t (s)
t0

(c) (d) None of these


x
(a) speeds up continuously
(b) first speeds up and then speeds down
77. Two fixed points A and B are 20 metres apart. At time t = 0, (c) speeds down continuously
the distance between a third point C and A is 20 meters and (d) first speeds down and then speeds up
the distance between C and B is 10 metres. The component
81. A body is projected horizontally from the top of a tower
of velocity of point C along both CA and CB at any instant
with initial velocity 18m/s. It hits the ground at angle 45°.
is 5m/s. Then the distance between A and C at the instant all
What is the vertical component of velocity when it strikes
the three points are collinear will be
the ground ?
(a) 5 m (b) 15 m
(c) 10 m (d) None of these (a) 18 2 m / s (b) 18 m/s
78. A ball is thrown upward with initial velocity v0 = 15.0 m/s at
an angle of 30° with the horizontal. The thrower stands near (c) 9 2 m/s (d) 9 m/s
the top of a long hill which slopes downward at an angle of 82. A bomber plane moving at a horizontal speed of 20 m/s
20°. When does the ball strike the slope ? releases a bomb at a height of 80m above ground as shown.
(a) 2.49s (b) 1.13s At the same instant a Hunter starts running from a point
(c) 2.12s (d) 5.12s below it, to catch the bomb at 10 m/s. After two seconds he
79. A small ball is thrown from a height of 15m above the ground realized that he cannot make it, he stops running and
and at a horizontal distance d from a vertical wall. The ball immediately holds his gun and fires in such direction so
first hits the wall and then strikes the ground and then it that just before bomb hits the ground, bullet will hit it. What
flies back to its initial position of throwing. Take both should be the firing speed of bullet.
collisions to be perfectly elastic and neglect friction. The (Take g = 10 m/s2)
initial speed of the ball is 10 2 m/s and angle of projection
is 45° with the horizontal as shown. Find the horizontal
distance of point of throwing from the wall ‘d’ in meters. 20m/s
(Neglect air resistance and take g = 10 m/s2)
//////////////////////////////////////

80m

u = 10 2m/s

45°
d

15m 10m/s
Ground

(a) 10 m/s (b) 20 10 m / s


//////////////////////////////////////
(c) 10 10 m / s (d) None of these
(a) 20 m (b) 10 m
(c) 40 m (d) 30 m

MARK YOUR 76. 77. 78. 79. 80.


RESPONSE 81. 82.
MOTION IN ONE AND TWO DIMENSIONS 31

83. A particle is projected from the ground with an initial velocity 88. Two stones A and B are projected from an inclined plane
of 20m/s at an angle of 30° with horizontal. The magnitude such that A has range up the incline and B has range down
of change in velocity in time interval of 0.5 sec starting from the incline. For range of both stones on the incline to be
instant of projection is equal in magnitude, pick up the correct condition. (Neglect
(Neglect air friction and take g = 10 m/s2) air friction). A
(a) 5 m/s (b) 2.5 m/s
(c) 2 m/s (d) 4 m/s B
84. A bullet is fired from horizontal ground at some angle passes
3R R
through the point , ÷ , where R is the range of the
4 4 /////////////////////////////////////
bullet. Assume point of the fire to be origin and the bullet (a) Component of initial velocity of both stones along the
moves in x-y plane with x-axis horizontal and y-axis vertically incline should be equal and also component of initial
upwards. Then angle of projection is velocity of both stones perpendicular to the incline
(a) 30° (b) 53° should be equal.
(c) 37° (d) None of these (b) Component of initial velocity of both stones
85. A swimmer crosses a river with minimum possible time 10 perpendicular to the incline should be equal and also
horizontal component of initial velocity of both stones
second. And when he reaches the other end, he starts
should be equal.
swimming in the direction towards the point from where he
(c) Horizontal component of initial velocity of both stones
started swimming. Keeping the direction fixed the swimmer
should be equal and also vertical component of initial
crosses the river in 15sec. The ratio of speed of swimmer velocity of both stones should be equal.
with respect to water and the speed of river flow is (Assume (d) None of these
constant speed of river and swimmer) 89. An engineer works at a factory out of town. A car is sent for
2 3 him from the factory every day and arrive at the railway
(a) (b) station at the same time as the train. One day the engineer
5 2
arrived at the station one hour before his usual time and
9 5 without waiting for the car, started walking towards factory.
(c) (d) On his way he met the car and reached his factory 10 minutes
4 2 before the usual time. For how much time (in minutes) did
86. A man throws a ball making an angle of 60° with the the engineer walk before he met the car ? The car moves
horizontal. He runs on a level ground and catches the ball. If with the same speed everyday.
he had thrown the ball with speed v, then his average velocity (a) 55 min (b) 35 min
must be (c) 45 min (d) 60 min
(a) v (b) 2v 90. A particle is thrown up inside a stationary lift of sufficient
(c) (d) v/2 height. The time of flight is T. Now it is thrown again with
v
same initial speed v0 with respect to lift. At the time of second
87. The acceleration versus time graph for a particle moving throw, lift is moving up with speed v 0 and uniform
along a straight line is shown in the figure. If the particle acceleration g upward. The new time of flight is
starts from rest at t = 0, then its speed at t = 30 sec will be (a) T/4 (b) T (c) T/2 (d) 2T
91. A stone is projected horizontally with speed v from a height
a (m/s)² h above ground. A horizontal wind is blowing in direction
opposite to velocity of projection and gives the stone a
4 constant horizontal acceleration f (in direction opposite to
initial velocity). As a result the stone falls on ground at a
10 20 30 t (sec) point vertically below the point of projection. Then the value
of height h in terms of f, g, v is (g is acceleration due to
–4 gravity)
2 gv 2 gv 2
(a) 2 (b)
f 2f2
(a) 20 m/sec (b) 0 m/sec
(c) – 40 m/sec. (d) 40 m/sec. gv 2 2 gv 2
(c) 2 (d) 2
f f

MARK YOUR 83. 84. 85. 86. 87.


RESPONSE 88. 89. 90. 91.
32 IIT-JEE P HYSICS Challenger
92. A particle moves along x-axis with initial position x = 0. Its
velocity varies with x-coordinate as shown in graph. The 93. Velocity of a stone projected, 2 second before it reaches the
acceleration ‘a’ of this particle varies with x as – maximum height, makes angle 53° with the horizontal, then
the velocity at highest point will be (Neglect air friction and
v take g = 10 m/s2)
(a) 20 m/s (b) 15 m/s
(c) 25 m/s (d) 80/3 m/s
94. You calculate that to throw an object vertically to a height h
it needs to be launched with an initial upward velocity v0,
x assuming no air resistance. The dashed lines in figure show
the motion according to this calculation. Which of the
velocity-time graphs shows the motion of an object tossed
a a with initial upward velocity v1 that will also rise to height h,
but this time with air resistance ?
(a) x (b) x
v v

(a) t (b) t

a a

(c) x (d) x
v v

(c) t (d) t

MARK YOUR
92. 93. 94.
RESPONSE

PASSAGE-1 Vcos Vcos

We know how by neglecting the air resistance, the problems of Vsin Vsin
v
projectile motion can be easily solved and analysed. Now we
consider the case of the collision of a ball with a wall. In this case
the problem of collision can be simplified by considering the case Components of velocity
Velocity just Components of velocity
of elastic collision only. When a ball collides with a wall we can before collision just before collision just after collision
divide its velocity into two components, one perpendicular to the
wall and other parallel to the wall. If the collision is elastic then the The other parallel component of velocity will remain
constant if wall is given smooth.
perpendicular component of velocity of the ball gets reversed
Now let us take a problem. Three balls A , B & C are projected
with the same magnitude.
from ground with same speed at same angle with the
horizontal. The balls A, B and C collide with the wall during
their flight in air and all three collide perpendicularly with
the wall as shown in figure.
MOTION IN ONE AND TWO DIMENSIONS 33

4. The vertical component of velocity of A at the time when he


got the impulse is
(a) 10 m/s (b) 5 m/s
A (c) 1 m/s (d) None of these
5. What should be the minimum constant acceleration A should
B have to catch the orange before B catches it
(a) 8 m/s2
(b) 26/3 m/s2
(c) 25/3 m/s2
(d) none of these
6. The horizontal distance from the base of orange tree from
C
where A has to gain impulse to just catch the orange before
B is
200 100
(a) m (b) m
3 3
1. Which of the following relation about the maximum height
H of the three balls from the ground during their motion in 400
(c) m (d) None of these
air is correct 9
(a) HA = HC > HB (b) HA > HB = HC
(c) HA > HC > HB (d) HA = HB = HC PASSAGE-3
2. If the time taken by the ball A to fall back on ground is 4
seconds and that by ball B is 2 seconds. Then the time
taken by the ball C to reach the inclined plane after projection Two particles A and B are projected in same vertical plane as
will be shown in the figure. Their initial positions (t = 0), initial speed and
(a) 6 sec. (b) 4 sec. angle of projection are indicated in the diagram.
(c) 3 sec. (d) 5 sec. uA= 60m/s
3. The maximum height attained by ball A from the ground is
(a) 10 m (b) 15 m B
53°
(c) 20 m (d) Insufficient information
A B
100 m
PASSAGE-2 7. If initial angle of projection B = 37°. What should be initial
speed of projection of particle B, so that it hits particle A
There is an orange tree at which there is an orange hanging at a (a) 80 m/s (b) 75 m/s
height 5 m from ground. Two persons A and B each standing at a (c) 40 m/s (d) 45 m/s
horizontal distance of 100 m from the orange starts running towards 8. Suppose that the speed of B is such that an observer on
the orange. B is standing on a platform at the same level as that of particle A observes that particle B is moving at angle 8° with
orange. B runs with a constant speed of 20 m/s towards orange. A horizontal in downward direction. Initial angles of projection
starts running first horizontally with a constant acceleration and of each particle remain same as previous question. Speed of
then gets a vertical impulse so as to reach the orange. If the particle B may be
maximum height that A can jump due to this impulse is just equal (a) 80 m/s (b) 75 m/s
to the height of orange from the ground then. (c) 60 m/s (d) 40 m/s
(Take g = 10 m/s2) 9. In the situation of previous question, when observer on
particle A, observes that particle B appears to be moving at
B 8° below horizontal ( B = 37°). After how much time (in
second) distance between A and B is minimum? (t = 0
situation is shown in figure)
A
3 2 3 2
(a) sin 8 (b) cos8
100 m 5 5
5m
5 5
(c) sin 8 (d) cos8
100 m 3 2 3 2

MARK YOUR 1. 2. 3. 4. 5.
RESPONSE 6. 7. 8. 9.
34 IIT-JEE P HYSICS Challenger

PASSAGE-4 PASSAGE-6

An observer having a gun observes a remotely controlled balloon. The graph given shows the position of two cars, A and B, as a
When he first notices the balloon, it was at an altitude of 800m function of time. The cars move along the x-axis on parallel but
and moving vertically upward at a constant velocity of 5 m/s. The separate tracks, so that they can pass each other position without
horizontal displacement of balloon from the observer is 1600m. colliding.
Shells fired from the gun have an initial velocity of 400m/s at a
x
fixed angle .
(sin = 3/5 and cos = 4/5). The observer having gun waits (for
some time after observing balloon) and fires so as to destroy the
balloon. Assume g = 10 m/s2. Neglect air resistance.
A
B

400m/s 5m/s

/////////////////////////////////////////////// t
1600m Ground t1 t2 t3 t4
10. The time of flight of the shell before it strikes the balloon is 16. At which instant of time is car-A overtaking the car-B ?
(a) 2 sec (b) 5 sec (a) t 1 (b) t 2
(c) 10 sec (d) 15 sec (c) t 3 (d) t 4
11. The altitude of the collision above ground level is
17. At time t3, which car is moving faster ?
(a) 1075 m (b) 1200 m
(c) 1250 m (d) 1325 m (a) car A (b) car B
12. After noticing the balloon, the time for which observer (c) same speed (d) None of these
having gun waits before firing the shell is 18. At which instant do the two cars have the same velocity ?
(a) 45 sec (b) 50 sec (a) t 1 (b) t 2
(c) 55 sec (d) 60 sec
(c) t 3 (d) t 4
PASSAGE-5
A ball is thrown at an angle of 53° with the horizontal from the PASSAGE-7
centre of a well with the speed of 50 m/s. The ball collides elastically
with the vertical wall of the well. The total number of collision on
both walls is 12. Two particles are simultaneously projected in the same vertical
13. The minimum height of the wall so that the ball may not plane from the same point with velocities u and v at angles and
come out of the well (Take g = 10 m/s2), is to the horizontal.
(a) 80 m 19. The line joining them moves
(b) 79.45 m (a) parallel to itself (b) perpendicular to itself
(c) 40.25 m
(c) 30° to itself (d) none of these
(d) 81.5 m
14. The diameter of the well so that the particle fall back to the 20. The time that elapses when their velocities are parallel is
initial point of projection
uv sin ( ) uv sin ( )
(a) 10 m (b) 20 m (a) (b)
(c) 30 m (d) 4 m g (v cos u cos ) g (v cos u cos )
15. The impulse provided by the wall during the 3rd collision
with the wall if the mass of the ball is 1 kg is uv sin ( ) 2uv sin ( )
(c) (d)
(a) 60 N-s (b) 70 N-s g (v cos u cos ) g (v cos u cos )
(c) 80 N-s (d) 90 N-s

10. 11. 12. 13. 14.


MARK YOUR
15. 16. 17. 18. 19.
RESPONSE
20.
MOTION IN ONE AND TWO DIMENSIONS 35

21. The time that elapses between their transits through the 25. The braking force required to bring the car to a stop within
other common point is one second from the maximum speed is
10000
2uv sin ( ) 2uv sin ( ) (a) N (b) 5000 N
(a) (b) 3
g (u cos v cos ) g (u cos v cos )
5000
(c) 10000 N (d) N
2uv sin ( ) uv sin ( ) 3
(c) (d) 26. Correct acceleration-time graph representing the motion of
g (u cos v cos ) g (u cos v cos )
car is
a(m/s²)
PASSAGE-8

An object has the acceleration v/s time graph as shown.


a(m/s²) 5
10 t(s)
(a) 2 7
2 –10/3
1

0 t(s)
1 2 3 4 5 6 7 8
–1 a(m/s²)
–1.5
–2
22. When does the object return to it’s initial velocity ? 5
(a) At t = 4s (b) At t = 7s 7 t(s)
(c) At t = 8s (b)
2 10
(d) Impossible to determine from the given information –10/3
23. When is the object at rest ?
(a) At t = 0s (b) At t = 4s
(c) At t = 4s and t = 8s a(m/s²)
(d) Impossible to determine from the given information

PASSAGE-9 5

(c) t(s)
The velocity-time graph of a car moving on a straight track is 2 7 10
given below. The car weighs 1000 kg.
Velocity (m/s)

a(m/s²)
10
5

time t(s)
2 7 10
2 7 10 (sec) (d) –10/3
24. The distance travelled by the car during the whole motion is
(a) 50 m (b) 75 m
(c) 100 m (d) 150 m

MARK YOUR 21. 22. 23. 24. 25.


RESPONSE 26.
36 IIT-JEE P HYSICS Challenger
31. The expression for the velocity vy (t) valid for the time
PASSAGE-10
interval t > tb is –

With reference to figure, the projectile is fired with an initial velocity 1


(a) vy gtb gt (b) vy = – g (t – tb)
v0 = 35 m/s at an angle = 23°. The truck is moving along X with 5
a constant speed of 15m/s. At the instant the projectile is fired, the 4
back of the truck is at (c) vy = g (t – tb) (d) vy
gtb gt
5
x = 45m.
32. The time taken for rocket to reach its maximum height is –
Y 3 4
(a) tb (b) tb
5 5
15m/s
v0 1 2
(c) tb (d) tb
Truck 5 5
P

PASSAGE-12
X
x

The plot of velocity versus time graph for a particle moving along
27. Find the time for the projectile to strike the back of the truck, a straight line is shown. Answer the following three questions
if the truck is very tall. based on given information.
(a) 0.614s (b) 3.614s
v (m/s)
(c) 1.614s (d) 2.614s
28. Find the total time if the truck is only 2.0m tall
(a) 1.123s (b) 3.614s
(c) 2.635s (d) 2.614s =45°
29. Find a value of v0, all other conditions remaining the same, t (sec.)
t = 3sec.
for which the projectile his the truck at y = 3m. dv
(a) 35.3 m/s (b) 50.3 m/s 33. If v is velocity at any time t, then the value of at t = 2 sec
dt
(c) 25.3 m/s (d) 27.2 m/s is
(a) 1 m/s2 (b) – 1 m/s2
2 (d) – 2 m/s2
PASSAGE-11 (c) 2 1 m/s
34. The value of dot product of velocity and acceleration of
particle at t = 2 sec is
A physics tutor launches a home-built model rocket straight up
(a) 1 m2/s3 (b) – 1 m2/s3
into the air. At t = 0, the rocket is at y = 0 with
(c) 2 m2/s3 (d) – 2 m2/s3
vy (t = 0) = 0. The acceleration of the rocket is given by
t 5
4 35. If v is velocity at any time t, then the value of v dt is
g g t ,0 t tb 1/ 4
ay g
where tb t 2
g ,t tb
(where v is in m/s and t is in seconds)
is the time at which fuel burns out. is a positive dimensionless (a) zero (b) 2.5m
number (> 1). (c) 1.5m (d) 2m
30. The expression for the velocity vy(t) valid at all times in the
interval 0 < t < tb is – PASSAGE-13
1 5 1 5
(a) v y ( 1) gt t (b) v y ( 1) gt t For a particle moving along x-axis, the acceleration ‘a’ of the particle
5 5
in terms of its x-coordinate x is given by
1 5 1 5 a =–9x, where x is in meters and ‘a’ is in m/s2. Take acceleration,
(c) v y ( 1) gt t (d) v y ( 1) gt t
5 5 velocity and displacement in positive x-direction as positive. The
initial velocity of particle at x = 0 is u = + 6m/s.

MARK YOUR 27. 28. 29. 30. 31.


RESPONSE 32. 33. 34. 35.
MOTION IN ONE AND TWO DIMENSIONS 37

36. The velocity of particle at x = 2m will be


PASSAGE-14
(a)
6 2 m/s (b) 6 2 m/s
(c) 72 m/s (d) 0 A particle moves in x-y plane with constant acceleration
37. The maximum distance of particle from origin will be
(a) 1m (b) 2m a 6iˆ 8 ˆj (in m / s 2 ) . At time t = 0, the particle is at point having
(c) 3m (d) 4m coordinates (0, 20 m) and its initial velocity is
38. The plot of velocity versus x-coordinate of particle in
the duration it moves from origin towards the positive u 12iˆ 16 ˆj (in m / s)
x-direction, is best represented by – 39. The instant of time when speed of the particle is zero will
v
be–
(a) 1 sec (b) 3 sec
v (c) 4 sec (d) 2 sec
40. The speed of the particle at the instant position vector and
(a) x (b) x
velocity vector of the particle are mutually perpendicular
will be

(a) 12 5 m / s (b) 4 5 m/s


v
v (c) 20 m/s (d) None of these
41. The instant of time when the particle crosses x-axis is
(a) 5 sec (b) 3 sec
x (c) 4 sec (d) 6 sec
(c) (d) x

MARK YOUR 36. 37. 38. 39. 40.


RESPONSE 41.

1. Statement - 1 : A body can have acceleration even if its 3. Statement - 1 : If a body is thrown upwards, the
velocity is zero at a given instant of time. distance covered by it in the last second
Statement - 2 : A body is numerically at rest when it of upward motion is about 5 m
reverses its direction. irrespective of its initial speed.
2. Statement - 1 : Retardation is directly opposite to the Statement - 2 : The distance covered in the last second
velocity. of upward motion is equal to that
covered in the first second of downward
Statement - 2 : Retardation is equal to the time rate of
motion when the particle is dropped.
decrease of speed.

MARK YOUR
1. 2. 3.
RESPONSE
38 IIT-JEE P HYSICS Challenger
4. A particle is projected in vertical plane from horizontal Statement - 2 : Throughout the motion of the body, a
surface, with speed of projection u and angle of projection constant force acts on it.
above the horizontal. 9. Statement - 1 : Three projectiles are moving in different
paths in the air. Vertical component of
relative velocity between any of the pair
does not change with time as long as
Range they are in air. Neglect the effect of air
Statement - 1 : The horizontal range must be the friction.
farthest displacement of the particle from Statement - 2 : Relative acceleration between any of the
point of projection along the horizontal. pair of projectiles is zero.
Statement - 2 : The projectile is farthest distance from 10. Statement - 1 : The magnitude of velocity of two boats
the point of projection, when its relative to river is same. Both boats start
instantaneous velocity vector and simultaneously from same point on the
position vector (w.r.t. point of bank. They may reach opposite bank
projection) are mutually perpendicular. simultaneously moving along different
5. Statement - 1 : Two projectiles are launched from the straight line paths.
top of a cliff with same initial speed with Statement - 2 : For above boats to cross the river in
different angles of projection. They same time, the component of their
reach the ground with the same speed. velocity relative to river in direction
Statement - 2 : The work done by gravity is same in normal to flow should be same.
both the cases. 11. Statement - 1 : A particle having zero acceleration must
6. Statement - 1 : A projectile, launched from ground, have constant speed.
collides with a smooth vertical wall and Statement - 2 : A particle having constant speed must
returns to the ground. The total time of
have zero acceleration.
flight is the same had there been no
12. Statement - 1 : Two projectiles having same range must
collision.
have the same time of flight.
Statement - 2 : Horizontal component of velocity is
constant in projectile motion under
gravity.
13. Statement - 1 : During a train journey, tree outside
appear to move backwards (opposite to
the train’s motion)
Statement - 2 : Rest or motion depends on choice of
Statement - 2 : The collision changes only the
frame.
horizontal component of velocity .
14. Statement - 1 : For a particle undergoing rectilinear
7. Statement - 1 : Two stones are simultaneously
projected from level ground from same motion with constant acceleration, the
point with same speeds but different average velocity cannot be equal for
angles with horizontal. Both stones two different time intervals.
move in same vertical plane. Then the Statement - 2 : The average velocity of particle moving
two stones may collide in mid air. with constant acceleration in a time
Statement - 2 : For two stones projected simultaneously u v
from same point with same speed at interval is , where u is the initial
2
different angles with horizontal, their
velocity and v is the final velocity..
trajectories may intersect at some point.
8. Statement - 1 : A body is thrown with a velocity u 15. Statement - 1 : Acceleration of a particle in motion
inclined to the horizontal at some angle. always depends on its velocity.
It moves along a parabolic path and falls dv
to the ground. Linear momentum of the Statement - 2 : Acceleration of a particle is , where
dt
body, during its motion, will remain
conserve. v is velocity of particle at any time t.

4. 5. 6. 7. 8.
MARK YOUR
9. 10. 11. 12. 13.
RESPONSE
14. 15.
MOTION IN ONE AND TWO DIMENSIONS 39

16. Statement-1 : Let v be the speed of particle at any time Statement-2 : The horizontal range is same for two
projectiles projected with same speed if
dv
t. If is a non-zero constant then one is projected at an angle with the
dt
horizontal and other is projected at an
speed of particle may be minimum at angle (90° – ) with the horizontal.
some time. (Neglect air friction)
Statement-2 : The speed v of a moving particle may be 20. Statement-1 : A particle starts from rest under action
minimum at time t = t0 when v = 0. The of (nonzero) variable acceleration such
speed v of a moving particle may also be that direction of acceleration is always
dv fixed (or constant). Then this particle
minimum at time t = t0 when = 0 and moves in a straight line.
dt
Statement 2 : The motion of particle starting from rest
d 2v
= + ve . is always along a straight line.
dt 2
21. Statement-1 : The rate of change of velocity with time
17. Statement-1 : The velocity of a point sized observer in for a projectile in motion depends on
motion as measured by himself is always direction of its velocity. (Neglect air
zero. friction)
Statement-2 : The velocity of a particle A as measured Statement-2 : Acceleration a of a particle is defined
by another particle B is v A v B , where as rate of change of velocity v with time

v A and v B are velocities of A and B dv


t, that is, a = .
respectively. dt
18. Statement-1 : A stone is projected from ground and 22. Statement-1 : Two stones are projected with different
moves on a parabolic path. Then the velocities from ground from same point
magnitude of displacement of particle and at same instant of time. Then these
from point of projection may increase stones cannot collide in mid air. (Neglect
with time over the entire time of flight. air friction)
(Neglect air friction) Statement 2 : If relative acceleration of two particles
Statement-2 : The horizontal component of velocity initially at same position is always zero,
of projectile remains constant for the then the distance between the particles
duration of time of flight. Hence the either remains constant or increases
horizontal component of displacement continuously with time.
of particle from point of projection keeps 23. Statement-1 : In a given time interval, the average
on increasing with time. (Neglect air speed of a particle may be equal to
friction) magnitude of average velocity.
19. Statement-1 : A man projects a stone with speed u at Statement-2 : The magnitude of instantaneous
some angle. He again projects a stone velocity is equal to instantaneous speed
with same speed such that time of flight of a particle in motion.
now is different. The horizontal ranges
in both the cases may be same. (Neglect
air friction)

MARK YOUR 16. 17. 18. 19. 20.


RESPONSE 21. 22. 23.
40 IIT-JEE P HYSICS Challenger

1. ABCD is a wire frame of identical wires in which point D is 4. A particle has an initial velocity of 60m/s up to the right at a
given velocity v as shown in figure. Choose the correct slope of 0.75. The components of acceleration are constant
statement (s) at ax = –3.6 m/s2 and ay = – 6 m/s2. Choose the correct
(a) velocity of point A along x-axis will be v/2 option(s).
(b) speed of point A will be v (a) The radius of curvature at the start of the path is
1363.64m
(c) speed of point A along y-axis will be v/2
(b) The radius of curvature at the top of the path is 384m
(d) velocity of point A will be equal to velocity of point C
(c) The radius of curvature at the start of the path is
323.64m
y (d) The radius of curvature at the top of the path is 184m
x
A 5. A particle is moving along X-axis whose position is given

t3
by x = 4 9t . Mark the correct statement(s) in relation
3
B 30° to its motion.
D v (a) Direction of motion is not changing at any of the
instants
(b) Direction of motion is changing at t = 3s
C (c) For 0 < t < 3s, the particle is slowing down
(d) For 0 < t < 3s, the particle is speeding up
2. A particle moves uniformly with speed v along the parabolic 6. The upper end of the string of a simple pendulum is fixed
path y = k x2. Taking k as a positive constant, the magnitude to a vertical z-axis, and set in motion such that the bob
of acceleration of the particle at x = 0 is given by moves along a horizontal circular path of radius 2m, parallel
(a) 2k v2 in magnitude (b) along x-axis to the x-y plane, 5m above the origin. The bob has a speed
of 3m/s. The string breaks when the bob is vertically above
k v2 the x-axis, and it lands on the xy plane at a point (x, y). Then
(c) in magnitude (d) along y-axis
2 z
2m
3. A man can swim with a velocity v relative to water. He has to
cross a river of width d flowing with a velocity u (u > v). The
distance through which he is carried downstream by the
river is x. Which of the following statements are correct? 5m
(a) If he crosses the river in minimum time, x = du/v
y
(b) x cannot be less than du/v O
(c) For x to be minimum, he has to swim in a direction
making an angle of /2 + sin–1 (v/u) with the direction
of the flow of water. x
(d) x will be maximum if he swims in a direction making an (a) x = 2m (b) x > 2m
angle of /2 – sin–1 (v/u) with the direction of the flow (c) y = 3m (d) y = 5m
of water.

MARK YOUR 1. 2. 3. 4. 5.
RESPONSE 6.
MOTION IN ONE AND TWO DIMENSIONS 41

(a) >
7. An imperfectly elastic ball is projected with velocity gh
(b) t cos = (t – T) cos
at an angle with the horizontal, so that it strikes a vertical
(c) (t – T) cos = t cos
wall distant c from the point of projection, and returns to the
point of projection. If e is the coefficient of restitution, then 1 2 1
(d) (u sin ) t – gt = (u sin ) (t – T) – g (t T )2
2 2
10. A stunt man is to drive an auto across the water-filled gap
A
shown in figure. Choose the correct option(s).
(In FPS g = 32.2 ft/s2)
gh
v0

28.56
A

/////////////////////////
O c N
1 B
2 ///

////////////////
c ///
(a) Time from A to O = ///
///
gh cos ///

c
(b) Time from A to O = ///////////////////////////////
e gh cos
40ft
c
(c) Time from O to A =
gh cos (a) The auto’s minimum take-off velocity is 16.76 ft/sec
c (b) The angle of the landing ramp is 45°
(d) The coefficient of restitution = (c) The angle of the landing ramp is 30°
(h sin 2 c)
8. A particle is projected with velocity V along a smooth (d) The auto’s minimum take-off velocity is 32.76 ft/sec
horizontal plane in a medium whose resistance per unit of 11. A ball is projected from a building of height 20m at a speed
mass is µ times the cube of the velocity. Choose the correct of 30m/sec making an angle of 30° with the horizontal. Then
options(s).
30m/s
(a) The distance it has described in time t is
30°
1
[ (1 2 V 2t ) 1]
V

V
(b) Velocity at time t is .
(1 2 V 2 t ) 20m

(c) The distance it has described in time t is


1
[ (1 2 V 2t ) 1]
V
D
V (a) time after which ball strikes the ground is 4sec
(d) Velocity at time t is
(1 2 V 2 t ) (b) ball comes to a height of 20m again after 3 sec

9. Two shells are fired from a cannon with speed u each, at 5 3


angles of and respectively with the horizontal. The time (c) value of is tan–1
9
interval between the shots is T. They collide in mid air after
time t from the first shot. Which of the following condition(s) (d) Value of D is 60 3m
must be satisfied ?

MARK YOUR
7. 8. 9. 10. 11.
RESPONSE
42 IIT-JEE P HYSICS Challenger
12. A trapezium with parallel sides of length 2a and 4a and non- (a) The value of t at the instant when the train reaches P
parallel sides each of length 2a is placed on the ground with is 40 sec.
its plane vertical and with the longest side in contact with (b) The magnitude of the acceleration of the train in the
the ground. A ball is projected from the corner lying on the
1
ground so as just to graze the other three corners of the time interval 50 < t < 80 (t in sec) ism / s2 .
trapezium. Choose the correct options 15
(c) The distance of the train from A at the end of the 120
(a) The greatest height reached by the ball is 2 a / 3 second period is 35m.
(d) The distance of the train from A at the end of the 120
32a
(b) The total time of flight is . second period is 105m.
g 3 15. A particle moves in a straight line, its acceleration directed
towards a fixed point O in the line and is always equal to µ
(c) The greatest height reached by the ball is 4 a / 3 (a5/x2)1/3 when it is at a distance x from O. If it starts from
rest at a distance ‘a’ from O. Choose the correct option(s)
8a
(d) The total time of flight is . (a) Particle will arrive at O with a velocity a 6
g 3
13. A battle ship is streaming ahead with velocity u. A gun is (b) Time in which particle will arrive at O is (8 /15) 6
mounted on the ship so as to point straight backward and is
set at an angle of elevation . If v be the velocity of (c) Particle will arrive at O with a velocity a 3
projection relative to the gun, choose the correct option(s)
(a) The range is (2v/g) sin (v cos – u). (d) Time in which particle will arrive at O is (4 / 15) 6
(b) The angle for maximum range is 16. Which of the following statement(s) is/are true
(a) The magnitude of the sum of two vectors must be
1 u u 2 8v 2 greater than the magnitude of either vector
cos .
4v (b) If the speed of a moving particle is constant, the
acceleration must be zero
(c) The range is (2v/g) sin (v cos + u) (c) Distance travelled by an object must be greater than or
(d) The angle for maximum range is equal to magnitude of displacement vector
(d) If a particle is moving along a circle it must be
1 u u 2 8v 2 accelerating
cos .
2v 17. Two small balls A and B are launched in the same vertical
plane simultaneously, with same speed of 20 m/s at t = 0.
14. The figure shows the (v, t) graph for a miniature train as it Ball A has an initial horizontal velocity and ball B has initial
moves along a straight track. At time t = 0 the train passes a velocity at an angle above the line joining A and B as
point A and is moving at 3m/s. The farthest point from A shown. If the projectiles collide in mid-air at time t, then
reached by the train in the 120 second period is P. Choose
the correct option(s) A
20m/s
v(m/s)

3 100m
20m/s

50 80 120
0 t(s) B
30 40
100 3 m
(a) = 30° (b) = 60°
–2
20 10
(c) t s (d) t s
3 3

MARK YOUR 12. 13. 14. 15. 16.


RESPONSE 17.
MOTION IN ONE AND TWO DIMENSIONS 43

18. A car starts from rest at t = 0 and for the first 4 second of its (a) The maximum height of the stone (above the ground)
rectilinear motion the acceleration ‘a’ (ms–2) at time t (sec) is 71.25m
after starting is given by a = 6 – 2t. Choose the correct (b) The time when the stone hits the ground is 6 sec
option(s).
(c) The (v, t) graph for the motion of the stone up to the
(a) The maximum velocity of the car is 6 m/s time when it hits the ground is
(b) The velocity of the car after 4 seconds is 8 m/s (d) The (h, t) graph showing the height of the stone as a
(c) The distance travelled up to 4 seconds is 80/3 m function of time is
(d) The maximum velocity of the car is 9 m/s
19. A velocity-time graph of a particle moving rectilinearly can 71.25m
give which of the following quantities with time
(a) Change in speed (b) Change in velocity 30m
(c) Distance covered (d) Change in position
t
20. A row-boat is observed travelling downstream on a flowing 2.5sec 6sec
river at a speed of 8 m/s with respect to the shore. A motor-
boat comes from the opposite direction at a speed of 10m/s 23. If a point moves with constant acceleration and u1 and u2
with respect to the water. 10 seconds after meeting each are the initial and final velocities then choose the correct
other they are at 160 meters from each other. Choose the option(s).
correct option(s)
(a) The space average of the velocity over any distance is
(a) Speed of the river flow is 2 m/sec
(b) The speed of row-boat in still water is 4 m/sec 2 u12 u1u2 u22
.
3 u1 u 2
(c) Speed of the river flow is 3 m/sec
(d) The speed of row-boat in still water is 6 m/sec
u1 u2
21. As a car passes the point A on a straight road, its speed is 10 (b) The time-average is
2
m/s. The car moves with constant acceleration a m/s2 along
the road for T seconds until it reaches the point B, where its (c) The space average of the velocity over any distance is
speed is V m/s. The car travels at this speed for a further 10
1 u12 u1u2 u22
seconds, when it reaches the point C. From C it travels for a .
3 u1 u2
further T seconds with constant acceleration 3a m/s2 until it
reaches a speed of 20 m/s at the point D. Choose the correct
option(s). 2u1u 2
(d) The time-average is
u1 u 2
[Given that the distance between A and D is 675m]
(a) Value of V is 12.5 m/sec(b) Value of a is 1/8 m/sec2 24. Two particles are projected from a horizontal plane with the
(c) Value of T is 10 sec (d) Value of T is 20 sec same initial velocity v0 at two different angles of projection
22. A stone is thrown vertically upwards with an initial velocity 1 and 2 , such that their ranges are the same. The ratio of
of 25 m/s from a point 30 meters above the ground. their maximum heights reached is/are
Choose the correct option(s). [Given g = 10 m/s2] (a) tan2 1
v
(b) cot2 2
25m/s
(c) sin2 1 cosec2 2
6sec
2.5sec t
(d) sin2 1 cos2 2

MARK YOUR 18. 19. 20. 21. 22.


RESPONSE 23. 24.
44 IIT-JEE PHYSICS Challenger
25. In figure for the motion of an object along the x-axis, choose a a
the correct options (approximately)
x,m

15 (a) (b)
D

t2 t t2 t
10 t1 t1
C F
5 a a
E

0
J 10 20 H t,s
(a) The instantaneous velocity of the object at point D is t1 t2 t1 t2
zero t t
(b) The instantaneous velocity of the object at point C is (c) (d)
1.3 m/s
(c) The instantaneous velocity of the object at point E is
–0.13m/s
28. Two balls are projected with same speed from a point on
(d) The instantaneous velocity of the object at point E is
ground at same moment. For the horizontal range of both
– 1.3m/s
balls to be same, the angles of projection for both balls
26. Figure shows the velocity of a particle as it moves along the respectively is/are
x-axis. (a) 40°, 60° (b) 57°, 33°
(c) 53°, 37° (d) 45°, 45°
v (m/s) 29. An inclined plane as in the figure makes an angle with the
horizontal. A groove OA cut in the plane makes an angle
A
7 with OX. A short smooth cylinder is free to slide down the
6 groove under the influence of gravity, starting from rest at
5 the point (x0, y0). Let = 30°, x0 = 3m, y0 = 4m.
4
Choose the correct options –

3 Y
B D
2
C
1 A
x0
0
1 t (s) 2 g=9.8m/s²
a
y0
Short
(a) The acceleration at A is –9.6 m/s2 cylinder

(b) The acceleration at C is zero O X


(c) The acceleration at A is zero
(d) The acceleration at C is –9.6 m/s2 (a) Downward acceleration along the groove is 3.92 m/s2
27. Each of the three graphs represents acceleration versus time (b) The time to reach O is 1.597s
for an object that already has a non-zero positive velocity at (c) Velocity at O is 6.26 m/s
time t1. Which graph shows an object whose speed is (d) Velocity at O is 3.12 m/s
increasing for the entire time interval between t1 and t2 ?

MARK YOUR
25. 26. 27. 28. 29.
RESPONSE
MOTION IN ONE AND TWO DIMENSIONS 45

30. A stone is projected from level ground at time t = 0. Let vx (a) net work done on the object in 30 sec is zero
and vy are the horizontal and vertical components of velocity (b) the average acceleration from 0 to 30 sec of the object
at any time t ; x and y are displacements along horizontal is zero
and vertical from the point of projection at any time t. Then (c) the average velocity from 0 to 30 sec of the object is
(a) vy – t graph is a straight line zero
(b) y – t graph is a straight line passing through origin (d) the average force from 0 to 30 sec on the object is zero.
(c) x – t graph is a straight line passing through origin 34. A ball is released from rest at the edge of a deep ravine.
(d) vx – t graph is a straight line
dy
31. Two stones are projected from level ground. Trajectories of Assume that air resistance gives it an acceleration of b
,
dt
two stones are shown in figure. Both stones haves same
where y is measured positive downward. (This negative
maximum heights above level ground as shown. Let T1 and
T2 be their time of flights and u1 and u2 be their speeds of dy
projection respectively (neglect air resistance). Then acceleration is proportional to its speed, , the positive
dt
constant b can be found by experiment.)
y
dy
The ball has a total acceleration of b g , and so
dt

1 2 d2y dy
2
b g ............ (1)
dt dt
x is the differential equation of motion. Choose the correct
options
(a) T2 > T1 (b) T1 = T2 (a) Solution to differential equation (1) is
(c) u1 > u2 (d) u1 < u2 y = k (e–bt – 1) + (g/b) t
32. If a particle moving along a straight line under uniform (b) At t = 0, k = g/b2
acceleration covers successive equal distances (s each) in (c) Assuming b = 0.1 s–1, the distance fallen and the speed
time intervals t1, t2 and t3 respectively. Then the expression reached after 10s are 360m and 62 m/s.
for average speed of particle in covering the given distance (d) After 1 min the ball will have essentially reached its
3s is (Assume the particle is speeding up for the entire terminal velocity of 9.8 m/s.
journey)
35. A person, standing on the roof of a 40m high tower, throws
s s s s s s a ball vertically upwards with speed 10m/s. Two seconds
(a) (b) later, he throws another ball again in vertical direction (use
t1 t2 t3 t1 t2 t3
g = 10 m/s2). Both the balls hit the ground simultaneously.
3s s s s (a) The first stone hits the ground after 4 seconds
(c) t1 t2 t3 (d) (b) The second ball was projected vertically downwards
t1 t2 t3
with speed 5m/s
33. v-t graph of an object of mass 1 kg is shown
(c) The distance travelled by the first ball is 10m greater
than the distance travelled by the second ball.
v(m/s) (d) Both balls hit the ground with same velocities.
36. A particle is projected vertically upwards in absence of air
resistance with a velocity u from a point O. When it returns
20 to the point of projection
(a) its average velocity is zero
10 (b) its displacement is zero
(c) its average speed is u/2
(d) its average speed is u
10 20 30 t( sec)

MARK YOUR 30. 31. 32. 33. 34.


RESPONSE 35. 36.
46 IIT-JEE P HYSICS Challenger
37. A helicopter is trying to land on a submarine deck which is (b) Speed of helicopter relative to the air is
moving south at 17 m/s. A 12 m/s wind is blowing into the
west. If to the submarine crew the helicopter is descending
( 12iˆ 17 ˆj 5kˆ) m/s
vertically at 5 m/s, choose the correct options (a) Speed of helicopter relative to the water is
( 12iˆ 17 ˆj 5kˆ) m/s
Z (b) Speed of helicopter relative to the air is
(17 ˆj 5kˆ ) m/s
38. A particle is moving rectilinearly so that its acceleration is
^ given as a = 3t2 + 1 m/s2. Its initial velocity is zero –
k
(a) The displacement of the particle in 1 sec will be 2m
(b) The velocity of the particle at t = 1 sec will be 2m/s
N S (c) The particle will continue to move in positive direction
^
j Y (d) The particle will come back to its starting point after
^
i some time
39. A ball of small size is rolled off the edge of a horizontal table.
The ball has an initial speed v0 and lands on the floor at
X some distance from the base of the table. Which of the
following statements concerning the fall of the ball is/are
true ?
(a) Speed of helicopter relative to the water is (a) The time of flight depends on the height of the table
(17 ˆj 5kˆ ) m/s (b) The horizontal component of average velocity for the
duration of flight will be v0.
(c) The bat will accelerate for the duration of flight
(d) The ball will have a longer flight time if v0 is increased.

MARK YOUR
37. 38. 39.
RESPONSE

1. Match the situation given in column I with the possible curves in column II
Column I Column II
Position

(A) Particle moving with constant speed (p)

time
Position

(B) Particle moving with increasing acceleration (q)

time
MOTION IN ONE AND TWO DIMENSIONS 47

Velocity
(C) Particle moving with constant negative acceleration (r)

time

Position
(D) Particle moving with zero acceleration (s)

time

2. Figure shows a rigid solid hemisphere of mass m and radius R and a wall in y-z plane in a gravity free space. The velocity of centre
of mass of body is ux i + uy j before collision and vx i + vy j after an elastic collision with the wall. Consider that the wall is
frictionless and the body had zero angular velocity before the collision.

Column I Column II
(A) If = 45°, ux 0, uy 0 (p) ux vx
(B) If = 45°, ux 0, uy = 0 (q) uy vy
(C) If = 90°, ux 0, uy 0 (r) Body will rotate after collision
(D) If = 90°, ux 0, uy = 0 (s) None of these
3. A body is projected with velocity v and angle of projection is .
Column I Column II
(A) Variation of kinetic energy (K) with respect (p) Linear
to horizontal position (x) is
(B) Variation of potential energy (U) with respect to (q) Parabolic
horizontal position (x) is
(C) Kinetic energy at the highest point is (r) Minimum
(s) Maximum

1. 2. 3.

MARK YOUR
RESPONSE
48 IIT-JEE PHYSICS Challenger
4. A particle is projected vertically upwards with speed 20 m/s from the top of a tower of height 20m as shown in figure. Given B is
top most point of trajectory and C is at same height as A.

B
20m/s
A C

20m
D

Column I Column II
(A) Ratio of maximum height from ground (BD) (p) 1 / 2
to the initial height from ground (AD) is
(B) Ratio of distance travelled in 1st second to (q) 1
the distance travelled in 2nd second is
(C) Ratio of initial speed at A to the final just before (r) 2
reaching to ground (D) is
(D) ratio of time taken from A to C and time taken (s) 3
from A to B is
5. Column I shows the position-time graph of particles moving along a straight line.
Column I Column II
x

A B
(p) Acceleration a > 0
C D (q) Acceleration a < 0
(r) Speeding up
t (s) Speeding down
6. Match the columns.
Column I Column II
(A) Particles are projected at same speed from same (p) Time of flight will be same
point on level ground such that their ranges are same
(B) Particles are projected from same point on ground (q) Speed just before reaching ground will be same
such that maximum heights reached are same.
(C) Particles are projected horizontally from same (r) Vertical component of velocity just before reaching
point at a height with different initial velocities ground will be same
(D) Particles are projected from the same point at a height (s) Minimum kinetic energy during the flight will be equal.
with same initial speed, direction of velocity makes
equal angle with horizontal one below and the other
above horizontal

4. 5. 6.

MARK YOUR
RESPONSE
MOTION IN ONE AND TWO DIMENSIONS 49

7. For a particle moving in x-y plane initial velocity of particle is u u1iˆ u2 ˆj and acceleration of particle is always a a1iˆ a2 ˆj
where u1, u2, a1, a2 are constants. Some parameters of motion is given in column I, match the corresponding path given in
column II.
Column I Column II
(A) If u1 0, u2 = 0, a1 0, a2 0 (p) path of particle must be parabolic
(B) If u1 0, u2 0, a1 0, a2 0 (q) path of particle must be straight line
(C) If u1 = 0, u2 = 0, a1 0, a2 0 (r) path of particle may be parabolic
(D) If u1 0, u2 0, a1 0, a2 0 (s) path of particle may be straight line

8. A truck is moving on a straight horizontal road towards east with a constant speed 10m/s. In each situation of column I, a ball is
thrown in different ways. Match each situation of column I with the nature of trajectory of ball as seen by different observers as
given in column II.
Column I Column II
(A) Ball is thrown vertically upward from the truck (p) Motion of ball as seen from ground is straight line.
(B) Ball is thrown at 60° to horizontal from ground (q) Motion of ball as seen from truck is
with a speed of 20m/s in east direction straight line.
(C) Ball is thrown vertically upward from ground (r) Motion of ball as seen from ground is parabola
(D) Ball is thrown at 60° with horizontal towards west (s) Motion of ball as seen from truck is parabola.
at a speed of 20m/s from the truck

7. 8.

MARK YOUR
RESPONSE
50 IIT-JEE P HYSICS Challenger
1. Three particles A, B and C are situated at the vertices of an 6. A body is projected downwards at an angle of 30º to the
equilateral triangle ABC of side d = 50 cm at t = 0. Each of the horizontal with a velocity of 9.8 m/s from the top of a tower
particles moves with constant speed v = 2 m/s. A always has 29.4 m high. How long will it take before striking the ground?
its velocity along AB, B always along BC and C along CA.
7. A particle having a mass 0.5 kg is projected under gravity
At what time( in sec), will all the particles meet each other?
2. Two cars started simultaneously towards each other from with a speed of 98 m/s at an angle of 60º. What is the
towns A and B which are 480 km apart. It took first car magnitude of the change in momentum in kg m/s of the
travelling from A to B 8 hours to cover the distance and particle after 10 s ?
second car travelling from B to A 12 hours. Determine the 8. A stair case contains three steps, each 10 cm high and 20 cm
distance (in km) from town A where the cars meet. Assuming wide. What should be the minimum horizontal velocity
that both the cars travelled with constant speed. (in m/s) of a ball rolling off the uppermost plane so as to hit
3. Two trains A and B, 100 km. apart, are travelling towards each directly the lowest plane (take g = 10m/s).
other with starting speeds of 50 km/hr. for both. The train A is
accelerating at 18 km/hr² and B is decelerating at 18 km/hr².
Find the distance (in km) from the initial position of A of the
point when the engines cross each other.
A P
x B
100km
4. A ball is thrown upwards from the ground with an initial 9. A particle is projected from a point O with an initial speed of
speed of u. The ball is at a height of 80m at two times, the 30 m/s to pass through a point which is 40m from O
time interval being 6s. Find u (in m/s). Take g = 10 m/s2. horizontally and 10m above O. There are two angles of
5. Two swimmers leave point A on one bank of the river to projection for which this is possible. If these angles are
reach point B lying right across on the other bank. One of and then find the value of tan[–( + )].
them crosses the river along the straight line AB while the
10. Two boys stationed at A and B fire bullets simultaneously
other swims at right angles to the stream and then walks the
distance that he has been carried away by the stream to get at a bird stationed at C. The bullets are fired from A and B at
to point B. What was the velocity u (in kph) of her walking if angles of 53° and 37° with the vertical. Both the bullets fire
both swimmers reached the destination simultaneously ? The the bird simultaneously. What is the value of v A if
stream velocity is 2km/hr and the velocity of each swimmer vB = 60 units? Given : tan 37° = 3/4
with respect to water is 2.5 km/hr.

1. 2. 3. 4. 5. 6.

MARK
YOUR
RESPONSE 7. 8. 9. 10.
MOTION IN ONE AND TWO DIMENSIONS 51

1 (b) 11 (b) 21 (a) 31 (c) 41 (b) 51 (a) 61 (b) 71 (a) 81 (b) 91 (a)
2 (c) 12 (d) 22 (c) 32 (a) 42 (d) 52 (b) 62 (a) 72 (b) 82 (c) 92 (a)
3 (a) 13 (c) 23 (a) 33 (b) 43 (a) 53 (a) 63 (a) 73 (d) 83 (a) 93 (b)
4 (b) 14 (a) 24 (b) 34 (b) 44 (a) 54 (a) 64 (a) 74 (b) 84 (b) 94 (b)
5 (b) 15 (a) 25 (b) 35 (b) 45 (b) 55 (a) 65 (a) 75 (d) 85 (a)
6 (b) 16 (d) 26 (c) 36 (c) 46 (d) 56 (b) 66 (b) 76 (d) 86 (d)
7 (c) 17 (c) 27 (a) 37 (d) 47 (b) 57 (a) 67 (a) 77 (b) 87 (c)
8 (b) 18 (b) 28 (b) 38 (b) 48 (a) 58 (a) 68 (c) 78 (a) 88 (b)
9 (d) 19 (a) 29 (b) 39 (c) 49 (c) 59 (b) 69 (a) 79 (a) 89 (a)
10 (d) 20 (d) 30 (a) 40 (b) 50 (a) 60 (a) 70 (a) 80 (b) 90 (c)

1 (a) 6 (b) 11 (a) 16 (a) 21 (a) 26 (d) 31 (d) 36 (d) 41 (a)


2 (c) 7 (a) 12 (b) 17 (b) 22 (b) 27 (d) 32 (b) 37 (b)
3 (c) 8 (c) 13 (b) 18 (b) 23 (d) 28 (c) 33 (a) 38 (d)
4 (a) 9 (d) 14 (b) 19 (a) 24 (b) 29 (a) 34 (b) 39 (d)
5 (c) 10 (b) 15 (a) 20 (b) 25 (c) 30 (b) 35 (c) 40 (a)

1 (a) 4 (c) 7 (d) 10 (a) 13 (a) 16 (a) 19 (a) 22 (a)


2 (a) 5 (a) 8 (d) 11 (d) 14 (d) 17 (a) 20 (d) 23 (b)
3 (a) 6 (a) 9 (a) 12 (c) 15 (d) 18 (b) 21 (d)

1 (a, b) 6 (a, c) 11 (a, b, c, d) 16 (c, d) 21 (a, b, d) 26 (a, b) 31 (b, d) 36 (a, b, c)


2 (a, d) 7 (b, c, d) 12 (b, c) 17 (a, d) 22 (a, b, c, d) 27 (a, b) 32 (b, c) 37 (a, b)
3 (a, c) 8 (a, b) 13 (a, b) 18 (b, c, d) 23 (a, b) 28 (b, c, d) 33 (a, b, d) 38 (b, c)
4 (a, b) 9 (a, b, d) 14 (a, b, c) 19 (a, b, c, d) 24 (a, b, c) 29 (a, b, c) 34 (a, b, c, d) 39 (a, b, c)
5 (b, c) 10 (b, d) 15 (a, b) 20 (a, d) 25 (a, b, c) 30 (a, c, d) 35 (a, c, d)

1. A - q, s; B - r; C - p; D - s 2. A - p, r; B - p, r; C - p; D - p
3. A - q; B - q; C - r 4. A - r; B - s; C - p; D - r
5. A - q, s; B - q, r; C - p, r; D - p, s 6. A - q; B - p, r; C - p, r; D - q, r
7. A - p, B - p, C - q, s ; D - r, s 8. A - q, r ; B - q, r; C - p, s; D - p, s

1 0.83 2 288 3 59 4 50 5 3.0


6 2 7 49 8 2 9 4 10 80
52 IIT-JEE P HYSICS Challenger

1. (b)
u u B
P H
Q u A C
u=0 a=gcos h
uII
D
a=gsin
a=gsin 1 2 1
Now y uyt ayt h u sin 3 g 32
2 2
It can be observed from figure that P and Q shall collide
if the initial component of velocity of P on inclined 1
h 10 3 10 9 h 30 45
plane i.e., along incline uII = 0 that is particle is projected 2
perpendicular to incline. h = 45 – 30 = 15m
Time of flight on an inclined plane of inclination is
given by 1 72v 2 2
4. (b) As when they collide vt t R vt
u 2 25 R
2u sin
T ,
g cos 5 R
t
6v
2u sin 90
4 u 10 m/s vt
1 Now, angle covered by A
10 R
2
2. (c) The acceleration vector shall change the component 11
of velocity u|| along the acceleration vector. Put t , angle covered by A
6
v2 u 2 sin 2
r 5. (b) h1
an 2g
Radius of curvature rmin means v is minimum and an is
u 2 sin 2 (90 ) u 2 cos 2
maximum. Thus at point P when component of velocity h2
parallel to acceleration vector becomes zero, that is 2g 2g
u|| = 0 2 2
1 u 2 sin cos 1 R
h1h2
u|| = 8 cos 30° 4 g 4 2

30° R 4 h1h2
6. (b) ( vbc ) x ( vb ) x ( vc ) x
20 cos 60 (vb ) x 30
a=2m/s² u = 8sin30° P
= 4m/s (vb ) x 40
a=2m/s² u =4m/s
(vbc ) y (vb ) y (vc ) y
u 2 42 20 sin 60 (vb ) y 0
R 8 metres.
a 2
3. (a) TAC = 2 sec.
(vb ) y 10 3

2u sin (vb ) y 10 3 3
So, 2 u sin g 10m/s tan
g (vb ) x 40 4
MOTION IN ONE AND TWO DIMENSIONS 53

dx 1
7. (c) x = 6t, v x 6 = 1 + 4 × (0.6) 8 (0.6)2 = 4.84 m
dt 2
h=vt sinα-1/2 gt^2=vt cosα
y = 8t – 5t2 , tanβ.(resolve para v)
dy 0.75 m
vy 8 10t
dt 37°
Initial (t = 0) v y 8 1m

v v x2 v 2y 62 82 10 m/s u2 u2 100 10
16. (d) tan R m
2u sin ( ) gt cos Rg 10 3g tan
3
8. (b) t .; u 17. (c) Say at any instant, the velocity makes an angle with
g cos 2 sin ( )
9. (d) Ranges for complementary angles are same the x-axis.
5 13 u u (cos i sin j )
Required angle =
2 36 36
dv d d
1 2 1 3 a u – sin i cos j ...(1)
10. (d) KEhighestpoint = mu cos 2 mu 2 dt dt dt
2 2 4
3 dy
K Now, tan cos x
4 dx
11. (b) pi mu cos 45 iˆ mu sin 45 ˆj sec2
d
– sin x
dx
dt dt
mu ˆ mu ˆ
= i j
2 2 d dx
– cos 2 sin x
dt dt
mu ˆ mu ˆ
pf i j
2 2 dx
Now, at x = /2, = 0°, u
dt
pf pi 2mu ˆj
d
12. (d) Use 45 in the formula for range down the –u
dt
incline plane.
Putting this in (1), we get
13. (c)
(a) u2
u= 2gh
v2 36
18. (b) aT 0, aR
h R R
(–h,x)
x 1 d 2 y / dx 2 3/ 2
3
10 2
R
Using equation to trajectory [1 (dy / dx) 2 ] 2
2
gx
h x tan(0 ) x = 2h dy x d2y 1
2(2gh)(cos 2 0 ) and 2
dx 10 dx 10
u2 10sin 37 10sin 37
14. (a) H H 9 2
2g 2 g cos 53 aR
10
36
H 3m (vr t )2 (vmr t )2
3
2 10 19. (a) Avg. speed = 3
5 t
15. (a) Speed of plank when the rod leaves the plank
vr2 5 9 vr 2m / s
v2 = 2 × 8 × 1 = 16 v = 4 m/s
By constraint relation speed of the rod. 20. (d) a = 2 + |t – 2|
u/4 = tan 37° u = 3 m/s (vertically upwards) For t 2
a=2–t+2
2 3 a=4–t
Time period of its flight = 0.6s
10
54 IIT-JEE P HYSICS Challenger
dv = (4 – t) dt 25. (b) It is possible only when 45
v = 4t – t2/2 26. (c)
at t = 2, v = 6 m/s
For t > 2 x
a=2+t–2=t u
v t
g cos 30° g sin 30°
dv tdt g
6 2

t 30°
v–6 t2 / 2 y
2
X-direction
t2
v 4 vx = u x + a x t
2 0 = u cos 30° – g sin 30° .t
at t = 4, v = 12 m/s
21. (a) From v = u + at, we have u 3 gt ... (i)
Y-direction
v 1
xy = u y t + ayt 2
A B 2
1
20 cos 30° = u cos 60° t + g cos 30° t2
2
O C t 1 2
t1 t2 t3 20 3 ut gt 3
2
v 0 t1 t1 v/ u2 3 3 3u 2
20 3 g. 2
0 v – t3 t3 v/ g 2 g

1 1 v2 v2 5u 2
Now, S vt1 vt2 vt3 vt2 20 =
2 2 2 2 2g
S v 1 1 u = 80 m/s = 4 5 m/s.
t2 –
v 2 27. (a) From figure,

S v 1 1 3
Hence, t1 t2 t3 tan 3 30
v 2 1

22. (c) v cg v CP v Pg Y ( 3, 1)

–R i 2 j vSC

–2i 2 j
3m
23. (a) v SB vj vs 3i
1m
O X
100 vBC
vS v j – 3i and v = 2m/s
50
28. (b) v= 2
vS v 2 (3)2 t = 6 s.

22 9 v
13 m/s a = m / s2 .
Drift = 50 × 3 = 150 m t 3 2
24. (b) 10 – v cos 60° = 0 29. (b) Resistance = µv
v = 20 m/s Equation of motion is
v 2 sin 2 60 d2x
H 15m m mg v
2g dt 2
MOTION IN ONE AND TWO DIMENSIONS 55

d2x tan
20 4
tan 1
4
i.e. g v
dt 2
m 15 3 3
This may be written as 31. (c) n balls are thrown per second.

dv dv 1
g v or v g Time interval between two balls thrown = sec
dt m dt m n
In this time it reaches highest point
This is a linear differential equation in v and t, so the
v = u + at
integrating factor is eµt/m, hence its solution is
t/m t/m
1 g
ve ge dt C 0 u g u
n n
t/m m t/m 2
i.e. ve ge C ............... (1) g
v2 u 2 2as 02 2 ( g) h
n
m g
Initially, when t = 0, v = 0, C g. h
2n 2
Hence eq. (1) becomes 32. (a) The equation for the given v-x graph is
t m t v0
ve m g em 1 v=– x v0 ... (i)
x0
Differentiating the above equation w.r.t x, we get

dx m t dv v0
or g 1 e m
dx x0
dt
Multiplying the above equation both sides by v, we
Integrating it, we have get

m m t dv v0 v0 v0
x g t e m D .......... (2) v v x v0 From (i)
dx x0 x0 x0

v02 v02 dv
m2 a= x .... (ii) a v
Initially, when t = 0, x = 0, D 2
g x02 x0 dx

Hence eq. (2) becomes

m m m
t m2
x g t e 2
g

m2 m
t
= g 2
e 1 t
m
On comparing the equation (ii) with equation of a
straight line
u 2 sin 2 y = mx + c
30. (a) 20 u sin 20 ....... (1)
2g
v02
For total time of light we get m = 2 ve, i.e. tan = + ve, i.e., is acute.
x0
1 2
25 u sin t gt
2 v02
Also c = , i.e., the y-intercept is negative
5t2 – 20t – 25 = 0 x02
t2 – 4t – 5 = 0
The above conditions are satisfied in graph (a).
4 16 20
t 5 sec. 33. (b) v vx2 v 2y ( 12 sin 4t ) 2 ( 12 cos 4t ) 2
2
In 5 sec, horizontal displacement = 75m. v = 12 m/s
Now u cos × 5 = 75 u cos = 15 ........ (2) Speed is constant.
From (1) and (2) Distance = speed × time = 12 × 2 = 24m
56 IIT-JEE P HYSICS Challenger
34. (b) Assuming particle ‘2’ be at rest
1200
2 37. (d) aA 0.960 m/s 2 ,
Substituting in y = x tan –
gx 1.25 103
2u 2 cos
1200
aB 1.00 m/s 2
gs 2
g 1.20 103
h v .s ,
2 4v2 8h
1
which is a straight line passing through the origin with 1.00 102 (0.960)t 2 t 14.4s
2
g 1
slope . dB =(1.00) (14.4)2 104m
8h 2
ds Car B will be 4 m ahead of Car A.
35. (b) Resistance = kv k 38. (b) Area under a-t curve gives change in velocity. Between
dt
0 to 2 second speed will increase and between 2 to 4
Equations of motion are second speed will decreases.
39. (c) Let the points A, B, C and D be separated by 1 km.
d 2x dx
k ............. (1) Then
dt 2 dt
1 1
tAB = hr , t BC hr
d2y dy 60 40
k g ............. (2)
dt 2 dt
1 1 1
Integrating (1) and (2) and using the initial conditions, <vAC> = = 48 km/hr tCD hr
1 1 48
we get
60 40
dx kt
u cos .e ............. (3) 1 1 1
dt Now <vAD> = = 48 km/hr
1 1 1
dy kt 60 40 48
and k g (ku sin g ).e
dt 40. (b) Initial relative velocity, vAB = v – 0 = v
dy 1 kt
Acceleration aAB = 0 – a = – a
i.e., [(ku sin g ).e g ] ......... (4) For max. separation vAB = 0
dt k
kt v2
dy dy / dt [(ku sin g ).e g] 0 = v2 – 2as s=
kt .... (5) 2a
dx dx / dt ku cos .e
Direction of projection was with the horizontal, when
the direction of motion again makes the angle with
the horizontal, it really makes the angle ( – ) with the
horizontal in the sense of the direction of projection. If u
this happens after the time t, we have from (5), P
41. (b)
90°–
( ku sin g ).e kt
g gsin g v
tan ( ) kt O
ku cos .e
Horizontal components of velocity at O and P are equal.
kt
(ku sin g ) ge v cos (90° – ) = u cos
i.e., tan or v sin = u cos
ku cos
or v = u cot
i.e., ku sin kt
ku sin g g .e
vT2 u 2 cot 2
At P, ac ; R
2ku R g sin
or e kt 1 sin
g 42. (d) This is a straight forward problem in calculating the
total distance. The only trick is to make sure that in the
1 2ku
or t log 1 sin cross-over lap you use the hypotenuse. The total
k g distance must be precisely 400 m, so we must have
36. (c) The distance travelled by each car is equal to the area 400 = 112 + (R + 0.05) + (R + 5.05) + (1122 + 52)1/2.
under the graph. These areas are equal at 10.0 s. Solving for R gives 25.44 m.
MOTION IN ONE AND TWO DIMENSIONS 57

a 47. (b) vx = const. No acceleration.


43. (a) sn = (2n – 1); 2s
2 48. (a) T0
a a v
sn + 1 = [2(n + 1) – 1] = (2n + 1)
2 2 s s 2v 2s v2
Tw s
sn 2n 1 v vw v vw v2 v w2 v v 2 v w2
sn 1 2n 1
1
b 2b Tw T0
44. (a) vy = 0 at t = , T
c
, 1 (v w / v ) 2
c
2ab 40km
Range = aT = 49. (c)
c
45. (b) P be the point where the tangent is parallel to the vL = 8 km/h, s = v0 × t
inclined plane. If PN = z be perpendicular from P on the
inclined plane and PM the vertical altitude of P then 40
t 5h ,
evidently for all points on the path, P is the point where 8
z is the greatest and consequently PM is greatest. Total distance flown by the bird = 10 × 5 = 50 km.

P
50. (a) vB v BA vA
usin
u
u
z
A

M N vA = v
ucos
Now for the point P, velocity perpendicular to the
inclined plane is zero. Now the velocity and acceleration
perp. to the plane at O is u sin ( – ) and g cos and 1 v
u cos v cos
this velocity becomes zero at P. u
0 = u2 sin2 ( – ) – 2g cos .z
srel 2H
u 2 sin 2 ( ) 51. (a) t H2 L2
z vrel g
2 g cos
52. (b) y
For max. range or
4 2 4 2
2
u D
Hence, z sin 2
2 g cos 4 2 v BA 40m/s
90°
2
u
= 1 cos B
4 g cos 2 x
A 30m/s
u2
= (1 sin ) or PM = z sec
4 g cos

u2 u2 1
= (1 sin ) (max. range)
4 g cos 2 4 g (1 sin ) 4 40
vBA vB vA 40 ˆj 30iˆ ; tan
Maximum range = 4 × PM 30
46. (d) By definition of relative velocity | v BA | 50m/s
v1 v0 v2
BD AB cos
v0 v2 ( v1 ) 0 Required time =
vBA vBA
v0, v1 and v2 will be sides of a triangle and we know
that the sum of any two sides is greater than third side 104 m 3
of the triangle. 120sec.
50ms 1 5
58 IIT-JEE P HYSICS Challenger
If the swimmer walks this distance with uniform velocity
400 m/s w
53. (a) T 37° 3 w', then time taken = .
vw
500 m/s Since both the swimmers reach B simultaneously, we
100m/s 3 have
w
v cos v vw
1 w
1
cos w
We have w = 2 km/hr and v = 2.5 km/hr.
x
vy v2 w2 2.52 2 2 1.5
cos
1 2 v v 2.5 2.5
1500 100t 10t ; t = 10 sec.
2 1.5 w
400 4000 2.5 w 2
x t m
3 3 Velocity of walking of the second swimmer
54. (a) In a, b, c and d time taken are respectively w' = 3 km/hr.
2d 2d d d 2du 2du v0 v0
, , , 57. (a) a
v 2
v u 2 u v uu v v u 2
v 2 2 2 t
On substituting
[u = stream speed]
v'0 = 2 m/s, v''0 = 12 m/s and t = 10 sec.
55. (a) The distance travelled by the body A is h1 given by
we get, a = 1 m/s2.
gt 2 gt 2 58. (a) Let the unit vectors towards east and north be iˆ, ˆj and
v1t and that travelled by the body B is h2 =
2 2
The distance between the bodies = x = h – (h1 + h2). the true velocity of the wind be aiˆ bjˆ and the man’ss
Since h1 + h2 = v1t, the relation sought is x = h – v1t
56. (b) The swimmer who wants to cross the river along the velocity be uiˆ . Now the relative velocities of the wind
line AB should swim in such a direction such that his in the 3 cases are :
velocity along the direction of stream is zero. (aiˆ bjˆ) uiˆ, (aiˆ bjˆ) 2uiˆ, (aiˆ bjˆ) 3uiˆ
His velocity along the direction AB = v cos From the given conditions they are of the form

Time taken by the swimmer to reach B where aiˆ bjˆ uiˆ ˆj ........... (1)
v cos
is the width of river. 1 ˆ 1 ˆ
aiˆ bjˆ 2uiˆ i j ........... (2)
Now the second swimmer crosses the river in a direction 2 2
perpendicular to the stream.
aiˆ bjˆ 3uiˆ cos iˆ sin ˆj ........... (3)
Time taken to reach the opposite bank = .
v
On the RHS of (1), the iˆ component is 0. So also on the
But during this time the swimmer is carried away by LHS. So, a – u = 0 or a = u
stream with velocity w and he reaches the point C.
On the RHS of (2), the iˆ and ˆj component are equal.
w
Distance CB = velocity × time = . So also on the LHS. Therefore a – 2u = b or b = – u
v
From (3), we get
C B – cos = a – 3u, – sin = b
b u 1
w tan
a 3u u 3u 2
59. (b) In the first and the second cases let the retardations be
h a m/sec2, a' m/sec2.
vy= v cos Now, for the first phase of the first case, we have
v u = 30, v = 10 m/s, s = 240 m
vx= v sin So, by v2 = u2 – 2as, we get a from
A 100 = 900 – 2a × 240 as a = 5/3.
MOTION IN ONE AND TWO DIMENSIONS 59

Let s1 be distance travelled by the train in the second Integrating,


phase. In that phase the initial velocity is 10 m/s and
10
the final velocity is 0 and therefore 0 = 10 2 – 2as1 or dv · t dt 10 dt
11
100 100 3
s1 = 30m.
2a 2 5 10 t 2
In the second case, the retardation is v . 10t
11 2
12½ 9a 9 5 15 For v to be maximum,
a a a
100 8 8 3 8
If s2 is the distance in this case dv
0
dt
15
0 302 2 s2 or s2 = 240 10 t 110 0
8
t = 11s
60. (a) Let v be the maximum velocity attained and t the total
time of journey. t' is the duration of acceleration and 10 11 11
retardation. Then v = 0 + at'. vmax . 110 = (–55 + 110) m/s = 55 m/s
11 2
1 2 1 2 62. (a) v = 4t – 3t2
L at v (t 2t ) at
2 2
s t
ds (4t 3t 2 ) dt
v
2
2v v 2
2v 2
v2 4t 3t 2 ; ds
= a v t = vt vt dt 0 0
a a a a a

L v 4t 2 3t 3
t s ; s = 2t2 – t3
v a 2 3

dt L 1 s (2) s(0)
2
vavg [s(2) = 8 – 8 = 0]
dv v a 2 0

dt 0 0
When t is minimum, 0 vavg 0
dv 2
63. (a) v2 = u2 + 2as
vmax La 400 > 2a (100) ; a < 2
61. (b) Change in velocity = area under the graph 64. (a) Let v = velocity of A at S ° E
1 The velocity triangles for the two cases are,
Area under the graph = 10 11 55 m / s
2

30°
30°

Since, initial velocity is zero, final velocity is 55 m/s. vC


vA,C
Alternatively :
From the given graph, a – t equation is
30°
10
a t (10)
11 30°
v – 30°
dv 10
or t (10)
dt 11

10
or dv t dt (10) dt
11
60 IIT-JEE P HYSICS Challenger
The distance moved by the first body in time (t + T)
1
= u (t + T) – g (t + T)2.
2
vB And the distance moved by the second body in time T
1 2
12 km/hr = vT – gT = h (supposed above). ........... (1)
2
The two bodies meet each other,
They are equidistant from the point of projection.
vA,B
v 1 1
Hence, u (t + T) – g (t + T)2 = vT – gT2
2 2
1
or u (t + T) – g (t2 + 2tT) = vT
2
By sine rule for triangles, or gt2 + 2t (gT – u) + 2 (v – u) T = 0 ........... (2)

v 8 1 2
Also from (1) we get, h = vT – gT
sin 60 sin ( 30 ) ........... (1) 2
dh
v gT
v 12 dT
........... (2) h increases as T increases
sin 90 sin
T is minimum when h is minimum i.e., when
Eq. (1) and Eq. (2) give
dh
= 0, i.e. when v – gT = 0 or T = v/g.
sin ( 30 ) 8 3 2 1 dT
sin 60
sin 12 2 3 3 Substituting this value of T in (2), we get
gt2 + 2t (v – u) + 2 (v – u) (v/g) = 0
sin cos 30 cos sin 30 3 1 1 or g2t2 – 2gt (u – v) + 2v (u – v) = 0
cot
sin 2 2 3 2 g (u v ) 4 g 2 (u v ) 2 8vg 2 (u v )
or t
1 2g 2
3 3 cot 2 cot = 60°
3 u v u 2 v2
or t
g
12 2 neglecting the negative sign which gives negative
From (2) v 12 8 3 km / hr.
sin 60 3 value of t.
67. (a) Let CAB = .
Speed of A = 8 3 km / hr , Direction = S 60° E b
Then cos =
65. (a) Before hitting the ground, the velocity v is given by a 2 b2
v2 = 2 gd (quadratic equation and hence parabolic path)
To reach B, the resultant velocity must be along AB.
Downwards direction means negative velocity. After Take the river bank as X-axis and line perpendicular to
collision, the direction become positive and velocity
decreases. it as Y-axis. Then vriver v0iˆ
Let the boat start at an angle with river bank.
2 d
Further, v ' 2g gd ; Then uboat u cos iˆ u sin ˆj .
2
v (resultant velocity of boat)
v v
2 or v = v ' 2 v' = (v0 u cos ) iˆ u sin ˆj
v' 2
u sin
As the direction is reversed and speed is decreased tan
graph (a) represents these conditions correctly. 2 v0 u cos
66. (b) Let the two bodies meet each other at a height h after u sin
time T of the projection of second body. Then before cot
v0 u cos
meeting, the first body was in motion for time (t + T)
whereas the second body was in motion for time T. v0 cot + u cos cot = u sin
MOTION IN ONE AND TWO DIMENSIONS 61

Differentiating, This is the required equation of the trajectory.


C a B 3
Substituting t in eq. (6), we get
2
b
v0
3 3
x u0 and y a sin a
A 2 2
du du 3
0 cot cos u sin sin u cos distance of the particle from the origin at t is
d d 2
du 2
When u is minimum, 0 3
d s x 2
y 2 = u0 . ( a )2
2
– u cot sin = u cos or tan = – tan
tan ( – ) = tan
2 2
+ = 9 u0
s 2
a2
If u0 be the minimum value of u, then 4
u sin ( ) 70. (a) To solve the problem you should have knowledge of
cot =
v0 u cos ( ) conic section-parabola.
v0 cos –u cos2 = u sin2 The enveloping parabola is x2 = – 4h (y – h) where

v0b u2
u = v0 cos = h
a 2 b2 2g
68. (c) v = 0 at t = 2 sec. Its focus is the point of projection O, vertex is A where
OA = h and directrix is XM.
1
s 10 (2) (5)(2)2 10m Let this enveloping parabola cut the sphere at P so
2 that OP is the max. range. Draw PM perp. to directrix
1
st 3 (5)(1) 2 2.5m and PN perpendicular to CO, C the centre of the sphere.
2 Since O is the focus, PO = PM = XN = XO + ON = 2h +
69. (a) The velocity of the particle is PO cos .
u u0iˆ ( a cos t ) ˆj ........... (1)
PO
Writing u in terms of Cartesian components in XY Also, PO = 2r cos Hence, PO 1 2h
2r
plane as u uxiˆ u y ˆj
R
or R 1 2h where R = PO
Comparing coefficients of iˆ and ˆj , the components of 2r
velocity along X and Y axes are
X M
dx
ux u0 ........ (2)
dt
O A
dy
and u y a cos t ........ (3)
dt
Integrating (2) and (3) with respect to t, we get N P
x = u0t + C1 ........ (4)
a sin t
and y C2 a sin t C2 ........ (5) C
where C1 and C2 are constants of integration. or R2 – 2r + R + 4rh = 0
At t = 0, the particle is at origin x = 0, y = 0
So C1 = 0 and C2 = 0 R r 4rh r2
Eq. (4) and (5) take the form The plus sign corresponds to the second point where
x = u0t and y = a sin t ........ (6) the parabola cuts the sphere again hence the minus
x x sign is to be taken
Substituting t , we get y a sin .
u0 u0 R r r2 4 rh
62 IIT-JEE P HYSICS Challenger
From this result is follows that the max. value of h 73. (d) Take downward as positive. Then a = g = 32 ft/s2 and
possible is r/4 when the two values of R become same v0 = – 40 ft/s.
and equal to r. In this case the parabola touches the 1 2 1
sphere and the velocity of the projection is s at = – 40 (20) + (32) (20)2
v0 t
2 2
r 1 = – 800 + 6400 = 5600 ft.
2 g.h 2 g.gr . The particle will then clear
4 2 The balloon was 5600 ft above the ground.
74. (b) Let the required time be t. Then
the sphere and the least value of the velocity of
vx = u cos 60° = 5 m/s, vy = u sin 60° – 10t
1 v2 = (u sin 60° – 10t)2 + (u cos 60°)2
projection must be gr .
2 u
71. (a) BQ is the vertical target, OB = d. P is the position of the But v =
2
shot at any instant, OP is the line of sight cutting BQ at
Q. We have to find the velocity of Q. Y
Let BQ = z and QOB =
vy vy
PM
Then z = d tan = d. u = 10 m/s
OM
vx
1 2
u sin t gt 60°
y 2
= d. d.
x u cos .t ux X
Q
2
u2 3 u2 10 3 3
u 10t 10t t
4 2 4 2 2
P
75. (d) At the highest point, the vertical component of the
z velocity is zero but acceleration acts vertically
downwards.

O d M B v = ucos
u
1 a=g
u sin gt
2 1 g
= d. = d tan .t
u cos 2 u cos
76. (d) It is clearly visible from all graphs that as x-increases
1 gd velocity change sign. Since this is not possible, no
z = constant since u cos = const.
2 u cos graph represents the possible motion.
72. (b) From the slopes of the graph, we can obtain the ratio of 77. (b) The length of side CA at any time t is = 20 – 5t
accelerations of the two cyclists : The length of side CB at any time t is = 10 – 5t
At the instant A, B and C are collinear
B
v 0 20 m
a2 4 3 A
4
a1 v 0 5 m/s
4 C (t = 0)
5 m/s

The distances travelled by the two cyclists must be (20 – 5t) + (10 – 5t) = 20.
equal Solving we get t = 1.
Therefore, length of CA at t = 1 is 20 – 5 = 15 m.
a1t 2 a2 (t 3) 2 78. (a) We choose the launch point at the origin (figure). The
2 2 equations of motion of the ball are
x = v0 cos 30°t = (13.0 m/s) t
2
a2 t t 1 2
2 t 6 min . y = v0 sin 30° – gt = (7.5 m/s) t – (4.9 m/s2) t2
a1 t 3 t 3 2
MOTION IN ONE AND TWO DIMENSIONS 63

82. (c) In 2 sec, horizontal distance travelled by bomb


y = 20 × 2 = 40m
v0 In 2 sec, vertical distance travelled by bomb
1
× 10 × 22 = 20m =
30° 2
x In 2 sec, horizontal distance travelled by Hunter
O 20° = 10 × 2 = 20m
L Time remaining for bomb to hit ground

A 2 80
2 2 sec =
10
The equation of the straight line incline is Let vx and vy be the velocity components of bullet
along horizontal and vertical direction.
y = – x tan 20° = – 0.364x.
Thus we have,
We want the time at which the (x, y) values for the ball
satisfy this equation. We thus substitute the time 2v y 20
2 vy 10m/s and 2
expression for y and x. g vx 20
(7.5 m/s) t – (4.9 (m/s2) t2 = – 0.364 [(13.0 m/s) t] vx = 30 m/s
or 12.2t = 4.9t2.
The solutions are t = 0 (corresponding to x = y = 0) and Thus velocity of firing is v v x2 v 2y 10 10 m/s
t = 2.49s. 83. (a) As horizontal component of velocity remains constant.
79. (a) Assume the wall to be absent. Let C and E be two Hence, magnitude of change in velocity = magnitude
points lying on trajectory at same horizontal level as of change in vertical component of velocity
point of projection. = | vy – uy | = gt = 10 × 0.5 = 5 m/s

u x R 3R 3R
B 84. (b) y x tan 1 tan 1
R 4 4 4R
C E
A 1 4
1 3 tan tan 53
4 3
///////////////////////////////////////////////
D 85. (a)

AE B
Then the wall must be placed a distance d = from
2 v
A. u
u
The maximum height of ball above ground at B is d u

102 v
H 15 20m
2 g
Time taken to fall from B to C is given by A C
v = velocity of man w.r.t. river
1 2
5 gt1 or t1 = 1 sec. u = velocity of river
2
t d d
A B = 10 d 10v ........ (1)
2 20 v v
Time taken to fall from B to D is t2 2 sec.
10 t d
Time taken by projectile to move from A to E = 4 sec. B C=
v cos
Hence 2d = u cos × 4 = 40 or d = 20m. d
80. (b) From t = 0 sec to t = t0 second, the speed decreases 15 = d 15v cos ....... (2)
v cos
and after t = t0 second the speed increases. Hence
particle first decelerates and then accelerates. 2 3
Eq. (1) and (2) cos sec
81. (b) Horizontal velocity vx = ux = 18 m/s 3 2
u u
vy tan sec 2 1
Now, tan 45 v v
vx
u 9 5 v 2
vy = vx = 18 m/s 1
v 4 2 u 5
64 IIT-JEE P HYSICS Challenger
86. (d) vm = (vx)ball = v cos 60° = v/2 Since horizontal displacement in time t is zero
87. (c) For a–t curve, area under the graph gives change in 2v
velocity at t = 10 sec, v = 40 m/s t .......... (2)
f
For 10 – 30 sec, v = – 80, v30sec – 40 = –80
Speed at 30 sec = – 40 m/s From eq. (1) and (2)
88. (b) If component of velocity normal to incline are equal, 2 gv 2
time of flight is same. Also if horizontal components h 2
f
are equal, range on inclined plane will be equal for both.
89. (a) In the figures S Station, F Factory and P is the 92. (a) The linear relationship between v and x is
place where he meets the car. v = – mx + c where m and c are positive constants.
Usual day : dv
Acceleration, a v m ( mx c )
T t=0 dx
S F = Factory a = m2x – mc
= station
Hence the graph relating a to x is
2T a
Car starts from F at t = 0, reaches station at T and again
reaches at the factory at time 2T. x
This day :

t=0
93. (b)
T – 60 P
C
S y
T – 60 + t
F
vy
C (2T – 10) v

Person reaches S at T– 60. Car starts at t = 0 from F. 53°


vx
Person walks for time t and reaches point P at time
T – 60 + t. At this time car also reaches P. Car comes
back at F at time (2T – 10). That means car takes x
time T – 5 from F to P.
That means car reach at P at time T – 5.
vy
Now T – 5 = T – 60 + t t = 55 min tan 53° =
90. (c) With respect to lift initial speed = v0 vx
a = – 2g, displacement = 0
T
1 2 1 u sin g 2
s = ut + at 0 = v0T' + × 2g × T'2 2
2 2 tan 53° =
u cos
v0 1 2v0 1
T T u sin
g 2 g 2 u sin 10 2
4 g
91. (a)
3 u cos
f
g
4 u sin u sin 20
h 4u cos 60
3 u cos
60
u cos 15 u cos = 15
//////////////////// 4
Time taken to reach the ground is given by At maximum height vertical component of velocity is
zero so velocity at maximum height = u cos = 15m/sec
1 2
h gt ..........(1) 94. (b) Initial velocity v1 must be greater than v0 if it has to
2 reach same height and finally while coming back it will
approach a terminal velocity.
MOTION IN ONE AND TWO DIMENSIONS 65

1. (a) HA = HC > HB
60 | vB |
Obviously A just reaches its maximum height and C
has crossed its maximum height which is equal to A as sin 37 sin 53
u and are same. But B is unable to reach its maximum 4
height. 60
5
2. (c) | vB | = = 80 m/s.
3/ 5
8. (c) Velocity triangles
Q
vBA 8°
45°
45°
|vA|
37° 53°
O R
Time of flight of A is 4 seconds which is same as the vB 60
time of flight if wall was not there. sin 45 sin 45
Time taken by B to reach the inclined roof is 1 sec. vB = 60 m/s.
TOR = 4 9. (d)
TQR = 1 vBA 8°
45°
TOQ = TOR – TQR = 3 sec. 45° 60°
90°
2u sin 37° 53°
3. (c) From above T 4s
g
u sin 20m / s | vBA | 60
vertical component is 20 m/s sin 90 sin 45
for maximum height | vBA | = 60 2,
v2 = u2 + 2as 02 = 202 – 2 × 10 × s Particle B has to travel 100 cos 8° (w.r.t. A)
s = 20m
4. (a) 5. (c) 6. (b) 100 cos8 5
t= cos8 second
60 2 3 2
100
T 5 sec.
20 sx 1600 1600
10. (b) t 5sec.
ux 400 cos 4
v 2gH 2 10 5 = 10 m/sec. 400
5
2H 2 5 1
T–t = 1 sec. 11. (a) H = (sy)bullet = v y t ayt2
g 10 2
t = 4 sec.
3 1
= 400 5 10 52
1 2 25 5 2
at at (T – t ) 100 a
2 3 = 1200 – 125 = 1075m
7. (a) Velocity triangle 12. (b) Let t0 be the waiting time.
|vBA| 5(t0+t)
37° 53°
|vA| = 60 800
|vB| = ? 90°
37° 53°
66 IIT-JEE P HYSICS Challenger

1600 20. (b) If 1 , 2 be the angles that directions of motion make


t 5 sec
400cos at time t,
800 + 5 (t0 + 5) u sin gt
tan 1 and
3 1 u cos
= 400 × 5 10 52 = 1200 – 125 = 1075m.
5 2 v sin gt
800 + (t0 + 5) × 5 = 1075 tan 2
v cos
(t0 + 5) 5 = 275 ;
t0 = 55 – 5 = 50 sec Hence, when 1 = 2 we have
13. (b) 14. (b) 15. (a)
u sin gt v sin gt
d 11d u cos v cos
dx (d d d d d ) .......... (1)
2 2
uv sin ( )
t
u 2 sin 2 g (v cos u cos )
Total d x 12d ,d 20m
g 21. (a) The paths of the two particles are

Now,
11d 11 20
110m u cos .t 1 gx 2
y x tan
2 2 2 u 2 cos 2
110 11
t sec. 1 gx 2
30 30 and y x tan
2 u 2 cos 2
So H of wall
If R (a, b) be the common point then
1 2 11 40 6 110
= Hmax = u sin t gt
1 gx 2
2 3 6 b = a tan – and
= 79.44 m 2 u 2 cos 2

3 1 ga 2
m (ucos ( u cos )) 2 50 60N-s b = a tan –
5 2 v 2 cos 2
16. (a) 17. (b) 18. (b)
At t1 car A overtaking car B 1 ga 2 ga 2
B has more slope than A Hence a tan = a tan
2 u 2 cos 2 2v cos 2
2
Same slope.
19. (a) Let O be the point of projection, P (x1, y1) and Q (x2, y2) 1 1 1
be the two particles at time t. i.e., tan tan ga 2 ..... (1)
2 u cos 2 v 2 cos 2
1 2
Then, x1 = u cos t, y1 = u sin t– gt If t1, t2 be their times to pass through the common
2 point then a = u cos .t1; also a = v cos t2
1 2 1 1
x2 = v cos t, y2 = v sin t– gt t t1 t2 a ......... (2)
2 u cos v cos
Y Hence dividing eq. (1) by eq. (2), we get

y 2) tan tan 1 1 1
x 2, g.
Q( t 2 u cos v cos
)
a, b
v u
P (x1, y1) R( g (u cos v cos )
=
2uv cos cos
X 2uv sin ( )
O Hence, t
g (u cos v cos )
y1 y2 u sin v sin 22. (b) 23. (d)
Slope of PQ =
x1 x2 u cos v cos A1 = 4 × 1.5 = 6,
independent of t, hence same for all times i.e., PQ A2 = 2 × t' = 2t'
remains parallel to itself. t' = 3 ; t = 3 + 4 = 7s
MOTION IN ONE AND TWO DIMENSIONS 67

24. (b) Distance travelled by the car in 10 seconds is equal to At this time
displacement in 10 seconds and it is same as area under 1
the v-t curve. y 3 (v0 sin ) t (9.8) t 2
2
1 1 2
Distance = 2 10 10 5 3 10 45 1 45
2 2
= (v0 sin ) (9.8)
= 10 + 50 + 15 = 75m. v0 cos 15 2 v0 cos 15
Inserting the numerical values of sin and cos we
10ms 1
25. (c) Retardation = 10ms 2 obtain the following quadratic equation for
1 v0 :
Braking force = ma v02 (4.55) – v0 (60.3) – 3532 = 0.
= 1000 × 10 = 10000 N Solving, v0 = 35.3 m/s
26. (d) From v-t graph we can analyze in t = 0 to t = 2 sec slope dv y
is positive and constant. Hence acceleration is positive 30. (b) ay dv y a y dt
dt
10
and constant and it is = 5 ms–2. vy tb tb
2
dv y a y dt ( g g t 4 ) dt
Between t = 2 to t = 7, slope is zero, so the acceleration 0 0 0
is zero. Between t = 7 to t = 10, slope is negative and
constant. Hence acceleration is negative and constant 1 5
vy 0 [ 1] gt t [ vy (t = 0) = 0]
10 5
and its value is – m / s2 .
3 1 5
vy ( 1) gt t .......... (1)
27. (d) In this case, the projectile hits the back of the truck at 5
the moment of overtaking it, which is the moment at vy t
which the distance of the back of the truck, x1 = 45 +
31. (d) dy a dt
15t, equals the horizontal distance of the projectile,
v y (t tb ) tb
x = (v0 cos ) t = 32.22t.
t
45
Thus t = 2.614s. vy – vy (t = tb) = ( g ) dt g (t tb )
32.22 15
tb
28. (c) At t = 2.614s, when the projectile overtakes the
back of the truck, faster its height is, noting
vy = vy (t = tb) – g (t – tb)
v0 sin = 13.67 m/s,
1
1 = ( – 1) gtb – t 5 –g (t – tb)
y = (13.67) (2.614) – (9.8) (2.614)2 = 2.25m, i.e. 25cm. 5 b
2
[Using (1)]
above the top of the truck. Since the projectile travels
horizontally than does the truck, it is clear that thereafter 4
gtb gt
the projectile remains ahead of the back of the truck, 5
and so never hits the back.
The projectile will reach (for the second time) a height 32. (b) For maximum height, vy = 0 (at t = tf > tb)
of 2m in a total time t2 given by 4
0 gtb gt f
1 5
2 = (13.67) t2 – (9.8) t22, or t2 = 2.635s, that is
2 4
tf tb
2.635 – 2.614 = 0.021s after overtaking the back of the 5
truck. Thus the projectile hits the top of the truck of a
dv
distance of (32.22 – 15) (0.021) = 0.36m = 36cm. in front 33. (a) From v-t graph tan 45 1
of the rear edge. dt
29. (a) The time taken to overtake the back of the truck is 34. (b) From v-t graph velocity at t = 2 sec. , v = – 1m/sec
given by acceleration at t = 2 sec is a = 1 m/s2
so do not product of v and a = – 1m2/s3
45 + 15t = (v0 cos ) t
35. (c) From v-t graph
45 t 5
or t 1 1 1
v0 cos 15 v dt ( 1) 2 2 2 1.5 metre
t 2
2 2 2
68 IIT-JEE P HYSICS Challenger
v 2 The position vector at any time t is
vdv
36. (d) 9x vdv 9 x dx 1 2
dx 6 0
r r0 ut at
2
v2 62 1 ˆ
18 or v = 0 = ( 12iˆ 16 ˆj ) t (6i 8 ˆj ) t 2 ...... (2)
2 2
37. (b) Since v = 0 at x = 2 8
Hence maximum distance of particle from origin will be Solving for r .v 0 we get t sec.
x = 2m. 3 5 5
38. (d) Quadratic relation hence parabola. Putting t in eq. (1) we get
39. (d) 40. (a) 41. (a) | v | 12 5 m / s
The velocity vector at time t is
The x-coordinate as function of time is
v u at 12iˆ 16 ˆj (6iˆ 8 ˆj ) t ..... (1)
1 2
Solving for v x x0 uxt axt 12t 3t 2
0 we get t = 2 sec. 2
Solving for x = 0 we get t = 5 sec.

1. (a) When a body is thrown upwards vertically, at the Trajectories may intersect at some point but not at the
highest point its velocity becomes zero but gravitational same instant
force continuous to act on it so it has acceleration in 8. (d) Linear momentum during parabolic path changes
downward direction even at the highest point. So continuously.
statement-1 is true. 9. (a) Acceleration of all three projectiles = g
A body is numerically at rest but it reverses its direction Relative acceleration = 0
due to acceleration present in it. Statement-2 is true &
it supports statement-1. L
10. (a) t ,
v cos
decrease in velocity
2. (a) Retardation angle between boat velocity relative to river and
time
normal.
It acts opposite to velocity.
11. (c) In uniform circular motion speed is constant but
3. (a) The distance covered in the last second, final velocity acceleration exists due to change in direction of
becomes zero. So if we drop an object with zero velocity velocity. So that statement-2 is wrong. If acceleration
it will cover the same distance in one second while is 0 then velocity is constant. Hence the magnitude of
going downwards. the velocity will be constant. Hence speed will be
Now distance travelled in the later case constant.
1 2 1 12. (d) Statement-1 is false because angles of projection and
S ut gt 0 10 1
2 2 (90° – give same range but time of flight will be
S = 5m different. Statement-2 is true because in horizontal
4. (c) Statement – 1 is true and Statement – 2 is false because direction acceleration is zero.
it is also possible that when position vector and velocity 13. (a) Statement-1 is true on the basis of our personal
vector are perpendicular, the distance from the point of experience, statement-2 is true and explain statement-1
projection not a maximum. properly.
5. (a) Since W = K implies that the final speed will be same. 14. (d) Average velocity of particle moving with constant
6. (a) The time of flight depends only on the vertical acceleration in time interval (t1 + t2) is equal to the
component of velocity which remains unchanged in average velocity in time interval (t1 + t) to (t2 + t).
collision with a vertical wall. 15. (d) For a projectile, acceleration is constant and hence does
not depend on velocity.
7. (d)
16. (a) For a stone projected vertically upwards, the speed v
u dv
u is least at highest point even though is constant.
dt
Hence statement-1 is true.
MOTION IN ONE AND TWO DIMENSIONS 69

17. (a) Both statements are true and statement-2 is a correct 20. (c) The motion of particle starting from rest is always along
explanation for statement-1. a straight line if and only if direction of acceleration is
18. (b) Statement-1 is True, Statement-2 is True; Statement-2 fixed (constant).
is NOT a correct explanation for Statement-1. 21. (d) Statement-1 is wrong because on earth, the acceleration
19. (a) In statement-2, if speed of both projectiles are same, is g which is downwards. Statement-2 is true.
horizontal ranges will be same. Hence statement-2 is 22. (a) Both statements are true and statement-2 is a correct
correct explanation of statement-1. explanation for statement-1.
23. (b) Both statements are correct and independent.

1. (a, b) vx v / 2 4. (a, b) v0 = 60 m/s at 36.87°


vy = vx cot 30° vox = 48m/s and voy = 36 m/s

v v 48iˆ 36 ˆj 0kˆ and a 3.6iˆ 6 ˆj 0kˆ


vy 3
2
vA = v iˆ ˆj kˆ
v a 48 36 0 ( 158.4) kˆ
y 3.6 6 0
x
A
| v a | 158.4

v3 603
R
|v a| 158.4
B 30°
D v R = 1363.64m at the start.

v3 v2 v2
[ R is basically R ; ac ]
|v a| ac R
C
Now at the top, vx = vox = 48 m/s, vy = 0
2. (a, d) At x = 0, vy = 0
vx = v and ax = 0 [ particle moves uniformly) v 48iˆ 0 ˆj 0kˆ v = 48 m/s
vy = 2k x vx
iˆ ˆj kˆ
ay 2k v x2 xa x
v a 48 0 0 ( 288) kˆ
ay = 2k v2 3.6 6 0
a 2kv 2 j
v3 483
d |v a| 288 R 384 m
3. (a, c) t= |v a| 288
v cos
R = 384m at the top
5. (b, c) v = 0 – 9 + t2
v a = 0 – 0 + 2t
u At t = 3, v = 0, a 0
6. (a, c) Time taken to reach x-y plane is given by
x (u v sin ) t 1 2
gt 5 t 1 sec
ud 2
= sec d tan
v Initial horizontal velocity of bob is 3jˆ
For x to minimum
y = 3 × 1 = 3m
dx v x = 2m, y = 3m
0 sin
d u
70 IIT-JEE P HYSICS Challenger
9. (a, b, d)
7. (b, c, d) gh sin
gh

x
gh cos x u cos t u cos (t T ) ;
cos cos
1 2 1
2 gh sin c c y u sin t gt u sin (t T ) g (t T )2
T 2 2
g gh cos e gh cos
1 1
10. (b, d) tan 28.56
2( gh)sin cos 1 2
1 Let v0 = initial velocity of auto
gc e
sx = vox × t
c 40 = v0 cos 28.56° × t
e
h sin 2 c 45.54
t ............. (1)
8. (a, b) Equation of motion is v0

dv dv 1 2 1 2
v3 i.e. dt Also sy = voy t gt = v0 sin 28.56 t gt
dt 3 2 2
v
2
45.54 1 45.54
1 sy = v0sin 28.5° × 32.2
Integrating it, we have 2 t A ....... (1) v0 2 v0
v2
Initially, when t = 0, v = V = – 10 (given)
Solving this, v0 = 32.76 ft/sec.
1 vox = 32.76 cos 28.56° = 29.3 ft/sec
A
V2 voy = 32.76 sin 28.56° = 15.66 ft/sec
Also
Hence, equation (1) becomes
v 2y v02 y 2a y s y = (15.66)2 – 2 (32.2) (–10)
1 1 1 2 V 2t
2 t
v2 V2 V2 v
1 y
vy = 29.3 ft/s, vx = 29.3 ft/s ; tan = 45°
vx
dx V
i.e. v 11. (a, b, c, d)
dt (1 2 V 2 t ) (a) – 20 = 15t – 5t2
t = – 1, t = 4
This proves the second option. (b) 0 = 15t – 5t2 ; 0 = 5t (3 – t) = 0 ;
Integrating it, we have t=3

x V (1 2 V 2t ) 1/ 2
dt D (c) v 2y (15)2 2 10 20

1 vy (625)1/ 2 25 ; vx 15 3;
= (1 2 V 2t ) D
V vy 25 5 3
tan
Initially when t = 0, x = 0 vx 15 3 3 3 3
1 5 3
D 5 3 tan 1
V tan ;
9 9

1 (d) x = vx × t ; D 15 3 4 60 3 m
So that, we have x [ (1 2 V 2t ) 1]
V
MOTION IN ONE AND TWO DIMENSIONS 71

12. (b, c) Let ABCD be the given trapezium whose sides are
ga 4 1
as follows : or 2 2
3
AD = 4a, AB = BC = CD = 2a. 2u cos 3 3
From B and C draw BB' and CC' perpendiculars
2 2 1
to AD. Then B'C' = 2a or u cos ga 3 .......... (4)
2
1
AB' = C'D = (2a) a u 2 cos 2
2 2 2
4a 2
From (2), we have (u sin ) 2
g
BB AB 2 AB 2 4a 2 a 2 a 3
[squaring both sides of (2)]
Let u and be the velocity and angle of projection
of the ball from the point A. Take A as origin and 4a 2 g 2 8a 2 g 2 8ag
the horizontal line AD and vertical line through A or u 2 sin 2
2 2
u cos ag 3 3
as coordinate axes. Then the coordinates of B are
(a, a 3) . 2u sin 4u 2 sin 2
Time of flight =
g g2
Y

4 8ag 32a
= 2
B C g 3 g 3
a a
Maximum height attained by the ball
u
u 2 sin 2 8 ga 4a
2a =
2a 2g 2g 3 3
13. (a, b) As the ship is moving horizontally with a velocity
X u in a direction opposite to that of the projection
A B' C' D
4a from the gun and the horizontal and vertical
components of velocity of projection relative to
Also the equation of the trajectory of the ball is the gun are v cos and v sin respectively, so
initially the actual horizontal components of
gx 2
y x tan velocity = v cos – u and the vertical component
2u 2 cos 2 of velocity = v sin .
Also we know that horizontal range of a particle
B = (a, a 3) lies on it, so we have = (2/g) (horizontal comp. of velocity)
× (initial vertical comp. of velocity)
ga 2 If R be the required range, then
a 3 a tan .......... (1) R = (2/g) (v cos – u) (v sin ) ......... (1)
2u 2 cos 2
Now if R is maximum, then dR/d = 0 and
Also horizontal range = AD d 2 R/d 2 = –ve.
2u 2 sin cos From (1),
or 4a
g dR
= (2g) [(v cos – u) (v cos ) +
2
d
u sin cos
or 2a .......... (2) (v sin ) (– v sin )]
g = (2/g) [v2 (cos2 – sin2 ) – uv cos ]
Substituting the values of u2 from (2) in (1) we get
dR
ga sin cos 1 If = 0, then
3 tan tan tan d
4ag cos 2 4 v2 (cos2 – sin2 ) – uv cos = 0
or v (2cos2 – 1) – u cos = 0,
4 4
or tan 3 .......... (3)
3 3 cos2 – sin2 = 2cos2 – 1
From (1), we get or 2v cos2 – u cos – v = 0

3
4 ga u u 2 8v 2
2 2 or cos =
3 2u cos 4v
72 IIT-JEE P HYSICS Challenger
The negative sign before the radical renders the putting x = a sin6
value of cos negative i.e. is obtuse which is
/2
against the problem. 6a 6 4 2
= sin 5 d . .
6 a 2
0 6 5 3
u u 2 8v 2
Hence, cos
4v 8 6
14. (a, b, c) Train reaches the farthest point at t = 40 sec. 15
Acceleration in the interval 50 < t < 80 (t in sec) is Also v, the velocity at O i.e. at x = 0 is given by
v 2 1 v2 = 6µa5/3 (a1/3), putting x = 0 in (1)
|a| m/s 2
t 30 15 or v a 6
Area under curve from t = 0 to 40 sec.
16. (c, d) (a) is wrong because a – b < | a b | a b , (b) is
1 wrong because in uniform circular motion, speed
= 30 × 3 + × 10 × 3 = 90 + 15 = 105m.
2 is constant but there exists acceleration, (c) is
correct because by definition distance
1
Area from t = 50 to 120 sec. = × 70 × 2 = 70m.
2 displacement ; (d) is correct because ac exists.
Distance from A = [105 – 70] = 35m.
15. (a, b) The equation of motion is 17. (a, d)
1/ 3
d2x a5 A
2 2 30°
dt x

dv a5 / 3 d2x dv v AB
or v 2
v
dx 2/3 dt dx
x 20 m/s

a5 / 3
or v dv = dx
x2 / 3
30° B
1 2
Integrating, v 3 a5 / 3 .x1/ 3 C , where C is
2
Condition for collision in mid air
constant.
Initially, x = a, velocity = 0, a AB 0 and v A B should be directed from A to B.
C = 3µa5/3 a1/3 v AB vA vB
v2 = 6µa5/3 (a1/3 – x1/3)
or (dx/dt)2 = 6µa5/3 (a1/3 – x1/3) .............. (1) = 20iˆ [ 20cos ( 30 ) iˆ 20sin ( 30 ) ˆj]
dx
or 6 a5 / 3 . a1/ 3 x1/ 3 )
dt = [20 20cos ( 30 )] iˆ 20sin ( 30 ) ˆj
1 dx (v AB ) y
or dt . 20sin ( 30 )
6 a 5/3
a 1/ 3
x 1/ 3 tan
(v AB ) x 20 20 cos ( 30 )
Required time from x = a to O, where x = 0,
1 cos ( 30 ) 3 sin ( 30 )
0
1 dx
= . 1 3 sin ( 30 ) cos ( 30 )
6 a5 / 3 x a a1/ 3 x1/ 3
1 3 1
a sin ( 30 ) cos ( 30 )
1 dx 2 2 2
.
6 a5 / 3 x 0 a1/ 3 x1/ 3 1
sin ( 30 30) ; sin sin ;
/2 5 2 6
1 6a sin cos d
= .
5/3 1/ 3
6 a 0 a cos 30
6
MOTION IN ONE AND TWO DIMENSIONS 73

10 + aT = V ........... (1)
v AB (20 20 cos 60 ) i 20 sin 60 ˆj
10 + 4aT = 20 ........... (2)
= 30iˆ 10 3 ˆj From (1) and (2),
V = 12.5 m/sec
| v AB | (30)2 (10 3) 2 10 9 3 20 3 ms 1
Distance between A and D = 675 m = Area under
the curve
relative distance
Time to collide = 1 1
relative speed T (10 + V) + 10V + (V + 20) T = 675
2 2
200 10 T = 20 sec.
Time to collide =
20 3 3
12.5 10 1
Hence, answers are (a) and (d). From eq. (1), a m / sec 2
20 8
dv
18. (b, c, d) a = 6 – 2t = 22. (a, b, c, d)
dt 25m/s
dv
For maximum velocity, 0
dt
6 – 2t = 0, t = 3 sec.
v t ///////////////////
dv Ground
6 2t ; dv (6 2t ) dt
dt 0 0 Height above ground
v = 6t – t2 = 18 – 9 = 9 m/s u2 25 25 125
After 4 sec, v = 6t – t2 = 6 × 4 – 16 = 24 – 16 = 8 m/s = 30 = 30 30
2g 2 10 4
v t
dx (6t t 2 ) dt = 30 + 41.25
0 0
= 71.25m.

t3 1 2 1
x 3t 2
80
(putting t = 4 sec.) h ut gt ; 30 25t 10t 2
3 3 2 2
5t2 – 25t – 30 = 0
tf rf
t2 – 5t – 6 = 0
19. (a, b, c, d) Area of v-t graph = v dt dr rf ri (t – 6) (t + 1) = 0 t = 6
ti ri
v
Hence area of v-t graph gives change in positive
vector. Hence (d) is correct. 25m/s
|change in position | = | displacement |
= distance 6sec
We can read data v on the v-axis. So we can get t
2.5sec
change in speed and velocity.
Hence (a) and (b) are correct.
20. (a, d) 8 × 10 + (10 – vr) 10 = 160
vr = 2 m/sec. h
8 = vb/r + vr 71.25m
vb/r = 8 – vr = 8 – 2 = 6 m/sec.
30m
21. (a, b, d) v
t
2.5sec 6sec
0 D
23. (a, b) If v be the velocity at a distance s from the starting
B C point and S be the total space covered, then
V
S
1
10 A Space-average of velocity = v ds
S0
and if T be the total time,
T
1
t time average of velocity = v dt
O T T+10 (2T–20) T0
74 IIT-JEE P HYSICS Challenger
Now if f be the constant acceleration, 29. (a, b, c)
v = u1 + ft and v2 = u12 + 2fs (a) The downward component of g parallel to OY is g sin ,
S hence the downward component along the groove is a
1
So that space-average = u12 2 fs ds = g sin sin . Since
S0
Y
1 S
= (u12 2 f S )3/ 2
3f S 0
A
x0
1 g=9.8m/s²
= (u12 2 f S )3 / 2 u13 , but u22 u12 2 f S
3f S a
y0
Short
2 u 32 u13 2 u12 u1u 2 u 22 cylinder
= 3 2 .
u2 u12 3 u1 u 2
O X

T
1
and time-average = (u1 ft ) dt y0
T0 sin 0.8
( x02 y02 )1/ 2
1 T and sin = 0.5.
(u1 ft ) 2
2 fT 0 a = (9.8) (0.5) (0.8) = 3.92 m/s2

1 1 2
= (u1 fT )2 u12 , (b) s = v0t + at , where
2 fT 2
but u2 = u1 + fT
s ( x02 y02 )1/ 2 5m and v0 = 0.
1 u22 u12 u2 u1
= 1
2 u2 u1 2 Thus, s (3.92)t 2 or t = 1.597s
2
24. (a, b, c) 2 90 1 (c) v = 0 + (3.92) (1.597) = 6.26 m/s.
30. (a, c, d) Let ux and uy be horizontal and vertical components
h1 sin 2 1
tan 2 1 cot 2 2 of velocity respectively at t = 0.
h2 sin 2 (90 1) Then, vy = uy – gt
25. (a, b, c) Hence, vy – t graph is straight line.
(a) Point D is a maximum of the x vs t curve. Therefore v x = vx t
= dx/dt = 0. Hence, x–t graph is straight line passing through origin.
(b) Without the exact equation for x as function of t one
cannot get a precise answer. The best we can do is to 1 2
The relation between y and t is y = uyt – gt
draw the tangent line at point C and get the slope. 2
This yields the answer. Hence, y-t graph is parabolic
dx vx = constant
vC 1.3 m / s
dt C Hence vx-t graph is a straight line.
(c) We proceed as in part (B), but here the tangent line has 31. (b, d) Since maximum heights are same, their time of flight
a negative slope and the answer should be should be same
T1 = T2
dx Also, vertical components of initial velocity are same
vE 0.13 m / s
dt E Since range of 2 is greater than range of 1.
26. (a, b) The acceleration at any time is the slope of the v vs t Horizontal component of velocity of 2 > horizontal
curve. component of velocity of 1.
(a) At A the slope is, from where the tangent line through Hence, u2 > u1.
A cuts the coordinate axes, a = –7.0/0.73 = –9.6 m/s2.
(b) The slope and therefore a is zero. 3s
32. (b, c) By definition average speed =
27. (a, b) v > 0 initially and a > 0 always t1 t2 t3
v > 0 always and increasing.
u1 u2 u3 u4
28. (b,c,d) It is known that range is same for complementary
angles of projection
s,t 1 s,t 2 s,t 3
1 + 2 = 90° is satisfied by (B, C, D)
MOTION IN ONE AND TWO DIMENSIONS 75

u1 u2 (c) If b = 0.1/s and using g = 9.8 m/s2,


s
For t1 : .......... (1) we have k = g/b2 = 980m. Then at t = 10s,
t1 2
y = (980m) (e–1 – 1) + (98 m/s) (10s) = 360m.
s u2 u3 dy
For t2 : .......... (2) v ( 98m / s) e 1
(9.8m / s 2 ) (0.1/ s ) 62 m / s
t2 2 dt
(d) At t = 60s,
s u3 u4
For t3 : ........ (3)
t3 2 dy 6
( 98m / s) e 98m / s
dt
3s u1 u4
For t1 t2 t3 : ........ (4) e 6
0.0025 ,
t1 t2 t3 2
From eq. (1), (2) and (3) dy
so 9.8 m / s
s s s u1 u4 dt
t1 t2 t3 2 35. (a, c, d)
The first ball is in air for 4 seconds and second ball is in
s s s air for 2 seconds.
Hence average speed is also =
t1 t2 t3 36. (a, b, c) When the particle returns to same point,
33. (a, b, d) displacement is zero, therefore average velocity is also
Work by Fnet = Change in KE zero.
In our case it is zero.
Total distance 2 (u 2 / 2 g ) u
vf vi Average speed = Total time 2 (u / g ) 2
aaverage 0 vi = vf
t 37. (a, b)
displacement area of v-t curve (a) vhel / water vsub / water vhel / sub
vaverage 0
time time
34. (a, b, c, d) = 17 ˆj ( 5) kˆ (17 ˆj 5kˆ) m/s
dy bt (b) vhel / air vhel / water vwater / air vhel / water vair / water
(a) bke g /b .
dt
= (17 ˆj 5kˆ ) 12iˆ ( 12iˆ 17 ˆj 5kˆ) m/s
2
d y (b, c) a = 3t2 + 1
Differentiating once more b 2 ke bt 38.
dt 2 v 1
dv
dy 3t 2 1 dv (3t 2 1) dt
Multiplying our expression for by –b and adding g dt 0 0
dt
yields b2ke–bt. v (t 3 1)10 2m / s v t3 1
Thus (1) is satisfied. Substituting t = 0 and recalling
that e0 = 1, we get y = 0. s 1
1 1
(b) Since the ball is released from rest, y = 0 at t = 0. ds (t 3 1) dt s 0.75
0 0
4 2
dy 39. (a, b, c) The time of flight is independent of horizontal
Using our expression for from (a),
dt component of velocity of sphere. Hence whatever be
we have 0 = –bk + g/b, which yields k = g/b2. the value of v0, the time of flight shall remain same.

1. (A) (q, s); (B) (r); (C) (p); (D) (s) 1


2. (A) (p, r); (B) (p, r); (C) (p); (D) (p) (C) KEH mu 2 cos 2 Min. KE
3. (A) q; (B) q; (C) r 2
4. (A) r; (B) s; (C) p; (D) r
1
(A) K mu 2 mgy , y = Ax – Bx2 Parabolic v2 20 20
2 (A) AB = maximum height = 20 m
(B) U = mgy = mg (Ax – Bx2) 2a 2 10
76 IIT-JEE P HYSICS Challenger

BD 20 20 (D) v 2 u2 2 gh
= 2
AD 20
v 2y u 2y 2 gh
1
20 1 10 12
s (1st ) 2 15 above horizontal
(B) 3
s (2 ) nd 20 [15] 5 –h = u sin t – (1/2) gt2 .... (1)
below horizontal
vA 20 20 1
– h = – u sin t – (1/2) gt2 .... (2)
(C) vB 2 2 2
20 2 10 20 From (1) and (2), t will be different.
7. (A) – p, (B) – p, (C) q, s ; (D) r, s
t AC 2t AB
(D) t 2 If angle between constant acceleration vector a and velocity v
AB t AB
is zero or 180° then path is straight line otherwise path must be
5. (A) q, s; (B) q, r; (C) p, r; (D) p, s parabolic.
(A) Slope is positive and decreasing a<0 8. (A) – q,r ; (B) – q,r ; (C) – p, s ; (D) – p, s
(B) Slope is negative and its magnitude is increasing (A) Since truck is moving, as seen from ground the ball will
(C) Slope is positive and increasing have a horizontal component of velocity hence the path
(D) Slope is negative and its magnitude is decreasing of the ball seen from ground will be parabola.
6. (A) q; (B) p, r; (C) p, r; (D) q, r (B) In for the truck, the horizontal component of velocity
(A) For same range, angles of projection are and (90° – will be zero, hence straight line w.r.t. truck.
R1 R2 , T1 T2 , h1 h2 , v y1 v y2 , same speed (C) Ball will have a horizontal component with truck hence
parabola as seen from truck.
(B) For same height, v y1 v y2 , T1 T2 (D) As seen from ground, the horizontal component of the
velocity will be zero.
2h
(C) t , v 2y 2 gh
g

1. 0.83 2
2 2 d d
Initially, AO d
3 2 3
A
Therefore, the time taken for AO to become zero
A1
d
3 2d 2d 2 0.5
A2 = 0.83 s
v cos 30 3v 3 3v 3 2
2. 288
O
480
C2 Velocity of car from A = 60 km / hour
B2 C1 8
480
B B1 C Velocity of car from B = 40 km / hour
12
Let the two cars meet at t hour
Figure shows the motion of the particles. These particles 480
t 4.8 hours
will meet at the centroid O of the triangle. At any instant the 60 40
particles will form an equilateral triangle ABC with the same The distance s = vA × t = 60 × 4.8 = 288 km.
centroid O. Let us fix our attention on the motion of any one 3. 59
Such problems can be tackled by keeping in mind, that for
particle, say A. At any instant, its velocity makes an angle
any event, to occur, time remains same for both bodies.
30° with AO. The component of this velocity along AO is v Let P be the point, where the two engines cross each other.
cos 30°. This component will represent the rate of decrease If t hr be the time to occur this event, then total distance
of the distance AO. covered by the two trains should be equal to 100 km.(fig.)
MOTION IN ONE AND TWO DIMENSIONS 77

i.e., AP + BP = 100 Time taken to reach point B by walking is,


1 1 x vr d 2 d d
50t 18t 2 50t 18t 2 100 t2 = = = = hr
2 2 u uv u 2.5 12.5u
100t = 100 Given that both the swimmers reach the destination
A P simultaneously, so we have
x B
t = 1 hr. d d d
100km t = t1 + t2 or = +
1.5 2.5 12.5u
1 or u = 3.0 kph.
x = AP = 50 (1) + 18(1) x = 50 + 9 = 59 km.
2 6. 2
4. 50 The situation is shown in fig.
Here, u = u m/s, a = g = – 10 m/s2 and s = 80m.
Substituting the values in 30°
1 –
s = ut + at2, we have, 80 = ut – 5t2
2 u
+

29.4 m
u u 2 1600
or 5t2 – ut + 80 = 0 or t = and
10

u u 2 1600
10
Now it is given that, A B
2 2
u u 1600 u u 1600 The time taken by the body is equal to the time taken by the
– =6
10 10 freely falling body from the height 29.4 m with initial velocity
u sin = 9.8 sin 30º. This is given by
u 2 1600
or =6 u 9.8
5 4.9 m/s
2 2
or u 2 1600 = 30 1
or u2 – 1600 = 900 Applying the formula, s = u t + g t2, we have
u2 = 2500 2
or u = ± 50 m/s 1
Ignoring the negative sign, we have, u = 50 m/s 29.4 = 4.9 t + (9.8) t2
2
5. 3.0
or 4.9 t2 + 4.9 t – 29.4 = 0
Let us take velocity of swimmer with respect to water is v and
that of river current is vr. The swimmer which crosses the (because s, u & g are all in downward direction)
river along the straight line AB, has to swim in upstream t2 + t – 6 = 0
direction such that its resultant velocity becomes toward or t = 2s or –3s
AB as shown in figure. If the width of river is assumed to be Time taken to reach ground = 2 second
d, then 7. 49
Vertical velocity after 10s is,
Resultant velocity of first swimmer is v1 = v2 vr2
3
Time taken by her to cross the river is v (98sin 60º ) (9.8) 10 98 98
2
d d d
t= = = hr.. 3
v 2
vr2 2
2.5 2 2 1.5 = 98 1 98 (0.866 1)
2
Second swimmer if swims along AB, she is drifted towards
point C, due to river flow as shown in figure and then she 98 ( 0.134) .
has to walk down to reach point B with velocity u.
Vertical momentum of the ball after
Here crossing velocity of second swimmer is v, as its is
swimming along normal direction. 10s = mv = 0.5 × (–98 × 0.134) kg m/s;
Time taken to cross the river by her is, Initial vertical momentum of the ball

d d 3
t1 = = hr 0.5 98 0.5 98 0.866
v 1.5 2
Her drift due to river flow is, Change in vertical momentum
d = 0.5 × 98 × 0.866 – (–0.5 × 98 × 0.134)
x = vr × = 0.5 × 98 [0.866 + 0.134] = 0.5 × 98 = 49 kg m/s;
v
78 IIT-JEE P HYSICS Challenger
Horizontal component of velocity remains same, so there is This is quadratic equation in tan with two positive roots.
no change in momentum along horizontal direction. Therefore there are two values of less than 90°, so there
Change in momentum = 49 kg m/s. are two possible angles of projection.
8. 2 Now we are asked to calculate tan( ) which is equal to
For body to hit the lowest plane with minimum velocity, it
should be able to just clear the second step tan tan
Let R = range for minimum velocity 1 tan tan
h = height for minimum velocity where tan and tan are the roots of equation (1).
u = minimum velocity
t = time of motion 36 9
Hence tan tan (sum of roots)
R = u × t t = R/u ....(1) 8 2
1 2 17
h gt ....(2) and tan tan (product of roots)
2 8
From (1) & (2), 9/2
Therefore tan( ) 4
2 2 1 (17 / 8)
1 R gR
h g h ....(3)
u22 2u 2 tan
R = 2 × 20 cm = 40cm = 0.4 m
= – tan ( + )
h = 2 × 10cm = 20 cm = 0.2m
= – (–4)
1 R2 1 0.4 0.4 = 4
u2 g u2 10 4
2 h 2 0.2 10. 80
u = 2m/s The vertical components must be equal.
9. 4
Let the angle of projection of the particle be . C

y
30

vB
vA h
37°

10
53°
x
O A
10 m
The path of the particle has to pass through the point where
x = 40, y = 10. vA cos 53° = vB cos 37°
Using the equation of the path of the projectile in the form cos 37
2 or vA = vB cos (90 37 )
x g
y x tan (1 tan 2 )
2v 2 60
2 or vA = 60 cot 37° = tan 37
x
gives y (1 tan 2 )
x tan
180 60 4
The point (40, 10) lies on this path so = = 80 units
3
80
10 40 tan (1 tan 2 )
9
8 tan 2 36 tan 17 0 ....(1)
1. A bob is hanging over a pulley inside a car through a string. (a) 20N (b) 10N
The second end of the string is in the hand of a person (c) 12N (d) 15N
standing in the car. The car is moving with constant 4. Three blocks A, B and C of equal mass m are placed one
acceleration a directed horizontally as shown in figure. Other over the other on a smooth horizontal ground as shown in
end of the string is pulled with constant acceleration a figure. Coefficient of friction between any two blocks of A,
vertically. The tension in the string is equal to B and C is 1/2.
car
a
a

The maximum value of mass of block D so that the blocks A,


B and C move without slipping over each other is
(a) m g2 a2 (b) m g2 a2 ma (a) 6 m (b) 5 m (c) 3 m (d) 4 m
5. Two blocks of masses m1= 4kg and m2 = 6 kg are connected
(c) m g2 a2 ma (d) m (g + a)
by a string of negligible mass passing over a frictionless
2. The two blocks, m = 10 kg and M = 50 kg are free to move as pulley as shown in the figure below. The coefficient of
shown. The coefficient of static friction between the blocks friction between the block m1 and the horizontal surface is
is 0.5 and there is no friction between M and the ground. A 0.4. When the system is released, the masses m1 and m2
minimum horizontal force F is applied to hold m against M start accelerating. What additional mass m should be placed
that is equal to over mass m1 so that the masses (m1 + m) slide with a uniform
speed?
F m
M Smooth m

m1
(a) 100 N (b) 50 N (c) 240 N (d) 180 N
3. What is the maximum value of the force F such that the
block shown in the arrangement, does not move?
F
1 m2
60º 2 3
m 3kg (a) 12 kg (b) 11kg
(c) 10 kg (d) 2 kg

MARK YOUR
1. 2. 3. 4. 5.
RESPONSE
80 IIT-JEE P HYSICS Challenger
6. A ship of mass 3 × 107 kg initially at rest, is pulled by a force
of 5× 104 N through a distance of 3m. Assuming that the
resistance due to water is negligible, the speed of the
ship is
(a) 1.5 m/sec. (b) 60 m/sec.
(c) 0.1 m/sec. (d) 5 m/sec.
7. is the angle of the incline when a block of mass m just
starts slipping down. The distance covered by the block if
thrown up the incline with an initial speed u0 is

(a) u02 / 4g sin (b) 4u02 / g sin The velocity of the block will be [assume that the block
does not rotate]
(c) u02 / sin / 4g (d) 4u02 sin / g (a) 3 m/s (b) 2 m/s
8. A given object takes n times as much time to slide down a (c) 1/2 m/s (d) 1 m/s
45° rough incline as it takes to slide down a perfectly smooth 12. A plank of mass M 1 = 8 kg with a bar of mass
45° incline. The coefficient of friction between the object M2 = 2 kg placed on its rough surface, lie on a smooth floor
and the incline is of elevator ascending with an acceleration g/4. The
coefficient of friction is µ = 1/5 between m1 and m2.
(a) (1 – 1/n2) (b) 1/(1 – n2)

(c) (1 1/ n 2 ) (d) 1/ (1 n 2 ) g/4


9. The acceleration of the block B in the above figure, assuming
the surfaces and the pulleys P1 and P2 are all smooth. m2
2m P1 4m F m1
F A B
P2
A horizontal force F = 30 N is applied to the plank. Then the
(a) F/4m acceleration of bar and the plank in the reference frame of
(b) F/6m elevator are
(c) F/2m (a) 3.5 m/s2, 5 m/s2
(d) 3F/17m (b) 5 m/s2, 50/8 m/s2
(c) 2.5 m/s2, 25/8 m/s2
10. A block P of mass m is placed on a horizontal frictionless
(d) 4.5 m/s2, 4.5 m/s2
plane. A second block of same mass m is placed on it and is
13. All the surfaces are frictionless then acceleration of the block
connected to a spring of spring constant k, the two blocks
B is
are pulled by distance A. Block Q oscillates without slipping.
(a) 2g/13 (b) 3g/13
What is the maximum value of frictional force between the
(c) 4g/13 (d) g/13
two blocks.

A B
k
Q
s
P m m

(a) kA/2 (b) kA


(c) s mg (d) zero
11. A system is shown in the figure. A man standing on the
block is pulling the rope. Velocity of the point of string in
m C
contact with the hand of the man is 2 m/s downwards.

MARK YOUR 6. 7. 8. 9. 10.


RESPONSE 11. 12. 13.
LAWS OF MOTION 81

14. A smooth ring P of mass m can slide on a fixed horizontal


h/4
rod. A string tied to the ring passes over a fixed pulley and
carries a block Q of mass (m/2) as shown in the figure. At an
h m
instant, the string between the ring and the pulley makes an
angle 60° with the rod. The initial acceleration of the ring is
M F
m (a) 30 N
60° (b) 4 0 N
P
(c) 120 N
(d) none of the above
18. There are two blocks A and B in contact with vertical and
Q m/2 horizontal smooth surfaces respectively, as shown in the
figure. Acceleration of A and B are aA and aB respectively
2g 2g g along their constrained direction of motions. Relation
2g
(a) (b) (c) (d) between aA and aB is (Assume sin 23° = 2/5).
3 6 9 3
15. A massive wooden plate of unknown mass M remains in
equilibrium in vacuum when n bullets are fired per second
on it. The mass of each bullet is m (M >> m) and it strikes the
plate at the centre with speed v. If the coefficient of restitution
is e, then M is equal to

m (a) 2a A 21 aB (b) 2aA = 5aB

mv n mev n (c) 5aA = 2aB (d) 21 a A 2aB


(a) (b)
g g 19. A bead of mass m is located on a parabolic wire with its axis
vertical and vertex at the origin as shown in the figure and
m v (1 e) n whose equation is x2 = 4ay. The wire frame is fixed and the
(c) (d) n m
g bead can slide on it without friction. The bead is released
from point y = 4a on the frame from rest. The tangential
16. A block is resting on a horizontal plate in the xy plane and
acceleration of the bead when it reaches the position given
the coefficient of friction between block and plate is . The by y = a is
plate begins to move with velocity u = bt2 in x direction. At
what time will the block start sliding on the plate. y
m
b bg
(a) (b)
g 2

g g x
(c) (d)
b 2b
17. A block of mass m = 2 kg is placed on a plank of mass M = 10 kg
which is placed on a smooth horizontal plane. The coefficient
1 3g
of friction between the block and the plank is . If a g
3 (a) (b)
2 2
horizontal force F is applied on the plank, then find the maximum
value of F for which the block and the plank move together. g g
(c) (d)
(Take g = 10 m/s2) 5 2

MARK YOUR 14. 15. 16. 17. 18.


RESPONSE 19.
82 IIT-JEE P HYSICS Challenger
20. In the figure acceleration of bodies A, B and C are shown
with directions. Values b and c are w.r.t. ground whereas a is
acceleration of block A w.r.t. wedge C. Acceleration of block
A w.r.t. ground is

B C u A

(a) u = 5m/s (b) u = 10 m/s


(c) u = 20 m/s (d) u = 15 m/s
23. If 1 = 2 in figure, (pulley is frictionless). Choose the correct
(a) (b c) 2 a 2 option
(b) c – (a + b) cos W1
(a) T1 = T2 = T3 = W2 =
(c) 2 2 2 cos 1
(b c ) c 2(b c).c.cos
W1
(d) (b c)2 c 2 2(b c).c.cos (b) T1 T2 = T3 = W2 =
2 cos 1
21. The system shown in the adjacent figure is in equilibrium.
All the blocks are of the equal mass m and springs are ideal.
When the string between A and B is cut then

k
1
T1
2 T3
T2
A
W1

B
W1

k W1
(c) T1 = T2 T3 = W2 =
2 cos 1

C W1
(a) the acceleration of block A is 2g while the acceleration (d) T1 = T2 = T3 W2
2 cos 1
of block C is g
24. The inclined plane shown in figure has an acceleration ‘a’
(b) the acceleration of block B is 2g while the acceleration
to the right. The block will side on the plane if (µs tan is
of block C is zero
the coefficient of static friction for the contacting surfaces)
(c) the acceleration of block A is 2g while the acceleration
Y
of block B is g
(d) the acceleration of block B is g while the acceleration N
of block C is zero f
22. In the arrangement shown in the adjacent figure, the two 90° –
pulleys are fixed and the two blocks A and B are made to
move downwards so that they decelerate at 10 m/s2. The
arrangement achieving the above is not shown in the a
diagram. The block C which is fixed to the middle of the
string, moves upward with a constant velocity u. At a certain
mg
d
instant, (shown in the figure) = 30 ° and 1 radian/s. X
dt (a) a < g tan ( – ) (b) a > g tan ( + )
It can be concluded that (c) a > g tan ( – ) (d) a > g cot ( – )

MARK YOUR
20. 21. 22. 23. 24.
RESPONSE
LAWS OF MOTION 83

25. A student’s life was saved in an automobile accident


because an air bag expanded in front of his head. If the car
B
had not been equipped with an air bag, the windshield would
have stopped the motion of his head in a much shorter time.
Compared to the windshield, the air bag
(a) exerts a much smaller impulse
(b) causes a much smaller change in kinetic energy A
(c) exerts a much smaller force 45°
m1
(d) does much more work
26. Five persons A, B, C, D and E are pulling a cart of mass 100 P
kg on a smooth surface and cart is moving with acceleration m2
3 m/s2 in east direction. When person A stops pulling, it
moves with acceleration 1 m/s2 in the west direction. When
person B stops pulling, it moves with acceleration 24 m/s2 (a) (b) 24 N
40 2 N
in the north direction. The magnitude of acceleration of the
cart when only A and B pull the cart keeping their directions (c) 40 N (d)
24 2 N
same as the old directions, is 30. Two blocks of mass 2 kg are connected by a massless ideal
spring of spring constant k = 10 N/m. The upper block is
(a) 26 m/s2 (b) 3 71 m/s2
2
suspended from roof by a light inextensible string A. The
(c) 30 m/s (d) 25 m/s2 system shown is in equilibrium. The string A is now cut, the
27. What is the tension in cord AB ? acceleration of upper block just after the string A is cut will
be (g = 10 m/s2)
///////////////////////////////////////////////
A
40° A B 40° 2kg
30° 30°

70N 2kg
(a) 0 m/s2 (b) 20 m/s2
(a) 25.2 N (b) 18.3 N (c) 10 m/s 2 (d) 15 m/s2
(c) 15.5 N (d) 21.5 N 31. In the figure shown, the pulleys and strings are massless.
28. A box weighing 100N is at rest on a horizontal floor. The The acceleration of the block of mass 4m just after the
coefficient of static friction between the box and the floor is system is released from rest is ( = sin–1 3/5)
0.4. What is the smallest force F exerted eastward and upward
at an angle of 30° with the horizontal that can start the box in
motion ?
(a) 27.5 N (b) 37.5 N
(c) 14.2 N (d) 45.4 N
29. The block of mass m1 = 20kg lies on the top of block of 4m
mass m2 = 12 kg. A light inextensible string AB connects m m
mass m1 with vertical wall as shown. The coefficient of
friction is µ = 0.25 for all surfaces in contact. A horizontal
2g 2g
force P is applied to block of mass m2 such that it just (a) downwards (b) upwards
5 5
slides under block of mass m1. Then the tension in the
string AB is (Take g = 10 m/s2) 5g 5g
(c) downwards (d) upwards
11 11

MARK YOUR 25. 26. 27. 28. 29.


RESPONSE 30. 31.
84 IIT-JEE P HYSICS Challenger
32. Two small masses m1 and m2 are at rest on a frictionless, Student A : vA cos = vB.
fixed triangular wedge of angles 30° and 60° as shown. They Student B : vB cos = vA.
are connected by a light inextensible string. The side BC //////// ////////
of wedge is horizontal and both the masses are 1 metre
vertically above the horizontal side BC of wedge. There is
no friction between the wedge and both the masses. If the
string is cut, which mass reaches the bottom of the wedge
first ? (Take g = 10m/s2) vA

B vB A
m2
m1 (a) A is correct, B is wrong
fixed wedge
B 60° 30° (b) B is correct, A is wrong
C
///////////////////////////////////////// (c) Both are correct
(D) Both are wrong
(a) Mass m1 reaches the bottom of the wedge first.
36. A string of negligible mass going over a clamped pulley of
(b) Mass m2 reaches the bottom of the wedge first.
mass m supports a block of mass M as shown in the figure.
(c) Both reach the bottom of the wedge at the same time.
The force on the pulley by the clamp is given by
(d) It’s impossible to determine from the given information.
33. Two weights are hung over two frictionless pulleys as shown
in figure. What weight W will cause the 300kg block to just
m
start moving to the right ?

///////////////////////////////////////////////////////

M
45° 300kg 30°

(a) 2 Mg (b) 2 mg
µs=0.3 (c) M m 2 m2 g (d) M m 2 M2 g
40kg W
37. A small mass slides down a fixed inclined plane of inclination
(a) 108 kg (b) 115 kg with the horizontal. The coefficient of friction is
(c) 98 kg (d) 120 kg µ = µ0x where x is the distance through which the mass
34. In figure, block 1 is one-fourth the length of block 2 and slides down and µ0 is a constant. Then the speed is maximum
weighs one-fourth as much. Assume that there is no friction after the mass covers a distance of
between block 2 and the surface on which it moves and that cos sin
the coefficient of sliding friction between blocks 1 and 2 is (a) (b)
µk = 0.2. After the system is released find the distance block 0 0
2 has moved when only one-fourth of block 1 still on block 2 tan
tan
2. Block 1 and block 3 have the same mass. (c) (d)
0 0
38. Two smooth spheres each of radius 5cm and mass 10kg rest
1
m one on the other inside a fixed smooth cylinder of radius
8cm. Then the magnitude of normal reaction exerted by one
2
4m sphere on the other sphere is
m3
Configuration at t = 0

(a) 1/7.47 (b) 2/7.47


(c) 1/3.37 (d) 4/7.47
35. Two masses A and B are connected with two an inextensible
string to write constraint relation between vA and vB. (a) 125N (b) 225N (c) 150N (d) 100N

MARK YOUR 32. 33. 34. 35. 36.


RESPONSE 37. 38.
LAWS OF MOTION 85

39. A body of mass 400 kg is suspended at the lower end of a block of mass m is initially at rest. The block of mass M is
light vertical chain and is being pulled up vertically (see released from rest with spring in unstretched state. The
figure). Initially, the body is at rest and the pull on the chain minimum value of M required to move the block up the
is 6000g N. The pull gets smaller uniformly at the rate of plane is (neglect mass of string and pulley and friction in
360g N per meter through which the body is raised. What is pulley.)
the velocity of the body when it has been raised 10m?

m M
37°
/////////// /////////////////////////////////////////
3 4
(a) m (b) m
5 5

6 3
(c) m (d) m
5 2
Position at time t
42. A uniform rope of length L and mass M is placed on a smooth
W = 400g fixed wedge as shown. Both ends of rope are at same
horizontal level. The rope is initially released from rest, then
T0 = 6000g the magnitude of initial acceleration of rope is
y

Initial position

(a) 44.1 m/s (b) 43.2 m/s


(c) 54.1 m/s (d) 33.3 m/s /////////////////////////////////////////////////////
40. A system is shown in the figure. Block A moves vertically
upwards with velocity 10 m/s. The speed of the mass B at (a) M (cos – cos ) g (b) zero
the shown instant will be (c) M (tan – tan ) g (d) None of these
43. An insect crawls up a hemispherical surface very slowly
////////////////////////////////////////////
(see fig.). The coefficient of friction between the insect and
the surface is 1/3. If the line joining the center of the
45° 45° hemispherical surface to the insect makes an angle with
10m/s the vertical, the maximum possible value of is given by
A B

(a) 10 2 m / s (b) 10 m/s

20
(c) 5 3 m/s (d) m/s
3
41. A block of mass m is attached with massless spring of force (a) cot = 3 (b) tan = 3
constant k. The block is placed over a fixed rough inclined
(c) sec = 3 (d) cosec = 3
surface for which the coefficient of friction is µ = 3/4. The

MARK YOUR
39. 40. 41. 42. 43.
RESPONSE
86 IIT-JEE P HYSICS Challenger
44. A 0.11 kg baseball is thrown with a speed of 17 m/s towards cases. At the highest point of the track, the normal reaction
a batsman. After the ball is stuck by the bat, it has a speed of is maximum in
34 m/s in the direction shown in figure. If the ball and bat are
in contact for 0.025s, find the magnitude of the average
force exerted on the ball by the bat.
(a) (b) v
v
vf =34m/s

(c) v (d) v

48. A weight W is supported by two cables as shown. The


tension in the cable making angle with horizontal will be
the minimum when the value of is
mball=0.11 kg
60°

////////

////////
vi=17m/s T2 T1

(a) 167.90 N (b) 171.90 N 60°


(c) 197.90 N (D) 122.20 N
45. In the figure (i) an extensible string is fixed at one end and
the other end is pulled by a tension T. In figure (ii) another W
identical string is pulled by tension T at both the ends. The
(a) 0 (b) 30°
ratio of elongation in equilibrium of string in (i) to the
(c) 60° (d) None of these
elongation of string in (ii) is
49. An object of mass M is moving on a conveyor belt. The
T object and the belt move together at a constant velocity v .
(i) The coefficient of static friction is µs, the coefficient of kinetic
friction is µk, and the acceleration due to gravity is g. What
///////////////////////////////////////// is the magnitude of the force of friction on the object ?
(a) (µs – µk) Mg (b) µs Mg
T (ii) T (c) µkMg (d) zero
(a) 1 : 1 (b) 1 : 2 (c) 2 : 1 (d) 0 50. A mass M is suspended by two springs A and B of force
46. Two monkeys of masses 10 kg and 8 kg are moving along a constants k1 and k2 respectively as shown in the diagram.
vertical rope which is light and inextensible, the former The total stretch of springs in equilibrium is
climbing up with an acceleration of 2m/s2 while the latter ///////////////
coming down with a uniform velocity of 2m/s. Find the
tension (in newtons). k1
A

B k2

Mg 2Mg
(a) (b)
(a) 200 N (b) 150 N (c) 300 N (d) 100 N k1 k 2 k1 k 2
47. A small block is shot into each of the four tracks as shown
below. Each of the tracks rises to the same height. The Mg Mg ( k1 k 2 )
(c) (d)
speed with which the block enters the track is the same in all 2 ( k1 k 2 ) k1k 2

MARK YOUR 44. 45. 46. 47. 48.


RESPONSE 49. 50.
LAWS OF MOTION 87

51. A minimum horizontal force of 10N is necessary to just hold


a block stationary against a wall. The coefficient of friction F x F a2 - x2
(c) (d)
between the block and the wall is 0.2. The weight of the 2m a 2m x
block is 54. If in the arrangement shown, motor runs winding rope at a
rate of v m/s the upward speed of platform will be

////////////////////
///////////////
10N

(a) 100 N (b) 50N (c) 20N (d) 2N


52. A different forces is applied to each of four 1 kg blocks to
slide them across a uniform steel surface at constant speed
as shown. In which diagram is the coefficient of friction
between the block and the steel smallest ?
F=5N 1 kg v v
block (a) (b)
(a) 3 4
////////////////////////////////////////
Steel v
(c) (d) None of these
F=3N 1 kg 7
block 55. A and B are two smooth pegs on the same horizontal line.
(b)
//////////////////////////////////////// An inextensible light string carrying a heavy smooth ring
Steel (which can freely slide on the string) passes over the pegs
F=2N as shown in the figure.
1 kg
The free ends of the string are pulled down vertically with
(c) block
//////////////////////////////////////// speed u and v respectively. The speed with which the ring
Steel goes up at any instant is

F=4N 1 kg /////////////// ///////////////


(d) block A B
////////////////////////////////////////
Steel
53. Two particles of mass m each are tied at the ends of a light
string of length 2a. The whole system is kept on a frictionless u v
horizontal surface with the string held tight so that each
mass is at a distance 'a' from the centre P (as shown in the
figure). Now, the mid-point of the string is pulled vertically (u v )
(a) (u + v) cos (b) cos
upwards with a small but constant force F. As a result, the 2
particles move towards each other on the surface. The
magnitude of acceleration, when the separation between u v u v
(c) (d)
them becomes 2x, is 2cos cos
56. A block of mass m is pulled in the direction shown in the
F figure on a rough horizontal ground with a constant
acceleration of magnitude ‘a’. The magnitude of the
frictional force is –
m m
P
a a

F a F x
(a) (b) (a) T cos – ma (b) µkmg
2m a 2 - x 2 2m a 2 - x 2
(c) µk (T – mg) (d) µk T sin

MARK YOUR 51. 52. 53. 54. 55.


RESPONSE 56.
88 IIT-JEE P HYSICS Challenger
57. For the arrangement shown in the figure let ‘a’ and T be the (a) 0° (b) 30°
acceleration of the blocks and tension in the string (c) 45° (d) 90°
respectively. The string and the pulley are frictionless and 60. A 10 kg block A resting against 50 kg block B is shown in
massless. Which of the graphs show the correct relationship figure. The coefficient of static friction between block A and
between ‘a’ and T for the system in which sum of the two the wall is negligible. If P = 100N, determine the value of µs
masses m1 and m2 is constant. (as shown) for which motion is impending.
P =100N
/////////////////////////////

/////////////////////////////////
A

m1 > m2
µs Wall
m2 B

m1
////////////////////////////////////////////
Floor µs
(a) 0.962 (b) 0.162
T T (c) 0.362 (d) 0.562
61. An overweight acrobat, "weighing" in at 115 kg, wants to
perform a single hand stand. He tries to cheat by resting
one foot against a smooth frictionless vertical wall. The
(a) (b) horizontal force there is 130 N. What is the magnitude of the
force exerted by the floor on his hand? Answer in N.
a2 a2 (a) 1134 (b) 1257
(c) 997 (d) 1119
T T 62. A parabolic bowl with its bottom at origin has the shape

x2
y= . Here x and y are in meter. The maximum height at
20
(c) (d) which a small mass m can remain on the bowl without slipping
(coefficient of static friction is 0.5) is
1/a2 1/a2 (a) 2.5m (b) 1.25m
(c) 1.0m (d) 4.0m
58. The force exerted by the rough incline on stationary block
of mass 1 kg is closest to
y
1kg µN
N

(a) 1N (b) 10N


(c) Impossible to determine without the coefficient of static mg
friction x
(d) Impossible to determine without the angle of the incline
59. A particle moves in the X-Y plane under the influence
of a force such that its linear momentum is
p(t ) A [iˆ cos(kt ) ˆj sin( kt )], where A and k are constants.
The angle between the force and the momentum is

MARK YOUR 57. 58. 59. 60. 61.


RESPONSE 62.
LAWS OF MOTION 89

63. A car is coming down hill with constant velocity. A bob is 67. A shopper pushes a shopping cart of a store with a constant
hung from its ceiling. Choose correct statement force of 75 N [forward]. The shopping cart exerts a force of
75 N [backward] on the shopper
(a) only if the velocity of the cart is constant.
(b) only if there is no friction between the cart and the
floor.
(c) only if the velocity of the cart is increasing.
(d) under all circumstances assuming the system to be the
(a) The string joining bob to ceiling is vertical shopper and cart.
(b) The string is perpendicular to the inclined plane
68. In figure, two blocks are separated by a uniform strut
(c) The string is parallel to the inclined plane
attached to each block with frictionless pins. Block A
(d) None of these
weighs 400N, block B weighs 300N, and the strut AB weigh
64. A cart initially moving on a frictionless track filled with sand
200N. If µ = 0.25 under B, determine the minimum coefficient
has a hole at its bottom from where sand leaks out –
of friction under A to prevent motion.
Student-A says : The velocity of cart remains constant
although mass of sand in cart decrease.

//////
Student- B say : Since mass is decreasing, the velocity of B

//////
cart must increase.
(a) Student-A is correct, Student-B is wrong

//////
(b) Student-A is wrong, Student-B is correct 30°
A

//////
(c) Both are correct (d) Both are wrong 60°
65. The string between blocks of mass m and 2m is massless //////////////////////////////////
and inextensible. The system is suspended by a massless
(a) 0.4 (b) 0.2
spring as shown. If the string is cut find the magnitudes of
(c) 0.8 (d) 0.1
accelerations of mass 2m and m (immediately after cutting)
69. A block of mass m is placed on an inclined surface.
Coefficient of friction between plane and block is
µ > tan . A force F = kt is applied on block at t = 0, then
which of the following represents variation of magnitude
of frictional force with time ?

F=kt
m
2m
µ > tan
m

g
(a) g, g (b) g, fr fr
2
(a) (b)
g g g Time Time
(c) ,g (d) ,
2 2 2
66. A bowling ball is tied to a rope such that the rope lifts the
ball straight up at a constant velocity. The magnitude of the
tension in the rope is fr fr
(a) greater than the force of gravity on the ball (c) (d)
(b) equal to the net force acting on the ball Time Time
(c) less than the force of gravity on the ball
(d) equal to the force of gravity on the ball

MARK YOUR 63. 64. 65. 66. 67.


RESPONSE 68. 69.
90 IIT-JEE P HYSICS Challenger
70. Two masses m and M are attached to strings as shown in

////////////////////////////
the figure. In equilibrium, tan is

m=3kg
T

////////
A
P
B m=5kg
C
m=4kg
///////////////////////////////
/////////

(a) 12.34 N (b) 32.34 N


m
45° 45° (c) 62.12 N (d) 6.34 N
73. Three blocks A, B, C of weights 40N, 30N, 80N respectively
are at rest on an inclined plane as shown in figure. Determine
the smallest value of coefficient of limiting friction (µs) for
M
which equilibrium of system is maintained.
y
(a) 1 + (2M/m) (b) (1 + 2m/M)

////
(c) 1 + (M/2m) (d) None of these

///
O

////
71. What force P must be applied to the weightless wedges

/
x

/
shown to start them under the 1000N block ? The angle of

/ /
////
friction at all contact surfaces is 10°. A
B ///
/////
C /////////
1000N ///
//// /////
///
///30°

=15° (a) 0.1757 (b) 0.2757


(c) 0.5757 (d) 0.8757
P 74. A pail filled with sand has a total mass of 60 kg. A crane is
P lowering it such that it has an initial downward acceleration
of 1.5 m/s2. A hole in the pail allows sand to leak out. If the
force exerted by the crane on the pail does not change, what
(a) 221.32 N (b) 121.32 N
(c) 421.32 N (d) 321.32 N mass of sand must leak out before the downward acceleration
72. Determine the force P required to impend the motion of the decreases to zero?
block B shown in figure. Take coefficient of friction = 0.3 for (a) 9.2 kg (b) 20 kg
all surfaces in contact. (c) 40 kg (d) 51 kg

MARK YOUR
70. 71. 72. 73. 74.
RESPONSE

y
PASSAGE-1

B
Two smooth blocks are placed at a smooth corner as shown. Both
the blocks are having mass m. We apply a force F on the small smooth
block m. A m
F m =37°

x
LAWS OF MOTION 91

Block A presses the block B in the normal direction, due to which 6. If applied force F = 120 N, then magnitude of acceleration of
pressing force on vertical wall will increase and pressing force on 15 kg block will be
the horizontal wall decrease, as we increase F. ( = 37° with (a) 8 m/s² (b) 4 m/s²
horizontal). (c) 3.2 m/s² (d) 4.8 m/s²
As soon as the pressing force on the horizontal wall by block B
becomes zero, it will loose the contact with the ground. If the
value of F is further increased, the block B will accelerate in upward PASSAGE-3
direction and simultaneously the block A will move toward right.
1. What is minimum value of F, to lift block B from ground ? For the object to be in equilibrium
Fx = 0, Fy = 0, = 0 and =0
25 5 3 4 7. As shown in figure, the tension in the horizontal cord is
(a) mg (b) mg (c) mg (d) mg
12 4 4 3 30N. The weight of the object is
2. If both the blocks are stationary, the force exerted by ground
on block A is ////////////////////////////////

//////////////////////////
40°
3F 3F
(a) mg (b) mg cord 2 50°
4 4
4F 4F P 30N
(c) mg (d) mg
3 3 cord 1
3. If acceleration of block A is a rightward, then acceleration of weight = W
block B will be
3a 4a (a) 25.2N (b) 22.2 N
(a) upwards (b) upwards (c) 39.2 N (d) 42.5 N
4 3
8. If W = 40N in the equilibrium situation shown in figure, find
3a 4a T1 and T2.
(c) upwards (d) upwards
5 5
///////////////////////////////////////////////

//////////////////////////
PASSAGE-2 60°
T1
A block of mass 15 kg is placed over a frictionless horizontal T2
surface. Another block of mass 10 kg is placed over it, that is 70°
connected with a light string passing over two pulleys fastened W
to the 15 kg block. A force F = 80N is applied horizontally to the
free end of the string. Friction coefficient between two blocks is
(a) T1 = 48.3 N, T2 = 25 N
0.6. The portion of the string between 10 kg block and the upper
pulley is horizontal. Pulley, string and connecting rods are (b) T1 = 58.3 N, T2 = 31 N
massless. (Take g = 10 m/s²) (c) T1 = 65.3 N, T2 = 15 N
(d) T1 = 54.4 N, T2 = 21 N
9. The weight W1 in figure is 300N. Find T1, T2, T3, and W2.
10kg

//////////////////////////////////////////////////
µ=0.6 T3 T1
15 kg
53° T 2 37°
F=80N
A
C
Smooth
4. The magnitude of acceleration of the 10 kg block is W1 W2
(a) 3.2 m/s² (b) 2.0 m/s²
(c) 1.6 m/s² (d) 0.8 m/s² (A) T1 = 500 N, T2 = 400 N, T3 = 670 N, W2 = 530 N
5. The magnitude of acceleration of the 15 kg block is (B) T1 = 500 N, T2 = 300 N, T3 = 670 N, W2 = 530 N
(a) 4.2 m/s² (b) 3.2 m/s² (C) T1 = 400 N, T2 = 400 N, T3 = 670 N, W2 = 530 N
(c) 16/3 m/s² (d) 2.0 m/s² (D) T1 = 500 N, T2 = 400 N, T3 = 300 N, W2 = 530 N

MARK YOUR 1. 2. 3. 4. 5.
RESPONSE 6. 7. 8. 9.
92 IIT-JEE P HYSICS Challenger
13. At the instant the speed of block of mass m1 is maximum,
PASSAGE-4
the compression in the spring is
(a) 5 cm (b) 10 cm
In figure, the weights of the objects are 200 N and 300 N.
The pulleys are essentially frictionless and massless. Pulley (c) 20 cm (d) 60 cm
P1 has a stationary axle but pulley P2 is free to move up and 14. At the instant speed of block of mass m1 is maximum, the
down. speed of block of mass m2 is
///////////////////////////////// //////////// (a) zero (b) 2 m/s
2 4
(c) m/s (d) m/s
3 3
P1
15. The maximum force exerted by block of mass m1 on block of
mass m2 is

T2 P2 40
(a) zero (b) N
3
A
80
(c) N (d) 40N
T1 3
200N
B
PASSAGE-6

300N An elevator is moving in vertical direction such that its


10. Tension T1 is velocity varies with time as shown in figure. Their upward
(a) 327 N (b) 164 N (c) 315 N (d) 300 N direction is taken as positive. A man of mass 60 kg is standing
11. Tension T2 is in the elevator on a weighing machine. When the lift is
(a) 327 N (b) 164 N (c) 315 N (d) 300 N accelerated up, the reading of the weighing machine
12. Acceleration of A is increases and when the lift is accelerating down, the reading
(a) 1.78 m/s2 (b) 0.39 m/s2 decreases.
(c) 3.56 m/s 2 (d) 2.72 m/s2
5m/s
PASSAGE-5
t(sec)
Two blocks of mass m1 = 10 kg and m2 = 20 kg are placed on 0 1 2 3 4 5 5.5 6
a fixed inclined surface making an angle = 37° with 5m/s
horizontal. One end of a light spring of spring constant k =
100 N/m is free and other end is connected to a support S
rigidly fixed to inclined surface. The coefficient of friction 16. The man will not be in contact with weighing machine at
between block of mass m1 and inclined plane is µ1 = 0.5 and time
that between block of mass m2 and inclined plane is µ2 = 1. (a) t = 1s (b) t = 5.1s
At time t = 0 both blocks are released at rest from shown (c) t = 2.4s (d) t = 4.2s
position. (Take g = 10 m/s2) 17. What will be the maximum reading of the weighing machine?
(a) 90 kg (b) 100 kg
8m m2
=1.4 m1 (c) 6 kg (d) None of these
L
m 18. If the total mass of (elevator + man) system is 150 kg, the
N/
0 00 maximum tension in the cable supporting the lift will be
k =1
(a) 1500 N (b) 3000 N
S = 37° (c) 2250 N (d) None of these
/////////////////////////////////////////

MARK YOUR 10. 11. 12. 13. 14.


RESPONSE 15. 16. 17. 18.
LAWS OF MOTION 93

block reduces and velocity of lower block increases. This


PASSAGE-7
will continue till velocity of both the block get equal.

Two blocks A and B of equal masses m kg each are connected rough A


by a light thread, which passes over a massless pulley as 5 kg
shown. Both the blocks lie on wedge of mass m kg. Assume 5 kg B
friction to be absent everywhere and both the blocks to be //////////////////////////
Smooth
always in contact with the wedge. The wedge lying over
smooth horizontal surface is pulled towards right with 22. The net energy loss due to friction in the entire process is
constant acceleration a (m/s2) (g is acceleration due to (a) x + y (b) x – y (c) x (d) y
gravity) 23. Which curve shows the correct variation of velocity of each
block v/s time ?
v

A B
m m vA

37° 53° a
///////////////////////////////////////////////////// (a)
vB
19. Normal reaction (in N) acting on block B is t
m m
(a) (3g 4a) (b) (4 g 3a) v
5 5
m m vA
(c) (3g 4a) (d) (4 g 3a)
5 5
20. Normal reaction (in N) acting on block A is (b)
vB
m m
(a) (3g 4a) (b) (4 g 3a)
5 5 t

m m
(c) (3g 4a) (d) (4 g 3a) v
5 5
21. The maximum value of acceleration a (in m/s2) for which
normal reactions acting on the block A and block B are non vA
zero is (c)
3 3 5 4 vB
(a) g (b) g (c) g (d) g
4 5 3 3 t

v
PASSAGE-8
vA
Block A of 5 kg is placed on long block B also having 5 kg.
The coefficient of friction between two block is 0.4. There is
no friction between the ground and lower block. The upper (d)
block is given a sudden velocity of 10 m/s. The upper block vB
slides toward right, so friction force acting on it will be t
toward left. Its reaction will act on the lower block, which
will act forward. The friction force absorb x mechanical 24. After what time, both the blocks start moving together ?
energy from upper block but supply only y mechanical (a) 1 sec. (b) 2.25 sec.
energy to lower block. Due to friction the velocity of upper (c) 1.25 sec. (d) None of these

MARK YOUR 19. 20. 21. 22. 23.


RESPONSE 24.
94 IIT-JEE P HYSICS Challenger

PASSAGE-9 PASSAGE-10

A lift can move upward or downward. A light inextensible A body of mass m = 1.8 kg is placed on an inclined plane, the
string fixed from ceiling of lift with a frictionless pulley and angle of inclination is = 37°, and is attached to the top end
tension in string T1. Two masses of m1 and m2 are connected of the slope with a thread which is parallel to the slope.
with inextensible light string and tension in this string T2 as Then the slope is moved with a horizontal acceleration of a.
shown in the figure.
Friction is negligible.

T1

m a
T2
m1
///////////////////////////////////////////////
m2 28. The acceleration if the body pushes the slope with a force
3
of mg is
4
25. If m1 + m2 = m and lift is moving with constant velocity then
value of T1 5
(a) mg (b) = mg (c) mg (d) > mg (a) m / s2 (b) 0.5 m/s2
3
26. If m1 is very small as compared to m2 and lift is moving with
constant velocity then value of T2 is nearly 5
(a) m2g (b) 2m1g (c) 0.75 m/s2 (d) m / s2
6
(c) (m1 + m2) g (d) zero
27. If m1 = m2 and m1 is moving at a certain instant with velocity 29. The tension in thread is
v upward with respect to lift and the lift is moving in upward (a) 12 N (b) 10 N
direction with constant acceleration (a < g) then speed of (c) 8 N (d) 4 N
m1 with respect to lift 30. At what acceleration will the body lose contact with plane?
(a) increases
40
(b) decreases (a) m/s2 (b) 7.5 m/s2
(c) remains constant 3
(d) depend upon acceleration of lift (c) 10 m/s2 (d) 5 m/s2

MARK YOUR 25. 26. 27. 28. 29.


RESPONSE 30.

1. Statement - 1 : On a rainy day it is difficult to drive a car or 2. Statement - 1 : A rocket moves forward by pushing the
bus at high speed. surrounding air backwards.
Statement - 2 : The value of coefficient of friction is Statement - 2 : It derives the necessary thrust to move
lowered due to wetting of the surface. forward according to Newton’s third law
of motion.

MARK YOUR
1. 2.
RESPONSE
LAWS OF MOTION 95

3. Statement - 1 : The driver in a vehicle moving with a but his foot does not slip on the ground. There is no friction
constant speed on a straight road is an between the box and the ground, whereas there is sufficient
inertial frame of reference. friction between the man’s foot and ground to prevent him
Statement - 2 : A reference frame in which Newton's laws from slipping.
of motion are applicable is non-inertial. Statement - 1 : The force applied by the man on the box is
4. Statement - 1 : Use of ball bearings between two moving equal and opposite to the force applied by
parts of a machine is a common practice. the box on the man.
Statement - 2 : Ball bearings reduce vibrations and provide Statement - 2 : Friction force applied by the ground on
good stability. the man is 200 N.
5. Statement - 1 : A man in a closed cabin falling freely does 11. Statement - 1 : A uniform elastic rod lying on smooth
not experience gravity. horizontal surface is pulled by constant
Statement - 2 : Inertial and gravitational mass have horizontal force of magnitude F as shown
equivalence. in figure (i). Another identical elastic rod is
6. Statement - 1 : A cloth covers a table. Some dishes are pulled vertically upwards by a constant
kept on it. The cloth can be pulled out vertical force of magnitude F (figure ii). The
without dislodging the dishes from the extension in both rods will be same.
table.
because F
Statement - 2 : For every action there is an equal and
opposite reaction.
7. A solid sphere and a hollow sphere of same mass M and
same radius R are released from the top of a rough inclined F
/////////////////////////////////////////
plane. Friction coefficient is same for both the bodies. If
Fig. (i) Fig. (ii)
both bodies perform imperfect rolling, then
Statement - 1 : Work done by friction for the motion of
Statement - 2 : In a uniform elastic rod, the extension
bodies from top of incline to the bottom
depends only on forces acting at the ends
will be same for both the bodies.
of rod.
Statement - 2 : Force of friction will be same for both the
bodies. 12. Statement - 1 : Two blocks of mass m1 and m2 connected
by a massless spring are lying on smooth
8. Statement - 1 : Maximum value of friction force between
horizontal surface as shown. The block of
two surfaces is × normal reaction.
mass m1 is pulled to right by a horizontal
where = coefficient of friction between
force of magnitude F. When the spring is
surfaces.
compressed state, the acceleration of block
Statement - 2 : Friction force between surface of a body of mass m2 is towards left even if its
is always less than or equal to externally velocity is towards right.
applied force.
9. Statement - 1 : If we assume that there are only two m2 m1 F
/////////////////////////////////////////////////////
bodies, earth and sun, in the universe.
Their size, shape and motion remains same. smooth horizontal surface
A frame placed on sun is an inertial frame. Statement - 2 : A massless spring exerts force on a block
Statement - 2 : Inertial frame is non-accelerating in nature. fixed at one end of spring in direction
10. A man of mass 80 kg pushes a box of mass 20 kg horizontally. opposite to that of velocity of block as a
The man moves the box with a constant acceleration of 2m/s2 consequence of Newton’s third law.

MARK YOUR 3. 4. 5. 6. 7.
RESPONSE 8. 9. 10. 11. 12.
96 IIT-JEE P HYSICS Challenger
13. Statement - 1 : Two blocks of masses m1 and m2 are Statement - 2 : Internal forces with in a system does not
connected by a light extensible string depend on net force on the system
(having non zero tension) and this system because sum of all internal forces within a
lies on smooth horizontal surface. The system is zero.
block of mass m1 is pulled to right by
horizontal force of magnitude F as shown. m2 m1 F
/////////////////////////////////////////////////////
Then the value of m1a1 + m2a2 does not
depend on tension in the string connecting smooth horizontal surface
both the blocks, where a1 and a2 are the 14. Statement - 1 : Newton’s first law of motion can be derived
magnitudes of acceleration of blocks of using Newton’s second law.
masses m1 and m2 respectively. Statement - 2 : In inertial frame a = F/m hence a = 0 when
F = 0. (Symbols have usual meanings)

MARK YOUR
13. 14.
RESPONSE

1. A rope extends between two poles. A 90N boy hangs from (a) can be (m1 + m2)g
it, as shown in figure. Choose the correct option(s) (b) can be greater than (m1 + m2)g
(c) is equal to (m1g + m2g – T)
(d) can be less than (m1 + m2)g
3. Choose the correct option(s) for the figure shown.
10° 5°
T1 T2 ///////////////////////////////////////////////

//////////////////////////
70° 70°
T4 T5
T3
C B
30° 30°
T1 T2
A
(a) T1 = 346 N (b) T2 = 342 N
600N
(c) T2 = 322 N (d) T1 = 342 N
2. The mass of block is m1 and that of liquid with the vessel is (a) T1 = 346 N (b) T2 = 346 N
m2. The block is suspended by a string (tension T) partially (c) T3 = 877 N (d) T4 = 877 N & T5 = 651 N
in the liquid. The reading of the weighing machine placed 4. In the two cases shown, the coefficient of kinetic friction
below the vessel
between the block and the surface is the same, and both the
identical blocks are moving with the same uniform speed. If
///////////////
T sin q = mg / 4 F2 , then
F1 F2

(a) F1 = F2 (b) F1 < F2


(c) F1 > F2 (d) F1 = 2F2

MARK YOUR
1. 2. 3. 4.
RESPONSE
LAWS OF MOTION 97

5. Two blocks of mass M and m, are used to compress two 8. Two blocks of masses m1 = 2 kg and m2 = 4 kg over a
different massless springs as shown. The left spring is massless pulley as shown in the figure. The string
compressed by 3cm, while the right spring is compressed connecting both the blocks is light and inextensible. A force
by an unknown amount. The system is at rest, and all F0 = 100N acting at the axis of the pulley accelerates the
surfaces are fixed and smooth. Which of the following system upwards. Then (Take g = 10m/s2)
statements is/are true ? F0=100N

/////////////
k1=2N/cm k2=?
M m
//////////////////////////////////////////////
2 kg
(a) The force exerted on block of mass m by the right spring
is 6N to the left 4 kg
(b) The net force on block of mass m is zero (a) acceleration of both the masses is same
(c) The force exerted on block of mass m by the right spring (b) magnitude of acceleration of 2 kg mass is 15 m/sec2
is impossible to determine (c) magnitude of acceleration of 4 kg mass is 2.5 m/sec2
(d) The normal force exerted by block of mass m on block (d) acceleration of both the masses is upward
of mass M is 6N 9. Two uniform and equal ladders AB and AC, each of weight
6. A block of mass m is lying at rest on a rough horizontal W lean against each other and a string is tied between E and
surface having coefficient of kinetic and static friction µk F. They stand on a smooth horizontal surface. Then
and µs respectively. Now a constant horizontal force is A
applied on the block in the horizontal direction. Choose the
correct alternative(s).
(a) If F > µsmg, block will start moving and a constant
W
friction force of magnitude µs mg will act on the block
(b) If µkmg < F < µsmg, block will start moving and a E F
T T
constant friction force of magnitude µk mg will act on
B C
the block //////////////////////////////////////////////
(c) If F < µsmg, block will not start moving (a) the force exerted by one ladder on the other at A is
(d) If F > µsmg, block will move with a constant acceleration equal in magnitude to the tension T in the string
F (b) tension T = (W/2) cot
k mg
given by (c) the normal reaction at B and C are equal
m
(d) the normal reaction at B or C is greater than W
7. In the figure shown all the surface are smooth. All the blocks 10. All the blocks shown in the figure are at rest. The pulley is
A, B and C are movable, x-axis is horizontal and y-axis vertical smooth and the strings are light. Coefficient of friction at all
as shown. Just after the system is released from the position contacts is 0.2. A frictional force of 10N acts between A and
as shown. B. The block A is about to slide on block B. The normal reaction
y
and frictional force exerted by the ground on the block B is
A
A
B C x
5kg
//////////////////////////////////////
B
Horizontal surface
(a) acceleration of A relative to ground is in negative (a) the normal reaction exerted by the ground on the block
y-direction. B is 110N
(b) acceleration of A relative to B is in positive x-direction. (b) the normal reaction exerted by the ground on the block
(c) the horizontal acceleration of B relative to ground is in B is 50N
negative x-direction (c) the frictional force exerted by the ground on the block
(d) the acceleration of B relative to ground directed along B is 20N
the inclined surface of C is greater than g sin . (d) the frictional force exerted by the ground on the block
B is zero

MARK YOUR 5. 6. 7. 8. 9.
RESPONSE 10.
98 IIT-JEE P HYSICS Challenger
11. A horizontal force P is exerted on a 20 kg box in order to
slide it up a 30° incline. The friction force retarding the motion
A 30° 55° B
is 80N. Choose the correct option(s)
(a) If the acceleration of the moving box is to be zero then
P is 223N
(b) If the acceleration of the moving box is to be 0.75 m/s2 C
then P is 206 N
(c) If the acceleration of the moving box is to be zero then F
P is 206 N (a) the magnitude of the largest force F which may be
(d) If the acceleration of the moving box is to be 0.75 m/s2 applied at C is 1210 N
then P is 223 N (b) the value of is 57.5°
12. The system in figure is given an acceleration. Weight of the
(c) the magnitude of the largest force F which may be
ball is W. applied at C is 1510 N
(b) the value of is 37.5°
a
30° 15. In the figure, the blocks are of equal masses. The pulley is
fixed. In the position shown, A moves down with a speed
///////////////////////////////////////////// u, and vB = the speed of B.

(a) The force on the ball from vertical surface is 1.15 W P


(b) The force on the ball from inclined surface is B

a /////////////////////////////////
W 0.58 u
g A

(c) The force on the ball from vertical surface is (a) B will never lose contact with the ground
(b) The downward acceleration of A is equal in magnitude
a to the horizontal acceleration of B.
W 0.58
g (c) vB = u cos
(d) The forces on the ball from inclined surface is 1.15 W (d) vB = u/cos
13. Two blocks A and B of equal mass m are connected through 16. The block shown is acted on by its weight W = 400N, a
a massless string and arranged as shown in figure. The horizontal force F = 600N and the pressure P exerted by the
wedge is fixed on horizontal surface. Friction is absent inclined plane. The resultant R of these forces is parallel to
everywhere. When the system is released from rest the incline. Choose the correct option(s).

W = 400N

A ///
/////
///
Fixed ///
F = 600N // ///
//
30° B ///
/////
///
/ ///// P
// 45°
(a) tension in string is mg/2
// ///
/
(b) tension in string is mg/4 //// 30°
(c) acceleration of A is g/2
(d) acceleration of A is 3g/4
(a) P = 670 N
14. Two cables tied together at C are loaded as shown. Knowing (b) R = 146.2 N
that the maximum allowable tension is 1400N in cable AC (c) Block is moving upwards
and 700N in cable BC, choose the correct options (d) Block is moving downwards

MARK YOUR 11. 12. 13. 14. 15.


RESPONSE 16.
LAWS OF MOTION 99

17. A particle P of mass m is attached to a vertical axis by two (a) The acceleration of the system is 6.53 m/s2
strings AP and BP of length each. The separation AB = . (b) The tension in the two strings are 36.65 N and 13.32 N
P rotates around the axis with an angular velocity . The (c) The acceleration of the system is 3.53 m/s2
tensions in the two strings are T1 and T2. Then (d) The tension in the two strings are 16.65 N and 18.32 N
A 20. In the arrangement shown in the Fig, the ends P and Q of an
unstretchable string move downwards with uniform speed
P U. Pulleys A and B are fixed.

B
(a) T1 = T2 A B
(b) T1 + T2 = m 2
(c) T1 – T2 = 2mg
(d) BP will remain taut only if 2g /
18. Figure shows a beam AC supported by cable BC. Force at C
is 400N. Choose the correct option(s).
P Q
B

60° C M
30°
45°
400N Mass M moves upwards with a speed
A
(a) 2Ucos (b) U/ cos
(a) magnitude of force along AC is 107 N
(c) 2U / cos (d) U cos
(b) the tension in cable BC is 200 N
(c) magnitude of force along AC is 207 N 21. A reference frame attached to the earth
(d) the tension in cable BC is 400 N (a) is an inertial frame by definition.
19. Three bodies A, B and C are connected by inextensible light (b) cannot be an inertial frame because the earth is
strings as shown. The weights of these bodies are 40N, revolving round the sun.
50N and 150N respectively. The pulleys are frictionless. A is (c) is an inertial frame because Newton’s laws are applicable
placed on a horizontal plane while B is placed on an inclined
in this frame.
plane which is at an inclination of tan–1 (3/4) with the
horizontal. The coefficient of kinetic friction between bodies (d) cannot be an inertial frame because the earth is rotating
A and B and the planes are same equal to 0.25. Choose the about its own axis.
correct option(s). 22. A simple pendulum of length L and mass (bob) M is
a
B oscillating in a plane about a vertical line between angular
a T2 limit – and + . For an angular displacement (| |< ),
T1
A
the tension in the string and the velocity of the bob are T
WB=50N
and V respectively. The following relations hold good under
T1 T2
the above conditions :
WA=40N
(a) T cos = Mg

Mv 2
(b) T – Mg cos =
L
Wc=150N
(c) The magnitude of the tangenial acceleration of the bob
a
| aT | = g sin
(d) T = Mg cos

MARK YOUR 17. 18. 19. 20. 21.


RESPONSE 22.
100 IIT-JEE P HYSICS Challenger
23. A particle P is sliding down a frictionless hemispherical bowl. tP tQ
(a)
It passes the point A at t = 0. At this instant of time, the
horizontal component of its velocity is v. A bead Q of the (b) tP tQ
same mass as P is ejected from A at t = 0 along the horizontal
string AB, with the speed v. Friction between the bead and (c) tP tQ
the string may be neglected. Let tP and tQ be the respective
times taken by P and Q to reach the point B. Then : tP length of arc ACB
(d) tQ length of arc AB
Q
A B

P C

MARK YOUR
23.
RESPONSE

1. In the shown assembly on wheels, a force F is applied at point P as shown in the figure. If C denotes the centre of mass of the
whole assembly and N1 & N2 are the normal reaction as shown. Consider the following two cases.
Case I : Wheels are frictionless.
Case II : Wheels are jammed such that the van doesn’t move.
When F = 0, N1 = N10 and N2 = N 20

F
P

N1 N2

Column-I Column -II


(A) Case I, C lies above P (p) N1 N10

(B) Case II, C lies above P (q) N1 N10

(C) Case I, C lies below P (r) N2 N 20

(D) Case II, C lies below P (s) N2 N 20


LAWS OF MOTION 101

2. System of blocks are placed on a smooth horizontal system as shown. For a particular value of F, 3 kg block is just about to leave
ground. (tan 37° = 3/4)
String 2
String 1
5 kg 37° 3 kg 1 kg F

Column I Column II
(A) Tension in string 1 (p) 80 N
(B) Tension in string 2 (q) 64 N
(C) Net force by ground on 5 kg block (r) 50 N
(D) Net force on 3 kg block (s) 24 N
3. A block of mass 2 kg is placed on 4 kg block and force F is applied on the block of mass 2 kg as shown in the figure. Coefficient
of friction between any two surfaces is = 0.2, choose the correct choice if it is true for any block
F
2 kg

4 kg

Column I Column II
(A) Limiting friction force on block (p) 12 N
of 2 kg or 4 kg or ground is
(B) Net friction exerted by 4 kg (q) Zero
block on ground when F = 3 N
(C) Net friction force on a block, (r) 3 N
when F = 10 N can have the
values (s)
(D) Net friction exerted on a block, if (s) 4 N
F = 3 N can have the value(s)
4. A block A is placed on wedge B, which is placed on horizontal surface. All the contact surfaces are rough but friction is not
sufficient to prevent sliding at any surface. Match Column I and II. Column II indicates possible direction(s) of the physical
quantities mentioned under Column I. X and Y axes are along the incline and perpendicular to the incline.

Column I Column II
Vertical Y
s
(A) Acceleration of A (p)

(B) Net force applied by A on B (q)


Horizontal

(C) Acceleration of A relative to B (r)


p
(D) Net force applied by ground on B (s)
q
r
X

1. 2. 3. 4.

MARK YOUR
RESPONSE
102 IIT-JEE P HYSICS Challenger
5. Column II gives certain situations involving two blocks of mass 2 kg and 4 kg. The 4 kg block lies on a smooth horizontal table.
There is sufficient friction between both the blocks and there is no relative motion between both the blocks in all situations.
Horizontal forces act on one or both blocks are shown. Column I gives certain statement related to figures given in column II.
Match the statements in column I with the figure in column II.
Column I Column II
2kg
4kg 12N
(A) Magnitude of frictional force is maximum (p) \\\\\\\\\\\\\\\\\\\\\

2kg 12N
4kg
(B) Magnitude of frictional force is least (q) \\\\\\\\\\\\\\\\\\\\\

2kg 6N
4kg 6N
(C) Frictional force on 2 kg block is towards right (r) \\\\\\\\\\\\\\\\\\\\\

8N 2kg
20N 4kg
(s)
\\\\\\\\\\\\\\\\\\\\\
6. Three blocks of masses m1, m2 and M are arranged as shown in figure. All the surfaces are frictionless and string is inextensible.
A constant horizontal force of magnitude F is applied on block of mass m1 as shown. Pulleys and string are light. Part of the
string connecting both pulleys is vertical and part of the strings connecting pulleys with masses m1 and m2 are horizontal.

m2

M
m1 F
///////////////////////////////////////////////////////////////////
smooth horizontal surface

Column I Column II
F
(A) Acceleration of mass m1 (p)
m1

F
(B) Acceleration of mass m2 (q)
m1 m2
(C) Acceleration of mass M (r) zero
m2 F
(D) Tension in the string (s)
m1 m 2

5. 6.

MARK YOUR
RESPONSE
LAWS OF MOTION 103

7. Column II shows normal force acting on the block in each 8. Match the column to value of tension assuming mass of
of the equilibrium situations shown in column I. particle equal to m :
Column I Column II Column I Column II

///////////////
53°
4mg
20N (A) (p)
5
(A) 55° (p) 46 N
50N
//////////////////////////////////// Extreme position in simple pendulum

///////////////
37°
5mg
(B) (q)
4
N
60

Conical pendulum
(B) (q) 33.6 N
40°
53° a
//////////////////////////////////// 3mg
(C) (r)
5

Cart is accelerated such that


particle remains at rest as
N

70N shown w.r.t. cart


60

40°

(C) (r) 91 N
40° a = g sin 37° 5mg
//////////////////////////////////// (D) (s)
3
37°

Cart is accelerating freely on


the incline. Particle remains at
rest w.r.t. cart as shown.

7. 8.

MARK YOUR
RESPONSE
104 IIT-JEE P HYSICS Challenger
9. A small block lies on a rough horizontal platform above its centre C as shown in figure. The plank is moved in vertical plane such
that it always remains horizontal and its centre C moves in a vertical circle of centre O with constant angular velocity . There
is no relative motion between block and the plank and the block does not loose contact with the plank anywhere. P, Q, R and S
are four points on circular trajectory of centre C of platform. P and R lie on same horizontal level as O. Q is the highest point on
the circle and S is the lowest point on the shown circle. Match the statements in column I with points in column II.
Q
block
C plank
R P
O

S
Column I Column II
(A) Magnitude of frictional force on block is maximum (p) when block is at position P
(B) Magnitude of normal reaction on block is equal to mg (q) when block is at position Q
(C) Magnitude of frictional force is zero (r) when block is at position R
(D) Net contact force on the block is directed towards (s) when block is at position S
the centre
10. Two blocks of masses 20kg and 10kg are kept on a rough horizontal floor. The coefficient of friction between both blocks and
floor is µ = 0.2. The surface of contact of both blocks are smooth. Horizontal forces of magnitude 20N and 60N are applied on
both the blocks as shown in figure. Match the statements in column I with the statements in column II.
F1=20N
20kg 10kg F2=60N
left ////////////////////////////////////
µ=0.2 right
rough horizontal floor
Column I Column II
(A) Frictional forces acting on block of mass 10 kg (p) has magnitude 20N
(B) Frictional forces acting on block of mass 20 kg (q) has magnitude 40N
(C) Normal reaction exerted by 20kg block on 10 kg block (r) is zero
(D) Net force on system consisting of 10kg block and (s) is towards right (in horizontal direction)
20kg block
11. Match the columns – (g = 10 m/s2)
Column I Column II
(A) Block of mass 2 kg on a rough horizontal surface (p) Tension at the mid point of block is 10N
pulled by a horizontal force of 20N, µs = 0.5
(B) Block of mass 2 kg pulled with constant speed up an (q) Acceleration of block is 5 m/s2
incline of inclination 30° and coefficient of friction 1 / 3
(C) Block of mass 0.75kg pulled by a constant force of (r) Force of friction acting is 5N
7.5N upon incline of inclination 30° and coefficient of
friction 1 / 3
(D) Block of mass 2 kg pulled vertically by a force 20N (s) Resultant force on the block is zero
(t) Force of friction is 10 N

9. 10. 11.

MARK YOUR
RESPONSE
LAWS OF MOTION 105

12. A bicycle moves forward and assume that the rider is not braking and pedaling at the same time. Match the columns regarding
friction on the wheels of bicycle.
Column I Column II
(A) When rear brakes are applied on a bicycle and (p) Friction force will be forward on both wheels
it does not slide
(B) When front brakes are applied on a bicycle and (q) Friction force will be backward on both wheels
it does not slide
(C) When front and rear brakes are applied (r) Friction force will be forward on rear wheel and
simultaneously so hard that both wheels stop backward on front wheel
rotating
(D) When force on pedals is applied to accelerate (s) Friction force will be backward on rear wheel and
bicycle without sliding forward on front wheel

12.

MARK YOUR
RESPONSE

1. A wire of mass 9.8 × 10–3 kg per metre passes over a 2. A block slides down a smooth inclined plane to the ground
frictionless pulley fixed on the top of an inclined frictionless when released at the top, in time t seconds. Another block is
plane which makes an angle of 30º with the horizontal. dropped vertically from the same point, in the absence of
Masses M1 and M2 are tied at the two ends of the wire. The the inclined plane and reaches the ground in t/2 second.
mass M1 rests on the plane and mass M2 hangs freely Then find the angle (in degree) of inclination of the plane
vertically downwards. The whole system is in equilibrium. with the vertical.
Now a transverse wave propagates along the wire with a
velocity of 100 ms–1. Find the value of (M1 + M2) in kg.

1. 2.

MARK
YOUR
RESPONSE
106 IIT-JEE P HYSICS Challenger
3. The masses of 10 kg and 20 kg respectively are connected 1
2ms . The friction coefficient between road and bigger
by a massless spring in fig. A force of 200 newton acts on
the 20 kg mass. At the instant shown, the 10 kg mass has block is µ ( = 0.3) and between the blocks is µ/2. Find the
acceleration 12 m/sec2. What is the acceleration (in m/s2) of time (in second) elapsed before the smaller block separates
20 kg mass? from the bigger block.

m
20 kg M
10 kg
200 newton
6. In the figure masses m1, m2 and M are 20 kg, 5 kg and 50 kg
respectively. The coefficient of friction between M and ground
4. is zero. The coefficient of friction between m1 and M and that
between m2 and ground is 0.3. The pulleys and the strings are
T massless. The string is perfectly horizontal between P1 and
F A m1 and also between P2 and m2. The string is perfectly vertical
f2 f1 between P1 and P2. An external horizontal force F is applied
B to the mass M. Take g = 10 m/s2.
T
f3

P1 m1
A block A of mass m1(= 10 kg) rests on a block B of mass
m2 (=20 kg). B rests on fixed surface. The coefficient of friction
between any two surfaces is =0.3 . A and B are F
P2
connected by a massless string passing around a frictionless
m2
pulley fixed to the wall as shown in fig. With what force
should A be dragged so as to keep both A and B moving
with uniform speed?
5. Figure shows a small block of mass m (= 10 kg) kept at the Let the magnitude of the force of friction between m1 and M
left hand of a larger block of mass M (= 20 kg) and length be f1 and that between m2 and ground be f2. For a particular
L (= 4 cm). The system can slide on a horizontal road. The F it is found that f1 = 2f2. Find f1 f 2 in newton.
system is started towards right with an initial velocity v

3. 4. 5. 6.

MARK
YOUR
RESPONSE
LAWS OF MOTION 107

7. Two blocks A and B of equal masses are placed on rough Find the acceleration (in m/s2) of the block with respect to
inclined plane as shown in figure. When will the two blocks
4 3
come on the same line on the inclined plane if they are disc. (Given cos , sin .)
5 5
released simultaneously? Initially the block

2m
2
A 25 m/s

B
A

B
45º 9. In the arrangement shown in figure mA = 1 kg and mB = 2 kg,
while all the pulleys and strings are massless and frictionless.
At t = 0, a force F = 10 t starts acting over central pulley in
A is 2 m behind the block B. Coefficient of kinetic vertically upward direction. Find the velocity of A (in m/s)
friction for the blocks A and B are 0.2 and 0.3 respectively when B loses contact with floor.
(g =10 m/s2).
8. A circular disc with a groove along its diameter is placed F=10t
horizontally on a rough surface. A block of mass 1 kg is placed
as shown. The co-efficient of friction between the block and
all surfaces of groove and horizontal surface in contact is

2
= . The disc has an acceleration of 25 m/s2 towards left.
5
A B

7. 8. 9.

MARK
YOUR
RESPONSE
108 IIT-JEE P HYSICS Challenger

1 (c) 8 (a) 15 (c) 22 (c) 29 (a) 36 (d) 43 (a) 50 (d) 57 (b) 64 (a) 71 (d)
2 (c) 9 (d) 16 (d) 23 (a) 30 (b) 37 (c) 44 (c) 51 (d) 58 (b) 65 (c) 72 (b)
3 (a) 10 (a) 17 (a) 24 (c) 31 (c) 38 (a) 45 (a) 52 (c) 59 (d) 66 (d) 73 (a)
4 (c) 11 (b) 18 (d) 25 (c) 32 (a) 39 (b) 46 (a) 53 (b) 60 (c) 67 (d) 74 (a)
5 (b) 12 (c) 19 (b) 26 (d) 33 (a) 40 (b) 47 (a) 54 (b) 61 (a) 68 (a)
6 (c) 13 (a) 20 (c) 27 (d) 34 (a) 41 (a) 48 (b) 55 (c) 62 (b) 69 (c)
7 (a) 14 (c) 21 (b) 28 (b) 35 (a) 42 (b) 49 (d) 56 (a) 63 (a) 70 (b)

1 (c) 4 (a) 7 (a) 10 (a) 13 (c) 16 (b) 19 (c) 22 (b) 25 (c) 28 (d)
2 (c) 5 (b) 8 (b) 11 (b) 14 (a) 17 (a) 20 (b) 23 (a) 26 (b) 29 (a)
3 (a) 6 (b) 9 (a) 12 (a) 15 (a) 18 (c) 21 (d) 24 (c) 27 (c) 30 (a)

1 (a) 3 (c) 5 (a) 7 (d) 9 (d) 11 (c) 13 (c)


2 (a) 4 (c) 6 (b) 8 (c) 10 (b) 12 (c) 14 (d)

1 (a, b) 4 (c, d) 7 (a, b, c, d) 10 (a, d) 13 (b, d) 16 (a, b, c) 19 (a, b) 22 (b, c)


2 (a, c, d) 5 (a, b, d) 8 (b, c, d) 11 (c, d) 14 (b, c) 17 (c, d) 20 (b) 23 (a)
3 (a, b, c, d) 6 (c, d) 9 (a, c) 12 (c, d) 15 (a, d) 18 (c, d) 21 (b, d)

1. A - p, s; B - q, r; C - q, r; D - q, r 2. A - q; B - r; C - p; D - s
3. A - p, s; B - r; C - q, s; D - q, r 4. A - q, r; B - r, q; C - p; D - s
5. A - s; B - r; C - p, s 6. A - q; B - q; C - r; D - s
7. A - q; B - p; C - r 8. A - r; B - q; C - s; D - p
9. A - p, r; B - p, r ; C - q, s ; D - q, s 10. A - p, s; B - p, s; C - q, s; D - r
11. A - p, q, t; B - s; C - s; D - p, s 12. A - s; B - r; C - q; D - r

1 30 2 60 3 4 4 150 5 0.12 6 15 7 2 8 10 9 10
LAWS OF MOTION 109

F cos 60° = µ [F sin 60° + mg]


1. (c)
mg
F=
cos 60 sin 60
T
ma m a 1
3 10
a 2 3 5
= 20 N
mg 1 1 3 1
2 2 3 2 4
(Force diagram in the frame of the car)
Applying Newton’s law perpendicular to string
mg sin ma cos 4. (c) Blocks A and C both move due to friction. But less
friction is available to A as compared to C because
a normal reaction between A and B is less. Maximum
tan
g friction between A and B can be :
Applying Newton’s law along string 1
f max m Ag mg
T m g2 a2 ma 2
Maximum acceleration of A can be
or T m g 2 a 2 ma
f max g
2. (c) As m would slip in vertically downward direction, then amax
mg = µN m 2

mg 100 mD g
N 200 Newton Further, amax
0.5 3m mD
Same normal force would accelerated M,
g mD g
200 or
thus aM 4 m / s2 2 3m mD
50
Taking m + M as system 5. (b) For uniform speed of the blocks
F = (m + M) 4 = 240 N T = 6g ...(i)
3. (a) The forces acting on the block are shown. Since the T = f = µN ...(ii)
block is not moving forward for the maximum force F µN = 6g
applied, therefore or 0.4 (4 + m)g = 6g
F cos 60° = f = µN ... (i) (Horizontal Direction) m = 15 – 4 = 11 kg
For maximum force F, the frictional force is the
limiting friction = µN]
N
and F sin 60° + mg = N... (ii)
From (i) and (ii) m
T
N 4 kg
f

F cos 60o (4 + m) g
o
T
mg 60
f
6 kg

F 6g
Fsin60o
110 IIT-JEE P HYSICS Challenger
6. (c) F = ma 9. (d) C
1 2 3
4
F 5 10 5 0
a 10 3 ms 2 1 2 3
m 3 107 3
Also, v2 – u2 = 2as 2m l1 v B 4m
vA
5 3 2 F A P1 T
l 3T B
v2 – 0 2 = 2 10 3 10 l2 P2
3
v = 0.1 ms–1
vB vA vB vA vB 0
7. (a) Here is the angle of repose.
µ = tan 3vB = 2vA
3aB = 2aA
Applying Newton’s law on A and B
s in
µN mg F – 2T = 2mA
3T = 4maB
3F = (6aA + 8aB) m
The retardation of the block
F N mg sin 3F
a= 3F = (9 + 8) aBm aB
m m 17 m
10. (a) The forces acting on the masses are shown.
tan mg cos mg sin
= = 2g sin Applying Newton's second law on mass Q, we get
m
F – f = ma ... (i)
Let s is the distance travelled, then by third equation of
where a is the acceleration at the extreme position. Now
motion
applying Newton's second law on mass P
2
v2 = u0 – 2 a s f = ma ... (ii)
2 [acceleration is same as no slipping occurs between Q
or 0 = u0 – 2 × 2g sin × s and P]
u02 From (i) and (ii)
or s=
4g sin a
8. (a) Q
F m
N N f
f f m
P
in sin A
m gs mg
(Mean position) (Extreme
position)
mg mg cos mg mg cos
Smooth surface Rough surface F kA
F = 2ma a= [ F = kA]
2m 2m
1
For smooth surface, s g sin t12 ...(i) Substituting this value of a in eq. (ii), we get
2
kA kA
For rough surface, a = g (sin – cos ) t 22 f=m×
2m 2
1 Alternatively,
s g(sin cos )t22 ...(ii)
2 Let be the angular frequency of the system.
From (i) and (ii), The maximum acceleration of the system,
1 1
s g sin t12 g(sin cos )t22 2 k k k
2 2 a A A m m 2 m
2m
Given, = 45° t12 (1 )t22 The force of friction provides this acceleration.
Also, given that, t2 = nt1 t12 (1 )n 2 t12 f = ma
kA kA
1 1 m
1 1 2m 2
n2 n2
LAWS OF MOTION 111

11. (b) 1 2 3 4 C m m
14. (c) g –T a ....(i)
d 1 d 2 d 3 d 4 2 2
0
dt dt dt dt ma
–v–v+0+v+2=0 T cos 60 ...(ii)
cos 60
v = 2 m/s
Solving (i) and (ii)
2g
acceleration of ring
9
15. (c) Mg = n m v (1 + e)
16. (d) µ m g = m 2b t
µg
t
v 2b
17. (a) For no slipping between m and M,
2m/s F (M + m) g/3
F 40 N
12. (c) FBD in reference frame of the lift For no toppling of m block
F (M + m) g/4
F 30 N
2 Fmin = 30N
.2(g+a) 18. (d) Perpendicular to surface, their acceleration components
.2(g+a) must be equal.
F 8 aA cos aB sin

21 a A 2aB
1 g g
a2 g 2.5 m / s 2
5 4 4 aAcos aA

0.5
g 2
30 .2 g aA
4 90° –
a8 21
8
1 50
30 2 aAsin
5 4 25
m / s2
8 8
19. (b) y
13. (a) a1 a2
a x
A x1 x2 B

m m x2 = 4a (a) [since y = a]
x2 = 4a2
x = ± 2a
The slope of curve at point (2a, a)
2x 2
m = tan 2a 1
4a 4a
C m
g
so, tangential acceleration = g sin
x = 3x1 + 4x2 2
a = 3a1 + 4a2 ... (1) 20. (c) a = b + c
3T = ma1 ... (2)
4T = ma2 ... (3) Net acceleration of A = a2 c2 2ac cos ( )
mg – T = ma ... (4)
From (1), (2), (3) & (4) = (b c)2 c2 2 (b c).c.cos
a = 2g /13
112 IIT-JEE P HYSICS Challenger
21. (b) FBD of C kx = mg ... (i)
a g tan
FBD of B T = 2 mg ... (ii) satisfy tan
FBD of A kx = mg + T ... (iii) g a tan
kx = 3 mg tan tan
or a g g tan ( )
1 tan tan
kx If a > g tan ( – ) the block will slide.
25. (c) Since impulse would have been same
F dt = constant and as time interval increases.
T – mg average force decreases.
T 26. (d) When all are pulling
B
Fnet 100 3iˆ ......... (1)
When A stops
mg + kx kx
Fnet FA 100 1 ( iˆ) ......... (2)
When B stops
C
Fnet FB 100 24 ˆj ......... (3)
From these three we get
mg
When string is out T = 0 FA FB 100 7iˆ 24 ˆj
kx – mg = maA
n 2
3 mg – mg = maA aA = 2 g Required acc 72 24
For B
T – mg – kx = maB 25 m/s2
– 2 mg = maB 27. (d) First find the tension in the cords below the cord in
aB = – 2g question by balancing forces at the lower junction in
22. (c) The component of the velocity of the block C along figure: 2T cos 30° = 70, so T = 40.4 N.
the slanted segment of the string equals the velocity T T
of the block B downwards, i.e., u cos = v B . 30° 30°
Differentiating this,
d dvB
u sin
dt dt
Putting the given values,
dvB 70 N
u sin 30 1
dt Equilibrium conditions for junction B (figure) are
T' cos 40° = TAB + T sin 30°.
u T' sin 40° = T cos 30°.
10 u 20 m/s
2 Solving above equations, TAB = 21.5 N
23. (a) Where the three ropes join, T1 sin 1 = T2 sin 2,
so T1 = T2. T'
Also, T2 = T3 and W2 = T3. Further, the equilibrium
condition for the vertical direction is W1 = 2T1 cos 1.
W1
Therefore, T1 = T2 = T3 = W2 = 40°
2 cos 1 TAB B
24. (c) If the block is not to slide, it must have the same
30°
acceleration as the plane.
Hence, f cos – N sin = ma
f sin + N cos – mg = 0 T
From these,
f = m (a cos + g sin ) 28. (b) First draw a force diagram.
N = m (g cos – a sin ) F
N F sin
f a cos g sin a g tan
and =30°
N g cos a sin g a tan Fcos
Now the maximum value of f/N in the absence of f
slipping is µs = tan . Thus the acceleration a must
W
LAWS OF MOTION 113

Next, consider the forces in the x-direction and apply and T – mg = ma .......... (2)
the conditions for equilibrium, noting f equals its cos = 4/5 and from constraint we get
maximum value to start motion. a = A cos .......... (3)
Fx = 0 , F cos – f = 0
F cos = f ; 0.0866 F = f = µsN = 0.4 N T 2T cos
Now apply the conditions for equilibrium to the forces
in y-direction
Fy = 0 , N + F sin – W = 0 m 4m A
a
N + 0.5 F – 100 = 0
N = 100 – 0.5 F
Substituting this equation for N in
0.866 F = 0.4 N above, mg 4mg
0.866 F = 0.4 (100 – 0.5 F) Solving eq. (1), (2) and (3)
0.866 F + 0.2 F = 40 we get acceleration of block of mass 4m,
F = 37.5 N
5g
29. (a) A= downwards.
11
N 1 Tsin 45° 32. (a) Acceleration of bodies of masses m1 and m2 are g sin 1
T and g sin 2 respectively down the plane.
45°
m1 Tcos 45°
N2
0.25N 2
N1 m2
W1 m1
For m1 : g cos g sin 2
g cos 2
N1 + T sin 45° = 200 .......... (1) 1 1 2
g sin 1
T cos 45° = 0.25 N1 .......... (2)
g
T 40 2 N
30. (b) Before the string A is cut : Let x be elongation in the
spring. As system is in equilibrium. Then for lower
block, m2
m1

A h1 h2 x2
x1 2
1
2kg
g
kx
k Time taken by body of mass m1 is given by
2 x1 2 h1cosec 1
2kg t1
a1 g sin 1

mg 2 1 cosec60
10×sin 60°
kx = mg = 20N
Just after the string A is cut : For upper block, 2 2 2 8
ma = kx + mg 10× 3 3 30
2a = 20 + 20
2kg a 4
0.51 sec.
15
mg Time taken by body of mass m2 is given by
kx
2 x2 2 h2cosec 2
2kg t2
a2 g sin 2
a = 20 m/s2
31. (c) The FBD of blocks is as shown 2 1 cosec30
From Newton’s second law 10×sin 30°
4mg – 2T cos = 4mA .......... (1)
114 IIT-JEE PHYSICS Challenger
get a1 = (g/2) (1 – µk), from the second equation, a2 =
2 2 2
= = 0.89 sec. (g/4)µk. Then the displacements of blocks 1 and 2 are
10 1 given by
It is clear that the body with mass m1 will reach the
1 2 g g
bottom of wedge first. x= at , i.e., x1 (1 k ) t 2 , x2 kt
2

Alternatively, 2 4 8
The vertical component of acceleration of mass 1 and At the instant that one-fourth of block 1 remains on
mass 2 are block 2, x2 + = x1 + ( /16), where is the length of
a1 = g sin2 60°, a2 = g sin2 30° g g
Since vertical displacement for both masses is 1m, the block 2. Therefore, kt
2
+ = (1 k)t
2
+
8 4 16
block with larger acceleration will reach the base of
wedge first. Hence block of mass m1 shall reach base 15
or t 2 =
of wedge first. 2 g (2 3 k)
33. (a) From the force diagram figure,
g 15 15 k
and x2 k÷ =
T 40g W 8 2 g (2 3 k) 16 (2 3 k)
N
1
Ty Wy =
45° 30° 7.47
Tx Fy Wx 35. (a) vB = vA cos .
//////// ////////
vB
300g
vAcos
Tx = (40 cos 45°) = 28.3g vA
vB
Ty = (40 sin 45°) = 28.3g
Fy = 0 , Ty + Wy + N – 300g = 0 vB A
28.3g + W sin 30° + N = 300g
N = 300g – 28.3g – 0.5W
vB
Fx = 0 , Wx – Tx – µsN = 0
36. (d) At equilibrium T = Mg
µs = 0.3
W cos 30° – 28.3g – 0.3 N = 0
Substitute N = 271.7g – 0.5 W T
Solving for W gives W = 108kg
34. (a) From figure, the equations of motion are
F1 = T – µkW1 = ma1 T
mg T
F2 = µkW1 = 4ma2
F3 = W1 – T = ma1

a1 Mg

1 F.B.D. of pulley
f1=µk W1 T T=Mg
W1 R1=W1

a2 F
T
f2=µk W2 F1=(m+M)g
2 3
F1 = (m + M) g
a3 = a1 The resultant force on pulley is

W1 F= F12 + T 2
4W1 R = 5W
2 1

F = [ (m + M ) + M ] g
2 2
Solve the first and third equations simultaneously to
LAWS OF MOTION 115

37. (c) Acceleration of mass at distance x


3 3 4 3
a = g (sin – µ0 x cos ) 2 Mg mg mg or M m
Speed is maximum, when a = 0 5 4 5 5
g (sin – µ0 x cos ) = 0 42. (b) Let L1 and L2 be the portions (of length) of rope on left
and right surface of wedge as shown.
tan
x= Magnitude of acceleration of rope
0
M
38. (a) NAB sin = mg ( L sin L2 sin ) g
a L 1 0
4 M
N AB 100N N AB 125N
5
39. (b) At time t, let y be the height (in meters) of the body ( L1 sin L2 sin )
above its initial position. The pull in the chain is 43. (a) For the maximum possible value of ,
then mg sin will also be maximum.
T = (6000 – 360y) g and Newton’s second law gives In this case f is the limiting friction. The two forces
acting on the insect are mg and N. Let us resolve mg
d2y
T – 400g = 400 into two components.
dt 2 mg cos balances N.
d2y mg sin is balanced by the frictional force.
or (5600 – 360y)g = 400
dt 2
=1/3
dy
The equation may be changed into one for v
dt f N
(the velocity of the body) by use of the identity
mgcos
d2y dv dv dy dv d (v 2 )
2 2 2 2v mg mgsin
dt 2 dt dy dt dy dy

d (v 2 ) N = mg cos
Thus 200 = (5600 – 360y) g f = mg sin But f = µN = µ mg cos
dy
µ mg cos = mg sin
or d (v2) = g (28 – 1.8y) dy
Let v be the velocity at height 10m. 1
Then, on integrating cot =
µ
v2 10 cot =3
d (v 2 ) g (28 1.8 y ) dy
44. (c) The impulse is I F t . The impulse momentum
0 0
relation is shown in figure.
v 2 g [28 y 0.9 y 2 ]10 0 From the law of cosines,
= g [28 (10) – 0.9 (100)] = 190g I2 pi2 p 2f 2 pi p f cos120
v 190g 43.2 m/s = [(0.11) (17)]2 + [(0.11) (34)]2
The choice of the +sign for v (upward motion) should – 2 [(0.11) (17)] [(0.11) (34)] (–0.5)
be checked. For 0 y 10, the net force, (5600 – 360y) p I
f
g, is positive, and so the acceleration is positive. Then,
since the body started from rest, v must be positive.
40. (b) String is light so speed of A = speed of B. I = pf – pi
41. (a) As long as the block of mass m remains stationary, the
block of mass M released from rest comes down by
2Mg 120°
(before coming it rest momentarily again). Thus
k
the maximum extension in the spring is pi
2Mg I = (0.11) (17) ( 7 ) = 4.974 Ns
x .......... (1)
k
For block of mass m to just move up the incline I 4.947
and F = 197.90 N
kx = mg sin + µ mg cos .......... (2) t 0.025
116 IIT-JEE P HYSICS Challenger
45. (a) Tension in both string shall be same which can be 53. (b) The acceleration of mass m is due to the force T cos
observed by making FBD of string in figure (i) T cos = ma
T T cos
T T
(i) (ii) a= ... (i)
m
FBD of string in figure (i)
/////////////////////////////////////////
also,F = 2T sin
46. (a) T = T1 + T2 = m1 (g + a) + m2g
= 10 (10 + 2) + 8 (10) = 120 + 80 = 200N F
T= ... (ii)
47. (a) Since the body presses the surface with a force N hence 2sin
according to Newton's third law the surface presses
the body with a force N. The other force acting on the F
body is its weight mg.
N T T
a
Tsin Tsin a
N T
Tcos
mg
x x
For circular motion to take place, a centripetal force is
required which is provided by (mg + N). From (i) and (ii)
2
mv F cos F F x
mg + N = a=
r 2sin m 2m tan 2m a 2 x 2
where r is the radius of curvature at the top.
If the surface is smooth then on applying conservation
of mechanical energy, the velocity of the body is always a2 x2
tan
same at the top most point. Hence, N and r have inverse x
relationship. From the figure it is clear that r is minimum
for first figure, therefore N will be maximum. 54. (b) 4TXA = TXB ; 4vA = vB ; vA = v/4
If we do not assume the surface to be smooth, we 1 u v u v
cannot reach to a conclusion. 55. (c)
2 cos cos 2 cos
48. (b) T2 sin + T1 sin 60° = W
56. (a) T cos – f = ma
T1 cos 60° = T2 cos
f = µN
T2 sin + T2 cos . tan 60° = W
T sin + N = mg
T2 (sin 3 cos ) W 57. (b) Let m1 + m2 = m = constant and
2T2 (sin 30° sin + cos 30° cos ) = W m1 = x m2 = m – x
cos (30° – ) is maximum x (m x)
a g
If 30° – = 0 ; = 30° m
49. (d) No relative motion no force.
2 x (m x) m 2 2
T g T (g a )
Mg Mg k k m 2g
50. (d) Total stretch = Mg 1 2
k1 k2 k1k2 58. (b)

µN f N
/////////////////////

N=10
51. (d)
10N
mg
W
W = µN ; W = 0.2 (10) ; W = 2 Total force f N ; f = mg sin ; N = mg cos
52. (c) For constant speed ext. force = frictional force (max.)
Total force = mg (cos ) 2 (mg sin ) 2 = mg = 10N
Fext = Fmax = µR
LAWS OF MOTION 117

Fx = 0, NAB cos 30° – NA – FAB sin 30° = 0


59. (d) p (t ) A [i cos (kt ) j sin (kt )]
N AB 3 FAB
dp 688 s 0
F Ak [ i sin (kt ) j cos (kt )] 2 2
dt
FAB
F .P 0 Also s
N AB (by definition)
F . p Fp cos Using (2) and (3), we get

But F . p 0 cos =0 3 N AB FAB 344 s

= 90°. N AB 3FAB 99
60. (c) Given : mA = 10 kg, WA = mAg = 98 N, mB = 50 kg
WB = 50 × 9.8 = 490 N 3 s 344 s
or 99
µs = 0 between wall and block A 1 3 s
P = 100 N (vertically downward)
or 344 3 2
FBD of system and FBD of block A are shown in figure: s 344 s 99 3 99 s
Since slip is impending, we can use F = µsN 2
From FBD of combined system : or 595.8 s 443 s 171.5 0

443 [(443)2 4 (595.8) ( 171.5)]1/ 2


s
P =100N 2 595.8
y
443 874 431
+ = (taking positive sign)
x NA 1191.6 1191.6
WA
µs = 0.362
61. (a) The acrobat has a force acting on his hand that we
B
resolve into two perpendicular components: the vertical
one is the reaction to the weight (115 × 9.8 N = 1127 N)
WB 60° and the horizontal one balances the 130 N force from
FB=µsNB the wall. These two forces give a resultant force F of
NB
F 11272 1302 1134 N

Fy = 0 : NB – P – WA – WB = 0 dy x
62. (b) tan ; µN cos = N sin .
NB = 100 + 98 + 490 = 688 N dx 10
Fx = 0 : NA – FB = 0
NA = FB = µsNB 1 25
tan = µ = x = 5m y 1.25m
= 688 µs ........... (1) 2 20
From FBD of block A : 63. (a) Tension will balance the weight.
Equilibrium equations yield Fy = 0
64. (a)
FAB cos 30° + NAB sin 30° – P – WA = 0
where FAB is the frictional force between blocks A and
B. v

P =100N

FAB NA dv
30° Fthrust + Fext = m ; Fext = 0
dt
WA
NAB dm
Fthrust = urel × and
dt
3 N AB As urel = 0 Fthrust = 0
FAB 100 98 0
2 2 mdv
0
dt
3FAB N AB constant velocity.
198 ........... (2)
2 2
118 IIT-JEE P HYSICS Challenger
65. (c) Just before the string is cut by equilibrium of mass m,
B
T ' = mg ... (i) BH
By equilibrium of mass 2m, T = 2mg + T' ... (ii) L
From (i) and (ii), T = 2mg + mg = 3mg ... (iii) A BV
200N
AH AV
Applying equilibrium equations,
Av + Bv = 200 N ....... (1)
AH = BH ....... (2)
From FBD of block B,
BV
T T
2m 2m
2mg
T' B
2mg BH 60°
T'
m m NB
60°
mg mg FB
300N
Situation 1 Situation 2 BH + FB cos 60° – NB sin 60° = 0
NB cos 60° – BV – 300 + FB sin 60° = 0
FB = 0.25 NB
When the string is cut : BH – 0.74 NB = 0 ....... (3)
For mass m : – BV + 0.71 NB = 300 ....... (4)
Fnet = mam mg = mam am = g (downwards) FBD of block A
For mass 2m : AV
Fnet = 2ma2m
2mg – T = 2ma2m A AH
g
2mg – 3mg = 2ma2m a2m = FA
2
400N
The negative sign indicates that the acceleration is in NA
upwards direction. FA – A H = 0
Alternatively, NA – AV = 400 ....... (5)
FA = µA NA
In situation 1, the tension T has to hold both the masses
µANA – AH = 0 ....... (6)
2m and m therefore,
On solving above equations, we get
T = 3mg NA = 650 N, FA = 260N, FA = µANA
In situation 2, when the string is cut, the mass m is a
260
freely falling body and its acceleration due to gravity µA = 0.4
is g. 250
For mass 2m, just after the string is cut, T remains 3mg 69. (c) At t = 0, fr = mg sin
because of the extension of string (or otherwise it would when kt = mg sin , fr = 0
quickly changed to 2mg). mg sin mg sin mg cos
3mg – 2mg = 2m × a t
k k
g Frictional force increases,
a
2 mg sin mg cos
t
66. (d) Since the ball is moving upwards at a constant velocity, k
the net force acting on it is zero. The only forces are fr = µ mg cos = constant
gravity and the force of the rope. These must be equal 70. (b) T1 cos = T2 cos 45°
and opposite. T2 cos 45° = T3 cos 45°
67. (d) This is an example of Newton's Third Law of Motion. T1 sin = T2 sin 45° + mg
68. (a) Consider FBD of structure. T2 sin 45° + T3 sin 45° = Mg
LAWS OF MOTION 119

Also we can write,


P = R1 sin 25° + R3 sin 10°

////////
= 551.7 × sin 25° + 507.72 × sin 10°
= 321.32 N
T1 72. (b).

y NB
/////////

45°
T3 T2 NA FBA
m
45° 45°
T A B P
x

FAB=µ.NA WA
NB WA FBC
M WB
FBD of block A FBD of block B
Mg
T2 = T3 = The FBD of block A and block B shown in figure. Taking
2
y
Mg
T1 cos = , sign convention for force system as +
2 x
O
Mg 2m Consider FBD of block A
T1 sin = mg tan 1
2 M The equilibrium equation for block A yields :
71. (d) Consider FBD of block. Fx = 0; T – µ NA = 0
We can write from force polygon as follows : or T = µ NA = 0.3 × NA .......... (1)
Consider R1 as resultant of F1, N1 and R2 as resultant Fy = 0; NA – WA = 0
of F2 and N2, then, NA = WA = MAg
1000 N NA = 29.4 N .......... (2)
T = 8.82 N
Considering FBD of block B :
Equilibrium equation for block B yields :
Fx = 0; P – FAB – FBC = 0
or P – µ.NA – µ NB = 0
F2 P = µ (NA + NB)
F1 P = 0.3 (29.4 + NB) .......... (3)
N2 Fy = 0; NB = NA + WB
N1 NB = WA + WB
NB = 29.4 + 5 × 9.8
R1 1000 NB = 78.4 N .......... (4)
sin 25 sin130 Putting value of NB = 78.4 N in (3) we get
R1 = R2 = 551.7 N P = 0.3 (29.4 + 78.4) N = 32.34 N
Consider FBD of wedge, F3 = µN3, 73. (a) For the impending motion, block A must slip up and
Resultant of F3 & N3 is R3. block C down the inclined plane. Since the normal force
between A and B is less than that between block B and
R1 C, the maximum frictional force (limiting friction) will be
reached first between A and B while B and C will stay
P together.
From FBD of block A :
x
y WA

F3 30°
R3
N3 A

Applying equilibrium equations we get


NA
R1 cos 25° = R3 cos 10°
R3 = 507.72 N FAB
120 IIT-JEE P HYSICS Challenger
Writing equilibrium equations :
FC s .NC 75 3 s ......... (3)
Fy = 0 ;
NA – WA cos 30° = 0 For Fx = 0, we have
NA = WA cos 30° TA + (FBA + FC ) – (WB + WC) sin 30° = 0

NA = 20 3N 110
T [20 3 75 3 s] 0
Also for impending motion if FAB is frictional force 2
between blocks A and B, then
T (55 95 3 s)
FAB s .N A 20 3 sN ......... (1) Since tension is same, so from (2) and (4), we get
Fx = 0 :
20 (1 3 s) (55 95 3 s)
T – WA sin 30° – FAB = 0
1 Solving for µs we get, 115 3. s 35
T 40. 20 3 s 0
2 35
or s 0.1757
T 20 (1 3 s) ......... (2) 115 3
From FBD of block B and C combined Minimum µs = 0.1757
74. (a)
NA x
WB
FBA
Fc

B Free body diagram at start

NB

FBC m1g
Writing equilibrium equation
Fy = 0 ;
NC – NA – (WB + WC) cos 30° = 0 Fc

3
NC 20 3 110. 0
2
Free body diagram when a = 0
NC 75 3 N
Also for impending motion :
m2g
WB
FBA x
m1g – Fc = m1a ; Fc = m1 (g – a)
NA F c – m2 g = 0 ; F c = m2 g
WC m1 (g – a) = m2g
B
m1 ( g a )
m2 50.8 kg ; m = 9.2 kg
C FC g

NC
LAWS OF MOTION 121

1. (c) For equilibrium of block (A) moving together. Using the FBDs
F = N sin .
N = F/sin . a1
F
To lift block B from ground
y f
f F
F
B 15 kg
Ncos F
A N
a2
F Nsin
N mg 10 kg block will not slip over the 15 kg block till acceleration
x of 15 kg block becomes maximum as it is created only by
N cos mg friction force exerted by 10 kg block on it.
3 a1 a2(max)
F mg tan mg
4 F f f
for limiting condition as f maximum is 60N.
3 10 15
So Fmin mg F = 100 N
4
2. (c) If both the blocks are stationary, Therefore for F = 80N, both will move together.
Balancing forces along x-direction Their combined acceleration, by applying NLM using
both as system, F = 25a
F N sin N / F sin
Balancing forces along y-direction 80
a 3.2 m / s 2
Ny mg N cos 25
5. (b) See explanation of previous question.
Ny 6. (b) If F = 120N, then there will be slipping, so using FBDs of
both (friction will be 60 N)
For 10 kg block,
F Nsin 120 – 60 = 10a
N a = 6 m/s².
For 15 kg block,
Ncos
mg 60 = 50a
a = 4 m/s².
7. (a) The tension in cord 1 is equal to the weight of the
F
= mg cos mg F cot object hanging from it. Therefore, T1 = W, and we wish
sin to find T1 or W.
4F Note that the unknown force, T1, and the known force,
Ny mg 30N, both pull on the knot at point P. It, therefore,
3
makes sense to isolate the knot at P as our object.
3. (a) To keep regular contact The free-body diagram showing the force on the knot
a sin b cos is drawn as given in figure. The force components are
3 also found there.
b a tan a y
4
T2sin 40°

bsin b T2
bcos

in
as a 40°
x
acos T2cos 40° 30N
4. (a) First, let us check upto what value of F, both blocks
move together. Till friction becomes limiting, they will be T1= W
122 IIT-JEE P HYSICS Challenger
Next write the first condition for equilibrium for the Let a = downward acceleration of A.
knot. From the free body-diagram,
1
Fx = 0 becomes 30N – T2 cos 40° = 0 Then a = upward acceleration of B.
Fy = 0 becomes T2 sin 40° – W = 0 2
Solving the first equation for T2 gives T2 = 39.2 N. [As the cord between P1 and A lengthens by 1 unit, the
Substituting this value in the second equation gives W 1
= 25.2 N as the weight of the object. segments on either side of P2 each shorten by unit.
2
8. (b) The knot is in equilibrium under the action of three
forces, and the free-body diagram is as shown in 1
aB t 2
figure (b). 1 SB 2 aB
T3 = W = 40 N. Hence, 2 SA 1 aA
a t2
2 A
y
Write Fy = may for each mass in turn, taking the
direction of motion as positive in each case. We have
T1
1
T1 300 N mB a and 200 N – T2 = mAa
2
60° But m = w/g and so mA = (200/9.8) kg and mB = (300/9.8)
x
kg. Further T1 = 2T2.
70° Substitution of these values in the two equations
T2
allows us to compute T2 and then T1 and a. The results
T3 are :
T1 = 327 N, T2 = 164 N, a = 1.78 m/s2.
13. (c), 14. (a), 15. (a)
Fx = 0 m2 will not move as applied force (m2g sin ) is less
T2 sin 70° – T1 cos 60° = 0 than the maximum frictional force (µ2m2g cos ).
or (0.940) T2 = (0.500) T1, T1 = 1.88 T2 At the instant maximum speed, net force on the m1
Fy = 0 block should be zero.
or T1 sin 60° – T2 cos 70° – T3 = 0 m1g sin 37° = kx + µ1m1g cos 37°
or (0.866) T1 – (0.342) T2 = T3 = 40 N m1 g (sin 37 1 cos 37 )
Substituting for T1, x
k
(0.866) (1.88 T2) – (0.342) T2 = 40N
1.29 T2 = 40 N, 3 4
T2 = 31.0 N and T1 = (1.88) (31.0) = 58.3 N 100 0.5
5 5
9. (a) From figure (a), T1 sin 37° = 300 so T1 = 500N. x 20cm
100
T1 T3
kx
T2 T2 µ1m1g cos 37°
37° 53°
m1

300 W2
m1gsin 37°
(a) (b)
At this instant, speed of m2 block is zero.
Also T2 = T1 cos 37° = 400 N. Hence, maximum force exerted by block of mass m1 on
From figure (b), block of mass m2 will be zero.
T3 sin 53° = T2 so T3 = 670 N. 16. (b) For 5 < t < 5.5
But T3 sin 53° = W2 so W2 = 530 N. N = m (g – a) = m (10 – 10) = 0
10. (a), 11. (b), 12. (a). So man leaves the contact when t > 5 sec
Mass B will rise and mass A will fall. You can see this So correct option t = 5.1 sec.
by noting that the forces acting on pulley P2 are 2T2 17. (a) Nmax = m (g + a) = 60 (10 + 5) = 900 N
up and T1 down. Therefore, T1 = 2T2 (the inertialess Weighing machine reads 90 kg.
object transmits the tension). Twice as large a force is 18. (c) Tmax = (m + m) (g + a) = 150 (10 + 5)
pulling upward on B as on A. 150 × 15 = 2250 N
LAWS OF MOTION 123

19. (c), 20. (b), 21. (d)


The FBD of A and B are as shown in the fig. T1
Applying Newton’s second law to block A and B along
normal to inclined surface
(2)
NA a T2
a sin 53°
a sin 37° NB m1 m2

In frame of lift

A mg cos 53° T2
B
mg cos 37°
m1a0 T2 – m1g – m1a0 = m1a
NB – mg cos 53° = ma sin 53°
mg cos 37° – NA = ma sin 37°
mg
m
Solving N A (4 g 3a ) and
5 T2

m a m2g + m2a0 – T2 = m2a


NB (3g 4a)
5
For NA to be non-zero m 2g m2a0
4g
4g – 3a 0 or a 2 (m1m2 ) ( g a0 )
3 Solving, T2
22. (b) The friction force absorb x mechanical energy from m1 m2
upper block but supply only y mechanical energy to From m1 << m2
lower block. (3) If m1 = m2
Net loss = x – y Relative velocity remains constant.
23. (a) vA vB and attain common velocity.. 28. (d)

24. (c) v = 10 – 0.4 × 10 × t for upper block


v = 0 + 0.4 × 10 × t for lower block N T
10 ma
t = 1.25 sec. mg
2 0.4 10
25. (c), 26. (b), 27. (c). 37°
(1) If lift has acceleration zero, then the acceleration of 3 5
centre of mass of the two blocks is downwards N = mg cos 37° – ma sin 37° = mg ; a = m / s 2
4 6
T1 < (m1 + m2) g 29. (a) T = mg sin 37° + ma cos 37° ; T = 12N
If masses are equal then no acceleration of centre of 30. (a) mg cos 37° = ma sin 37°
mass
T = (m1 + m2) g 40
a= m / s2
3

1. (a) Due to rain, path becomes slippery, so coefficient of 4. (c) Ball bearings are used to convert sliding friction to
friction which provides driving power to a car becomes rolling friction. Sliding friction is less than rolling
lowered. So it is difficult to drive a car on a rainy day. friction.
2. (a) A rocket moves forward taking the help of reaction 5. (a) A and R are correct, R explains A.
force. For that it has to exert a force on the surrounding 6. (b) Cloth can be pulled out without dislodging the
air so that it receives reaction force as per Newton’s dishes from the table because of inertia. Therefore,
third law. statement – 1 is true.
3. (c) A vehicle moving with constant speed on a straight Statement 2 is Newton's third law and hence true.
road is an inertial frame. Newton’s laws of motion is But statement 2 is not a correct explanation of
applicable only in inertial frame. statement 1.
124 IIT-JEE P HYSICS Challenger
7. (d) W = (force) × (displacement of point of application) cases. Hence weight has no effect on tension in
8. (c) Statement – 2 is false because friction force may be situation of figure (ii). Extension in rod occurs due to
more than applied force when body is retarding and force acting at any point on the rod. In certain cases
external force is acting on body. when net force acts at the centre of rod like weight,
9. (d) Statement – 1 is false because sun is also rotating about extension due to this force may not occur like the given
their common centre of mass. case.
10. (b) Both statements are correct. 12. (c) A compressed spring always pushes both the blocks
But reason is Newton's 3rd law. attached at its ends.
Friction force = m × a = 100 × 2 = 200 N 13. (c) Net force on system does not depend on internal forces
11. (c) Tension at a point on rod (length L) at a distance x from within the system but vice-versa is not true.
14. (d) Newton’s first law can’t be derived from Newton’s
x
point of application of force is T F 1 in both second law.
L

1. (a, b) Label the two tensions T1 and T2, and isolate the 3. (a, b, c, d) Let us select as our object the knot at A because
rope at the boy’s hands as the object. The free- we know one force acting on it. The weight pulls
body diagram for this object is shown in figure. down on it with a force of 600N and so the
free-body diagram for the knot is as shown in
y figure (a).
T1 Applying the first condition for equilibrium to that
T2 diagram, we have
T1sin10° 5° Fx = 0 or T2 cos 60° – T1 cos 60° = 0
10° T2sin5°
Fy = 0 or T1 sin 60° + T2 sin 60° – 600 = 0
T1cos10° T2cos5° The first equation yields T1 = T2. Substitution of
T1 for T2 in the second equation gives T1 = 346 N,
W = 90N and this is also T2.
y
Fx = 0 becomes
T2 cos 5° – T1 cos 10° = 0 T1 T2
Fy = 0 becomes
T2 sin 5° + T1 sin 10° – 90N = 0
60° 60°
Evaluating the sines and cosines, these equations
becomes
0.996T2 – 0.985T1 = 0
and 0.087 T2 + 0.174 T1 – 90 = 0
600N
Solving the first for T2 gives T2 = 0.987 T1.
Substituting this in the second equation gives
0.086 T1 + 0.174 T1 – 90 = 0
from which T1 = 346 N. Then, because T2 = 0.989 (a)
T1, we have T2 = 342 N. Let us now isolate knot at B as our object. Its free-
2. (a, c, d) body diagram is shown in figure (b).
//////////// y
T

T3
m1g
T5 20°
x

m2g N T2
30°
N = (m1g + m2g) – T
if T = 0 N = (m1g + m2g)
if T > 0 N < (m1g + m2g)
and T cannot be negative.
(b)
LAWS OF MOTION 125

We have already found that T2 = 346 N and so the 6. (c, d) The FBD for the moving block is
equilibrium equations are
Fx = 0 m F
or T3 cos 20° – T5 – 346 sin 30° = 0
Fy = 0 µkmg
or T3 sin 20° – 346 cos 30° = 0 F k mg
The last equation yields T3 = 877 N. Clearly, a =
m
Substituting this in the prior equation gives 7. (a, b, c, d) There is no horizontal force on block A, therefore
T5 = 651 N
it does not move in x-direction, whereas there is
We can now proceed to the knot at C and the
net downward force (mg – N) is acting on it, making
free-body diagram of figure (c).
y its acceleration along negative y-direction.
Block B moves downward as well as in
negative x-direction. D o wn wa r d s
acceleration of A and B will be equal due to
constrain, thus w.r.t. B, A moves in positive
T4 x-direction.
T5
x
20°
B
30°
T1
Normal reaction
due to C
Due to the component of normal exerted by C on
(c) B, it moves in negative x-direction.
Recalling that T1 = 346 N, NA
Fx = 0 becomes
T5 + 346 sin 30° – T4 cos 20° = 0
Fy = 0 becomes
T4 sin 20° – 346 cos 30° = 0
The latter equation yields T4 = 877 N B
From the symmetry of the system T1 = T2 and
T4 = T3.
4. (c, d) If is the angle made by the direction of force NC
with the horizontal, we have
Mg
F1 cos = µ (mg + F1 sin ) and
The force acting vertically downward on block B
F2 cos = µ (mg – F2 sin ).
Clearly F1 > F2 so that option (c) is correct. are mg and NA (normal reaction due to block A).
If sin = mg /4F2, two relations written above Hence the component of net force on block B
becomes along the inclined surface of B is greater than mg
F1 cos = µ [mg + mg F1/(4F2)] and sin . Therefore the acceleration of B relative to
F2 cos = µ [mg – mg F2/4F2)]. ground directed along the inclined surface of C is
greater than g sin .
F1 1 ( F1 / 4F2 )
From this we get . 8. (b, c, d)
F2 (3/ 4) 50N 50N
Solving this we get F1 = 2F2, so that (d) is correct.
5. (a, b, d) Let F be the force exerted by mass m on mass M.
FBD of mass M a b
k1x1 F
F = k x = 2 × 3 = 6N M
1 1
FBD of mass m
2g 4g
k2 x2
F m 50 – 2g = 2a and also 50 – 4g = 4b
k2x2 = F = 6N
50 20 50 4 10
Hence, the force exerted on block of mass m by a b
the right spring (k2x2) is 6N to the left. From FBD, 2 4
a = 15 m/sec2 upwards b = 2.5 m/sec2
the normal reaction (F) between blocks is 6N.
As system is at rest, net force on block of mass m upwards
is zero. The acceleration of both the masses is upward.
126 IIT-JEE P HYSICS Challenger
9. (a, c)
W
(a) Fx = 0 (c) By symmetry R1 1.15W
cos30
10. (a, d) The frictional force on block A is
W
10 R2 R1 sin 30 a
µN1 = 10 N1 50N g
0.2 W
10N = (1.15 W) (0.5) + a
g
10N N1 10N
N1 a
= W 0.58
10N g
10N
50N 13. (b, d) Let a be acceleration of system and T be tension
in the string.
f F.B.D. of block A
N2 mg sin 30° + T = ma
The net force on block B in vertical direction is
mg
zero T ma ....... (1)
2
N2 = 50 + N1 + 10 = 110 N
Normal reaction exerted by ground on block B
is 110N. The net force on block B in horizontal a
direction is zero A
f + 10 – 10 = 0 T
Frictional force exerted by ground on block B mg sin 30°
is zero.
11. (c, d) Here we choose the x-axis along the incline with F.B.D. of block B
positive upward. All the forces on the block are mg – T = ma ....... (2)
shown in figure. Adding equation (1) and (2), we get

3mg 3
2ma a g
2 4
P
N T
f W
B a
30°

From Fx = max we have P cos 30° – W sin 30°– f mg


= max, where m = 20 kg, W = mg = 196 N, and f = 80 mg
N From eq. (1) T
4
For ax = 0, P = 206 N.
14. (b, c) Free-body diagram at point C :
For ax = 0.75 m/s2, P = 223 N
12. (c, d) From figure, Fver = R1 cos 30° – W = maver = 0
and Fhor = R2 – R1 sin 30° = ma. 700N
1400N

30° R2

a 30° 55°
R1 C

F
W
(a) FBD at C
Thus, the acting forces are
LAWS OF MOTION 127

B Now as resultant lies along X-direction summation


55° of forces along Y-direction should equal to zero.
FBC x = F cos 30° – W cos 60° – P cos 75°= R .... (1)
F
y = P sin 75° – W sin 60° – F sin 30° = 0...... (2)
30° 85°
C From eq. (1),
FAC 600 cos 30° – 400 cos 60° – P cos 75° = R
A
From eq. (2),
(b) Force triangle P sin 75° = 400 sin 60° + 600 sin 30°
(a) Using law of cosines
F2 = (1400 N)2 + (700)2 – 2 (1400 N) (700 N) cos 85° (346.41 300)
P 670N
F = 1510 N sin 75
(b) Using law of sines So from eq. (1),
sin sin 85 R = 146.2 N
1400N 1510N As the value of R is positive, it means that the
assumed direction was correct. So the block is
sin = 0.92362
= 67.461° moving upwards.
= 180° – 55° – 67.461° = 57.5°
17. (c, d) T1
T T1 2
30° 3
T1 3 + T2 3 m 2
a 2 2 30° 2
15. (a, d) A mg – T = ma T = m (g – a)..... (1)
T2
T2
mg+
mg 2

T1 T2
T sin + N mg
T 2 2
T1 – T2 = 2mg
B T cos N = mg – T sin . (2)
3 2 3
(T1 T2 ) m
2 2
mg
T1 + T2 = m 2
From (1), T < mg ; T sin < mg N 0 T1 + T2 > T1 – T2 = 2mg
As string length remains constant, velocity of m 2 > 2mg
both ends along length remains same.
2g
vB cos = u vB = u sec .
16. (a, b, c) By geometry, = (45° + 30°) = 75°
18. (c, d)
Free-body diagram Force triangle
Wcos60° Wsin60° TBC FAC FAC
Fcos30° 60° 60° 45° 45°
W 75° 400N
30° C TBC 30°
P 400N 30°
F Psin75° 30° 60°
Fsin30°
(a) (b)
Pcos75°
From force triangle and applying Lami’s theorem

For resolution refer figure shown. Let R be the FAC TBC 400 N
resultant force acting along incline towards right, sin 30 sin 75 sin 75
so that x = R.
128 IIT-JEE P HYSICS Challenger

400N 50
FAC sin 30 207 N or T2 + 20 = a ............. (2)
sin 75 g

400 Considering motion of C :


TBC sin 75 400 N WC – (T1 + T2) = mCa
sin 75
150
40 150 – T1 – T2 = a ............ (3)
19. (a, b) Given WA = 40 N, m A kg g
g

50
WB = 50 N, mB kg T1 T2
g

150
WC = 150 N, mC kg
g
a
1 3
tan
4
sin = 0.6 and cos = 0.8 WC=150N
µ1 = µ2 = 0.25
Let acceleration of the bodies be ‘a’ and tension
in the two strings connected to A and B be T1 and Adding (1), (2) and (3)
T2 respectively. 240 160 9.8
Considering the motion of A : 160 a or a 6.53 m / s 2
g 240

a From (1),
R1 40 40
T1 10 a 10 6.53 36.65 N
g 9.8
µR1 T1
From (2),

50 50 6.53
WA=40N T2 a 20 20 13.32 N
g 9.8

F1 = µ1R1 = 0.25 × 40 = 10 N 20. (b) This is a problem based on constraint motion. The
motion of mass M is constraint with the motion of P
40 and Q. Let PN = x, NO = z. Then velocity of mass is
T1 – F1 = mAa or T1 10 a
g
dz
.
Considering the motion of B : dt
R2
F2 d
Also, let OP = . then U
T2 dt
From PNO, using pythagorous theorem
WBsin WBcos x2 + z2 = 2
WB =50N
A P x N B
R2 = WB cos = 50 × 0.8 = 40 N
F2 = µ2R2 = 0.25 × 40 = 10 N Z
l
T2 + WB sin – F2 = mBa P U U
Q
50 O
or T2 + 50 × 0.6 – 10 = M
g
LAWS OF MOTION 129

Here x is a constant. Differentiating the above equation


w.r.t to t Mv 2
T – Mg cos
dz d
0 + 2z 2
dt dt
zvM = U

U U z
vM = U cos
z z/ cos
21. (b, d)
Since earth is an accelerated frame and hence, cannot
gsin g gcos
be an inertial frame.
Strictly speaking Earth is accelerated reference frame.
Earth is treated as a reference frame for practical Also, the tangential acceleration acting on the mass is
examples and Newton's laws are applicable to it only g sin .
as a limiting case.
23. (a) At A the horizontal speeds of both the masses is the
22. (b, c)
same. The velocity of Q remains the same in horizontal
Since the body is moving in a circular path therefore it
as no force is acting in the horizontal direction. But in
Mv 2 case of P as shown at any intermediate position, the
needs centripetal force
horizontal velocity first increases (due to N sin ),
reaches a max value at O and then decreases. Thus it
always remains greater than v. Therefore tP < tQ.

Q
l A B
v
Ncos
T N
Nsin
P
v mg O
Mgsin Mg Mgcos

1. (A) p, s; (B) q, r; (C) q, r; (D) q, r 72 – T1 = 1 × 8


For case I, take torque about COM T1 = 64 N.
For case II, take torque about any point on the ground. On 5 kg
2. (A) q; (B) r; (C) p; (D) s
T2 30

3 N1
50
T2 sin 37° = 30
T2 = 50 N N1 = 80 N.
50 cos 37 Net force on 3 kg block = T 1 – T2cos37°
Acceleration of 5 kg = 8 m/s2 = (64 – 40) N = 24 N
5
F = 8 × 9 = 72 N 3. (A) p, s; (B) r; (C) q, s; (D) q, r
On 1 kg block F
2 kg
T1 72 f1
130 IIT-JEE P HYSICS Challenger
6. (A) q; (B) q; (C) r; (D) s
f1
FBD’s
4 kg
f2
N2 T
a N
Limiting friction on 2 kg = 0.2 × 2 × 10 = 4 N
Limiting friction on 4 kg at lower surface T
= 0.2 × 6 × 10 = 12 m2 T T
M
When force is 3 N, 2 kg block does not move as F < 4. T = m2a
friction on 2 kg is 3 N. T
m2g
When F = 10 N, friction on 2 kg is 4 and net friction on Mg
4 kg will be zero.
As it cannot move since (f2)max > (f1)max.
4. (A) q, r; (B) r, q ; (C) p; (D) s N1
(A) This vector sum is along q and r
a
Acc of B
T m1 F

Acc of A
m 1g
(B) Sum of these forces can be r and q.
F – T = m1a
F = (m1 + m2) a T = m2 a

N Friction F m2 F
a T , Fx = 0, aM = 0
(C) Only P. m1 m2 m1 m2
(D) Sum of normal and friction is s.
7. (A) q; (B) p; (C) r
Normal In each case friction is necessary for equilibrium. But friction
(s) does not enter in solving for N.
(A) Free-body diagram in figure (a) (F = 20N, W = 50N)
Fy = 0 N – W + F sin 55° = 0
N = 50N – (20N) (0.819) = 33.6 N
Friction
F
5. (A) s; (B) r; (C) p, s
55°
2kg
fr
fr fr 2 2 4 fr 4 f
(p) 4kg 12N 12 fr 4 2 fr 4 N
\\\\\\\\\\\\\\\\\\\\\ W
(a)
(B) Free-body diagram in figure (b) : (W = 60 N)
2kg
fr 12N Fy = 0 N – W cos 40° = 0
fr 12 fr 2 2 fr 8
(q) 4kg fr 4 2 fr 8 N = (60N) (0.766) = 46.0 N
\\\\\\\\\\\\\\\\\\\\\

2kg
fr 6N
fr 6 fr 2 3 fr 2
(r) 4kg 6N fr 6 4 2 fr 2 N
40°
\\\\\\\\\\\\\\\\\\\\\

8N 2kg
fr 20 fr 4 2 fr 12 f W
(s) fr fr 8 2 2 fr 12
4kg 20N (b)
\\\\\\\\\\\\\\\\\\\\\
LAWS OF MOTION 131

(C) Free-body diagram in figure (c) :


(F = 70 N, W = 60N) F1 =20N F2 =60N
Fy = 0 N – W cos 40° – F sin 40° = 0
N = 60 (0.766) + 70 (0.643) = 91.0 N
fmax =40N fmax=20N

F1= 20N N=40N N=40N F2= 60N


F N 20kg 10kg
40°
f = 20N f = 20N
FBD of both blocks
f W
Hence, magnitude of friction force on both blocks is 20N
(c)
and is directed to right for both blocks. Normal reaction
8. (A) r; (B) q; (C) s; (D) p
exerted by 20kg, block on 10 kg. block has magnitude 40N
(A) As velocity = 0 no radial acceleration
directed towards right. Net force on system of both blocks
3mg is zero.
T = mg cos 53° =
5 11. (A) p, q, t; (B) s; (C) s; (D) p, s
(B) Motion in horizontal plane no acceleration in vertical (A) 20 – 0.5 × 2 × 10 = ma ; a = 5 m/s2, f = 10N
direction Tension at mid point,
T – 0.5 × 1 × 10 = 1 × 5 T = 10 N
mg 5
T cos 37° = mg T= (B) Speed constant a=0
4
Pulling force = mg sin +f
(C) Acceleration of particle w.r.t. cart is zero
1
5mg (C) Fnet = 7.5 – 0.75 × g × sin 30° – × 0.75g cos 30°
T cos 53° = mg T= 3
3

4mg 1 1 3
(D) T = mg cos 37° T= = 7.5 – 0.75g × 0.75 g =0
5 2 3 2

9. (A) p, r ; (B) p, r ; (C) q, s ; (D) q, s (D) Fnet = 20 – 2g = 0 ; T – 1g = 1 × 0 T = 10 N


At positions P and R frictional force is maximum and 12. (A) s; (B) r; (C) q; (D) r
normal reaction is mg. At positions S and Q frictional In (A) we try to decrease of the rear wheel then friction
force is zero and normal reaction points towards centre. will try to increase it. On front wheel at the point of contact
10. (A) p, s; (B) p, s; (C) q, s; (D) r
with the ground ( r ) is greater than v as v decreases due
The minimum horizontal force required to push the two
block system towards left to break. So that friction will act in forward direction on it.
= 0.2 × 20 × 10 + 0.2 × 10 × 10 = 60 N In (B) we try to decrease of the front wheel, friction will try
to increase it.
Hence the two block system is at rest. The FBD of both
of blocks is as shown. The friction force f and normal In (C) sliding will start hence fk will act.
reaction N for each block is as shown. In (D) we try to increase of rear wheel then friction will try
to decrease it.
132 IIT-JEE P HYSICS Challenger

1. 30 3. 4
Resolving M1g into rectangular components, we have M1g Force on 10 kg mass = 10 × 12 = 120 N
sin 30º acting along the plane downwards, and M1g cos30º The mass of 10 kg will pull the mass of 20 kg in the backward
acting perpendicular to the plane downwards. The situation direction with a force of 120 N.
has been shown in Fig. Net force on mass 20 kg = 200 – 120 = 80 N
80 N
R Its acceleration 4 m/s2
20 kg
T
F 4. 150
s in The situation is shown in fig.
M 1g M1 g cos Let F be the horizontal force applied on A.
M 1g M2g For block A, F = T + f1 = T + m1g .....(1)
( Block A moves towards left, frictional force f1 acts towards
Let T be the tension in the wire and R be the reaction of right)
plane on the mass M1. Since the system is in equilibrium, For block B, fB = f2 + f3
therefore,
T = M1g sin30º ...(i)
and R = M1g cos30º ...(ii)
T = M2g ...(iii) T
F A
From (i) and (iii), we have
T = M1g sin30º = M2g ...(iv) f2 f1
B
T T
Velocity of transverse wave, v , f3
m
where m is the mass per unit length of the wire. ( Block B moves towards right, frictional forces f2 and f3
2 2 acts towards left).
v T / m , or T = v m = (100)2 × (9.8 × 10–3) = 98N
From (iii), M2 = T/g = 98/9.8 = 10kg. T m1g ( m1 m2 ) g g (2 m1 m2 ) ...(2)
From (iv), M1 = 2M2 = 2 × 10 = 20kg. From eqs. (1) and (2), we get
M1 M 2 30 kg F g (2 m1 m2 ) m1 g or F g (3 m1 m2 )
2. 60
F 0.3 10 3 10 20
If is the angle which the inclined plane makes with the
= 150 N
vertical direction, then the acceleration of the block sliding
5. 0.12
down the plane of length will be g cos .
Make free body diagram of m
1 2 Take right as the positive direction. Let a 1/g be the
Using the formula, s ut at , we have s = , u = 0,
2 acceleration of m w.r.t. ground.
t = t and a = g cos .
N
1 1 N'
So 0 tg cos t 2 (g cos )t 2 ...(i) N
2 2 f1
Taking vertical downward motion of the block, we get m M
1 1 2 f1 f2 Mg
h 0 g(t / 2) 2 gt / 4 ...(ii) mg
2 2
h 1 f1
Dividing (ii) by (i), we get [ cos h/ ] a1/ g f1 mg
4cos m 2
1 1 1
or cos ; or cos 2 ; or cos a1/ g
g g
4 cos 4 2 ...(1)
2 2
or = 60º N' = N + Mg and N' = (m + M)g ...(2)
LAWS OF MOTION 133

f2 = N' = (m +M)g ...(3) These conditions will be satisfied when T < 15 N say T = 14
then f1 = f2 = 14 N.
( f 2 f1 )
a2 / g Therefore the condition f1 = 2f2 will not be satisfied. Thus
M
m1 and m2 both can't remain stationary.
[ a2/g is acceleration of M w.r.t. ground] In the second case, when m1 and m2 both move
f2 = (f2)max = 15 N
{ (m M )g / 2 mg} m
Mg 1 Therefore, f1 = 2f2 = 30 N
M 2M
f1 f2 15 N
a1/2 = acceleration of m w.r.t. to M = a1/g – a2/g
7. 2
g m 1 m
Mg 1 Mg 1 mg sin k mg cos
2 2M 2 2M a=
m
[m M ] aA = g sin – µk, A g cos ... (i)
Mg
2M and aB = g sin – µk,B g cos ... (ii)
Putting values, we get
1 4 ML
Now L a1/ 2t 2 t
2 ( M m) g 0.89 0.79
aA = and aB =
2 2
4 20 0.04
= 0.12 s
20 10 0.3 10 µN = fK N mg cos
6. 15
Given a
m1 = 20 kg, m2 = 5 kg, M = 50 kg, µ = 0.3 and g = 10 m/s2
mgcos mgsin
(A) Free body diagram of mass M is :
mg

T N1 aAB is relative acceleration of A w.r.t. B


T f1 a AB = a - a
T A B
Mg F
T 1
L= 2m L= a AB t 2
2
N
[where L is the relative distance between A and B]
(B) The maximum value of f1 is
2L 2L
(f1)max = (0.3) (20) (10) = 60 N or t2 =
aAB aA aB
The maximum value of f2 is
Putting values we get, t2 = 4 or t = 2s.
(f2)max = (0.3) (5) (10) = 15 N
8. 10
Forces on m1 and m2 in horizontal direction are as follows :
Applying pseudo-force ma and resolving it.
T m1 m2 T Applying Fnet = max for x-direction
f1 f2
ma cos – (f1 + f2) = max
Now, there are only two possibilities. ma cos – µN1 –µN2 = max
(1) either both m1 and m2 will remain stationary (w.r.t. ma cos – µma sin – µ mg = max
ground) or (2) both m1 and m2 will move (w.r.t. ground). First
case is possible when ax = a cos – µa sin – µg

T < (f1)max or T < 60 N 4 2 3 2


= 25 25 10 10m/s 2
and T < (f2)max or T < 15 N 5 5 5 5
134 IIT-JEE P HYSICS Challenger
9. 10 While block B will lose contact, when
10 t = 2T T' mB g
T = 5t
5t = 2mBg
Block A will lose contact when
T = mAg 2g
or t2 sec = 4 sec
5t = mAg 5

mA g At t1 t for block A
t1 sec 2sec T – mg = ma
5
mdv
5t – mg
F=10t dt

v t2
m dv (5t – mg )dt
0 t1

v = 10 m/s
A B
1. A particle A of mass 10/7 kg is moving in the positive 3. A uniform rope of linear mass density and length is
direction of x. Its initial position is x = 0 & initial velocity is coiled on a smooth horizontal surface. One end is pulled up
1 m/s. The velocity at x = 10 is : (use the graph given) with constant velocity v. Then the average power applied
Power (in watts) by the external agent in pulling the entire rope just off the
horizontal surface is
v
4

2
x 1 g
10 (in m) (a) v2 + (b) gv
2 2
(a) 4 m/s (b) 2 m/s
1 3 vg 1 3
(c) v + (d) vg v
(c) 3 2 m/s (d) 100/3 m/s 2 2 2
2. As shown in the figure, there is no friction between the 4. In the above question the maximum power delivered by the
horizontal surface and the lower block (M = 3 kg) but friction agent in pulling up the rope is
coefficient between both the blocks is 0.2. Both the blocks
v3
move together with initial speed v towards the spring, (a) gv (b) gv
2
compresses it and due to the force exerted by the spring,
moves in the reverse direction of the initial motion. What gv v3
can be the maximum value of v (in cm/s) so that during the (c) gv v3 (d)
2 2
motion, there is no slipping between the blocks.
5. A particle falls from a height h on a fixed horizontal plane
(use g = 10m/s²) and rebounds. If e is the coefficient of restitution, the total
distance travelled by the particle before it stops rebounding
1 kg is
K=400N/m µ=0.2
3 kg
M h [1 e 2 ] h[1 e 2 ]
(a) (b)
2 [1 e 2 ] [1 e 2 ]
smooth
(a) 20 cm/s (b) 10 cm/s h [1 e 2 ] h[1 e 2 ]
(c) (d)
(c) 30 cm/s (d) 40 cm/s 2 [1 e 2 ] [1 e 2 ]

MARK YOUR
1. 2. 3. 4. 5.
RESPONSE
136 IIT-JEE PHYSICS Challenger
6. A man places a chain of mass m and length L on a table 12. A stationary particle explodes into two particles of masses
slowly. Initially the lower end of the chain just touches the m1 and m2 which move in opposite directions with velocities
table. The man drops the chain when half of the chain is in v1 and v2. The ratio of their kinetic energies E1/E2 is
vertical position. Then work done by the man in this process (a) m1v2/m2v1 (b) m2/m1
is (c) m1/m2 (d) 1
L mgL 13. A block lying on a smooth surface with spring connected to
(a) mg (b)
2 4 it is pulled by an external force as shown. Initially the velocity
of ends A and B of the spring are 4 m/s and 2 m/s
3mgL mgL respectively. If the energy of the spring is increasing at the
(c) (d)
8 8 rate of 20 J/sec, then the stretch in the spring is
7. In one dimensional motion, a 1 kg object experiences a force,
k=100 N/m
which is a linear function of time t viz. F = 2t acting in the
direction of motion. The work done by the force in first 4 B A
seconds is
(a) 16 J (b) 32 J
(a) 1.0 cm (b) 2.0 cm
(c) 64 J (d) 128 J
(c) 10 cm (d) 2.0 cm
8. A toy gun uses a spring of force constant k. When charged 14. A rectangular plate lie in the horizontal plane as shown in
before being triggered in the upward direction the spring is the figure. At one end a peg is situated and a particle of
compressed by x. If the mass of the shot is m, on being mass m tied with a string of length through the peg and
triggered, it will go up to a height of initially particle is just below the peg. A horizontal velocity
(a) kx2/2mg (b) kx2/mg of magnitude 130g is given to the particle then after
2 2
(c) k x /2mg (d) x2/3mg how many collisions particle does not touch the plate if
9. What will be the power output of the heart if in each coefficient of restitution between the plate and the particle
heartbeat, it pumps 80 ml of blood at an average pressure of is 1/2.
100 mm of mercury? (assume 60 heartbeats/min).
Plate
(a) 1.07 W (b) 1.04 W
(c) 1.035 W (d) 1.02 W
10. An electron of mass m moving with a velocity v collides P
head on with an atom of mass M. As a result of the collision
a certain fixed amount of energy E is stored internally in
the atom. The minimum initial velocity possessed by the
electron is : l

2( M m) E 2M E v m
(a) (b)
Mm ( M m) m (a) 2 (b) 3
(c) 4 (d) 5
2( M m) E
(c) (d) None of these 15. A block of mass 1 kg is pulled along the curve path ACB by
Mm a tangential force as shown in figure. The work done by the
11. Hail storms are observed to strike the surface of the frozen frictional force when the block moves from A to B is
lake at 300 with the vertical and rebound at 600 with the = 0.2
vertical. Assume contact to be smooth, the coefficient of C
restitution is
1 1
(a) e (b) e A B
3 3 x = 10 m
(a) 5 J (b) 10 J
(c) e 3 (d) e = 3 (c) 20 J (d) none of these

MARK YOUR 6. 7. 8. 9. 10.


RESPONSE 11. 12. 13. 14. 15.
WORK, POWER, ENERGY & CONSERVATION LAWS 137

16. A uniform block of mass m and length is kept stationary 20. Consider a one-dimensional collision where a body of mass
on a frictionless track as shown. The centre of mass of the m1originally moving in the positive x direction with speed
block is initially at a height 10 from the horizontal ground. v0 collides with a second body of mass m2originally at rest.
The circular track is followed by a horizontal frictionless The collision could be completely inelastic, with the two
track after travelling a long distance, the box slides on a bodies sticking together, completely elastic, or somewhere
rought track of coefficient µ = 0.2. Calculate the distance in between. After the collision, m1 moves with velocity v1 ,
covered by the block on the rough surface.
while m2 moves with velocity v 2 . If m1> m2, then
(a) v0 < v1 < v2 (b) v2 < v1 < v0
=0 10 v0
(c) v0 /2 < v2 < v1 (d) < v2 < 2v0
2
21. Two blocks of masses 10 kg and 4 kg are connected by a
(a) /2 (b) spring of negligible mass and placed on a frictionless
(c) 49.5 (d) Data insufficient horizontal surface. An impulse gives a velocity of 14 m/s to
17. Two blocks of masses m and M are joined with an ideal the heavier block in the direction of the lighter block. The
spring of spring constant k and kept on a rough surface as velocity of the centre of mass is
shown. The spring is initially unstretched and the coefficient (a) 30 m/s (b) 20 m/s
of friction between the blocks and the horizontal surface is (c) 10 m/s (d) 5 m/s
µ. What should be the maximum speed of the block of mass
22. Mass m1 strikes m2 which is at rest. The ratio of masses for
M such that the smaller block does not move?
which they will collide again (collision between ball and
v0
wall are elastic, coefficient of restitution between m1 and m2
k is e and all the surfaces are smooth.)
m M

µg
Mm ( M + m) k
(a)
( M + m) k (b) µg
Mm
m1 m2
µg
( 2M + m) m
(c) (d) None of these
kM
e 2e
18. A block is suspended by an ideal spring constant k. If the (a) (b)
2+e 2+e
block is pulled down by constant force F and if maximum
displacement of block from it’s initial position of rest is z, then e
(c) (d) none of the above
(a) z = F/k (b) z = 2F / k 2(2 + e)
(c) work done by force F is equal to 2Fz.
23. A bowler throws a ball horizontally along east direction with
1 2 speed of 144 km/hr. The batsman hits the ball such that it
kz
(d) increase in potential energy of the spring is
2 deviates from its initial direction of motion by 74° north of
east direction, without changing its speed. If mass of
19. A collision occurs between two identical balls of mass m
the ball is 1/3 kg and time of contact between bat and ball is
each, moving with velocities u1 and u 2 . If the collision is 0.02 s, average force applied by batsman on ball is
head on and the energy lost in the collision is (a) 800 N, 53° East of North

E=
3
16
(
u1 – u 2 )2 then the coefficient of restitution is (b) 800 N, 53° North of East
(c) 800 N, 53° North of West
(a) 0.25 (b) 0.75 (d) 800 N, 53° West of North
(c) 0.5 (d) 0.9

MARK YOUR 16. 17. 18. 19. 20.


RESPONSE 21. 22. 23.
138 IIT-JEE PHYSICS Challenger
24. A block of 1 kg is kept on a rough surface of an elevator 27. The adjacent figure shows an interaction force between two
moving up with constant velocity of 5m/s. In 10 seconds particles A and B against the separation between them, when
work done by normal reaction (the block does not slide on the separation between them changes from zero to infinity.
the inclined surface) is Parts of the curve above and below the x-axis denote
attractive and repulsive forces, respectively. Then which of
the following statement is incorrect?
F
5m
1 kg

x1 x2
37°
x3 x4 x
(i) from ground frame it is 320 J
(ii) is equal to work done by friction force in elevator frame
(iii) is equal to work done by friction in ground frame (a) Kinetic energy of the system increases from x1 to x2
Of the three statements given above, the ones that are true (b) Potential energy of the system decreases from x1 to x2
is given by the choice (c) Kinetic energy of the system increases from x3 to x4
(a) (i) (b) (ii), (iii) (d) Kinetic energy of the system decreases from x3 to x4
(c) (i), (ii) (d) only (iii). 28. A body of mass 10 g is attached to a hanging spring whose
25. A small block of mass m is placed on a bigger block of mass force constant is 10 N/m. The body is lifted until the spring
M which is placed on a horizontal frictionless plane. The is in its unstretched state. The body is then released.
two blocks are given equal speeds u but in opposite Calculate the speed of the body when it strikes a table 15 cm
directions as shown in the figure. After sometime, it is below the release point.
observed that both the blocks are moving in the direction (a) 0.831 m/s (b) 0.512 m/s
of motion of the lower block, with a velocity greater than (c) 1.121 m/s (d) 2.142 m/s
u 29. A block of mass m = 1kg is moving with a constant acceleration
. It can be concluded that a = 1 m/s2 on a rough horizontal plane. The coefficient of
2
friction between the block and plane is µ = 0.1. The initial
Friction u velocity of block is zero at t = 0. The power delivered by the
m Horizontal external agent at a time t = 2 sec from the beginning is equal to
frictionless (Take g = 10m/s2)
u M (a) 1 watt (b) 2 watt
surface
(c) 3 watt (d) 4 watt
(a) M > 3m 30. The net power of all the forces acting on a particle (P) versus
(b) 3M < m time curve is shown. Work done upon the particle from A to B
(c) m > 2M
(d) M.m can have any value such that M > m. P
26. Two particles of masses m1 and m2 in projectile motion have A B
velocities v1 and v2 respectively at time t = 0. They collide
at time to. Their velocities become v1 ' and v2 ' at time 2to
while still moving in air. The value of t

( m1v1 ' m2 v2 ') ( m1v1 + m2 v2 ) is (a) increases


(b) decreases
(a) zero (b) (m1 + m2 ) gto (c) first increases then decreases
1 (d) first decreases then increases
(c) ( m1 + m2 ) gto (d) 2(m1 + m2)gto
2

MARK YOUR 24. 25. 26. 27. 28.


RESPONSE 29. 30.
WORK, POWER, ENERGY & CONSERVATION LAWS 139

31. For the path PQR in a conservative force field (fig.), the 33. A block of mass m is suspended by a light thread from a lift.
amount of work done in carrying a body from P to Q and The lift is moving upward with uniform velocity v. From the
from Q to R are 12J and 5J respectively. The work done in frame of lift, the work done by tension on the block in t
carrying the body from P to R will be seconds will be

P
T v

Q R
(a) – mgvt (b) 0
(a) 7J (b) 13J mgvt
(c) (d) mgvt
(c) 17J (d) zero 2
32. A smooth track in the form of a quarter circle of radius 6m 34. An isolated particle of mass m is moving in horizontal plane
lies in the vertical plane (see figure). A particle of weight 4N (x – y), along the x-axis, at a certain height above the ground,
it suddenly explodes into two fragments of masses m/4 and
moves from P1 to P2 under the action of forces F1 , F2 , and 3m/4. An instant later, the smaller fragment is at y = +15 cm.
The larger fragment at this instant is at
F3 . Force F1 is always towards P2 and is always 20N in
(a) y 5 cm (b) y = +20 cm
magnitude, force F2 always acts horizontally and is always (c) y = +5 cm (d) y = – 20 cm
35. A massive disc of radius R is moved with a constant velocity
30N in magnitude, force F3 always acts tangentially to the u on a frictionless table. Another small disc collides with it
track and is of magnitude (15 – 10s) N when s is in meters. If the elastically with a speed of v0 = 0.3 m/s, the velocities of the
particle has speed 4 m/s at P1, what will its speed be at P2? discs being parallel. The distance d shown in the figure is
equal to R/2, friction between the discs is negligible. For
which u (in m/s) will the small disc move perpendicularly to
its original motion after the collision ?
6m
v0
P2
2 u d
R

(a) 0.1 (b) 0.5


6m
F1= 20N (c) 1.0 (d) 0.01
36. A man of mass m on an initially stationary boat gets off the
F3= 15–10s(N) boat by jumping to the left in an exactly horizontal direction.
F2= 30N Immediately after the jump, the boat of mass M, is observed
to be moving to the right at speed v. How much work did the
s w = 4N man do during the jump (both on his own body and on the
P1 boat)
1 1 M2 2
(a) ( M + m )v 2 (b) M+
m ÷
v
2 2
(a) 11.3 m/s (b) 15.4 m/s
(c) 22.6 m/s (d) 6.5 m/s 1 Mm 2
(c) ÷v (d) None of these
2 M +m

MARK YOUR 31. 32. 33. 34. 35.


RESPONSE 36.
140 IIT-JEE PHYSICS Challenger
37. An engine pumps water continuously through a hose. If Fz (in N)
the speed with which the water passes through the hose
nozzle is v, and if k is the mass per unit length of the water jet
as it leaves the nozzle. What is the rate at which kinetic
energy is being imparted to the water ?
1 2 1 3
(a) kv (b) kv
2 2 Z (in m)
(c) kv 3 (d) kv 2 12
38. The cause of increase in kinetic energy when a man starts
running without his feet slipping on ground is asked from
two students. 16
Rishabh : Cause of increase in kinetic energy is work done
400
by friction force without which he cannot run. (a) 192 J (b) J
Bappy : Cause of increase in kinetic energy is work done by 3
internal forces of the body. 287
(a) Rishabh is correct, Bappy is wrong (c) J (d) None of these
2
(b) Bappy is correct, Rishabh is wrong 42. A bomb travelling horizontally at 90km/hr suddenly explodes
(c) Both are correct into two parts of mass ratio 1 : 3. Immediately after explosion,
(d) Both are wrong the lighter mass is found to be travelling vertically upwards
39. If a machine is lubricated with oil
(a) the mechanical advantage of the machine increases. with speed 360 3 km / hr . The heavier one must be
(b) the mechanical efficiency of the machine increases. travelling at
(c) both its mechanical advantage and efficiency increase. (a) 240 km/hr vertically downwards
(d) its efficiency increases, but its mechanical advantage (b) 240 3 km / hr downwards 30° to horizontal (initial
decreases. direction)
40. Two blocks of masses m1 = 10 kg and m2 = 20 kg are (c) 240 km/hr downwards 60° to horizontal (initial direction)
connected by a spring of stiffness k = 200N/m. The (d) None of the above
coefficient of friction between the blocks and the fixed 43. A 1 kg particle at a height of 8m has a speed of 10m/s down
horizontal surface is µ = 0.1. Find the minimum constant a fixed incline making an angle 53° with horizontal as shown
horizontal force F (in newtons) to be applied to m1 in order in figure. It slides on a horizontal section of length 10 m at
to slide the mass m2. [Take g = 10 m/s2] ground level and then up a fixed incline making an angle 37°
m2 g with horizontal. All surfaces have µk = 0.5. How far (in meters)
(a) m1 g + (b) m1 g m2 g from point O (bottom of right inclined plane), along the incline
2 making an angle 37° with horizontal, does the particle first
m2 g m1 g m2 g come to rest ?
(c) m1 g (d)
2 2
41. The components of a force acting on a particle are varying 10m/s
according to the graphs shown. When the particles move
from (0, 5, 6) to (2, 10, 0) then the work done by this force is 8m
53° O 37°
Fx(in N) Fy (in N) ///////////////////////////////////////////////
20
10m
37° (a) 1 m (b) 0.5 m
10 (c) 2 m (d) 5 m
44. Two masses of 1 gm and 4 gm are moving with equal kinetic
energies. The ratio of the magnitudes of their linear momenta
X (in m) Y (in m)
is
15
(a) 4 : 1 (b) 2 :1
(c) 1 : 2 (d) 1 : 16

MARK YOUR 37. 38. 39. 40. 41.


RESPONSE 42. 43. 44.
WORK, POWER, ENERGY & CONSERVATION LAWS 141

45. A particle moves along x-axis under action of force 48. An ideal spring with spring-constant k is hung from the
F = ax – bx2, where a = 1 N/m, b = 1 N/m2, F is in newtons ceiling and a block of mass M is attached to its lower end.
and x is x-coordinate of the particle in meters. The work The mass is released with the spring initially unstretched.
done by this force as the particle moves from x = 0 to Then the maximum extension in the spring is
x = 1.5m is
4Mg 2Mg
1 (a) (b)
J k k
(a) Zero (b)
6
Mg Mg
2 3 (c) (d)
J J k 2k
(c) (d)
6 6 49. A 2 kg mass moves along x-axis. Its acceleration as function
46. A ball of mass m collides horizontally with a stationary of its position is shown in the figure. What is the total work
wedge on a rough horizontal surface, in the two orientations done (in joule) on the mass by the force as the mass moves
as shown. Neglect friction between ball and wedge. Two from x = 0 to x = 10 m ?
student comment on system of ball and wedge in these
situations. 1

a(m/sec²)
y 2

m m 3
m m 4
x x(m)
rough rough 10 8 6 4 2
(I) (II) (a) 25 J (b) 30 J
(c) 50 J (d) 40 J
Saurav : Momentum of system in x-direction will change by
50. A particle of mass m = 1kg is lying at rest on x-axis,
significant amount in both cases.
experiences a net force given by law F = x (3x – 2) newton,
Rahul : There are no impulsive external forces in y-direction
where x is the x-coordinate of the particle in meters. The
in both cases hence the total momentum of system in y-
magnitude of minimum velocity in negative x-direction to
direction can be treated as conserved in both cases.
be imparted to the particle placed at x = 4 meters such that it
(a) Saurav is wrong and Rahul is correct
(b) Saurav is correct and Rahul is wrong reaches the origin is P / 27 m/s. Find the value of P..
(c) Both are correct (a) 3500 (b) 3000
(d) Both are wrong (c) 2600 (d) 2000
47. Two objects each of mass m are connected with a uniform 51. A bead of mass m is sliding down the fixed inclined rod
rope having same mass m and length = 10/3 m are shown without friction. It is connected to a point P on the horizontal
in the figure. The rope can slide on a frictionless pulley surface with a light spring of spring constant k. The bead is
which is fixed to the edge of the table. Initially 1/3 of the initially released from rest and the spring is initially
rope hangs vertically. Friction is negligible everywhere. unstressed and vertical. The bead just stops at the bottom
of the inclined rod. Find the angle which the inclined rod
(2/3) makes with horizontal.
m
m
///////////////////////////////////////////
//////////////////////////

(1/3)
h
k
m
//////////////////////////////////////
P

2mg 2mg
(a) cot 1
1+ (b) 1
1+
kh ÷
tan
kh ÷
If the system is released from rest and object on the right
hand side descend further by a distance of /3, what would
be the speed of the object (in m/s) ?
(a) 2.12 m/s (b) 3.33 m/s mg mg
(c) 1
1+ 1
1+
kh ÷
cot (d) tan
(c) 0.33 m/s (d) 4.12 m/s kh ÷

MARK YOUR 45. 46. 47. 48. 49.


RESPONSE 50. 51.
142 IIT-JEE PHYSICS Challenger
52. Three springs, labelled (A), (B) and (C) are arranged as
shown with three identical balls all the springs are parts cut
from a single long spring such that A and B have equal
natural length but c has natural length 2 . All surfaces are
smooth. Which spring is compressed by the smallest amount 2h
in equilibrium ? h

(A)
(a) (b)
(a) dgha (b) dgh 2a
(c) 2gdh 2a (d) 4dgh 2a
(C) 56. An inelastic collision occurs between two particles such
(B) that external forces are absent on the system of these two
particles. Consider the following statements based on above
situation.
(a) A (b) B S1 : The velocity of both the particles may be same after
(c) C the collision.
(d) cannot be determine with given information. S2 : Kinetic energy of the system of both particles is not
53. A particle, which is constrained to move along the x-axis, conserved
is subjected to a force in the same direction which S3 : Linear momentum of the system of both particles is
varies with the distance x of the particle from the origin as conserved.
F(x) = –kx + ax3. Here k and a are positive constants. For (a) FTF (b) TTT
x 0 , the functional form of the potential energy U(x) of (c) TTF (d) FFT
the particle is 57. A block of mass 2 kg was moving along a straight line on a
smooth surface with a speed of 5 m/s. At t = 0, a force given
U(x) U(x) by F = (3 + 2t) N directed in the direction of motion of the
body starts acting on the block. The kinetic energy of the
(a) X (b) X
block after 2 sec is
(a) 20 J (b) 200 J
(c) 100 J (d) None of these
58. A wind-powered generator converts wind energy into
U(x) U(x) electrical energy. Assume that the generator converts a fixed
(c) X (d) X fraction of the wind energy intercepted by its blades into
electrical energy. For wind speed v, the electrical power
output will be proportional to
54. Consider the following statements for a rigid object (a) v (b) v2
(c) v 3 (d) v4
undergoing pure translational motion
S1 : If an object receives an impulse its kinetic energy must 59. Two small beads each of mass m are attached to a smooth
ring of mass m as shown in the figure. The whole system is
change.
kept on smooth horizontal table. Initially, each bead has
S2 : An object’s kinetic energy can change without the
given velocity v0 as shown in figure. What will be the
object receiving any impulse.
velocity of bead just before impact ?
S3 : An object can receive a net impulse without any work m
being done on it. v0
S4 : A force may do work on an object without delivering
any impulse. m
State, in order, whether S1, S2, S3, S4 are true or false.
v0
(a) FTTF (b) TFTF m
(c) TTFF (d) FFTF 7 7
55. A liquid in a U tube held in vertical plane is changed from (a) v0 (b) v0
3 3
position (a) to position (b) with the help of a pump. The
density of liquid is d and area of cross section of the tube is 7
‘a’. The work done in pumping the liquid will be (c) v0 (d) None of these
3

MARK YOUR 52. 53. 54. 55. 56.


RESPONSE 57. 58. 59.
WORK, POWER, ENERGY & CONSERVATION LAWS 143

60. The energy content of gasoline is 3.6×107 J/L. A motor with 65. A particles of mass m is moving in a circular path of constant
an efficiency of 20% is needed at full output power of 45kW radius r such that its centripetal acceleration ac is varying
for 50.0 minutes. How many litres of gasoline are required to with time t as ac = k2rt2 where k is a constant. The power
operate the motor for this amount of time ? delivered to the particles by the force acting on it is:
(a) 0.31 L (b) 0.38 L
(c) 1.6 L (d) 19 L (a) 2 mk 2 r 2 t (b) mk2r2t
61. A ball is dropped from a height h above the landing and
bounces down a flight of stairs. If e is coefficient of (mk 4 r 2 t 5 )
(c) (d) Zero
restitution, determine the value of h so that the ball bounces 3
by the same height h above each step. The vertical distance 66. Denis is riding his bicycle at 12.0 km/h. When he applies the
between successive steps is d. brakes, locking the wheels and sliding to a stop. Denis has
d d a mass of 15.0 kg, while the bicycle has a mass of 10.0 kg.
(a) (b) Assuming any energy conversion into heat, calculate the
1 e 2
1 + e2
amount of heat generated by the friction between the tyres
2d 2d and the road while coming to a stop.
(c) (d)
1 e 2
1 + e2 (a) 195 J (b) 139 J
62. A particle is acted by a force F = kx, where k is a +ve (c) 108 J (d) 95.0 J
constant. Its potential energy at x = 0 is zero. Which curve 67. An escalator has 2400 W of power available to move
correctly represents the variation of potential energy of passengers from the first floor of a mall to the second, 6.0 m
the block with respect to x vertically. If the average mass of the passengers is 65 kg,
what is the maximum number of passengers that can be
U
U carried to the second floor in 1.0 minute?

(a) x (b) x

U
6.0m
U

(c) x (d) x

(a) 4 (b) 5
(c) 22 (d) 37
63. Two particles interact by conservative forces. In addition,
an external force acts on each particle. They complete round 68. A spring of force-constant k is cut into two pieces such that
trips, ending at the points where they started. Which of the one piece is double the length of the other. Then the long
following must have the same values at the beginning and piece will have a force-constant of
end of this trip ? (a) (2/3)k (b) (3/2)k
(a) the total kinetic energy of the two-particle system (c) 3 k (d) 6 k
(b) the potential kinetic energy of the two-particle system 69. A Ping-Pong ball is thrown at a stationary bowling ball
(c) the mechanical energy of the two-particle system hanging from a wire. The Ping-Pong ball makes a one-
(d) the total linear momentum of the two-particle system. dimensional elastic collision and bounces back along the
64. A railroad car is carrying sand and is rolling without friction same line. After the collision, the Ping-Pong ball has,
on the railroad tracks. There is a small hole at the bottom of compared with the bowling ball
the car and the sand is pouring out. As the sand pours out, (a) a larger magnitude of momentum and more kinetic energy
the speed and kinetic energy of the car will respectively (b) a smaller magnitude of momentum and more kinetic
(a) decrease, decrease. energy
(b) increase, decrease. (c) a larger magnitude of momentum and less kinetic energy
(c) remain constant, decrease. (d) a smaller magnitude of momentum and less kinetic energy
(d) increase, remain constant

MARK YOUR 60. 61. 62. 63. 64.


RESPONSE 65. 66. 67. 68. 69.
144 IIT-JEE PHYSICS Challenger
70. A body of mass m is thrown vertically upward into air with B
initial velocity v0. A constant force due to air resistance acts m
on body opposite to the direction of motion of the body.
The velocity of the body at a height h above the surface 1 2
during ascending is
3
Fh Fh
(a) v02 2 gh + ÷ (b) v02 2 gh + ÷
m 2m A
(a) W1> W2 > W3 (b) W1= W2 = W3
2 Fh (c) W1< W2 < W3 (d) W2> W1 > W3
(c) v02 2 gh + ÷ (d) v02 2 gh
m 73. For a completely inelastic two-body collision, the total
kinetic energy retained by the objects is the same as
71. An iron ball of mass m, suspended by a light inextensible
(a) the total kinetic energy before the collision
string of length from a fixed point O, is shifted by an angle
(b) the difference in the kinetic energies of the objects
0 as shown so as to strike the vertical wall perpendicularly.. before the collision
The maximum angle made by the string with vertical after
1 2
the first collision, if e is the coefficient of restitution, is (c) Mvcom , where M is the total mass and vcom is the
2
O velocity of the center of mass
//////////////////////////////////////// (d) the kinetic energy of the less massive body before the
///////////////////////////

collision
0
74. In a particular fission process of a uranium atom at rest,
fragments with mass numbers A1 and A2 are produced. A
good estimate of the ratio of kinetic energies of the
fragments, K1/K2 is
(a) A2/A1 (b) (A2/A1)2
(c) A1/A2 (d) (A1/A2)2
(a) sin–1 {1 – e2 (1 – cos 0 )} 75. An object of mass m is projected vertically upwards with a
speed of v0. At the same moment another object of mass M,
(b) cos–1 {1 – e2 (1 – cos 0 )} which is initially above the projected one, is dropped from a
(c) tan–1 {1 – e2 (1 – cos 0 )}
height of h. The two point like objects collide completely
inelastically, and they stick to each other. Find kinetic energy
(d) zero
(in J) of combined mass just before it hits the ground. (Given
72. If W1, W2 and W3 represent the work done in moving a : m = 1 kg, v0 = 20 m/s, M = 3 kg, h = 20m, g = 10 m/s2).
particle from A to B along three different paths 1,2 and 3 (a) 550 J (b) 650 J
respectively (as shown) in the gravitational field of a point (c) 450 J (d) 250 J
mass m, find the correct relation between W1, W2 and W3

MARK YOUR 70. 71. 72. 73. 74.


RESPONSE 75.

position where force is zero. Suppose potential energy at a distance


PASSAGE-1
r from centre of the field is given as
A B
In a conservative force field, we can find the component of force U(r) = – where A and B are positive constants.
from the potential energy at a point in the field. A positive force r2 r
means repulsion and a negative force means attraction. From the
given potential energy function U (r) we can find the equilibrium
WORK, POWER, ENERGY & CONSERVATION LAWS 145

1. The conclusion that can be drawn from the form of potential 5. Co-ordinate of centre of mass of ring when particle reach
energy given is back to point A for the first time
(a) the field is surely non-conservative (a) (10, 10) (b) (0, 0)
(b) the field is surely conservative (c) (20, 20) (d) (– 20, – 20).
(c) data insufficient to decide anything 6. Which of the following conclusion can be drawn
(d) for some range of ‘r’ the field is conservative and rest
(i) between t = 8 to 12 s particle is at rest
everywhere it is non-conservative
(ii) between t = 12 to 16 s particle is moving along +y
2. The work required to move the particle from equilibrium to
direction
infinity is
(iii) between t = 4 to 8 s particle is moving along +x direction
B 4B (iv) between t = 8 to 12 s ring is moving at 45° with +x
(a) (b)
4A A direction
Choose the correct alternatives
B2 4B 2
(c) (d) (a) (i), (ii), (iii) (b) (ii), (iii), (iv)
4A A
(c) (i), (iv) (d) (i), (ii), (iii), (iv)
3B 2
3. If the total energy of the particle is E = – and it is
16 A PASSAGE-3
known that the motion is radial only. Then the velocity is
zero at
Block A(1 kg) is placed on smooth horizontal surface and
A B connected with a block B (2 kg), as shown in the figure, by an
(a) (b)
B A inextensible string. A bullet of mass 250 gm, strikes the block A
horizontally with speed 200 m/s. The bullet penetrates through
3A 4A
(c) (d) the block A and comes out with velocity 100 m/s.
4B 3B
A m = 250gm speed = 200 m/s
PASSAGE-2 1 kg

y
B
5
v= m/s
2 B 2 kg
A x
7. Velocity of 2 kg block just after bullet comes out of block A
15
(a) 8 m/s (b) m/s
3
A ring of mass 200 gm and radius 10 m is placed on a smooth 25 25
horizontal surface with centre at the origin. A small particle of the (c) m/s (d) m / s.
3 7
5 8. Impulse produced by string on block B is
same mass as ring, is given velocity m/s from point A (very
2 (a) 50 N-s (b) 25 N-s
close to inner surface of the ring) towards point B (at t = 0). Initially 50 50
particle was not in contact with ring. Assume all collisions between (c) N-s (d) N-s
3 4
the ring and the particle as perfectly elastic.
4. Particle will collide with point A for the first time after a time 9. Max. displacement of block A in left direction is
interval of approximately (g = 10m/s2)
(a) 8 s (b) 16 s (a) 2.2 m (b) 3.2 m
(c) 12 s (d) 24 s. (c) 4.2 m (d) 5.2 m

MARK YOUR 1. 2. 3. 4. 5.
RESPONSE 6. 7. 8. 9.
146 IIT-JEE PHYSICS Challenger
A
PASSAGE-4
60°
A U shaped tube of mass 2m is placed on a smooth horizontal mg
surface. Two identical spherical balls each of mass m and of
diameter slightly less than the inner diameter of tube enters into
the tube with a velocity u as shown. (Assume no loss of energy B
anywhere and all collisions to be elastic). 13. The speed of the centre of mass of the rod will be, after the
blow,
B
m u g 3
(a) (b) g
2 2
g
A (c) g (d)
4
m u
14. The angular velocity of the rod about its centre of mass,
10. Speed of each spherical ball, just before their collision after the blow has been delivered, is
u 2u 6g 3g
(a) (b) (a) (b)
3 3 3

3u u 3 3g 3 3g
(c) (d) (c) (d)
2 2 2
11. At the time of collision, angle between direction of motion 15. The rod is observed to vibrate along the transverse direction,
of spherical ball A and B is, as observed in the ground frame, emitting sound of wavelength 0. The velocity of transverse
vibrations along the rod is vT and that of sound in air cS.
1 The number of nodes formed on the rod is
(a) 2 tan 1
2 (b) 2 tan 1
2 2 cS 2
(a) (b) .
vT 0
(c) tan 1 2 (d) 0

12. In the given situation the maximum velocity that the U– 1 4


1 1 cS 4
2 0 ÷
tube can gain (choose one of the given option) (c) (d) . 1÷
2 vT 0
u
(a) when spheres are about to collide.
2 PASSAGE-6
(b) 2u when spheres move in backward direction with speed
A small particle of mass m/10 is moving horizontally at a height of
u.
3R/2 from ground with velocity 10 m/s. A perfectly inelastic
(c) u, finally when sphere emerge from U-tube. collision occurs at point P of sphere of mass m placed on smooth
(d) some extra informations are required to calculate it. horizontal surface. The radius of sphere is R. (m = 10 kg and
R = 0.1 m) (Assume all surfaces to be smooth). Answer the
following questions.
PASSAGE-5
m/10 P
A uniform rod of mass m and length lying on a smooth horizontal 10 m/s R/2
plane, is struck sharply at one end by means of a hammer which m
delivers a sudden blow at one end of the rod, making an angle of R
60° with the rod. The blow delivers an impulse of mg to the rod.
Due to this blow, the rod undergoes translation, rotation and also B
vibrates. The effect of vibration can be neglected while the 16. Speed of particle just after collision is
translational and rotational motion are being analysed. (a) approx 5.0 m/s (b) approx 10 m/s
(c) approx. 15.0 m/s (d) approx 20.0 m/s

MARK YOUR 10. 11. 12. 13. 14.


RESPONSE 15. 16.
WORK, POWER, ENERGY & CONSERVATION LAWS 147

17. Speed of sphere just after collision is


(a) 27/43 m/s (b) 30/43 m/s Mallet v0 = 0
(c) 35/43 m/s (d) 40/43 m/s mass M
18. Angular speed of sphere just after collision is
(a) Zero (b) 2 rad/sec y
(c) 2.5 rad/sec (d) 3 rad/sec
v
PASSAGE-7 mass m
Stake
One of the forces acting on a certain particle depends on the particle’s Ground
f
position in the xy plane. This force F2 , expressed in newtons, is

given by the expression F2 = ( xy xˆ + xy yˆ ) (1 N/m 2 ) , where x 22. The resistance of the ground, assuming it to be constant
and the stake and mallet to stay together on impact is
and y are expressed in meters.
y (in m) M 2 gy
(a) ( M + m) g +
M +m d
B (0,1) (b)
C (1,1) M 2 gy
(b) (M m) g
M +m d
M 2 gy
(b) (c) (c) ( M + m) g +
(a) 2M + m d
M 2 2 gy
(d) ( M + m) g +
M +m d
(a) A (1,0) 23. Time the stake is in motion is
O (0,0) x (in m)
19. Calculate the work done for path OAC M m 2 M +m 2
(a) d (b) d
M gy M gy
1 2
(a) J (b) J
2 3 M +m 2 M +m 1
(c) d (d) d
1 2M gy 2M gy
(c) 1 J J
(d)
3 24. The kinetic energy lost at impact is
20. Calculate the work done for path OBC m m
(a) (b)
1 2 M +m 2M + m
(a) J (b) J
2 3 2m
(c) (d) None of these
1 M +m
(c) 1 J (d) J
3 PASSAGE-9
21. Calculate the work done for path OC
Block A is placed on wedge B at a height h above ground. Block and
1 2 the two wedges are all of same mass m. Neglect friction every where
(a) J (b) J
2 3 m
A
1
(c) 1 J (d) J
3
h
PASSAGE-8
m
m
A camper lets fall a heavy mallet of mass M from a height y upon B C
the top of a tent stake of mass m and drives it into the ground a
distance d (Figure).

MARK YOUR 17. 18. 19. 20. 21.


RESPONSE 22. 23. 24.
148 IIT-JEE PHYSICS Challenger
25. The velocity of B when A has slide down from it is
x
gh x
(a) gh (b)
2 A
A
gh t
(c) (d) None of these 2 t
B
2
26. The maximum height upto which block A rises on wedge C is
B
(a) h (b) h/2
(V) (VI)
(c) h/4 (d) None of these
27. The velocity of A when it has slide down to ground from
wedge C is 28. Which graphs pertain to physically possible explosions –
(a) II, IV and V (b) I, III, VI
gh (c) I, III, V (d) II, III, VI
(a) 0 (b)
2 29. Base on the above question, Match column A with the
column B
gh Column A Column B (Graph number)
(c) (d) None of these
4 (P) mA = mB I
(Q) mA > mB II
(R) mA < mB III
PASSAGE-10
IV
V
An initially stationary box on a frictionless floor explodes into
VI
two pieces, piece A with mass mA and piece B with mass mB. Two
pieces then move across the floor along x-axis. Graph of position (a) P-VI, Q-III, R-I (b) P-II, Q-V, R-IV
versus time for the two pieces are given. (c) P-II, Q-IV, R-V (d) P-VI, Q-II, R-IV
30. If following graphs are possible then, in which of the
x following cases external impulse must be acting on the box.
x (a) II (b) IV
A
A (c) V (d) VI
t
B
2 B PASSAGE-11
t
In the case of a moving object making a head-on collision with
(I) (II) another object of equal mass at rest we have momentum
conservation
Mv0 = Mv1 + Mv2 .......... (1)
x x A
and for elastic collision, energy conservation
1 1 1
2 Mv02 = Mv12 + Mv 22 .......... (2)
t t 2 2 2
In eq. (1) and (2), v0 is the incoming speed, v1 is the speed of the
A B incident object after the collision, and v2 is the final speed of the
B initially stationary object. Eliminating common factors from both
sides of eq. (1) and (2) yields.
(III) (IV)
v1 + v2 = v0 .......... (3)

and v12 + v 22 = v 02 .......... (4)

MARK YOUR 25. 26. 27. 28. 29.


RESPONSE 30.
WORK, POWER, ENERGY & CONSERVATION LAWS 149

v2 Unique physical
solution v2 v2
v1 + v2 = v0 v0

v2 = v1

v0 (c) v1(d) v1
v1

33. In a case where both objects are of mass 1 kg, and 1st object
is travelling with velocity of 2 m/s. Their v2 – v1 graph is
shown here. What is the velocity v2 after the collision?
v12 + v22 = v02
v2
The v 1 – v 2 graph for a two-body collision with M1= M 2
Eq. (3) and (4) are two simultaneous equations in two unknowns, 2 (0.5, 1.5)
which are readily solved. Rather than solve them algebraically it is A
very helpful to plot these two equations in a two-dimensional v1
(1.5, 0.5)
– v2 graph. This plot is shown in figure the circle of radius v0 B
arising from energy conservation and the straight line from v1
2
momentum conservation. The two points of intersection provide
two mathematical solutions to eq. (3) and (4). One of these
solutions
(v1 = v0, v2 = 0) is prohibited on physical grounds since it would
required object 1 to pass through object 2 in the collision. The (a) 1.5 m/s (b) 0.5 m/s
other intersection point (v1 = 0, v2 = v0) provides the well known (c) 2 m/s (d) zero
and unique physical solution.
31. For an inelastic collision in a similar situation, momentum is
still conserved, but the kinetic energy of the system reduces. PASSAGE-12
In this case
(a) The momentum conservation equation will yield a
different straight line, but the energy equation will yield A spring having initial unstretched length 0 is lying on a smooth
a circle of smaller radius. table. It’s one end is fixed and the other one is fastened to a small
(b) The momentum conservation equation will yield the particle of mass m. The particle is imparted an initial speed v0
same straight line, but the energy equation will yield a horizontally in a direction perpendicular to the spring. In the course
circle of larger radius.
of the motion in horizontal plane, the maximum elongation of the
(c) The momentum conservation equation will yield a
different straight line, but the energy equation will yield spring is = 0/10. (Given m = 2.1 kg, 0 = 21 cm, v0 = 1.1 m/s)
a circle of larger radius.
(d) The momentum conservation equation will yield the
same straight line, but the energy equation will yield a v0
circle of smaller radius.
32. If the collision is completely inelastic, how can this m
represented on v1 – v2 graph
v2
v2
34. In the course of motion, which of the quantities relating to
spring-block system are conserved ?
(a) kinetic energy (b) momentum
(a) v1 (b) v1 (c) angular momentum (d) potential energy

MARK YOUR
31. 32. 33. 34.
RESPONSE
150 IIT-JEE PHYSICS Challenger
35. Which of the following is correct about initial situation and
situation at maximum elongation ? (3e 1) 2 (3e 1) 2
(a) h (b) h
(a) the orientation of spring in both positions should be 16 4
perpendicular to each other (e 1) 2
(b) the velocity at maximum extension should be zero (c) h (d) (3e – 1)2 h
4
(c) the velocity at maximum extensions as well as at initial
39. If the pile meets a constant resistive force F from the material
position should be perpendicular to spring.
in which it is lodged, calculate how far into the material it
(d) acceleration should be zero at the maximum extensions penetrates before coming to rest.
as well as at initial position.
36. Student A : At maximum extension mv' ( 0 + ) = mv0 0. m (1 + e)2 gh 3 (1 + e) 2 mgh
(a) (b)
16 ( F 3mg ) ( F 3mg )
2
mv
Student B : At maximum extension k =
0 6 (1 + e) 2 mgh 3 (1 + e) 2 mgh
(c) (d)
16 ( F 3mg ) 16 ( F 3mg )
1 2 1 1
Student C : mv0 = mv + k
2 2
.
2 2 2
where v' is velocity at instant of maximum extension. PASSAGE-14
(a) Only student A and B are correct
(b) Only student A and C are correct A crash test car of mass 1000 kg moving at constant speed of 12m/
(c) Only student B and C are correct s collides completely inelastically with an object of mass M at time
(d) All are correct t = 0. The object was initially at rest. The speed v in m/s of car-
object system, after the collision is given as a function of time t in
seconds by the expression
PASSAGE-13
8
v=
1 + 5t
A particle of a mass m at a height h above top of a pile which has
40. Assuming an initial position of x = 0, the position of the car-
a mass 3m is released from rest as shown in figure.
object system after the collision as a function of time t is
m
given by
8
h (a) ln (1 + 5t) (b) 8 ln (1 + 5t)
5
5
3m (c) ln (1 + 5t) (d) ln (1 + 5t)
8
41. The resisting force on the car-object system after the collision
as a function of time t is given by
37. What will be the possible range of the coefficient of
40000 60000
restitution e if the particle has to rebound ? (a) (b)
(1 + 5t ) 2
(1 + 5t ) 2
1
(a) 1 > e >
3 6000
(c) (d) None of these
(1 + 5t ) 2
1
(b) 0 < e < 42. The impulse delivered to the car-object system from t = 0 to
3
t = 2.0s is
(c) It will rebound for any value of e
(d) It will not rebound for any value of e 80000
(a) N-s (b) 80000 N-s
38. To what height does the particle has rebound if the coefficient 11
of restitution between the pile and the particle is e.
120000
(e is in the range as calculated in above question) (c) 120000 N-s (d) N-s
11

MARK YOUR 35. 36. 37. 38. 39.


RESPONSE 40. 41. 42.
WORK, POWER, ENERGY & CONSERVATION LAWS 151

PASSAGE-15 a a

Block, Ma = 0.50kg
Block, Ms = 3.0kg (c) x (d) x
4m/s

48. Speed at mean position is

3 2
A small block of mass Ma = 0.50 kg is placed on a long slab of (a) m/s (b) m/s
10 5
mass Ms = 3.0 kg as shown above. Initially, the slab is at rest and
the block has a speed v0 of 4.0 m/s to the right. The coefficient of 7
kinetic friction between the block and the slab is 0.20, and there is (c) m/s (d) None of these
10
no friction between the slab and the horizontal surface on which
it moves.
At some moment later, before the block reaches the right end of PASSAGE-17
the slab, both the block and the slab attain identical speeds vf,
(Take g = 10 m/s2)
43. The value of vf is A system consists of block A and B each of mass m connected by
(a) 0.57 m/s (b) 0.50 m/s a light spring as shown with block B in contact with a wall. The
(c) 0.52 m/s (d) 0.47 m/s block A compresses the spring by 3mg/k from natural length of
44. The distance the slab has travelled at the moment it reaches spring and then released from rest. Neglect friction any where.
vf is
(a) 0.40 m (b) 0.49m
(c) 0.65m (d) 0.55m 3mg/k
45. The work done by friction on the slab from the beginning of k
its motion until it reaches vf is
B A
(a) 0.40 m (b) 0.49 m
(c) 0.65 m (d) 0.55 m 49. Acceleration of centre of mass of system comprising A and
B just after A is released is
PASSAGE-16 (a) 0 (b) 3g/2
(c) 3g (d) None of these
Two rubber threads of spring constant 100 N/m each is connected 50. Velocity of centre of mass of system comprising A and B
to the two sides of a body of mass 1 kg standing on a horizontal when block B just loses contact with the wall
frictionless surface. The rubber threads are stretched by a length
m 3g m
of = 1 cm and their ends are fixed. Then the body is displaced (a) 3g (b)
by a distance of 2 towards one of the fixing points and it is k 2 k
released.
46. The initial acceleration of the mass is m
(c) 2g (d) None of these
(a) 1 m/s2 (b) 2 1 m/s2 k
(c) 3 m/s 2 (d) None of these 51. Maximum extension in the spring after system loses contact
47. Acceleration-position graph of the mass is best represented with wall
by
a a 3mg 3mg
(a) (b)
2k 2k

x (b) 3mg
(a) x (c) (d) None of these
2k

MARK YOUR 43. 44. 45. 46. 47.


RESPONSE 48. 49. 50. 51.
152 IIT-JEE PHYSICS Challenger

1. A particle of mass m strikes a wedge of mass M horizontally Statement - 2 : On a smooth horizontal floor we can not
as shown in the figure. walk.
6. Statement - 1 : Sum of work done by the Newton’s 3rd law
m pair internal forces, acting between two
M particles may be zero.
Statement - 2 : If two particles undergo same
displacement then work done by Newton’s
Statement - 1 : If collision is perfectly inelastic then, it can 3rd law pair forces on them is of opposite
be concluded that the particle sticks to the sign and equal magnitude.
wedge. 7. Statement - 1 : If resultant of external force on a system of
Statement - 2 : In perfectly inelastic collision velocity of two spheres is always zero and net
both bodies is same along common normal momentum of system of both spheres
just after collision. (undergoing pure translational motion)
2. Statement - 1 : Centre of mass of a body in pure rolling on before undergoing head-on collision is
a horizontal surface always moves in a zero, then the minimum kinetic energy of
straight line. system during collision is zero.
Statement - 2 : Centre of mass of a body may be inside or Statement - 2 : During a head-on collision between two
outside a body. bodies, at instant of maximum deformation
velocities of both bodies are equal. Also if
3. As shown in the figure, a uniform sphere is rolling on a
horizontal surface without slipping, under the action of a Fext = 0 then p system = constant .
horizontal force F.
8. Statement - 1 : A particle moves along a straight line with
F constant velocity. Now a constant non-
zero force is applied on the particle in
direction opposite to its initial velocity.
After the force is applied, the net work
done by this force may be zero in certain
Statement - 1 : Power developed due to friction force is time intervals.
zero. Statement - 2 : The work done by force acting on a
Statement - 2 : Power developed by gravity force is non- particle is zero in any time interval if the
zero. force is always perpendicular to velocity
4. Statement - 1 : If bodies slide down on a smooth inclined of the particle.
plane without rolling then all bodies reach 9. Statement - 1 : In an elastic collision between two bodies,
the bottom simultaneously. the relative speed of the bodies after
Statement - 2 : Acceleration of all bodies are equal and collision is equal to the relative speed
independent of the shape. before the collision.
5. Statement - 1 : When we run on rough ground without Statement - 2 : In an elastic collision, the linear momentum
slipping our kinetic energy increases due of the system is conserved.
to positive work done by friction which
accelerates us.

MARK YOUR 1. 2. 3. 4. 5.
RESPONSE 6. 7. 8. 9.
WORK, POWER, ENERGY & CONSERVATION LAWS 153

10. Statement - 1 : One end of ideal massless spring is surface after moving through a certain
connected to fixed vertical wall and other distance. The surface is now tilted to an
end to a block of mass m initially at rest on angle of 30° with the horizontal and the
smooth horizontal surface. The spring is same block is made to go up on the surface
initially in natural length. Now a constant with the same initial velocity v. The
decrease in the mechanical energy in the
horizontal force F acts on block as shown.
second situation is smaller than that in the
Then the maximum extension in spring is
first situation.
equal to maximum compression in spring.
Statement - 2 : The coefficient of friction between the
////////////////////

block and the surface decreases with the


increase in the angle of inclination.
13. Statement - 1 : In an elastic collision in one dimension
m F
between two bodies, total momentum
/////////////////////////////////////////////// remains the same before, during and after
Statement - 2 : To compress and to expand an ideal spring the collision.
by equal amount, same work is to be done Statement - 2 : In an elastic collision in one dimension
on spring. between two bodies, total kinetic energy
11. Statement - 1 : Internal forces can change total linear remains the same before, during and after
momentum of system. the collision. [Assume external forces are
Statement - 2 : Due to work of internal force kinetic energy absent in both the above statements.
of system may change. 14. Statement - 1 : Some work (non zero) has to be done on a
12. Statement - 1 : A block of mass m starts moving on a rough moving particle to change its momentum.
horizontal surface with a velocity v. It stops Statement - 2 : To change momentum of a particle a non-
due to friction between the block and the zero net force should act on it.

MARK YOUR
10. 11. 12. 13. 14.
RESPONSE

1. The curve shown is for potential energy U of the interaction (b) Repulsive force acts between objects during segment
between two objects on the distance r separating them. (ab)
Choose the correct options for the given objects. (c) Attraction force acts between objects during segment
U (bc)
f
(d) Attraction force acts between objects during segment
(abc)

d 2. A force F = K ( yiˆ + xjˆ) (where K is a positive constant)


acts on a particle moving in the xy plane. Starting from the
e
origin, the particle is taken along the positive x axis to the
a c r point (a, 0), and then parallel to the y axis to the point (a, a),
b The total work done by the force F on the particle is
(a) Position of stable equilibrium for two objects in curve (a) – 2Ka2 (b) 2Ka2
is (b). (c) – Ka2 (d) Ka 2

MARK YOUR
1. 2.
RESPONSE
154 IIT-JEE PHYSICS Challenger
3. Which of the following statements about momentum and (b) the kinetic energy of the A-B system, at maximum
kinetic energy are true ? compression of the spring, is mv2/4.
(a) The momentum depends on the direction of motion,
(c) the maximum compression of the spring is v (m / K )
the kinetic energy does not
(b) The kinetic energy depends on the speed, the (d) the maximum compression of the spring is v (m / 2K )
momentum does not
(c) Whenever there is a net force on an object, both its 7. An elevator is rising at constant speed. Consider the
following statements and pick up the statements which is/
momentum and kinetic energy must change.
are correct
(d) The total amount of momentum or kinetic energy in a
system is found by adding the contributions from each Cable
object in the system.
4. A shell is fired from a cannon with a velocity v (m/sec.) at an
angle with the horizontal direction. At the highest point Elevator
in its path it explodes into two pieces of equal mass. One of
the pieces retraces its path to the cannon and the speed (in
m/sec.) of the other piece immediately after the explosion is (a) The force exerted by cable on elevator is constant
(b) The kinetic energy of the elevator is constant.
(c) The gravitational potential energy of the Earth-elevator
(a) 3v cos (b) 2v cos
system is constant
3 3 (d) The mechanical energy of the Earth-elevator system is
(c) v cos (d) v cos constant.
2 2
8. A load W is suspended from a self-propelled crane by a
5. As shown in figure, a smooth rod is mounted horizontally cable of length d (figure a). The crane and load are moving
just above a tabletop. A 10 kg collar, which is able to slide on at a constant speed v0. The crane is stopped by a number
the rod with negligible friction is fastened to a spring whose and the load on the cable swings out, as shown in figure b.
other end is attached to a pivot at O. The spring has Choose the correct option(s).
negligible mass, a relaxed length of 10cm, and a spring
constant of 500 N/m. The collar is released from rest at point
d
S. Choose the correct option(s). v0
10cm 15cm

B A S

(a)
20cm

(a) Velocity as collar passes point A is 0.791m/s


(b) Velocity as collar passes point B is 0.601 m/s
(c) Velocity as collar passes point A is 0.601m/s
(d) Velocity as collar passes point B is 0.791 m/s
6. Two blocks A and B, each of mass m, are connected by a
massless spring of natural length L and spring constant K. (b)
The blocks are initially resting on a smooth horizontal floor
with the spring at its natural length, as shown in fig.. A third 1 v0
(a) Angle through which the load swings is 2sin ÷
identical block C, also of mass m, moves on the floor with a 2 gd
speed v along the line joining A and B, and collides elastically (b) If the angle is 60° and d = 5m then the initial speed of
with A. Then the crane is 9m/s
v (c) If the angle is 60° and d = 5m then the initial speed of
L the crane is 7m/s
C A B
1 v0
(d) Angle through which the load swings is 2 tan ÷
(a) the kinetic energy of the A-B system, at maximum 2 gd
compression of the spring, is zero.

MARK YOUR 3. 4. 5. 6. 7.
RESPONSE 8.
WORK, POWER, ENERGY & CONSERVATION LAWS 155

9. Choose the correct statement (s) regarding inertial frames. Y


(a) Impulse is independent of frame of reference a
C
(b) Momentum depends upon frame of reference D

(c) Work done by a force depends on frame of reference


(d) Change in kinetic energy is independent of frame of a
reference
10. A ball hits the floor and rebounds after an inelastic collision.
In this case
(a) the momentum of the ball just after the collision is the B X
A
same as that just before the collision.
(a) F is a conservative force
(b) the mechanical energy of the ball remains the same in
the collision a5
(b) Work done for path ABC is (J )
(c) the total momentum of the ball and the earth is 3
conserved
a5
(d) the total energy of the ball and the earth is conserved (c) Work done for path ADC is (J )
3
11. A sphere of mass m moving with speed v collides elastically
with a stationary sphere of mass km, where k is some 2a5
(d) Work done for path AC is (J )
numerical factor. Let their resulting velocities be v1 and v2 3
respectively. Then pick up the correct statement(s) 15. A uniform chain of length L and mass M is lying on a smooth
table and one third of its length is hanging vertically down
(a) If the collision is oblique, v2 must be perpendicular to over the edge of the table. If g is acceleration due to gravity,
2v1 (k 1)v2 the work required to pull the hanging part on to the table is
(a) MgL (b) MgL/3
(b) If the collision is oblique, v2 must be perpendicular to (c) MgL/9 (d) MgL/18
16. When two blocks of different masses connected by a
v1 (2 k 1)v2
stretched spring (as shown) start moving from rest towards
1 k each other under mutual interaction, then pick up the correct
(c) If the collision is head-on, v1 = ÷ v2 alternative or alternatives.
2
k
(d) If the collision is head-on, v1 (1 2 k ) v2 m1 m2
12. An escalator is moving down with constant speed. You are /////////////////////////////////////////
Smooth horizontal floor
moving on it such that you remain at rest with respect to
ground. Choose correct statement(s) from ground frame. (a) Their velocities are equal and opposite
(b) Their acceleration are equal and opposite
(a) Work done by you is zero (c) The force acting on them are equal and opposite
(b) Work done by escalator on you is zero (d) Their momentum are equal and opposite
(c) Work done by gravity on you is zero 17. A stone tied to a string of length L is whirled in a vertical
(d) Work done by escalator on man is negative circle with the other end of the string at the centre. At a
13. A body is moved along a straight line by a machine delivering certain instant of time, the stone is at its lowest position,
constant power. The distance moved by the body in time t and has a speed u. The magnitude of the change in its
is proportional to velocity as it reaches a position where the string is horizontal
(a) t 1/2 (b) t 3/4 is
(c) t 3/2 (d) t 2 (a) u2 2 gL (b) 2gL
14. A force F = x y i + x y j ( N ) acts on a particle which
2 2ˆ 2 2ˆ
(c) u2 gL (d) 2(u 2 gL)
moves in the XY plane. Choose the correct option(s)

MARK YOUR 9. 10. 11. 12. 13.


RESPONSE 14. 15. 16. 17.
156 IIT-JEE PHYSICS Challenger
18. Among the following choose the correct statement(s)
P v
(a) Work done by a centripetal force is zero
A
(b) Work done by a conservative force is independent of 60m B
the path
45°
(c) The area under the force-displacement curve gives
work
(a) v = 20 m/s (b) v = 10 m/s
(d) The product of pressure and volume has same
(c) AB = 80 2 m (d) AB = 80 m
dimensions as that of force
22. The diagram given shows how the net interaction force
19. A smooth sphere A of mass m collides elastically with an between two particles A and B is related to the distance
identical sphere B at rest. The velocity of A before collision between them, when the distance between them varies from
is 8m/s in a direction making 60° with the line joining the x1 to x4. Then
centres at the time of impact. Which of the following is/are
possible ? Attraction
(a) The sphere A comes to rest after collision.
(b) The sphere B will move with a speed of 8m/s after
O x1
collision
x2 x3 x4
(c) The directions of motion of A and B after collision are
at right angles
(d) The speed of B after collision is 4m/s Repulsion
20. A particle is acted upon by a force of constant magnitude
which is always perpendicular to the velocity of the particle. (a) the potential energy of the system increases from x1 to
The motion of the particle takes place in a plane. It follows x2
that : (b) potential energy of the system increases from x2 to x3
(c) potential energy of the system increases from x3 to x4
(a) its velocity is constant
(d) kinetic energy increases from x1 to x2 and decreases
(b) its acceleration is constant from x2 to x3
(c) its kinetic energy is constant. 23. A ball of mass 3 kg moving with a velocity of 3m/s has an
(d) it moves in a circular path. indirect collision with a ball of equal mass moving with a
velocity of 4.5 m/s. The velocity of first ball makes an angle
21. A particle is to be projected horizontally with velocity v of 30° with the line of impact while the velocity of second
from a point P, which is 60m above the foot of a plane inclined ball makes an angle of 60° with the line of impact from the
at angle 45° with horizontal as shown in figure. The particle other side. If coefficient of restitution is 0.9, choose the
hits the plane perpendicularly at A. After rebound from correct option(s).
inclined plane it again hits at B. If coefficient of restitution (a) The final velocity of ball A is 2.51m/sec
(b) The final velocity of ball B is 4.55m/sec
1
between particle and plane is , then (c) The final velocity of ball A is 1.55m/sec
2 (d) The final velocity of ball B is 9.75m/sec

MARK YOUR 18. 19. 20. 21. 22.


RESPONSE 23.
WORK, POWER, ENERGY & CONSERVATION LAWS 157

1. A body initially moving towards the right explodes into two pieces 1 and 2. The magnitudes of v1 and v2 (the final velocities)
are completely arbitrary. Directions of motion of the pieces are shown in Column I and possible mass ratios are shown in
Column II.
Column I Column II

v1

1
(A) (p) m1 > m2
2
v2

1 v1
(B) (q) m1 = m2
2
v2

v1

1
(C) (r) m1 < m2
2

v2

v1

1
(D) (s) impossible for any masses
2
v2

1.

MARK YOUR
RESPONSE
158 IIT-JEE PHYSICS Challenger
2. Initially spring are in natural length. An application of external varying force F causes the block to move slowly distance x
towards wall on smooth floor.

\\\\\\\\\\\\\\\\
S1 S2
k1 k2
F
Smooth
//////////////////////////////////////////////////////////
Column I Column II
(A) Work done by S2 on block (p) Zero

1 k1k 2
(B) Work done by S2 on S1 (q) x2
2 k1 + k 2 ÷

1 k1k2
(C) Work done by F on block (r) x2
2 k1 + k2 ÷

1 k1k 22 x 2
(D) Work done by S1 on wall (s)
2 ( k1 + k 2 ) 2
3. A block of mass m is placed on wedge also of mass m. The wedge is placed on smooth horizontal fixed surface. One end of a light
spring is connected to block and the other end to a light support S rigidly fixed to wedge as shown. Friction is absent every
where. The system is initially released from rest with spring unstressed. Match statements in column I with corresponding
statements in column II.

S m

///////////////////////////////////////////
Column I Column II
(A) At the instant compression in spring is maximum (p) speed of block is zero
(B) At the instant spring has natural length, that is, it is (q) speed of block is non-zero
unstressed
(C) At the instant net force on wedge is zero (r) speed of block is maximum
(D) At the instant elastic potential energy stored in (s) speed of block is minimum
spring is least
4. Match the columns
Column I Column II
(A) Collision of two light nuclei to form a heavier nucleus (p) Elastic collision
(B) Speeding bullet getting embedded in a wooden plank (q) Perfectly inelastic collision
(C) Collision of neutron with heavy unstable nucleus (r) Nuclear fission
(D) Collision in which there is no loss of external kinetic (s) Nuclear fusion.
energy

2. 3. 4.

MARK YOUR
RESPONSE
WORK, POWER, ENERGY & CONSERVATION LAWS 159

5. A block suspended from a spring is released from rest when spring is unstretched. ‘x’ represents stretch in spring. Select the
appropriate graph taking quantities in column I as y-axis.
Column I Column II
(A) The KE of block (p) 1
4

(B) The work done on the block by gravity (q) 2


1 2
(C) The magnitude of work done on the block by spring (r) 3
3 X
(D) The total mechanical energy of block-earth-spring system (s) 4 xmax

6. Two blocks A and B of mass m and 2m respectively are connected by a massless spring of spring constant k. This system lies over
a smooth horizontal surface. At t = 0, the block A has velocity u towards right as shown while the speed of block B is zero, and
the length of spring is equal to its natural length at that instant.
B k A
2m m u
///////////////////////////////////////////////
Smooth horizontal surface
Column I Column II
(A) The velocity of block A (p) can never be zero
(B) The velocity of block B (q) may be zero at certain instant of time
(C) The kinetic energy of system of two blocks (r) is minimum at maximum compression of spring
(D) The potential energy of spring (s) is maximum at maximum compression of spring
7. Match entries of column I with most appropriate entry of column II.
Column I Column II
(A) Work done by all forces (p) KE
(B) Work done against internal conservative forces (q) PE
(C) Work done by external forces (r) Total ME
(s) Total energy

5. 6. 7.

MARK YOUR
RESPONSE
160 IIT-JEE PHYSICS Challenger
1. A ball of mass 100 gm is projected vertically upwards from 4. A rod AB of mass M and length L is lying on a horizontal
the ground with a velocity of 49 m/sec. At the same time frictionless surface. A particle of mass m travelling along
another identical ball is dropped from a height of 98 m to the surface hits the end ‘A’ of the rod with a velocity v0 in a
fall freely along the same path as that followed by the first direction perpendicular to AB. The collision is elastic. After
ball. After some time the two balls collide and stick together the collision the particle comes to rest. Find the ratio m/M.
and finally fall to the ground. Find the time of flight
(in second) of the masses. 5. A car P is moving with a uniform speed of 5 3 m/s towards
2. An object of mass 5 kg is projected with a velocity of 20 m/s a carriage of mass 9 kg at rest kept on the rails at a point B
at an angle of 60° to the horizontal. At the highest point of as shown in figure. The height AC is 120 m. Cannon balls
its path the projectile explodes and breaks up into two of 1 kg are fired from the car with an initial velocity 100 m/
fragments of masses 1 kg and 4 kg. The fragments separate s at an angle 30o with the horizontal. The first cannon ball
horizontally after the explosion. The explosion releases hits the stationary carriage after a time t0 (in second) and
internal energy such that the kinetic energy of the system sticks to it. Determine t0.
at the highest point is doubled. Calculate the separation
(in meter) between the two fragments when they reach the
ground.
C
3. A cart is moving along + x direction with a velocity of 4 m/s. P
A person on the cart throws a stone with a velocity of 6 m/s
relative to himself. In the frame of reference of the cart the A
B
stone is thrown in y-z plane making an angle of 30° with
vertical z-axis. At the highest point of its trajectory, the stone 6. A uniform chain of mass M (= 1 kg) and length L (= 10 cm)
hits an object of equal mass hung vertically from the branch lies on a frictionless table with length l0 (= 6 cm) hanging
of a tree by means of a string of length L. A completely over the edge. The chain begins to slide down. What is the
inelastic collision occurs, in which the stone gets embedded speed (in m/s) with which the chain slides away from the
in the object. Determine the length L of the string (in meter) edge ?
such that the tension in the string becomes zero when the (Take g = 1000 cm/s2)
string becomes horizontal during the subsequent motion of 7. A cyclist moves against a resistance to motion which is
the combined mass. proportional to his speed. At a power output of 75W he has
a maximum speed of 5 m/s on a level road. If the cyclist and
his machine together weigh 800 N, find the maximum speed
(in m/s) he reaches when travelling down a hill inclined
at sin–1 (1/40) to the horizontal when he is working at the
rate of 25W.

1. 2. 3. 4.

MARK
YOUR
RESPONSE 5. 6. 7.
WORK, POWER, ENERGY & CONSERVATION LAWS 161

1 (a) 14 (c) 27 (c) 40 (a) 53 (d) 66 (b)


2 (a) 15 (c) 28 (a) 41 (c) 54 (d) 67 (d)
3 (c) 16 (c) 29 (d) 42 (c) 55 (b) 68 (b)
4 (b) 17 (c) 30 (a) 43 (a) 56 (b) 69 (b)
5 (d) 18 (b) 31 (b) 44 (c) 57 (c) 70 (c)
6 (c) 19 (c) 32 (a) 45 (a) 58 (c) 71 (b)
7 (d) 20 (d) 33 (b) 46 (d) 59 (a) 72 (b)
8 (a) 21 (c) 34 (a) 47 (b) 60 (d) 73 (c)
9 (a) 22 (c) 35 (a) 48 (b) 61 (a) 74 (a)
10 (c). 23 (c) 36 (b) 49 (d) 62 (b) 75 (b)
11 (b) 24 (a) 37 (b) 50 (c) 63 (b)
12 (b) 25 (a) 38 (b) 51 (a) 64 (c).
13 (c) 26 (d) 39 (b) 52 (a) 65 (b)

1 (b) 10 (c) 19 (a) 28 (a) 37 (a) 46 (c)


2 (c) 11 (a) 20 (a) 29 (b) 38 (a) 47 (a)
3 (d) 12 (c) 21 (b) 30 (d) 39 (d) 48 (c)
4 (b) 13 (c) 22 (a) 31 (d) 40 (a) 49 (b)
5 (c) 14 (c) 23 (b) 32 (a) 41 (b) 50 (b)
6 (d) 15 (b) 24 (a) 33 (a) 42 (d) 51 (a)
7 (c) 16 (a) 25 (a) 34 (c) 43 (a)
8 (c) 17 (b) 26 (c) 35 (c) 44 (b)
9 (d) 18 (a) 27 (a) 36 (b) 45 (b)

1 (d) 4 (a) 7 (a) 10 (d) 13 (c)


2 (d) 5 (d) 8 (b) 11 (d) 14 (d).
3 (c) 6 (a) 9 (b) 12 (c)

1 (a, b, c) 5 (a, b) 9 (a, b, c) 13 (c) 17 (d) 21 (a, c)


2 (c) 6 (b, d) 10 (c, d) 14 (b, c, d) 18 (a, b, c) 22 (b, c, d)
3 (a, d) 7 (a, b) 11 (a, c) 15 (d) 19 (c, d) 23 (a, b)
4 (a) 8 (a, c) 12 (c, d) 16 (c, d) 20 (c, d)

1. A-s; B-s; C-p, q, r; D-p, q, r 2. A-q; B-s; C-r; D-p 3. A-p, s; B-p, s; C-q, r; D-p, s
4. A-q, s; B-q; C-r; D-p, r, s 5. A-r; B-p; C-q; D-s 6. A-q; B-q; C-p, r; D-q, s
7. A-p; B-q; C-s

1 6.53 2 44.2 3 0.318 4 0.25 5 12 6 28.3 7 7.7


162 IIT-JEE PHYSICS Challenger

1. (a) Area under P-x graphs


vdm
dv 4. (b) Instantaneous force = mg where mg is the force
pdx m vdx dt
=
dt needed for supporting the weight of already hanging
v v vdm
2 mv 3 10 section of the rope and is the force needed to
= mv dv (v 3 1) dt
1
3 7 3
1 supply momentum to the portion of the rope which is
1 vdm d
From graph : area = (2 4) 10 30 to be pulled up v. ( ) v 2 , a constant.
2 dt dt
10 The force mg is maximum when the whole rope has
(v3 1) 30 been pulled up, (mg)max ( )g
7 3
v = 4 m/s Now power due to the force mg = mg .v
2. (a) Maximum chance of slipping occurs when spring is While power due to force
maximum compressed. At this moment, as force exerted
by the spring is maximum, acceleration of the system is dm dm v 2 dm
v v. v
maximum. Hence maximum friction force is required at dt dt 2 dt
this moment.
By W/E theorem where v = average velocity of mass dm which is pulled
off the ground in the time interval at dt = v/2.
1 1
( M + m )v 2 = kx m2
2 2 Hence Fmax g v 2 and

( M + m )v 2 1 3
xm = Pmax gv v
k 2
kxm 5. (d) The velocity of particle after falling through height h
Now for upper block am =
M +m u 2 gh ...(i)
Force on upper block is provided by the friction force.
After first rebounding, the velocity of ball is eu and
kxm .m after attaining maximum height it will come to the ground
Therefore , mg
M +m with same velocity eu. So, after second rebounding its
velocity will be e2u.
M +m
For limiting values v g Similarly, after third fourth ... etc reboundings its
k
velocities will be e2u, e4u, ... etc.
Using values vmax imum = 20 cm / s Since, it first rebounds with velocity eu so if it attains
3. (c) Initial KE. = 0, Initial P.E. = 0 height h then from
When the rope is just pulled off the table, v2 = u2 – 2gh
1 0 = e2u2 – 2gh1
final K.E. = ( )v 2 , final P.E. = ( ) g / 2
2
e2u 2 e 2 2gh
time taken = t = / v or h1 e 2 h [from Eq. (i)]
2g 2g
net change in energy
Average power = The same height the ball travels while approaching
time ground. Now, it rebounds with velocity e2u so if it
1 attains a height h2 then
v2 g /2
2 1 3 vg 0 = e4u2 – 2gh2
= v +
/v 2 2 or h2 = e4h
WORK, POWER, ENERGY & CONSERVATION LAWS 163

The similar process will follow for further reboundings Number of heartbeats per minute = 60
Hence, the total distance travelled by the practice before Time taken for one hearbeat = 1 sec.
it stops rebounding. Word done
h 2 h1 2h2 ... Power output = Time taken = 1.07 W

h 2e 2 h 2e 4 h ... 1 mM
10. (c) v2 E
h 2e2 h 1 e2 e4 ... 2 M m

1 2( M
m) E
h 2e 2 h v .
1 e2 Mm
11. (b) Components of velocity before and after collision
2 2 parallel to the plane are equal, so
2e 1 e
h 1 h v sin 60° = u sin 30° .......(1)
2
1 e 1 e2
Components of velocity normal to the plane are related
6. (c) The work done by man is negative of magnitude of to each other
decreases in potential energy of chain v cos 60° = e u (cos 30°) ........(2)
Dividing (2) by (1),
cot 60°
cot 60° = e cot 30° e=
cot 30°
1
L/2 3 1
e e
L/4 3 3
12. (b) From law of conservation of momentum, before collision
and after collision linear momentum (p) will be same or
L m L L
U mg g 3mg initial momentum = final momentum.
2 2 4 8
p2
3m g L E=
W 2m
8 According to question,
7. (d) Final velocity
E1 p12 2 m2 E1 m2
4 4 4 =
F 2t 2t 2 E2 2 m1 p22 E2 m1 [p1 = p2]
= dt = dt = = 16 ms–1
m 1 2 13. (c) Let xA and xB be the position of ends A and B at time t
0 0 0
from the block, then stretched length of the spring will
Hence, work done = change in kinetic energy be
1 = x A – xB
1 16 2 0 = 128 J 2
2 and so the stretch
8. (a) Elastic potential energy = 2 – 1 = ( x A – xB ) – 1
= gravitational potential energy.
By the conservation of energy, we have ( 1 natural length of the spring)
Elastic potential energy = Gravitational potential energy 1 1
So, U k 2
= k[( x A – xB ) – 1]
2
1 2 2 2
kx = m gh,
2 dx A dxB
= k .2 ( x A – xB – )
dU 1
P=
dt ÷
where h is the height attained by the shot. 1 –
dt 2 dt
kx 2 P = F (vA – vB)
h .
2 mg P
F=
9. (a) Work done = P V v A – vB
= (100 mm of Hg) × (80 ml)
= (100 × 10–3 × 13, 600 × 9.5 Nm–2) × F P 20
= = =
(80 × 10–3 × 10–3 l) k (v A – vB )k (4 – 2) 100
= 1.07 J = 0.1 m = 10 cm
164 IIT-JEE PHYSICS Challenger
14. (c) Just before collision velocity of the particle is

v = ( 130 g ) – 2 g = 128 g
2
3
16
(
u1 – u 2 )2 = 12 murel2 (1 – e2 )
e = 0.5
v 64 2 g = 8 2 g
20. (d) m1v0 = m1v1 + m2v2
e = 1/2, after 1st collision velocity = 4 2g If m1 > m2 , v1 > 0.
From this it follows v0 > v1 > 0
After 2nd collision velocity = 2 2g 21. (c) Just after collision
m v + m2 v2 10 14 4 0
vc = 1 1 = = 10 m / s;
After 3rd collision velocity = 2g m1 + m2 10 + 4
Spring force is an internal force, it cannot change the
2 g < 3g linear momentum of the (two mass + spring) system.
So, no circular motion. Therefore vc remains the same.
15. (c) Work done by friction 22. (c) m1v = – m1v1 + m2v2,
x 2v1
dx e= for limiting condition
= F .ds = µmg cos v
cos
0
m1 e
= µ mg x = 20 J <
m2 2 + e
N
5
23. (c) 144 km/hr = 144 × = 40 m/s
18
f
dx ds dy v2 v1 40
mg cos = =
ds sin 74 sin (90 37°)
dx

16. (c) Wg + W fr = KE N

mg (10 ) – µ mg ( /2) – µ mgs = 0


40 m/s |v2–v1|
µ 0.2
s= 10 – = 10 – ÷ = 49.5
53°
µ 2 0.2 2 74° 37°
E
40 m/s
17. (c) For the smaller block to move k x0 = µmg
and from work energy theorem 40 2sin 37° cos 37°
| v2 – v1 | = = 48 m/s
1 1 cos 37°
– µMgx0 – k x02 = – M v02
2 2 1
Change in momentum = 48 = 16 kgm/s
2 3
µmg 1 µmg 1
+ µMg + k ÷ = Mv
2
k 2 k 2 16
Average force applied by batsman = = 800 N
0.02
(2M + m)m
v = µg 24. (a) W.D. from ground frame = 6.4 × 50 = 320 J
kM From elevator frame, displacement = 0.
18. (b) Apply work-energy theorem
N cos 37° = 8 × 0.8 = 6.4 N
2 2
1 mg 1 mg N = 8N
mgz – k +z – k ÷ + Fz = 0 37°
2 k 2 k

z = 2F / k
19. (c) Energy lost in the collision is

E=
1
2
2
murel (
1 – e2 ) 10 × 0.8 = 8 N
WORK, POWER, ENERGY & CONSERVATION LAWS 165

25. (a) Total momentum is conserved along the horizontal Kf + Uf = Ki + Ui


direction and so
1 2 1 2
or mv f + khi = 0 + mghi
u 2 2
(M + m) < (M – m) u
2 Solving for vf, we find that
or M > 3 m.
vf 2 ghi ( khi2 m)
26. (d) At t = 0, mass m1 has velocity v1 and
With k = 10 N/m and m = 0.100 kg, we find that
mass m2 has velocity v2 .
vf 2 (9.80) (0.15) [(10) (0.15)2 /(0.1)]
At time t = 2t0, mass m1 has velocity v1' and
= 0.831 m/s
mass m2 has velocity v '2 . 29. (d) a
Fext
Both are moving under gravitational pull.
f=1N
Applying v u = at
v '1 v1 = g (2t0 ) and v '2 v2 = g (2t0 ) m = 1 kg, a = 1 m/s2
µ = 0.1
m1v '1 m1v1 = m1 g (2t0 ) ... (i) f = µ mg = 0.1 × 1 × 10 = 1N
and m2 v '2 m2 v2 = m2 g (2t0 ) ... (ii) Fext = ma + f
Fext = 1 + 1 = 2N
Adding (i) and (ii)
v = u + at = 0 + 1 × 2 = 2 m/sec.
[m2 v2 ' m2 v2 ] [m1v1' m1v1 ] = 2 gt0 (m1 + m2 ) P = F ext. v = 2 × 2 = 4 watt.
30. (a) Area under graph increases. Hence work done upon
( m1v1' m2 v2 ' ) (m1v1 + m2v2 ) = 2 ( m1 + m2 ) gt0 the particle from A to B increases.
Alternatively 31. (b) Work done by conservative force in a closed path is zero.
If we consider the two particles as a system then the So, WPQ + WQR + WRP = 0
external force acting on the system is the gravitational WPQ + WQR = WPR = 12 + 5 = 17J
pull (m1 + m2) g. P2

p 32. (a) The work done by F1 is W1 F1 cos ds


Fext = P1
t
From figure,
p = Fext t = ( m1v1 ' m2 v2 ') ( m1v1 + m2v2 ) ds = (6m) d (– 2 ) = – 12 d , and F1 = 20 N.
= (m1 + m2) g × 2t0 0
27. (c) From x1 to x2 interaction force is repulsive, so potential Hence, W1 240 cos d = 240sin = 120 2
energy decreases and from x3 to x4 interaction force is 4
/4
attractive, so kinetic energy decreases.
28. (a) Figure shows the system just as it is released and just Note that W1 = (20N) (6 2m) , just as if the chord
as the body strikes the table. P 1 P 2 and not the circular arc were the path of
///////////////////////// ///////////////////////// integration. The reason for this is that F1 is a
Spring conservative force. For such a force, the work is the
relaxed same along all paths joining two given points.
si = 0 sf = hi
vf = 0 m The work done by F3 is
hi
6 /2
vf
m hi = 0 W3 = F3 ds = (15 10s) ds
0

The potential energy of the system may be written as 2 3


= [15s 5s ]0 302.8 J
1 2
U = mgh + ks , h is the height above the table and s To calculate the work done by F2 and W , it is
2
is the amount of stretching of the spring. When the convenient to take the projection of the path in the
object is released, h = hi = 0.15 m, s = si = 0, and the direction of the force, instead of vice versa. Thus,
speed v = vi = 0. When the object strikes the table, h = W2 = F2 (OP2 ) = 30 (6) = 180 J
hf = 0, s = hi = 0.15m, and v = vf. The conservation of
energy requires that and W ( w) ( PO
1 ) ( 4) (6) 24 J
166 IIT-JEE PHYSICS Challenger
The total work done is 35. (a) v0y = v0 sin this component does not change
W1 + W3 + W2 + W = 23 J. v0x = v0 cos and ux cos
Then, by the work-energy principle,
For elastic collision relative velocity before collision is
K P2 K P1 = 23 J equal to relative velocity after collision along x-axis
v0 cos – u cos = u cos + vx
1 4 2 1 4
÷ (4) = 23
2
v2 if vx cos = v0y sin
2 9.8 2 9.8
By solving u = 0.1
v2 = 11.3 m/s
33. (b) In the frame of lift displacement is zero.
W = 0.
34. (a) Since the particle was initially moving in the x-direction, x
the momentum in the y-direction is zero.

y = 30° v0

m u d
v
4 x
3m R
v 4

36. (b) The required work done by man = kinetic energy of


Ground
man + kinetic energy of boat
Therefore finally the momentum in the y-direction is
zero. Let v and v be the component of velocities in the 1 p2 1 p2
= + (where p = Mv)
y-direction at the time of explosion. Then 2M 2 m
3m m
v' = v ... (i) 2
1 M 2 M 2 2 = 1 M + M v2
4 4 W= +
m ÷ m ÷
v
After time t second from the explosion, fragment of 2 M 2
m 37. (b) During a small elapsed time, t, the mass of water
mass is at y = +15 cm. Let the other fragment be
4 ejected is k (v t). The kinetic energy of the water is
at y . Both the fragments will move with constant speed
1
along y-direction. ( kv t ) v 2 . The rate at which kinetic energy is
2
m y 15 1 3
For mass , v= = ... (ii) kv t
1
4 t t imparted lim 2 = kv 3
t 0 t 2
3m y'
For mass , v' = ... (iii) 38. (b) If slipping is not there friction can do no work.
4 t
From (i), (ii) and (iii), Output Work
39. (b) Mechanical efficiency =
3m y ' m 15 Input energy
4 t 4 t The output work will increase because the friction
y' = 5 cm becomes less. Thus the mechanical efficiency increases.
The displacement will be in – y direction.
40. (a) WF + WSp + Wfric = k
Alternatively
As the particle is moving in the x-direction, after
explosion, the y-coordinate of centre of mass of the x=0
fragments is at zero. m2 m1 F

m1 y1 + m2 y2 µ µ
0=
m1 + m2 µ 1

m1 m2 m1 F
m/4
y2 y1 15 5 cm
m2 m µ 2
3 x
4
WORK, POWER, ENERGY & CONSERVATION LAWS 167

1 2 p12 p22
Fx kx m1 g x = 0 and kx = µm2g =
2 m1 m2
1
F m2 g m1 g = 0
2 p12 ml pl m1 1 1
= = = =
m g p22 m2 p2 m2 4 2
F m1 g + 2
2
41. (c) From given graph : 1.5
x2 x3 x2 2
45. (a) W ( x x 2 ) dx 1 x =0
3 4 4 2 3 2 3
F= x + 10 iˆ + 20 y ˆj z 16÷ kˆ 0
4 3 3
46. (d) In Case-I no horizontal external impulse on ball and
mass system while in Case-II friction is impulsive as
W = F.ds
normal with ground is impulsive.
(2,10,0) both are wrong.
3 4 4
= x + 10 iˆ + 20 y ˆj z 16÷ kˆ
(0,5,6)
4 3 3 47. (b) Initial Final
2m/3 m/3
287
[dx iˆ + dy ˆj + dz kˆ] = J
2
Work done can also be found by finding area under m/3
these curves. m
42. (c) By conservation of linear momentum along y-axis,
we get,
3mv y m 360 3 v y = 120 3 m

4m 90 360 3 3m vx By using mechanical energy conservation


m v 10
vy v= = 3.33 m/s
3
Similarily, along y-axis, we get 48. (b) Let the maximum extension of the spring be x as shown
4m . 90 = 3mvx vx = 120 in the figure. Work is done by the gravitational and the
spring force. There is no change in the kinetic energy
vy between the initial and final position of the mass.
tan = = 3, = 60° ;
vx

v= v x2 + v 2y = 240 m/s
43. (a) Let x be the distance travelled by the particle along the
37° incline, after which it comes to rest. From work-
energy theorem, Work done by gravity on block on x
incline 53° + work done by friction on block on incline
53° + work done by friction on ground level + work
done by gravity on block on incline 37° + work done
by friction on block on incline 37° = change in K.E.
mg(8) – µmg cos 53°(10) – µmg(10) – µmg cos 37°(x) From Work-energy theorem;
1 Wg + Ws = 0
– mg sin 37° (x) = 0 – (1) (10)2 where Wg = work done by gravity
2
and Ws = work done by spring
On solving we get x = 1m
1 2
p2 + Mgx – kx = 0
44. (c) K.E. = 2
2m
2Mg
E1 = E2 x=
k
168 IIT-JEE PHYSICS Challenger
Alternatively Also, when we put x = 0 in the given function,
The above situation can also be looked upon as the decrease
dU
in the gravitational potential energy of spring mass system we get F = 0. But F =
is equal to the gain in spring elastic potential energy. dx

1 2 dU
Mgx = kr At x = 0; = 0 i.e. the slope of the graph
2 dx
should be zero. These characteristics are represented
2Mg
x= by (d).
k 54. (d)
49. (d) W = Area under F-x graph = m (Area under a-x graph) S1 : For head-on elastic collision of sphere with wall, K.E.
1 of sphere does not change but the sphere receives an
=2 4 10 = 40J impulse. Hence false.
2
S2 : If KE of a sphere changes its momentum must change.
50. (c) The particle is at equilibrium at x = 0 and x = 2/3
Hence false.
F
S 3 : A force acting perpendicular to velocity changes
momentum, but does no work. Hence true.
S4 : When a force does work, kinetic energy of body
changes. If K.E. changes, momentum must change.
Hence false.
x 55. (b) Gain in P.E. = mass of liquid × height of rise of COM
x=0 ×g
x=2/3
The particle is in stable equilibrium at x = 0 metre and = (h × da) × h × g = dah2g
unstable equilibrium at x = 2/3 metre. 56. (b) S1 : If e = 0, v1 = v 2
The minimum speed imparted to the particle should be S2 : Collision is inelastic so loss of KE must take place.
such that it just reaches x = 2/3 from there on it shall
automatically reach x = 0. dPsys
S3 : Fext = 0 ; =0 Psys = constant
2/3 2/3 dt
1 2 1300
mv F dx x (3x 2) dx = 2
2 27
4 4
57. (c) p f = pi + F . dt
0
2600
or v= m/s
27 2

or P = 2600 Total change in momentum = 10 kg-m/s + (3 + 2t ) dt


51. (a) Loss in gravitational P.E. = gain in spring P.E. 0

1 p 2f 400
mgh k (h cot h) 2 pf = 20 kg-m/sec ; KE f = = = 100 J
2 2m 4
2mg 58. (c) Number of particles string the blades/time µ velocity
or (cot 1) = of wind
kh
K.E. of particle µ (Velocity of wind)2
2mg Power output µ (No. of particles striking/time) × (K.E.
or cot = 1+ of particle)
kh
Power output µ v3
mg 2mg 2mg Alternatively
52. (a) xA = , xC = , xB =
k k k dm d d
53. (d) dU(x) = – Fdx F= v ÷ v ( Volume) v (Volume)
dt dt dt
x
Ux = Fdx = v × (Av) = A v2
0 Power = Force × Velocity = A v2 × v = A v3
P µ v3
kx 2 ax 4 59. (a)
2 4 vx
v0 vx
vy
2k
U = 0 at x = 0 and at x = ; we have potential vx vy
a vx
energy zero twice (out of which one is at origin). v0 vx
WORK, POWER, ENERGY & CONSERVATION LAWS 169

By conservation of linear momentum, 2mv0 = 3mvx 62. (b) We know that U = – W for conservative forces
vx = 2v0/3 x x
By energy conservation U= Fdx or U = k xdx
0 0
1 1 1
mv02 2= m (v x2 + v 2y ) 2 + mv x2
2 2 2 kx 2
U(x) – U(0) =
2
2v02 = 2 (v x2 + v 2y ) + v x2
Given U(0) = 0
2v02 = 3v x2 + 2v 2y kx 2
U(x) =
2
4v02 4v02
2v02 3 + 2v 2y 2v02 = 2v 2y 63. (b) Wext + Wnc + WC = KE
9 3
WC is zero for closed path
2v02 v0 Hence WC = – U, U = 0
= 2v 2y vy =
3 3 64. (c) Since no external force is acting on the car, its speed
will not change. Since it is losing mass, its kinetic energy
4v02 v02 7v02 7 v0 will decrease.
vbead = v x2 + v 2y = + = = 65. (b) The centripetal acceleration
9 3 9 3
60. (d) E = Pt = 45 × 103 × 50 × 60 = 1.35 × 108 v2
ac = k2 r t2 or = k 2 rt 2
1.35 108 r
V = = 3.75 L v = krt
3.6 107
Since the motor is 20% efficient, five times as much dv
So, tangential acceleration, at = = kr
gasoline is needed. dt
61. (a) Let the ball fall from A at a height h above the landing. Work is done by tangential force.
Power = Ft .v.cos 0°
A
= (m at ) (krt )
h e
B = (mkr ) (krt )
////////////////////////////////
h e = mk 2 r 2 t
d
C Alternatively
//////////// The centripetal acceleration
h ac = k2 r t2
d
D
////////////////
v2
= k 2 rt 2
r
When it hits the surface at B, if its velocity is v then
1 2 m 2 22
v = 2 gh (v2 = u2 + 2gh = 2gh as u = 0) mv = k r t ... (i)
2 2
As it bounces to a height h again above the next (lower)
step, it actually bounces to a height (h-d) above the m 2 22
first step. If the velocity by which it bounces is v' then K.E. = k r t
2
v' = ev
d
v 2 g (h d ) (K.E.) = mk 2 r 2 t
or e= = dt
v 2 gh Power = mk2r2t
66. (b) 12.0 km/h = 3.33 m/s
h d
e2 =
h 1
Eheat = Ek = (25) (3.33)2 = 139 J
or h (1 – e2) = d 2
d 67. (d) Pt = nmgh
h= 2400 (60) = n (65) (9.8) (6) ; n = 37
1 e2
170 IIT-JEE PHYSICS Challenger
68. (b) The force constant of a spring is inversely proportional 73. (c) From momentum conservation,
to the length of the spring. m1v1 + m2v2 = (m1 + m2) v .... (i)
Let the original length of spring be L and spring 1
constant is k (given) KEf = ( m1 + m2 ) v 2
2
Therefore,
2
1 m1v1 + m2 v2
2L = ( m1 + m2 )
m1 + m2 ÷
k×L= ×k' from (i)
2
3

3 1 2
k = Mvcom
k'= 2
2
69. (b) Because momentum of the two-ball system is m1v1 + m2 v2
where, M = m1 + m2 and vcom = .
m1 + m2
conserved, p Pi + 0 = p Pf + p B .
74. (a) By COM we can say that both the fragments have
Because the Ping-Pong p Pf p Pi . same linear momentum.
p2
As a consequence, pB = 2 p Pi . Kinetic energy can be We have, K =
2m
expressed as K = p2/2m. Because of the much larger
1
mass of the bowling ball, its kinetic energy is much Kµ when p is same
smaller than that of the Ping-Pong ball. m
70. (c) From conservation of energy , K1 m2 A2
= =
K 2 m1 A1
1 2 1
mv0 = mgh + F .h + mv 2 75. (b) Using relative velocity, time of flight before collision
2 2
will be
2 Fh 20
v02 v 2 = 2 gh + t= = 15
m 20

2 Fh T B h' = 1/2 × 10 × 1 2
v v02 2 gh + ÷ =5m
m
O B

71. (b) Speed just before collision = 2 g (1 cos 0) W


gt = 10m/s
h – h' = 15m E
Speed just after collision = e 2 g (1 cos 0) B 20 – 10 × 1 = 10m/s 15

From conservation of energy, R

1 2 By COM at the time of collision


me [2 g (1 cos 0 )] mg (1 cos )
2 3 × 10 – 1 × 10 = 4 × v
2 × 10 = 4 × v ; 5 = v
cos 1{1 e 2 (1 cos 0 )} v = 5 m/s
For 1-D motion
72. (b) In a conservative field work done does not depend on
v2 = u2 + 2as
the path. The gravitational field is a conservative field. = 52 + 2 × 10 × 15
W1 = W2 = W3 = 25 + 300 = 325
K = 650 J

1. (b) The field is surely conservative. where r0 equilibrium position


2. (c) Work required can be calculated from P.E. at the
dU
B A To find equilibrium position, use =0
equilibrium position = 2 – dr
r 0 r0
WORK, POWER, ENERGY & CONSERVATION LAWS 171

3. (d) Since K.E. = 0 Apply COE


1 1 1
3B 2 mu 2 ÷ 2= (2m)v y 2 + m (v y 2 + v 2y )
P.E. = total energy 2 2 2
16 A
u
A B 3B 2 vy =
– 2
2 r 16 A
r
Solve for ‘r’ u
4. (b) Time taken by particle to move from A to B is 2

2R 2R 20 u
t1 = = = = 4 second. 2
5 5 5
2 2 2
u u u 3
vnet = ÷ + ÷ = .
Since collision is perfectly elastic, relative velocity 2 2 2
between ring and particles will not change. Hence after
11. (a) In the previous solution
4th time it reaches to A.
5. (c) For particle, x component of velocity (2 initial)
5 u
= m/s 2
2
5 u
y component of velocity (2 initial) = m/s
2 2
u
for 8 second ring move along x direction and for 8 second
it move along y direction. So, its centre of mass = (2R, tan = 2 = tan 1 2
2R) = (20, 20) u/2
6. (d) Since collision are elastic (e = 1) along normal direction Angle between sphere A and B direction is 2 tan 1 2.
velocities will be exchanged, hence all these 12. (c) Since collision are elastic, velocity will be exchanged.
conclusions are correct. Spheres comes to rest and U-tube move with max speed
7. (c) Use impulse momentum equation of u.
For bullet 13. (c) Impulse delivered = change in momentum.
14. (c) Moment of the impulse delivered by the blow = angular
p2 p1 = F. T momentum about centre of mass.
– 0.25 × 200 + 0.25 × 100 = F. T ... (i) 15. (b) The frequency of emitted sound = the frequency of
Block A, – 1 × v = T. t – F. T ... (ii) nvT
vibration of the rod =
Block B, 2 × v = T. t ... (iii) 2
16. (a) Speed of particle after the collision
25
Solving we get, v = m/s.
3 2
15
3 ÷ + 25 = 5.036 m /s
50 43
8. (c) From equation (iii) T. t = 2.v = N-s.
3 30
9. (d) K.E. = W.D. by all forces 17. (b) Speed of the sphere just after collision = m/s
43
1 18. (a) Angular speed of sphere is zero as impulse due to
(1 + 2)v 2 = (2) (10) h collision passes through centre of sphere.
2
19. (a) 20. (a) 21. (b)
3 25 25 625 Sol. 19-21: For each of the paths from O to C, the work done by
h= = = 5.2 m.
4 10 3 3 12 10 C
F2 is given by W = F2 .d s
10. (c) Apply COM along x direction
0
u
2mu = 2mvx + mvx + mvx v= where F2 = ( xy xˆ + xy yˆ ) and ds = xˆ dx + yˆ dy .
2
Along y direction, Pi = Pf The product F2 .ds = xy dx + xy dy .
172 IIT-JEE PHYSICS Challenger
The path OAC consists of the straight lines OA and
Mv
AC. Along OA, y = 0 and dy = 0, so Substituting v = and v2 = 2gy,
( M + m)
F2 .ds = 0 M 2 v2
f = (M + m) g + (M + m)
OA 2 (M + m)2 d
Along AC, x = 1m and dx = 0, so
M 2 gy
F2 .ds = y dy = ( M + m) g +
M +m d
Therefore, 23. (b) F = p/ t, where t, is the time interval from just
after (or just before, since momentum is conserved)
1 1
y2 1 the impact to cessation of motion of the stake plus
F2 .ds = y dy = = J mallet. Then,
CA 0
2 2
0
p 0 [ ( M + m) v ]
t= =
The work done along the entire path OAC is given by F f ( M + m) g
1 1
WOAC = F2 .ds + F2 .ds = 0 + J = J Mv M +m 2
OA AC
2 2 = = d
2 M gy
M gy
The path OBC consists of the straight line OB and BC. ÷
m+m d
Along OB, x = 0 and dx = 0, so F2 .ds = 0 . 24. (a) Just before impact the kinetic energy of the system
OB
1
Along BC, y = 1m and dy = 0, so was Mv 2 , just afterward it was
2
F2 .ds = x dx .
1 1 M2 2
( M + m) v 2 = v
1 1 2 2 M +m
x2 1
Therefore, F2 .ds = x dx = = J So the amount lost by the mallet was
2 2
BC 0 0 1 1 M2 2 m 1
Mv 2 v = Mv 2 ÷
The work done along the entire path OBC is given by 2 2 M +m M +m 2

1 1 m
WOBC = F2 .ds + F2 .ds = 0 + J = J or the fraction lost was
2 2 M +m
OB AC
25. (a) Conserving linear momentum : m1v1 = m2v2
Along the straight line OC, y = x, so dy = dx and as m1 = m2 = m v1 = v2
Work energy theorem
F2 .ds = xy dx + xy dy = 2 x 2 dx . Then
1 2
mgh = 2 mv ÷ v = gh
1 1 2
3
2x 2
WOC = F2 .ds = 2 x 2 dx = = J 26. (c) Momentum conservation
OC 0
3 3
0
gh
22. (a) The speed of the mallet on just striking the stake is m gh m 0 2m v v =
2
v = 2 gy . Momentum is conserved at the instant of Work energy theorem
collision, so that Mv = (M + m) v', where v' is the speed 2
of the stake plus mallet just after impact. 1 gh 1
mgh 2 m ÷ m ( gh )2
The resultant upward force on the stake plus mallet is 2 2 2
F = f – (M + m) g, where f is the resistive force of the
mgh mgh
ground. Then the work-energy principle gives mgh
4 2
K ( F) ( d)
mgh h
mgh h =
1 4 4
0 ( M + m)v 2
=[f (M m) g ] ( d )
2 27. (a) Momentum conservation
gh
v2 m1v1 + m2 v2 = 2 m
f = (M + m) g + ( M + m) 2
2d
WORK, POWER, ENERGY & CONSERVATION LAWS 173

Work energy theorem Sol. 37-39: We need to know the velocities of particle and pile
immediately after the inelastic collision. Let the velocity
2 of the particle just before strike be v and velocity of the
mgh 1 2 1 2 1 gh
= mv1 + mv2 ÷ 2m ÷ particle after the strike be v1 and velocity of pile after
4 2 2 2 2
strike be v2.
v = 2 gh
v1 = 0 , v2 = gh
From momentum conservation
block comes to rest & wedge moves with velocity m1i v1i + m 2 i v 2i = m1 f v1 f + m 2 f v 2 f
gh . mv = mv1 + 3mv2
28. (a) Physically possible explosions are those in which both v = v1 + 3v2 ........... (1)
particles move in opposite directions. i.e. signs of v2 v1
velocities are opposite. e=
v 0
II, IV & V ev = v2 – v1 ........... (2)
29. (b) If mA = mB, magnitude of velocities are same II From (1) and (2),
If mA > mB vA < vB V (1 + e) v = 4v2
If mA < mB vA > vB IV (1 + e) v
30. (d) As in VI momentum is not conserved. v2 =
4
Force must be acting on it. From (1),
31. (d) v1 + v2 = v (remains same straight line) v1 = v – 3v2
1 2 1 2 1 (1 3e) v
mv1 + mv2 + C1 = mv02 v1 = ,
2 2 2 4
C1 is loss in kinetic energy Note that (1 – 3e) < 0
as e > 1/3, i.e.the velocity of particle is opposite to the
On solving, v02 = v12 + v 22 + C 2 assumed positive reference direction. This further
means that it rebounds back in the upward direction.
v12 + v 22 = v02 C2
v2
h = 1 (height attained after the collision) =
v12 + v22 = (v02 C2 )2 2g

(3e 1) 2 h
R v02 C 2 ( < v0 ) .
16
therefore yield circle of smaller radius. Now consider the motion of pile into the material.
32. (a) In inelastic collision radius will be less but straight line Two forces are acting on pile :
of momentum conservation will be same. (i) Constant resisting force F
33. (a) Velocity of incident I object will always be less than II (ii) Force due to gravity,
object, therefore from above two solutions (1.5, 0.5) is F 3mg
correct. a=
3m
Pile penetrates the distance,
v2
(1 + e)2 3mgh
2 (0.5, 1.5) y=
16 ( F 3mg )
A
(1.5, 0.5) x t
8 5 dt 8
B
v1 40. (a) dx = ; x = ln (1 + 5t )
2 0
5 0 1 + 5t 5

41. (b) Just after collision, v = 8 m/s (t = 0)


(M + 1000) 8 = 1000 × 12
34. (c) = 0, angular momentum conserved. (Using momentum conservation)
35. (c) At maximum extension velocity along spring length 2M = 1000
should be zero. M = 500 kg
36. (b) At maximum extension Fnet 0 1 60000
Fr = 1500 × a = 1500 × 8 × 5=
37. (a) 38. (a) 39. (d) (1 + 5t ) 2
(1 + 5t ) 2
174 IIT-JEE PHYSICS Challenger
47. (a) a µ x with jerk when string slacks.
8
42. (d) Impulse = p = 1500 8 ÷ 1 1 2 1
11
48. (c) k (3 )2 = mv + k ( )2 2
2 2 2
1500 80 120000
== N-s
11 11 7
k.7 ( )2 = mv2 v= m/s
43. (a) From conservation of momentum 10
Mav0 = (Ma + Ms) vf
m1a1 + m2 a2 m 0 m 2 g 3 g
0.50 49. (b) acm = = =
vf = ÷ 4.0 = 0.57 m / s m1 + m2 m1 + m2 2
0.50 + 3.0
50. (b)
44. (b) Only friction is responsible to move the slab
f = Msas
MBg
as = ........ (1)
Ms 0
0 m/s v
v 2f = v02 + 2 a s x , where v0 = 0 ........ (2) m m
From eq. (1) and (2), x = 0.49m.
45. (b) W = Fd = µMBgx 2
W = 0.20 (0.5) (10) (0.49) = 0.49 Joule 1 3mg 1 2
By COE, k ÷ = mv
2 k 2
1
or W K= M s v 2f
2
9mg 2 m
46. (c) v= = 3g
k k
1 kg
m 0 mv v 3 g m
vcm = = =
m+m 2 2 k
1 2 1
2 51. (a) By COE in CM-frame, vref = kx 2
2 2
2
1m m 1
= kx 2

3g
2 2 2

Slack Total ext.


9 2 m2 x=
3mg
=3 g = kx 2 ;
2 k 2k
F = ma = 100 (3 ) ; a = 3 m/s2

1. (d) When e = 0, velocity of separation along common When object slips k = 0


normal zero, but there may be relative velocity along A = g sin ; which is independent on shape.
common tangent.
5. (d) The gain in KE is due to work done by muscles & work
2. (d) Statement – 1 is False but statement – 2 is true.
done by friction is zero as it is static friction.
3. (c) Statement – 1 is True but statement – 2 is False.
6. (a) Work done by action reaction force may be zero only if
g sin displacement of both bodies are same.
4. (a) a =
1+ k2 / R2 7. (a) Statement-2 clearly explains statement-1.
8. (b) Both statements are true and independent.
9. (b) Both the statements are true but statement-2 is not
correct explanation of statement-1. In fact the
momentum is conserved both in elastic as well as in
inelastic collision. But in elastic collision the total
kinetic energy of the system is also conserved.
WORK, POWER, ENERGY & CONSERVATION LAWS 175

10. (d) net work done = change in KE.


////////////////////

(nothing specified for internal/external forces)


0 12. (c) On horizontal surface, Wf = µmgs = (1/2) mv2
m On inclined surface, Wf = µmg cos s'
//////////////////////////////////////////////////////////////////////////////// = (1/2) mv2 – mgh
equilibrium extreme The coefficient of friction between two surfaces is
independent of angle of inclination of the surface.
2E 13. (c) Momentum remains constant before, during and after
In this example, maximum elongation =
k the collision but KE does not remain constant during
But maximum compression = 0 the collision as the energy gets converted into elastic
dp potential energy due to deformation.
11. (d) Fext internal forces have no effect on 14. (d) Think about uniform circular motion in horizontal plane,
dt
momentum. p exists because it is vector and W = 0.

3. (a, d) Momentum is vector, KE is scalar


dU
1. (a, b, c) F = – =0
dr Psystem = P1 + P2 + ....
2 gives stable equilibrium position
d U
and >0 Ksystem = k1 + k2 + ....
dr 2
4. (a) As one piece retraces its path, the speed of this piece
The fact that a system always tends to a state in
just after explosion should be v cos
which its potential energy is minimal implies that
repulsive forces act on (a – b) and (d – e) segments
– +
and an attraction force acts on (b – d) segment.
2. (c) The expression of work done by the variable force F on vcos
m
the particle is given by vsin v

W = F .d
vcos
In going from (0, 0) to (a, 0), the coordinate of x varies
from 0 to 'a', while that of y remains zero. Hence, the
work done along this path is : vcos v'
a m m
W1 = ( Kx j ). dx i = 0 [ j .i = 0 ] 2 2
0 (At highest point just after explosion)
In going from (a, 0) to (a, a) the coordinate of x remains
Applying conservation of linear momentum at the
constant (= a) while that of y changes from 0 to 'a'.
Hence, the work done along this path is highest point;

a m m
W2 = [( K ( yi + a j ). dy j ] m (v cos ) = v' v cos
0 2 2
a 3 v cos = v'
= Ka dy Ka 2
0 5. (a, b) We denote the spring constant, the relaxed length,
and the collar mass by k, 0, and m, respectively.
Y We let vi and i represent the initial collar speed
(a, a) and spring length, v and represent the
instantaneous speed and spring length at some
dy
other time. The conservation of mechanical energy
implies that

O 1 2 1 1 2 1
dx (a, 0) X mv k( 0)
2
= mvi + k ( i 0)
2
2 2 2 2
Hence, W = W1 + W2 = – Ka2
176 IIT-JEE PHYSICS Challenger
Since vi = 0, we have K.E. of the system in situation 3 is
k 1 1
v [( i 0)
2
( 0)
2 1/ 2
] mv '2 + mv '2 = mv '2
m 2 2
Referring to figure,
mv 2 v
v' = ÷
=
= (0.20)2 + (0.15) 2 = 0.25m 4 2
i
This is the kinetic energy possessed by A – B system (since,
As the collar passes point A, = A = 0.20m. C is at rest).
With 0 = 0.10 m, k = 500 N/m, and m = 10 kg, Let x be the maximum compression of the spring.
we obtain Applying energy conservation
1 2 1 1 1
vA
500
[(0.25 0.10)2 (0.20 0.10)2 ]1/ 2 mv = mv '2 + mv '2 + Kx 2
2 2 2 2
10
1 2 1 2 1 2
= 50 0.0225 0.0100 = 0.791 m/s mv = mv + Kx
2 4 2
As the collar passes point B, = B
1 2 1 2
= (0.20)2 + (0.10)2 = 0.2236 m. Kx = mv
2 4
Then we have,
m
vB 50 [(0.15) 2
(0.1236)2 ]1/ 2 = 0.601 m/s x= v
2K
6. (b, d) In situation 1, mass C is moving towards right with 7. (a, b) T = m (g + a)
velocity v. A and B are at rest.
In situation 2, which is just after the collision of C with 1
But a = 0 T = mg and KE = mv 2
A, C stop and A acquires a velocity v. [head-on elastic 2
collision between identical masses] i.e. constant.
When A starts moving towards right, the spring suffer 8. (a, c) The cable does no work on the load, so the load’s
a compression due to which B also starts moving energy is conserved.
towards right. The compression of the spring continues Ki + Ui = Kf + Uf
till there is relative velocity between A and B. When 1W 2
this relative velocity becomes zero, both A and B move v0 + 0 = 0 + W (d d cos )
2 g
with the same velocity v' and the spring is in a state of
maximum compression.
v02 2 gd (1 cos ) = 4 gd sin 2
Applying momentum conservation in situation 1 2
and 3,
v0
= 2sin 1
÷
2 gd
v
C A B
1
Fig(i) v0 = 2 gd sin = 2 (9.8) (5) ÷ = 7 m/s
2 2
9. (a, b, c) Impulse is independent of frame of reference.
v
Vel=0 Momentum depends upon frame of reference.
C A B Work done by a force depends on frame of
Fig(ii) reference.
10. (c,d) (a) is wrong because the momentum of ball changes
v'
in magnitude as well as direction.
Vel=0 v'
(b) is wrong because on collision, some mechanical
C A B energy is converted into heat, sound energy.
Fig(iii) Max Compression (c) is correct because for earth + ball system the impact
force is an internal force.
(d) is correct.
mv = mv' + mv'
11. (a, c) Putting k = 1, v 2 v1 for oblique collision
v
v' = and putting k = 1, v1 = 0, v2 = v for head-on
2
collision.
WORK, POWER, ENERGY & CONSERVATION LAWS 177

12. (c, d) Since you are not moving in ground frame so no 15. (d) The hanging part of the chain which is to be pulled up
work by gravity. can be considered as a point mass situated at the centre
of the hanging part. The equivalent diagram is drawn.
E
13. (c) P= = constt The work done in bringing the mass up will be equal to
t the change in potential energy of the mass.
1 2
mv
2 = constt
t

v2
= constt (k )
t T
L/6 = distance of the centre
M_g
v = kt1/2 3
from top of the table.
ds
= kt1/ 2 or ds = kt1/ 2 dt W = Change in potential energy
dt
= mgh
By integrating, we get
M L MgL
2kt 3 / 2 = g =
s= +C 3 6 18
3 Alternatively,
s µ t3/2
M
14. (b, c, d) If F is conservative, then The mass per unit length of the chain =
L
U U Let us consider length x of the chain, see fig.
Fx , Fy To move this length upwards, a force equal to the
x y
weight of chain of length x will have to be applied
Fx 2
U 2
U Fy upwards.
and so =
y y x x y x -dx
But, for the given force, x
Fx Fy
= 2 x 2 y and = 2 xy 2
y x M
Hence, the given force is not conservative. Weight of chain of length x is x÷ g
L
The work done by F is given by Small amount of work done in moving the length dx
upwards is
W = F .ds = ( x 2 y 2iˆ + x 2 y 2 ˆj ) . (dx iˆ + dy ˆj )
M
dW = F . dx = Fdx = x÷ g dx
= x y dx + x y dy
2 2 2 2
L
Along AB, y = 0 and so WAB = 0. The total amount of work done in moving the one third
Along BC, dx = 0 and length of the hanging chain on the table will be
a
a5 x2
L/3
WBC = a 2 y 2 dy = W=
L/3 M
xgdx =
M
g
L/3
xdx =
M
g
0
3 0 L L 0 L 2
0
5
a
Thus, W ABC = W AB + WBC = (J) MgL
3 =
Along AD, x = 0 and so WAD = 0. 18
Along DC, dy = 0 and 16. (c, d) The spring pulls both the blocks with same force.
a Hence force on both blocks is equal and opposite.
a5
WDC = x 2 a 2 dx = Since no net external force acts on system and its
0
3 initial momentum is zero, therefore net momentum
of system is always zero. Hence momentum of
a5
Thus, WADC = WAD + WDC = (J) blocks are equal and opposite
3 17. (d) Applying the principle of conservation of energy
Along AC, x = y and dx = dy. Thus,
(K.E.)B + (P.E.)B = (K.E.)A + (P.E.)A,
a
2a 5 we get
W AC = 2 x 4 dx = (J)
0
3
178 IIT-JEE PHYSICS Challenger
21. (a, c)
1 2 1
mv + mgL = mu 2
2 2 v
y A

60m
y' x'
Hence, v = u2 2 gL ... (i) B
x 45°
v ux' = v cos 45° ax' = – g sin 45°
uy' = – v sin 45° ay' = – g cos 45°
At point A,
vx' = 0 0 = v cos 45° – g sin 45°t ; t = v/g
1 2
u h gt
2 1 2
tan 45° = ; h gt = vt
vt 2
Change in velocity = | v u | = v + u
2 2
1 v2 v2
h g 2÷ =
2(u 2 g ) 2 g g
= [From (i)]
18. (a, b, c) Work by centripetal force = 0
v 2 v 2 3v 2 2 gh
because FC to displacement h= + = ; v= = 20m/s
g 2g 2g 3
Area under F-x curve = F.dx = W.
v' = velocity just before collision = 2v
By definition, for conservative force work is v'' = velocity after collision = v
independent of path length.
2v v
[Pressure × volume] = [work] t AB = = 2 2 = 4 2 sec
19. (c, d) Sphere A moving with velocity v has a component g cos 45° g
v/2 along the line joining the centres of the spheres
1 1
at the time of collision and another component AB 10 16 2 = 80 2 m
2 2
v 3 / 2 perpendicular to the previous direction.
After collision the component along the line will x2 U
dU
interchange i.e. B will move with v/2 velocity i.e. 4 22. (b, c, d) F F .dx = dU
m/s along the line joining the centres and A moves dx x1 0

with v 3 / 2 velocity at perpendicular direction In repulsion F and dx are in same direction.


to B. In a ttra cti on F a n d d x a r e in o p p o s i t e
directions.
20. (c,d) When the force is perpendicular to the velocity and
constant in magnitude then the force acts as a 23. (a, b) Given : m1 = 3 kg, m2 = 3 kg
centripetal force, and the body moves in a circular path. u1 = 3m/s at an angle of 30° with the line of impact
The force is constant in magnitude, this show the speed 1 = 30°
is not changing and hence kinetic energy will remain u2 = 4.5 m/s at an angle of 60° with the line of
constant. impact
1 = 180° – 60° = 120°
v e = 0.9

3m/s 4.5m/s
F
v
x 30° 60°
+ Line of
F impact
36° 58.8°
F + A B
v y
v v1=2.51m/s v2 =4.55m/s

The velocity changes continuously due to change in Let v1 and v2 be the final velocities of the balls
the direction. The acceleration also changes after impact making angles 1 and 2 with the
continuously due to change in direction.
line of impact, respectively.
WORK, POWER, ENERGY & CONSERVATION LAWS 179

Along the line of impact applying the conditions v1 sin 1 = 1.5 ....... (5)
of direct impact, we have, by law of conservation
of momentum, u2 sin 1 = v2 sin 2
m1u1 cos 1+ m2u2 cos 2 4.5 × 0.866 = v2 sin 2
= m1v1 cos 1 + m2v2 cos 2
v2 sin 2 = 3.897 ........ (6)
or 3 cos 30° + 4.5 cos 120° = v1 cos 1 + v2 cos 2 Squaring and adding (3) and (6)
or v1 cos 1 + v2 cos 2 v22 = 5.55 + 15.19 = 20.74 or v2 = 4.55 m/s
= 3 × 0.866 – 4.5 × 0.5 = 0.348 ..... (1) Dividing eq. (6) by eq. (3)
By Newton’s law of collisions 3.897
tan 2 = = 1.654
v2 cos 2 – v1cos 1 = e (u1 cos 1– u2 cos 2) 2.356
= 0.9 (3 × 0.866 + 4.5 × 0.5) 2 = 58.8°
= 0.9 × 4.848 = 4.363 .... (2) Squaring and adding eq. (4) and eq. (5)
Adding eq. (1) and eq. (2)
v12 ( 2.008)2 + (1.5)2 = 4.032 + 2.250 = 6.282
4.711
v2 cos 2 = = 2.3555 ....... (3) v1 = 2.506 m/s 2.51 m/s
2
Dividing eq. (5) by eq. (4)
and by eq. (1)
1.5
v1 cos 1 = 0.348 – 2.356 = – 2.008 ....... (4) tan 1 = = 0.7470 ;
2.008
As components of velocities perpendicular to line
of impact remain unchanged. 1 36.8°
u1 sin 1 = v1 sin 1 Hence, v1 = 2.51 m/s in a direction – 36.8° with
or 3 × 0.5 = v1 sin 1 line of impact.
v2 = 4.55 m/s in a direction 58.8° with the line of
impact.

1. A-s; B-s; C-p, q, r; D-p, q, r 3. A-p, s; B-p, s; C-q, r; D-p, s


(A) not possible, Px cannot be conserved The velocity of centre of mass of system of block and wedge
(B) not possible, Py cannot be conserved is always zero.
(C) possible, for any mass ratio – p, q, r
(A) At maximum compression velocities of block and wedge
(D) possible for any mass ratio – p, q, r
2. A-q; B-s; C-r; D-p must be same and equal to velocity of centre of mass.
F1 = k1x1 | F2 = k2x2 ; k1x1 = k2x2 = kx Hence velocity of both is zero and also minimum.
(B) At natural length of spring the velocity of system is
k2 x
x1 = and x1 + x2 = x zero as given.
k1 + k 2
(C) At the instant acceleration of wedge is zero its velocity
k1 x kk must be maximum. Hence velocity of block must be
& x2 = & k= 1 2 maximum.
k1 + k 2 k1 + k 2
(D) When elastic energy stored in spring is least it must be
1 2 in state of natural length.
(A) W.d. by S2 on block = kx
2 4. A-q, s; B-q; C-r; D-p, r, s
1 In perfectly inelastic collision bodies combine together after
(B) W.d. by S2 on S1 = k1 x12 collision. It is similar to nuclear fusion process.
2
5. A-r; B-p; C-q; D-s
1 2
(C) W.d. by F on block = + kx Initial position, KE = 0,
2
Equilibrium, KE is maximum
(D) W.d. by S1 on wall = zero (no displacement of point of
application)
180 IIT-JEE PHYSICS Challenger
Work done on block by gravity = mgx 6. A-q; B-q; C-p, r; D-q, s
////////////////// Since vcm will never be zero and some part of initial KE is
converted in spring potential energy.
0 At the time when spring is at its natural length, potential
energy is zero and at maximum extension, PE is maximum.
x
7. A-p; B-q; C-s
(WC)i + (WNC)i + WE = KE
(WC)i = PE
1 2
Magnitude of work done on block by spring = k x (WNC)i = ME
2
WE = TE
By conservation of energy, total mechanical energy will be
conserved.

1. 6.53 Substituting this value of h in (i) and (ii), we get


For the ball thrown up v12 – 2401 = – 19.6 × 78.4 v22 = 19.6 (98 – 78.4)

v12 = 864.36 v22 = 384.16


98m +
v1 = 29.4 m/s v2 = 19.6 m/s
C – At point C where the two bodies collide, thereafter both
78.4m bodies stick and behave as a single body, we apply
conservation of linear momentum.
m1v1 – m2v2 = 2mv
v v2 29.4 19.6
v= 1 = = 4.9 m/s
2 2
v12 u12 = 2a1s1
For the combined body
u = 4.9 m/s; s1 = – 78.4; a1 = – 9.8 m/s2; t = ?
v12 (49) 2 2( 9.8) h
1 2
v12 2401 = 19.6 h. ... (i) s = ut + at
2
– 78.4 = 4.9t – 4.9t2
1 2
s1 = u1t1 +
a1t1 t2 – t – 16 = 0
2
h = 49t – 4.9 t2 1 ± 1 + 64
... (ii) t= = 4.53
For the ball downward from height 2
Total time = 4.53 + 2 = 6.53 sec.
v22 u22 = 2a2 s2 2. 44.2
2
v22 02 2 9.8 (98 h) u sin 2 20 20 3
= 15.3m
Max Height =
2g 2 9.8 4
v22 = 19.6 (98 – h) ... (iii) Initial momentum of the object is horizontal direction at the
1 highest point = 5 × 10
s2 = u2 t 2 + a2 t22 = 50 kg m/s
2
At the highest point the projectile explodes and breaks into
98 – h = 4.9 t2 ... (iv) two particles which separate horizontally.
From (ii) and (iv)
98 – (49t – 4.9t2) = 4.9t2 20 cos 60° =10m/s
98 – 49t = 0 u=20m/s
t = 2 sec H
h = 49 × 2 – 4.9 × 22 = 78.4 m (from (ii)) 60°
WORK, POWER, ENERGY & CONSERVATION LAWS 181

Applying conservation of linear momentum (ii) Applying conservation of linear momentum at Q for
4v1 – 1v2 = 50 ... (i) collision with an mass of equal magnitude
The kinetic energy at the highest point initially m × 5 = 2m × v
1 [Since, the collision is completely inelastic the two masses
= 5 10 10 = 250 J
2 will stick together. v is the velocity of the two masses just
The kinetic energy after collision after collision]
v = 2.5 m/s
1 1
4 v12 1 v22 = 500 When the string is undergoing circular motion, at any
2 2
arbitrary position
4v12 + v22 = 1000 ... (ii)
2mv 2
Substituting the value of v2 from (i) in (ii) T – 2g cos =
4v12 (4v1 50)2 = 1000
Given that T = 0 when = 90°
4v12 + 16v12 + 2500 400v1 = 1000
2mv 2
0–0= v=0
20v12 400v1 + 1500 = 0
Velocity is zero when a = 90°, i.e., in the horizontal
2v12 40v1 + 150 = 0
position.
v12 20v1 + 75 = 0 M

v12 15v1 5v1 + 75 = 0


v1 = 15 or 5 m/s v2= + 10 or – 30 m/s
Taking the pair v1 = 15 m/s and v2 = 10 m/s.
Horizontal distance covered by mass m1
Horizontal distance covered by mass m2 Q
uy = 0; sy = H = 15.3; ay = 9.8 m/s2; t = ? 2mgSin 2mgCos
2mg
1 2
s = ut + at Applying energy conservation from Q to M, we get
2
15.3 = 4.9t2 1
2mv 2 = 2mg
t = 1.77 sec 2
x1 = 15 × 1.77 = 26.55 m
x2 = 10 × 1.77 = 17.7 m v 2 (2.5) 2
Separation between the two masses = = = 0.318 m
2g 2 9.8
= 26.55 + 17.7 = 44.2 m.
3. 0.318 4. 0.25
When the stone reaches the point Q, the component of
velocity in the + Z direction (v cos ) becomes zero due to v0 m
the gravitational force in the – Z direction. m L/2 L/2

Z COM COM v x
+Z L v'
+Y v =4m/s
Q Y x
v
vcos L/2 L/2
vsin
P vx=4m X
t=0 t=t Before collision After collision

Let just after collision, velocity of COM of rod is v and


The stone has two velocities at Q
angular velocity about COM is . Applying following three
(i) vx in the + X direction (4 m/s)
laws
(ii) v sin in the + Y direction (6 sin 30° = 3 m/s)
(i) External force on the system (rod + mass) in horizontal
The resultant velocity of the stone
plane along x-axis is zero.
v' = (vx ) 2 (v sin ) 2 Therefore applying conservation of linear momentum
in x-direction.
= 42 + 32 = 5 m / s mv0 = mv ... (i)
182 IIT-JEE PHYSICS Challenger
(ii) Net torque on the system about COM of rod is zero.
M
Therefore, applying conservation of angular momentum P.E. of the rest part of chain g( L l0 ) H
about COM of rod, we get L
P.E. of chain when it starts falling off
L L ML2 ML
mv0 ÷ I mv 0 mv0 =
6
M
g( H
L
) ×L
2 2 12
L 2
(iii) Since the collision is elastic, kinetic energy is also
By energy conservation
conserved.
1 L M l0 Mg
1 1 1 Mv 2 ÷

Therefore, mv02 + mv 2 + I 2 Mg H l0 H g ( L l0 ) H
2 2 2 2 2 L L

ML2 2 g 2 2 1000 ( 2 2 )
or mv02 = mv 2 + ... (iii) v= (L - l 0 ) = 10 - 6 = 28.3 cm/s
12 L 10
From equation (i), (ii) and (iii), we get the following 7. 7.7
result
P
m 1 When travelling at any speed v, F = and R = kv
= = 0.25 v
M 4
5. 12 N
Consider the vertical motion of the cannon ball
uy = + 100 sin 30°
sy = – 120 m
ay = – 10 m/s2
R F
t = t0

+ P o
_ 30 v = 5 3 m/s
800
C m = 9kg
120m When travelling on the level, F = 75/5 = 15
A B uT=0 As there is no acceleration F = R
But R=k×5
1 Therefore 15 = k × 5 k = 3
s = ut + at 2 R = 3v at any velocity v
2
When travelling down the hill at maximum speed v,
2
– 120 = 50 t0 – 5 t0
P 25
F= = and R = 3v
5t02 50t0 120 = 0 v v

t02 – 10 t0 – 24 = 0 N
R
( 10) 100 4(1) ( 24)
t0 = –
2
= 12s or – 2s [Not valid]
6. 28.3
Let H be the height of table top from ground. F
–1 800
L– l0 sin (1/40)

L/2 There is no acceleration so, resolving parallel to the hill, we


l0
C.G. l0 C.G. L have
H 2 H 1 25
F 800 =R + 20 = 3v
40 v
3v² – 20v – 25 = 0
Hence v = 7.7 (the negative root is not applicable).
M/L = mass per unit length of chain. The maximum speed downhill is 7.7 ms–1.
M l0
Then P.E. of hanging part

l0 H g
L
1. When a bucket containing water is rotated fast in a vertical
circle of radius R, the water in the bucket doesn’t spill b b
(a) ,0 (b) ,b
provided the bucket is 2 2
(a) whirled with a maximum speed of 2gR
(b) whirled around with a minimum speed of (1/ 2)gR b 2
(c) ,b (d) b, b
3 5
(c) having an r.p.m of n = 900g /( 2 R)
Here m and 2m represent the masses of the particles.
(d) having an r.p.m of n = 3600g /( 2 R) 5. A uniform rod of mass m and length L lies radially on a disc
2. A disc of uniform thickness and radius 50.0 cm is made of rotating with angular speed in a horizontal plane about its
two zones. The central zone of radius 20.0 cm is made of
metal and has a mass of 4.00 kg. The outer zone is of wood axis. The rod does not slip on the disc and the centre of the
and has a mass of 3.00 kg. The moment of inertia of the disc rod is at a distance R from the centre of the disc. Then the
about a transverse axis through its centre is kinetic energy of the rod is
(a) 0.510 kg-m2 (b) 0.515 kg-m2
(c) 0.500 kg-m2 (d) 0.525 kg-m2
3. A particle is confined to rotate in a circular path decreasing
linear speed, then which of the following is correct?
(a) L (angular momentum) is conserved about the centre L
(b) only direction of angular momentum L is conserved R
(c) It spirals towards the centre
(d) its acceleration is towards the centre.
4. With O as the origin of the coordinate axis, the X and Y-
coordinates of the centre of mass of the system of particles
shown in the figure may be given as :
Y
1 L2 1 2
m 2m m 2m (a) m 2
R2 (b) m R2
2 12 2
h
X X 1 2 2 1 2 2
O (c) m L (d) m L
b b b 24 12

MARK YOUR
1. 2. 3. 4. 5.
RESPONSE
184 IIT-JEE PHYSICS Challenger
6. A disc of mass M and radius R is rolling with angular speed 9. A cylinder rolls up an inclined plane, reaches some height,
on a horizontal plane as shown in Figure. The magnitude and then rolls down (without slipping throughout these
of angular momentum of the disc about the origin O is motions). The directions of the frictional force acting on the
cylinder are
Y (a) up the incline while ascending and down the incline
descending
(b) up the incline while ascending as well as descending
M
(c) down the incline while ascending and up the incline
X while descending
O
(d) down the incline while ascending as well as
(a) (1/2) MR2 (b) MR2 descending.
(c) (3/2) MR 2 (d) 2 MR2 10. A train of mass M is moving on a circular track of radius R
7. A sphere of mass ‘m’ is given some angular velocity about with constant speed V. The length of the train is half of the
a horizontal axis through its centre and gently placed on a perimeter of the track. The linear momentum of the train will
plank of mass ‘m’. The coefficient of friction between the be
two is µ. The plank rests on a smooth horizontal surface. (a) 0 (b) 2MV/
The initial acceleration of the plank is (c) MVR (d) MV
11. A small block of mass 'm' is rigidly attached at 'P' to a ring of
m mass '3m' and radius 'r'. The system is released from rest at
= 90° and rolls without sliding.
m
P
(a) zero (b) (7/5) µg
(c) µg (d) 2 µg
8. A uniform triangular plate ABC of moment of inertia I (about
an axis passing through A and perpendicular to plane of the
plate) can rotate freely in the vertical plane about point 'A'
as shown in figure. The plate is released from the position
shown in the figure. Line AB is horizontal. The acceleration
The angular acceleration of hoop just after release is
of centre of mass just after the release of plate is
(a) g/4r (b) g/8r
a (c) g/3r (d) g/2r
A× B 12. A running man has half the K.E. that a body of half his mass.
whenthe man speeds up by 1 ms–1 then he has the same
a a K.E. as that of the body. The original speeds of the man and
the boy in ms–1 are
(a) 1.41 each (b) 2.42, 4.84
(c) 4.84, 0.8 (d) 2.41, 0.41
C 13. A horizontal circular plate is rotating about a vertical axis
passing through its centre with an angular velocity o. A
man sitting at the centre having two blocks in his hands
mga 2 mga 2 stretches out his hands so that the moment of inertia of the
(a) (b)
3I 4I system doubles. If the kinetic energy of the system is K
initially, its final kinetic energy will be
mga 2 mga 2 (a) 2 K (b) K/ 2
(c) (d)
2 3I 3I (c) K (d) K/ 4

MARK YOUR 6. 7. 8. 9. 10.


RESPONSE 11. 12. 13.
CIRCULAR AND ROTATIONAL DYNAMICS 185

14. A particle of mass m is attached to a thin uniform rod of 17. A small object of uniform density rolls up a curved surface
length a and mass 4 m. The distance of the particle from the with an initial velocity v. It reaches up to a maximum height
centre of mass of the rod is a/4. The moment of inertia of the
combination about an axis passing through O normal to the 3v 2
of with respect to the initial position. The object is
rod is 4g

a/2

a/4
v

(a) ring (b) solid sphere


(c) hollow sphere (d) disc
18. Two thin rods of mass m and length each are joined to
64 2 91 2 form L shape as shown. The moment of inertia of rods about
(a) ma (b) ma an axis passing through free end (O) of a rod and
48 48
perpendicular to both the rods is
27 2 51 2
(c) ma (d) ma
48 48
15. The spool shown in figure is placed on a rough horizontal
surface has inner radius r and outer radius R.

F
O
R
2 2 m 2
r (a) m (b)
7 6

2 5m 2
(c) m (d)
3
The angle between the applied force and the horizontal 19. The figure shows a hollow cube of side 'a' of volume V.
can be varied. The critical angle ( ) for which the spool does There is a small chamber of volume V/4 in the cube as shown.
not roll and remains stationary is given by This chamber is completely filled by m kg of water. Water
r 2r leaks through a hole H and spreads in the whole cube. Then
1 1
(a) cos (b) cos the work done by gravity in this process assuming that the
R R
complete water finally lies at the bottom of the cube is
r r
(c) cos 1
(d) sin 1 a/2
R R a

16. The free end of a thread wound on a bobbin is passed round


a nail A hammered into the wall. The thread is pulled at a H Hole
constant velocity. Assuming pure rolling of bobbin, find
the velocity v0 of the centre of the bobbin at the instant a

when the thread forms an angle with the vertical.


vR vR
(a) (b) a
R sin r R sin r
2vR v (a) (1/2) mga (b) (3/8) mga
(c) (d) (c) (5/8) mga (d) (1/8) mga
R sin r R sin r

MARK YOUR 14. 15. 16. 17. 18.


RESPONSE 19.
186 IIT-JEE P HYSICS Challenger
20. A solid sphere of mass M and radius R having moment of 24. A particle of mass m is attached to a rod of length L and it
inertia I about its diameter is recast into a solid disc of radius rotates in a circle with a constant angular velocity . An
r and thickness t. The moment of inertia of the disc about an observer P is rigidly fixed on the rod at a distance L/2 from
axis passing the edge and perpendicular to the plane remains
the centre. The acceleration of m and the pseudo force on m
I. Then R and r are related as
from the frame of reference of P must be respectively.
2 2
(a) r R (b) r R
15 15
P
2 2 m
(c) r R (d) r R
15 15 L
21. A heavy particle of weight W, attached to a fixed point by a O
light inextensible string describes a circle in a vertical plane.
The tension in the string has the values mW and nW 2 L
respectively, when the particle is at the highest and the (a) zero, zero (b) zero, m
2
lowest points in the path. The value of (n – m) is
(a) 5 (b) 4
2 L 2 L 2
(c) 6 (d) 7 (c) ,m (d) zero, m L
2 2
22. A sphere of mass M and radius R is moving on a rough fixed
surface, having coefficient of friction µ as shown in figure. 25. A solid cylinder is wrapped with a string and placed on an
It will attain a minimum linear velocity after a time inclined plane as shown in the figure. Then the frictional
force acting between cylinder and plane is
0

v0

µ
= 0.4
(a) v0/µg (b) 60°
0R/µg
(c) (v0 – 0R)/µg (d) 2 (v0 – 0R)/7µg
23. A block of mass m is at rest under the action of force F
against a wall as shown in figure. Which of the following (a) zero (b) 5 mg
statement is incorrect?
7m g mg
(c) (d)
2 5
a
26. A disc is rolling without slipping with angular velocity . P
a and Q are two points equidistant from the centre C. The
order of magnitude of velocity is
F

C P
Q
(a) f = mg [where f is the friction force]
(b) F = N [where N is the normal force]
(c) F will not produce torque (a) vQ > vC > vP (b) vP > vC > vQ
(d) N will not produce torque (c) vP = vC , vQ = vC/2 (d) vP < vC > vQ

MARK YOUR 20. 21. 22. 23. 24.


RESPONSE 25. 26.
CIRCULAR AND ROTATIONAL DYNAMICS 187

27. A solid sphere of mass M and radius R is kept on a rough 30. A circular disc of mass m and radius R is rotating on a rough
surface. The velocities of air (density ) around the sphere surface having a coefficient of friction µ with an initial angular
are as shown in the figure. Assuming R to be small and velocity . Assuming a uniform normal reaction on the entire
contact surface, the time after which the disc comes to rest
4 R2
M kg, what is the minimum value of coefficient is
g
of friction so that the sphere starts pure rolling? (Assume R 3 R
force due to pressure difference is acting on centre of mass (a) (b)
µg 4µg
of the sphere)

7 m/s 14 m/s 1 R 3 R
(c) (d)
2 µg 2 µg
R
M 31. Consider the two bobs are shown in the figure. The bobs
are pivoted to the hinges through massless rods. If tA be the
time taken by the bob A to reach the lowest position and tB
Horizontal be the time taken by the bob B to reach the lowest position.
(a) 0.25 (b) 0.50 (Both bobs are released from rest from a horizontal position)
(c) 0.75 (d) 1.0 then ratio tA / tB is
28. A racing car driver drives his car on a flat circular track of
radius 25/3 m and a coefficient of friction 0.5. He drives the
car in such a manner that he may attain the maximum possible m m
velocity on the track in a minimum possible time. The magnitude A B
of his tangential acceleration at an instant when his speed /2
is 5m/s is
(a) 2 m/s2 (b) 3 m/s2 (a) 3 (b) 5
(c) 4 m/s 2 (d) 1 m/s2
29. From a circular disc of radius R and mass 9M, a small disc of 1
radius R/3 is removed from the disc. The moment of inertia (c) 2 (d)
2
of the remaining disc about an axis perpendicular to the
plane of the disc and passing through O is 32. A uniform solid cube of mass M has edge length a. The
moment of inertia of the cube about its face diagonal will be

3 1
R/3 (a) Ma 2 (b) Ma 2
2 2
2R/3
5 7
R (c) Ma 2 (d) Ma 2
12 12
33. A particle undergoes uniform circular motion. About which
point on the plane of the circle, will the angular momentum
of the particle remain conserved?
(a) centre of the circle
40
(a) 4MR2 (b) MR 2 (b) on the circumference of the circle.
9
(c) inside the circle
37
(c) 10 MR2 (d) MR 2 (d) outside the circle.
9

MARK YOUR 27. 28. 29. 30. 31.


RESPONSE 32. 33.
188 IIT-JEE P HYSICS Challenger
34. A system consists a ball of mass M2 and a uniform thin rod
of mass M1 and length d. The rod is attached to a frictionless
horizontal table by a pivot at point P and initially rotates at A
an angular speed as shown in figure. The rod strikes the
ball, which is initially at rest. As a result just after collision, O
the rod stops and ball moves in the direction shown. If
R
collision is elastic, the ratio M1/M2 is
v
B
P P

d
v 3 v 3
(a) (b)
M2 2l 2R
M2

Before collision After collision 2 3v 2v


(c) (d)
R l 3
(a) 3 (b) 2
37. A long horizontal rod has a bead which can slide along its
(c) 1 : 2 (d) 1 : 3
length and initially placed at a distance L from one end A of
35. A hemispherical shell of mass m and radius R is hinged at
point O and placed on a horizontal surface. A ball of mass m the rod. The rod is set in angular motion about A with
moving with a velocity u inclined at an angle = tan–1(1/2) constant angular acceleration . If the coefficient of friction
strikes the shell at point A (as shown in the figure) and between the rod and the bead is , and gravity is neglected,
stops. What is the minimum speed u if the given shell is to then the time after which the bead starts slipping is
reach the horizontal surface OP ?
A B
O
P L

(a) / (b) /
A

m 1
u (c) (d) infinitesimal
(a) Zero
38. A hollow sphere of mass 2 kg is kept on a rough horizontal
2gR surface. A force of 10 N is applied at the centre of the sphere
(b)
3 as shown in the figure. Find the minimum value of so that
the sphere starts pure rolling. (Take g = 10m/s2)
gR
(c) F = 10 N
5
(d) it cannot come on the surface for any value of u.
30°
36. A disc is fixed at its centre O and rotating with constant
angular velocity . There is a rod whose one end is
connected at A on the disc and the other end is connected
with a ring which can freely move along the fixed vertical
smooth rod. At an instant when the rod is making an angle
30° with the vertical the ring is found to have a velocity v in
the upward direction. Find of the disc. Given that the (a) 3 0.16 (b) 3 0.08
point A is R/2 distance above point O and length of the rod (c) (d) Data insufficient
3 0.1
AB is l

MARK YOUR
34. 35. 36. 37. 38.
RESPONSE
CIRCULAR AND ROTATIONAL DYNAMICS 189

39. A uniform circular disc of radius r is placed on a rough


a2 (a2 g)
horizontal surface and given a linear velocity v0 and angular (a) (b)
R R
velocity 0 as shown. The disc comes to rest after moving
some distance to right. It follows that 2(a2 g)
(a) 3v0 = 2 0r (b) 2v0 = 0r (c) (d) None of these
R
(c) v0 = 0r (d) 2v0 = 3 0r
43. A cubical block of side L rests on a rough horizontal surface
with coefficient of friction . A horizontal force F is applied
on the block as shown. If the coefficient of friction is
v0 sufficiently high so that the block does not slide before
toppling, the minimum force required to topple the block is
0
F P
40. A thin wire of length L and uniform linear mass density is M
bent into a circular loop with centre at O as shown. The L
moment of inertia of the loop about the axis XX' is
X X' (a) infinitesimal (b) mg/4
90º (c) mg/2 (d) mg (1 )
44. A horizontal force F is applied at the top of an equilateral
triangular block having mass m and side a as shown in
figure. The minimum value of the coefficient of friction
required to topple the block before translation will be
L3 L3
(a) (b)
8 2 16 2 F

5 L3 3 L3
(c) (d)
16 2 8 2
41. Two point masses A of mass M and B of mass 4M are fixed
at the ends of a rod of length and of negligible mass. The 2 1
(a) (b)
rod is set rotating about an axis perpendicular to its length 3 2
with a uniform angular speed. The work required for rotating
the rod will be minimum when the distance of axis of rotation 1 1
(c) (d)
from the mass A is at 3 3
2 8 45. A solid spherical ball of radius R collides with a rough
(a) (b) horizontal surface as shown in figure. At the time of collision
5 5
its velocity is v0 at an angle to the horizontal and angular
4 velocity 0 as shown. After collision, the angular velocity
(c) (d)
5 5 of ball may
42. Linear acceleration of cylinder of mass m2 is a2. Then angular
acceleration 2 is (given that there is no slipping). 0

m1, R v0

m2, R (a) decrease (b) increase


(c) remains constant (d) none of these.

MARK YOUR 39. 40. 41. 42. 43.


RESPONSE 44. 45.
190 IIT-JEE P HYSICS Challenger
46. A particle of mass m is moving in a circle of radius r. The
centripetal acceleration (ac) of the particle varies with the
time according to the relation, ac = Kt2, where K is a positive
constant and t is the time. The magnitude of the time rate of
change of angular momentum of the particle about the centre
of the circle is
14 15
(a) mKr (b) m 2 Kr 3 (a) (b)
15 14
(c) mKr (d) mKr2
47. A horizontal uniform beam AB of length 4m and a mass of 20 3 10
(c) (d)
kg is supported at the end B by means of a string which 10 3
passes over a fixed, smooth pulley supporting a 50. Two vertical walls are separated by a distance of 2 metres.
counterbalancing weight of 8kg on the other side. What Wall ‘A’ is smooth while wall B is rough with a coefficient of
force, F, when applied at the point A in a suitable direction, friction = 0.5. A uniform rod is probed between them. The
will hold the beam in static equilibrium? length of the longest rod that can be probed between the
walls is equal to
2m

20 kg P
30°
A 4m B
Q

(a) F = 160 N (b) F 40 19N


(c) F (200 80 3)N Wall Wall
(d) no force F, as specified, can hold the beam in static A B
equilibrium.
(a) 2 metres (b) 2 2 metres
48. x, y, z are the cartesian axes of an inertial frame of reference. A
particle of mass 1 kg moves with a uniform velocity of 1m/s 17
from A (0, 3 m, 0) to B (4m, 0, 0). The motion of the particle is (c) 2 metres metres
(d)
2
observed from a frame K which rotates with constant angular
51. Three identical rods are hinged at point A as shown. The
velocity about the z-axis by an observer O located at
angle made by rod AB with vertical is
(0, 0, 7m) in xyz system. What is the magnitude of average
pseudo force that observer O should consider as acting on A
the particle during its motion from A to B, if
3
k rad / s; k being a unit vector in the direction of
10
positive z - axis? B 90°
D
3
(a) zero (b) N
10
2 90°
3 5 3
(c) N (d) N
50 2 10 C
49. A thin uniform spherical shell and a uniform solid cylinder 1 3
of the same mass and radius are allowed to roll down a fixed (a) tan 1
(b) tan 1
3 4
incline, without slipping, starting from rest. The times taken
by them to roll down the same distance are in the ratio, 4
tsph : tcyl (c) tan –1 (1) (d) tan 1
3

MARK YOUR 46. 47. 48. 49. 50.


RESPONSE 51.
CIRCULAR AND ROTATIONAL DYNAMICS 191

52. What is the average angular speed of the second hand on a 56. A and B are moving in 2 circular orbits with angular velocity
clock (in rad/s) ? 2 and respectively. Their positions are as shown at t = 0.
(a) 6.28 (b) 0.105 Find the time when they will meet for the first time.
(c) 0.0167 (d) 1.745 × 10–3
B
53. The pulley in the figure has radius R = 3 cm and moment of
inertia I = 36 × 10–4 kg m2 about its axis of rotation. The high
A
cord that connects the masses (Ma = 2 kg and Mb = 3 kg)
goes over the pulley and does not slip over the pulley.
If coefficient of friction between Ma and the tabletop is
µk = 0.15 and if the system starts from rest, determine the
speed (in m/s) of the masses when Mb has descended a 3
(a) (b)
distance d = 1.5 from its original position. 2 2
(g = 10 m/s2).
I (c) (d) they will never meet
Ma R 57. A homogeneous disc with a radius 0.2m and mass 5 kg
rotates around an axis passing through its centre. The angular
velocity of the rotation of the disc as a function of time is
Mb given by the formula = 2 + 6t. The tangential force applied
to the rim of the disc is
d (a) 1 N (b) 2 N (c) 3 N (d) 4 N
(a) 6 (b) 4 (c) 3 (d) 5 58. A particle of mass m moves without friction along a
54. A uniform rigid rod hinged at one end is released from rest semicubical parabolic curve, y2 = ax3, with constant speed
in the position shown in the vertical plane. Find the v. Find the reaction force of the curve on the particle.
magnitude of reaction force (in N) at hinge just after its 3/ 2
3 1/ 2 1/ 2 9
release. Express your answer after rounding it to nearest (a) a x 1 ax mv 2
4 4
integer. (Use M = 0.8 kg, = 45°, g = 10 m/s2)
3/ 2
(a) 6 (b) 4 (c) 3 (d) 5 3 1/ 2 1/ 2 9
55. A ring of mass M and radius R lies in x-y plane with its (b) a x 1 ax mv2
4 4
centre at origin as shown. The mass distribution of ring is
non-uniform such that at any point P on the ring, the mass 1/ 2
3 1/ 2 1/ 2 9
per unit length is given by = 0 cos2 (where 0 is a positive (c) a x 1 ax mv2
4 4
constant). Then the moment of inertia of the ring about
z-axis is (d) None of these
59. A semicircle of radius R = 5m with diameter AD is shown in
y figure. Two particles 1 and 2 are at points A and B on diameter
at t = 0 and move along segments AC and BC with constant
P u1
M speeds u1 and u2 respectively. Then the value of for
R u2
both particles to reach point C simultaneously will be
x C

1 2
A B D
1 2m
(a) MR2 (b) MR 2
2 5 2 5
(a) (b)
4 4
1M 1M
(c) R (d) R
2 0 0 2 2
(c) (d) 2 2
5

MARK YOUR 52. 53. 54. 55. 56.


RESPONSE 57. 58. 59.
192 IIT-JEE P HYSICS Challenger
60. Different locations of a cyclist moving with constant speed 64. A truss is made by hinging two uniform 150 N rafters, as
on road of hilly region are as shown in the figure below. At shown in figure. They rest on an essentially frictionless
which of these locations would he feel heaviest floor and are held together by a tie rope. A 500 N load is held
at their apex. Find the tension in the tie rope.
C

D g
B

m
3.0
(a) A (b) B 500N
Tie rope
(c) C (d) D

m
0.5
61. Consider the bowling ball in pure rolling motion and suppose
///////////////////////////////// ////////////
that it is rotating with an angular velocity of magnitude . In 3.5m

applying the principles of classical mechanics to a rigid


body, it is useful to regard the rigid body as being composed
(a) 280 N (b) 460 N
of an infinite number of point masses. The point masses
that make up the sphere will have linear speeds (relative to (c) 320 N (d) 400 N
the ground) 65. A boy of mass 30 kg starts running from rest along a circular
(a) that are all exactly R path of radius 6 m with constant tangential acceleration of
(b) that range from –R to R magnitude 2 m/s2. After 2 sec from start he feels that his
(c) that range from 0 to R shoes started slipping on ground. The friction between his
(d) that range from 0 to 2R shoes and ground is (Take g = 10 m/s2)
62. A cylinder of height h, diameter h/2 and mass M and with a
homogeneous mass distribution is placed on a horizontal (a) 1/2 (b) 1/3
table. One end of a string running over a pulley is fastened (c) 1/4 (d) 1/5
to the top of the cylinder, a body of mass m is hung from the 66. As shown in figure, the hinges A and B hold a uniform 400 N
other end and the system is released. Friction is negligible door in place. The upper hinge supports the entire weight
everywhere. At what minimum ratio m/M will the cylinder of the door. Find the resultant force exerted on the door at
tilt? the hinges. The width of the door is h/2, where h is the
h/2 distance between the hinges.
h M y

m
M
A

(a) 1 (b) 2
(c) 3 (d) 4
400N
63. A ring of radius R rolls without sliding with a constant
velocity with respect to ground. The radius of curvature of B
the path followed by any particle of the ring at the highest
point of its path will be
(a) 312 N (b) 280 N
(a) R (b) 2R
(c) 4R (d) None of these (c) 412 N (d) 480 N

MARK YOUR 60. 61. 62. 63. 64.


RESPONSE 65. 66.
CIRCULAR AND ROTATIONAL DYNAMICS 193

67. A bowling ball of mass m, which can be treated as a uniform (b) In comparison to , the angular speed of the disc now
rigid sphere, is rolling without slipping on a horizontal decreases
surface. The coefficient of static friction between the ball (c) In comparison to , the angular speed of the disc now
and the surface is µs, the coefficient of kinetic friction is µk, remains same
and the acceleration of gravity is g. What is the force of (d) The block will move tangentially and fall off the disc
friction acting on the ball ? 70. A metal sheet 14 cm × 2 cm of uniform thickness is cut into
(a) zero (b) µsmg two pieces of width 2 cm. The two pieces are joined and laid
2 along XY plane as shown. The centre of mass has the
(c) µkmg (d) µ mg coordinates
5 s
y
68. A thin, uniform square plate ABCD of side ‘a’ and mass
m = 1 kg is suspended in vertical plane as shown in the
figure. AE and BF are two massless inextensible strings.
The line AB is horizontal. Find the tension (in N) in the
string AE just after BF is cut. (Take g = 10 m/s2)

x
O
E F (a) (1, 1) (b) (7/2, 7/2)
(c) (13/4, 9/4) (d) (12/7, 8/7)
71. A particle moves in a circle of radius r = 4/3cm at a speed
A B given by v = 2.0 t2 where v is in cm/s and t in seconds. Find
the magnitude of the acceleration (in cm/s2) at t = 1s.
a
(a) 8 (b) 6
D C (c) 3 (d) 5
a
72. A light T bar, 10cm on each arm, rests between two vertical
walls, as shown in figure. The left wall is smooth, the
(a) mg/5 (b) 2mg/5 coefficients of static friction between the bar and floor, and
(c) 3mg/5 (d) 4mg/5 between the bar and right wall, are 0.35 and 0.50, respectively.
69. A smooth disc is rotating with uniform angular speed The bar is subjected to a vertical load of 1N, as shown.
What is the smallest value of the vertical force F for which
about a fixed vertical axis passing through its centre and
the bar will be in static equilibrium in the position shown ?
normal to its plane as shown. A small block of mass m is
µ1 = 0
gently placed at the periphery of the disc. Then (pickup the µ3 = 0.50
/////////////////////////////////////////////////////////////////////////////////
/////////////////////////////////////////////////////////////////////////////////

correct alternative or alternatives). F 5cm

6 cm
5cm 5cm

r m
8 cm
M 8 cm
5cm
1N 4 cm

µ2 = 0.35
///////////////////////////////////////////////////////////////////////////////
(a) In comparison to , the angular speed of the disc now 6 cm 5 cm

increases (a) 1/19 N (b) 2/19 N (c) 1/15 N (d) 20/19 N

MARK YOUR 67. 68. 69. 70. 71.


RESPONSE 72.
194 IIT-JEE P HYSICS Challenger
73. A 2.0 kg block is attached to one end of a spring with a (c) its speed is a minimum
spring constant of 100 N/m and a 4.0 kg block is attached to (d) it experiences a net upward force
the other end. The blocks are placed on a horizontal 77. Consider the following statements :
frictionless surface and set into motion. At one instant the
S1 : The locations of centre of mass and centre of gravity
2.0kg block is observed to be travelling to the right with a
may be different for an object.
speed of 0.5 m/s and the 4.0 kg block is observed to be
S2 : Internal forces can change the momentum of a non-
travelling to the left with a speed of 0.30 m/s. Since the only
rigid body.
force on the blocks is the force of gravity, the normal force
of the spring, we conclude that S3 : If the resultant force on a system of particles is non-
zero, then the distance of the centre of mass of system
(a) the spring is compressed at the time of the observation
may remain constant from a fixed point.
(b) the motion was started with the masses at rest
State, in order, whether S1, S2 and S3 are true or false
(c) the motion was started with at least one of the masses
moving (a) FTT (b) TFT (c) FFT (d) FTF
(d) the motion was started by compressing the spring 78. Two satellites S1 and S2 revolve around a planet in coplanar
74. An automobile of mass m is going around a curve in an arc circular orbits in the same sense. Their periods of revolution
of a circle of radius R at a speed v. The curve is banked at an are 1 hour and 8 hours respectively. The radius of the orbit
angle to the horizontal and the coefficient of static friction of S1 is 104 km. When S1 is closest to S2, the angular speed
between the tires and the road is µs. If is not very big, the of S2 as observed by an astronaut in S1 is
maximum speed the car can be moving without skidding is (a) rad/hr (b) /3 rad/hr
(c) 2 rad/hr (d) /2 rad/hr
gR (sin s cos ) gR (cos s sin ) 79. A hollow smooth uniform sphere A of mass m rolls without
(a) (b)
cos s sin cos s sin sliding on a smooth horizontal surface. It collides head on
elastically with another stationary smooth solid sphere B of
mgR (sin cos ) gR (cos ssin ) the same mass m and same radius. The ratio of kinetic energy
s
(c) (d) of B to that of A just after the collision is
sin s cos sin s cos
A B
75. A thin circular ring of mass ‘M and radius r is rotating about
its axis with a constant angular velocity , Two objects,
v0
each of mass m,, are attached gently to the opposite ends of
a diameter of the ring. The wheel now rotates with an angular
velocity
(a) 1 : 1 (b) 2 : 3
M ( M – 2 m) (c) 3 : 2 (d) None of these
(a) (b)
( M m) ( M 2 m) 80. What is the moment of inertia I of a uniform solid sphere of
mass M and radius R, pivoted about an axis that is tangent
M ( M 2 m) to the surface of the sphere
(c) (d)
(M 2m) M
76. A rectangular block is moving along a frictionless path when
it encounters the circular loop as shown. The block passes R
points 1, 2, 3, 4, 1 before returning to the horizontal track. At
point 3 : M

3
sphere
4 2
2 3
1 (a) MR 2 (b) MR 2
5 5
(a) its mechanical energy is a minimum 6 7
(b) it is not accelerating (c) MR 2 (d) MR 2
5 5

MARK YOUR 73. 74. 75. 76. 77.


RESPONSE 78. 79. 80.
CIRCULAR AND ROTATIONAL DYNAMICS 195

81. A bowling ball rolls without slipping down an inclined plane


3v 3v
inclined at an angle to the horizontal, as shown. The (a) (b)
coefficient of static friction between the ball and the surface 2 4
is µs, and the coefficient of kinetic friction is µk. What is the
magnitude of the force of friction acting on the ball ? 3 3v 3 3v
(c) (d)
4 2
R g
85. The center of mass of a quadrant of thin elliptical section
made of material of mass per unit area (figure) is

(a) zero (b) µs mg cos


Y
7
(c) µs mg sin (d)
2
mg sin x² y²
7
+ =1
5 a² b²
82. Two students were given a physics problem for finding
maximum extension of spring if blocks are imparted velocities
v1 and v2 when spring is unstretched.
v1 v2 b
m m y

1 1 2
By student A : m (v1 v2 ) 2 kx
2 2 X
dx
a
1 1 1 2
By student B : mv12 mv22 kx
2 2 2
(a) Student A is correct, Student B is wrong (a) 4a/ , 4b/3 (b) 4a/3 , 4b/
(b) Student B is correct, Student A is wrong
(c) Both are correct (c) 2a/3 , 4b/3 (d) 4a/3 , 4b/3
(d) Both are wrong 86. A box of mass 1 kg is mounted with two cylinders each of
5 mass 1 kg, moment of inertia 0.5 kg m2 and radius 1m as
83. A particle is moving along a circular path of radius r = m shown in figure. Cylinders are mounted on their control axis
with a uniform speed 5 m/s. What is the magnitude of average of rotation and this system is placed on a rough horizontal
acceleration (in m/s2) during the interval in which particle surface. The rear cylinder is connected to battery operated
completes half revolution ? motor which provides a torque of 100N-m to this cylinder
(a) 2 m/s2 (b) 10 m/s2 via a belt as shown. If sufficient friction is present between
(c) 20 m/s2 (d) 5 m/s2 cylinder and horizontal surface for pure rolling, find
84. A particle is moving with constant speed v m/s along the acceleration of the vehicle in m/s2. (Neglect mass of motor,
circumference of a circle of radius R meters as shown. A, B belt and other accessories of vehicle).
and C are three points on periphery of the circle and ABC
is equilateral. The magnitude of average velocity of particle,
as it moves from A to C clockwise sense, will be Electric motor
A v
m
R

//////////////////////////////////////

C B (a) 20 m/s2 (b) 10 m/s2


(c) 25 m/s2 (d) 30 m/s2

MARK YOUR 81. 82. 83. 84. 85.


RESPONSE 86.
196 IIT-JEE P HYSICS Challenger
87. The given figure shows a small mass connected to a string, 90. A uniform solid sphere of mass m is lying at rest between a
which is attached to a fixed vertical post. If the mass is vertical wall and a fixed inclined plane as shown. There is no
released from rest when the string is horizontal as shown, friction between sphere and the vertical wall but coefficient
the magnitude of the total acceleration of the mass as a of friction between the sphere and the fixed inclined plane is
function of the angle is µ = 1/2. Then the magnitude of frictional force exerted by
fixed inclined plane on sphere is
///////////// (g is acceleration due to gravity)
Initial
position

(a) 2g sin (b) 2g cos

vertical wall
(c) g 3cos2 1 (d) g 3sin 2 1
///
1/2 //// ne
88. A particle of mass m = 1 kg moves in a circle of radius R = 2m µ= /////// d pla
//// ine
with uniform speed v = 3 m/s. The magnitude of impulse /////// incl
/
given by centripetal force to the particle in one second is ////
30°
(a) 2 Ns (b) 3 Ns horizontal level

(c) 2 3 Ns (d) 3 2 Ns
89. A hoop of radius 0.10m and mass 0.50 kg rolls across a table mg 3
(a) (d) mg
parallel to one edge with a speed of 0.50 m/s. Refer its motion 2 4
to a rectangular coordinate system with the origin at the left (c) mg (d) 0
rear corner of the table. At a certain time t, a line drawn from 91. Two point masses of 0.3 kg and 0.7 kg are fixed at the ends
the origin to the point of contact of the hoop with the table of a rod of length 1.4 m and of negligible mass. The rod is set
has length 1m and makes an angle of 30° with the X-axis rotating about an axis perpendicular to its length with a
(figure). What is the spin angular momentum of the hoop uniform angular speed. The point on the rod through which
with respect to the origin at this time t ? the axis should pass in order that the work required for
rotation of the rod is minimum, is located at a distance of
(a) 0.42 m from mass of 0.3 kg
Z (b) 0.70 m from mass of 0.7 kg
(c) 0.98 m from mass of 0.3 kg
(d) 0.98 m from mass of 0.7 kg
30° 92. A cylinder A rolls without slipping on a plank B. The
Y velocities of center of the cylinder and that of the plank are
X vcm 4m/s and 2m/s respectively in same direction, with respect
to the ground. Find the angular velocity of the cylinder (in
rad/s) if its radius is 1m.

B
(a) 0.25 iˆ kg m 2 /s (b) 0.005 iˆ kg m 2 /s
Horizontal floor
(c) 0.025 iˆ kg m 2 /s (d) 0.5 iˆ kg m 2 /s
(a) 2 rad/sec (b) 4 rad/sec
(c) 6 rad/sec (d) 10 rad/sec

MARK YOUR 87. 88. 89. 90. 91.


RESPONSE 92.
CIRCULAR AND ROTATIONAL DYNAMICS 197

93. A small sphere A of mass m and radius r rolls without slipping


r r
inside a large fixed hemispherical bowl of radius (a) (b)
3 3
R (>> r) as shown in figure. If the sphere starts from rest at
the top point of the hemisphere find the normal force exerted r r
by the small sphere on the hemisphere when it is at the (c) (d)
2 2
bottom B of the hemisphere.
96. A uniform ladder of length L rests against a smooth
frictionless wall. The floor is rough and the coefficient of
A static friction between the floor and ladder is µ. When the
R ladder is positioned at angle , as shown in the
accompanying diagram, it is just about to slip. What is ?

////////////////////////////////
(a) cos = µ
B (b) tan = 2µ
1 L
(c) tan =
10 17 2
(a) mg (b) mg
7 7
1
5 7 (d) sin =
(c) (d) ////////////////////
mg mg
7 5 97. Two identical rings A and B are acted upon by torques A
94. A weight W rests on the bar AB as shown in figure. The and B respectively. A is rotating about an axis passing
cable connecting W and B passes over frictionless pulleys. through the centre of mass and perpendicular to the plane
If bar AB has negligible weight, the vertical component of 1
reaction at A is of the ring. B is rotating about a chord at a distance
2
////////// ////////// times the radius from the centre of the ring. If the angular
acceleration of the rings is the same, then
(a) A = B
(b) A > B
(c) A < B
(d) Nothing can be said about A and B as data are
a insufficient
98. A particle is moving along a circular path as shown in the
W figure. The instantaneous velocity of the particle is
B A
v (4m/s) iˆ (3m/s) ˆj . The particle is moving through
/////////
...... quadrant if it is travelling clockwise and through ..............
quadrant if it is travelling anticlockwise, respectively around
L a L a the circle
(a) W (b) W
L a L a Y

L 2a L a II I
(c) W (d) W
L a L 2a
95. A constant force acting at the centre of a uniform disc of X
radius r is always perpendicular to the plane of the disc.
III IV
The disc can rotate about a chord at a distance x from the
centre of the disc. For what value of x will the angular (a) First, first (b) First, second
acceleration of the disc be maximum ? (c) First, third (d) Third, first

MARK YOUR 93. 94. 95. 96. 97.


RESPONSE 98.
198 IIT-JEE P HYSICS Challenger
99. A uniform plank of weight W and total length 2L is placed as 103. A particle collides with a uniform rod at rest lying on a smooth
shown in figure with its ends in contact with the inclined horizontal plane. Initially the velocity of the particle is not
planes. The angle of friction is 15°. Determine the maximum along the length of the rod. Then the ratio of the velocity of
value of the angle at which slipping impends. the centre of the rod to its angular velocity will
/////

W (a) depend on the coefficient of restitution


/////

(b) depend on the masses of the particle and that of the


/////

L
/
/////

rod

////
L

////
// /////

(c) depend on the initial velocity of the particle

////
/

////
/////

(d) depend on the length of rod and the position of the


////
/
/////

//// point where the particle strikes


////
////

////

60°
104. A pulley of 1m radius, supporting a load of 500N is mounted
////

45°
at B on a horizontal beam as shown in figure. The beam
(a) 18.1° (b) 48.4°
weighs 200N and the pulley weighs 50N, find the hinge
(c) 36.2° (d) 88.8°
force at C.
100. A wheel of radius 0.1m (wheel A) is attached by a non-

//////////
stretching belt to a wheel of radius 0.2m (wheel B). The belt
does not slip. By the time wheel B turns through 1 revolution, 4
wheel A will rotate through 3

B A A C
B

///////// /////////

1 3m 1m 1m
(a) revolution (b) 1 revolution
2
(c) 2 revolution (d) 4 revolution
101. A right circular cone of base diameter 3 cm. and height 6cm. W
is cut from a solid cylinder of diameter 5cm. and height 12cm. 500N
Find the position of CG of rest of the body.
(a) 2.1 cm. (b) 6.3cm (c) 8.2 cm. (d) 5.3cm.
(a) 271.69 N (b) 671.69 N
102. Two identical bricks of length L are piled one on top of the
other on a table as shown in the figure. The maximum distance (c) 371.69 N (d) 471.69 N
S the top brick can overhang the table with the system still 105. A uniform rod AB of length three times the radius of a
balanced is hemisphered bowl remains in equilibrium in the bowl as
L shown. Neglecting friction find the inclination of the rod
with the horizontal.

///////////////////////////////////// r B
C
/////////////

S
3r
A
1 2 ///////////////////////////////
(a) L (b) L
2 3

3 7 (a) sin–1 (0.92) (b) cos–1 (0.92)


(c) L (d) L
4 8 (c) cos–1 (0.49) (d) tan–1 (0.92)

MARK YOUR 99. 100. 101. 102. 103.


RESPONSE 104. 105.
CIRCULAR AND ROTATIONAL DYNAMICS 199

106. Find the moment of force about point A as shown in the


(a) mh/bg (b) mb 2 gh
figure.
(c) mh 2 gh (d) None of these

6kN 108. A frustum of a solid right circular cone has a base diameter
of 20cm, top diameter of 10cm. and height 20cm. It has an
axial cylindrical hole of diameter 5cm. Determine the position
///////////////////////////////////

C 30°
of centre of gravity of this body

10cm
0.3

20cm
A 0.5m B 5

(a) 0.06 kNm clockwise 20cm


(b) 0.06 kNm anticlockwise (a) 7.6cm. (b) 4.3cm
(c) 0.03 kNm clockwise (c) 12.6cm. (d) 15.3 cm.
(d) 0.03 kNm anticlockwise 109. Consider the following statements :
107. A particle of mass m is released from rest at point A in the S1 : Zero net torque on a body means absence of rotational
figure falling freely under gravity parallel to the vertical Y- motion of the body.
axis. The magnitude of angular momentum of particle about S2 : A particle may have angular momentum even though
point O when it reaches B is the particle is not moving in a circle.
(where OA = b and AB = h) S3 : A ring of rolling without sliding on a fixed surface. The
O b centripetal acceleration of each particle with respect to
A
the centre of the ring is same.
State in order, whether S1, S2, S3 are true or false.
h (a) FTT (b) FFT
(c) TTF (d) FTF

Y
B

MARK YOUR
106. 107. 108. 109.
RESPONSE
200 IIT-JEE PHYSICS Challenger

PASSAGE-1 PASSAGE-2

Two blocks of equal mass m are connected by an unstretched


B
spring and the system is kept at rest on a frictionless
horizontal surface. A constant force F is applied on the first
F –F
block pulling it away from the other as shown in figure.
IB k
m m F
IA
4 The displacement of the centre of mass at time t is
Ft 2 Ft 2
(a) (b)
F –F 2m 3m

Ft 2 Ft 2
(c) (d)
4m m
I A+ I B 5. If the extension of the spring is x0 at time t, then the
displacement of the first block at this instant is
The figure has two disks : one an engine flywheel, and the
other a clutch plate attached to a transmission shaft. Their 1 Ft 2 1 Ft 2
(a) x0 (b) x0
moments of inertial are IA and IB; initially, they are rotating 2 2m 2 2m
with constant angular speeds A and B, respectively. We
then push the disks together with forces acting along the 1 Ft 2 Ft 2
axis, so as not to apply any torque on either disk. The disks (c) x0 (d) x0
2 2m 2m
rub against each other and eventually reach a common final
angular speed . Suppose flywheel A has a mass of 2.0 kg, a 6. If the extension of the spring is x0 at time t, then the
radius of 0.20 m and an initial angular speed of 50 rad/sec. displacement of the second block at this instant is
(about 500 rpm) and that clutch plate B has a mass of 4.0 kg,
a radius of 0.10 m, and an initial angular speed of 200 rad/sec. Ft 2 1 Ft 2
(a) x0 (b) x0
1. Find an expression for ? 2m 2 2m
IA A IB B IA A IB B
(a) (b) 1 2 Ft 2 1 Ft 2
IA IB IA IB (c) x0 (d) x0
2 m 2 2m
IA A IB B IA A IB B
(c) IA + IB (d) I A + IB
PASSAGE-3
2. Find the common final angular speed after the disks are
pushed into contact?
Rigid uniform L-shaped rod AOB has mass 2m and is free to
(a) 10 rad/sec (b) 100 rad/sec
rotate about a fixed point O on a horizontal frictionless plane.
(c) 1000 rad/sec (d) .010 rad/sec
Now massless rigid rod CD is connected at end B of L-
3. What happens to the final kinetic energy during this
shaped rod such that it can freely rotate about B, as shown
process?
in the figure. Two masses, of mass m each, are connected to
(a) 300 J (b) 3 J ends C and D. Now an impulse is given at point A
(c) 30 J (d) 3000 J (perpendicular to OA) such that the total assembly gets an
initial angular speed .

MARK YOUR 1. 2. 3. 4. 5.
RESPONSE 6.
CIRCULAR AND ROTATIONAL DYNAMICS 201

A force F = 55 N is applied on the plank such that the plank


l moves and causes the cylinder to roll. The plank always
A remains horizontal. There is no slipping at any point of
O contact.
10. Calculate the acceleration of cylinder.
(a) 20 m/s2 (b) 10 m/s2
l
(c) 5 m/s2 (d) None of these
11. Find the value of frictional force at A
(a) 7.5 N (b) 5.0 N
B (c) 2.5 N (d) None of these
C D 12. Find the value of frictional force at B
l l (a) 7.5 N (b) 5.0 N
(c) 2.5 N (d) None of these
7. Angular velocity of AO will
(a) be constant at .
(b) be varying, with least value PASSAGE-5
(c) be varying, with maximum value .
(d) linearly drop to zero with time n identical rods each of mass m are welded at their ends to
8. Distance between A and C will form a regular polygon and the corners are then welded to a
(a) be constant at 5 metal ring of radius R and mass M, such that the plane of
(b) be varying, with max value 3 polygon and plane of ring are in same plane and centres of
polygon and ring coincide.
(c) be varying, with max value 5 13. The moment of inertia of the system about an axis passing
through the centre of mass of system and perpendicular to
(d) be varying, with max value 2 1
the plane of system will be
9. Magnitude of relative velocity of A w.r.t. C
(a) be constant at 2 sin 2
nmR 2 n cos 2 MR 2
(b) keeps varying, with a maximum value (a)
3 n
(c) keeps varying, with a max value 2

(d) keeps varying, with a maximum value 2 1


tan 2
(b) nmR 2 n sin 2 MR 2
PASSAGE-4 4 n
Consider a cylinder of mass M = 1kg and radius R = 1 m lying
on a rought horizontal plane. It has a plank lying on its stop
as shown in the figure. sin 2
(c) nmR 2 n cos 2 MR 2
4 n
F
m = 1kg 60°
A
sin 2 cos 2
(d) nmR 2 n n MR 2 .
M R 4 3

MARK YOUR 7. 8. 9. 10. 11.


RESPONSE 12. 13.
202 IIT-JEE P HYSICS Challenger
14. The moment of inertia of system about any one of the corner 16. The moment of inertia of the combined ring system about
of polygon and perpendicular to its plane will be the centre of ring will be

sin 2
(a) nmR 2 n cos 2 1 MR 2 . F
3 n

sin 2 M
(b) nmR 2 n cos 2
1 2 MR 2
3 n
12 48
(a) MR 2 (b) MR 2
5 15

sin 2 24 48
(c) MR 2 (d) MR 2
(c) nmR 2 n cos 2
2 2 MR 2 5 5
3 n
17. The magnitude of F equals
(a) Mg (b) 2Mg
sin 2 Mg
(d) nmR 2 n cos 2 4 MR 2 . (c) (d) none of these
3 n 2
18. If the masses of the spheres were doubled keeping their
15. If the rigid assembly of rods and hoop is allowed to roll dimensions same, the force of friction between the ring and
down the incline of inclination , the minimum value of the the horizontal surface would
coefficient of static friction that will prevent slipping will be (a) be doubled
(b) increase but be less than double
I tan I sin
(a) (b) (c) remain the same
2I (M nm) R 2 I (M nm) R 2 (d) decrease
I cos I tan
(c) (d) .
2I ( M nm) R 2
I (M nm) R 2 PASSAGE-7
(where I is moment of inertia about centre of mass)
A disc of radius R is spun to an angular speed 0 about
PASSAGE-6 0R
its axis and then imparted a horizontal velocity v0 =
4
Four identical spheres having mass M and radius R are fixed (at t = 0) with its plane remaining vertical. The coefficient
tightly within a massless ring such that the centres of all of friction between the disc and plane is µ. The direction
spheres lie in the plane of ring. The ring is kept on a rough of v0 and 0 are shown in the figure.
horizontal table as shown. The string is wrapped around
the ring which can roll without slipping. The other end of
the string is passed over a massless frictionless pulley to a v
block of mass M. A force F is applied horizontally on the
ring, at the same level as the centre, so that the system is in
equilibrium. 19. The disc will return to starting point at time

25 R 0 5 R 0
(a) (b)
48 µ g 12 µ g

5 R 0 R 0
(c) (d)
48 µ g 6µg

MARK YOUR 14. 15. 16. 17. 18.


RESPONSE 19.
CIRCULAR AND ROTATIONAL DYNAMICS 203

20. The disc will start rolling without slipping at time (c) 2

(d) the wheel will continue to roll without slipping forever,


R 0 5R 0
(a) (b) independent of .
6µg 12µg

R 0 R 0 PASSAGE-9
(c) (d)
µg 4µg
21. The angular momentum of the disc about the point of A uniform rod AB of length 2 falls without rotation on a
contact, when slipping ceases is equal to smooth horizontal surface at an angle to the horizontal.
The speed of rod just before collision is v0 and the collision
MR 2 0 is elastic. The magnitude of the angular velocity and
(a) MR 2 0 (b)
2 12 magnitude of the velocity of centre of mass after collision
are and v' respectively.
(c) MR 2 0
(d) MR 2 0
B
4 3

PASSAGE-8 A

A uniform disc rolls on a rough horizontal surface without 25. The direction of force of impact on the rod is
slipping. It starts rolling with a small initial velocity v0, and (a) along the surface
is continuously acted upon by a torque (provided through (b) vertically upward
the axle), that delivers constant power. The initial kinetic (c) along the rod
energy of the disc can be ignored. (d) any direction is possible
26. The relation between v0, and v' is
torque
(a) v0 = v' (b) v0 = v' +
(c) v0 = v' – (d) v0 = v' + cos .
27. The angular momentum before collision about the point on
the ground at which the rod strikes has the magnitude (m :
mass of rod)
Rough horizontal surface mv0
(a) (b) mv0 cos
22. The velocity v, of the centre of the wheel varies with time (t) 2
according to (assume that t is large enough) mv0 m v0
(c) (d) .
1 2cos 3
(a) v t (b) v
t

. v
1 PASSAGE-10
(c) v t (d)
t2
23. If the magnitude of the force of friction acting on the wheel Rolling is the combination of translation and rotation. The
be f when it travelled a distance x (which is large), then point of contact plays a crucial role as it decides the direction
(a) f = constant of friction. A uniform disc of mass M and radius R is given a
(b) fx = constant linear velocity v0 and angular speed 0 as shown in the
(c) f 2 x = constant diagram on a rough horizontal surface.
(d) f 3 x = constant.
24. If the coefficient of static friction between the wheel and the 0
horizontal surface be , the time for which the wheel rolls v0 2v0
without slipping is proportional to 0=
t=0 R
(a) 1/2 R
(b) –2 Rough

MARK YOUR 20. 21. 22. 23. 24.


RESPONSE 25. 26. 27.
204 IIT-JEE P HYSICS Challenger
28. The graph of angular speed of disc about its centre is best PASSAGE-11
represented by
A particle continuously moves in a circular path at constant
speed in a counter clockwise direction. Consider a time
interval during which the particle moves along this circular
path from point P to point Q. Point Q is exactly half-way
around the circle from point P.
(a) (b) 31. What is the direction of the average velocity during this
t0 t t time interval ?

Q P

(c) (d) (a) (b)


t0 t t0 t

(c) (d)
29. The disc is now replaced by a uniform solid sphere of mass
M and radius R. Again sphere is given same velocity and 32. What is the direction of the average acceleration during
angular speed under same situation. Then group of angular this time interval ?
speed about centre vs time ‘t’ is given by (a) (b)

(c) (d)

(a) (b)
t t PASSAGE-12

In the figure shown a uniform solid sphere is released on


the top of a fixed inclined plane of inclination of 37° and
height h. It rolls without sliding. (Take sin 37° = 3/5 and g is
acceleration due to gravity)

(c) (d)
t t
fixed h
37°
/////////////////////////////////////////
30. The velocity of centre of mass of sphere in previous question 33. The acceleration of the centre of the sphere is
when pure rolling starts will be equal to (a) 3g/5 (b) 4g/5 (c) 4g/7 (d) 3g/7
34. The speed of the point of contact of the sphere with the
2v0 5 v0 inclined plane when the sphere reaches the bottom of the
(a) (b)
5 6 incline is
10 gh
v0 (a) 2gh (b)
(c) (d) zero 7
7
(c) zero (d) 2 2gh

MARK YOUR 28. 29. 30. 31. 32.


RESPONSE 33. 34.
CIRCULAR AND ROTATIONAL DYNAMICS 205

35. The time taken by the sphere to reach the bottom is 37. The least tension in string is
2h 70h mg mg
(a) (b) (a) (b)
g 9g 6 5
25h 25h 3 mg
(c) (d) (c) mg (d)
18 g 6g 2 2
38. Tangential acceleration of block at highest position is

PASSAGE-13 g 3
g
(a) (b)
2 2
The cross-section of a fixed cylinder (not allowed to rotate
g g
and translate) with horizontal axis is as shown. One end of a (c) (d)
light inelastic string is fixed at top of cylinder of radius R 3 6
and a small block of mass m is tied to the other end of string.
Initially the block is at rest with the portion of string not in
PASSAGE-14
contact with cylinder being vertical and having length L as
shown. At the lowest position the block is given initial
An 80 kg cyclist riding a 16 kg bicycle is travelling along a
horizontal velocity u = 2 gL and the block moves in city street at 18 km / hour. A taxi pulls in front of him and
vertical plane. When the block reaches the highest point of stops suddenly to pick up a passenger. By the time the
its trajectory, the length of string not in contact with cylinder cyclist assesses the situation and begins to apply his brakes,
he is 5 m away from the taxi. The cyclist knows from
R
is L + (where g is acceleration due to gravity). experience that if he jams on the brakes and his wheels skid,
3 he will have less braking force, so he avoids this. The brake
pads have a surface area of 8 mm by 70mm and a coefficient
of friction of 0.5, the bicycle has 700 mm diameter wheels
and the diameter of the middle of the rim (where the brake
R
fixed pads are applied) is 650 mm.
cylinder L 39. If the braking force is distributed equally between the front
and back wheels, calculate the force that must be applied to
each pad in order to stop in time.
m u = 2 gL (a) 129 N (b) 25.8 N
(c) 240 N (d) 258 N
36. The distance between block and centre of cylinder when 40. If the brake lever (on the handlebar) moves 2 cm for each
block is at highest position will be brake pad to move 1 mm, calculate the force which must be
(a) 2R (b) 5R applied to the brake lever.
(a) 129 N (b) 25.8 N
(c) 3R (d) 2R / 3 (c) 240 N (d) 258 N

MARK YOUR 35. 36. 37. 38. 39.


RESPONSE 40.
206 IIT-JEE P HYSICS Challenger
1. Statement - 1 : A rigid disc rolls without slipping on a fixed 5. Statement - 1 : Radius of gyration of a body is not a
rough horizontal surface with uniform constant quantity.
angular velocity. Then the acceleration of Statement - 2 : The radius of gyration of a body about an
lowest point on the disc is zero. axis of rotation may be defined as the root
Statement - 2 : For a rigid disc rolling without slipping on a mean square distance of the particle from
fixed rough horizontal surface, the velocity the axis of rotation.
of the lowest point on the disc is always 6. Statement - 1 : Two bodies A and B are attracted towards
zero. each other due to gravitation. If A is much
2. Statement - 1 : A uniform thin rod of length L is hinged heavier than B then the center of mass of
about one of its end and is free to rotate the bodies moves towards A.
about the hinge without friction. Neglect Statement - 2 : The centre of mass depends upon mass
the effect of gravity. A force F is applied at distribution of a body or a system of bodies.
a distance x from the hinge on the rod such 7. Statement - 1 : A wheel moving down a perfectly
that force always is perpendicular to the frictionless inclined plane will undergo
rod. As the value of x is increased from slipping (not rolling motion).
zero to L, the component of reaction by Statement - 2 : For perfect rolling motion, work done
hinge on the rod perpendicular to length against friction is zero.
of rod increases. 8. Statement - 1 : Centrifugal force is reaction force of the
Statement - 2 : Under the conditions given in statement 1 centripetal force.
as x is increased from zero to L, the angular Statement - 2 : A rotating frame of reference is a non-
acceleration of rod increases. inertial frame.
3. Statement - 1 : If two different axes are at same distance 9. Statement - 1 : In non-uniform circular motion, velocity
from centre of mass of a rigid body, then vector and acceleration vector are not
moment of inertia of the given rigid body perpendicular to each other.
about both axis will always be same. Statement - 2 : In non-uniform circular motion, particle has
Statement - 2 : From parallel axis theorem I = Icm + md2, normal as well as tangential acceleration.
where all terms have usual meaning.
4. Statement - 1 : A uniform cubical block (of side a) 10. Statement - 1 : When a body rolls on a rough surface,
undergoes translational motion on a friction force is always zero.
smooth horizontal surface under action of Statement - 2 : A particle cannot roll on a surface.
horizontal force F as shown. 11. Statement - 1 : A particle is moving in circular path. The
a net work done on the particle is zero.
Statement - 2 : For a particle undergoing uniform circular
motion, net force acting on the particle and
a velocity of the particle are always
F
a/4 perpendicular.
12. Statement - 1 : If two different axes are at same distance
Under the given condition, the horizontal from centre of mass of a rigid body, then
surface exerts normal reaction non- moment of inertia of the given rigid body
uniformly on lower surface of the block. about both axes will always be same.
Statement – 2 : For the cubical block given in statement-1, Statement - 2 : From parallel axis theorem I = Icm + md 2,
the horizontal force F has tendency to where all terms have usual meaning.
rotate the cube about its centre in clockwise 13. Statement - 1 : Kinetic energy of a system is minimum in
sense. Hence, the lower right edge of cube centre of mass frame of reference.
presses the horizontal surface harder in Statement - 2 : In centre of mass frame kinetic energy of
comparison to the force exerted by lower all particles is smaller than their respective
left edge of cube on horizontal surface. kinetic energy in ground frame.

1. 2. 3. 4. 5.
MARK YOUR 6. 7. 8. 9. 10.
RESPONSE
11. 12. 13.
CIRCULAR AND ROTATIONAL DYNAMICS 207

14. Statement - 1 : A cyclist is cycling on rough horizontal 18. Statement - 1 : A rigid disc rolls without slipping on a fixed
circular track with increasing speed. Then rough horizontal surface with uniform
the frictional force on cycle is always angular velocity. Then the acceleration of
directed towards centre of the circular lowest point on the disc is zero.
track. Statement - 2 : For a rigid disc rolling without slipping on
Statement - 2 : For a particle moving in a circle, radial a fixed rough horizontal surface, the
component of net force should be directed velocity of the lowest point on the disc is
towards centre. always zero.
15. Statement - 1 : If two different axes are at same distance 19. A uniform thin rod of length L is hinged about one of its end
from centre of mass of a rigid body, then and is free to rotate about the hinge without friction. Neglect
moment of inertia of the given rigid body the effect of gravity. A force F is applied at a distance x from
about both axis will always be same. the hinge on the rod such that force is always perpendicular
to the rod. As the value of x is increased from zero to L,
Statement - 2 : From parallel axis theorem I = Icm + md2,
where all terms have usual meaning. F
16. Statement - 1 : KE of rotating rigid body in CM frame is
1 x
I cm 2 , where symbols have usual
2 Statement - 1 : The component of reaction force by hinge
meaning. on the rod perpendicular to length of rod
Statement - 2 : In CM frame rigid body has pure rotational increases.
motion. Statement - 2 : The angular acceleration of rod increases.
20. Statement - 1 : Net external torque ( ext) on a system of
17. Statement - 1 : If net force F acting on a system is particles is equal to rate of change of
changing in direction only, the linear
dL
momentum ( p) of system changes in angular momentum , if ext and L are
dt
direction.
measured with respect to any fixed point
Statement - 2 : In case of uniform circular motion,
in an inertial frame.
magnitude of linear momentum is constant
Statement-2 : If a body is in rotational equilibrium, then
but direction of centripetal force changes
net torque on a body about any fixed point
at every instant.
is zero.

MARK YOUR 14. 15. 16. 17. 18.


RESPONSE 19. 20.

1. As shown in figure, a planner assembly, having six rods, (a) Iz will have its highest value for = 45°
each of mass m is lying in x-y plane with O at origin, lengths (b) Iz will have its highest value for = 90°
of AD and BC are . If Iz denotes the moment of inertia of (c) Iz will have its highest value for = 0°
the assembly about z-axis and Iy denotes moment of inertia (d) Iy will have its highest value for = 90°
about y-axis. 2. A particle is moving along an expanding spiral in such a
manner that magnitude of normal acceleration of particle
A B remains constant. Choose the correct options
y (a) Linear speed of particle is increasing
(b) Linear speed of particle is decreasing
x (c) Angular speed of paricle is increasing
O
(d) Angular speed of particle is decreasing
C D

MARK YOUR
1. 2.
RESPONSE
208 IIT-JEE P HYSICS Challenger
3. The moment of inertia of a hollow cylinder of mass M and
inner radius R1 and outer radius R2 about its central axis is A
1 1 R
(a) M ( R22 – R12 ) (b) M ( R12 R22 )
2 2

1
(c) M ( R12 – R22 ) M ( R2 – R1 )2
(d) B
2
4. A uniform rod moves in a vertical circle. Its ends are
constrained to move on the track without friction. The
angular frequency of small oscillation is given by
(a) its velocity v on arriving at B is proportional to cos .
R (b) its velocity v on arriving at B is proportional to tan .
O (c) time to arrive at B is proportional to cos .
d (d) time to arrive at B is independent of .
7. A small ball is connected to a block by a light string of
L length . Both are initially on the ground. There is sufficeint
friction om the ground to prevent the block from slipping.
The ball is projected vertically up with a velocity u, where
1/ 2 2g < u2 < 3g . The centre of mass of the block + ball
2 L2 2 L2
R R – system is C.
4 4
(a) g (b) g
L2
2 L2
2
R – R –
4 6

(a) C will move along a circle.


2 (b) C will move along a parabola
L
R2
(4 R 2 2
L ) 6 (c) C will move along a straight line
(c) g (d) g
2
(R – L ) 2 2
L (d) The horizontal component of the velocity of the ball
R2 – will be maximum when the string makes an angle q =
6
sin–1 (u2/3g ) with the horizontal
5. A torque on a body about a given point is found to be 8. A wheel is rolling on a horizontal plane without slipping. At
a certain instant, it has velocity ‘v’ and acceleration ‘a’ of
equal to C L , where C is a constant vector and L is the
c.m. as shown in the figure. Acceleration of
angular momentum of the body about that point. From this,
it follows that C
(a) L does not change with time. a
B
dL v
(b) is perpendicular to L at all instants of time. A
dt

(c) the magnitude of L does not change with time. (a) A is vertically upwards
(d) all the above (b) B may be vertically downwards
6. A bead is free to slide down a smooth wire tightly stretched (c) C cannot be horizontal
between the points A and B on a fixed vertical circle. If the
(d) Some point on the rim may be horizontal leftwards.
bead starts from rest at A, the highest point on the circle.

MARK YOUR 3. 4. 5. 6. 7.
RESPONSE 8.
CIRCULAR AND ROTATIONAL DYNAMICS 209

9. A ball of mass 1 kg is thrown up with an initial speed of (c) The tangential acceleration of the upper end of the
4 m/s. A second ball of mass 2 kg is released from rest from pole is 2k (1 – cos )
some height as shown in the figure. (d) The centripetal acceleration of the upper end of the
pole is k sin
u=0 2 kg 12. A thin rod AB of mass M and length L is rotating with angular
speed 0 about vertical axis passing through its end B on a
4 m/s horizontal smooth table as shown. If at some instant the
hinge at end B of rod is opened then which of the following
1 kg statement is/are correct about motion of rod ?
A M
(a) The centre of mass of the two balls comes down with
L
acceleration g/3.
(b) The centre of mass first moves up and then comes
down B
(c) The acceleration of the centre of mass is g downwards 0
(d) The centre of mass of the two balls remains stationary.
10. A disc is given an initial angular velocity 0 and placed on
rough horizontal surface as shown. The quantities which (a) The angular speed of rod after opening the hinge will
will not depend on the coefficient of friction is/are remain 0.
(b) The angular speed of rod after opening the hinge will
be less than 0.
(c) In the process of opening the hinge the kinetic energy
of rod will remain conserved.
(d) Angular momentum of rod will remain conserved about
centre of mass of rod in the process of opening the
(a) The time until rolling begins. hinge.
(b) The displacement of the disc until rolling begins. 13. A rod leans against a stationary cylindrical body as shown
(c) The velocity when rolling begins. in figure, and its right end slides to the right on the floor
(d) The work done by the force of friction. with a constant speed v. Choose the correct option(s).
11. The angular acceleration of the toppling pole shown in figure
is given by = k sin , where is the angle between the axis
of the pole and the vertical, and k is a constant. The pole
starts from rest at = 0. Choose the correct options R R v

x
as
an Rv 2 (2 x 2 R2 )
(a) the angular speed is
x2 ( x2 R 2 )3 / 2
Rv
(b) the angular acceleration is
x x2 R2

O Rv
(c) the angular speed is
(a) The tangential acceleration of the upper end of the
x x2 R2
pole is k sin
(b) The centripetal acceleration of the upper end of the Rv 2 (2 x 2 R2 )
pole is 2k (1 – cos ) (d) the angular acceleration is
x2 ( x2 R 2 )3 / 2

MARK YOUR
9. 10. 11. 12. 13.
RESPONSE
210 IIT-JEE P HYSICS Challenger
14. A small object moves counter clockwise along the circular 18. The uniform 120 N board shown in figure is supported by
path whose centre is at origin as shown in figure. As it two ropes. A 400 N weight is suspended one-fourth of the
moves along the path, its acceleration vector continuously way from the left end. Choose the correct options
points towards point S. Then the object
y
B
30°
T2 T1

x
C S O A
0.25 L 0.75 L

400 N
D
(a) Speeds up as it moves from A to C via B
(a) T1 = 185 N (b) T2 = 371 N
(b) Slows down up as it moves from A to C via B
(c) T2 = 185 N (d) tan = 0.257
(c) Slows down as it moves from C to A via D
19. Neglecting the weight of the beam in figure, choose the
(d) Speeds up as it moves from C to A via D
correct options
15. Which of the following are not correct about centre of mass?

////////////////////////////////////
(a) Centre of mass of a system of four particles in a plane
must lie within the quadrilateral formed by the four 70°
particles. 0.2L
(b) In centre of mass frame momentum of a system is always
zero. 50°
0.8L
(c) Internal force may affect the motion of centre of mass. W
(d) Centre of mass and centre of gravity are synonymous
in all situations.
16. A ball tied to the end of a string swings in a vertical circle (a) Tension in the tied rope is 1.80 W
under the influence of gravity. (b) The force components at the hinge are 1.69 W, 1.62 W
(a) When the string makes an angle 90° with the vertical (c) If the uniform beam weighs W/2 then T = 2.35 W
the tangential acceleration is zero and radial acceleration (d) Force component at the hinge are 2.21 W and 2.30 W if
is somewhere between maximum and minimum. uniform beam weighs W/2.
(b) When the string makes an angle 90° with the vertical 20. A light rope passes over a light frictionless pulley attached
the tangential acceleration is maximum and radial to the ceiling. An object with a large mass is tied to one end
acceleration is somewhere between maximum and and an object with a smaller mass is tied to the other end.
minimum. Both masses are released from rest. Which of the following
(c) At no place in the circular motion, tangential statement (s) is/are false for the system consisting of the
acceleration is equal to radial acceleration. two moving masses while string remains taut?
(d) Throughout the path whenever radial acceleration has
//////////////////////
its extreme value, the tangential acceleration is zero.
17. A cylinder rolls without slipping on a rough floor, moving
with a speed v. It makes an elastic collision with smooth
vertical wall. After impact
M m
(a) it will move with a speed v initially
(b) its motion will be rolling without slipping (a) the centre of mass remains at rest
(c) its motion will be rolling with slipping initially and its (b) the net external force is zero
rotational motion will stop momentarily at some instant. (c) the velocity of the center of mass is a constant
(d) its motion will be rolling without slipping only after (d) the acceleration of the center of mass is g downward
some time

MARK YOUR 14. 15. 16. 17. 18.


RESPONSE 19. 20.
CIRCULAR AND ROTATIONAL DYNAMICS 211

21. Consider a woman lifting a 60N bowling ball as shown in (b) The magnitude of torque of the gravitational force on
figure (a). Approximate the situation as shown in figure (b) the particle about O is decreasing
and assume the upper part of her body to weigh 250N with (c) The moment of inertia of the particle about O is
center of gravity as indicated. Choose the correct options decreasing
(d) The magnitude of angular velocity of the particle about
Back muscle O is increasing
24. Four samples of a colloidal aqueous mixture each weighing
Pivot point
12.0 g are placed in the rotor of a high speed centrifuge,
Hip Tm equally spaced around the circumference of the rotor. The
12° H
samples are located at 10 cm from the axis of rotation of the
250 N rotor. The centrifuge motor delivers a constant torque of
V 0.25 Nm and the empty rotor has a moment of inertia of 0.06
60 N L/2 kg m2. Choose the correct options
2L/3
L
(b) Sample

(a)

(a) The tension in her back muscle is 1335 N


(b) The compressional force in her spine when her back is
horizontal is 1305 N
(c) The tension in her back muscle is 1305 N 20cm.
(d) The compressional force in her spine when her back is Cross section of rotor
horizontal is 1335 N
22. A sphere of radius 0.10m and mass 10 kg rests in the corner (a) In 456.2 sec. rotor accelerate to its operating state of
formed by a 30° inclined plane and a smooth vertical wall. 18,000 rpm (rotations per minute).
Choose the correct options (b) When the centrifuge is up to speed 18,000 rpm the
force exerted on the sample by the rotor is 2264 N
(c) In 756.2 sec. rotor accelerate to its operating state of
18,000 rpm (rotations per minute).
(d) When the centrifuge is up to speed 18,000 rpm the
force exerted on the sample by the rotor is 4264 N
f 25. A horizontal beam PQRS is 12m long. Forces of 1kN, 1.5kN,
mg N1
1 kN and 0.5 kN act at P, Q, R, S respectively in downward
direction. The line of action of these forces make angle of
30° 90°, 60°, 45°, 30° respectively with line PS. If PQ = QR = RS
= 4m, choose the correct options related to resultant force.
1 kN 1.5 kN 1 kN
0.5 kN
N2
(a) N1 = 56.5 N 90° 60° 45°
Q R 30°
(b) N2 = 113 N P
S
(c) f = 0
(d) f 0 4m 4m 4m
23. A particle falls freely near the surface of the earth. Consider (a) the magnitude of resultant force is 3.77 kN
a fixed point O (not vertically below the particle) on the (b) position of the resultant force is 3.67m from P
ground. Then pickup the correct alternative or alternatives. (c) the magnitude of resultant force is 1.77 kN
(a) The magnitude of angular momentum of the particle (d) position of the resultant force is 1.67m from P
about O is increasing

MARK YOUR
21. 22. 23. 24. 25.
RESPONSE
212 IIT-JEE P HYSICS Challenger
26. A tube of length L is filled completely with an incomressible
dL
liquid of mass M and closed at both the ends. The tube is (a) is perpendicular to L at all instants of time.
dt
then rotated in a horizontal plane about one of its ends with
(b) the component of L in the direction of A does not
a uniform angular velocity . The force exerted by the liquid
change with time.
at the other end is
(c) the magnitude of L does not change with time.
M 2
L (d) L does not change with time
(a) (b) M 2L 30. A solid cylinder is rolling down a rough inclined plane of
2
inclination . Then
2 2 2 (a) The friction force is dissipative
M L M L
(c) (d) (b) The friction force is necessarily changing
4 2
(c) The friction force will aid rotation but hinder translation
27. A car is moving in a circular horizontal track of radius 10 m (d) The friction force is reduced if is reduced
with a constant speed of 10 m/s. A pendulum bob is 31. A particle is moving along a circular path. The angular
suspended from the roof of the car by a light rigid rod of velocity, linear velocity, angular acceleration and centripetal
length 1.00 m. The angle made by the rod with track is acceleration of the particle at any instant respectively are
(a) zero (b) 30° , v, and ac . Which of the following relations is/are
(c) 45° (d) 60° correct ?
28. Let I be the moment of inertia of a uniform square plate (a) (b)
.v 0 . 0
about an axis AB that passes through its centre and is parallel
(c) .ac 0 (d) v .ac 0
to two of its sides. CD is a line in the plane of the plate that
passes through the centre of the plate and makes an angle 32. If the resultant of all the external forces acting on a system
with AB. The moment of inertia of the plate about the axis of particles is zero, then from an inertial frame, one can surely
CD is then equal to say that
(a) linear momentum of the system does not change in
(a) I (b) I sin2
time
(c) I cos2 (d) I cos2 ( /2)
(b) kinetic energy of the system does not change in time
29. The torque on a body about a given point is found to be
(c) angular momentum of the system does not change in
equal to A × L where A is a constant vector, and L is the time
angular momentum of the body about that point. From this (d) potential energy of the system does not change in time
it follows that

MARK YOUR 26. 27. 28. 29. 30.


RESPONSE 31. 32.
CIRCULAR AND ROTATIONAL DYNAMICS 213

1. A particle of 500 gm mass moves along a horizontal circle of radius 16 m such that normal acceleration of particle varies with time
as an = 9t2
Column I Column II
(A) Tangential force on particle at t = 1 second (in newton) (p) 72
(B) Total force on particle at t = 1 second (in newton) (q) 36
(C) Power delivered by total force at t = 1 sec (in watt) (r) 7.5
(D) Average power developed by total force over first one (s) 6
second (in watt)
2. A rigid body of mass M and radius R rolls without slipping on an inclined plane of inclination , under gravity. Match the type
of body with magnitude of the force of friction.
Column I Column II

(A) For ring (p) Mg sin


2.5

Mg sin
(B) For solid sphere (q)
3

Mg sin
(C) For solid cylinder (r)
3.5

Mg sin
(D) For hollow spherical shell (s)
2
3. Column I Column II
(A) Axial vector (p) Rotational K.E.
(B) Scalar quantities (q) Translational K.E.
(C) Turning ability of force (r) Angular momentum
(D) A rolling body can have (s) Torque
4. A rigid body is rolling without slipping on horizontal surface. At a given instant BD is perfectly horizontal and CD is perfectly
vertical.
C
= v/R

D R
B

Column I Column II
(A) Velocity at point A; vA (p) v 2
(B) Velocity at point B; vB (q) Zero
(C) Velocity at point C; vC (r) v
(D) Velocity at point D; vD (s) 2v

1. 2. 3. 4.

MARK YOUR
RESPONSE
214 IIT-JEE P HYSICS Challenger
5. A uniform disc is acted upon by some forces and it rolls on a horizontal plank without slipping from north to south. The plank,
in turn lies on a smooth horizontal surface. Match the following regarding this situation
Column I Column II
(A) Frictional force on the disc by the surface (p) May be directed towards north
(B) Velocity of the lowermost point of the disc (q) May be directed towards south
(C) Acceleration of centre of mass of the disc (r) May be zero
(D) Vertical component of the acceleration of centre of mass (s) Must be zero
6. Match the following :
Column I Column II
(A) Work done by friction force may be (p) Dependent on the property of metal
(B) Work done by pseudo force may be (q) Positive
(C) Work done by torque may be (r) Negative
(D) Work function of a metal surface is (s) Zero
7. A motorcycle moves around a vertical circle with a constant speed under the influence of the force of gravity W , friction
between wheel and track f and normal reaction between wheel and track N .
Column I Column II
(A) Constant magnitude (p) N
(B) Directed towards centre when value is non-zero (q) N f
(C) Total reaction force by track (r) f W
(D) When motion is along vertical the value is zero (s) N W f
8. A uniform solid cylinder of mass m and radius R is placed on a rough horizontal surface where friction is sufficient to provide
pure rolling. A horizontal force of magnitude F is applied on cylinder at different positions with respect to its centre O in each
of four situations of column-I, due to which magnitude of acceleration of centre of mass of cylinder is ‘a’. Match the appropriate
results in column-II for conditions of column-I.
Column I Column II
F
R
(A) O (p) Friction force on cylinder will not be zero
//////////////////////////
F
R/2 F
(B)
O (q) a
m
//////////////////////////

F
O
F
(C) (r) a
////////////////////////// m

R/2
O
(D) F
(s) the direction of friction force acting on cylinder is
//////////////////////////
towards left

5. 6. 7. 8.

MARK YOUR
RESPONSE
CIRCULAR AND ROTATIONAL DYNAMICS 215

9. A small object of mass 0.5 kg is attached to an end of a massless 2 meter long inextensible string with the other end of the string
being fixed. Initially, the string is vertical and the object is at its lowest position having initial horizontal velocity of magnitude
u. The tension in string is T when the object is at its lowest position. The object subsequently moves in vertical plane. The
forces acting on object are tension exerted by string and gravitational pull by earth. Match the statements in column I with
corresponding results in column II (Take g = 10 m/s2).
Column I Column II
(A) u = 3.5 m/s (p) There will be some point on the trajectory of object
at which speed of the object is zero but tension in the
string is not zero.
(B) u = 9.5 m/s (q) There will be some point on the trajectory of object
for which tension in the string is zero but speed of the
object is not zero.
(C) T = 15N (r) There will be some point on the trajectory of object
for which tension in the string as well as speed of object
are both zero.
(D) T = 35N (s) The acceleration of the object will be in vertical direction
10. Match the columns
Column I Column II
(Object) (Moment of Inertia)
8MR 2
(A) Uniform rod (p)
11

30°
=R

MR 2
(B) Uniform semicircular ring. Axis is perpendicular (q)
12
to plane of ring

[ = 22/7]

13MR 2
(C) Uniform triangular plate of mass M (r)
8

R
60° 60°

R R
216 IIT-JEE P HYSICS Challenger

MR 2
(D) Uniform disk of initial mass M from which circular (s)
8
portion of radius R is then removed.
M.I. of remaining mass about axis which is perpendicular
to plane of plate.

R
2R

11. If net external force on a system of particles is zero, then match the following
Column I Column II
(A) Acceleration of centre of mass (p) must be constant
(B) Kinetic energy of the system (q) must be zero
(C) Velocity of centre of mass (r) may not be zero
(D) Velocity of an individual particle of the system (s) may not be constant
12. A block is placed on a horizontal table which can rotate about its axis. A block is placed at a certain distance from centre as
shown in figure. Table rotates such that particle does not slide. Select possible direction of net acceleration of block at the
instant shown in figure. Then match the column.

2
1

Column I Column II
(A) When rotation is clockwise with constant (p) 1
(B) When rotation is clockwise with decreasing (q) 2
(C) When rotation is clockwise with increasing (r) 3
(D) Just after clockwise rotation begins from rest (s) 4

9. 10. 11. 12.

MARK YOUR
RESPONSE
CIRCULAR AND ROTATIONAL DYNAMICS 217

13. A particle moves with position given by r 3tiˆ 4 ˆj . Where r is measured in meters and t ( > 0) in seconds
Column I Column II
(A) Rate of change of distance from origin (p) Increasing with time
(B) Magnitude of linear acceleration of particle (q) decreasing with time
(C) Magnitude of angular velocity of particle about origin (r) constant
(D) Magnitude of angular momentum of particle about origin (s) zero
14. In each situation of column I a mass distribution is given and information regarding x and y coordinates of centre of mass is
given in column II. Match the figures in column I with corresponding information of centre of mass in column II.
Column I Column II
(A) An equilateral wire frame is made using three thin uniform (p) xcm 0
rods of mass per unit lengths , 2 and 3 as shown.
y

x
2
(B) An square frame is made using four thin uniform rods of mass (q) ycm 0
per unit lengths , 2 , 3 and 4 as shown.

3 4

(C) A circular wire frame is made of two uniform semicircular wires (r) xcm < 0
of same radius and of mass per unit length and 2 as shown.

2
(D) A circular wire frame is made of four uniform quarter circular (s) ycm < 0
of same radius and of mass per unit length , 2 , 3 and
4 as shown.

3 4

13. 14.

MARK YOUR
RESPONSE
218 IIT-JEE P HYSICS Challenger
15. A solid sphere, hollow sphere, solid cylinder, hollow cylinder and ring each of mass M and radius R are simultaneously released
at rest from top of incline and roll (pure rolling) down the incline then match column I and column II.
Column I Column II
(A) Time taken to reach bottom is maximum for (p) Solid sphere
(B) Angular acceleration is maximum for (q) Hollow cylinder
(C) Kinetic energy at bottom is same for (r) Hollow sphere
(D) Rotational kinetic energy is maximum for (s) Ring
16. An object is allowed to roll down the incline starting from rest. All are uniform and have same mass and radius.
Column I Column II
(A) The object which has largest rotational inertia (p) Solid sphere
about its axes of symmetry
(B) The object which will experience the largest net (q) Spherical shell
torque
(C) The object which will have the largest speed at the (r) Solid disc
bottom of the incline
(D) The object which will reach the bottom of incline in (s) Thin hollow cylinder
the shortest time
17. A block of mass m is tied with an inextensible light string of length . One end of the string is fixed at point O. Block is released
(from rest) at A. Find acceleration of particle during its motion in vertical plane at positions specified in column I and match them
with column II. Given that A and O are at same horizontal level.

O
A

Column I Column II
(A) At highest point (p) Acceleration is horizontal
(B) At lowest point (q) Acceleration is vertically upwards
(C) At tan 1 ( 3) with vertical (r) Acceleration is vertically downwards
(s) Acceleration has both horizontal and vertical
components

15. 16. 17. p q r s


A p q r s
MARK YOUR B p q r s
RESPONSE C p q r s
CIRCULAR AND ROTATIONAL DYNAMICS 219

1. A rectangular plate of mass M and dimension (a × b) is held


in horizontal position by striking n small balls (each of mass
cue
m) per unit area per second by a velocity v. The balls are
striking in the shaded half region of the plate. The collision
of the balls with the plate is elastic. What is the value of
(0.1 v) in m/s ?
(Given n = 100, M = 3 kg, m = 0.01 kg; b = 2 m; a = 1 m;
g = 10 m/s2). 5. A small solid ball (mass = 0.1kg) rolls without slipping along
the track shown in Fig. The radius of the circular part of track
b is R. If the ball starts from rest at a height 8R above the
bottom, what is the horizontal forced (in N) acting on it at P?

v
2. A circular plate of uniform thickness has a diameter of
56 cm. A circular portion of diameter 42 cm is removed from 8R
one edge of the plate as shown. At what distance (in cm) to R
the left from the centre of the disc is the centre of mass of
O P
the remaining portion ?
R

6. A small sphere rolls down without slipping from the top of a


track in a vertical plane. The track has an elevated section
O and a horizontal part. The horizontal part is 1.0 metre above
42 cm the ground level and the top of the track is 2.4 metres above
cm

the ground. Find the distance (in m) on the ground with


56

respect to the point B (which is vertically below the end of


the track as shown in fig.) where the sphere lands.
3. Two blocks of masses m1 (=1kg) and m2 (= 2kg) connected
by a weightless spring of stiffness k (= 0.2 N/m)rest on a
smooth horizontal plane as shown in fig. Block 2 is shifted a
small distance x (= 0.1m) to the left and then released. Find
the velocity (in m/s) of centre of mass of the system after
block 1 breaks off the wall.
2.4 m

4. A sphere at rest on a horizontal rough surface is hit by a


cue. At what height (in cm) above the centre of the sphere
should it be hit so that it starts pure rolling, just after 1.0 m
hitting ? Assume the radius of the sphere to be 0.5 cm. B

1. 2. 3. 4. 5. 6.

MARK
YOUR
RESPONSE
220 IIT-JEE P HYSICS Challenger

1 (c) 12 (b) 23 (d) 34 (a) 45 (a) 56 (d) 67 (a) 78 (b) 89 (c) 100 (c)
2 (b) 13 (b) 24 (d) 35 (d) 46 (b) 57 (c) 68 (b) 79 (c) 90 (d) 101 (b)
3 (b) 14 (b) 25 (d) 36 (c) 47 (d) 58 (a) 69 (c) 80 (d) 91 (c) 102 (c)
4 (c) 15 (a) 26 (b) 37 (a) 48 (b) 59 (b) 70 (c) 81 (d) 92 (a) 103 (d)
5 (a) 16 (a) 27 (a) 38 (b) 49 (d) 60 (d) 71 (d) 82 (d) 93 (b) 104 (d)
6 (c) 17 (d) 28 (c) 39 (b) 50 (d) 61 (d) 72 (a) 83 (b) 94 (a) 105 (b)
7 (c) 18 (d) 29 (a) 40 (d) 51 (b) 62 (a) 73 (c) 84 (c) 95 (c) 106 (a)
8 (c) 19 (c) 30 (b) 41 (c) 52 (b) 63 (c) 74 (a) 85 (d) 96 (c) 107 (b)
9 (b) 20 (b) 31 (c) 42 (c) 53 (c) 64 (a) 75 (c) 86 (a) 97 (a) 108 (a)
10 (b) 21 (c) 32 (c) 43 (c) 54 (a) 65 (b) 76 (c) 87 (d) 98 (c) 109 (d)
11 (b) 22 (d) 33 (a) 44 (c) 55 (a) 66 (c) 77 (c). 88 (d) 99 (c)

1 (c) 5 (a) 9 (a) 13 (a) 17 (b) 21 (c) 25 (b) 29 (d) 33 (d) 37 (d)
2 (b) 6 (b) 10 (b) 14 (b) 18 (c) 22 (c) 26 (d) 30 (c) 34 (c) 38 (b)
3 (a) 7 (a) 11 (a) 15 (d) 19 (a) 23 (d) 27 (b) 31 (b) 35 (b) 39 (a)
4 (c) 8 (d) 12 (c) 16 (d) 20 (b) 24 (d) 28 (c). 32 (d) 36 (a) 40 (b)

1 (d) 3 (d) 5 (a) 7 (b) 9 (a) 11 (d) 13 (c) 15 (d) 17 (b) 19 (d)
2 (d) 4 (c) 6 (c) 8 (d) 10 (d) 12 (d) 14 (d) 16 (a) 18 (d) 20 (c)

1 (a, b, c, d) 5 (b, c) 9 (b, c) 13 (c, d) 17 (a, c, d) 21 (a, b) 25 (a, b) 29 (a, b, c)


2 (a, d) 6 (a, d) 10 (c, d) 14 (a, c) 18 (a, b, d) 22 (a, b, c) 26 (a) 30 (c, d)
3 (b) 7 (a, d) 11 (a, b) 15 (a, c, d) 19 (a, b, c, d) 23 (a, c, d) 27 (c) 31 (a,c)
4 (b) 8 (a, b, c, d) 12 (a, c, d) 16 (b, d) 20 (a, b, c, d) 24 (a, d) 28 (a) 32 (a)

1. A-s; B-r; C-p; D-q 2. A-s; B-r; C-q; D-p 3. A-r, s; B-p, q; C-s; D-p, q, r, s
4. A-q; B-p; C-s; D-r 5. A-p, q, r; B-p, q, r; C-p, q, r; D-s 6. A-q, r, s; B-q, r, s; C-q, r, s; D-p, q
7. A-s; B-p, s; C-q; D-r 8. A- p, r; B-q, r; C-p, r, s; D-p, r, s 9. A-p, s; B-q, s; C-r, s; D-s
10. A-q; B-p; C-s; D-r 11. A-p, q; B-r, s; C-p, r; D-r, s 12. A-r; B-s; C-q; D-p
13. A-p; B-r, s; C-q; D-r 14. A-q, r; B-p, s; C-p, s; D-p, s 15. A-q, s; B-p; C-p, q, r, s; D-q, s
16. A-s; B-s; C-p; D-p 17. A-r; B-q; C-s

1 1 2 9 3 0.02 4 0.2 5 10 6 2
CIRCULAR AND ROTATIONAL DYNAMICS 221

1. (c) At the highest point, 6. (c) The disc has two types of motion namely translational
weight = centrifugal force gives mg = mR 2
and rotational. Therefore there are two types of angular
1 g g momentum and the total angular momentum is the
g
= =2 n n= vector sum of these two.
R 2
R R
In this case both the angular momentum have the same
g 900g direction (perpendicular to the plane of paper and away
r.p.m = 60 n = 60 2
R 2
R from the reader).
2. (b) I = Imetal + Iwood
y
m1 R12 R12 R2 2
= 2 m2
2

2
0.2 0.22 0.32 R
v
=4 2
3
2
0.515
C
3. (b) Since v is changing (decreasing), L is not conserved in
magnitude. Since it is given that a particle is confined O x
to rotate in a circular path, it cannot have spiral path.
Since the particle has two accelerations ac and a t
therefore the net acceleration is not towards the centre. L LT LR
LT = angular momentum due to translational motion.
v
LR = angular momentum due to rotational motion
about C.M.
at L = MV × R + Icm
L ac Icm = M.I. about centre of mass C.

1
= M (R ) R + MR 2
2
(v = R in case of rolling motion and surface at rest)

3
= MR 2
The direction of L remains same even when the speed 2
decreases. 7. (c) FBD for sphere and block
m( 2b) 2m( b) m 0 2m(b) b
4. (c) x = and fr mg fr mg
m 2m m 2m 3 a1 , a2
m m m m
y = + b.
5. (a) Moment of inertia of the rod w.r.t. the axis through a1 giˆ , a2 giˆ
centre of the disc is (by parallel axis theorem)
fr
mL2
I mR 2 and K.E. of rod w.r.t. disc m m
12 a1
1 1 2 L2
= I
2
= 2m R2 fr a2
2 12
222 IIT-JEE P HYSICS Challenger

a As the linear momentum of any system = MVCM.


8. (c) Torque about A : mg I
2 2MV
The linear momentum of the train =
a/2
A× B 11. (b) f = 4 ma ........ (1)
a/2 3 (mg – f) r = (3mr² + mr²) a
mg – f = 4ma ........ (2)
a/ 3 a a
From (1) and (2)
mg 8ma = mg

C
mga
2I

a mga 2 mg
acceleration = f
3 2 3I
9 g
9. (b) Imagine the cylinder to be moving on a frictionless a
8 8r
surface. In both the cases the acceleration of the centre
of mass of the cylinder is g sin . This is also the 1 1 1m 2
12. (b) mvm2 vb
acceleration of the point of contact of the cylinder with 2 2 2 2
the inclined surface. Also no torque (about the centre
of cylinder) is acting on the cylinder since we assumed 1 1m 2
m ( vm 1) 2 vb
the surface to be frictionless and the forces acting on 2 2 2
the cylinder is mg and N which pass through the centre Divide both equation,
of cylinder. Therefore the net movement of the point of
contact in both the cases is in the downward direction vm 1
2
as shown. Therefore the frictional force will act in the vm
upward direction in both the cases.
1
vm 2 1 2.414 m/s
2 1
F
g sin Put in any above equation vb = 4.84 m/s
N Point of 13. (b) The angular momentum (L) is conserved, since is
contact ext
g sin zero.
Here, Ii = I0 If = 2I0
mg sin =
mg i 0 f= (say)

0
Then I0 0 = 2I0 =
In general we find the acceleration of the point of 2
contact due to translational and rotational motion and 1 2
then find the net acceleration of the point of contact. Ki = I0 0 K
2
The frictional force acts in the opposite direction to
that of net acceleration of point of contact. 2 2
1 0 1 0
10. (b) If we treat the train as a ring of mass M then its COM Kf = (2 I 0 ) 2 I0
2 2 2 4
2R
will be at a distance from the centre of the circle.
1 2 1 K
= I0 0
Velocity of centre of mass is : 2 2 2
VCM RCM . Alternatively,
2R 2R V V L2
= (K.E.)rotation = .
R R 2I
2V 2 MV Here , L = constant
VCM MVCM (K.E.)rotational × I = constant.
When I is doubled, K.E.rotational becomes half.
CIRCULAR AND ROTATIONAL DYNAMICS 223

14. (b) Moment of inertia 16. (a) When the thread is pulled, the bobbin rolls to the right.

R
a/2 v
v0
3 a/2
B

a/4

a/4 C

Resultant velocity of point B along the thread


is v v0 sin r , where v0 sin is the component
2
3a a2 of translational velocity along the thread and r linear
=m m1
4 3 velocity due to rotation. As the bobbin rolls without
For the centre of rod slipping, v0 R . Solving the obtained equations,

vR
m1a 2 m1a 2 m1a 2 we get v0
R sin r
12 4 3
17. (d) By the concept of energy conservation
m1 = 4m
1 2 1 2 3v 2
2
mv I mg
3a 4ma 2 9ma 2 4ma 2 2 2 4g
Total I = m
4 3 16 3 For rolling motion v = R

27 64 91 2 1 2 1 v2 3 2
= ma 2 ma mv I mv
48 48 2 2 R2 4
15. (a) If spool is not to translate
1 v2 3 2 1 2 1 2
F cos = f ....... (1) I mv mv mv
2 2 4 2 4
If spool is not to rotate R
Fr = f R ........ (2)
1 v2 1 2
From eq. (1) and (2) we get static friction I 2
mv
2 R 4
fR
cos f 1
r I= mR 2
2
This is the formula of the moment of inertia of the disc.
Fsin
18. (d) Moment of inertia of rod 2 about an axis passing
F
m 2
through O perpendicular to both roots, I2 =
3

r Fcos
1
f
static friction
2 x
r 1 r
or cos or cos O
R R
224 IIT-JEE P HYSICS Challenger
The distance from O to the parallel axis through the From (i) and (ii),
2 3
L F 1 I OP MR 2 Mr 2
1
2 2
centre of mass of rod 2 is M
MN GH 2 JK PQ
2
5 2
2
r= R
15
Moment of inertia of rod 1 about an axis passing
21. (c) Let a be the length of the string.
through O perpendicular to both rods,
Tension T at any point is given by
m 2 m 2 5m 2 4 2 M 2
I1 = + mx2 = = = m T [u 3ag cos 2ag ]
12 12 4 3 a ...(i)

2 2 2
where M is the mass of the particle and as such
4m m 5m
I = I1 + I2 = + = W
3 3 3 M .
g
19. (c) Let h be the height of water surface, finally
From (i), we get :

a/4 W 2
T [u 3ag cos 2ag ] ...(ii)
ag
At the highest point T = mW (given) and
W 2
From (ii), mW [u 3ag 2ag ]
h ag
a/8
mag u 2 5ag ...(iii)
At the lowest point T nW (given) and 0
2 a a a
a h a. . h W 2
2 2 4 From (ii), nW [u 3ag 2ag ]
ag
a a 3a 5a
C.M. gets lowered by a a nag u 2 ag ...(iv)
4 8 8 8
Eq. (iii) – Eq. (iv)
5a mag nag 5ag ag
Work done by gravity = mg
8 m 6 n n–m=6
20. (b)
5µg
22. (d) v0 – µgt R 0 .t
2R
A' Y
2(v0 – 0 R )
t
M B 7 µg

M 23. (d) The cubical block is in equilibrium.


r For translational equilibrium
R (a) Fx = 0 F=N
A (b) Fy = 0 f = mg

B' Y' y
f
2a x
For solid sphere
a
2
IAB = MR 2 I (given) ... (i)
5 C
b F
For solid disc N
1 3
IA'B' = IYY' + Mr2 = Mr 2 Mr 2 Mr 2
2 2 mg
IAB = IA'B' (given) ... (ii)
CIRCULAR AND ROTATIONAL DYNAMICS 225

For Rotational Equilibrium Alternatively


c=0
In pure rolling, the point of contact is the instantaneous
Where c = torque about c.m. centre of rotation of all the particles of the disc.
Torque created by frictional force (f) about C = f × a in On applying
clockwise direction. v=r
There should be another torque which should counter We find is same for all the particles then v r.
this torque. The normal reaction N on the block acts as Farther the particles from O, higher is its velocity.
shown. This will create a torque N × b in the 27. (a) Force due to pressure difference is
anticlockwise direction. N
Such that f × a = N × b
The normal force does not act through the centre of
the body always. The point of application of normal
force depends on all the forces acting on the body. F
M
24. (d) In the shown frame the particle appears to be at rest.
Net force on it must be zero. Therefore pseudo force
must be equal and opposite to the tension.
f
P Mg
m
1 7 R2
L F R 2 (v22 – v12 )
O 2 2
Now, F – f = M a
25. (d) Let us assume cylinder is not moving then
T + fs = mg sin 2 a 2
fR MR 2 f Ma
T. R – fs R = 0 3 R 5
2
fs
mg 3 f F R2 µMg R2
4 5
But (fs)max = µN = µmg cos R2 1
µ 0.25
1 mg Mg 4
= 0.4 × mg
2 5 28. (c) F = m a
( fs) < ( fs)max, our assumption is wrong. So, friction
2
existing must be kinetic v2
µ mg m at2
1 mg r
fk = µ m g cos 0.4 × mg
2 5
26. (b) Here vC = vc 2
2 v22
at µ g – 4 m/s 2
vP = vc2 2 2
r r

29. (a) Let be the mass per unit area.


vQ = vc2 2 2
r 2vc r cos

R/ 3
O'

2R /3
vc P vc O
C
r= v'
v'= r
Q vc
O
226 IIT-JEE P HYSICS Challenger
The total mass of the disc
2 xdx 2µ mgx
= × R2 = 9M Now dN = µmg. = dx
2
The mass of the circular disc cut R R2
2
R 2µmgx 2 dx
= d xdN
3 R2

R2 2µ mg R 2 2
= M d x dx µmgR
9 R2 0 3
Let us consider the above system as a complete disc of
mass 9M and a negative mass M super imposed on it. 1
I mR 2
Moment of inertia (I 1 ) of the complete disc = 2
1 4 µg
9MR 2 about an axis passing through O and , If t be the time for complete stop.
2 3 R
perpendicular to the plane of the disc.
M.I. of the cut out portion about an axis passing through 3 R
t t
O' and perpendicular to the plane of disc 4µ g
2 31. (c) Consider a situation when the bob A has fallen through
1 R
= M an angle .
2 3
Loss in PE = Gain in KE
M.I. (I2) of the cut out portion about an axis passing
through O and perpendicular to the plane of disc
2 2
1 R 2R
= M M
2 3 3

[Using perpendicular axis theorem]


The total M.I. of the system about an axis passing
through O and perpendicular to the plane of the disc is
I = I1 + I2
2 2
1 1 R 2R
= 9 MR 2 M M 1 2
2 2 3 3 I mg sin
2
1 1 4
= 9 MR 2 MR 2 2mg sin 2mg sin
2 18 9 A
I m 2
1 1 8
= 9 MR 2 MR 2
2 18 2g sin
A

9 MR 2 9MR 2 In the similar position


=
2 18
4g sin
(9 1) MR 2 B
= 4MR 2
2
30. (b) Consider a small element of disc of thickness dx at a A tB 1
radius x. B tA 2

tA
2
dx tB
x 32. (c) Consider a square lamina of mass (dm).
Moment of inertia from this about the shown diagonal
2
is (dm) a .
12
CIRCULAR AND ROTATIONAL DYNAMICS 227
a 35. (d) The angular momentum of the system about O = 0
= 0.

a 36. (c) vB vA vB
A

vB v
Now consider an element of the cube of thickness dx ' is for rod AB
and a distance x from the face diagonal. v
tan 30
R
2

2v 3 2 3v
R R

' vB
dx A
a x vB v
30°
a2 vA R /2
Now dI = (dm) + (dm)x2
12 37. (a) When we are giving an angular acceleration to the rod,
( Parallel axis theorem) the bead is also having an instantaneous acceleration
a = L . This will happen when a force is exerted on the
Mdx a 2
dI x2 bead by the rod. The bead has a tendency to move
a 12 away from the centre. But due to the friction between
the bead and the rod, this does not happen to the extent
a
M a a2 M a2 x x3 to which frictional force is capable of holding the bead.
As I 2
x dx The frictional force here provides the necessary
a 0 12 a 12 3
0 centripetal force. If instantaneous angular velocity is
then
1 1 5
Ma 2 Ma 2
12 3 12

33. (a) | L | mvr


L
L
mL 2 = µ(ma)

mL 2 = µ mL
r
O p=mv 2=µ
By applying
The direction of L (about the center) is perpendicular = 0 + t,
to the plane containing the circular path. Both We get = t
2t2 = µ
magnitude and direction of the angular momentum of
the particle moving in a circular path about its center O µ
is constant. t=
Alternatively, The net force acting on a particle
undergoing uniform circular motion is centripetal force 38. (b) 10 cos 30° – f = 2a ... (1)
which always passes through the centre of the circle.
The torque due to this force about the centre is zero, Fy=10 sin 30°

therefore, L is conserved about O.


Fx=10 cos 30°
M1d 2 a
34. (a) M 2v d
3
M1 3 f N
v=d
M2 1 20 N
228 IIT-JEE P HYSICS Challenger
I 43. (c) The applied force shifts the normal reaction to one
corner as shown. At this situation, the cubical block
2
fr= 2 r2 ... (2) starts topping about O. Taking torque about O
3
(where r is radius of sphere)
From (1) and (2), we get F L
N
f = 2 3 newton, N = 20 + 10 sin 30° = 25
L/2
f 2 3
f=µN 0.08 3
N 25
O
39. (b) Linitial = Lfinal mg
mv0r = I0 0
1 2 L mg
mr 0
mv0r = 2v0 = 0r F × L = mg × F=
2 2 2
40. (d) Moment of inertia about the diameter of the circular 44. (c) Torque about point A
1
loop (ring) = MR 2 a 3 a
2 F. mg.
2 2
Using parallel axis theorem
The moment of inertia of the loop about XX' axis is mg
F=
2 3
MR 3
IXX' = MR 2 MR 2
2 2
F
Where M = mass of the loop and R = radius of the loop
L
Here M = L and R = ;
2 f
2
A
3
3 L 3L
IXX' = ( L ) For equilibrium of block
2 2 8 2 F–f=0
1 2 F=f
41. (c) W = I
2 1
µ should be greater than .
Let x is the distance of CM from A 3
I = M.x2 + 4M ( x)2 45. (a) During collision with the rough surface, a frictional
and if I is minimum, W will be minimum torque will act in the clockwise direction.
dI 46. (b) v = Kr .t
2 Mx 4 M 2( x) ( 1)
dx dL
L = mvr = m Kr 3 t m Kr 3
= 2 Mx – 8M ( x) dt
dI 47. (d) Taking moments of all forces about A, the net torque
10 Mx 8M due to tension (T) in the string and the weight of the
dx
beam (W) is non-zero. Hence, the beam can never be in
10 Mx 8M 0 equilibrium.
4 48. (b) Average pseudo force
x=
5
change in momentum in non-inertial frame
42. (c) m2g – T = m2a2 ... (1) =
time elapsed
m2 R 2
TR = 2 ... (2) 3
2 = N
10
1R = a2 – 2R ... (3)
49. (d) From the F.B.D., it is clear that we get the equations,
m1 R 2 mg sin f ma
TR = 1 ... (4)
2 fr = I
2T 2 2(a2 g) a
2=
.( m2 a2 m2 g) = =I since
m2 R m2 R R r r
CIRCULAR AND ROTATIONAL DYNAMICS 229

Solving, 51. (b)


A
mg sin 2 /3
a= ; the time taken by the bodies to roll down
I
m 2 P
r
B /2
2 O D
a distance, is .
a

1 2 2 2
Using I = mr for the solid cylinder and I = mr
2 3
C
for the hollow spherical shell we get the required result. Vertical line from hinge A must pass through C.M. of
rod system.
N OP /2
tan
AP 2 /3
f a
mgsin 3 1 3
tan tan
4 4
mgcos
2 rad 3.14
52. (b) rad/sec 0.105 rad/sec
t 60sec 30
53. (c) Applying work-energy theorem :
50. (d) Drawing F.B.D. of rod PQ 1 1
M b gd M a gd (M b M a ) v2 I pulley 2
[0]
2 2
and v = R, v = 3.
N P µN 54. (a) =I
Q
N|| N
N
Mg /////////////////////////////
1m 1m
45°

Fx = 0
Fy = 0
µN = mg ... (1) m 2 3g
mg sin 45° ×
about centre of mass = 0 2 3 2 2
Tangential acceleration of centre of mass
N sin [2] µN cos 3g
2 2
(at) =
2 4 2
µ 2
tan m
2 Radial acceleration of centre of mass (ar) = 0
2
(initially is 0)
( µ2 4)
Hence sec mg
2 mg sin 45° – N = mat N
4 2

/2 µ2 4 mg
Thus, µ2 4 mg cos 45° – N|| = mar N||
1 2 2

17
17 Net hinge force N 2 N||2 mg 34 5.83 6
0.25 4 metres 4 2
2 (rounded to nearest integer)
230 IIT-JEE P HYSICS Challenger
55. (a) Divide the ring into infinitely small lengths of mass 60. (d) At a point such as B or D (the centripetal force
dm1. Even though mass distribution is non-uniform, necessary for circular motion) = (the normal reaction
each mass dm1 is at same distance R from origin. given by the road) – (the weight of the cyclist).
MI of ring about z-axis is Therefore, the normal reaction = (the weight of the
= dm1R2 + dm2 R2 + ..............+ dmnR2 = MR2
cyclist) + (the centripetal force). The radius of curvature
56. (d) Case 1 : When they rotate in same sense
is smaller at D than it is at point B, giving a larger
2m = 2 t
normal reaction at D than at B. This is why the cyclist
3 3 feels heaviest at D. Note that at points A or C, the
2n t; 2m 2 2n
2 2 necessary centripetal force is weight minus the normal
3 3 reaction.
2m = 3 + 4n ; m 2n m 2n 61. (d) Point O can be taken to be the instantaneous axis of
2 2
Not possible for m and n being integer. rotation. Distance of this point from any other point is
Case 2 : When they rotate in opposite sense between 0 & 2R.
2m = 2 t These linear speeds vary between 0 & 2R

2n t ; 2m 2 2n 2R
2 2
2m = + 4n ; 2m – 4n = 1
Not possible for m and n integer. r
57. (c) FR = I
where R is the radius of the disc. r
1
F MR O
2
1 d 1 62. (a) a T
MR MR(6) T
2 dt 2
1 N = Mg a
5 0.2 6 = 3.0 N
2
58. (a) The local radius of curvature of the curve is mg
[1 (dy / dx )2 ]3 / 2 [1 (9 / 4)ax]3 / 2
mg – T = ma ........ (i) and T = Ma ............ (ii)
2 2
d y / dx (3/ 4) a1/ 2 x 1/ 2 Solving (i) and (ii),
In this motion of the particle, the curve exerts a normal
or centripetal force, causing the particle momentarily mg Mmg
a T = Ma =
to move in an arc of a circle of radius . Thus, M m M m
3/ 2
mv 2 3 1/ 2 1/ 2 9 h h
F a x 1 ax mv 2 T Mg
Mmg h h
4 4 Mg
2 4 M m2 4

59. (b) From geometry BC = 52 32 = 4m M M m


2 1 1 or 1
m m M
and AC AB 2 BC 2 22 42 2 5m
63. (c)
C
2 R
5m v2 (2 R ) 2
R R 2
4R
a R
3m
A 2m B O D
64. (a) There are many approaches to this problem, but the
For both particles to reach C simultaneously, we have
one to take should yield the desired answer as quickly
t AC t BC
as possible. We are interested in only one unknown,
AC BC u AC 2 5 the tension in the tie rope. Considering one of the rafters
or 1 to be our system, we show all forces acting on it in
u1 u2 u2 BC 4
figure.
CIRCULAR AND ROTATIONAL DYNAMICS 231

F y
V

150N M
T 0.5m A
H

400N
The force F includes the effect of the 500 N weight as F2
B
well as the force due to the other rafter. If we take
moments about an axis through the top of the rafter, we
get
=0
To find the resultant force R on the hinge at A, we
(3.0 sin ) (1.5 150 sin )
have R = (400)2 (100)2 412 N .
– 2.5 × T × sin (90° – )
= 1.75 – (1.75/2) (150) – 2.5 cos T = 0. The tangent of the angle that R makes with the
Since sin = (1.75/3.0), = 35.7°, and cos = 0.812, negative x-direction is V/H, so the angle is tan–1 (4) =
therefore 2.03 T = 1.75 – 131. 76°
67. (a) If friction acts backwards its will increase & vcm will
To find we return to the complete system in figure decrease hence voiding pure rolling condition & if
and appeal to symmetry. The normal force due to the friction acts in forward direction the will decrease &
floor on each rafter is the same. Furthermore, vcm will increase, again voiding pure rolling condition
considering the entire system as one rigid body, hence no friction acts.
Fy = 0 yields 2 = (500 + 150 + 150) N, and 68. (b) Let b and are linear acceleration of centre of mass
and angular acceleration of the plane, just after BF is
= 400 N. cut.
Substituting into our moment equation and solving we
get T = 280 N.
65. (b) After 2 sec speed of boy will be
v = 2 × 2 = 4 m/s a
T
At this moment centripetal force on boy is 2

mv 2 30 16 A
Fr 80 N
R 6
C
Tangential force on boy is
Ft = ma = 30 × 2 = 60N
mg
Total friction acting on boy is
b
F Fr2 Ft2 100 N
mg – T = mb ......... (1)
At the time of slipping F = µmg
Taking torques about COM
or 100 = µ × 30 × 10 µ = 1/3.
66. (c) Only a horizontal force acts at hinge B, because hinge Ta ma 2
......... (2)
A is assumed to support the door’s weight. Let us take 2 6
torques about A as axis.
a a
= 0 becomes (F2) (h) – (400 N) (h/4) = 0 , g b and b
3 2
from which F2 = 100 N.
We also have 2b 5b 3g
g b b
Fx = 0 or F2 – H = 0 3 3 5
Fy = 0 or V – 400 N = 0 m.3g 2mg
We find from these that H = 100 N and V = 400 N T mg
5 5
232 IIT-JEE P HYSICS Challenger
69. (c) The only force block exerts on disc is parallel to axis of and the torque condition (about the contact point with
rotation of disc. This additional force does not cause the floor) is
any torque on disc. Hence angular momentum of disc – N1 (0.08) + 1 (0.03) – F (0.01) = 0 .......... (3)
remains same. Since there is no friction between block Elimination of F and N1 between these three equation
and disc, the block remains in its position. gives f2 = 0.5 – 0.125 N2.
But the largest possible value of f2 is µ2N2 = 0.35N2.
70. (c) y Hence, 0.5 – 0.125 N2 0.35 N2
0.5 20
or N2 N
0.475 19
and, from (1)
7 1
F N2 1 N
19
2 The minimum force is thus (1/19) N, corresponding to
x
O 2 which
7
7 20
N1 N f2 , N2 N
m 5.5 1 m 6.5 13 19 19
X cm cm. The force may be increased above this value, still
2m 2 4
keeping N3 = f3 = 0, up to F = 3N, at which point N1 and
m 3.5 1 m 4.5 9 f2 vanish. Thus there is a whole range of solutions
Ycm cm.
2m 2 4 such that the right wall might just as well not be there.

du 0.5 2 0.3 4
71. (d) at 4t 4m / s 2 73. (c) Speed of COM = 0.2 m / s
dt 6
74. (a)
v2 4 N
an 3 m / s2 a a 2t a c2 5
R 4/3
72. (a) The solution of this problem is greatly simplified by an
intuitive consideration of the situation when F is very
small. The 1N force then sets up a counter-clockwise s N cos N sin
torque that because of the low frictional resistance
offered by the left wall and the floor immediately causes
the bar to lose contact with the right wall. Therefore, if
a value of F can be found that puts the bar in equilibrium mg sN
with its end just touching the right wall (i.e. N3 = f3 = 0),
this value of F must represent the desired minimum. mv 2
s N cos N sin .............. (1)
R
µ1 = 0
N3 µ3 = 0.50 N cos s N sin mg .............. (2)
/////////////////////////////////////////////////////////////////////////////////
/////////////////////////////////////////////////////////////////////////////////

F 5cm v2 sin scos


N1 (1) and (2) give,
f3 Rg cos s sin
6 cm
5cm 5 cm
sin scos
v gR
cos s sin

8 cm
8 cm 75. (c) Since the objects are placed gently, therefore no
5cm
external torque is acting on the system. Hence angular
1N
4 cm momentum is constant.
N2
f2 µ2 = 0.35 1 2
///////////////////////////////////////////////////////////////////////////////
6 cm 5cm

r
With N3 = f3 = 0, the force conditions are m m
N2 – 1 – F = 0 .......... (1)
N1 – f2 = 0 .......... (2) 2 2 2
I = Mr I2=Mr +mr
CIRCULAR AND ROTATIONAL DYNAMICS 233

i.e., I1 1 = I2 2 2
fR MR2 (torque = I about COM)
Mr × 1 = (Mr2 + 2mr2)
2
2 1 5

M 2mg sin
= acm = R f =
2M 2m 7
76. (c) Its mechanical energy is conserved. 82. (d) Velocity of centre of mass is non zero.
It has a centripetal acceleration, downward. v vf vi
Its speed is minimum. 83. (b) a
t t
77. (c) S1 : If the object is large so that gravitational
acceleration is not same at every point, both will have r 5/
t 1
different locations. v 5
S2 : Internal forces cannot change momentum of any 10
kind of system. = 10 m/s2
a
1
S3 : If resultant force on a system of particles is non-
84. (c) In AOC, by sine rule,
zero, the centre of mass shall accelerate and in some
condition it may move along a circular path. Thus the AC AO
distance of centre of mass from centre of circle shall be sin120 sin 30
constant. Hence the statement is true.
AC = 3R
78. (b) v2 A v
v1
104 R
120°
O
R R
C B

2 2 Time taken in moving from A to C will be


1 rad / hr , 2 rad / hr
1 8 A to C distance 4 R
=
v 3v
2 3
T1 R1 R2
4 R2 4 10 4 km 3R 3 3v
T2 R2 R1 vav
4 R 4
2 R1 3v
v1 2 10 4 km/hr
1h x2 y2
85. (d) The quadrant is bounded by the ellipse 1
2 R2 a2 b2
v2 10 4 km/hr
8h
and the coordinate axes, its area is A = ab/4
At closest separation
vrel to line joining 10 4 km/hr Y x² y²
4
rad/hr. + =1
length of line joining 3 10 km 3
a² b²
79. (c) After collision velocity of COM of A becomes zero and dm = dA = y dx
that of B becomes equal to initial velocity of COM of A.
But angular velocity of A remains unchanged as the
two spheres are smooth. b
y
7
80. (d) I = Icm + MR2 = MR 2
5
81. (d) X
f dx
a

mg sin
mg cos a
xy dx a
x dm 0 1
mg sin – f = macm xcm xy dx
dm A A0
234 IIT-JEE P HYSICS Challenger
But, along the ellipse, 88. (d) The angular displacement of the particle in t = 1 sec is
2 v 3
2 x dx 2 y dy a
0 or x dx y dy t t
2 2 2 R 2
a b b
0
1 a2 v
and so xcm 2
y 2 dy v
A b b

1 a2 b3 4a
= 2
A b 3 3
By symmetry, ycm = 4b/3
86. (a) For whole system
f1 f2 3 (1)a ......... (1) The magnitude of impulse by centripetal force in t =
1 second is equal to change in momentum
= 2mv 3 2 Ns
m 89. (c) The position vector of the center of mass at the time t
is
rcm iˆ (cos 30 ) ˆj (sin 30 ) kˆ (0.10)
//////////////////////////////////////
f1 f2 = 0.866 iˆ 0.5 ˆj 0.10 kˆ
and the total momentum of the hoop is
For rear cylinder
100 – f1 = 0.5 (a) ......... (2) p mvcm (0.50) (0.50 ˆj ) 0.25 ˆj
For front cylinder
f2 = 0.5 (a) ......... (3) Thus, Lorb rcm p
From (1), (2) and (3)
100 = 4a = (0.866 iˆ 0.5 ˆj 0.10 kˆ) 0.25 ˆj
a = 20 m/s2
87. (d) From conservation of energy = 0.025 iˆ 0.216 kˆ kg m 2 /s
1 To find the spin angular momentum, note that every
mgh mv 2 element of mass of the hoop is at the same distance
2
from the centre of mass r' = 0.10m, and every element
////////// rotates about the center of mass with a velocity v (of
magnitude 0.50m/s) perpendicular to r . Thus,
h Lspin r v dm

= r v ( iˆ) dm

= mr v iˆ 0.025 iˆ kg m 2 /s
mg
90. (d) All forces on sphere pass through its centre except the
1 2 force of friction exerted by inclined plane. Since net
mg sin mv torque on sphere in equilibrium about its centre is zero,
2
the torque on sphere due to frictional force about its
v2 centre must be zero. Hence frictional force on sphere is
2 g sin ac zero.
91. (c) The moment of inertia of the system about axis of
g cos at
rotation O is
Total acceleration, a ac2 at2 I = I1 + I2
= 0.3x2 + 0.7 (1.4 – x)2
g cos 2 (2sin )2 g 3sin 2 1 = 0.3x2 + 0.7 (1.96 + x2 – 2.8x)
= x2 + 1.372 – 1.96x
CIRCULAR AND ROTATIONAL DYNAMICS 235

94. (a) FBD of rod will be as shown below


0.3 kg
TB
x
TB
O a RAH
B A
C W
1.4 – x L

RAV
0.7 kg
Summation of moments about A should be zero
MA = 0 = – TB × L – (TB – W) × a = 0
The work done in rotating the rod is converted into its
rotational kinetic energy. Wa
TB
( L a)
1 2 1 2 2
W= I [ x 1.372 1.96 x] Summation of moments about B should be zero
2 2 MB = 0 = (TB – W) × (L – a) + RAV × L = 0
For work done to be minimum
Wa
dW W × (L – a) + RAV × L = 0
0 ( L a)
dx
2x – 1.96 = 0 Wa WL Wa
RAV × L = – (L a)
( L a)
1.96
x= 0.98 m
2 WL ( L a ) ( L a)
R AV W
92. (a) For rolling, vA = 2m/s ( L a) L (L a)
or 4 – 1. = 2
axis Fx
95. (c)
I axis 2
Mr
4m/s Mx 2
4
2m/s For maximum ,

or = 2 rad/sec (clockwise) d
0
dx
93. (b)
r2 r
x2 2 x2 0 x
4 2
N
96. (c) p 0
At equilibrium F = 0, P N2
N1 = mg
L
mg L
mg cos f1L sin N1 L cos
2
N1
1 2 1 2 mg cos
mgR mv I L mgL sin mgL cos
2 2 2 f1=µN1
mg
v 1 2 1 2 2 v2 1
; mgR mv . mr tan
r 2 2 5 r2 2
97. (a) A= B ;
7 10 gR
gR mv 2 v A B
10 7 IA IB

mv 2 17mg A B
N mg N A B
R 7 mR 2 mR 2
236 IIT-JEE P HYSICS Challenger
98. (c)
5cm.
Y Y

^ ^ ^ ^
v =4i –3j v = 4i–3j
X X 12cm.

6cm.
^ ^
v = 4i–3j

99. (c) Consider FBD of rod


/////
////

12
/////

W Position of CG, y1 6cm.


FA
////

2
////

A
////

L
/////

1 2
////

L FB For right circular cone : Volume = r1 h1


////

////

B 3
/////

////

NA
////
////

2
1 3 9
////
/////

NB or V2 6 cm3
////

3 2 2
///

////

60° 45° Position of CG,


Given = 15° µ = tan = 0.268 1 1 3
y2 h1 6 cm.
Applying equilibrium equations, we get 4 4 2
X = FA cos 60 + NA sin 60 + FB cos 45 – NB sin 45 = 0 Position of CG of given body above its base
Also we know that 9 3
FA = 0.268 NA and FB = 0.268 NB V1 y1 V2 y 2 75 6
y = 2 2
Solving above equations we get V1 V2 9
75
NA = 0.518 NB 2
450 6.75 443.25
Y = NA cos 60 – FA sin 60 + NB cos 45 + FB sin 45 = W = 6.3cm.
75 4.5 70.5
Solving these equations we get
102. (c)
NB = 0.966 W and F = 0.259 W m C2
Taking moment about A and equating it to zero, we get C1
MA = (W × L cos ) – (NB cos 45° × 2L cos ) + /////////////////////////////////////
/////////////

[(L/2)–x]
(NB sin 45° × 2L cos ) – (FB sin 45° × 2L cos ) S
– (FB cos 45° × 2L × sin ) = 0
By putting the values of known quantities in above L
equation we get = 36.2° mx m x
2
100. (c) Let speed of belt be v
L
Angular speeds of wheels x ;
4
v v A RB L 3L L
2
2 RB , 2 RA ;
B A S
B RA 2 44
101. (b) The given body = Solid cylinder – Right circular cone 103. (d) mu = Mvcm + mv1
mu – mv1 = Mvcm
As the body is symmetrical about y-axis, the center of
mux = I + mv1x
gravity will lie on the y-axis i.e. x 0
1
For solid cylinder : Volume = r2h mux = ML2 mv1 x ;
12
2
5 Vcm L2
or V1 12 75 cm 3
2 12 x
CIRCULAR AND ROTATIONAL DYNAMICS 237

104. (d) Free body diagram for pulley and for the beam is shown or 2 cos2 – 0.75 cos – 1 = 0
in figure (a) and (b).
500 0.75 (0.75)2 4 2
Solving it, cos
4
3
0.75
8.5625
4 = = 0.92 (using the +sign)
4
VB = cos–1 (0.92) = 23°
106. (a) The moment of force F (6kN) about point A is equal to
HB the sum of the moment of its components about the
same point A.

y
50N 500 6kN
6 sin 30°
VB

///////////////////////////////////
C 30°
HB x
RCH 6 cos 30°

0.5m 0.3 0.3


2.5m
RCV
RAV 200N

Pulley : FV = 0 : VB = 500 + 50 – (0.6 × 500) = 250 N A 0.5m B


FH = 0 : HB = 500 × 0.8 = 400 N
Beam : FH = 0 Moment of x-component of force is
RCH = 400 Mx = 6 cos 30° × 0.3
RCV × 5 = (250 × 3) + (200 × 2.5)
3
i.e. RCV = 250 N =6× 0.3 = 1.56 kN-m – clockwise
RAV = 250 + 200 – 250 = 200 N 2
RC = (4002 + 2502)1/2 = 471.69 N moment of y-component of force is
105. (b) The forces acting on the rod are : My = 6 sin 30° × 0.5
(i) Weight W of the rod acting vertically downwards from 1
centre of gravity. =6 0.5 1.50 kN-m +
2
(ii) Reaction R at A acting normally at A i.e. along AO.
(iii) Reaction R' at C acting at right angles to rod. Net moment about A = My – Mx = (1.50 – 1.56) kNm
For equilibrium the three forces will be concurrent. = – 0.06 kNm
By geometry, OCA = OAC = GDA = or 0.06 kNm clockwise.
AC = AD cos = 2r cos , DC = 2r sin 107. (b) b
O
and AG = 1.5r
h
In triangle GDC,
GC AC AG sin 2r cos 1.5r
tan or
DC DC cos 2r sin Velocity at B = 2gh

D Angular momentum = m × 2gh × b


108. (a) The given body can be considered as a right circular
R' cone ABC from which a cone ADE and a cylindrical
B PQRS have been cut out as shown.
O Let the x-axis be along the base and y-axis as the axis of
C
G symmetry. For the cone ABC of height h is :
R
h2 h
A W r1 r

2sin2 = 2 cos2 – 1.5 cos h 20 h


or
or 1 – cos2 = cos2 – 0.75 cos 5 10
238 IIT-JEE P HYSICS Challenger
For cylindrical hole
y 2
A 5
Volume, V3 r32 h1 = 20 125 cm3
2
h2
R h1 20
D S r1 E
Position of CG on y-axis, y3 10 cm.
2 2
h The given body has a volume = V1 – V2 – V3
r3 h1 Position of CG of given body on y-axis
Vy V1 y1 V2 y2 V3 y3
Q y
B P r C V V1 V2 V3

4000 500
2h 40 h h 40cm. 10 25 125 10
3 3
= 4000 500
1 2 1 4000
Volume V1 r h (10)2 40 cm3 3 3
125
3 3 3
Position of CG on y-axis, 40000 12500 3750 23750
y = 7.6cm.
1 4000 500 375 3125
y1 40 10 cm. 109. (d) S1 : For a rotating body, if is constant then
4
For cone ADE, Volume =0 = 0.
S2 : A particle moving along x-axis with some velocity,
1 2 1 500
V2 = r1 h2 = (5) 2 20 cm3 at some distance from origin then its angular momentum
3 3 3 not equal to zero.
Position of CG on y-axis, S3 : Direction continuously changes so it is not
1 constant.
y2 = 20 + × 20 = 25cm.
4

1. (c) As shown in the figure that all of the angular velocities 3. (a) The initial kinetic energy is
are in the same direction, so we can regard A, B, and 1 1
2 2
as the components of angular velocity along the K1 = IA A IB B
rotation axis. Conservation of angular momentum then 2 2
gives 1
IA A + IB B = (IA + IB) = (0.040 kg.m2) (50 rad/s)2 +
2
IA A IB B 1
IA IB (0.020 kg.m2) (200 rad/s)2
2
2. (b) The moment of inertia of the two disks are The final kinetic energy is
1 1 1 1
IA = m r 2 = (2.0 kg) (0.20 m)2 K2 = (I A IB ) 2= (0.040 kg.m2
2 AA 2 2 2
= 0.040 kg.m2 + 0.020 kg.m2) (100 rad/s)2 = 300 J
1 1 F
IB = m r 2 = (4.0 kg) (0.10 m)2 4. (c) The acceleration of the centre of mass is aC .M
2 BB 2 2m
= 0.020 kg.m2 The displacement of the centre of mass at time t will be
IA A IB B 1 Ft 2
=
IA IB x aC.M t 2
2 4m
5. (a), 6. (b)
(0.040 kg.m 2 ) (50 rad / s) Suppose the displacement of the first block is x1 and
= (0.020 kg.m 2 ) (200 rad / s) that of the second is x2. Then,
0.040 kg.m 2 0.020 kg.m 2
mx1 mx2 Ft 2 x1 x2
= 100 rad/s x or
2m 4m 2
CIRCULAR AND ROTATIONAL DYNAMICS 239

f1R – f2R = I .....(4)


Ft 2 A=R .....(5)
or x1 x2 ....... (1)
2m
1
Further, the extension of the spring is x1 x2 . 4 55
4 F cos 2
a 10 m/s 2
Therefore, x1 x2 x0 ........ (2) 3M 8m 3 1 8 1
From eq. (1) and eq. (2), 1
3 1 55
1 Ft 2 3MF cos 2
x1 x0 f1 7.5 N
2 2m 3M 8 m 3 1 8 1
1
1 55
1 Ft 2 and f MF cos 2 2.5 N
and x2 x0 2
2 2m 3M 8m 3 1 8 1

7. (a); 8. (d); 9. (a) 13. (a) Rcos /n


Taking origin at O
Coordinate of A = ( cos , – sin ) Rcos /n
Coordinate of C = (– – sin – cos )
/n
O

2
A m 2 R sin
n
I= m.R 2 cos 2 MR 2
12 n

B
C D sin 2
I = n.m.R2 n cos 2 MR 2 .
3 n
10. (b); 11. (a); 12. (c)
Drawing the F.B. D of the plank and the cylinder.

F sin sin 2
N1 14. (b) I = nmR2 n cos 2 MR 2 (M mn) R 2
3 n

F cos sin 2
f1 = nmR2 n cos 2 1 2 MR 2 .
3 n

mg
sin 2
15. (d) Where I = nmR2 n cos 2 MR 2
f1 3 n
N1

Mg (M + nm) gsin f = (M + nm) a ... (i)


f R = I a/R ... (ii)
N2 From (i) and (ii)
f2 (M nm)g sin I
f=
Equations of motion are [I ( M nm) R 2 ]
F cos – f1 = ma ....(1)
F sin + N1 = mg ....(2) (M nm)g sin I
µ (M + nm) g sin
f1+ f2 = MA .....(3) [I ( M nm) R 2 ]
240 IIT-JEE P HYSICS Challenger

I tan 0
µ= = 0 – 2
I (M nm) R 2 2

0 0
3 = or, =
2 6

R 0
Now from (ii), t2 =
6µg
2 Maximum displacement of the disc in forward direction,
16. (d) I = 4 MR2 M (R 2)2
5 R R 0
2
0 1 R 0
S= µg
2 2 4 4µg 2 4µg
= 4MR 2
5
R 2 02 1 1 R 2 02
4 MR 12 48MR 2 2 = =
= . µg 16 32 32µg
5 5 The displacement of the disc when it starts pure rolling
17. (b) Let a be the acceleration of centre of mass
2
Mg – T = 0 ... (i) 1 R 2 02 1 R 0 5 ( R 0 )2
F.x = T.2x ... (ii) = µg =
32 µg 2 6µg 32 9 µg

0R 5 ( 0 R)2
F t3
x 6 32 9 µg

M 5 0R
or, t3 =
48 µg
18. (c) remain the same
19. (a) Let linear velocity of the disc will become zero after a 25 0 R
time t1. Then it starts moving in backward direction Total time = t1 + t2 + t3 = .
48 µg
and at time t2 it comes in pure rolling. When disc starts
pure rolling its linear and angular velocities will become 20. (b) Time after which disc starts pure rolling.
constant and friction will be zero. R 0 5R 0 R 0
t = t1 + t2 = =
6µg 12µg 4µg
R 0 21. (c) Angular momentum of disc after it starts pure rolling,
v0= L = MvR + I
4
MR 0 R MR 2 0
=
At time t1,
6 2 6

R 0
0= µgt1 2 1 1 MR 2 0
4 = MR 0 =
6 12 4
R 0
t1 = ... (i) 22. (c) The power delivered is constant; the kinetic energy of
4µg
the wheel is proportional to t and so the velocity is
At time t2,
v = µg t2 proportional to t .
µg t2 = R dx
23. (d) Since v = t
R dt
t2 = ... (ii) We get, x t3/2;
µg
dv 1/ 2
2µg and acceleration = a = t
= 0– (t1 t2 ) dt
R
friction, f = ma t–1/2
2µg R 0 R or, f 3 x = constant.
= 0 –
R 4µg µg 24. (d) The required time is independent of .
CIRCULAR AND ROTATIONAL DYNAMICS 241

25. (b) The force of impact at A is vertically upward.


26. (d) v( ˆj ) vjˆ
vf vi2vjˆ
32. (d) Average acceleration =
t t t
v' 33. (d) Let P be the point on sphere in contact with incline.
Then P is also instantaneous centre of rotation
A
A 7
v0 mg sin × R = (Icm + mR2) = mR 2
5
vA = v' + cos
collision is elastic
P
v0 = v ' cos
27. (b) Angular momentum L r mv
| L | mv0 cos . sin
mg
28. (c) The disc will stop (translation) at time
v0 5 3g
t= acm = R g sin
=
µg 7 7
Since, = I 34. (c) Speed of point P is always zero.

MR 2 h 5h
µMg.R = . 35. (b) Distance covered = S =
2 sin 37 3

2µg 3g
= acm
R 7
and = 0 – t
2s 70h
time t =
acm 9g
2v0 2µg v0
0= .t t= 36. (a), 37. (d), 38. (b).
R R µg
Also at the same time, linear speed also ceases.
R
v = v0 – µgt 0 = v0 – µgt L+
R 3
v0 60° 30°
t=
µg O x
60°
Thus it will not regain .
Since linear speed and angular speed becomes zero at mg
the same time.
29. (d) At time t = 4v0/5µg the angular speed will be zero.
At this instant v = v0/5. Since u 2 gL , the highest point to which the block
Now when pure rolling starts, it will take a time
u2
shall reach is L distance above its initial position.
2v0 4v0 2g
from t = 0
35µg 5µg Hence at highest point, the thread has rotated by
v0
Thus finally . Hence first will decrease from 60 and the block is at same horizontal level as
7R 3
centre of cylinder as shown.
2v0 v
to zero and then increase from zero to 0 .
R 7R R
x 2R
30. (c) COM : about point of contact sin 30

2 2 v0 7 v mg
Mv0 R MR 2 MvR ; v = 0 T = mg cos 60° =
5 R 5 7 2

Total displacement 3g
31. (b) Average velocity = Tangential acceleration, at = g sin 60° =
Total time 2
242 IIT-JEE P HYSICS Challenger
39. (a), 40. (b) OD= rim
Some deceleration required: FOD .ROD 120 0.350
v2 – v02 = 2as Fp r 129 N
Rrim 0.325
v0 = 18 km / hour = 5 m/s
Fp-r = µ × Fperpendicular
v2 v02 0 25 Fperpendicular = Fp-r / µ = 129 / 0.5 = 258 N
a 2.5 m/s 2 Fperpendicular per pad = 129 N
2s 2 5
The combination of handlebar lever and caliper
F = m a = – 96 × 2.5 = – 240 N
assembly has a mechanical advantage of 10.
i.e., 120 N per wheel opposite to the direction of rotation
(Think of a lever with 20 units on one side of the fulcrum
The brake pads apply a torque to the rim, but at a and 2 units, 1 per pad, on the other.)
shorter distance from the axle than the outside diameter Flever per pad = Fperpendicular / 10 = 12.9 N
of the wheel (contact point with the road). However, as the lever acts on two pads, the total force
which must be applied to the lever is 25.8 N.

1. (d) For a disc rolling without slipping on a horizontal rough 9. (a) In non-uniform circular motion acceleration vector
surface with uniform angular velocity, the acceleration makes some angle with radius hence it is not
of lowest point of disc is directed vertically upwards perpendicular to velocity vector.
and is not zero (Due to translation part of rolling, 10. (d) Statement – 1 is False, Statement – 2 is True.
acceleration of lowest point is zero. Due to rotational Friction force is not always zero and a point object
part of rolling, the tangential acceleration of lowest
cannot roll as a point object will translate.
point is zero and centripetal acceleration is non-zero
11. (d) In non-uniform circular motion, particle’s kinetic energy
and upwards). Hence statement 1 is false.
2. (d) As x increases, the required component of reaction changes with time. By work-energy theorem, net work
decreases to zero and then increases (with direction done on the particle is non-zero. In uniform circular
reversed). Hence statement-1 is false. motion, total force on the particle is centripetal in nature.
3. (d) The moment of inertia about both given axis shall be 12. (d) The moment of inertia about both given axis shall be
same if they are parallel. Hence statement-1 is false. same if they are parallel. Hence statement-1 is false.
4. (c) The applied horizontal force F has tendency to rotate
1 2
the cube in anticlockwise sense about centre of cube. 13. (c) KE KECM MVCM
2
Hence statement-2 is false.
5. (a) Radius of gyration of body is not a constant quantity. KE in CM frame is least as VCM = 0 in that frame.
Its value changes with the change in location of the 14. (d) If speed is increasing there is a tangential acceleration.
axis of rotation. Radius of gyration of a body about a Net acceleration is not pointing towards centre.
given axis is given as 15. (d) S-1 will be true only if both the axis in comparison are
parallel to each other.
r12 r22 ............. rn2
k 16. (a) While seen from CM rigid body appears in pure rotation
n
6. (c) The two bodies will move towards their common centre 1 2
around CM. So KE = I cm .
of mass. But the location of the centre of the mass will 2
remain unchanged. Therefore, the centre of mass will 17. (b) Both statements are true but statement-2 is not correct
remain at rest with respect to A as well as B.
explanation for statement-1.
7. (b) Rolling occurs only on account of friction which is
18. (d) Acceleration of lowest point on the disc is 2R
tangential force capable of providing torque. When
the inclined plane is perfectly smooth, it will simply
slip under the effect of its own weight. Once the perfect
rolling begins, force of friction becomes zero. Hence
2
work done against friction is zero. R a
8. (d) Statement – 1 is false because, it is applicable only in
rotational frame.
CIRCULAR AND ROTATIONAL DYNAMICS 243

19. (d) Component of reaction by hinge on the rod


Fx Ny perpendicular to rod

Fx Nx 3Fx 3x
F Ny M Ny F 1
m 2
/3 2M 2
x as x increases, Ny decreases increases.
3Fx 20. (c) Torque about all points should be zero.
acm
2 2m

2 2
m( sin )2 cos
5. (b, c)
dL
C L
1. (a, b, c, d) I z m cos m
12 2 12 dt

2 dL
m 2 m 2 is to C and L both. Hence B is correct.
m sin dt
2 12 12
Further, L L = L2
m 2 m 2 m 2 m 2 Differentiating w.r.t. time
12 4 12 12
dL dL dL
L. 2L
m 2 dt dt dt
(constant independent of )
2
dL dL
[I z will be maximum for any value of or, 2 L. 2L
(Obviously)] dt dt

v2 dL
2. (a, d) an 2
R But L
R dt

M dL dL
3. (b) dm (2 r )drL 2 rdrL L. 0 0
( R22 – R12 ) dt dt
Magnitude of L does not change with time.
2M 6. (a, d) Acceleration of the bead down the wire is g cos
r dr
( R22 – R12 ) and the length of wire is 2R cos

R2 v 2as 2( g cos )(2 R cos )


2M 3 1
I r dr M ( R12 R22 ) v cos
( R22 – R12 ) R1
2
v 2 gR cos R
4. (b) The external torque about O is contributed only Further, t 2
a g cos g
by mg. The equation of motion is
– mg (d sin ) = I0 i.e., t is independent of .
7. (a, d) As the block does not move, the ball moves along
mL2 mL2 L2 a circular path of radius . The centre of mass of
I0 md 2 m( R 2 – )
12 12 4 the system always lies somewhere on the string.
where is small, sin Let v = spseed of the ball when the string makes
an angle with the horizontal.
1/ 2
2 L2 v
R – v'
4
g
L2
R2 –
6
u
244 IIT-JEE P HYSICS Challenger

1 2 1
mv mu 2 – mg sin Then d k sin d
2 2
0 0
Horizontal component of v = v' = v sin
2
= sin u 2 – 2 g sin or 2k (1 cos ) and
For v to be maximum, dv/d = 0, which gives 2
an 2k (1 cos )
sin = u2/3g 12. (a, c, d) As no external torque is present, using angular
8. (a, b, c, d) At A : momentum conservation about any point we can
say that f = i.
v R
13. (c, d) From the geometry, x
R sin
a
R A
for a rolling wheel, a = R
(a) is correct. R R v
At B :
x
v B a d
R Also, . Therefore,
dt
R
dx d R
v 2 v
If a then aB may be vertically downwards dt dt sin
R
(b) is correct. R (d / dt ) cos R cos
2
At C : sin sin 2
C R
a v sin 2 Rv
v R sin x x2 R2
R
d d Rv
(c) is correct. dt dt x x 2 R 2
Consider this
v2 Rv 2 (2 x 2 R2 )
R =
x2 ( x2 R 2 )3 / 2
a
R 14. (a, c) y
(d) is correct.
9. (b, c) The initial velocity of c.m. is upward. The B
acute
acceleration of the c.m. is 'g' downward.
10. (c, d) The velocity of the disc when rolling begins can a
be obtained using the conservation of angular
momentum principle about the point through x
C S O A
which the friction force acts. So, the coefficient of
obtuse
friction has no bearing on final velocity. The work a
done by the force of friction will simply be change
in kinetic energy. D
dv d
11. (a, b) (a) as ( ) k sin As the object moves from A to C via B the angle
dt dt
between acceleration vector and velocity vector
d
(b) From , decreases from 90° and then increases back to
dt 90°. Since the angle between velocity and
dt k acceleration is acute, the object speeds up.
d k sin dt k sin d sin d As the object moves from C to A via D the angle
d
between acceleration vector and velocity vector
CIRCULAR AND ROTATIONAL DYNAMICS 245

increases from 90° and then decreases back to Taking moments about the left edge and resolving
90°. Since the angle between velocity and T1 into x and y components,
acceleration is obtuse, the object slows down. = 0 yields LT1 cos 30° – (0.25 L) (400)
15. (a, c, d) Internal force does not change the motion of centre – (0.5 L) (120) = 0
of mass. Dividing throughout by L and solving, we get
If “g” varies, COG doesn’t coincide with centre of T1 = 185 N.
mass. Substituting into our earlier equations, we get
16. (b, d) In the shown diagram at position A and C radial T2 sin = 92.5 N andT2 cos = 360 N
acceleration has its maximum values and there is Dividing the equations yields tan = 0.257,
no component of acceleration in tangential or = 14.4°.
direction.
Then 0.249 T2 = 92.5, and T2 = 371 N.
C
One can always check moment problem results
by taking moments about another point, such as
the right end of the bar for this problem.
D B 19. (a, b, c, d) Refer to figure.

20°
T
A
At position B and D tangential acceleration is L
0.8
maximum (g) and radial acceleration in some where W
between maximum and minimum. H 40°
17. (a, c, d)
V

Taking torques about the hinge,


– W (0.8 L cos 40°) + (T cos 20°) (L sin 40°) –
(T sin 20°) (L cos 40°) = 0.
This yields T = 1.80 W.
From Fx = 0, H – T cos 20° = 0,
Giving H = 1.69 W.
Because the wall is frictionless, so there will be no
From Fy = 0,
torque during collision. Hence will not change
instantaneously. But velocity will be same in V – W – T sin 20° = 0,
magnitude and in opposite direction as collision and so V = 1.62 W.
is elastic with a fixed wall. If the beam has weight, then an additional W/2
Hence, now the slipping will start and the friction must be added acting vertically downward through
present on the ground will reduce . After some the center of the beam. The torque about the hinge
time, pure rolling starts. will be – (W/2) (0.5 L cos 40°) due to the weight.
18. (a, b, d) Remembering that the weight acts at the center of With these additions to the equations above, we
gravity, obtain T = 2.35 W, from which H = 2.21 W and V =
2.30 W.
20. (a, b, c, d) If larger mass comes down with acceleration a,
T cos T1 cos 30°
aM am M m
T1 acm = a
30° M m M m
T2
//////////////////////
T2 sin T1 sin 30°

400 N 120 N (=W)


M m
0.25 L 0.75 L
Fx = 0 yields T1 sin 30° – T2 sin = 0 COM is not at rest and net force on system
or T2 sin = 0.50 T1.
M m
Fy = 0 yields T1 cos 30° + T2 cos = (M + m) a = (M – m) a
– 120 – 400 = 0 M m
or T2 cos + 0.866 T1 = 520 N. Velocity of COM changes continuously.
246 IIT-JEE P HYSICS Challenger
21. (a, b) The most convenient point to take the sum of
1885 0.0605
torques is at the hinge. The torque equation is t = 456.2 s
+ 60L – (2L/3) (Tm sin 12°) + 250 (L/2) = 0, 0.25
leading to Tm = 1335 N. Force on the sample:
Also T cos 12° – H = 0, so H = 1305 N Fcentral = m 2 r = 0.012 × (1885)2 × 0.1
22. (a, b, c) The possible forces are shown in figure. = 4264 N.
25. (a, b) The system of coplanar-unlike forces are shown
above in figure. Sign convention used :
y
+
for forces ; +M for moments
x
f
mg N1 Let R be the resultant of given system of forces
and Rx and Ry be the horizontal and vertical
components of resultant force.
30°
R R x2 R y2

N2 Also, Rx Fx and Ry Fy
If we take moments about an axis through the Resolving all force in xx' direction with proper
center of the sphere, only f can have a torque and sign convention
= 0 implies f = 0.
Fx = (–) 1 cos 90° – 1.5 cos 60° – 1 cos 45° – 0.5
Then Fy = 0 yields
cos 30° = –1.890 kN
N2 cos 30° = mg = (10 kg) (9.8 m/s2)
Resolving all force in yy' direction with shown
Fx = 0 yields N2 sin 30° – N1 = 0,
sign convention
or N1 = 56.5 N, N2 = 113 N
Fy = (–) 1 sin 90° – 1.5 sin 60° – 1 sin 45° – 0.5 sin
23. (a, c, d) 30°
m = –3.265 kN
v
Putting value in eq. (1) we get
mg r R ( Fx ) 2 ( Fy ) 2 ( 1.89) 2 ( 3.265)2
R = ± 3.77 kN
///////////////////////////////////////// As the forces in the system are directed
O downwards (–ve direction) so the resultant R
d
will also be negative i.e.
The magnitude of angular momentum of particle The resultant force is given by : R = – 3.77 kN.
about O = mvd
Since speed v of particle increases, its angular Fy 3.265
tan or 59.93 60
momentum about O increases. Fx 1.890
Magnitude of torque of gravitational force about To find position of resultant force on beam use
O = mgd constant
R.d = MP
Moment of inertia of particle about O = mr2
where R = resultant force = (–) 3.77 kN
Hence MI of particle about O decreases.
d = distance between point of application of
v sin resultant force about P.
Angular velocity of particle about O
r MP = sum of the moments of all forces about P.
v and sin increases and r decreases MP = (1 × sin 90° × 0) + (1.5 × sin 60° × 4) +
angular velocity of particle about O increases. (1.0 × sin 45° × 8) + (0.5 × sin 30° × 12) = 13.852 kN-
24. (a, d) From – 0 = t and = I m
as Ry = R sin
( 0 ) I total
t Ryd = R sin d
MP = R sin d
where Itotal = Irotor + Isamples = Irotor + 4msampler2 So, 13.852 = 3.77 sin 60°.d
= 0.06 + 4 × 0.012 × (0.1)2 = 0.0605 kg m2
and = 18,000 (rpm) / 60 (seconds/minute) 13.852
d= 3.67m from P
× 2 (radians/revolution) = 1885 s–1 3.265
CIRCULAR AND ROTATIONAL DYNAMICS 247

26. (a) The force acting on the mass of liquid dm of length dx 28. (a) A'B' AB and C' D' CD
at a distance x from the axis of rotation O. From symmetry IAB = IA'B' and ICD = IC'D'
From theorem of perpendicular axes,
A'
F C' D
x
dx
dF

O A
B
dF = (dm) x 2

M 2
dF = dx x
L
M C D'
where is mass of liquid in unit length. B'
L Izz = IAB + IA'B' = ICD + IC'D' 2IAB = 2ICD
The force acting at the other end is for the whole IAB = ICD
liquid in tube.
dL
LM M L 29. (a, b, c)
F= 2
x dx 2
x dx dt
0 L L 0 Given that
L dL
M 2 x2 M 2 L2 ML 2
A L A L
= 0 dt
L 2 L 2 2
0
dL
27. (c) When the car is moving in a circular horizontal track of From cross-product rule, is always perpendicular
dt
radius 10 m with a constant speed, then the bob is also
undergoing a circular motion. The bob is under the to the plane containing A and L.
influence of two forces. By the dot product definition
(i) T (tension in the rod) L . L L2
(ii) mg (weight of the bob) Differentiating with respect to time
dL dL dL
L L 2L
v dt dt dt
dL dL
2L 2L
Tcos dt dt
T
Tsin dL
mg Since, i.e. is perpendicular to L
dt
dL
L 0
dt

Resolving tension, we get dL


0
T cos = mg ... (i) dt
L = constant
mv 2 Thus, the magnitude of L always remains constant.
And T sin = ... (ii)
r As A is a constant vector and it is always
(Here T sin is producing the necessary centripetal perpendicular to ,
force for circular motion) Also, L is perpendicular to A
Dividing (ii) by (i), we get
L A
2
v 10 10
tan = 1 L. A 0
rg 10 10
Thus, it can be concluded that component of L along
= 45°
A is zero i.e., always constant.
248 IIT-JEE P HYSICS Challenger
30. (c, d) As shown in the figure, the component of weight
g sin g sin 2
mg sin tends to slide the point of contact (of the But ac = 2
g sin ... (ii)
Ic mR / 2 3
cylinder with inclined plane) along its direction. The 1 1
mR 2 mR 2
sliding friction acts in the opposite direction to oppose
this relative motion. Because of frictional force the mg sin
From (i) and (ii), f =
cylinder rolls. 3
If is reduced, frictional force is reduced.
N 31. (a, c) is perpendicular to the plane of circular motion and
f v lies in this plane.
is perpendicular to the plane of circular motion and
ac also lies in this plane
s in mg cos v is tangential and ac is radial hence perpendicular..
mg mg
d psystem
32. (a) Fext
Thus frictional force adds rotation but hinders dt
translational motion. Given Fext 0 psystem = Constant
Applying Fnet = ma along the direction of inclined
Due to internal forces acting in the system, the kinetic
plane, and potential energy may change with time.
we get mg sin – f = mac, Also zero external force may create a torque if the line
where ac = acceleration of centre of mass of the cylinder of action of forces are along different direction. Thus
f = mg sin – mac … (i) the torque will change the angular momentum of the
system.

1. A-s; B-r; C-p; D-q


2 2
2 For hollow sphere I = mr
v 3
an = 9t 2 v = 12 t
16 mg sin mg sin
f=
dv 5/ 2 2.5
12
dt mg sin
For solid cylinder, f = ;
dv 3 3
Tangential force m. 16 6N
dt 2 mg sin
For solid sphere, f =
2 3.5
2
2 mv 2 9 3. A-r, s; B-p, q; C-s; D-p, q, r, s
Total force = 6 62 7.5 N.
R 2 4. A-q; B-p; C-s; D-r
5. A-p, q, r; B-p, q, r; C-p, q, r; D-s
Power = FT .v = 6 × 3 16 72 watt. 6. A-q, r, s; B-q, r, s; C-q, r, s; D-p, q
7. A-s; B-p, s; C-q; D-r
72
Average power = 36 watt. mv 2
2 (A) Constant magnitude = net force
2. A-s; B-r; C-q; D-p R

mg sin (B) N W f 0, N 0
f= 2
mr (C) Total reaction force = N f (D)
1
I When motion is along vertical

For ring I = mr2 mv2


N , f W 0
mg sin R
f=
2
CIRCULAR AND ROTATIONAL DYNAMICS 249

8. A- p, r; B-q, r; C-p, r, s; D-p, r, s


Assume friction to be absent and horizontal force F is
applied at a distance x above centre 2R/
CM
F
a .......... (1)
m
R – 2R/
2
mR
and Fx
2 A
2Fx
or R .......... (2) 2
mR 2R
IA Icm M R
R
If a = R then from eq. (1) and (2) x
2 2 2
2 2
F IA MR 2 MR 2 1
The friction force will be zero and a
m
R 4
If a > R or x , friction force is towards left and = MR 2 1
2
F 4 8 MR 2
a or I A MR 2 2
m 22 / 7 11
R
If a < R or x , friction force is towards right and
2 R 3
x
2 y
F (C)
a R 3 R
m
9. A-p, s; B-q, s; C-r, s; D-s 2
(A) u 2 g , Hence string will never get horizontal R
(p) and (s) are true 60° 60°
x
(B) 2g u 5 g , At one point tension will be zero. R 3
(q) and (s) are true R R 2
(C) Tmax = 3mg. Hence string will just become horizontal y
(r) and (s) are true
(D) Tmax > 6mg
The object will undergo complete vertical circular 2x
motion. I (dm) x 2 ( dx) yx 2 R x 2 dx
10. A-q; B-p; C-s; D-r 3
2 R 3
x
(A) I dm M x 3
2 x 4 2 MR 2
2 = R
3R 2 3 3 4 0
8
3 2 4
dm 2 m 2 m x m
=I x x dx
4 4 4 3 12
0
1 1M 2 M 2
(D) M (2 R)2 R R
2 2 4 4

x sin 30° 3 13
2 MR 2 MR 2 MR 2
8 8
30°
11. A-p, q; B-r, s; C-p, r; D-r, s
x (A) Fext = Maccln
2
Fext = 0 acm = 0
2 2R (C) Vcm must be constant and may not be zero.
(B) MR I cm M
(D) Nothing can be said for sure
250 IIT-JEE PHYSICS Challenger
12. A-r; B-s; C-q; D-p 16. A-s; B-s; C-p; D-p
mg sin
(B) = fR =
mR 2
Case - A : r + 1÷
ac= ²r =constant I

mR 2
If is minimum t will be maximum.
I
a aT= r
I is mR2 is maximum for ring.
Case B :
ac= ²r (C) v =
g sin I
2 s ; v is maximum when 2
I m R
1+ ÷
mR 2
ac= ²r minimum. v is minimum for solid sphere
Case C :
a 2s I I
r= aT (D) t µ 1+ 2÷
; For tmin , ÷ should
g sin mR mr 2
be min.
17. A-r; B-q; C-s
0=0

At ‘A ’ : v = 0, no centripetal acceleration. So
Case D :
acceleration is downward (Due to mg)
aT = r At B : T and mg both are vertical so acceleration
is vertically upward (centripetal acceleration)
13. A-p; B-r, s; C-q; D-r
mv 2
At C : T – mg cos = ...... (1)
v = 3iˆ , at = 0, L = mvr = (mr )
2
;

12
= O
A
9t 2 + 16

distance = | r | = (3t ) + (4) ;


2 2 T
C
T
d 3t
(distance) = ÷ v
(3t ) 2 + (4) 2 ÷
dt mg
B
this is increasing function with time (t). mg
14. A-q, r; B-p, s; C-p, s; D-p, s
Centre of mass lies where concentration of mass is more. 1
mg cos = mv 2 ......... (2)
(A) y > 0, x < 0 (B) y < 0, x = 0 2
(C) x = 0, y < 0 (D) x = 0, y < 0 From (1) and (2),
15. A-q, s; B-p; C-p, q, r, s; D-q, s T – mg cos = 2mg cos
T = 3mg cos
g sin k2 If T cos = mg then vertical component of acceleration
a= ; a T v k2/r2
k 2 r2 will become zero.
1+ (3mg cos ) cos = mg
r2
1
Solid sphere 2/5 cos = tan = 2
Hollow cylinder 1 3
Hollow sphere 2/3 So, at = tan 1 ( 2) acceleration has only horizontal
Ring 1
component.
CIRCULAR AND ROTATIONAL DYNAMICS 251

1. 1 2. 9
Since the plate is held horizontal therefore net torque acting
on the plate is zero.
m
F
3b/4
C2 O C1

m2 m m1
a

b/2
m' Area of whole plate = (56/2)2 = 784 sq. cm.
b 3b Area of cutout portion = (42/2)2 = 441 sq. cm. ;
Mg × F ... (i)
2 4 Area of remaining portion = 784 – 441 = 343 cm2;
dp b As mass µ area.
F = n (Area) = n × (2mv) × a × ... (ii)
dt 2 mass of cutout portion m 441 9
From (i) and (ii) = 1 = =
mass of remaining portion m2 343 7
b b 3b
Mg × = n × (2mv) × a × × Let C2 be centre of mass of remaining portion and C1 be
2 2 4
centre of mass of cutout portion.
3 2 O is centre of mass of the whole disc.;
3 × 10 = 100 × 2 × 0.01 × v × 1 ×
4 OC1 = r1 = 28 – 21 = 7 cm.
v = 10 m/s 0.1v = 1 m/s OC2 = r2 = ?;
Alternatively : Equating moments of masses about O,
b m1 9
Torque due to weight of plate 1 = Mg × we get m2 × r2 = m1 × r1 r2 r1 7=9
2 m2 7
torque due to small element shown dotted in the figure
Centre of mass of remaining portion is at 9 cm to the left
dp of centre of disc.
= x × dF = x n a dx
dt 3. 0.02
dx When block m1 breaks off the wall, length of spring is
x
unstretched. Therefore, KE of block m2 = P.E. of compression
1 1 k
m 2 v22 = k x 2 ; or v2 = x .
2 2 m2

Total torque
Wall
b a 2 b2 m1 k m2
2=
n(2mv ) ax dx 2mnv b
b/2 2 4
1 2
2
a 3b
= 2nmv Co-ordinates of centre of mass are given by
2 4
m x + m 2 x2
As = x= 1 1
1 2
m1 + m 2
b a 3b 2 a 3b
Mg = 2nmv Mg = 2nm v dx m1 dx1 m2 dx2
2 2 4 2 2 = +
dt m1 + m 2 dt m1 + m 2 dt ÷
4Mg 4 3 10
v= = = 10 m/s dx1
2nma 3b 2 100 0.01 1 3 2 To start with, x1 = 0, =0
0.1v = 1 m/s dt
252 IIT-JEE PHYSICS Challenger
This appears as total KE of ball at P.
dx m2 dx2 m 2 v2 m2 k
= ÷ = = x Thus mg × 7R = KE of translation + KE of rotation
dt m1 + m2 dt m1 + m2 m1 + m2 m2
1 2 1 1 1 2 2 7
(0.1 0.2 2) = mv + I 2 = mv2 + mr ÷ 2
= mv 2
= x km2 /( m1 + m 2 ) = = 0.02 m/s. 2 2 2 2 5 10
(1 + 2)
v2 = 10 g R (Where v = r & r is radius of solid ball)
4. 0.2 The horizontal force acting on the ball,
Cue exerts a force on sphere, so there will be change in
Fh = centripetal force towards O
momentum and change in angular momentum. Let the time
of collision be t and force exerted by cue during the collision mv 2 m (10g R )
= = = 10 m g = (10 × 0.1 × 10) N = 10 N
is F. R R
F t = p or F t = MVcm ....(1) 6. 2
[here impulse by frictional force is supposed to be zero Applying law of conservation of energy at point D and
since f < < F] point A.
Now take linear momentum about horizontal surface so that P.E. at D = P.E. at A + (K.E.)T + (K.E.)R
change in linear momentum by frictional force becomes zero. where (K.E.)T = Translational K.E.
Li = 0, Lf = I + MVcm R (K.E.)R = Rotational K.E.
Vcm
Since pure rolling is taking place, = ....(2) 1 2 1 2
R mg (2.4) = mg (1) + mv I
2 2
2 Since the case is of rolling without slipping.
Lf = 2/5MR2 + MVcmR = MVcm R + MVcm R
5
D
7
L f = MVcm R 2.4m
5 A
(F t) × (h ) = Lf – Li (F t)h = 7/5 MVcm R ...(3) 1m
h = vertical height from horizontal surface B C
7 MVcm R 7 2 v=r
From (1) & (3), h ' = = R = R+ R
5 MVcm 5 5 v
2 = where r is the radius of the sphere
Hence h = 2/5 R = 0.5 = 0.2 cm above centre of sphere. r
5
5. 10 2 2
Also, I= mr
5
v = 4.43 m/s
After point A, the body takes a parabolic path.
The vertical motion parameters of parabolic motion will be
uy = 0 sy = 1m
ay = 9.8 m/s2 ty = ?
8R
1 2
R s = ut + at
2
O P
R 1 = 4.9 t y2

1
Suppose m is the mass of the ball of radius r. ty = = 0.45sec
4.9
On reaching P, the net height through which the ball
descends is 8R – R = 7R, (from the fig. shown) Applying this time in horizontal motion of parabolic path,
Decrease in P.E. of ball = mg × 7R ; BC = 4.43 × 0.45 = 2 m
1. A certain pendulum clock keeps good time on the earth.If 5. A straight rod of length L extends from x = a to x = L +a.Find
the same clock were placed on the moon, where objects the gravitational force it, exerts on a point mass m at x = 0 if
weigh only one sixth as much as on the earth,how many the linear density of rod µ =A + Bx2
seconds will the clock tick out in an actual time of 1 minute?
A 1 1
(a) 24 sec (b) 24.5 sec (a) Gm + BL (b) Gm A ÷ + BL
a a a+L
(c) 25 sec (d) 25.5 sec
2. A satellite of mass ms revolving in a circular orbit of radius rs A
A
round the earth of mass M has a total energy E. Then its (c) Gm BL + (d) G m BL
a+L a
angular momentum will be
6. A geo-stationary satellite orbits around the earth in a circular
(a) (2 Ems rs 2 )1/ 2 (b) (2 Ems rs 2 ) orbit of radius 36,000km. Then, the time period of a spy
satellite orbiting a few hundred km above the earth's surface
(c) (2 Ems rs )1/ 2 (d) (2 Ems rs ) . (Rearth = 6,400km) will approximately be
Vesc (a) 1/2 hr (b) 1 hr (c) 2 hr (d) 4 hr
3. A shell is fired vertically from the earth with speed , 7. In a certain region of space, gravitational field is given
N
where N is some number greater than one and Vesc is escape by I= –(K/r). Taking the reference point to be at r = r0
speed for the earth. Neglecting the rotation of the earth and with V=V0, find the potential.
air resistance, the maximum altitude attained by the shell r r
will be (RE is radius of the earth) (a) K log + V0 (b) K log 0 + V0
r0 r
N 2 RE NRE r r0
(a) (b) (c) K log V0 (d) log V0 r
2
N2 1 r0 r
N 1
8. A projectile leaves the earth’s surface with a speed equal to
RE RE
(c) (d) 2 gRe , where Re is the radius of earth. Its speed far away
N2 1 N2
4. The percentage change in the acceleration of the earth from the earth would be
towards the sun from a total eclipse of the sun to the point (a) 2 gRe (b) g Re
where the moon is on a side of earth directly opposite to the
sun is (c) 2gRe (d) g Re
2
9. An artificial satellite is moving in a circular orbit around the
M s r2 M s r2 earth with a speed equal to half the magnitude of the escape
(a) (b) 100
M m r1 ÷
100 velocity from the earth. Find the height (h) of the satellite
M m r1
above the earth’s surface : (Take radius of earth as Re)
2 2
r Mm r1 Ms (a) h Re2 (b) h = Re
(c) 2 1÷ 100 (d) 100
r2 Ms r2 ÷ M m (c) h = 2Re (d) h = 4Re

MARK YOUR 1. 2. 3. 4. 5.
RESPONSE 6. 7. 8. 9.
254 IIT-JEE PHYSICS Challenger
10. A planet is revolving around a star in an elliptic orbit. The
ratio of the farthest distance to the closest distance of the R R
(a) (b)
planet from the star is 4. The ratio of kinetic energies of the 6 3
planet at the farthest to the closest position is
2R
(a) 1 : 16 (b) 16 : 1 (c) 1 : 4 (d) 4 : 1 (c) (d) R
11. A spherical uniform planet is rotating about its axis. The 3
velocity of a point on its equator is v. Due to the rotation of 18. A small satellite of mass m is revolving around earth in a
planet about its axis the acceleration due to gravity g at circular orbit of radius r0 with speed v0. At certain point of
equator is 1/2 of g at poles. The escape velocity of a particle its orbit, the direction of motion of satellite is suddenly
on the pole of planet in terms of v is changed by angle = cos–1 (3/5) by turning its velocity
(a) ve = 2v (b) ve = v vector, such that speed remains constant. The satellite
(c) ve = v/2 (d) ve = 3v consequently goes to elliptical orbit around earth. The ratio
12. A point P lies on the axis of a fixed ring of mass M and radius of speed at perigee to speed at apogee is
R, at a distance 2R from its centre O. A small particle starts (a) 3 (b) 9
from P and reaches O under gravitational attraction only. Its
speed at O will be (c) 1/3 (d) 1/9
19. With what minimum speed should m be projected from point
2GM
(a) zero (b) C in presence of two fixed masses M each at A and B as
R
shown in the figure such that mass m should escape the
2GM 2GM 1 gravitational attraction of A and B
(c) ( 5 1) (d) (1 )
R R 5 vmin
13. A planet of mass m is in an elliptical orbit about the sun
(m << Msun) with an orbit period T. If A be the area of orbit, m 30°
then its angular momentum would be C
(a) 2mA/T (b) mAT (c) mA/2T (d) 2mAT
14. If the radius of earth shrinks by one percent and its mass
remains the same, then acceleration due to gravity on the
earth surface will R
(a) increase by 1% (b) decrease by 1%
M M
(c) decrease by 2% (d) increase by 2%
15. The work done required to increase the separation distance A R R B
from x1 to x1+ d between two masses m1 and m2 is
G m1 m 2 x1 2GM 2 2GM
G m1m2 d (a) (b)
(a) (b) R R
x1 ( x1 + d ) ( x1 + d ) d

G m1m 2 x1 GM GM
(c) (d) none 2 2 2
( x1 + d ) (c)
R
(d)
R
16. Two satellites of same mass are orbiting round the earth at
20. The radius of a planet is n times the radius of earth (R). A
the heights of R and 4R above the earth's surface; R being
the radius of the earth. The kinetic energies are in the ratio satellite revolves around it in a circle of radius 4nR with
(a) 4 : 1 (b) 3 : 2 (c) 4 : 3 (d) 5 : 2 angular velocity . The acceleration due to gravity on
17. A body is projected vertically upward from the surface of planet’s surface is
the earth with a velocity equal to half the escape velocity. If (a) R 2 (b) 16 R 2
R is the radius of the earth, the maximum height attained by 2
(c) 32 nR (d) 64 nR 2.
the body is

10. 11. 12. 13. 14.


MARK YOUR 15. 16. 17. 18. 19.
RESPONSE
20.
GRAVITATION 255

21. Two concentric uniform shells of mass M1 and M2 are as where x is horizontal, y is vertically upwards; both being
shown in the figure. A particle of mass m is located just measured in meter. The x–component of the velocity of the
within the shell M2 on its inner surface. Gravitational force plane is constant throughout, and has the value of 360 km/h.
on ‘m’ due to M1 and M2 will be The effective g (“g – force”) experienced by an astronaut
on the plane equals
M2
(a) 4g (b) 3g
a M1
g
(c) (d) 5g
b 5
m
25. A planet of radius R has an acceleration due to gravity of gs
G M 1m on its surface. A deep smooth tunnel is dug on this planet,
(a) zero (b) radially inward, to reach a point P located at a distance of
b2
R
G ( M1 + M 2 ) m from the centre of the planet. Assume that the planet
(c) (d) None 2
b2
has uniform density. The kinetic energy required to be given
22. The minimum and maximum distances of a satellite from
to a small body of mass m, projected radially outward from
centre of earth are 2R and 4R respectively, where R is the
P, so that it gains a maximum altitude equal to the thrice the
radius of earth. The minimum and maximum speeds of the
radius of the planet from its surface, is equal to
satellite will be

GM 2GM GM 2GM
(a) , (b) , A
R R 6R 3R
OP
2GM 4GM
(c) , (d) None
3R 3R
Note that M represents the mass of the earth.
63 3
23. An artificial satellite is first taken to a height equal to half (a) mg s R (b) mg s R
the radius of earth. Assume that it is at rest on the earth’s 16 8
surface initially and that it is at rest at this height. Let E1 be
9 21
the energy required. It is then given the appropriate orbital (c) mg s R (d) mg s R
speed such that it goes in a circular orbit at that height. Let 8 16

E1 26. Six stars of equal mass are moving about the centre of mass
E2 be the energy required. The ratio is of the system such that they are always on the vertices of a
E2
regular hexagon of side length a. Their common time period
(a) 4 : 1 (b) 3 : 1 will be
(c) 1 : 1 (d) 1 : 2
24. In order to simulate different values of g, aspiring astronauts
a3 4 3a3
are put on a plane which dives in a parabola given by the (a) (b) 2
( )
4
equation : Gm Gm 5 3+4
x2 = 500 y
y
(c) 3a3 (d) None of these
4
Gm

MARK YOUR 21. 22. 23. 24. 25.


RESPONSE 26.
256 IIT-JEE PHYSICS Challenger
27. An earth satellite of mass m orbits along a circular orbit C1 31. A spaceship is sent to investigate a planet of mass M and
at a height 2R from earth’s surface. It is to be transferred to radius R. While hanging motionless in space at a distance
a circular orbit C2, of bigger radius, at a height 5R from 5R from the centre of the planet, the spaceship fires an
earth’s surface. The transfer is affected by following an instrument package with speed v0 as shown in the figure.
elliptical path as shown in figure. Calculate the change in The package has mass m, which is much smaller than the
the energies required at the transfer points A and B. mass of the spaceship. For what angle will the package
[R = radius of earth] just graze the surface of the planet ?
v0
m R
2R M
v'1 5R
v'2
A B
1 8GM 1 8GM
(a) sin 1
1 + 2 ÷ (b) sin 1
1+ 2 ÷
2 5v0 R 5 5v0 R
5R C1
1 2GM 1 8GM
(c) sin 1
1+ 2 ÷ (d) sin 1
1+ 2 ÷
C2 5 5v0 R 5 3v0 R
mgR mgR 32. In older times, people used to think that the earth was flat.
(a) (b)
36 18 Imagine that the earth is indeed not a sphere of radius R, but
mgR mgR an infinite plate of thickness H. What value of H is needed
(c) (d) to allow the same gravitational acceleration to be experienced
2 9
as on the surface of the actual earth ? (Assume that the
28. If the radius of the earth were to shrink by one percent, its
mass remaining the same, the acceleration due to gravity on earth’s density is uniform and equal in the two models
the earth’s surface would (a) 2R/3 (b) 4R/3
(a) decrease (b) remain unchanged (c) 8R/3 (d) R/3
(c) increase (d) be zero 33 A planet revolves about the sun is elliptical orbit. The areal
29. The acceleration due to gravity on the surface of the moon dA
velocity 16 2
is 1/6 that on the surface of earth and the diameter of the ÷ of the planet is 4.0 × 10 m /s. The least
dt
moon is one-fourth that of earth. The ratio of escape
velocities on earth and moon will be distance between planet and the sun is 2 × 1012 m. Then the
maximum speed of the planet in km/s is
6 3 (a) 10 (b) 20
(a) (b) 24 (c) 3 (d)
2 2 (c) 30 (d) 40
30. An artificial satellite of mass m of a planet of mass M, 34. The mass M of a planet-earth is uniformly distributed over a
revolves in a circular orbit whose radius is n times the radius spherical volume of radius R. Calculate the energy needed
R of the planet. In the process of motion, the satellite to disassemble the planet against the gravitational pull
experiences a slight resistance due to cosmic dust. Assuming amongst its constituent particles.
resistance force on satellite depends on velocity as F = av2 Given : MR = 2.5 × 1031 kg-m and g = 10 m/s2
where ‘a’ is constant. Calculate how long the satellite will (a) 3.0 × 1032 J (b) 2.5 × 1032 J
stay in orbit before it falls onto the planet’s surface. 28
(c) 0.5 × 10 J (d) 1.5 × 1032 J
m R ( n 1) m R ( n + 1) 35. If gravitational forces alone prevent a spherical, rotating
(a) (b)
a GM a GM neutron star from disintegrating , the minimum mean density
of a star that has a rotation period of one second will be
2m R ( n 1) m R ( n 1) (a) 2.4 × 1010 kg/m3 (b) 1.4 × 1010 kg/m3
(c) (d) 11 3
a GM a 2GM (c) 1.4 × 10 kg/m (d) 3.4 × 1011 kg/m3

MARK YOUR 27. 28. 29. 30. 31.


RESPONSE 32. 33. 34. 35.
GRAVITATION 257

36. Inside a uniform sphere of density there is a spherical 40. The orbital speed of Jupiter is
cavity whose centre is at a distance from the centre of the (a) greater than the orbital speed of earth
sphere. Find the strength F of the gravitational field inside (b) less than the orbital speed of earth
the cavity at the point P. (c) equal to the orbital speed of earth
(d) zero
41. The eccentricity of the earth’s orbit is 0.0167. The ratio of its
maximum speed in its orbit to its minimum speed is
C P (a) 1.67 (b) 1.034
(c) 1 (d) 0.167
42. Two bodies of masses M1 and M2 are placed at a distance d
apart. What is the potential at the position where the
gravitational field due to them is zero ?

(a)
4
3
G (b)
1
3
G (a)
G
d
(
M1 + M 2 + 2 M1 M 2 )
(c)
2
G (d)
1
G (b)
G
d
(
M1 M 2 2 M1 M 2 )
3 2
37. If g is the acceleration due to gravity on the earth’s surface,
the change in the potential energy of an object of mass m
(c)
G
d
(
2M1 + M 2 + 2 M1 M 2 )
raised from the surface of the earth to a height equal to the
radius R of the earth is
(d)
G
2d
(
M1 + M 2 + 2 M1 M 2 )
(a) mgR/2 (b) 2mgR 43. The figure shows a planet in elliptical orbit around the sun
(c) mgR (d) – mgR S. Where is the kinetic energy of the planet maximum ?
38. If the distance between the earth and the sun were half its
present value, the number of days in a year would have P3
been
(a) 64.5 (b) 129 (c) 182.5 (d) 730 P4 S P2
39. A spherical cavity is made in a lead sphere of radius R such
that its surface touches the outside surface of the lead sphere
P1
and passes through its centre . The mass of the lead sphere
before hollowing was M. What is the force of attraction that (a) P 1 (b) P 2
this sphere would exert on a particle of mass m, which lies at
(c) P 3 (d) P 4
a distance d from the centre of the lead sphere on the straight
44. A missile, which missed its target when into orbit around
line joining the centres of the sphere and the centre of cavity
the earth at a mean radius 3 times as great as the parking
as shown in the figure.
orbit of the satellite. The period of the missile is
(a) 2 day (b) 3 day
(c) 3 day (d) 3 3 day
m
45. A satellite of mass m is orbiting the earth in a circular orbit of
d radius R. It starts losing energy due to small air resistance at
the rate of C J/s. Find the time taken for the satellite to reach
the earth.

GMm 1 1 GMm 1 1
3GMm 2GMm (a) (b) +
(a) (b) C R r 2C R r
2R2 R2
GMm 1 1 2GMm 1 1
GMm GMm (c) (d) +
(c) (d) 2C R r C R r
R2 2R 2

MARK YOUR 36. 37. 38. 39. 40.


RESPONSE 41. 42. 43. 44. 45.
258 IIT-JEE PHYSICS Challenger
46. The time taken by the earth to travel over half its orbit, 50. Two particles of masses m1 and m2 moving in coplanar
remote from the sun, separated by the minor axis is about 2 parabolas round the sun, collide at right angles and coalesce
days more than half the year, then the eccentricity of the when their common distance from sun is R. The subsequent
orbit is path of the combined particles is an ellipse of major axis
(a) 1/30 (b) 1/60
( m1 + m2 ) 2 ( m1 m2 ) 2
(c) 1/15 (d) 1/70 (a) R (b) R
2 m1m2 2 m1m2
47. If a spherically symmetric star of radius R collapsed under
its own weight, neglecting any forces other than
gravitational ones, what is the time required for collapse ? ( m1 + m2 ) 2 ( m1 + m2 ) 2
(c) R (d) R
m1m2 3m1m2
2 3
8 2 R3 2 R
(a) (b) 51. Inside a large isolated shell of mass M having inner and
(GM ) 0.5 (3GM )0.5 outer radius r and R respectively, a particle is projected with
a velocity v tangentially along the inner surface of the shell.
2 3
R Find the time taken for the particle to complete one
(c) (d) None of these
(8GM )0.5 revolution. The friction is absent.

48. A planet of mass m is in an elliptical orbit about the sun (m


R
<< Msun), with an orbital period T. If A be the area of orbit
then its angular momentum would be r

2mA v
(a) (b) mAT
T
2 r
mA (a)
(c) (d) 2mAT GM / R
2T
49. A binary star system consists of two stars A and B which 2 r3 / 2
have time period TA and TB, radius RA and RB and mass MA (b)
GM
and MB. Then
(a) if TA > TB then RA > RB (b) if TA > TB then MA > MB 2 r
(c)
2 3
v
TA RA
= (d) The particle will not be able to complete a revolution,
RB ÷
(c) (d) TA = TB
TB as it will fall off towards the centre

MARK YOUR 46. 47. 48. 49. 50.


RESPONSE 51.

Me Mass of earth
PASSAGE-1
Mm Mass of Mars.
Le Angular momentum of earth around the sun.
It can be assumed that orbits of earth and mars are nearly Lm Angular momentum of mars around the sun.
circular around the sun. It is proposed to launch an artificial Re Semi major axis of earth’s orbit.
planet around the sun such that its apogee is at the orbit of Rm Semi major axis of mars orbit.
mars while its perigee is at the orbit of earth. Let Te and Tm be M Mass of the artificial planet.
periods of revolution of earth and mars. Further the variables Ee is total energy of earth.
are assigned the meanings as follows. Em is total energy of mars.
GRAVITATION 259

1. Time period of revolution of the artificial planet about sun 6. Find the time period for the motion in previous question
will be (neglect gravitational effects of earth and mars)
R 2h R 2h
Te + Tm (a) 2 +4 (b) 2 +2
TeTm
g g g g
(a) (b)
2
R 2h
(c) +2 (d) None of these
3/ 2 g g
2TeTm Te2 / 3 + Tm2 / 3
(c) (d)
Te + Tm 2
PASSAGE-3
2. The total energy of the artificial planet’s orbit will be

2M Re Ee 2M Re Ee An intergalactic observation station is in a circular orbit of


M e Re + Rm ÷ M m Re + Rm ÷
(a) (b) radius 0.04 AU around a black hole of mass 10 solar masses.
The observation station is constructed in the shape of a
dumbbell with two spheres 20 m in diameter, connected by a
2 Ee M Re + Rm 2 Ee M Re + Rm
÷ hollow cylindrical beam of small cross section and 80 m
Rm ÷
(c) (d)
Re2 + Rm2 ÷
Mm Me long. [1 AU = 1.5 × 1011 m]

3. Areal velocity of the artificial planet around the sun will be


(a) less than that of earth (b) more than that of mars
(c) more than that of earth (d) same as that of the earth

PASSAGE-2
100m

If a tunnel is dug across the earth passing through centre


and a particle is dropped from surface it perform SHM inside

Radius
tunnel with time period, T = 2
g
0.04AU
4. Assume that a tunnel is dug across the earth (radius = R) Not to
passing through its centre. Find the time, a particle takes to scale
reach centre of the earth, if it is projected into the tunnel
from surface of earth with speed needed for it to escape the
gravitational field of earth.

1 1 Re 1 1 2Re
sin sin ÷
(a) ÷ g
(b)
3 g
3

1 1 Re
(c) sin ÷ (d) None of these
3 2g
7. The orbital period of the station is
5. If a particle is drop from height h (h << R) into the tunnel
then choose the correct option – (a) 7.99 × 104 s (b) 7.99 × 102 s
(a) Particle will perform SHM (c) 3.99 × 104 s (d) 7.99 × 103 s
(b) Particle will not perform SHM 8. The tangential velocity of the station is
(c) Particle will perform periodic motion (a) 4.72 × 102 m / s (b) 2.72 × 105 m / s
(d) Both (b) and (c) (c) 4.72 × 105 m / s (d) 4.72 × 103 m / s

MARK YOUR 1. 2. 3. 4. 5.
RESPONSE 6. 7. 8.
260 IIT-JEE PHYSICS Challenger
9. The station is aligned in orbit so that the long axis of the position.
dumbbell is aligned radially with respect to the centre of the (a) 2.66 × 10–1 N (b) 1.66 × 10–2 N
black hole. If each of the spheres weighs 100 tonnes, (c) 2.66 × 10 N–4 (d) 2.66 × 10–2 N
calculate the tension on the connecting beam. 11. The cylindrical beam is made of a titanium alloy which has a
(a) 1.124 N (b) 0.124 N (c) 3.114 N (d) 0.213 N yield strength of 830 MPa (mega Pascals). If the beam has a
10. If the alignment of the station is stable configuration, any diameter of 2.00 m and a wall thickness of 5 cm, calculate
change in that alignment would have to result in forces how close the station can come to the black hole and not be
which cause the station to return to its stable alignment. torn apart by the differential gravitational forces.
Calculate the magnitude of the gravitational forces which (a) 4.72 ×102 m. (b) 4.72 ×106 m.
would occur if the dumbbell axis were rotated 30° out of 3
(c) 4.72 ×10 m. (d) 4.72 ×108 m.

MARK YOUR
9. 10. 11.
RESPONSE

1. Statement - 1 : A balloon filled with hydrogen will fall with 5. Statement - 1 : We can not move even a finger without
g disturbing all the stars.
acceleration of the moon. Statement - 2 : Every body in this universe attracts every
6
other body with a force which is inversely
Statement - 2 : Moon has no atmosphere. proportional to the square of distance
2. Statement - 1 : The length of the day is slowly increasing between them.
Statement - 2 : The dominant effect causing a slowdown in 6. Statement - 1 : Orbital velocity of a satellite is greater than
its escape velocity.
the rotation of the earth is the gravitational
Statement - 2 : Orbit of a satellite is within the gravitational
pull of other planets in the solar system. field of earth whereas escaping is beyond
3. Statement - 1 : Generally the path of a projectile from the the gravitational field of earth.
earth is parabolic but it is elliptical for 7. Statement - 1 : If earth suddenly stops rotating about its
projectiles going to a very large height. axis, then the value of acceleration due to
gravity will become same at all the places.
Statement - 2 : The path of a projectile is independent of
Statement - 2 : The value of acceleration due to gravity is
the gravitational force of earth. independent of rotation of earth.
4. Statement - 1 : The time period of revolution of a satellite 8. Statement - 1 : The square of the period of revolution of a
close to surface of earth is smaller than planet is proportional to the cube of its
that revolving away from surface of earth. mean distance from sun.
Statement - 2 : The intensity of the gravitational field of
Statement - 2 : The square of time period of revolution of the sun acting on the planet is proportional
a satellite is directly proportional to cube to the mass of the sun and inversely
of its orbital radius. proportional to square of its distance from
the sun; it also acts towards the sun.

MARK YOUR 1. 2. 3. 4. 5.
RESPONSE 6. 7. 8.
GRAVITATION 261

9. Statement - 1 : Gravitational potential due to earth at its


own centre is negative. Q
Statement - 2 : Gravitational field due to earth is vector
quantity. S
10. Statement - 1 : A planet may orbit around a star either in O
orbit P or orbit Q. The speed of a planet at circle
P
O is same for both orbits.
Statement - 2 : The radius vector of point O from the sun
is same for both orbits P and Q.

MARK YOUR
9. 10.
RESPONSE

1. A particle is dropped from a height equal to the radius of the


F1 r1
=
F2 r2 if r1 > R and r2 > R
earth into a tunnel dug through the earth along one of the (c)
diameters as shown in the figure.

F1 r12
=
R (d) F2 r22 if r1 < R and r2 < R
3. Which of the following statements are true about
acceleration due to gravity ?
C (a) g decreases in moving away from the centre if r > R
(b) g decreases in moving away from the centre if r < R
(c) g is zero at the centre of earth
R : Radius of earth (d) g decreases if earth stops rotating on its axis
M : Mass of earth 4. A satellite is launched and attains a velocity of 30400 km/hr
(a) Particle will oscillate through the earth to a height R on relative to the centre of the earth at a height of 320km from
both sides the earth’s surface. It has been guided into a path that is
(b) Particle will execute simple harmonic motion parallel to the earth’s surface at burnout. Choose the correct
(c) Motion of the particle is periodic options
(a) Satellite moves along an elliptical orbit
2GM (b) Longest distance from the earth’s surface is 3550 km.
(d) Particle passes the centre of earth with a speed
R (c) he period of revolution for the satellite is 2.09 hrs.
2. The magnitudes of the gravitational field at distance r1 and (d) The minimum escape velocity for this position of
r2 from the centre of a uniform sphere of radius R and mass launching is 10930.08 m/s
m are F1 and F2 respectively. Then: 5. Imagine a light planet revolving around a very massive star
in a circular orbit of radius R with a period of revolution T. If
F1 r1 the gravitational force of attraction between the planet and
=
(a) F2 r2 if r1 < R and r2 < R the star is proportional to R–5/2
(a) T2 is proportional to R3
F1 r22 (b) T2 is proportional to R7/2
=
(b) F2 r12 if r1 > R and r2 > R (c) T2 is proportional to R3/2
(d) T2 is proportional to R3/73

MARK YOUR
1. 2. 3. 4. 5.
RESPONSE
262 IIT-JEE PHYSICS Challenger
6. Which of the following statements is/are correct – 10. A rocket starts vertically upwards with speed v0. Choose
(a) The radius R of planet’s orbit and period T bear the the correct option(s) (R is the radius of the earth).
relation R2/T3 = constant for all planets (a) Speed v at a height is given by
(b) The gravitational potential energy of a satellite earth
system decreases as the period of the satellite 2 gh
v02 v2 =
decreases. h
1+
(c) The period of an earth-stationary satellite, as observed R
from earth, is infinite
(b) The maximum height reached by the rocket fired with a
(d) The mass of a body placed in a orbiting satellite
becomes zero speed of 90% of escape velocity is 2.26 R
7. Consider an attractive central force of the form (c) The maximum height reached by the rocket fired with a
speed of 90% of escape velocity is 4.26 R
k
F (r) = – , k is a constant. For a stable circular orbit to (d) Speed v at a height is given by
rn
2 gh
exist v02 v2 =
h
(a) n = 2 (b) n < 3 1+
(c) n > 3 (d) n = –1 2R
8. A solid sphere of uniform density and radius 4 units is located 11. A communication earth satellite
with its centre at the origin O of coordinates. Two spheres
(a) goes round the earth from east to west
of equal radii 1 unit, with their centres at A (–2, 0 ,0) and
(b) can be in the equatorial plane only
B (2, 0, 0) respectively, are taken out of the solid leaving
(c) can be vertically above any place on the earth
behind spherical cavities as shown in fig
(d) goes round the earth from west to east
y 12. A satellite S is moving in an elliptical orbit around the earth.
The mass of the satellite is very small compared to the mass
of the earth.
-m (a) The acceleration of S is always directed towards the
A B
x
O centre of the earth.
(b) The angular momentum of S about the centre of the
z earth changes in direction, but its magnitude remains
Then : constant.
(a) The gravitational force due to this object at the origin (c) The total mechanical energy of S varies periodically
is zero. with time.
(b) the gravitational force at the point B (2, 0 ,0) is zero. (d) The linear momentum of S remains constant in
(c) the gravitational potential is the same at all points of magnitude.
circle y2 + z2 = 36. 13. Titan, the largest satellite of Saturn has a radius of 2,575 km
(d) the gravitational potential is the same at all points on and a mass of 1.36 × 1023 kg. The NASA Cassini probe has
the circle y2 + z2 = 4. measured its average surface temperature to be -178°C.
9. For two satellites at distance R and 7R above the earth’s Which of the following gases would you expect to remain in
surface, the ratio of their the atmosphere of Titan (A rule-of-thumb among
(a) total energies is 4 and potential and kinetic energies
astronomers is that if the average speed is below 1/6 of the
is2
escape velocity, the gas will remain for the age of the solar
(b) potential energies is 4
system)
(c) kinetic energies is 4
(d) total energies is 4 (a) ammonia (b) methane
(c) helium (d) hydrogen

MARK YOUR 6. 7. 8. 9. 10.


RESPONSE 11. 12. 13.
GRAVITATION 263

1. Match the following


Column-I Column-II
(A) Q(r) (p) Q (r) is variation of gravitational potential for
a uniform volumetric spherical distribution of mass

O r=R r
(B) Q(r) (q) Q (r) is variation of gravitational potential
for a uniform spherical shell

O r=R r

(C) Q(r) (r) Q (r) is variation of gravitational field intensity for a


uniform volumetric spherical distribution of mass

O r=R r

(D) Q(r) (s) Q (r) is variation of gravitational field


intensity for a uniform spherical shell

O r=R r
Where Q (r) is magnitude of physical quantity as a function of r (distance from centre of spherical distribution of radius R)
2. Column I lists some parameter of the electron in the hydrogen atom. Column II lists their dependences with respect to the
principle quantum number n. Match the entries in Column I with those in Column II.
Column I Column II
1
(A) Radius of the orbit (p)
n
1
(B) Momentum of the electron (q)
n2
(C) Time period of circular motion (r) n 3
(D) Kinetic energy (s) n 2

1. 2.

MARK YOUR
RESPONSE
264 IIT-JEE PHYSICS Challenger
3. Match the columns I and II
Column I Column II
(A) Elliptical orbit of planet (p) Kinetic energy conservation
(B) Circular orbit of satellite (q) Angular momentum conservation
(C) Escape velocity (r) Independent of mass of particle/satellite

GM
(D) Orbital velocity (s)
R

(t) Areal velocity constant


4. Match the columns I and II
Column I Column II
(A) Kinetic energy of a body projected from surface of earth, (p) must be zero
at large distance from surface of earth
(B) Gravitational potential energy of a bound system (q) may be zero
(C) Change in potential energy of a point mass if left free to (r) positive
itself, with time
(D) Change in areal velocity of earth as earth moves from (s) may be negative
apogee towards perigee
(t) must be negative
5. Considering earth to be a homogeneous sphere but keeping in mind its spin, match the columns I and II correctly.
Column I Column II
(A) Acceleration due to gravity (p) May change from point to point
(B) Orbital angular momentum of the earth as seen from a (q) Does not depend on direction of projection
distant star
(C) Escape velocity from the earth (r) Remains constant
(D) Gravitational potential due to earth at particular point (s) Changes with time
(t) Depends on earth mass

3. 4. 5.

MARK YOUR
RESPONSE
GRAVITATION 265

1. If the radius of the earth were to shrink by two percent, its velocity from the earth. If the satellite is stopped suddenly
mass remaining the same, by how much percentage would in its orbit and allowed to fall freely onto the earth, find the
the acceleration due to gravity on the earth’s surface would speed (in m/s) with which it hits the surface of the earth.
increase ? 5. A body is projected vertically upwards from the bottom of a
2. The period of revolution of planet A around the sun is 8
R
times that of B. The distance of A from the sun is how many crater of moon of depth where R is the radius of moon
100
times greater than that of B from the Sun?
with a velocity equal to the escape velocity on the surface
3. Suppose earth’s orbital motion around the sun is suddenly
of moon. How many times of the radius of the moon is the
stopped. What time (in days) will the earth take to fall into
maximum height attained by the body from the surface of
the sun?
the moon ?
4. An artificial satellite is moving in a circular orbit around the
earth with a speed equal to half the magnitude of escape

1. 2. 3. 4. 5.

MARK
YOUR
RESPONSE
266 IIT-JEE PHYSICS Challenger

1 (b) 7 (a) 13 (a) 19 (b) 25 (c) 31 (b) 37 (a) 43 (d) 49 (d)


2 (a) 8 (c) 14 (d) 20 (d) 26 (b) 32 (a) 38 (b) 44 (d) 50 (a)
3 (c) 9 (b) 15 (a) 21 (b) 27 (a) 33 (d) 39 (d) 45 (c) 51 (c)
4 (c) 10 (a) 16 (a) 22 (b) 28 (c) 34 (d) 40 (b) 46 (b)
5 (b) 11 (a) 17 (b) 23 (c) 29 (b) 35 (c) 41 (b) 47 (d)
6 (c) 12 (d) 18 (b) 24 (d) 30 (a) 36 (a) 42 (a) 48 (a)

1 (d) 3 (c) 5 (d) 7 (a) 9 (b) 11 (b)


2 (a) 4 (a) 6 (a) 8 (c) 10 (d)

1 (a) 3 (c) 5 (a) 7 (c) 9 (b)


2 (c) 4 (a) 6 (c) 8 (a) 10 (d)

1 (a, c, d) 4 (a, b, c, d) 7 (a, b, d) 10 (a, c) 13 (a, b)


2 (a,b) 5 (b) 8 (a,c,d) 11 (b, d)
3 (a, c) 6 (b, c) 9 (b, c, d) 12 (a)

1. A-q; B-r; C-s; D-p 2. A-s; B-p; C-r; D-q


3. A-q, t; B-p, q, t; C-r; D-r, s 4. A-q, r; B-t; C-q, s; D-p
5. A-p, t; B-r, t; C-p, q, t; D-r, t

1 4 2 4 3 65 4 7920 5 99.5
GRAVITATION 267

4. (c) During total eclipse :


l Total attraction due to sun and moon,
1. (b) On earth, T 2
g
GM s M e GM m M e
F1 = +
l r12 r22
On moon, T 2
g¢ When moon goes on the opposite side of earth.
Effective force of attraction,
mg GM s M e GM m M e
but mg F2
6
r12 r22
g
or g 2GM m M e
6 Change in force, F F1 F2 =
r22
6l Change in acceleration of earth
T 2 6T 6 min ( T 1min )
g
F 2GM m
So, in actual time of 1 minute on earth, on moon its ticks a= M =
e r22
60
out sec = 24.5 sec Fav GM s
6 Average force on earth, Fav = = 2 % age change
Me r1
2. (a) The velocity vs of the satellite is given by
in acceleration
2
G Mms ms vs GM
= vs = a 2GM m r12
rs 2 rs rs ÷ …(i) = a 100 100
av r22 GM s

1 1 GM GMms 2
K.E. = ms vs 2 = ms P.E. r1 M m
2 2 rs ÷ ; 2 100
r2 ÷ M s
rs

GMms
Total energy E = K.E. + P.E. …(ii)
2rs
Gm ( dx )
The angular momentum L is given by
5. (b) dF =
x2 dx
1/ 2 x
GM
L = ms vs rs = ms rs = (GMms 2 rs )1/ 2 …(iii)
rs ÷
x
From eqs. (ii) and (iii), we get
L = (2 E ms rs 2 )1/ 2 dx
( A + Bx )
a + L

F = Gm 2

3. (c) By conservation of energy a

x 2

1 1
GMm 1 m GM
+ =
GMm F Gm A ÷ + BL
RE 2 N 2 2 RE H a a+L
6. (c) A satellite revolving near the earth's surface has a time
N 2 RE period of 84.6 min.
H= We know that as the height increases, the time period
N2 1
increases. Thus the time period of the spy satellite
RE should be slightly greater than 84.6 minutes.
Altitude = H R= Ts = 2 hr
N2 1
268 IIT-JEE PHYSICS Challenger
7. (a) We know that intensity is negative gradient of potential, Since, L is constant, thererfore
i.e., I = – (dV/dr) and as here I = – (K/r), so 1 1
K.E. µ K .E. µ 2
dV K I r
, i.e., dV K
dr r E2 r12 1
= =
or V V0
r E1 r22 16
K log
r0 11. (a) v = R
2
r g g0 R [g = at equator, g0 = at poles]
so V K log V
r0
g0 g0 g0 R
g0 2
R ; 2
R= ; v =
2
8. (c) If a body is projected with ( 2 gRe ) greater than escape 2 2 2

velocity 2gRe then by conservation of energy ve = 2g 0 R = 4v 2 = 2v

1 2 GMm 1 GM
mv m (v ) 2 0 12. (d) Gravitational potential at P, V p = Gravitational
2 Re 2 5R

2GM GM
i.e., (v )2 v2 v2 ve2 potential at O, VO =
Re R

i.e., v v2 ve2 4gRe 2gRe = 2gRe M,R


5R
9. (b) The escape velocity from earth is given by
m
ve = 2g Re ...(i) P
O
The orbital velocity of a satellite revolving around earth
is given by
GM e
v0 = By work energy theorem,
( Re h)
1 2
where, Me = mass of earth, Re = radius of earth, h = height W K m[VP VO ] = mv
of satellite from surface of earth. 2
By the relation GMe = g Re2
GM GM 1 2 2GM 1
m = mv or (1 )
g Re2 R 5R 2 R 5
So, v0 ...(ii)
( Re h ) 13. (a) Using Kepler’s 2nd law
Dividing equation (i) by (ii), we get dA J
= [J = angular momentum]
ve 2( Re h) dt 2 m
v0 ( Re ) A
Integrating, 2m =J
v T
Given, v0 = e
2 GM
14. (d) Acceleration due to gravity g =
2ve 2( Re h ) R2
G = gravitational constant, M = mass and
ve Re R = radius of earth
Squaring on both side, we get R 99
2( Re h ) New radius R' = R – R
4= or Re + h = 2Re i.e., h = Re 100 100
Re 2
GM GM 100
10. (a) Angular momentum remains conserved during the g' = 2
revolution of planet. Because gravitational force is a 99 R2 99
R
central force. 100
Now
2
1 2 m2 v 2 r 2 L2 100
g' = g
K.E. = mv = = 99
2 2mr 2 2I
GRAVITATION 269

g' g Using conservation of energy


% change in g = × 100
g 1 2 – GM e m – GM e m 1 2
mv p + = = mv0
2 2 rA r0 2
100
2 (100)2 (99)
1 100 = 2
100
99 (99)
9v02 r02 v02 r0 v2 r0
=+ 0 Let =x
199 50r A2 rA 2 rA
= 100 2%
(99) 2 9x2 – 50x + 25 = 0
15. (a) Work done, W = Uf – Ui x = 5 or (5/9)
G m1 m 2 G m1m 2 vP rA
= rf ri = =9
v A rp

1 1 1 1 1 2 GMm
= Gm1m2
ri rf
= Gm1m2
x1 x1 + d
19. (b) mvmin – 2=0
2 R 2

v1 r2 4R
16. (a) = = =2 2GM 2
v2 r1 R vmin =
R

1 2 2=
GMm
20. (d) mr
K1 2 mv1 v2 r2
= = 1 =4
K2 1 2 v2 GM = r3 2 (GM = gR2)
mv 2
2 2
r 3. 2
2GM g=
R2
ve R GM
17. (b) v
2 2 2R (4nR)3 . 2
g' =
If h is the height reached by the body, then by n 2 .R 2
conservation of mechanical energy
g' = 64 nR 2.
2 21. (b) Since gravitational field intensity inside hollow sphere
GMm 1 GM GMm
+ m ÷ = +0 M2 will be zero.
R 2 2R R+h So, gravitational force will be due to M1 only.
22. (b) By conservation of angular momentum
3 1
or = mv1 (2R) = mv2 (4R)
4R R + h v1 = 2v2 ... (1)
R By conservation of energy,
h=
3 1 2 GMm 1 2 GMm
mv1 mv2 ... (2)
–GM e m 2 2R 2 4R
18. (b) TE =
2r0 From (1) and (2), we get

GM 2GM
A vmin = ; vmax =
6R 3R
rA
GM e m GM e m GM e m
23. (c) E1 = – =
R Re ÷ 3R e
O Re + e
r0 v0 2
rp
P 1 2 1 GM e GM e m
E2 = mv0 = m. =
2 2 R 3R e
R+ e
Using conservation of angular momentum about O. 2
mvp rp = mvA rA = mv0 r0 cos
E1
3v0 r0 = 1:1
vA rA = vp rp = E2
5
270 IIT-JEE PHYSICS Challenger
24. (d) Differentiating the equation of the curve w.r.t. t we get, Now, let v'1 and v'2 be the velocities of the satellite at A
2 and B while the transfer is pursued from orbit C1 to C2.
dx d2 y Evidently, the points A and B will be the positions of
2 ÷ 500
dt dt 2 perigee and apogee for the satellite along its elliptical
path.
d2y
2 104
or, = = 4g. the effective 'g' = 5g. Conserving angular momentum
dt 2 500 v'1 (3R) = v'2 (6R)
25. (c) The gravitational potential at a point Q (OQ = x) is v1
given by v2 =
2
3 1 x2 Conserving energy between A and B, we have
gs R ÷ , when x < R
2 2 R2 1 GMm 1 GMm
V (x) = m (v1 ) 2 m ( v2 ) 2
R 2 3R 2 6R
gs R ÷ , when x > R
x
1
v1 2 v22 = 2GM ÷
The energy required to project the body to a maximum 6R
altitude of 3R from its surface, is
v1 2 GM
9 v1 2 =
m VB R VP x = 4 R ÷ = mg s R. 4 3R
x= ÷ 8
2
3v1 2 GM 2 GM
v1 =
26. (b) F = F1 + F 2 + F 3 + F 4 + F 5 4 3R 3 R

m m 1 GM
So, v2 =
3 R
Now, change in energy required at point A
m F2 F3 m 1 1 2
E1 mv1 2 mv1
2 2
F1 F4
m 4 gR m gR mgR
( gR ) ÷=
m
F5
m 2 9 3 2 9 18
Similarly change in energy required at point B
F 2 = F 5 and F 2 = F 4 1 2 1 m gR gR
E2 mv2 mv22
2 2 2 6 9
F1 = F 3 + 2 F2 cos30° + 2 F1 cos60°
m gR mgR
Gm 2 Gm 2 Gm 2 = ÷=
F3 = ; F2 = ; F1 = 2 18 36
4a 2 3a 2 a2
GM
Gm 2 5 1 28. (c) g=
F= 2 + R2
4 ÷ =m 2a
a 3
GM
g' =
Gm 5 1 (0.99 R) 2
= + ÷
a3 4 3
2
g' R2
= ÷
4 3a3 g 0.99 R
T 2
Gm (5 3 + 4)
g' > g
27. (a) Let v1 and v2 be the velocity required for the satellite in
order to maintain its motion along the circular orbits 29. (b) ve = 2 g e Re ; vm = 2 g m Rm
C1 and C2 respectively.
ve ge Re ge Re
GM GM = = = 24
Then v1 = and v2 = vm g m Rm ge / 6 Re / 4
3R 6R
GRAVITATION 271

Conserving mechanical energy


mdv
30. (a) F= = av 2
dt GMm 1 2 GMm 1 2
+ mv0 = + mv
5R 2 R 2
GM
t 1 4GMm 8GM
R
dv M m (v 2 v02 ) v 2 v02 = ..... (2)
m = a dt 2 5R 5R
v2 R
GM 0 nR Substituting the value of v from eq. (1) in equation (2)
nR
8GM
25v02 sin 2 v02 =
5R
R nR v
+ m
GM GM F = av 2 1 8GM
=t = sin 1
1+ 2 ÷
a 5 5v0 R
m R ( n 1) 32. (a)
t=
a GM A
Energy method : Total energy in circular orbit of radius r,
GMm
E h
2r
dE GMm dr
Rate of change of energy =
dt 2r 2 dt
GMm dr
= F .v = av3 ....... (1)
Gauss law for gravitation
2r 2 dt
GM
Centripetal force = gravitational force g .ds min .4 G ; g =
R2
mv 2 GMm GM GM
3/ 2
v2 = v3 = ÷ GM M 2R
r r2 r r 2 2
A (h A) 4 G h=
R 4 3 3
From eq. (1), R
3
GMm dr (GM )3/ 2 L dA
= a 33. (d) = (L = angular momentum)
2r 2 dt r 3/ 2 2 m dt
GMm mvmax rmin dA
dr = a(GM )3/ 2 dt = ;
2r 1/ 2 2m dt
2 dA / dt
R t vmax = = 40
GMm rmin
1/ 2
dr = a(GM ) 3/ 2
dt
nR 2r 0 34. (d) If M is the mass and R is the radius of earth, then the
M
GMm m density =
. The spherical volume may be
t 3/ 2
[ nR R] ( nR R) 4 3
a (GM ) a GM R
3
m R ( n 1) supposed to be formed by a large number of their
= concentric spherical shells. Let the sphere be
a GM
disassembled by removing such shells. When there is
31. (b) Let the speed of the instrument package is v when it a spherical core of radius x the energy needed to
grazes the surface of the planet. disassemble a spherical shell of thickness dx is
v0 Gm1m2 4 3
R dW = ; m1 x
m ? x 3
m2 = mass of spherical shell of radius x and thickness
dx = 4 x2dx .
Conserving angular momentum of the package about
the centre of the planet 4 3
G x ÷ (4 x 2 dx )
mv0 × 5R sin ( – ) = mvR sin 90° 3 16 2 2 4
dW = Gx dx
v = 5v0 sin ......... (1) x 3
272 IIT-JEE PHYSICS Challenger
Total energy required Here T1 = 365 days ; T2 = ?
R R R
16 2 2 16 x5 R1 = R ; R2 =
W= Gx3 dx 2 2
G 2
3 3 5
0 0 3/ 2
R2
R1 ÷
T2 = T1
16 GR 5 16 2 M ÷ 3 GM 2
= 2 2
= ÷ GR 5 =
3 5 15 4 3 5 R
R ÷ 3/ 2
3 R/2
But GM = gR2 = 365 = 129days
R
2 2
3 gR M 3 3
W = = gMR = 10 2.5 1031 39. (d)
5 M R 5 5
= 1.5 × 1032 J
35. (c) For this (for a particle on its equator) d
4 3 p p
G r C1 C2
3 2
r C1 C2
r2
M
3 3 2 8
= R –d
4 G GT 2 m 2
if T = 1 second
> 1.4 × 1011 kg/m3. M R
36. (a) For calculation of gravitational field intensity inside G d÷
GMd 8 2
the cavity. (Ep) = +
R3 R
3
÷
2
P R
r1 r2
GM ÷
P r1 r2 = GMd 2 GMd
=
GM
R2 R3 R 3
R 3
2R 2
R1
GMm
(I2) Force =
Gravitational field 2R 2
intensity without
cavity (I1) + r1 = r2 40. (b)

rj
4 3 4 3
G R ÷ ( r1 ) G R ÷ ( r2 ) re
3 1 3 2 Jupiter
I1 = , I2 = M
Sun
R13 R23 Earth

I I1 I2 ( I – intensity inside the cavity)


GM
4 4 v=
= G [ r1 + r2 ] = G r
3 3 Distance of Jupiter from Sun is more than Earth.
GM1M 2 i.e., rj > re. vj < ve
37. (a) Ug 41. (b)
R
GMm GMm v2
Uf Ui ÷
2R R

GMm mgR GM
U= = g= 2 S
2R 2 R Perigee r1 r2 Apogee
38. (b) According to Kepler's law

T12 R13
=
T22 R23
v1
GRAVITATION 273

According to the law of conservation of angular 46. (b) Remote half from the sun is the arc B'A'B.
momentum, mv1r1 = mv2r2
B
v1 r2 a (1 + e)
= =
v2 r1 a (1 e) ,
where e is eccentricity of the earth’s orbit A A'
S C
(1 + 0.0167)
= = 1.034
(1 0.0167) B'
42. (a) Let the gravitational field be zero at a point distant x Let t be the time of description of the arc B'A'B, therefore
from M1. rate of description of the sectorial area SB'A'BS
GM1 GM 2 x M1 area SB A BS
2
= 2 ; d = =
x (d x) x M2 t
Since the whole area of the ellipse is described in a
x M2 M1 d x M1 year, therefore rate of description of the sectorial area
is
x M1 + M 2 = M 1 d area of the ellipse
= a year
d M1 d M2
x= , d x=
M1 + M 2 M1 + M 2 Hence, area SB A BS = area of the ellipse
Potential at this point due to both the masses will be t a year
GM1 GM 2 t area SB A BS
or =
x ( d x) year area of the ellipse

M1 ( M1 + M 2 ) + M2 ( M1 + M 2 ) 1
area of the ellipse SBB
G = 2
d M1 d M2 ab
1 1
ab + .2bae
=
G
d
( M1 + M 2 )
2
=
2 2 1 e
= +
ab 2
G
d
(
M 1 + M 2 + 2 M1 M 2 ) t=
1 e
2
1 e
+ ÷ year = year + year
2
43. (d) Angular momentum of the planet about S is conserved.
1
So, mvr = constant. =
year + 2 days nearly
v is maximum when r is minimum. 2
So, v is maximum at point P4. Hence K.E. is maximum at Solving e = 1/60
47. (d) 4 r2dr = volume of an element of thickness dr.
P4 .
The number of particles, ni = 4 r2dr. , assuming is
44. (d) From Kepler’s law, T2 µ R3
the number of particles per unit volume. The
Parking orbit, 12 µ R3
After missing, T2 µ (3R)3 acceleration due to gravity =
g
r for each particle.
R
T 2 27 R3
= = 27 ; T = 27 = 3 3 day g g 3
1 R3 ni g 4 r 2 dr .
r 4 r dr
R R
GMm Mean acceleration of the particles due to gravity,
45. (c) E
2r R
4 g 3
r dr ÷
dE GMm 1 dr R 4 g R4 1
= g= 0 = .
dt 2r r 2 dt (4 / 3) R 3 R 4 (4 / 3) R3
t R
GMm dr GMm 1 1 R3 1 3
dt t = 4 g. = g
0
2C r r 2 ; 2C R r 4 (4 / 3) R 3 4
274 IIT-JEE PHYSICS Challenger
This is the mean acceleration due to gravity.

ni r
Average distance travelled by the particle is = P
ni
90°
R
4 r 2 dr .r
0 R4 1
r r 4 S
4 3 4 4 3
R R
3 3

3
r = R
4

1
r = at 2 where a = average acceleration Since the two paths are parabolic, therefore
2
2
v12 = = v22 (at the point P) ........... (1)
3 1 3 2 2R R
R . g .t t=
4 2 4 g
acc. =
2 1/ 2 3 0.5 (distance)2
GM 2 R.R 2R
g t= = ÷ After collision the two masses combine into one,
R2 GM GM
let V be the velocity after impact of the single mass
Collision between particles is neglected. It is as if every (m1 + m2).
particle is moving unhindered with no other interaction By the principle of conservation of momentum the
towards the centre. The density of particles P is very resultant momentum must be the same after and before
small. Otherwise it will be making collisions like the the impact :
electrons hitting each other but moving under the ( m1 + m2 ) 2 V 2 = m12 v12 + m22 v22 ........... (2)
electric potential. Here it will be gravitational potential.
because v1 and v2 are at right angles.
The model used is a simplified one. From (1) it is seen that v2 = v1 and then from (2), we
48. (a) By Kepler’s law have
dA A L ( m1 + m2 ) 2 V 2 = ( m12 + m22 )v12
= =
dt T 2 m
m12 + m22
2mA i.e. V 2 = v12 ........... (3)
L= ( m1 + m2 ) 2
T
49. (d) The gravitational force of attraction between the stars It is seen from (3) that after the collision the (velocity)2
will provide the necessary centripetal forces. 2
In this case angular velocity of both stars is the same. becomes less than v12, that is less than , which is
R
2 the condition for describing an ellipse.
Therefore time period remains the same. = ÷. So after the collision the path will be an ellipse. Let 2a'
T
be its major axis.
For the path to be an ellipse,
2 1
v2 ÷
r a
RA
MB MA At the point P, just after the collision v = V and r = R.
RB
2 1
So, V 2 ÷
R a
Substituting for V2 from (3), we have
50. (a) Let P be the common point of the two parabolas where
the two particles collide. Let v1, v2 be the velocities of m12 + m22 2 1
v2
2 1 ÷
m1 and m2 at the time of collision. ( m1 + m2 ) R a
GRAVITATION 275

Now substituting for v1 2 from (1), we have


(m1 + m2 ) 2
i.e. 2a = .R
m12 + m22 2 2 1 2m1m2
÷
( m1 + m2 ) 2 R R a
( m1 + m2 ) 2
that is, major axis = .R
1 m12 + m22 2 2m1m2 2 2 m1m2
1 . = .
a ( m1 + m2 ) 2 R ( m1 + m2 ) R
2
51. (c) Inside the shell there will be no gravity. Hence,

2 r
T=
v

1. (d) T 2 µ R3
ve ve
Te2 = KRe3 ; Tm2 = kRm
3
; T 2 = kR3 R R
?
A

Re + Rm O
R=
2
3
Te2 / 3 Tm2 / 3 1
T =k2
+ According to energy conservation
k1/ 3 k1/ 3 2
GM m GM 1 2GM
3/ 2 +0= 3
(3 A2 R2 ) m + m ÷
Te2 / 3 + Tm2 / 3 A 2A 2 R
T=
2 A3 M A3 M M
(3 A2 R2 ) +
GM s M e GM s M R3 A 3
R 2A 3 R
2. (a) Ee =–
2 Re 2R 1
A2 (3 A2 R 2 ) + R 2
2
2 Re Ee M
3 2 R2
Me Re + Rm A2 A + + R 2 ; A = 3R
2 ÷
2 2 2
1 1 1
Re
sin = ; = sin ÷
2M 3 3
= Ee
M e Re + Rm ÷
1 A 1 Re
T= ÷ T = sin
1
= sin 1
÷
3. (c) Areal velocity of the artificial planet around the sun 2 3 gA 3 g
will be more than that of earth.
4. (a) We know if a particle is released (initial speed zero) Re 1
T = sin 1
÷
3 g
Radius 5. (d) At a height h from earth surface acceleration due to
from earth surface time period = 2 , so we
g gravity is not proportional to distance from centre hence
imagine a earth of radius A from where particle is particle will not perform SHM but the motion will be
released and when it reaches at distance R from earth periodic.
centre its speed is ve. Mass of assumed earth of radius
1 2 2h
is A. 6. (a) h gt t=
2 g
3
A
M = ÷ M R 2h
R
R Time period = 2 + 4t = 2 +4
g g g
276 IIT-JEE PHYSICS Challenger
7. (a), (8) (c), (9) (b), (10) (d), (11) (b). To calculate the displacement angle between one of
For a circular orbit, with the central mass much greater the spheres, the black hole and the radial axis, the
than the satellite mass:
distance from the sphere to the radial axis D = R
Fcentral = Fgravitational
m 2 r = GMm / r2 sin(30°) = 25 m.
= [GM/r3]1/2 D D
= [6.67 × 10–11 × 20 × 1030 / (0.04 × 1.5 × 1011)3]1/2 Then, sin ( ) =
R R cos (30°) R
= 7.86 × 10–5
The restoring force perpendicular to the radial axis,
Period = 1 / f = 2 / = 7.99 × 104 s
Tangential Velocity = r = 4.72 × 105 m / s F RA (a component of the force of the black hole on
The tension in the beam is due to the difference in one sphere) is
gravitational force between the inner and outer sphere. GMm GMmD 2
F RA = 2
sin ( ) 3
1.54 10 N
GMm1 GMm2 R R
T
(R R )2 (R R )2 The restoring force FR is a component of F RA,
perpendicular to the axis of the observation station is
GMm 1 1 given by
= 2 2
R (1 R / R) (1 R / R)2 FjR = F RA cos (30°) = 1.33 × 10–2 N.
As R / R << 1, we can use the binomial expansion on A similar restoring force acts on the other sphere.
both fractions. As , this force is equal to the force on the first
4GMm R
= sphere, so the total restoring force is 2.66 × 10–2 N.
R3 The maximum tension force Fmax that can be applied
11 30 3 to the beam is given by
4 6.67 10 20 10 100 10 50
= 11 3
(0.04 1.5 10 ) Fmax = Cross sectional area of beam x yield strength
= (1 – 0.952) × 830 × 106 = 2.54 ×108 N
= – 0.124 N The distance of closest approach is given by :

4GMm R
Fmax =
R3
30°
4GMm R
or R3 =
Fmax
R
Rmin = 4.72 ×106 m.

0.04AU
'
GRAVITATION 277

1. (a) A balloon will not experience any buoyant force on the distance between them. When we move our finger,
moon because it has no atmosphere, so it will have free the distance of the object with respect to finger
fall under gravitational pull of the moon with acceleration changes, hence the force of attraction changes,
g disturbing the entire universe, including stars.
equal to . 6. (c) The orbital velocity, if a satellite close to earth is
6
2. (c) The length of the day is slowly increasing not due to v0 = gRe , While the escape velocity for a body
gravitational pull of other planets in the solar system
but due to viscous force between the earth and the thrown from the earth’s surface is ve = 2gRe .
atmosphere around it. So Statement-1 is true but
Statement-2 is false. v0 gRe 1
Thus = =
3. (c) Upto ordinary heights, the change in the distance of a ve 2gRe 2
projectle from the centre of earth is negligible compared
to the radius of earth. Hence the projectile moved under or ve = 2v0
a nearly uniform gravitational force and the path is i.e., if the orbital velocity of a satellite revolving close
parabolic. But for the projection moving to a large
to the earth happens to increase to 2 times, the
height the gravitional forced decreases quite rapidly
satellite would escape.
1 7. (c) The value of g at any place is given by the relation,
as F µ 2÷ . Under such a rapidly decreasing
r g ' g Re 2 cos2
variable force, the path of projectile become elliptical. Where is angle of latitude and is the angular
4. (a) The time period of satellite, T µ r 3 / 2 velocity of earth. If earth suddenly stops rotating,
then
or T µ ( Re + h ) 3 / 2 g' = g
=0
For a satellite revolving close to surface of earth h = 0 i.e., the value of g will be same at all places.
T µ Re 3 / 2 . It is evident that the period of revolution 8. (a) Statement-1 is true and Statement-2 is true and
of a satellite depends upon its height above the earth’s Statement - 2 is correct explanation of Statement - 1
surface. Greater is the height of a satellite above the 9. (b) Statement – 1 is True, Statement – 2 is True; Statement
earth’s surface, greater is its period of revolution. – 2 is NOT a correct explanation for Statement – 1.
5. (a) According to Newton’s law of gravitation, every body 10. (d) Speed changes in elliptical orbit, angular momentum
in this universe attracts every other body with a force remain same.
which is inversely proportional to the square of the

1. (a, c, d) The force outside the earth varies as inverse


GM
square of the distance. F1 =
Motion is not simple harmonic. r12
However, from symmetry of motion, the motion
will be periodic. GM F1 r22
and F2 = =
From COE r22 F2 r12
1 2 GMm 3GMm
mv ÷ GM
2 2R 2R For r < R, the gravitational field is F = r
R3
2GM
v= GM GM
R F1 = 3
r1 and F2 = r2
R R3
GM
2. (a,b) For r > R, the gravitational field is F = F1 r1
r2 =
F2 r2
278 IIT-JEE PHYSICS Challenger
Eliminating v' from the above two equations, we
R2 R2
3. (a, c) If r > R, g = g =g 2 have
( R + h) 2 r
2
672
R d 3565.43 104 1 ÷
If r < R, g = g r
R
where d is the depth below the surface of earth.
If d = R, g = 0. 9.8 (640 104 )2 1 1
= 4
Further, due to rotation 10 672 r
g' = g – R 2 cos2
If = 0, then g increases. r
where r = max m
4. (a, b, c, d) Given that the velocity of the satellite is 104
5 2
v = 30400 × m/s = 8444.44 m/s r (672)2
18 3565.43 104 2 ÷
Let us calculate the orbital velocity v0 for the r
satellite to move along a circular orbit of height
320 km. r 672
4014.08 107
672r
GM gR 2
v0 = =
r r 672
r' + 672 = 1.675 r' r = = 995m
0.675
9.8 (6.4 106 )2
= = 7728.73 m / s rmax = 995 × 104m
672 104
The maximum height of the satellite
= rmax – R = 9950 – 6400 = 3550 km.
S (c) The semi-major axis of the elliptical path will be
v=30400km/hr
rmax + rmin 995 + 672
320km a= = 4
÷ 10 m
2 2

R=6400km 833.5 104 m


O
From Kepler’s IIIrd law, of planetary motion,
Earth
a3 a3
T 2 T 2
GM gR 2

(833.5 104 )3
T 2
9.8 (64 105 ) 2

Also, the escape velocity for the satellite in this 2 3.14 1201.06 = 7546.5 sec. = 2.09 hrs.
position is given by ve = 2 v0 = 10930.08 m/s (d) The escape velocity is given by
Evidently, 7728.73 < 8444.44 < 10930.08 ve = 2 v0 = 10930.08 m/s
i.e., v0 < v < ve
5. (b) The centripetal force is provided by the gravitational
So, the satellite moves along an elliptical orbit,
with the position of launch as the perigee. (figure). force of attraction
(b) Let rmax be the maximum distance of the satellite mR 2 = GMmR–5/2
from the earth’s centre, and v' the corresponding
mR 4 2 GMm
velocity.
2
=
From, conservation law of angular momentum and T R5 / 2
energy between perigee and apogee, we have T2 µ R7/2
8444.44 (672 × 104) = v' ( rmax ) 6. (b, c) As per Kepler’s law, T 2 µ R 3 as T decreases
1 gR 2 1 gR 2 GMm
and (8444.44)2 4
v 2
radius decreases, GPE = decreases.
2 672 10 2 rmax R
GRAVITATION 279

9. (b, c, d) Distances of the two satellites from the centre of


k
7. (a, b, d) F (r) = – n earth are r1 = 2R and r2 = 8R respectively. (R =
r earth’s radius). Their potential energies are :
k 1 GmM GmM
U (r ) F (r ) dr . V1 and V2
(n 1) r n 1
r1 r2
If L is the angular momentum of the particle of
V r2 8 R
mass m in an orbit of radius r, then Their ratio is : 1 = = =4
V2 r1 2 R
L2 L2
Kinetic energy = = = K (r ) The kinetic energy of a satellite can be obtained
2I 2mr 2 from relation
Since total energy E (r) = U (r) + K (r)
mv 2 GmM 1 GmM
2 = or K = mv 2 =
k 1 L 2 2 2r
E (r) = . n 1
+ r r
(n 1) r 2mr 2
GmM GmM
The criterion that a circular orbit of radius r0 be Thus, K1 = and K 2 =
stable is that E (r) is minimum. 2r1 2r2
For E (r) to be minimum, 2 conditions must be The ratio of their kinetic energies is
fulfilled.
K1 r2 8 R
2 = = =4
E E K 2 r1 2 R
= 0 and > 0,
r r = r0 r 2 r =r Their total energies are
0
GmM GmM GmM
L2 E1 + =
using both conditions (3 n) >0 r1 2r1 2r1
m
This is possible only when n < 3. GmM GmM GmM
and E2 + =
We also note that inverse square law belongs to r2 2r2 2r2
this category n = –1 also gives stable circular orbits
Their ratio is
[Law of direct distance]. But n = 3 gives circular
orbits which are unstable [Inverse cube law] K1 r2 8 R
= = =4
8. (a,c,d) The gravitational field intensity at the point O is K 2 r1 2 R
zero (as the cavities are symmetrical with respect
to O). Now the force acting on a test mass m0 E1 r2 8R
placed at O is given by = = =4
E2 r1 2 R
F = m0 E = m0 × 0 = 0
Now, y2 + z2 = 36 represents the equation of a 1
circle with centre (0, 0, 0) and radius 6 units the 10. (a, c) mv02
Kinetic energy on the surface of earth =
2
plane of the circle is perpendicular to x-axis. Potential energy on the surface of earth =
Y GMm
R
1 2 GMm
Total energy = mv0
2 R
A B
(-2,0,0)
O (2,0,0)
X 1
4 Kinetic energy at a height h = mv 2
2
GMm
Z Potential energy at this height =
( R + h)
Since the spherical mass distribution behaves as 1 2 GMm
if the whole mass is at its centre (for a point outside Total energy = mv
2 R+h
on the sphere) and since all the points on the
circle is equidistant from the centre of the sphere, By the principle of conservation of energy,
the circle is a gravitational equipotential. 1 2 GMm 1 2 GMm
The same logic holds good for option (d). mv mv0
2 R+h 2 R
280 IIT-JEE PHYSICS Challenger
is constant throughout. Since the force F is
1 2 GM GM
(v0 v 2 ) conservative in nature, therefore mechanical
2 R R+h energy of satellite remains constant. Speed of S is
But GM = gR2 maximum when it is nearest to earth and minimum
when it is farthest.
1 2 gR 2 h
(v0 v2 ) = 13. (a, b) The escape velocity ve of an object from a planet
2 R ( R + h) of radius R is obtained by setting the kinetic
energy of the object near the surface equal to the
2 gRh 2 gh
v02 v2 = = gravitational potential energy difference between
R+h h
1+ the surface and infinity. As the mass of the atoms
R and molecules m in this case is much smaller than
At maximum height v = 0. the mass of the satellite, we don’t need to use the
The initial velocity v0 = (90%)vescape = 0.9 2gR reduced mass of the system in the calculation.
then mve2 GMm
=0
2ghmax 2 R
(0.9 2gR)2 0 = ; 1/ 2
h 2GM
1+ max ve = = 2655 m/s
R R
The speed of a gas molecule or atom follows the
hmax Maxwell-Boltzmann distribution. Their average
0.81 R = h
1 + max speed, as a function of temperature is given by:
R
mv 2 3kT
0.81 R + 0.81 hmax = hmax =
2 2
0.19 hmax = 0.81 R
where k is Boltzmann’s constant and T the
Maximum height reached by the rocket
temperature in Kelvin. The following table gives
0.81R the masses and average velocity for the gases in
hmax = = 4.26 R
0.19 the question.
11. (b, d) Satellite revolve in equatorial plane with same Gas Mas Mass Average
angular velocity as that of earth in magnitude and (molar) (kg) speed
direction.
Ammonia (NH3) 17.031 2.83 × 10–26 364 m/s
Methane (CH4) 16.043 2.66 × 10–26 385 m/s
Helium (He) 4.0026 6.65 × 10–27 770 m/s
e E Hydrogen (H2) 2.016 3.35 × 10–27 1085 m/s
W Hydrogen (H) 1.008 1.67 × 10–27 1534 m/s
s= E s As the Maxwell-Boltzmann distribution has a long
tail in the direction of higher speeds, even though
the average speed is below the escape velocity,
12. (a) Force on satellite is always towards earth, some molecules (atoms) of gas will escape. A rule-
therefore, acceleration of satellite S is always of-thumb among astronomers is that if the average
directed towards centre of the earth. Net torque speed is below 1/6 of the escape velocity, the gas
of this gravitational force F about centre of earth will remain for the age of the solar system.
is zero. Therefore, angular momentum (both in So ammonia and methane will remain on Titan,
magnitude and direction) of S about centre of earth helium would for a shorter time but hydrogen
(molecular or atomic) will escape more quickly.

1. A-q; B-r; C-s; D-p In elliptical orbit as distance from focus changes, speed
2. A-s; B-p; C-r; D-q changes, hence KE, while in circular orbit speed remains
3. A-q, t; B-p, q, t; C-r; D-r, s same hence KE.
Gravitational force due to planet or satellite is a central force,
hence, torque = 0 Angular momentum conservation 2GM
Escape velocity = ,
Kepler’s 2nd law Constant areal velocity. R
where M = Mass of planet
GRAVITATION 281

5. A-p, t; B-r, t; C-p, q, t; D-r, t


GM
Orbital velocity = Due to rotation
R
4. A-q, r; B-t; C-q, s; D-p geff = g0 – 2R cos2
(A) Kinetic energy of a body projected from the surface of g changes with (altitude angle)
earth at large distance may be zero (body momentarily g0 depends on earth mass
comes to rest) or positive.
on earth = 0 Angular momentum constant
(B) Gravitational potential energy of a bound system must
be negative. Escape velocity is independent on direction of projection
(C) Change in potential energy of a point mass if left free but depends on earth mass and distance from earth centre.
to itself, with time may be zero (point mass on horizon-
tal surface) or negative (point mass falling freely GM
Gravitational potential, V .
vertically). r
(D) Change in areal velocity of earth as earth moves from
At particular point it is constant but depends on mass.
apogee towards perigee is zero as areal velocity
remains constant according to Kepler’s second law.

1. 4 3. 65
When the earth’s motion is suddenly stopped, it would fall
GM
g= ; If R decreases then g increases. Taking logarithm into the sun and (suppose) it comes back. If the effect of
R2 temperature of sun is ignored, we can say that the earth
of both the sides; would continue to move along a strongly extended flat ellipse
log g = log G + log M – 2 log R whose extreme points are located at the earth’s orbit and at
dg 2 dR the centre of the sun.
Differentiating it we get =0+0– ; The semi major axis of such ellipse is R/2.
g R
3
T 2 R 1
dg
= 2
2 4 Now =
÷= T 2
2 R3
g 100 100
Where T is the time period of normal orbit of earth.

dg 4 T2
% increase in g 100 = 100 = 4% or T 2
=
g 100 8
2. 4 T
or T =
2 3 2 2
TA RA
= (Kepler law);
TB RB ÷ Now, time required to fall into the sun,
T T 365
2 3 t= = = 65 days
8 TB RA 2 4 2 4 2
=
TB RB ÷
4. 7920
3
RA v
or 64 =
RB ÷ A m
3 B h
RA
or (4) 3 =
RB ÷ M R
E
RA
or 4=
RB
RA = 4 R B
282 IIT-JEE PHYSICS Challenger
For the satellite in the circular orbit, we have 5. 99.5
Total energy at A = Total energy at B
mvs 2 GMm
= (K.E.)A + (P.E.)A = (P.E.)B
( R + h ) ( R + h )2
2
1 2GM GMm 99 R GMm
GM = m 3R 2 ÷
vs 2
= 2 R 2R 3 100 R+h
R+h

2
1 2GM GM 1 B
÷ = vs = ve ÷
2 R R+h 2 h

GM GM Ve
= R
2R R+h 100 A 100
99R
R + h = 2R R
100
h=R
When the satellite is stopped, its kinetic energy is zero.
When it falls freely on to the Earth, its potential energy
decreases and converts into kinetic energy.
(P.E.)A – (P.E.)B = K.E.
On solving, we get h = 99.5 R.
GMm GMm 1 2
÷ = mv The maximum height attained by the body from the
2R R 2 surface of the moon is 99.5 times the radius of the moon.

GM
v= = gR = 9.8 6.4 106
R
= 7920 m/s
1. Two solid spherical balls of radii r1 and r2 (< r1) and of 3. A container filled with viscous liquid is moving vertically
density are tied up with a long string and released in a downwards with constant speed 3v0. At the instant shown,
viscous liquid column of lesser density with the string a sphere of radius r is moving vertically downwards (in liquid)
just taut as shown. The tension in the string when terminal has speed v0. The coefficient of viscosity is . There is no
velocity is attained, is relative motion between the liquid and the container. Then
at the shown instant, the magnitude of viscous force acting
on sphere is
r1

r2 3v 0

4 r24 – r14
(a) ÷ ( – )g
3 r2 – r1 (a) 6 rv0 (b) 12 rv0
(c) 18 rv0 (d) 24 rv0
(b)
2
3
(r
3
2 )
– r13 ( – ) g
4. A soap bubble of radius R is surrounded by another soap
bubble of radius 2R, as shown. Take surface tension = S.
Then the pressure inside the smaller soap bubble, in excess
(c)
4
3
(r
3
2 )
– r13 ( – ) g of the atmospheric pressure, will be

4 r24 – r14
(d) ÷ ( – )g
3 r2 + r1

2. Drops of liquid of density are floating half immersed in a


liquid of density . If the surface tension of liquid is T, the
radius of the drop will be Atmosphere

3T 6T (a) 4S/R (b) 3S/R


(a) (b) (c) 6S/R (d) None of these
g(3 ) g(2 )
5. A steel wire is 4.0m long and 2mm in diameter. How much is
it elongated by a suspended body of mass 20 kg ? Young’s
3T 3T
(c) (d) modulus for steel is 1,96,000 Mpa.
g(2 ) g(4 3 )
(a) 1.273 mm (b) 1.73 mm
(c) 0.123 mm (d) 2.124 mm

MARK YOUR
1. 2. 3. 4. 5.
RESPONSE
284 IIT-JEE PHYSICS Challenger
6. Water is flowing on a horizontal fixed surface, such that its 11. A platform is suspended by four wires at its corners. The
flow velocity varies with y (vertical direction) as wires are 3m long and have a diameter of 2.0mm. Young’s
modulus for the material of the wires is 1,80,000 MPa. How
2 y2 y3 far will the platform drop (due to elongation of the wires) if
v k ÷ . If coefficient of viscosity for water is ,
a2 a3 a 50 kg load is placed at the centre of the platform ?
what will be shear stress between layers of water at y =a. (a) 0.25 mm (b) 0.65 mm
(c) 1.65 mm (d) 0.35 mm
k 12. Two spherical bubbles are in contact with each other
(a) (b)
a ka internally as shown. The radius of curvature of the common
a surface is R, then
(c) (d) None of these
k
7. A rod of length 1000mm and coefficient of linear expansion
= 10–4 per degree is placed symmetrically between fixed R1
R2 Common
walls separated by 1001 mm. The Young’s modulus of the
rod is 1011 N/m2. If the temperature is increased by 20°C, surface
then the stress developed in the rod is (in N/m2)
//////////////////////////////////////

(a) R > R1 (b) R1 > R > R2


1000mm (c) R < R2 (d) R = R1
13. The force exerted by a special compression device is given
as function of compression x as Fx (x) = kx (x – ) for
0 x , where is maximum possible compression and k is
a constant. The force exerted by the device under
compression is maximum when compression is –
(a) 0 (b) /4
1001mm
(a) 10 (b) 108 (c) / 2 (d) /2
(c) 2 × 108 (d) cannot be calculated 14. A steel ball of diameter d = 3.0mm starts sinking with zero
8. A large open tank has two holes in the wall. One is a square initial velocity in olive oil whose viscosity is = 0.90 P. How
hole of side L at a depth y from the top and the other is a soon after the beginning of motion will the velocity of the
circular hole of radius R at a depth 4 y from the top. When ball differ from the steady state velocity by
the tank is completely filled with water, the quantities of n = 1.0%? Density of steel = 7.8 × 10 3 kg/m3.
water flowing out per second from both holes are the same. (a) 0.2 sec. (b) 0.8 sec.
Then, R is equal to (c) 0.6 sec. (d) 1.2 sec.
L L 15. The diagram shows bimetallic strip used as a thermostat in
(a) (b) 2 L (c) L (d) a circuit. The copper expands more than the invar for the
2 2
same temperature rise.
9. A rain drop of radius 0.3mm falling vertically downwards
in air has a terminal velocity of 1m/s. The viscosity of air signal lamp
is 18 × 10 –5 poise. The viscous force on the drop is
(a) 101.73 × 10–4 dyne (b) 101.73 × 10–5 dyne invar electric bell
–5
(c) 16.95 × 10 dyne (d) 16.95 × 10–4 dyne copper
10. A vertical capillary tube with inside diameter 0.5mm is motor
M
submerged into water so that the length of its part protruding
over the surface of water is equal to 2.5mm. Find the radius Which will be switched on when the bimetallic strip becomes
of curvature of the meniscus. hot ?
(a) 0.3mm (b) 0.6mm (a) bell only (b) lamp and bell only
(c) 0.9mm (d) 1.2mm (c) motor and bell only (d) lamp, bell and motor

MARK YOUR 6. 7. 8. 9. 10.


RESPONSE 11. 12. 13. 14. 15.
PROPERTIES OF MATTER & FLUID MECHANICS 285

16. What is the minimum diameter of a brass rod if it is to support


ln (r / R2 ) v0 ln ( r / R2 )
a 400N load without exceeding the elastic limit ? Assume (a) 2v0 (b)
ln ( R1 / R2 ) 2 ln ( R1 / R2 )
that the stress for the elastic limit is 379 MPa.
(a) 1.16mm (b) 2.32mm ln ( r / R2 ) 2v0 ln (r / R2 )
(c) v0 (d)
(c) 0.16mm (d) 1.35mm ln ( R1 / R2 ) 3 ln ( R1 / R2 )
17. A wooden block, with a coin placed on its top, floats in 21. Following are some statements about buoyant force on a
water as shown in figure. The distance and h are shown body of certain shape, immersed completely inside ideal
here. After some time the coin falls into the water. Then fluid. Select the correct statement(s) (Liquid is of uniform
density)
(a) Buoyant force depends upon orientation of the
concerned body inside the liquid
(b) Buoyant force depends upon the density of the body
immersed
(c) Buoyant force depends on the fact whether the system
is on moon or on the earth
(d) Buoyant force depends upon the depth of the body
inside the liquid
(a) decreases and h increases
22. Water is filled in a container upto height 3m. A small hole
(b) increases and h decreases of area ‘a’ is punched in the wall of the container at a
(c) both and h increase (d) both and h decrease height 52.5 cm from the bottom. The cross sectional area
18. A conical glass capillary tube A of length 0.1m has diameters of the container is A. If a/A = 0.1 then v2 is (where v is the
10–3 and 5 × 10–4m at the ends. When it is just immersed in velocity of water coming out of the hole)
a liquid at 0°C with larger diameter in contact with it, the (a) 50 (b) 51
liquid rises to 8 × 10–2m in the tube. If another cylindrical (c) 48 (d) 51.5
glass capillary tube B is immersed in the same liquid at 0°C, 23. A capillary tube with inner cross-section in the form of a
the liquid rises to 6 × 10–2m height. The rise of liquid in the square of side ‘a’ is dipped vertically in a liquid of density
and surface tension which wet the surface of capillary
tube B is only 5.5 × 10–2m when the liquid is at 50°C. Find
tube with angle of contact . The approximate height to
the rate at which the surface tension changes with
which liquid will be raised in the tube is (Neglect the effect
temperature considering the change to be linear. (The density
of surface tension at the corners capillary tube)
of the liquid is (1/14) × 104 kg/m3 and angle of contact is
zero. Effect of temperature on density of liquid and glass is 2 cos 4 cos
(a) (b)
negligible). a g a g
(a) – 1.4 × 10–4 N/m°C (b) – 2.4 × 10–4 N/m°C
–6
(c) – 4.4 × 10 N/m°C (d) – 5.2 × 10–8 N/m°C 8 cos
(c) (d) None of these
19. Four uniform wires of the same material are stretched by the a g
same force. The dimensions of wire are as given below. The 24. A U-tube whose ends are open and whose limbs are vertical
one which has the minimum elongation has – contains oil of specific gravity 0.8 and surface tension 28
(a) radius 3mm, length 3m dyne/cm. If one limb has a diameter of 2.2mm and the other
(b) radius 0.5mm, length 0.5m of 0.8mm, what is the difference in level of the oil in the two
(c) radius 2mm, length 2m limbs ?
(d) radius 3mm, length 2m (a) 22.8 mm (b) 6.2 mm
20. A long cylinder of radius R1 is displaced along its axis with (c) 15.2 mm (d) 11.4 mm
a constant velocity v0 inside a stationary co-axial cylinder 25. The pressure in an explosion chamber is 345 MPa. What
of radius R2. The space between the cylinders is filled with would be the percent change in volume of a piece of copper
viscous liquid. Find the velocity of the liquid as a function subjected to this pressure ? The bulk modulus for copper is
of the distance r from the axis of the cylinders. The flow is 138 Gpa (= 138 × 109 Pa)
laminar. (a) 0.1% (b) 0.5%
(c) 0.25% (d) 0.2%

MARK YOUR 16. 17. 18. 19. 20.


RESPONSE 21. 22. 23. 27. 25.
286 IIT-JEE PHYSICS Challenger
26. Water of density in a clean aquarium forms a meniscus, as 29. A right circular cone of density , floats just immersed with
illustrated in the figure. Calculate the difference in height h its vertex downwards in a vessel containing two liquids of
between the centre and the edge of the meniscus. The densities 1 and 2 respectively, the planes of separation of
surface tension of water is . the two liquids cuts off from the axis of the cone a fraction z
of its length. Find z.

1/3 1/ 3
2 2
(a) ÷ (b)
– ÷
1 2 1 2

h
1/2
2 2
(c) ÷ (d) ÷
1 2 1– 2

30. A hemispherical portion of radius R is removed from the


2 bottom of a cylinder of radius R. The volume of the remaining
(a) (b)
g g cylinder is V and its mass M. It is suspended by a string in
a liquid of density where it stays vertical. The upper surface
1 of the cylinder is at a depth h below the liquid surface. The
(c) (d) 2
2 g g force on the bottom of the cylinder by the liquid is

27. A cube of mass m = 3.2 kg floats on the surface of water.


Water wets it completely. The cube is 0.2m on each edge by
what additional distance is it buoyed up or down by surface
tension ? Surface tension of water = 0.07 Nm–1.
(a) 2.8 × 10–4 m (b) 1.4 × 10–4 m
(c) 3.2 × 10 m–6 (d) 6.1 × 10–2 m h
28. For the arrangement shown in the figure, find the time
interval in seconds after which the water jet ceases to cross
the wall. Area of the cross section of the tank A = 5m 2
and area of the orifice a = 4 cm2. [Assume that the container 2R
remaining fixed]
(a) Mg (b) Mg – V g
(c) Mg + R2h g (d) g(V + R2h)
31. The cross-section of a tank full of water under pressure is
0.81m
shown. Determine the magnitude of the resultant force acting
on the quarter-circle curved surface BC if the tank is 2m
long.

19.62kN/m²
2m
B O
A
/////////////////////////////////////////////////// y
C
x
1m

E D
(a) 1000s (b) 2000s
(c) 1500s (d) 500s (a) 73.43 kN (b) 55.12 kN
(c) 64.12 kN (d) 43.14 kN

MARK YOUR 26. 27. 28. 29. 30.


RESPONSE 31.
PROPERTIES OF MATTER & FLUID MECHANICS 287

32. A water clock (clepsydra) used in ancient Greece is designed


as a vessel with a small orifice O (figure). The time is
determined according to the level of the water in the vessel.
H
What should be equation corresponding to the shape of
the vessel for the time scale to be uniform ?

h h wood
water
(a) (b)
water wood water

h2
(c) h (d)
O R
35. Find the force acting on the blade of an undershot wheel
(figure) if the stream after impinging on the blade continues
to move with the velocity of the blade.
The height of the water head is h, the radius of the wheel is
R, the angular velocity of the wheel is and the cross-
section area of the stream is A.
2 2
v
(a) y = kx4 where k =
2 ga 2

2 2
v
(b) y = kx3 where k =
2 ga 2
h

2 2
v
(c) y = kx4 where k =
ga R

2 3
v
(d) y = kx4 where k =
2 ga 2
33. A cylindrical tumbler, half filled with liquid of density : is
(a) A ( 2 gh R )2 (b) A ( 2 gh R)2
filled up with a liquid of density ' which does not mix with
the former one. Find the ratio of the pressure on the base of (c) A ( 2 gh R) (d) A (2 gh R)
the tumbler to the whole pressure on its curved surface is
36. A pump is designed as a horizontal cylinder with a piston
(h is the height and r the radius of the base of the tumbler)
having an area of A and an outlet orifice having an area of
r( ) r( ) ‘a’ arranged near the cylinder axis. Determine the velocity
(a) (b) of outflow of a liquid from the pump if the piston moves
h( 3 ) 2h ( 3 )
with a constant velocity under the action of a force F. The
r( ) 2r ( ) density of the liquid is . (a << A)
(c) (d)
2h ( 2 ) h( 3 )
2F F
(a) (b)
34. A hollow wooden cylinder of height h, inner radius R and A A
outer radius 2R is placed in a cylindrical container of radius
3R. When water is poured into the container, the minimum F 2F
height H of the container for which cylinder can float inside (c) (d)
2A 3A
freely is

MARK YOUR
32. 33. 34. 35. 36.
RESPONSE
288 IIT-JEE PHYSICS Challenger
37. A tank and a trough are placed on a trolley as shown. Water
issues from the tank through a 5cm. diameter nozzle at 5m/s
and strikes the trough which turns it by 45° as shown. II I
Determine the compression of the spring of stiffnes 20N cm.
w v
B C
u
45° (a) 1.12 (b) 1.1
(c) 1.05 (d) 1.0
///////////////

x 42. A solid sphere floats just immersed in heterogeneous liquid


composed of three liquids which do not mix and whose
/////////////////////////////////////////////////////// D densities are as 1 : 2 : 3. If the thickness of the two upper
A
liquids be each one-third of the diameter of the sphere, then
(a) 1.74cm. (b) 1.12 cm.
(c) 0.78cm. (d) 2.12 cm. choose the correct option
38. A right circular cylinder of sp. gr. floats in water with its (a) the density of the liquid in the middle is equal to the
axis vertical, one third begin above the water. If be the sp. density of the sphere
gr. of air then choose the correct option (b) the density of the liquid in the middle is half of the
(a) 3 = 2 + (b) 3 = 1 + density of the sphere
(c) 2 = 2 + (d) 3 = 2 + 2 (c) the density of the liquid in the middle is double to the
39. In the figure shown, a light container is kept on a horizontal density of the sphere
(d) none of these
Sh
rough surface of coefficient of friction . A very small = 43 . A rectangular gate shown should tip automatically when
V the water rises above a certain level. Determine that level in
hole of area S is made at depth h. Water of volume V is filled terms of h.
in the container. The friction is not sufficient to keep the
container at rest. The acceleration of the container initially is
V h
(a) g
Sh h
1m
(b) g
(c) zero
Sh ////////////////////////////////////
(d) g
V (a) 2.123m (b) 1.212m
40. A cylinder of 1m diameter and 2m length stays in equilibrium (c) 1.512m (d) 1.732m
as shown. Calculate the specific gravity of the material of 44. An incompressible liquid flows through a horizontal tube as
the cylinder. shown in the figure.
s
m/
OIL (0.7)
A
=1.5
B O D v2
////////////////

//////////////

C
mete
OIL (0.8) A
A meter² v1=3.0 m/s
1.5
(a) 0.197 (b) 0.797 Am
ete
(c) 0.517 (d) 0.921 r²
41. A U-tube of uniform cross section (see Fig) is partially filled v
with a liquid I. Another liquid II which does not mix with
Then the velocity ‘v’ of the fluid is
liquid I is poured into one side. It is found that the liquid
(a) 3.0 m/s (b) 1.5 m/s
levels of the two sides of the tube are the same, while the
level of liquid I has risen by 2 cm. If the specific gravity of (c) 1.0 m/s (d) 2.25 m/s
liquid I is 1.1, the specific gravity of liquid II must be

MARK YOUR 37. 38. 39. 40. 41.


RESPONSE 42. 43. 44.
PROPERTIES OF MATTER & FLUID MECHANICS 289

45. A fireman must reach a window 40m above the ground with
2Mmg 1 Mmg 1
a water jet, from a nozzle 3cm, diameter discharging 30 kg/s. (a) 1÷ (b) 1÷
Assuming the nozzle height of 2m, determine the greatest (2M m) (2M m)
distance from the building where the fireman can stand and
2Mmg 1 2 Mmg 1
still reach the jet into the window. (c) + 1÷ (d) 1÷
(a) 142m. (b) 102m. (2M m) (M m)
(c) 182m. (d) 120m. 49. A hollow weightless hemisphere, filled with liquid is
46. A broad vessel with water stands on a smooth surface. The suspended freely from a point in the rim of its base, Find the
level of the water in the vessel is h. The vessel together with ratio of the thrust on the plane base to the weight of the
the water weighs G. The side wall of the vessel has at the contained liquid.
bottom a plugged hole (with rounded edges) with an area A.
At what coefficient of friction between the bottom and the (a) 12 : 73 (b) 12 : 27
surface will the vessel begin to move if the plug is removed ? (c) 6 : 73 (d) 8 : 73
ghA 2 ghA 50. An iceberg is floating in ocean. What fraction of its volume
(a) (b)
G G is above the water ? (Given : density of ice = 900kg/m3 and
ghA 2 ghA density of ocean water = 1030 kg/m3)
(c) (d)
2G 3G
47. A hydraulic jack consists of a handle cum lever of 30cm. 90 13
(a) (b)
length and an assembly of given dimensions. In order that a 103 103
load of 20 kN be supported by the jack, what should be the
10 1
force exerted on the handle ? The distance between the (c) (d)
103 103
fulcrum of the lever and the point where the plunger is
connected is 25mm. 51. A multitube manometer is employed to determine the pressure
200kN in a pipe. For the levels in the manometers as shown, compute
the pressure in the pipe. What would be the length of a
single mercury filled U-tube to record this pressure?

50mm
/////////////////////////////////////////////////////////////////////

Water Sp Gr = 1.0
/////////////////////////////////////////////////////////

300mm Sp Gr = 0.00012
25mm
F
AIR
Lever Handle
////////////////////////////////////

///////////////////////////////////////////////

40cm.

15mm S S4 S3

50cm.

S2 S1

Fluid

//////////////////////////////////////////////////////////////////////

(a) 2000 N (b) 2500 N


Mercury
(c) 700 N (d) 1500 N Sp Gr = 13.6

48. A cylindrical bucket with water in it balances a mass M over


a pulley. A piece of cork, or mass m and sp. gr. , is then tied (a) 193cm (b) 165cm
to the bottom of the bucket with the help of string so as to (c) 121cm (d) 153cm
be totally immersed. Find the tension in the string

MARK YOUR 45. 46. 47. 48. 49.


RESPONSE 50. 51.
290 IIT-JEE PHYSICS Challenger
52. Find the force exerted by the nozzle on the fireman for the 55. Two spheres of volume 250cc each but of relative densities
configuration and data in the figure. 0.8 and 1.2 are connected by a string and the combination is
x immersed in a liquid in vertical position as shown in figure.
The tension in the string is (g = 10 m/s2)
d m. Relative
2c
density 0.8

R 2
²
m
g/c c
a 7k e
ug Relative
Ga
density 1.2
1

(a) 5.0N (b) 0.5N


b (c) 1.0N (d) 2.0N
(a) 754 N (b) 1554 N 56. A solid hemisphere of radius a and weight W is floating in
(c) 1254 N (d) 1712 N liquid, and at a point on the base at a distance c from the
53. A vessel in the shape of a hollow hemisphere surmounted centre rests a weight w, the tangent of the inclination of the
by a cone is held with the axis vertical and vertex uppermost. axis of the hemisphere to the vertical for the corresponding
If it be filled with a liquid so as to submerge half the axis of position of equilibrium, assuming the base of the hemisphere
the cone in the liquid, and the height of the cone be double to be entirely out of the fluid, is –
the radius of its base, the resultant downward thrust of the 4c w 2c w
liquid on the vessel is x times the weight of the liquid that (a) (b)
3aW 3aW
the hemisphere can hold. Find the value of x.
(a) 15/8 (b) 1/8 8c w 8c w
(c) (d)
(c) 5/8 (d) 15/2 3aW 5aW
54. A homogeneous solid cylinder of length L (L < H/2), cross- 57. A closed tube in the form of an equilateral triangle contains
sectional area A/5 is immersed such that it floats with its axis equal volumes of three liquids which do not mix and is placed
vertical at the liquid-liquid interface with length L/4 in the with its lowest side horizontal. If the densities of the liquids
denser liquid as shown in the figure. The lower density are in A.P., their surface of separation will be at the points of
liquid is open to atmosphere having pressure P0. Then ........ of the sides of the triangle.
density D of solid is given by (a) 1/2 (b) 1/4
(c) 1/3 (d) 1/6
58. A cone full of water, is placed on its side on a horizontal
table, the thrust on its base is x times the weight of the
contained fluid, where 2 is the vertical angle of the cone.
Find the value of x.
3L/4 d (a) 3 cos (b) 3 sin
h (c) 2 sin (d) 2 cos
L/4
59. A balloon of volume V, contains a gas whose density is to
2d that of the air at the earth’s surface as 1 : 15. If the envelope
of the balloon be of weight w but of negligible volume, find
the acceleration with which it will begin to ascend.
5 4 7Vg w 2Vg w
(a) d (b) d (a) g (b) g
4 5 Vg w÷ Vg w÷
d
(c) 4d (d) 14Vg w 14Vg + w
(c) g (d) g
w ÷ w ÷
5
Vg Vg

MARK YOUR 52. 53. 54. 55. 56.


RESPONSE 57. 58. 59.
PROPERTIES OF MATTER & FLUID MECHANICS 291

60. An non-homogeneous small sphere having average density (a) 2.75 mm (b) 3.75 mm
same as that of the liquid. It is released from rest, in the (c) 5.25 mm (d) 6.50 mm
position as shown in figure. C being its centre of mass and 64. If the ratio of lengths, radii and Young’s moduli of steel and
O being the centre of sphere. brass wires in the figure are a, b and c respectively, then the
corresponding ratio of increase in their lengths is

//////////////////
steel
M
C brass
O Liquid
2M

(a) 2a2cb (b) 3a/2b2c


(a) O moves up (b) O moves down
(c) C moves left (d) None of these (c) 2ac/b2 (d) 3c/2ab2
2
65. A boat with base area 8m floating on the surface of a still
61. A sealed glass bulb containing mercury (incompletely filled)
just floats in water at 4°C. If the water and bulb are river is intended to move with a constant speed of 2 m/s by
(i) cooled to 2°C and (ii) warmed to 8°C, the bulb the application of a horizontal force. If the river bed is 2m
(a) (i) sinks and (ii) sinks (b) (i) sinks and (ii) floats deep find the force needed, assuming a constant velocity
(c) (i) floats and (ii) floats (d) (i) floats and (ii) sinks gradient. Coefficient of viscosity of water is 0.90 × 10–2 poise.
62. A uniform wooden stick of length L, cross-section area A (a) 720 dyne (b) 620 dyne
and density d is immersed in a liquid of density 4d. A small (c) 520 dyne (d) 360 dyne
body of mass m and negligible volume is attached at the 66. Two bodies of masses 2 kg and 3 kg are connected by a
lower end of the rod so that the stick floats vertically in metal wire of cross-section 0.04 mm2. Breaking stress of
stable equilibrium then metal wire is 2.5 GPa. The maximum force F that can be
applied to 3 kg block so that wire does not break is
d
(Neglect friction)
0.04 mm² F
2kg 3kg
4d ///////////////////////////////////////////////////
(a) 100 N (b) 150 N
m (c) 200 N (d) 250 N
(a) m > dAL (b) m < dAL 67. The lower end of a capillary tube of radius 2.00mm is dipped
(c) m < dAL/2 (d) m < dAL/4 10.00cm below the surface of water in a beaker. Calculate the
63. A load of 10kN is supported from a pulley which in turn is pressure within a bubble blown at its end in water, in excess
supported by a rope of sectional area 1 × 103 mm2 and of atmospheric pressure.
modulus of elasticity 103 N mm–2, as shown in figure. [Surface tension of water 72 × 10–3 N/m]
Neglecting the friction at the pulley, determine the deflection (a) 718 Nm–2 (b) 912 Nm–2
of the load. (c) 1160 Nm –2 (d) 1052 Nm–2
/////////// 68. The ice storm in the province of Jammu strained many wires
to the breaking point. In a particular situation, the
/////////// transmission pylons are separated by 500 m of wire. The top
grounding wire is 15° from the horizontal at the pylons, and
600mm 900mm
has a diameter of 1.5 cm. The steel wire has a density of 7860
kg/m3. When ice (density 900 kg/m3) built up on the wire to
a total diameter of 10.0 cm, the wire snapped. What was the
breaking stress (force/unit area) in N/m2 in the wire at the
breaking point? You may assume the ice has no strength.
(a) 7.4 × 107 N/m2 (b) 4.5 × 108 N/m2
(c) 2.6 × 10 N/m6 2 (d) 1.15 × 107 N/m2
10 kN

MARK YOUR 60. 61. 62. 63. 64.


RESPONSE 65. 66. 67. 68.
292 IIT-JEE PHYSICS Challenger
69. Two identical soap bubbles, each of radius x, coalesce to zero contact angle. Calculate the difference in the level of
form a bubble of radius y. If P be the atmospheric pressure, the mencius caused by the capillarity.
and assuming that the process is isothermal, what is the
(a) 0.68mm (b) 0.12mm
surface tension of soap solution ?
(c) 0.32mm (d) 0.92mm
P [2 x + y ]
3 3
P [2 x 3 3
y ] 71. An iron bar of length L cross section A and Young modulus
(a) (b)
4 (y 2 2
2x ) 4 ( y + 2 x2 )
2 Y is pulled by a force F, from both the ends so as to produce
an elongation e. The elastic energy stored in the deformed
rod will be
P [2 x3 y3 ] P [ x3 y3 ]
(c) (d)
4 ( y2 2 x2 ) 4 ( y2 2 x2 ) F 2L F 2L
(a) (b)
AY 2 AY
70. A U-tube is made up of two capillaries of diameters 1.0mm
F2
and 1.5mm respectively. The U-tube is kept vertically and (c) (d) None of these
partially filled with water of surface tension 0.0075 kg/m and 2 A 2Y

MARK YOUR
69. 70. 71.
RESPONSE

1. Let v1, v2 and v3 be the speed of efflux of water from the


PASSAGE-1
three orifices then
(a) v1= v3 < v2 (b) v1 < v2 < v3
Water is filled in a cylinder vessel upto a height 4h. Three small
(c) v1= v3 > v2 (d) v1 > v2 > v3
orifices O1, O2 and O3 are made on the wall of the vessel.
2. Let x1 be the horizontal distance of the point at which the
h water stream from orifice I strikes the horizontal surface.
O1
The distance x1 is
h
O2 (a) h (b) 2h
h
(c) 3 2h (d) 2 3h
O3
h

MARK YOUR
1. 2.
RESPONSE
PROPERTIES OF MATTER & FLUID MECHANICS 293

3. The correct diagram showing the trajectories is 4. The cube will


(a) remain in cubical shape and fly up
(b) remain in cubical shape and move down
O1
(c) become flat and water will stay on
O2 (d) become spherical
(a) 5. The surface energy of liquid thus formed will be equal to
O3 (a) (6) (a2) (T) (b) (5) (a3/2) (T)
2/3 2
(c) (3 ) (T.a ) (4 ) 1/3 (d) (12) (a2) (T)
6. Now if this melted liquid is dropped in air, in gravity field
and viscosity of liquid is , then
(a) liquid will move down with a constant speed
O1 2/3
2 a2 ( )g 3
÷ .
O2 9 4
(b) (b) liquid will remain at rest
O3
72 a 2 ( )g
(c) liquid will move up with speed ÷
9
(d) None of above

O1
PASSAGE-3
O2
(c) A tube of length and radius R carries a steady flow of fluid
O3 whose density is and viscosity . The velocity v of flow is given

R2 r2
by v = v0 ÷ , where r is the distance of the flowing fluid
R2
O1 from the axis.
7. Volume of fluid, flowing across the section of the tube, in
O2 unit time is
(d)
O3 v0 R 2
(a) v0 R 2 (b)
2

v0 R 2 v0 R 2
(c) (d)
PASSAGE-2 3 4
8. Kinetic energy of the fluid within the volume of the tube is
The property of a surface of liquid to shrink whenever it gets a
v02 R 2 v02 R 2
chance to do so is because of surface tension. The energy that a (a) (b)
surface layer has is called as surface energy and is given by product 2 4
of surface tension and surface area. Consider a solid cube of edge
‘a’ made of ice. The cube is kept in a gravity free cabin. Now the v02 R 2 v02 R 2
(c) (d)
whole cube is melted. Surface tension of water = T. Viscosity of 6 3
air = . 9. The frictional force exerted on the tube by the fluid is
(a) 4 v0 (b) v0
a (c) 2 v0 (d) 3 v0

MARK YOUR 3. 4. 5. 6. 7.
RESPONSE 8. 9.
294 IIT-JEE PHYSICS Challenger
15. Force exerted by external agent on the piston will
PASSAGE-4
(a) continuously increase
(b) continuously decrease
A tank of base area 4m2 is initially filled with water up to height (c) remain constant
2m. An object of uniform cross-section 2m2 and height 1m is (d) first increase then decrease
now suspended by wire into the tank, keeping distance between
base of tank and that of object 1m. Density of the object is 2000
PASSAGE-6
kg/m3. Take atmospheric pressure 1 × 105 N/m2 ; g = 10 m/s2.
Water is filled to a height h in a fixed vertical cylinder placed on
//////// horizontal surface. At time t = 0 a small hole is drilled at a height
h/4 from bottom of cylinder as shown. The cross-section area of
hole is a and the cross-section area of cylinder is A such that
A >> a.

2
A=2m 1m

1m

10. The downwards force exerted by the water on the top surface
of the object is h/4
(a) 2.0 × 105N (b) 2.1 × 105N
5
//////////////////////////////////////////////////////////////
(c) 2.2 × 10 N (d) 2.3 × 105N Horizontal surface
x
11. The tension in the wire is
16. Let the value of horizontal distance of point where the water
(a) 0.1 × 105N (b) 0.2 × 105N fall on horizontal surface from bottom of cylinder be x as
5
(c) 0.3 × 10 N (d) 0.4 × 105N shown. Then from time t = 0 till water comes out of hole, pick
12. The buoyant force on the object is the correct statement
(a) 0.1 × 105N (b) 0.2 × 105N (a) x increases with time
5
(c) 0.3 × 10 N (d) 0.4 × 105N (b) x decreases with time
(c) x first increases and then decreases with time
(d) x first decreases and then increases with time
PASSAGE-5
17. As long as water comes out of hole, the time taken by a
water particle starting from hole to reach the horizontal
A syringe is filled with water. Its volume is 20cm3, and the cross- surface
section of its interior part is 4cm2. The syringe is held vertically (a) increases
such that its nozzle is at its top, and its 100g piston is pressed by (b) decreases
external agent and it moves with a constant speed. The ejected (c) remains constant
water has an initial upward velocity of 2m/s, and the cross-section (d) increases and then decreases
of the beam of water at the nozzle is 1mm2. 18. The duration of time for which water flows out of hole is
(Neglect the dissipated energy due to friction)
A 3h a 3h
(a) (b)
13. Find the speed of the piston a 2g A 2g
(a) 5 mm/s (b) 5 cm/s
(c) 0.5 m/s (d) 0.5 mm/s A 2h
(c) (d) None of these
14. What is the total work done by external agent a 3g
(a) 0.04 J (b) 0.045 J
(c) 0.095 J (d) 4.5 mJ

MARK YOUR 10. 11. 12. 13. 14.


RESPONSE 15. 16. 17. 18.
PROPERTIES OF MATTER & FLUID MECHANICS 295

24. A uniform rod rests in position inclined to the vertical, with


PASSAGE-7
half its length immersed in water, and can turn about a point
A cylinder of radius R is kept embedded along the wall of a dam as
1
shown. Take density of water as . Take length as L. in it at a distance equal to
th of the length of the rod from
6
the extremity below the water. Find the specific gravity of
R rod .
(a) 5/9 (b) 2/9
R (c) 1/8 (d) 1/9

PASSAGE-9

19. The vertical force exerted by water on the cylinder is One way of measuring a person’s body fat content is by
(a) R2Lg (b) R2Lg/2 “weighing” them under water. This works because fat tends to
(c) zero (d) None of these float on water as it is less dense than water. On the other hand
20. The net torque exerted by liquid on the cylinder is muscle and bond tend to sink as they are more dense. Knowing
your “weight” under water as well as your real weight out of
2 R 3 Lg R3 Lg water, the percentage of your body’s volume that is made up of fat
(a) (b)
3 3 can easily be estimated. This is only an estimate since it assumes
that your body is made up of only two substances, fat (low density)
R3 Lg
(c) (d) 0 and everything else (high density). The “weight” is measured by
2
spring balance both inside the outside the water. Quotes are placed
21. The force exerted by liquid on the cylinder in horizontal
around weight to indicate that the measurement read on the scale
direction is [Neglecting atmospheric pressure]
is not your true weight, i.e. the force applied to you body by
(a) 2R2 gL (b) R2 gL
2 gravity, but a measurement of the net downward force on the
(c) 4R gL (d) 1.61 R2 gL
scale.

PASSAGE-8 25. Ram and Shyam are having the same weight when measured
If a body is floating partly immersed in liquid and with a string outside the water. When measured under water, it is found
attached to a point of it. Then to find condition of equilibrium use that weight of Ram is more than that of Shyam, then we can
Fy = 0, Fx = 0, =0 say that
(a) Ram is having more fat content than Shyam
22. A uniform rod capable of turning about one end, which is
(b) Shyam is having more fat content than Ram
out of the water, rests inclined to the vertical with one-third
(c) Ram and Shyam both are having the same fat content
of its length in some water. Find the specific gravity of rod .
(d) None of these
B
26. A person of mass 165kg having one fourth of his volume
V G consisting of fat (relative density 0.4) and rest of the volume
C consisting of everything else (average relative density 4/3)
2a g
is weighed under water by the spring balance. The reading
G1
A shown by the spring balance is
(a) 5/9 (b) 2/9 (a) 15 kg (b) 65 kg
(c) 4/9 (d) 1/9 (c) 150 kg (d) None of these
23. A uniform rod 6 ft. long can move about a fulcrum one foot 27. Suppose that Ram is floating in water with two-third of his
above the surface of the water. In the position of equilibrium volume immersed. Now the system is taken in a lift which is
four feet of the rod is immersed. Find the specific gravity of accelerating upward with acceleration g/3. The new fraction
rod . immersed is
(a) 8/9 (b) 5/9 (a) one-third (b) half
(c) 2/9 (d) 1/9 (c) two-third (d) three-fourth

MARK YOUR 19. 20. 21. 22. 23.


RESPONSE 24. 25. 26. 27.
296 IIT-JEE PHYSICS Challenger
( layer in contact with the tube is stationary)
PASSAGE-10
r2 PR 2
When viscous liquid flows, adjacent layers oppose their relative
v v0 1 2÷ , where v0 =
R 4
motion by applying a viscous force given by
The volume of the liquid per second through the tube,
dv
F A R R
dz r2
Q v.2 r dr v0 1 ÷ 2 r dr
0 0 R2

R
r2 r4
= v0 2 2 4R2 0
When = coefficient of viscosity, A = surface area of adjacent
layers in contact.
R2 R2 v R2 PR 4
dv = v0 2 = 0 =
= velocity gradient 2 4 2 8
dz
Now, a viscous liquid having coefficient of viscosity is flowing This is called Poiseuille's equation
through a fixed tube of length and radius R under a pressure
difference P between the two ends of the tube. 28. Force acting on the tube due to the liquid is .
Now, consider a cylindrical volume of liquid of radius r. Due to (a) v0 (b) 2 v0
steady flow, net force on the liquid in cylindrical volume should (c) 4 v0 (d) 6 v0
be zero. 29. The viscous force on the cylindrical volume of the liquid
dv varies as
2 r P r2 (a) F µ r2 (b) F µ r
dr
(c) F µ 1/r (d) F µ 1/r2
0 R
P 30. The momentum of the liquid confined in the tube is
dv = r dr
2 (a) R2 v0 (b) R2 v0/2
v r 2
(c) 2 R v0 (d) R2 v0/4

MARK YOUR
28. 29. 30.
RESPONSE

1. There are two needles of same length, same mass, made of 2. Statement - 1 : When two soap bubble’s of different radii
same material but having different radii. The lengths are are brought into contact, the common
very large compared to their radii (and radii are not too large) interface of contact bulges into the bubble
such that buoyancy does not come into picture. of larger radii as shown.
Statement - 1 : If one of the needle floats on water due to Statement - 2 : Pressure inside a soap bubble of lesser
surface tension effects, other will also float. radius is more than pressure inside a soap
Statement - 2 : Same material ensures that adhesive forces bubble of larger radius.
are same.

MARK YOUR
1. 2.
RESPONSE
PROPERTIES OF MATTER & FLUID MECHANICS 297

3. Statement 1 : A uniform elastic rod lying on smooth


horizontal surface is pulled by constant
C' C' C'
horizontal force of magnitude F as shown
C A' C
in figure (i). Another identical elastic rod is A B A B
B'

pulled vertically upwards by a constant


vertical force of magnitude F (figure ii). The 6.
extension in both rods will be same.
///////////////////////////////// /////////////////////////////////
(a) (b)
F
Statement - 1 : During storms, when the velocity of the
wind is very high, it tears off the roofs of
buildings. Two cases can be distinguished
F (1) if the roof is fastened more firmly at
/////////////////////////////////////////
Fig. (ii) points A and B than at ridge C, the wind
Fig. (i)
will break the roof along ridge C and raise
both halves up (fig. a) (2) if the roof is
Statement - 2 : In a uniform elastic rod, the extension secured more firmly at the ridge and less
depends only on forces acting at the ends firmly at points A and B, the wind will first
of rod. lift the roof up and then carry it aside
4. Statement - 1 : A wooden cylinder floats horizontally in a (fig. b).
pond of water. By attaching a large cube Statement - 2 : The pressure of the air streaming over the
of ice on one end of cylinder it can be made roof is less than of air at rest. It is this
to float vertically. surplus pressure of the stationary air under
Statement - 2 : To float a cylinder vertically in a liquid its the roof that causes the described
centre of gravity should exist below centre phenomena.
of buoyancy of the submerged part of it. 7. Statement - 1 : A light celluloid ball placed in a stream of
5. Statement - 1 : A ship gets a large hole in its underwater gas or water issuing at a high velocity from
portion (figure). As a result the motion will a tube with a narrow neck freely hover in
begin to retard. this stream (figure).

Statement - 2 : The gas in the stream has a high velocity,


Statement - 2 : The force on the wall is slightly greater the pressure inside the stream is above
than 2pA atmospheric.

MARK YOUR
3. 4. 5. 6. 7.
RESPONSE
298 IIT-JEE PHYSICS Challenger
8. Statement - 1 : The bottom of a broad vessel is provided water in the vessel is greater than in the
with a narrow tube through which the water tube. The velocity increases at the
can flow out of the vessel (figure). A screen boundary between the vessel and tube,
is placed between the vessel and the tube. and the pressure drops. For this reason
If a light ball is submerged to the bottom the ball is pressed against the screen and
of the vessel the water flows out of it, and does not rise.
the ball will not rise to the surface. If the 9. Statement - 1 : In case of motion of an ideal fluid in a
outflow of the water is stopped, the ball horizontal tube, where the area of cross-
will immediately rise to the surface. section is minimum, pressure is maximum.
Statement - 2 : Hydrostatic pressure in different ideal
liquids at points of different depth can be
same.
10. Statement - 1 : When a rod is pulled by a force acting on
one end as shown in diagram then there
will be no longitudinal stress in the rod.
Statement - 2 : The pressure diminishes in a stream of a
F
flowing liquid with an increase in its
velocity. The velocity with which the water Statement - 2 : Longitudinal stress is internal normal force
flows in the vessel is much smaller than in per unit area.
the tube and, therefore, the pressure of the

MARK YOUR
8. 9. 10.
RESPONSE

1. A completely filled closed aquarium is kept on a weighing 3. Two light wires A and B shown in the figure are made of the
machine. It can be assumed that the density of the fish is same material and have radii rA and rB respectively. The
greater than the density of the water. The total mass of the block between them has a mass m.
aquarium and its contents put together is M. If now all the
fish start accelerating upwards with an acceleration a, then
the incorrect option (s) is/are A
(a) the weight recorded will be equal to Mg. m
(b) the weight reading will be less than Mg.
(c) the weight reading will be more than Mg. B
(d) no conclusion can be drawn from the given information.
2. Two wires A and B have equal lengths and are made of the
same material, but the diameter of A is twice that of wire B. F
Then, for a given load When the force F is mg/3, one of the wires breaks
(a) the extension of B will be four times that of A (a) A breaks if rA = rB
(b) the extensions of A and B will be equal (b) A breaks if rA < 2rB
(c) the strain in B is four times that in A (c) either A or B may break if rA = 2rB
(d) the strains in A and B will be equal (d) the length of A and B must be known to predict which
wire will break.

MARK YOUR
1. 2. 3.
RESPONSE
PROPERTIES OF MATTER & FLUID MECHANICS 299

4. Four rods, A, B, C and D of the same length and material but (a) Total torque due to nozzles A and B is 0.355 Nm
of different radii r , r 2, r 3 and 2r respectively are held (b) If held stationary then angular velocity with which it
between two rigid walls. The temperature of all rods is will rotate freely is 9.72 rad/sec.
increased through the same range. If the rods do not bend, (c) If held stationary then angular velocity with which it
then will rotate freely is 6.14 rad/sec.
(a) the stress in the rods A, B, C and D are in the ratio (d) Total torque due to nozzles A and B is 0.0355 Nm
1:2:3:4 9. The spring balance A reads 2 kg with a block m suspended
(b) the forces on them exerted by the wall are in the ratio from it. A balance B reads 5 kg when a beaker with liquid is
1:2:3:4 put on the pan of the balance. The two balances are now so
(c) the energy stored in the rods due to elasticity are in the arranged that the hanging mass is inside the liquid in the
ratio 1 : 2 : 3 : 4 beaker as shown in the figure. In this situation:
(d) the strains produced in the rods are in the ratio
1:2:3:4
5. An upright open U-tube, which can contain 0.5m mercury in A
each limb has its limbs 20cm. apart. This is used as an
accelerometer in a horizontal flight. From this device m
acceleration that can be measured are B
(a) 13 m/sec2 (b) 12 m/sec2
(c) 24 m/sec2 (d) 20 m/sec2
6. Two holes with an area of A = 0.2 cm2 each are drilled one (a) the balance A will read more than 2 kg
above the other in the wall of vessel filled with water. The (b) the balance B will read more than 5 kg
distance between the holes H = 50cm. Every second (c) the balance A will read less than 2 kg and B will read
Q = 140cm3 of water is poured into the vessel. Find the more than 5 kg
point (x, y) where the streams flowing out of the holes
(d) the balance A and B will read 2 kg and 5 kg respectively
intersect.
10. An upright U-tube manometer with its limbs 0.6m high and
(a) x = 120 cm (b) x = 60cm
spaced 0.3m apart contains a liquid to a height of 0.4m in
(c) y = 130cm (d) y = 75cm
each limb. If the U-tube is rotated at 10 radians/second about
7. An open rectangular container 2.5m × 1m base and 2m height, a vertical axis at 0.1m from one limb. Choose the correct
half full with water, is accelerated at 4m/s2 up a 15° incline options
along its length. Choose the correct options
(a) Water will not spill for given acceleration.
(b) At acceleration 5.04 m/s2 the water begin to spill.
10rad/s
(c) At acceleration 5.04 m/s2 the water does not spill.
(d) Water will spill for given acceleration.
z2
8. A lawn sprinkler with two nozzles 0.5 cm. diameter each at
0.4m
20cm. and 15cm. radii is connected across a tap capable of 6
z1
litres/minute discharged. The nozzles discharge water
z min
upwards and outwards from the plane of rotation. Choose
the correct options
30° 0.1m 0.2m
B A
(a) z1 = 0.324 (b) z2 = 0.477
(c) zmin = 0.273 (b) z1 + z2 = 0.8
45° 15cm. 20cm.

MARK YOUR 4. 5. 6. 7. 8.
RESPONSE 9. 10.
300 IIT-JEE PHYSICS Challenger
11. A tank which is open at the top, contains a liquid up to a 13. Two identical straight wires PO and RS each of mass m and
height H. A small hole is made in the side of the tank at a length can move smoothly on a fixed rectangular frame.
distance y below the liquid surface. The liquid emerging Two thin films of a liquid of surface tension T are formed
from the hole lands at a distance x from the tank between each wire and the frame. The two wires are
connected by a massless spring of stiffness k and initially
in natural length position and released then choose the
correct option(s)

y P R
k
H
Q S

2T
x (a) maximum elongation of spring, xm =
k
(a) If y is increased from zero to H, x will first increase and
4T
then decrease (b) maximum elongation of spring, xm =
k
(b) x is maximum for y = H/2
(c) Each wire executes SHM with time period,
(c) the maximum value of x is H m
T0 2
(d) the maximum value of x will depend on the density of 2k
the liquid (d) None of these
12. A vertical bar of uniform section is fixed at both of its ends 14. A solid shaft 100mm in diameter, transmits 120kW power at
and a load W = 5000 N is applied axially at an intermediate 200 rpm. Choose the correct options
section as shown in figure. Choose the correct options.
(Modulus of rigidity is 8 × 1010 N/m., Length = 6m)
//////////////// (a) The maximum intensity of shear stress is 2.92×105 Nm–2
2 (b) The angle of twist is 2°30'
(c) The angle of twist is 1°30'
W 3 (d) The maximum intensity of shear stress is 2.92 × 107 Nm–2

\\\\\\\\\\\\\\\\
(a) Reaction at the top support is 3000 N
(b) Reaction at the bottom support is 2000 N
(c) Reaction at the top support is 1000 N
(d) Reaction at the bottom support is 3000 N

MARK YOUR
11. 12. 13. 14.
RESPONSE
PROPERTIES OF MATTER & FLUID MECHANICS 301

1. Column I Column II
(A) Temperature increases ...... (p) Surface tension decreases of liquids
(B) Temperature increases....... (q) Modulus of increases elasticity of metals
(C) Hooke’s law is related to ........ (r) Viscosity of liquid
(D) ....... is independent of shape and (s) Viscosity of gas
size of substance (liquid, gas or solid)
2. Bucket A contains only water, an identical bucket B contains water, but also contains a solid object in the water. Consider the
following four situations. Which bucket weighs more
Column I Column II
(A) The object floats in bucket B, and the buckets have (p) Bucket A
the same water level.
(B) The object floats in bucket B, and the buckets have (q) Bucket B
the same volume of water.
(C) The object sinks completely in bucket B, and the (r) Both buckets have the same weight
buckets have the same water level.
(D) The object sinks completely in bucket B, and the buckets (s) The answer cannot be determined from the information
have the same volume of water. given.
3. Match the columns correctly.
Column I Column II
(A) Bernoulli’s theorem (p) Elasticity
(B) Stoke’s law (q) Speed of efflux
(C) Torricelli’s theorem (r) Venturimeter
(D) Hooke’s law (s) Viscosity
(t) Conservation of energy
4. Column II depends on physical quantity/law given in column I. Match the column correctly.
Column I Column II
(A) Stoke’s law (p) radius
(B) Terminal velocity (q) density of the material of body
(C) Excess pressure inside mercury drop (r) coefficient of viscosity
(D) Viscous force (s) surface tension
(t) velocity gradient
5. Match the columns correctly.
Column I Column II
(A) With rise in temperature forces that decreases (p) Elastic force
(B) Forces involved in capillary action (q) Force due to surface tension
(C) Water flows in a continuous stream down (r) Frictional force
a vertical pipe whereas it breaks into drops
when falling freely because of
(D) Terminal velocity of rain drop (s) Viscous force
(t) Gravitational force

1. 2. 3. 4. 5.
MARK YOUR
RESPONSE
302 IIT-JEE PHYSICS Challenger

1. A column of mercury of 10 cm length is contained in the 4. A thin tube of uniform cross-section is sealed at both ends.
middle of a narrow horizontal 1m long tube which is closed It lies horizontally, the middle 5 cm containing mercury and
at both the ends. Both the halves of the tube contain air at the two equal end containing air at the same pressure P.
a pressure of 76 cm of mercury. By what distance (in cm) will When the tube is held at an angle of 60° with the vertical
the column of mercury be displaced if the tube is held direction, the length of the air column above and below the
vertically? mercury column are 46cm and 44.5 cm respectively. Calculate
the pressure P in centimeters of mercury. (The temperature
2. A cube of wood supporting 200 gm mass just floats in water.
of the system is kept at 30°C).
When the mass is removed, the cube rises by 2cm. What is
the side of the cube (in cm)? 5. A cubical block of wood has density 1 = 500 kg/m3 and
3. A wooden plank of length 1m and uniform cross-section is side = 30 cm. It is floating in rectangular tank partially
hinged at one end to the bottom of a tank as shown in fig.
filled with water of density 2 = 1000 kg/m and having base
The tank is filled with water upto a height 0.5 m. The specific
gravity of the plank is 0.5. Find the angle (in degree) that the area, A = 45 cm × 60 cm. Calculate the work done (in joule) to
plank makes with the vertical in the equilibrium position. press the block slowly so that it is just immersed in water. (g
(Exclude the case = 0°) = 10 m/s2) (Express your answer in J)

1. 2. 3. 4. 5.

MARK
YOUR
RESPONSE
PROPERTIES OF MATTER & FLUID MECHANICS 303

1 (d) 13 (d) 25 (c) 37 (a) 49 (a) 61 (a)


2 (c) 14 (a) 26 (a) 38 (a) 50 (b) 62 (a)
3 (b) 15 (b) 27 (b) 39 (d) 51 (d) 63 (b)
4 (c) 16 (a) 28 (a) 40 (b) 52 (b) 64 (b)
5 (a) 17 (d) 29 (b). 41 (b) 53 (a) 65 (a)
6 (a) 18 (a) 30 (d) 42 (a) 54 (a) 66 (d)
7 (b) 19 (d) 31 (a) 43 (d) 55 (b) 67 (d).
8 (a) 20 (c) 32 (a) 44 (c) 56 (c) 68 (b)
9 (a) 21 (c) 33 (d) 45 (a) 57 (c) 69 (c)
10 (b) 22 (a) 34 (b) 46 (b) 58 (b) 70 (a)
11 (b) 23 (b) 35 (a). 47 (d) 59 (c) 71 (b)
12 (c). 24 (d) 36 (a) 48 (a) 60 (a)

1 (b) 6 (a) 11 (b) 16 (b) 21 (c) 26 (a)


2 (d) 7 (b) 12 (b) 17 (c) 22 (a) 27 (c)
3 (c) 8 (c) 13 (a) 18 (a) 23 (a) 28 (c)
4 (d) 9 (a) 14 (c) 19 (b) 24 (c) 29 (a)
5 (c) 10 (b) 15 (b) 20 (d) 25 (b) 30 (b)

1 (b) 3 (c) 5 (a) 7 (c) 9 (d)


2 (a) 4 (d) 6 (a) 8 (a) 10 (d)

1 (a, b, d) 4 (b, c) 7 (a, b) 10 (a, b, c, d) 13 (b, c)


2 (a, c) 5 (a, b, c, d) 8 (b, d). 11 (a, b, c) 14 (b, d)
3 (a, b, c) 6 (a, c) 9 (b,c) 12 (a, b)

1. A-p, q, r; B-s; C-q; D-p, q, r, s 2. A-r; B-q; C-q; D-q


3. A-q, r, t; B-s; C-q, t; D-p 4. A-p, r; B-p, q, r; C-p, s; D-r, t
5. A-p, q, r, s; B-q, t; C-q; D-s, t

1 3 2 10 3 45 4 75.4 5 6.75
304 IIT-JEE PHYSICS Challenger

( )(
4 7. (b) The change in length of rod due to increase in
1. (d) r13 r23 – )g 6 (r1 + r2 ) v temperature in absence of walls is
3
= µ T = 1000 × 10–4 × 20mm = 2mm.
4 3 But the rod can expand upto 1001mm only.
and r2 ( – ) g = T + 6 r1v
3 At that temperature its natural length = 1002 mm.
Compression = 1mm
4 (r24 – r14 )( – ) g
T 1
3 (r1 + r2 ) Mechanical stress = Y 1011
1000
2. (c) Balancing the forces acting on the drop, we get
= 108 N / m 2
4 3 1 4 3 3T
r g 2 rT . r r= 8. (a) Equating the rate of flow, we have
3 2 3 (2 )g
3. (b) Relative to liquid, the velocity of sphere is 2v0 upwards. (2 gy ) L2 (2 g 4 y ) R 2
Viscous force on sphere = 6 r 2v0 downwards
[Flow = (area) × (velocity), velocity = 2gx ]
= 12 r v0 downwards
where x = height from top
4S
4. (c) P1 P0 = ......... (1) L2 = 2 R2
2R
4S L
P2 P1 = ......... (2) R=
R 2
9. (a) Viscous force = 6 rv
= 6 × 18 × 10–5 × 0.03 × 100 = 101.73 × 10–4 dyne
P2 10. (b) Let the liquid rise to a height h.

r1
P1
P0
6S
Add (1) and (2), P2 P0 = r
R
5. (a) Let L be the elongation. Then, by Hooke’s law,
F L
=Y where Y is Young’s modulus. r2
A L
1 F mgL
The elongation is L= L= 2T
Y A YA h rg hr =
T=
(20) (9.8) (4.0) 2 g
=
(196 109 ) (0.001) 2 If the tube is of height h1 < h
= 1.273 × 10–3 m = 1.273 mm 2T
h1r1 = hr =
dv g
6. (a) Newton’s law of viscosity, F A
dy 2T 2 0.073 25
r1 = = 25mm = m
2 h1 g 25 1000
F dv 4y 3y 1000 9.8
Stress =
dy ÷
k 2 ÷ 1000
A a a3
2 0.073
4 3 k = = 0.0006m = 0.6mm
At y = a, stress = k ÷= 9.8 25
a a a
PROPERTIES OF MATTER & FLUID MECHANICS 305

16. (a) To find the minimum diameter, and hence minimum


LF
11. (b) L= , where L = 3m, cross-sectional area, we assume that the force F = 400
AY N brings us to the elastic limit. Then from the stress,
A = (1.0 × 10–3 m)2 = 3.14 × 10–6 m2, F/A = 379 × 106 Pa, we get
and since each wire supports one-quarter of the load,
400N
A= 6
= 1.0554 10 6 m 2 .
2 379 10 Pa
(50 kg) (9.8 m / s )
F= = 123 N
4
D2
Then, A =
(3m) (123N) 4
L=
(3.14 10 m 2 ) (1.8 1011 N / m 2 )
6
4A 4 (1.0554 10 6 m 2 )
D2 = = = 1.344 × 10–6
= 65 × 10–5 m or 0.65 mm
12. (c) Pressure difference is largest between atmosphere and
smaller bubbles. Hence radius of curvature (R) is and D 1.344 10 6 m 2 = 1.16 × 10–3 m = 1.16 mm
smallest. 17. (d) decreases as the block moves up. h will also
decreases because when the coin is in water it will
dF
13. (d) = 0 at /2 displace a volume of water, equal to its own volume,
dx whereas when it is on the block it displaces more volume
14. (a) Net force on the ball = W – Fu – Fv than to own volume (because density of coin is greater
W = weight of ball, Fu = upthrust force, than density of water).
Fv = viscous force 18. (a) In tube A, the radius of the capillary tube

dv r2 r1
m W Fu 6 rv r r2 ÷x
dt
r1
(where is coefficient of viscosity)

dv h
m A Bv , Here A = W – Fu, B = 6 r r
dt
r2
(1 n ) vs t
m
dv = dt (vs = terminal velocity)
A Bv
0 0

m A B (1 n ) vs
ln =t At a height of 8cm,
B A
3
At steady state, the net force is zero. 3 (0.5 0.25) 10 2
r 0.5 10 8 10
0.1
A
A – Bvs = 0, vs = = 0.3 × 10–3 m
B Surface tension of liquid at 0°C
m 1
t ln 1 (1 n ) vs
6 r vs hr g 8 10 2 0.3 10 3 104 9.8
T0 = = = 0.084 N/m
2 2 14
4 3
r In an uniform tube B of radius r' at 0°C , the surface
m 3
= ln(n ) ln (0.01)
6 r 6 r r h0 g
tension of liquid T0 =
2
2 (1.5)2 10 6
7.8 103 r h50 g
= 1
ln100 At 50°C, T50 =
9 0.9 10 2

2 2.25 7.8 T50 h50 5.5 10 2 11


= 10 2
ln100 = 0.2 sec. = = =
8.1 T0 h0 6 10 2 12
15. (b) As copper expands more, it will bend and strip comes
11 11
in contact with lamp & bell. T50 = T0 = 0.084 = 0.077 N/m
12 12
306 IIT-JEE PHYSICS Challenger
Hence rate of change of surface tension with
temperature
A
2 2 2.475 = h
T T50 T0 7.7 10 8.4 10
= =
50
= – 1.4 × 10–4 N/m°C
3m
F
19. (d) µ 2 v
r2 y r AD
Only option radius 3mm, length 2m is satisfying the
0.525 m
given condition.
dv
20. (c) Viscous force F = , (2 r )
dr 23. (b) Upward force by capillary tube on top surface of liquid
where = length of the cylinder is fup = 4 a cos .
If liquid is raised to a height h then we use
dr 2
÷ dv 4 a cos = ha2 g or h =
4 cos
r F
a g
R2 0 24. (d) Pressure just below the meniscus of the left and the
dr 2 R2 2
dv ln ÷ = v0 .... (1)
r F R1 F 2T 2T
R1 v0 right limbs are P0 and P0
r1 r2
Suppose velocity of liquid at a distance r is v,
Given T = 28 dynes/cm = 28 × 10–3 N/m
r v = 0.8 × 103 kg/m3
dr 2 r 2
dv ln (v0 v ) .... (2)
r F R1 F 1 1
R1 v0 Hence pressure difference is 2T gh
r1 r2 ÷
R2 2
ln ÷ = v 2 28 103 1 1
r F h 3 3 3÷
0.8 10 9.8 0.4 10 1.1 10
On solving eq. (1) and (2),
= 11.36 × 10–3 m
ln ( R2 / r ) ln ( r / R2 )
v v0 v = v0
ln ( R2 / R1 ) ln ( R1 / R2 ) 2.2mm
0.8mm
21. (c) Buoyant force = Effective weight of the displaced liquid
and weight of displaced liquid depends on the h
acceleration due to gravity which is different for
different planets.
22. (a) The square of the velocity of efflux

2 gh
v2 =
2
a
1 ÷
A
or difference in level of the oil in the two limbs,
h = 3 – 0.525
h = 11.4mm.
2 10 2.475 p
or, v2 = = 50 m2/s2 25. (c) The bulk modulus is defined as B = , where
2
1 (0.1) V /V
the minus sign is inserted because V is negative when
p is positive.
V p 345 106
100 = 100 = 100 = 0.25%
V B 138 109
PROPERTIES OF MATTER & FLUID MECHANICS 307

26. (a)
A O B

(h–z)
h
2
p0 – g h h
p0 h
2
C O' D

z 1

V
Let VO' = z then OO' = h – z
The weight of the cone = (vol. of the cone) g
1 3 2
p0 h h tan = g
p0 – g h h 3
2
Volume of liquid (of density 1) displaced
1 3 2
Balancing forces in horizontal direction = volume of cone (VCD) = z tan
3
2 and volume of liquid of density 2 displaced = volume of
h
p0 g ÷ h = p0 h h=
2 g 1 3 2 1 3 2
the frustum ABDC = h tan z tan ÷
27. (b) If surface tension is neglected, by the law of floatation 3 3
mg = (a2x ) g For equilibrium,
where a is the side of cube, is the density of water weight of the cone = (weight of liquid of density 1
and m is the mass of cube. displaced) + (weight of liquid of density 2 displaced)
3.2 g = 0.22 × (103)g 1 3 1 3
The height to which the cube is immersed or h tan 2 g z tan 2 1g
3 3
3.2
x= = 0.08m. 1
(0.2) 2 103 ( h3 z 3 ) tan 2 . 2g
Since water wets the cube, the angle of contact is zero 3
and the force of surface tension acts vertically or h3 z3 ( h3 z3 )
1 2
downwards. So it is buoyed down by surface tension
3 3
mg + 4aT = a2x' g, where T is surface tension of water or h ( 2) z ( 1 2)
3.2g + 4 (0.2) (0.07) = (0.2)2 x' (103) g
1/ 3
4 (0.2) (0.07) or z = h 2
.
x = 0.08 + = (0.08 + 1.4 × 10–4) m ÷
(0.2) 2 (103 ) (9.8) 1 2

The additional distance it is buoyed down by surface 30. (d) According to Archimedes principle
tension = 1.4 × 10–4 m Upthrust = Wt. of fluid displaced
1
28. (a) v = 2 gx = =
t t
P1 h
1 2
For vertical motion x = gt
2
x = 0.25m (i.e. level goes down from 0.81m to 0.25m.) V
Using equation of continuity
dx
5. 2 gx .4 10 4
dt
Fbottom – Ftop = V g
On solving t = 1000s
Fbottom = Ftop + V g
29. (b) VAB is the given cone. Let its height be h and semi-
= P1 × A + V g
vertical angle . Let the base AB of the cone be in the
= (h g) × ( R2) + V g
surface. CD is the surface of separation of two liquids,
= g [ R2 h + V]
O and O' are the centres of the base AB and surface of
separation CD.
308 IIT-JEE PHYSICS Challenger
31. (a) The horizontal force on the curved surface BC equals If we assume that the vessel is axially symmetrical, then
the force on the projected area normal to x-axis, i.e. A = x2, where x is the horizontal coordinate of the
OC × 2 = 1 × 2 = 2m2
x2 a
The depth of the centroid of this projected area is 0.5m vessel wall. Therefore, = = const.
below the top edge of water. But the pressure of 19.62 2 gy v
kN/m2 is equivalent to a head of since in conformity with the initial condition, the water
19.62 1000 level should lower with a constant velocity. Hence, the
= 2m of water shape of the vessel can be determined from the equation
1000 9.81
y = kx4.
which makes z to be (0.5 + 2), i.e., 2.5m.
2 2
The horizontal force is, therefore, v
where k =
Fx = 1000 × 9.81 × 2.5 × 2 = 49050 N = 49.05 kN 2 ga 2
The vertical force on the curved surface BC equals the
weight of the liquid virtually supported by it. 33. (d) Pressure at C.G. of the base
1 1
B O = g . h g. h ....... (1)
2 2
48.1°

49.05kN
P
C h/2

73.43kN
54.65kN
Fz = ( × 12/4 + 1 × 2) × 2 × 1000 × 9.81
= (0.785 + 2) × 19620 = 54649.5 N
h/2
= 54.65 kN upwards
The second term in the parenthesis represents the
column of water equivalent to the pressure of 19.62
kN/m2 acting on the water.
r
The resultant force on the curved surface is
Pressure on the base of the cylinder
F = Fx2 + Fz2 = 49.052 + 54.652 = 73.43 kN = (pressure at the C.G. of the base) × area of the base
indicated at = tan–1 (54.65/49.05) 1 1 2 1 2
= tan–1 (1.114) = 48.1° with the horizontal = g . h g . h÷ . r = r ( ) gh
2 2 2
The resultant force must pass through the centre O
The whole pressure on the curved surface of the
since this is a circular curved surface.
cylinder = the whole pressure on the upper half + the
32. (a) Let us introduce the coordinate system depicted in
whole pressure on the lower half of the curved surface.
figure. According to Torricelli’s formula, the outflow
The whole pressure on the upper half of the curved
velocity of a liquid is V = 2 gy , where y is the surface
thickness of the water layer in the upper vessel. Since = (pressure at the C.G. of the upper half) × area of the
water is incompressible, aV = Av, where v is the velocity upper half
with which the upper layer on the water lowers, A is its 1 1 1
area, and a is the area of the orifice. = g . h 2 r . h÷ rh 2 g .... (2)
4 2 4
y
The whole pressure on the lower half of the curved
surface = (pressure at the C.G. of the lower half) × area
of this lower half
1 1 1
g . h g . h 2 r . h÷
2 4 2
x
1
rh2 (2 )g .... (3)
4
From (2) and (3), the whole pressure on the curved
surface
PROPERTIES OF MATTER & FLUID MECHANICS 309

The outflow velocity of the liquid v can be found from


1 1
= rh 2 g rh 2 (2 )g the equation Au = av. The change in the kinetic energy
4 4
of the liquid during the time t is
1
rh 2 (3 )g ... (4) v2 u2

4 Aut
2
From (1) and (4),
This change should be equal to the work performed by
The whole pressure on the base
the force F.
The whole pressure on the curved surface
v2 u2

1 F ut = Aut
rh 2 ( ) gh 2
2 2r ( )
= = Upon eliminating u, we find that
1 h(3 )
rh2 (3 )g
4 2F 1
v2 .
A a2
1
34. (b) 2R R A2

2F
If a << A, then v =
A
37. (a) Let the control volume be ABCD. The only external
force acting on it is the spring reaction Rx and the mass
the momentum leave normal to CD.

The mass flow rate m = 1000 × 0.052 × 5


4
= 9.82 kg/s
mg v wood g (3 R 2 h) wood g
v1 = 0, v2 cos = 5 cos 45° = 3.54 m/s
Applying the momentum equation in the x-direction,
(3 R 2 h) wood g (3 R 2 H ) wg Rx = 9.82 × 3.54 = 34.7 N
Force acting on the spring = – Rx = –34.7 which could
h tend to compress it.
wood
=H
water 34.7
Compression of the spring = = 1.74 cm.
20
35. (a) The initial velocity of the water with respect to the 38. (a) Let A be the area of the cross-section of the cylinder
blade is v 2 gh R . Therefore, a mass of water and 3h its height. Let 1 be the density of water.
Then the weight of the cylinder = A.3h 1g.
m A ( 2 gh R ) impinges on the blade in a unit
of time. After the impact, the velocity of the water with
h
reference to the blade is zero, and for this reason the
change in the momentum of the water in a unit time is
mv. According to Newton’s second law, the sought
force is
F A ( 2 gh R)2 .
2h
36. (a) The piston will cover the distance ut during the time t
(figure). The force F will perform the work W = Fut. The
mass of the liquid flowing out during the time t is Aut.

Then weight of air displaced = Ah. 1g, since the length


F 1
A a of cylinder in air is (3h) i.e. h.
ut 3
vt The weight of water displaced = A.2h 1g
For equilibrium, A.3h 1g = Ah 1g + A.2h 1g
or 3 = + 2.
310 IIT-JEE PHYSICS Challenger
39. (d) Let the density of water be , then the force by escaping 42. (a) Let ‘a’ be the radius of the sphere and its density.
Let the densities of three liquids be , 2 and 3 .
liquid on container = S ( 2 gh )2
Acceleration of container 4 3
The weight of the sphere = a g.
3
2 Sgh Vg 2Sh
a= = ÷g The volume of the uppermost liquid displaced = the
V V volume of the lowermost liquid displaced, by symmetry
Sh Sh 1
Now, = a= g = (d 2 d1 ) [3r 2 (d12 + d1d2 + d22 )]
V V 3
40. (b) In order to determine its specific gravity we are only
concerned with the vertical force on the cylinder. 1 a a2 a2
= a ÷ 3a 2 + + a2 ÷
On surface AB, the vertical force equals the weight of 3 3 9 3
the oil actually supported by it.

0.7 0.7
A
B B D
0.8 2
C

1 3
F1 0.5 0.5 1 ÷ 2 g
16
F1 = 0.054 × 2 × 1000 × 0.7 × 9.81 = 741.64 N downwards. [since the distances of the two parallel faces from the
On surface BCD, the vertical force equals the virtual centre are a and a/3 in this case]
weight of oil supported by it
1 2a 13 2 28 a3
1 = ÷ 3a 2 a =
F2 1 0.5 0.7 1 0.8÷ 2 g 3 3 9 81
water
8
The volume of the middle liquid (of density 2 )
F2 = 0.664 × 2 × 1000 × 9.81 = 13027.7 N displaced
Net hydrostatic force on the cylinder = volume of the whole sphere – 2 (volume of the upper
F2 – F1 = 13027.7 – 741.64 = 12286 N upwards most liquid displaced)
This must be equal to the weight of the cylinder
4 3 56 3 52 3
1 = a a a
3 81 81
c × 9.81 × × 1 × 4 × 2 = 12236,
The weight of liquid of density displaced
where, c = 797.3 kg/m3
23 3
The specific gravity of the material should, therefore, = a g,
be 0.797. 81
41. (b) The level of liquid is same in both the limbs. Pressure The weight of liquid of density 2 displaced
in limb I at B = Pressure in limb II at A 52 3
= a 2 g
81
and the weight of liquid of density 3 displaced
28 3
P1 P2 = a 3 g
h II h I 81
For equilibrium, the weight of the sphere = the sum of
A B
the weights of liquids displaced
4 3 28 3
or a g a g
3 81
52 3 28 3
a .2 g a .3 g
h 1g = h 2g 81 81
or 108 = 8 + 104 + 84 = 216
1= 2
PROPERTIES OF MATTER & FLUID MECHANICS 311

or =2 .
9.81 x2
43 . (d) Consider the forces per unit width of the gate. For the 38 x tan ...... (1)
level of water, h metre above the hinge, 2 42.52 cos 2
Force on horizontal part OA equals Differentiating each w.r.t.
1000 × 9.81 × h × (1 × 1) = 9810h N vertically upwards. dx
This force acts at a distance of 1/2 m from the hinge. 0 x sec2 tan 0.0027 x 2 .2sec2 tan
Force on the vertical part of the gate equals d
0.0027 dx
h h2 2
.2 x
1000 × 9.81 × × (h × 1) = 9810 . cos d
2 2
dx
h Maximum x for =0
demands that
This force acts at from the hinge. d
3
Tipping would occur when the overturning clockwise hence x sec2 – 2 × 0.0027x2 sec2 tan =0
moment just exceeds the resisting anticlockwise 1
or x tan = = 185 ........... (2)
moment about the hinge O. 0.0054
Solving (1) and (2) simultaneously
0.0027 x 2
38 = 1.85 –
cos 2
h 9810h²/2 1m
x2 x
2
= 54513 or = 233
h/3 cos cos
O 0.5m A x sin
But x tan = = 185
cos
9810h 185
So, sin = , = 52.4° and x = 142m.
2 233
h h 1
9810 9810 h
2 3 2 46. (b) The velocity of water outflow from a hole is v = 2 gh .
or h = 3 = 1.732m The impulse of the force acting from the side of the
44. (c) A1v1 = A2v2 + A3v3 vessel on the outflowing water F t = mv, where m
A × 3 = A × 1.5 + 1.5A × v3 = Av t is the mass of the water flowing out during the
v3 = 1 m/s time t. Hence, F = v2A = 2 ghA. The pressure at the
bottom p = gh and therefore F = 2pA. The same force
m 30
45. (a) v1 = = = 42.4 m/s acts on the vessel from the side of the stream.
A 1000 / 4 (0.03) 2 Thus, the water acts on the wall with the hole with a
(m mass flow rate) force 2pA smaller than that acting on the opposite wall,
and net with a force smaller by pA as might be expected.
This is due to a reduction in the pressure acting on the
2
wall with the hole, since the water flows faster at this
Window wall.
The vessel will begin to move if µG < 2pA or
2 ghA
<
38m

40m

(µ coefficient of friction)
G
47. (d) Area of cross section of the large piston
(0.05)2
1 A= × = 0.00196m2
2m 4
Hose Pressure of the hydraulic fluid under the piston should
be such as to balance the force applied on the piston
(Note that v12/2g should be more than the height of the 200 1000
p= = 10.2 106 N / m 2
window) 0.00196
v12/2g = 92m > 40m) By Pascal’s law, pressure is transmitted undiminished
The question requires an optimisation of for the in all directions. The pressure at the bottom of the small
maximisation of x to achieve the desired height of piston must be 10.2 × 106 N/m2. The force exerted on
z of 38m. the small piston must be
312 IIT-JEE PHYSICS Challenger
m
(0.015)2 T ma mg ( g a)
10.2 × 106 × × = 18000 N
4
The force desired to be exerted at the handle of the
1 mg 1
= m ( g a) 1 m g 1÷
level should be m 2M
[using eq. (3)
25
F = 18000 × = 1500 N 2 Mg 1
300 =m 1÷
m 2M
48. (a) Since mass M over the pulley balances the bucket with
water, hence the mass of bucket with water is M, which A L
49. (a)
becomes (M + m) when a cork of mass m is tied to the
bottom of the bucket.

O
a
G
T a C
T

B
Mg

Let G be the C.G. of the hemisphere full of liquid (solid


m hemisphere) and O the centre of the base of the
3
hemisphere. Then OG = a , where a is the radius of
8
the hemisphere. Let the hemisphere be suspended
Let in this case the acceleration of the system be a,
from the point A. Then the hemisphere (full of liquid)
then for the bucket with cork we have the equation of
is in equilibrium under the action of two force viz. wt.
motion
of liquid acting at G and the force at A. Hence A and G
(M + m) a = (M + m) g – T ............ (1)
must be in the same vertical line. Let OA i.e. the base
And for the mass M, Ma = T – Mg ............ (2) of the hemisphere be inclined to the vertical at an
where T is the tension in the string round the pulley. angle .
mg
Adding (1) and (2), a = ........... (3) Then, tan =
OG 3a / 8 3
= =
(m + 2M) OA a 8
The forces acting on the cork are :
3
(i) its weight mg acting vertically downwards. or tan = ......... (1)
(ii) the force of buoyancy, acting vertically upwards. 8
(iii) the tension T' in the string tied to the cork which also Also the depth of C.G. of the base = OL = AO
acts downwards in the vertical direction as the other cos = acos .
two forces are vertical. The thrust on the base of the hemisphere
Now the mass of the cork = m and its sp. gr. = . Let be = w.z.S = w.OL. a2 = w.a cos a2,
the density of water, then the mass of the liquid where w is the weight per unit volume of the liquid.
m m 2 3
displaced by the cork = (its volume) × = . = Also the weight of the liquid contained = a .w
3
But this mass is descending with acceleration a, and if And from (1),
P be the upwards force on this mass, we have 1 1 1 8
cos = = = =
m m m sec2 1 tan 2 9 73
.a g P or P ( g a) 1+
64
m The thrust on the plane base w.a cos . a 2 3cos
The upward thrust on the cork = P ( g a) = =
wt. of the contained liquid 2 3 2
For the motion of the cork in space a w
3
m
m × a = mg + T' – ( g a) 3 8 12
= =
2 73 73
PROPERTIES OF MATTER & FLUID MECHANICS 313

50. (b) Let V be the volume of iceberg and let x be the fraction 53. (a) Let r be the radius of the base of the hemisphere or
of volume above water. cone then the height of the base = 2r (given)
Using law of floatation, weight of floating body = weight i.e. VK = 2r or VL = LK = r
of liquid displaced by part of the floating body inside Also from similar triangles VLF and VKB, we have
the liquid.
LF VL r 1 1 1
Therefore, V ice g = (1 – x) V waterg. = = = or LF = KB = r
Using the value of ice and water, we get x = (13/103). KB VK 2r 2 2 3
The volume of the frustum
50
51. (d) ps1 = 50 cm. mercury = × 13.6 = 6.8m WG
100 h (r12 + r22 + r1r2 )
ABFE =
(WG water gauge) 3
ps2 = ps1 = 6.8m WG 2
1 1 1 7 3
= 3 r r + 2r +r r÷ =
2
r
ps3 = 6.8 – 0.5 = 6.3m WG 2 12

Equating ps4 to ps3 etc.


2 3
ps = 6.3 × 2 = 12.6m WG And the volume of the hemisphere = r
3
50 90
ppipe = 12.6 + × 13.6 – 0.0012 × = 19.4 m WG
100 100 V
= 19.4 × 1000 × 9.81 = 190 × 103 N/m2
= 190 kN/m2 (gauge)
The pressure, 19.4m WG equals 1.43m mercury gauge.
L
Allowing, say, 10 cm. to stay in the bottom U-space, the E F
single U-tube mercury manometer would be 153cm. long.
52. (b) Let the control volume be bounded by a surface
abcd cutting through the hand of the fireman. Take
the x-axis along the flow and neglect the weight of
the nozzle assembly and the water it contains. A K B
Applying the Bernouli equation between stations 1
and 2, neglecting their datum difference,
p1 v12 p v2
+ = 2+ 2
2 2
Since (p1 – p2) = p1 gauge = 7 × 9.81 × 1002 = 686700 N/m2 C

p1 p2 686700
= = 686.7 m 2 /s 2 Weight of the liquid contained in the vessel =
1000
(volume of the frustum + volume of the hemisphere) g
v22 v12 2 686.7 = 1373.4 m 2 /s 2
2 7 3 2 3 5 3
v2 6 = r r ÷g r g , where is the
But by continuity = ÷ =9 12 3 4
v1 2
density of the liquid.
Hence (81 – 1) v12 = 1373.4, v12 = 17.17 m2/s2 Now the resultant vertical thrust on the vessel, which
v1 = 4.14 m/s, v2 = 37.26 m/s is partly pressed upwards and partly downwards
Mass flow rate, = weight of the liquid contained
m 1000 0.062 4.14 = 11.71 kg/s 5 3 15 2 3 15
4 = r g r g ÷ = (the weight of the
4 8 3 8
Applying the momentum equation (i.e. Rx = mass flow
rate (v2 – v1) – (p1 – p2) A) between stations 1 and 2, liquid that the hemisphere can hold)
reaction Rx along the x-direction, 54. (a) Weight of cylinder = Upthrust due to upper liquid +
Upthrust due to lower liquid.
Rx = 11.71 × (37.26 – 4.14) – 686700 × 0.062 A A 3 A L
4 D L g =d ÷ L g + 2d ÷ ÷ g
= –1554 N 5 5 4 5 4
The force Fx exerted by the nozzle on the fireman is
equal and opposite to it. 5d
D=
Fx = 1554 N along the flow. 4
314 IIT-JEE PHYSICS Challenger
55. (b) T + 0.8 × 250 × 10–3g = 250 d
g Let AQ = x = CR = BP,
T + 250 d g = 1.2 × 250 × 10–3g then AP = BR = CQ = a – x
Solving, T = 0.5 N Pressure at B, due to the liquids in the tube
56. (c) The weight w is placed at P, such that OP = c (given) APB = g ( – ) AE + g ( + ).EF
The fluid thrust at each point of the curved surface = g ( – ) AP cos 30° + g ( + ) BP sin 60°
being normal to the surface passes through the centre
O of the base. 1 1
= g ( – ) (a – x) 3 +g( + )x 3
A 2 2
And pressure at C, due to the liquids in the tube AQC
O
= g ( – ) AQ cos 30° + g QC sin 60°
G P 1 1
=g( – )x 3 + g (a – x) 3
C B
2 2
V w
Since B and C are in the same horizontal line, so the
W
pressure at B = pressure at C
1 1
g ( – ) (a – x) 3 +g( + )x 3
Hence the resultant fluid thrust V on the curved surface 2 2
in contact with the fluid will be passing through O.
The hemisphere is in equilibrium under the action of 1 1
=g( – )x 3 + g (a – x) 3
the following forces, 2 2
(i) Its weight W acting vertically downwards through G, or ( – ) (a – x) + ( + ) x = ( – ) x + (a – x)
the C.G. of the hemisphere where OG = 3a/8.
1
(ii) The force of buoyancy V, acting vertically upwards or 3 x = a or a x=
through O, and 3
(iii) The weight w placed at P, acting vertically downwards. 58. (b) V is the vertex of the cone and VB is the generator in
Let be the inclination of the axis OC of the hemisphere contact with the horizontal table.
to the vertical.
Taking moment of these forces about O, we get A L
W.OG sin = w.OP sin (90° – )
3a
or W . sin w.c.cos
8
8cw O
or tan =
3aW
57. (c) ABC is the given closed tube. Let a be the length of
each side of the tube. P, Q and R are the points on the
surfaces of separation of the liquids. Let the densities V
of the liquids be ( – ), and ( + ). The portions N B

PAQ, QCR and RBP be filled with liquids of densities


– , and + respectively. A is the highest point of the cone and the horizontal
A plane passing through A is the level of the free-surface.
Let be the semi-vertical angle of the cone, then
x AOL = . Let h be the height of the cone, then
the radius of the base of the cone = h tan .

D Q Also the depth of O, the C.G. of the base of the cone
below the free surface
= OL = AO cos = h tan cos = h sin
Thrust on the base of the cone
P
E = wzs = w.OL. (h tan )2
1 3
x
R = w h sin h2 tan2 = 3 sin h tan 2 .w
F 3
C
B + x = 3 sin [weight of the liquid contained in the cone]
PROPERTIES OF MATTER & FLUID MECHANICS 315

59. (c) Let be the density of the gas, then that of the air is
15 . Then the weight of the balloon = weight of the gas v
+ weight of the envelope = Vg + w.
From viscous drag F = A y ÷ (numerically),
If f be the required acceleration of the balloon acting we have F = 0.90 × 10–2 × 8 × 104 × 1 = 720 dyne
vertically upward and then from “mass × acceleration
To maintain, uniform speed, an external force equal to
= forces acting in the sense of acceleration” we get
the viscous drag F should be applied. So, the required
(Vg + w) force is 720 dyne.
a = force of buoyance – wt. of the
g 66. (d) F – T = 3a ; T = 2a
balloon with gas = V 15 g – (Vg + w) T = 2.5 × 109 × 4 × 10–8
T = 100 N, T = 2a
14Vg w
or a g 100 = 2a ; a = 50 N
Vg w ÷ F = 5 × 50 = 250 N
60. (a) As net force on the sphere is zero therefore centre of 67. (d) The pressure at a point just outside the bubble
mass does not move. But as there is a torque on sphere p0 = Patm + h g
due to mg and buoyant force in clockwise direction = Patm + 10 × 10–2 × 103 × 9.8
there sphere rotates clockwise and hence equilibrium = Patm + 980 Nm–2
is achieved when O comes directly above C.
Now excess pressure within the bubble compared to a
61. (a) Density is maximum at 4°C.
point just outside
62. (a) For equilibrium
4dA g = (dAL + m) g
2T 2 72 10 3
= = = 72 Nm 2
and > Ycm (for rotational equilibrium) R 2 10 3
2
Inside pressure = excess pressure + outside pressure
m 0 + dAL ( L / 2) dAL 2 = Patm + 980 + 72
Ycm = = = Patm + 1052 Nm–2
m + dAL 2 ( m + dAL )
Pressure within the bubble in excess of atmospheric
AL2 d m + dAL AL2 d pressure = 1052 Nm–2.
>
( ALd + m) 4dA ( ALd + m ) 68. (b) The vertical component of the tension in the two ends
m > ALd of the wire, 2Tsin(15°), must equal the total loaded
weight W. The volume of the wire is Rw2 L, where Rw
63. (b) If T be the tension in the rope, then is the radius of the wire, and the total volume of ice is
2T = 10kN (Ri2 – Rw2) L, where Ri is the radius of the ice-covered
T = 5kN.
wire. Multiplying by the respective densities, the total
Longitudinal stress in the rope
mass of the 500m-long ice-covered wire is 694.5 + 3531.5
T 5kN
= = 3 = 5 Nmm 2 = 4226 kg, having a total weight of 41415N. The tension
A 10 mm 2 in the wire is thus 41415/(2sin15°) = 80000 N, and
Extension in the rope dividing by the cross-sectional area of 1.77 x 10-4 m3,
the stress is 4.52 x 108 N/m2
Stress 5 Nmm 2
= L 1500mm = 7.5mm 69. (c) Let P1 and P2 be the internal pressure of air, in the two
Y 103 Nmm 2 identical bubbles and the bubble resulting from the
7.5 combination of the two respectively,
Deflection of the load = = 3.75mm
2 4T 4T
P1 – P = and P2 – P =
3Mg x y
s
AY 3 Ab Yb 3 a Since, the total number of moles remain constant,
64. (b) steel
= s s = s
=
brass 2 Mg 2 b As Ys 2 b 2 c 2 (no. of moles in bubble of radius x) = (no. of moles
b
Ab Yb in bubble of radius y)
65. (a) For low velocity, the water layer in contact with the
river bed can be assumed to be stationary. i.e., 2 PV
1 1 PV
= 2 2 [where = temperature]
R R
Since the velocity of water layers, increase from 0 to 2
m/s over a vertical height of 2m so, the velocity gradient 4 3 4 3
i.e., 2 P1 x P2 y ÷
v 2 0 3 3
= = =1s 1
y 2
316 IIT-JEE PHYSICS Challenger
For the columns of water in the two vertical limbs of
P1 y3
= 3 ........ (1) the U-tube referred to datum.
P2 2 x Surface tension force at A = d1T upwards
and surface tension force at B = d2 T upwards
4T 4T The difference in the two surface tension forces
Since P1 = P + and P2 = P +
x y supports the weight of the column of water in the
smaller tube above the datum level.
P ( P + 4T / x)
So, 1 = ........ (2) d 22
P2 ( P + 4T / y ) d2T – d1T = h g
Equating eqns. (1) and (2), we get 4
4 (d 2 d1 ) T
y3 P + 4T / x whence, h =
= d 22 g
2 x3 P + 4T / y
Py3 + 4Ty2 = 2Px3 + 8Tx2 4 (1.5 1.0) 0.0075
= = 0.68 mm
4T [y2 – 2x2] = P [2x3 – y3] (1.5 / 1000)2 1000 9.81
P [2 x3 y3 ] F e LF
T= 71. (b) = y ; e=
4 ( y2 2 x2 ) A L AY
70. (a) 1
Energy stored = × stress × strain × volume
2
A B 1F e 1 F 2L
h = AL = Fe =
2 AL 2 2 AY
D D
d2
d1

1. (b) The speed of efflux µ depth of orifice from free surface 4. (d) The cube will become spherical because of surface
tension.
2. (d) The distance x1 = v1t
4 3
5. (c) r = a3
2.(3h) 3
= 2gh = 2 3h
g 1/ 3 1/ 3
3a3 3
3. (c) The horizontal distance (r) = = . (a)
4 4
2( H y )
x= 2gy .
g 32 / 3 a 2
U = (T) (4 r2) = (T) . 4
where H : total height, y : depth of hole (4 )2 / 3
U = (T) (4 )1/3.(3)2/3 (a2)
= 2 y( H y)
1/ 3
H 2 r2 ( )g 3a3
xmax for y = and same from y and H – y. 6. (a) vT = ; where r = ÷
2 9 4
PROPERTIES OF MATTER & FLUID MECHANICS 317

7. (b) Volume of fluid crossing the section of the tube of By F.B.D. T + P2A = mg = P1A
length and radius R in unit time T = mg + (P1 – P2) A = mg – (P2 – P1) A
= 2 × 2000 × 10 – (0.2 × 105)
R
R2 r2 = 0.4 × 105 – 0.2 × 105 = 0.2 × 105 N
V 2 rvdr 2 v0 r ÷ dr Fb = V. wg = 2 × 1000 × 10 = 0.2 × 105 N
0 R2
It is also equal to net contact force by the liquid
= P2A – P1A
R2 R2 v0 R 2 = 0.2 × 105 N
= 2 v0
2 4 2 Note : Net contact force and buoyant force are same.
13. (a) A1V1 = A2V2
8. (c) Kinetic energy of the fluid within the volume of the
4cm2 × 1 = 1mm2 × 2m/s
tube
400mm2 × V = 1 mm2 × 2m/s
1 1
= (dm) v 2 = (2 rdr ) v2 2 20
2 2 V = = mm / s = 5mm / s
400 4
R 2 14. (c) Height of spring = V/A = 5cm
R2 r2
= r v02 ÷ dr Work done = KE of water + gain in PE of water
0 R2 + gain in PE of piston

R 1
r5 2r 3 = (V )v2 + m1gh1 + m2gh2
= v02 r ÷ dr 2
0 R4 R2
1
= [(20 × 10–6) 1000]22 + [20 × 10–3 × 10 × 2.5 × 10–2]
2
R2 R2 R2 v02 R 2
= v02 ÷= + [0.1 × 10 × 0.05]
2 6 2 6 = 0.04 + 0.005 + 0.05 = 0.095 J
15. (b) As amount of water reduces continuously force also
r2 dv 2r reduces continuously.

9. (a) v v0 1 , v0 ÷
R dr R2 16. (b), 17. (c), 18. (a)
The initial velocity of water coming out of hole is
Friction force due to viscosity horizontal and hole is at a height h/4 from ground. Hence
dv dv 2( h / 4)
= A (2 R ) ÷ time taken by water to reach ground is t =
dr dr r = R g

2R which remains constant.


= (2 R ) v0 ÷ 4 v0 x = vt, where v is velocity of efflux.
R2 Since v decreases with time x will decrease.
10. (b), 11. (b), 12. (b) Let y be the height of water surface above hole
By conservation of volume
dy av a 2 gy
4 × h = 4 × 2 + 2 × 1 = 10 – = =
h = 2.5m. dt A A
Pressure at top of the object
= P0 + 0.5 × 1000 × 10 = 1.05 × 105 N/m2.
F = P1A = 1.05 × 105 × 2 = 2.1 × 105 N y
v
T
PA1

v
0.5m

h //////////////////////////////////////////////////////////////
A=2m ² 1m

mg 0 t
PA2 dy a A 3h
dt t=
2 gy A a 2g
4m² 3h / 4 0
318 IIT-JEE PHYSICS Challenger
19. (b), 20. (d), 21. (c). 23. (a) Given that the rod AB = 6 ft.
h The immersed length AC = 4ft.
Fv [ g (2 R R cos )] [ Rd L]cos
B
0 G
V C N
2 A
Fv gR L cos (2 cos ) d G1
W
0

The length outside the water = BC = 6 – 4 = 2'


If the rod makes an angle with the vertical, then

BN 1 3
cos = = or sin =
BC 2 2
The forces acting on the rod are :
2 (i) The weight W of the rod = 6 × g acting through
= gR L (2 cos cos2 ) d gR 2 L
2 G, the C.G. of the rod, where , and are the area
0
of cross-section of the rod, sp. gr. of the rod and
Force = gR 2 L upwards density of water respectively.
2 (ii) The force of buoyancy V = 4 × g, acting vetically
As all forces are radial and all pass through axis and upwards through G 1 , the C.G. of the water
hence torque is zero. displaced and
(iii) The reaction at the fulcrum at B.
Fx = g (2 R R cos ) ( Rd L) sin 4 R 2 gL Taking moments of these forces about B, we have
0 W.BG sin = V.BG1 sin .
22. (a) AB is the rod of length 2a say, with end B fixed and or 6 g .BG = 4 g.BG1
1 or 3 BG = 2BG1
length AC = (2a) immersed in water..
3
1
Let be the area of cross-section of the rod and
its 2 BC + AC ÷
2 BG 2
sp gr. or = . 1=
Let be the density of water. 3 BG 1
3. AB÷
The rod AB is in equilibrium under the action of the 2
following forces :
(i) The weight 2a g acting vertically downwards 2 (2 + 2) 8
through G, the C.G. of the rod. = =
(ii) The force of buoyancy V or the weight of the 3(3) 9

1 24. (c) AB = 2a is the rod whose middle point G is in the


water displaced = AC. g=
(2a) g, acting surface and the point C on the rod, such that AC
3
vertically upwards through G1 , the centre of 1 1
buoyancy or C.G. of the water displaced. = (AB) = a, is fixed and the rod can turn about
6 3
(iii) The reaction at the end B, which is fixed.
C.
If be the inclination of the rod with the horizontal,
then taking moment of these forces about B, we Let be the area of cross-section of the rod and be
have its sp. gr. Let be the density of the water.
B
1 V
2a g BG cos = (2a) g.BG1 cos G
3
1 G1
or BG = BG1 C
3 A
W
1
BG1 BC + 2 ( AC ) The forces acting on the rod are :
or = = (i) its weight W = 2a g, acting downwards through
3BG 1
3. (2a) G, the C.G. of the rod :
2
2 1 2 (ii) the force of buoyancy V = a g, acting vertically
(2a) + a÷ upwards through G 1 , the C.G. of the water
3 2 3 5
or = = displaced and
3a 9
PROPERTIES OF MATTER & FLUID MECHANICS 319

(iii) the reaction at C. 27. (c) Upthrust and effective weight changes by same factor
Let be the inclination of the rod to the vertical. hence fraction immersed remains same.
Taking moments of the forces about C, we have
dv 2 Rv0
W.CG sin = V.CG1 sin . 28. (c) f 2 R ÷ = 2 R ;F 4 v0
or W.CG = V.CG1 dr R2
or 2a g (AG – AC) = a g (AG1 – AC) 2rv0
29. (a) F 2 r 2
F µ r2
1 1 1 R
or 2 a a a a÷
3 2 3
30. (b)
4 1 1
or a÷ = a or =
3 6 8
25. (b) Weight of Ram is more than that of Shyam in water
means upthrust on Ram is less hence less volume and
less fat content.
26. (a) Let Fat mass = m1, Other mass = m2. dm = dV = dA. ; dm = 2 r dr
Total volume = V so momentum of mass dm
m1 V m2 3V
Given : = , = and p R
0.4d w 4 (4 / 3) d w 4 dp = v dm ; dp = v dm
0 0
m1 + m2 = 165 R 2 R
r r2 r4
p 2 v0 r 1 ÷ dr ; p 2 v0
1650 0 R2 2 4R2
Solving, V = 0
11d w
R2 R2 2 v0 R 2 R 2 v0
1650 = 2 v0 = ; p=
Spring balance reading = 165 d w = 15 kg 2 4 4 2
11d w

5. (a) At the first moment the ship will begin to move to the
right, since the pressure on the starboard side
diminishes by 2pA, where p is the pressure at the depth
h of the hole, and A is its area. As soon as the stream of
water reaches the opposite wall, this wall will be acted
1. (b)
upon by the force F = Av2, where v is the velocity of
the stream with respect to the ship. The force F is

mg = 2T sin slightly greater than 2pA, since v > 2 gh because


Radius does not affect the process. the ship moves towards the stream. As a result, the
2. (a) Since pressure inside bubble of smaller radius is more motion will begin to retard.
the common interface should bulge inside bubble of 6. (a) According to Bernoulli’s theorem, the pressure
larger radii. Hence statement-2 is correct explanation decreases in the region where velocity of the wind is
of statement-1. high. The pressure difference is enough to lift the roof
3. (c) Tension at a point on rod of (length L) at a distance x up and carry it aside.
7. (c) Since the gas in the stream has a high velocity, the
x pressure inside the stream is below atmospheric. The
from point of application of force is T F 1 ÷ in
L ball will be supported from the bottom by the thrust of
both cases. Hence weight has no effect on tension in the stream, and on the sides by the static atmospheric
situation of figure (ii). Extension in rod occurs due to pressure.
force acting at any point on the rod. in certain cases 8. (a) Statement-1 is True, Statement-2 is True; Statement-2
when net force acts at the centre of rod like weight, is a correct explanation for Statement-1.
extension due to this force may not occur like the given 9. (d) Applying Principle of continuity & Bernoullies theorem
case. A v P , h1d1 = h2d2
4. (d) For stable equilibrium centre of gravity should exist 10. (d) At distance x from where force is applied
below centre of buoyancy of submerged part. Hence
statement-1 is false. F
T= x
L
320 IIT-JEE PHYSICS Challenger

1. (a, b, d) Since acceleration of centre of mass is upwards


therefore N must be greater than mg.
d2
2. (a, c) Area of cross-section, A =
4
where d is the diameter of the wire. Therefore, 0.5m

4FL
=

0.25m
d 2Y
Since, F, L and Y are the same for wires A and B x
ax
1
µ
d2
i.e., the extension is inversely proportional to the
square of the diameter. 0.2m

4F 1 zmax = 0.5m = 0.2 tan hence tan = 2.5


The strain is = . Thus, strain µ 2
L 2 d
d Y ax
But, tan = = 2.5
mg mg 4mg g
3. (a, b, c) TB = and TA = mg + =
3 3 3 Hence, ax = 2.5g = 24.5 m/s2
TA = 4TB The range of operation of this accelerometer is
0 to 24.5 m/s2.
T
Stress, S = 6. (a, c) Let us denote the distance from the level of the
r2 water to the upper hole by h, the sought distance
(a) For rA = rB, SA = 4SB from the vessel to the point where the streams
A breaks intersect by x, and the distance from the level of
(b) For rA < 2rB, SA > SB the water in the vessel to this point by y (figure).
A breaks The point of intersection will remain at the same
(c) For rA = 2rB, SA = SB place if the level of the water in the vessel does
either may break not change. This will occur if Q = Av1 + Av2, where
4. (b, c) Thermal force = YA d = Y r2 d v1 = 2 gh and v2 = 2 g ( H + h) are the
r1 = r, r2 = r 2, r3 = r 3, r4 = 2r , outflow velocities of the streams from the holes.
On the basis of the law of kinematics,
F1 : F2 : F3 : F4 = 1 : 2 : 3 : 4
Thermal stress = Y d
As Y and are same for all the rods, hence stress h
developed in each rod will be same.
As strain = d , so strain will also be same. H y

1
E = Energy stored = Y (strain)2 A L
2
E1 : E2 : E3 : E4 = 1 : 2 : 3 : 4 x
5. (a, b, c, d) When subjected to horizontal acceleration, the
fluid in one limb rises by the amount equal to the
depression in the other limb. Thus, for maximum
gt12 gt 2
possible acceleration, i.e. for a maximum value of x = v1t1 = v 2 t 2 and y = h + = h+ H + 2
tan , the fluid should be initially filled to 50% 2 2
height in the instrument i.e. 0.25m in each limb, where t1 and t2 are the times during which the
and should be at the average of spilling over when water falls from the holes to the point of
accelerated. intersection.
PROPERTIES OF MATTER & FLUID MECHANICS 321

When rotating free, let the angular velocity be .


1 Q2 2 gA2
Hence, x H2 ÷ = 120 cm. Now the absolute velocities of the nozzle-
2 2 gA2 Q2 discharge in the circumferential direction are
for nozzle A, v A = (2.2 – 0.2 ) m/s
1 Q2 2 2 gA
2
for nozzle B, v B = (1.8 – 0.15 ) m/s
y= + H ÷ = 130 cm.
2 2 gA2 Q2 There being no external moment, the angular
momentum should be conserved,
7. (a, b) Let us compute the slope of the free surface of QA (2.2 – 0.2 ) × – 0.2 + QB (1.8 – 0.15 )
the water × 0.15 = 0
ax 4 cos15° Cancelling and using QA = QB, = 9.72 rad/s.
tan = = = 0.35
g + ax 9.81 + 4sin15° 9. (b,c) When the block of mass m is arranged as shown
in the figure, an upthrust FT will act on the mass
whence = 19.8° backwards
which will decrease the reading on A.
The maximum slope which the surface can make
with the horizontal, without spilling, is
–1
max = tan (2/2.5) – 15° = 23.66° backwards
A

2.5m
4m/s²
2m
/////
/////// FT FT
/ ///////
////////
////// ////// B
///////
///// ///////
////////
///////
//
////////15° According to Newton's third law, to each and every
///////
action, there is equal and opposite reaction.
The slope, 19.2°, made by the surface is less than So FT will act on the liquid of the beaker which
this value, the water will not spill. will increase the reading in B.
Let the acceleration at which water begins to spill 10. (a, b, c, d) Let zmin be the minimum reference level of the
be ‘a’ then, dotted parabola and z1 and z2 the liquid levels
above the base.
a cos15°
tan ( 23.66)° r12 2 0.12 10 2
g + a sin15° ÷
tan max = z1 = + zmin = + zmin ,
2g 2 9.81
0.438 r22 2 0.2 2 10 2
z2 = + zmin = + zmin
whence a cos 15° – a sin 15° × 0.438 2g 2 9.81
= + 0.438 × 9.81 But z1 + z2 = 2 × 0.4 = 0.8m
and a = 5.04 m/s2.
8. (b, d) Assume the discharge to be equally divided 0.22 102 0.1 102
Hence, 0.8 = + 2 zmin
between the two nozzles. 2 9.81
3 whence, zmin = 0.273m
3 10 6
QA = QB = 50 10 m3 / s
60 Consequently,
50 10 6 0.12 102
vA = vB = = 2.54 m / s z1 = 0.273 + = 0.324m
4 0.0052 2 9.81
v A = 2.54 cos 30° = 2.2 m/s 0.22 102
z2 = 0.273 + = 0.477m
v B = 2.54 cos 45° = 1.8 m/s 2 9.81
Assume that the water entering the sprinkler
2 ( H y)
through a tap does not involve any angular 11. (a, b, c) x = vt = 2 gy
momentum. g
When stationary, the torque due to nozzles action,
H
for nozzle A, A = 1000 × 50 × 10–6 × 2.2 × 0.20 x 2 y (H y) Max. when y =
2
= 0.0220 Nm
for nozzle B, B = 1000 × 50 × 10–6 × 1.8 × 1.5 H
as y x=H
= 0.0135 Nm 2
Total torque due to nozzles A and B,
= 0.0355 Nm
322 IIT-JEE PHYSICS Challenger
12. (a, b) Figure (a) and (b) shows the F.B.D. of two partitions
1 2T 4T
of the bar. Considering the vertical equilibrium of 13. (b, c) 2 2T x = k (2 x ) 2 ; x , xm =
2 k k
the part of length 3 .
R1 + R2 = W = 5000 N ..... (1) m
Fnet 2T 2kx ; T0 2
Now, stress in the two portions are 2k
14. (b, d) Power transmitted is given by P =
R1 R2
1 and 2 =
A A P 120 10 3
or = = = 5.73 × 103 N-m
R1 200
2 ÷
R1 60

W
r4 r3 r
2
3 From, = , we have = ÷
2 2
But maximum intensity of stress is at the periphery
R1 R2 max = shear strain × modulus of rigidity
(a) (b) r
Elongation in the portion of length 2 = ÷
1 R1
1 2 2
Y AY r3 2
max max =
and comparison in the portion of length 3 2 r3
2 R2
2 3
3 2 5.73 103
Y AY max =
2.92 107 Nm 2
Since there is no net change in the length of the 3.14 (5 10 2 )3
bar so, elongation in the upper portion = r4
compression in the lower portion. Again, from =
2
2 R1 3R2 3R2 We have
1 2 R1 = ... (2)
AY AY 2
2 2 5.73 103 6
Solving eq. (1) and (2), = =
r4 3.14 (5 10 2 ) 4 8 1010
R1 = 3000 N, R2 = 2000 N
= 4.37 × 20–2 radian = 2°30'

1. A-p, q, r; B-s; C-q; D-p, q, r, s


2. A-r; B-q; C-q; D-q
In A : Same water level implies

B is heavier.
3. A-q, r, t; B-s; C-q, t; D-p

Wt. of fluid displaced is the same as that of object hence Bernoulli’s theorem can be used to calculate speed of efflux;
both buckets have equal weight. it is based on conservation of energy and a device
In B, C : Mass of water in both buckets is equal and B has venturimeter works reading on basis of Bernoulli’s theorem.
additional mass of solid object hence B is heavier. Stoke’s law is based on viscosity.
In D : Same water level and object sinks 0 > i.e. some Torricelli’s theorem can be used to calculate speed of efflux
volume of is replaced by same volume of 0 mass and is based on conservation of energy.
increases. Hooke’s law is used to determine elastic limit.
PROPERTIES OF MATTER & FLUID MECHANICS 323

4. A-p, r; B-p, q, r; C-p, s; D-r, t 5. A-p, q, r, s; B-q, t; C-q; D-s, t


Stoke’s law : Elastic force, Force due to surface tension, Frictional force
F 6 rv and Viscous force decrease with rise in temperature.
Terminal velocity : Forces involved in capillary action are surface tension and
gravitational force.
2 r2 g ( ) Water flows in a continuous stream down a vertical pipe
v=
9 whereas it breaks into drops when falling freely because of
Excess pressure inside mercury drop : surface tension.
2T Terminal velocity of rain drop depends on viscous force
P= and gravitational force.
r
Viscous force :
dv
F A
dx

1. 3
1 1
M is the mid-point of tube AB at equilibrium 10 = 45 × 76
p1× A + mg = p2 × A 45 x 45 + x
p1 × A + 10 × A × dHgg = p2 × A
p1 + 10dHg × g = p2 ... (i) 45 x 45 + x
= 45 × 76
For air present in column AP (45) 2 x2
p × 45 × A = p1 × (45 + x) × A
45 45 76[2 x]
p1= 76d Hg g ... (ii) 10 = 2025 – x2 = 684 x
45 + x 2025 x 2
For air present in column QB
p × 45 × A = p2 × (45 – x) × A x2 + 684 x – 2025 = 0

45
76d Hg g (684)2 4 1 ( 2025)
p2 = ... (iii) x = – 684 ±
45 x 2
From (i), (ii) and (iii)
45 76 d Hg 45 475956
10 d Hg g 76 d Hg g = – 684 ±
45 x 45 x 4
= – 342 ± 345 = 3 cm.
45 76 45 76
+ 10 = 2. 10
45 x 45 x
Let the edge of cube be . When mass is on the cube of
wood
A A

45 cm

p1 P
x 10 cm
P'
M M
10 cm

p2 Q' Q 200g + 3
d wood g = 3
d H 2O g
mg
45 cm 200 + 3
d wood = 3
d H 2O

3 3
d wood d H 2O 200 ... (i)
when the mass is removed
B B 3 2
d wood ( 2) d H 2O ... (ii)
324 IIT-JEE PHYSICS Challenger
From (i) and (ii) P1x1 = P2x2
3
d H 2O 200 ( 2) 2
d H 2O where P2 = P1 + Pressure due to mercury column
x2
But d H 2O = 1 5cm P1
3 2 x x x1 D
200 ( 2)
P2 C
P P
3
200 3
2 2 = 10 cm 30 O
B
A B C D
3. 45 5cm A
For equilibrium Pressure due to mercury column
Fnet = 0
F mg sin 30° Vdg sin 30°
net = 0 P= = =
A A A

( A 5) dg sin 30°
= = 5 sin 30° cm of Hg
x A
F1 P2 = P1 + 5 sin 30° = P1 + 2.5
1m

0.5m Substituting this value in (iii)


mg P1 × x1 = [P1 + 2.5] × x2
P1 × 46 = [P1 + 2.5] × 44.5
O
44.5 2.5
Taking moment about O P1 =
1.5
x
mg sin FT ÷ sin ... (i) Substituting this value in (ii)
2 2
Also FT = weight of fluid displaced. 44.5 2.5
P×r= 46
1.5
FT = [ ( x ) A] g ... (ii)
46 44.5 44.5 2.5
and m = ( A) 0.5 w ... (iii) P× 46
2 1.5
where A is the area of cross section of the rod.
From (i), (ii) and (iii) x + x2
x= 1 P = 75.4 cm of Hg
x 2
( A) 0.5 wg sin = [(u x ) A] w g ÷ sin
2 2 5. 6.75
Here, = 1 1 , i.e., in equilibrium, block is half submerged in water..
(1 – x)2 = 0.5
1
=
2 2
1 – x = 0.707
Let h' be increase in level if block is pressed by an amount h.
x = 0.293 m
Then
From the diagram
0.5 0.5 (A – 2
)h ' = 2
h h ' = 0.5h
cos = =
1 x 0.707 We have to immerse further / 2 .
= 45°
/ 2 = h + h ' = 1.5h
4. 75.4
Let A be the area of cross-section of the tube. Extra thrust upon depressing block by h
Since temperature is the same, applying Boyle's law on the F = (h + h ') 2
g 1.5 2
2 2 gh
side AB
P × (x × A) = P2 × (x2 × A) ... (i) /3 /3
Applying Boyle's law in section CD W= Fdh = (1.5 2 2 gh) dh = 6.75 J
P × (x × A) = P1 × (x1 × A) ... (ii) 0 0
From (i) and (ii)
P1 × (x1 × A) = P2 × (x2 × A)
1. A particle of mass m is executing oscillations about the origin (a) 0.283 sec (b) 0.0283 sec
on the X-axis with amplitude A. Its potential energy is given (c) 2.83 sec (d) 28.3 sec
as U(x) = x4 where is a positive constant. The x 5. The displacement of a particle is given at time t, by:
coordinate of the particle where the potential energy is one
third of the kinetic energy is x A sin( 2 t ) B sin 2 t Then,

(a) (b) A/ 2 (a) the motion of the particle is SHM with an amplitude of
A/2
(c) (d) A/ 3 B2
A/3 A2 +
2. Two masses m 1 and m 2 are suspended together by a 4
massless spring of spring constant k. When the masses are (b) the motion of the particle is not SHM, but oscillatory
in equilibrium, m1 is removed without disturbing the system. with a time period of T = /
Find the angular frequency of m2 (c) the motion of the particle is oscillatory with a time period
k k of T = /2
(a) ( m1 m2 ) (b) m1 (d) the motion of the particle is aperiodic.
6. The displacements y of a particle executing a certain periodic
k k
1
(c) m2 (d) ( m1 m2 ) motion is given by y = 4 cos
2
t ÷ sin (1000t ) . This
2
3. Two bodies M and N of equal masses are suspended from expression may be considered to be the superposition of n
two separate massless springs of spring constants k1 and independent harmonic motions. Then, n is equal to
k2 respectively. If the two bodies oscillate vertically such (a) 2 (b) 3
(c) 4 (d) 5
that their maximum velocities are equal, the ratio of the
amplitude of vibration of M to that of N is 7. A particle is describing simple harmonic motion. If its
velocities are v1 and v2 when the displacements from the
k1 mean position are y1 and y2 respectively, then its time period
(a) k2 (b) k1 / k2
is
k2 v22 v12
(c) (d) k2 / k1 y12 + y22
k1 (a) 2 (b) 2
v12 + v22 y12 y22
4. A block of 4 kg produces an extension of 0.16 metre in a
spring. The block is replaced by a body of mass 0.50 kg. If v12 + v22 y12 y22
the spring is stretched and then released the time period of (c) 2 (d) 2
y12 + y22 v22 v12
motion will be

MARK YOUR 1. 2. 3. 4. 5.
RESPONSE 6. 7.
326 IIT-JEE PHYSICS Challenger
8. A body of mass 0.01 kg executes simple harmonic motion
(SHM) about x = 0 under the influence of a force shown A
x (t ) B
below
F(N)
t
80
0.2 x(m) 2
– 0.2
– 80

The period of the SHM is x (t ) B


A
(a) 1.05 s (b) 0.52 s
(c) 0.25 (d) 0.03 s
9. A particle free to move along the x-axis has potential energy t
given by U(x) = k [1–exp(–x2)] for x , where k is
a positive constant of appropriate dimensions. Then 3
(a) at points away from the origin, the particle is in unstable
equilibrium (a) + 180°, + 90°, – 90° (b) – 180°, – 90°, + 90°
(b) for any finite nonzero value of x, there is a force directed (c) + 360°, + 180°, – 180° (d) + 180°, + 180°, – 90°
away from the origin 12. The period of oscillation of a simple pendulum of length L
(c) if its total mechanical energy is k/2, it has its minimum suspended from the roof of a vehicle which moves without
kinetic energy at the origin. friction down an inclined plane of inclination , is given by
(d) for small displacements from x = 0, the motion is simple
harmonic L L
(a) 2 (b) 2
10. A particle describes SHM in a straight line about O. g cos g sin
O P
L L
(c) 2 (d) 2
If T is the time period of the motion, then its kinetic energy g g tan
at P be half its peak at O, if the time taken by the particle to
travel from O to P is 13. The velocity of the bob on a pendulum of length 10 m (figure)

1 1 2 t
(a) T (b) T is given by v = v0 cos where v0 = 1.00 m/s and T =
2 4 T

1 1
(c) T (d) T 2 s.The radial acceleration at t = s is
2 2 8 4
11. The figure shows three situations of displacements x(t) of a (a) 1 m / s2 (b) 1/10 m / s2
pair of two SHM (A and B) that are identical except for the (c) 1/20 m / s2 (d) 1/30 m / s2
phase difference. For each pair, find out the phase shift (in 14. A particle performs S.H.M. on x-axis with amplitude A and
degrees) needed to shift the curve for A to coincide with the time period T. The time taken by the particle to travel a
curve for B. distance A/5 starting from rest is :

x (t ) T T FG 4 IJ
A B (a)
20
(b)
2
cos–1 H 5K
t T FG 1IJ T FG 1IJ
(c)
2
cos–1 H 5K (d)
2
sin–1 H 5K
1

MARK YOUR 8. 9. 10. 11. 12.


RESPONSE 13. 14.
SIMPLE HARMONIC MOTION 327
15. A block of mass ‘m’ is suspended from a spring and executes 19. Four massless springs whose force constants are 2k, 2k, k
vertical SHM of time period T as shown in figure. and 2k respectively are attached to a mass M kept on a
The amplitude of the SHM is A frictionless plane (as shown in figure). If the mass M is
displaced in the horizontal direction, then the frequency of
the system.

k
2k 2k
M
2k

1 k 1 4k
(a) (b)
m 2 4M 2 M

Spring is never in compressed state during the oscillation. 1 k 1 7k


(c) (d)
The minimum force exerted by spring is never in compressed 2 7M 2 M
state during the oscillation. The minimum force exerted by 20. m1 and m2 are connected with a light inextensible string
spring on the block is
with m1 lying on smooth table and m2 hanging as shown in
4 2 4 2 figure. m1 is also connected to a light spring which is initially
(a) mg mA (b) mg + mA
T2 T2 unstretched and the system is released from rest
2 2
(c) mg 2
mA (d) m g +
mA k
T T2
16. On a smooth inclined surface a body of mass M is attached m1
between two spring. The other ends of the springs are fixed
to firm supports. If each spring has force constant k, the
period of oscillation of the body is
Mg sin mm2
(a) 2 (M / 2k)1/ 2 (b) 2 ÷
2k
(a) system performs SHM with angular frequency given
1/ 2 1/ 2
2M 2 Mg
(c) 2 ÷ (d) 2 ÷ k( m1 m2 )
k k by m1m2
17. A particle executes simple harmonic motion between x = A
and x = +A. The time taken for it to go from 0 to A/2 is T1 and (b) system performs SHM with angular frequency given
to go from A/2 to A is T2. Then
(a) T1 < T2 (b) T1 > T2 k
(c) T1 = T2 (d) T1 = 2T2 by m1 m2
18. A uniform cylinder of length L and mass M having cross-
sectional area A is suspended with its vertical length, from a (c) tension in string will be 0 when the system is released.
fixed point by a massless spring, such that it is half- m2 g
submerged in a liquid of density d at equilibrium position. (d) maximum displacement of m1 will be
When the cylinder is given a small downward push and k
released, it starts oscillating vertically with a small amplitude. 21. A simple pendulum with length L and mass M of the bob is
If the force constant of the spring is K, the frequency of vibrating with amplitude a. Then the maximum tension in
oscillation of the cylinder is : the string is
1/ 2 1/ 2
1 K Adg 1 K + dgL a
2
(a) ÷ (b) ÷ (a) Mg (b) Mg 1 + ÷
2 M 2 M L
1/ 2 1/ 2
1 K + Adg 1 K Adg a
2
a
2
(c) ÷ (d)
2 Adg ÷ (c) Mg 1 + (d) Mg 1 +
2 M L 2L

MARK YOUR 15. 16. 17. 18. 19.


RESPONSE 20. 21.
328 IIT-JEE PHYSICS Challenger
22. A man of mass 60 kg is standing on a plateform which is 25. A tube of given shape has total length 2 and area of cross
oscillating up and down with frequency 2 oscillations/sec section is S. Its bottom and upper halves are filled with two
and amplitude 50 cm. A machine on the platform indicates non-viscous, non-compressible liquids of densities 3 and
weight of the man with respect to time, then the maximum
respectively. If its motion is simple harmonic then find its
reading of the machine will be
angular frequency.
(g = 10 m/s2)
(a) 10 kg (b) 532.8 kg
3
(c) 10 kg (d) 104 kg
23. A uniform rod of mass m and length is hinged at its mid
point in such a way that it can rotate in the vertical plane
about a horizontal axis passing through the hinge. One of
its ends is attached to a spring of spring constant k which is
unstretched when the rod is horizontal. If this end is now 3
given a small displacement and released angular frequency
of the resulting motion is
/2 /2 3g
(a) It is not an S.H.M. (b)
× 2

2g g
k (c) (d)

26. For a particle executing SHM the displacement x is given by


x = A cos t. Identify the graph which represents the variation
of potential energy (PE) as a function of time t and
(a) k/m (b) 2k / m
displacement x
(c) 3k / m (d) g/
I II PE
24. A block of mass m, attached to a fixed position O on a smooth PE III
inclined wedge of mass M, oscillates with amplitude A and
t IV
linear frequency f. The wedge is located on a rough horizontal
surface. If the angle of the wedge is 60º, then the force of
x
friction acting on the wedge is given by (coefficient of static
friction = µ). (a) I, III (b) II, IV
O (c) II, III (d) I, IV
A 27. Three charges q, q and – 2q are fixed on the vertices of an
equilateral triangular plate of edge length a. This plate is in
m
equilibrium between two very large plates having surface
smooth M
rough charge density 1 and 2 respectively. Find time period of
60º
small angular oscillation about an axis passing through its
(a) ( M + m)g centroid and perpendicular to plane. Moment of inertia of
the system about this axis is I.
1 2
(b) m A sin t
2 (a) 0I (b) 0I
2 2
qa | 1 – 2 | 2qa | 1 – 2 |
(c) 2
(M m) A sin t
2 0I 2 0I
3 (c) 2 (d) 2
(d) ( M + m)g + m 2
A sin t 3qa | 1 – 2 | qa | 1 – 2 |
2

MARK YOUR 22. 23. 24. 25. 26.


RESPONSE 27.
SIMPLE HARMONIC MOTION 329

28. A block of mass m is suspended by means of an ideal spring 31. A particle of mass 1 kg is placed in a potential field. Its
of force constant k from ceiling of a car which is moving potential energy is given by U = 10x2 + 5. The frequency of
along a circular path of radius r with acceleration a. The oscillations of the particle is given by
time period of oscillation of the block when it is displaced
along the spring, will be (a) ( 10 ) (b) ( 5)
10 5
(c) ÷ (d) ÷

32. ABC is an equilateral triangle structure made up of a light


rigid material. Find the frequency of small vertical oscillations
of mass m along AG. Consider k1 = k2 = k3 = k4 = k.
m

mg + ma 2
m O
(a) 2 (b)
k k g +a 2 2

k1
m m A
(c) 2 (d) 2
k k 2 + g2 + a2 30°
k2
29. A block of mass M is kept in gravity free space and touches
the two springs as shown in the figure. Initially springs are
in its natural length. Now, the block is shifted (l0/2) from the k3 k4
given position in such a way it compresses a spring and G
released. The time-period of oscillation of mass will be
B C
k M 4k

0 2 0
5k 4k
(a) (b)
2m m
(a) M (b) M
2
2 k 5k
3k 10k
(c) (d)
3 M 5m 9m
(c) (d) M
2 k 2k 33. A point particle of mass 0.1 kg is executing SHM of amplitude
30. The particles execute simple harmonic motion of the same of 0.1m. When the particle passes through the mean position,
amplitude and frequency along close parallel lines. They its kinetic energy is 18 × 10–3 J. The equation of motion of
pass each other moving in opposite directions. Each time this particle when the initial phase of oscillation is 45° can
their displacement is half of their amplitude. Their phase be given by
difference is
7 4 (a) 0.1 cos 6t + ÷ (b) 0.1 sin 6t + ÷
(a) (b) 4 4
6 3

2 (c) 0.4 sin t + ÷ (d) 0.2 sin + 2t ÷


(c) (d) 4 2
6 3

MARK YOUR 28. 29. 30. 31. 32.


RESPONSE 33.
330 IIT-JEE PHYSICS Challenger
34. As shown in figure, a uniform bar is suspended in a 39. Two springs, each of spring constant k = 100 N/m, are
horizontal position by a vertical wire attached to its center. attached to a block of mass 2 kg as shown in the figure. The
When a torque of 5 Nm is applied to the bar as shown, the block can slide smoothly along a horizontal platform clamped
bar moves through an angle of 12°. If the bar is then released, to the opposite walls of the trolley of mass 5kg. If the block
it oscillates as a torsion pendulum with a period of 0.5s. Its is displaced by x cm. and released, the period of oscillation
moment of inertia is in seconds is

////////////////////////////////////
1 7
F (a) T 2 (b) T 2
20 1000
12° r
1 49
(c) T 2 (d) T 2
r 140 100
40. A block is attached to an end of a massless spring whose
F other end is fixed to ceiling. The block is released at rest
(a) 0.151 kg-m2 (b) 2.151 kg-m2 when the spring is in its relaxed state. The maximum
(c) 0.211 kg-m2 (d) 0.321 kg-m2 acceleration of the block during its motion in the vertical
35. A spring mass system performs S.H.M. If the mass is plane is (g is acceleration due to gravity)
doubled keeping amplitude same, then the total energy of (a) g (b) 2g (c) g/2
S.H.M. will become
(d) can be determined only if the values of spring and
(a) double (b) half
mass of the block are given.
(c) unchanged (d) 4 times
41. Two particles execute SHM on same straight line with same
36. A body is executing simple harmonic motion. At a
mean position, same time period 6 second and same amplitude
displacement x from mean position, its potential energy is
5cm. Both the particles start SHM from their mean position
E1 = 2J and at a displacement y from mean position, its
(in same direction) with a time gap of 1 second. The maximum
potential energy is E2 = 8J. The potential energy E at a
separation between the two particles during their motion
displacement (x + y) from mean position is
(a) 10J (b) 14J (a) 2 cm (b) 3 cm
(c) 18J (d) 4J (c) 4 cm (d) 5 cm
37. Starting from the mean position body oscillates simple 42. A small body of mass m is connected to two horizontal
harmonically with a period of 2s. After what time will its 3d
kinetic energy be 75% of the total energy springs of elastic constant k, natural length . In the
4
1 1 equilibrium position both springs are stretched to length d,
(a) s (b) s as shown in figure. What will be the ratio of periods of the
6 4
1 1 Ta
Tb ÷
(c) s (d) s motion if the body is displaced horizontally by a
3 12
38. A particle of mass m is attached to a massless string of small distance where Ta is time period when the particle
length and is oscillating in a vertical plane with the other oscillates along the line of springs and Tb is time period
end of the string fixed to a rigid support. The tension in the when the particle oscillates perpendicular to the plane of
string at a certain instant is T = k mg. Select the correct the figure. Neglect effects of gravity.
statement.
/////////////

d d
////////////

(a) k can never be greater than 1


k m k
(b) k can never be less than 1
(c) k can never be equal to 1 (a) 1/2 (b) 1/4
(d) k can never be greater than 3 (c) 1/8 (d) 1/3

MARK YOUR 34. 35. 36. 37. 38.


RESPONSE 39. 40. 41. 42.
SIMPLE HARMONIC MOTION 331

43. A small ball of density 4 0 is released from rest just below


the surface of a liquid. The density of liquid increases with
depth as = 0 (1 + ay) where a = 2m–1 is a constant. Find
the time period of its oscillation. (Neglect the viscosity
effects).
2
(a) sec (b) sec
5 5

3
(c) sec (d) sec
2 5 2 5
44. A simple pendulum has time period T1 . The point of
suspension is now moved upward according to the relation
y = Kt2, (K = 1 m/s2) where y is the vertical displacement.
T12
The time period now becomes T2. The ratio of is
T22
1 k1b 2 + k2 2
1 2k1b 2 + k2 2
(g = 10 m/s2) (a) (b)
2 m 2 m
(a) 5/6 (b) 6/5
(c) 1 (d) 4/5
45. A rod of mass m and length is connected by two springs 1 k1b 2 + k2 2
1 3 ( k1b 2 + k 2 2
)
(c) (d)
of spring constants k1 and k2, so that it is horizontal at 2 2m 2 m
equilibrium. What is the natural frequency of the system?

MARK YOUR
43. 44. 45.
RESPONSE

a R b
PASSAGE-1 P Q

A wire of negligible mass and length stretched between two


fixed points P and Q. The wire suffers from a tension T when a
2
1
mass m is attached to the wire. Distance of this mass from fix point
S
P is a and from Q is b. The attachement of mass to the stretched
wire sets it into oscillations of small amplitude. Since the amplitude 1. Force exerted while trying to bring back the mass attached
to the wire from the pulled distance x is given by
is small, the transverse movement of R is small so the change in
length of the wire can be ignored. Thus tension in the wire can be 1 ab ( a + b)
(a) xT (b) 2 xT
taken as practically same T itself. The linear vibratory motion of 2 ( a + b) a ab
the mass can be given by T = 2 / , where is a constant of SHM. ( a + b) ab
Now answer the following questions. (c) xT (d) xT
ab ( a + b) a

MARK YOUR
1.
RESPONSE
332 IIT-JEE PHYSICS Challenger
2. Value of constant of SHM of mass m is PASSAGE-3

1 ab T 2(a + b) T
(a) (b)
2 ( a + b) a m ab m Two identical blocks P and Q have mass m each. They are attached
to two identical springs (of spring constant k) initially unstretched.
ab T (a + b) T Both the blocks are initially in contact as shown. Now the left
(c) (d)
( a + b) a m ab m spring (attached with block P) is compressed by A/2 and the right
spring (attached with block Q) is compressed by A. Both the blocks
3. Let the mass m be tied to end Q removed from the fixture and
are then released simultaneously.
pulled by disetance x horizontally. The time period of mass
of string is considered to be elastic of constant is
A/2 A

/////////////

/////////////
m m P Q
(a) 2 (b) 2
2
///////////////////////////////////////////////
Smooth horizontal floor
2m m
(c) 2 (d) 2
–1 7. The speed of block P just before P and Q are about to
collide for the first time is

PASSAGE-2 k A k
(a) (b) A
m 2 m
A 2 kg block hangs without vibrating at the bottom end of a
spring with a force constant of 800 N/m. The top end of the spring k
is attached to the ceiling of an elevator car. The car is rising with (c) A (d) None of these
2m
an upward acceleration of 10 m/s2 . When the acceleration
suddenly ceases at time t = 0, the car moves upward with constant 8. The speed of block Q just before P and Q are about to
speed. (g = 10 m/s2) collide for the first time is
4. What is the angular frequency of oscillation of the block
k A k
after the acceleration ceases ? (a) (b) A
m 2 m
(a) 10 2 rad / s (b) 20 rad/s
k
(c) 20 2 rad / s (d) 32 rad/s (c) A (d) None of these
2m
5. The amplitude of the oscillation is
9. After what time when they were released from rest, shall the
(a) 7.5 cm (b) 5 cm
blocks collide for the first time ?
(c) 2.5 cm (d) 1 cm
6. The initial phase angle observed by a rider in the elevator, m m
taking downward direction to be positive and positive (a) (b)
2 k k
extreme position to have /2 phase constant, is equal to
(a) zero (b) /2 rad m
(c) rad (d) 3 /2 rad (c) (d) None of these
3 k

MARK YOUR 2. 3. 4. 5. 6.
RESPONSE 7. 8. 9.
SIMPLE HARMONIC MOTION 333

1. Statement - 1 : In simple harmonic motion, the velocity is Statement - 2 : The time period of a pendulum of length L
maximum when the acceleration is
L
minimum. near the surface of the earth is 2 , if L
g
Statement - 2 : Displacement and velocity of SHM differ
is reasonably small.
in phase by . 6. Statement - 1 : The combination of y = sin t + cos 2 t is
2 not a simple harmonic function even
2. Statement - 1: In simple harmonic motion, the motion is though it is periodic.
to and fro and periodic. Statement - 2 : All periodic functions satisfy the relation

d2y
Statement - 2 : Velocity of the particle (v) k2 x2 ky.
dt 2
(where x is the displacement).
7. Statement - 1 : A particle is moving along x-axis. The
3. Statement - 1 : The time-period of pendulum, on a satellite
resultant force F acting on it is given by F
orbiting the earth is infinity.
= – x – 3, where a and b are both positive
Statement - 2 : Time-period of a pendulum is inversely constants. The motion of this particle is
not SHM.
proportional to g.
Statement - 2 : In SHM resultant force must be
4. Statement - 1 : The amplitude of an oscillating pendulum proportional to the displacement from mean
decreases gradually with time. position.
Statement - 2 : The frequency of the pendulum decreases 8. Statement - 1 : For a particle performing SHM, its speed
with time. decreases as it goes away from the mean
5. Statement - 1 : The time period of a pendulum of infinite position.
length whose bob hangs near the surface Statement - 2 : In SHM, the acceleration is always
of the earth will be infinite. opposite to the velocity of the particle.

MARK YOUR 1. 2. 3. 4. 5.
RESPONSE 6. 7. 8.
334 IIT-JEE PHYSICS Challenger

1. The energy of a particular executing simple harmonic motion


is given by E = Ax2 + Bv2 where x is the displacement from (a) 2 + sin 1
÷ (b) 2 sin 1
÷
mean position x = 0 and v is the velocity of the particle at x g 2 g
then choose the correct statement(s)
1 1
2E
(c) 2 cos ÷ (d) 2 cos ÷
g g
(a) amplitude of S.H.M. is
A
5. A block of mass m is kept on a horizontal platform of mass
E M. The platform is doing SHM in horizontal plane with
(b) maximum velocity of the particle during S.H.M. is angular frequency . There is no slipping between the block
B
and the platform due to friction. Then
B
(c) Time period of motion is 2
A m

(d) displacement of the particle is proportional to the M


velocity of the particle. (a) the friction force on the block is directly proportional
2. A particle executes simple harmonic motion with a frequency. to the displacement of the platform from mean position.
f. The frequency with which its kinetic energy oscillates is (b) the contact force on the block is directly proportional
(a) f/2 (b) f to the displacement of the platform from mean position.
(c) 2f (d) 4f (c) the net contact force on the block is directly
3. A linear harmonic oscillator of force constant 2 × 106 N/m proportional to mass of block (m)
and amplitude 0.01 m has a total mechanical energy of 160 J. (d) the net contact force on the platform due to the block
Its is directly proportional to mass of plank (M)
(a) maximum potential energy is 100 J 6. A highly rigid cubical block A of small mass M and side L is
(b) maximum kinetic energy is 100 J fixed rigidly on to another cubical block B of the same
(c) maximum potential energy is 160 J dimensions and of low modulus of rigidity such that the
(d) maximum potential energy is zero lower face of A completely covers the upper face of B. The
4. A ball is hung vertically by a thread of length ‘ ’ from a lower face of B is rigidly held on a horizontal surface. A
point P of an inclined wall that makes an angle with the small force F is applied perpendicular to one of the sides
vertical. The thread with the ball is then deviated through a faces of A. After the force is withdrawn, block A executes
small angle ( > ) and set free. Assuming the wall to be small oscillations the time period of which is given by
perfectly elastic, the period of such pendulum is –
M
(a) 2 M L (b ) 2
P L

ML M
(c) 2 (d) 2
L
l
7. A particle executes simple harmonic motion between
x = – A and x = + A. The time taken for it to go from 0 to
A/2 is T1 and to go from A/2 to A is T2, then
(a) T1 < T2 (b) T1 > T2
(c) T1 = T2 (d) 2T1 = T2

MARK YOUR 1. 2. 3. 4. 5.
RESPONSE 6. 7.
SIMPLE HARMONIC MOTION 335

8. One end of a long metallic wire of length L is tied to the (a) the time period of motion is 1 sec
ceiling. The other end is tied to a massless spring of spring (b) the time period of motion is 1/7.5 sec
constant k.A mass m hangs freely from the free end of the (c) the maximum acceleration of the particle is 0.04 2 m/s2
spring. The area of cross-section and the Young’s modulus (d) the force acting on the particle is zero when the
of the wire are A and Y respectively. If the mass is slightly displacement is 0.05m
pulled down and released, it will oscillate with a time period
11. Three simple harmonic motions in the same direction having
T equal to:
the same amplitude a and same period are superposed. If
each differs in phase from the next by 45°, then.
m(YA + kL)
(a) 2 (m / k )1/ 2 (b) 2
YAk (a) the resultant amplitude is (1 + 2)a

(b) the phase of the resultant motion relative to the first is


(c) 2 [(mYA / kL)1/ 2 (d) 2 [(mL / YA)1/ 2
90°
9. A particle of mass m is executing oscillations about the (c) the energy associated with the resulting motion is
origin on the x axis. Its potential energy is V(x) = k | x |3
(3 + 2 2 ) times the energy associated with any single
where k is a positive constant. If the amplitude of oscillation
is a, then its time period T is motion
(d) the resulting motion is not simple harmonic.
(a) proportional to 1/ a (b) independent of a
12. The function x A sin 2 t B cos 2 t C sin t cos t
(c) proportional to (d) proportional to a 32
a represent SHM for which of the option(s)
10. The displacement (in m) of a particle of mass 100g from its (a) for all value of A, B and C (C 0)
equilibrium position is given by the equation :
(b) A = B, C = 2B
y = 0.01 sin 2 (t + 0.4)
(c) A = –B, C = 2B
(d) A = B, C = 0

MARK YOUR
8. 9. 10. 11. ` 12.
RESPONSE

1. Figure shows a bob of mass m suspended using a rigid rod at C, performing SHM. Gravitational potential energy of bob is
assumed zero at lowest point. Which quantity will change if

k k
A B

m P.E.= 0
336 IIT-JEE PHYSICS Challenger
Column-I Column -II
(A) spring A is cut at (p) Amplitude
extreme position
(B) mass m is added to the (q) Angular frequency
bob at mean position
(C) g is doubled at mean (r) Maximum value of K.E.
position
(D) g is doubled at extreme (s) Maximum value of P.E.
position.
2. For a particle executing SHM along a straight line, match the statements in column I with statement in column II.
(Note that displacement given is column I is to be measured from mean position).
Column I Column II
(A) Velocity-time graph will be (p) straight line
(B) Acceleration-velocity graph may be (q) circle
(C) Acceleration-displacement graph will be (r) ellipse
(D) Acceleration-time graph will be (s) sinusoidal curve
3. In the column I, a system is described in each option and corresponding time period is given in the column II. Suitably match
them.
Column I Column II
2
(A) A simple pendulum of length 3 oscillating with small (p) T 2
3g
amplitude in a lift moving down with retardation g/2.

(B) Two springs of force constants k and 2k are connected to (q) T 2


g
a mass as shown below

m
k 2k
2
(C) The time period of small oscillation of a uniform rod (r) T 2
g
of length smoothly hinged at one end. The rod
oscillates in vertical plane.

(D) A cubical block of edge 2 and specific density /2 is in (s) T m


2
3k
equilibrium with some volume inside water filled in a large
fixed container. Neglect viscous forces and surface tension.
The time period of small oscillations of the block in vertical
direction.

1. 2. 3.

MARK YOUR
RESPONSE
SIMPLE HARMONIC MOTION 337

4. A simple harmonic oscillator consists of a block attached to a spring with k = 200 N/m. The block slides on a frictionless horizontal
surface, with equilibrium point x = 0. A graph of block’s velocity v as a function of time t is shown. Correctly match the required
information in the Column I with the values given in the Column II. (use 2 = 10)
V(m/s)

0 0.10 0.20
t (s)

Column I Column II
(A) The block’s mass in kg (p) – 0.20
(B) The block’s displacement at t = 0 in metres (q) – 200
(C) The block’s acceleration at t = 0.10s in m/s2 (r) 0.20
(D) The block’s maximum kinetic energy in joule. (s) 4.0
5. Two blocks A and B of mass m and 2m connected by a light spring of spring constant k lie at rest on a fixed smooth horizontal
surface. Initially the spring is unstressed. Now at time t = 0 both the blocks are imparted horizontal velocities towards each other
of magnitudes 2u and u as shown in figure. In the subsequent motion, the only horizontal force acting on blocks is due to spring.
Match the conditions in column I with the instants of time they occur as given in column II.
2u u
k
m A B 2m
/////////////////////////////////////////////////////////////////////////////////
Smooth fixed horizontal surface
Column I Column II
2m
(A) The length of spring is least at time (p) t=
2 3k

2m
(B) The length of spring is maximum at time (q) t
3k

3m
(C) The acceleration of both blocks is zero simultaneously at time (r) t
2k

3m
(s) t=
2 2k

4. 5. p q r s
A p q r s
MARK YOUR
B p q r s
RESPONSE
C p q r s
338 IIT-JEE PHYSICS Challenger
6. Column I describes some situations in which a small object moves. Column II describes some characteristics of these motions.
Match the situations in Column I with the characteristics in Column II and indicate your answer by darkening appropriate
bubbles in 4 × 4 matrix given in the ORS.
Column I Column II
(A) The object moves on the x -axis under a (p) The object executes a simple harmonic motion.
conservative force in such a way that its
''speed " and position'' satisfy v c1 c2 x 2
where c1 and c2 are positive constants.
(B) The object moves on the x- axis in such a way that (q) The object does not change its direction.
its velocity and its displacement from the origin
satisfy v = – kx,where k is a positive constant.
(C) The object is attached to one end of a mass-less (r) The kinetic energy of the object keeps on
spring of a given spring constant. The other decreasing.
end of the spring is attached to the ceiling of an
elevator. Initially everything is at rest. The elevator
starts going upwards with a constant acceleration a.
The motion of the object is observed from the elevator
during the period it maintains this acceleration.
(D) The object is projected from the earth's surface (s) The object can change its direction only once.
vertically upwards with a speed 2 GM e / R e ,
where, Me is the mass of the earth and Re is the
radius of the earth, Neglect forces from objects other
than the earth.

6.

MARK YOUR
RESPONSE
SIMPLE HARMONIC MOTION 339

1. In the figure shown, a plate of mass 60gm is at rest and in 2. A particle is moving on x-axis has potential energy
equilibrium. A particle of mass m = 30gm is released from U = 2 – 20x + 5x2 joules along x-axis. The particle is released
at x = – 3. If the mass of the particle is 0.1 kg, then find the
4.5 mg
height from the plate. The particle sticks to the plate. maximum velocity of the particle (in m/s).
k
3. A horizontal platform executes S.H.M. to and fro about the
Neglecting the duration of collision, find time from the
mean position. It's time period of oscillation is 2 seconds.
collision of the particle and the
A mass m is resting on it. Determine the greatest amplitude
(in m) the platform can have so that the mass resting on it
may not leave it ?
m
4.5mg
k 4. If two SHMs are represented by y1 10sin 4 t + ÷ and
2
2m
y2 5sin [2 t 8 cos 2 t ]. Find the ratio of their
amplitudes.
k
5. A mass M attached to a spring oscillates with a period of 2
sec.If the mass is increased by 2 kg, the period increases by
one second. Find the initial mass M (in kg) assuming that
Hooke's law is obeyed.
plate to the moment when the spring has maximum
compression. Spring has force constant 100 N/m. Calculate
value of time in the form x ms (millisecond).

1. 2. 3. 4. 5.

MARK
YOUR
RESPONSE
340 IIT-JEE PHYSICS Challenger
6. Two identical balls A and B each of mass 0.1 kg, are attached 8. What fraction of the total energy is kinetic energy when the
to two identical massless springs. The spring-mass system displacement is one half of the amplitude?
is constrained to move inside a rigid smooth pipe bent in 9. A point particle of mass 0.1 kg is executing S.H.M. of amplitude
the form of a circle as shown in Fig. The pipe is fixed in a of 0.1 m. When the particle passes through the mean position,
horizontal plane. The centres of the balls can move in a its kinetic energy is 8 × 10–3 joule. Obtain the positive value
circle of radius 0.06 meter. Each spring has a natural length of angular frequency (in rad/s).
of 0.06 meter and spring constant 0.1 N/m. Initially, both 10. A small spherical steel ball is placed a little away from the
the balls are displaced by an angle = / 6 radian with centre of a large concave mirror whose radius of curvature R
respect to the diameter PQ of the circle (as shown in Fig.) = 2.5 cm. When the ball is released, it begins to oscillate
and released from rest. Calculate the frequency (in s–1) of about the centre. Find the period (in second) of motion.
oscillation of ball B. Neglect friction and take g = 10 m/sec2.
11. A sphere of mass M and radius R is on a smooth fixed inclined
plane in equilibrium as shown in the figure. If now the sphere

A B is displaced through a small distance along the plane, what


6m will be the angular frequency (in radian/sec) of the resulting
/6 0.0 /6
P Q SHM? (Given k = 4M/3)

k
7. A spring is stretched by 0.20 metre when a mass of 0.50 kg is
suspended. Calculate the period (in sec) of the spring when 30°
M
a mass of 0.25 kg is suspended and put to oscillation 30°
(g = 10 m/sec2).

6. 7. 8. 9. 10. 11.

MARK
YOUR
RESPONSE
SIMPLE HARMONIC MOTION 341

1 (b) 9 (d) 17 (a) 25 (d) 33 (b) 41 (d)


2 (c) 10 (d) 18 (c) 26 (a) 34 (a) 42 (a)
3 (d) 11 (a) 19 (b) 27 (c) 35 (a) 43 (a)
4 (a) 12 (a) 20 (b) 28 (c) 36 (c) 44 (b)
5 (a). 13 (c) 21 (b) 29 (c) 37 (a) 45 (d)
6 (b) 14 (b) 22 (b) 30 (c) 38 (d)
7 (d) 15 (a) 23 (c) 31 (d) 39 (c)
8 (d) 16 (a) 24 (b) 32 (c) 40 (a)

1 (c) 3 (a) 5 (c) 7 (a) 9 (a)


2 (d ) 4 (b) 6 (b ) 8 (b )

1 (a) 3 (a) 5 (d ) 7 (d )
2 (b ) 4 (c) 6 (c) 8 (c)

1 (b, c) 4 (a, d) 7 (a, d) 10 (a, c)


2 (c) 5 (a,c) 8 (b) 11 (a, c)
3 (b, c) 6 (d) 9 (a) 12 (a, c, d)

1. A-q, r, s; B-p, q, r, s; C-p, q; D-q, r, s 2. A-s; B-q, r; C-p; D-s


3. A-r; B-s; C-p; D-q 4. A-r; B-p; C-q; D-s
5. A-p; B-r; C-q 6. A-p; B-q, r; C-p; D-q, r

1 20 2 50 3 9.8 4 0.67 5 1.6


6 0.32 7 0.628 8 0.75 9 4 10 3.14
11 1
342 IIT-JEE PHYSICS Challenger

1. (b) Energy of oscillation is E = A4 4. (a) According to question


Kinetic energy of mass at x = x is, F Mg 4 9.8
K = E – U = (A4 – x4) K= = = = 245 N/m
x x 0.16
1
Given, U = K K = 3U m 0.5
3 T= 2 = 2 × 3.14 = 0.283 sec.
K 245
( A4 x4 ) 3 x 4 , A4 x4 = 3x4 5. (a) The displacement of the particle is given by:
1/ 4 x A sin( 2 t ) B sin 2 t
A4 A4
A4 = 4 x 4 , x =
4
, x= ÷ B
4 4 A sin 2 t
(1 cos 2 t )
2
A B B
X =± . ( A sin 2 t cos 2 t ) +
2 2 2
2. (c) Suppose that masses m1 and m2 produce extension x1 This motion represents SHM with an amplitude:
and x2 respectively. Since, k is the spring constant then
B2 B
initially (m1 + m2) g = k (x1 + x2) A2 + , and mean position .
4 2
when mass m1 is removed from the system, we have
m2g = kx2 1
6. (b) y = r cos 2 t ÷ sin(1000t ) = 2(1 + cos t )sin(1000t )
2
g k
x2
=
m2 = 2sin(1000t ) + 2sin(1000t ) cos t
= 2 sin(1000t ) + sin(1001t ) + sin(999t )
2 2
Now angular frequency = 7. (d) v1 = A2 y12
T x2
2
g and v2 = A2 y22 .
or v22 = 2A2 – 2
y 22 ...(1)
k
m2 v12 = 2A2 – 2
y12 ...(2)

3. (d) Both the bodies oscillate in simple harmonic motion, Solving (1) and (2), we get
for which the maximum velocities will be v22 – v12 = 2(
y12 – y22 )
2
v1 = a 1 1 = a1 × ... (i) v22 v12
T1 =
y12 y22
2
v2 = a 2 2 = a2 × ... (ii)
T2
2 y12 y22
T= 2
Given that v1 = v2 v22 v12
2 2 8. (d) We have a = – 2 x
a1 × a2
T1 T2 a 2
or
x
m ma
2 or m 2
a1 T1 k1 k2 x
= = =
a2 T2 m k1 F
2 or m 2
k2 x
SIMPLE HARMONIC MOTION 343

80 2 ax = gsin cos
or 0.01
0.2 ay = g – gsin2 = g(1 – sin2 ) = gcos2
2
= 200, T = = 0.03s a= ax2 + a 2y
200
9. (d) Let us plot the graph of the mathematical equation
= g 2 sin 2 cos2 g 2 cos4
2
x
U (x) = K 1 e
= g cos sin 2 cos 2
dU x2 = g cos
F= = 2kxe
dx
L
T=2
U(x) g cos
Whenever point of suspension is accelerating use

C 1 B
T=2
geff
A
x=0 x

From the graph it is clear that the potential energy is 2


2 t 4 = 1
minimum at x = 0. Therefore, x = 0 is the state of stable
13. (c) v = v0 cos = 1cos
T 2 2
equilibrium. Now if we displace the particle from x = 0
then for displacements the particle tends to regain the
2kx an =
v2
=
1/ 2 ( )
2

=
1
m/s 2
position x = 0 with a force F = 2
. Therefore for small r 10 20
ex
values of x we have F µ x.
14. (b) Particle is starting from rest, i.e. from one of its extreme
1 1 2 2 1 2 2 2 position.
10. (d) m A ÷ m (A y )
2 2 2 As particle moves a distance A/5, we can represent it
A on a circle as shown.
y=
2 4A/5 4
cos = =
We have y = A sin t A 5
A 2 FG 4 IJ
2
= A sin
T
t = cos–1
H 5K
FG 4 IJ
t=
T
8
t = cos–1
H 5K
11. (a) For curve A to coincide with B, the shift in A should be
y
/2 or phase shift = rad. Similarly for others.
12. (a) As shown in the figure, gsin is the pseudo
acceleration applied by the observer in the accelerated
frame 4A/5 A/5
P
x
A
gsin sin Q
gsin
gcos sin

1 FG 4 IJ = T FG 4 IJ
H 5K 2 H 5K
g
t= cos–1 cos–1
344 IIT-JEE PHYSICS Challenger
Alternatively Alternatively
As starts from rest i.e. from extreme position x = A sin
( t+ )
At t = 0; x = A k

=
2 M
A k
A– = A cos t
5
4
= cos t
5
4
t = cos–1 ; Frestoring = 2kx
5
T FG 4 IJ M M
t=
2
cos–1 H 5K T 2
coeff . of x
2
2k
15. (a) The spring is never compressed. Hence spring shall 17. (a) The velocity of a body executing S.H.M. is maximum at
exert least force on the block when the block is at its centre and decreases as the body proceeds to the
topmost position. extremes. Therefore if the time taken for the body to go
from O to A/2 is T1 and to go from A/2 to A is T2 then
obviously
Natural length T1 < T2
of spring Alternatively
Any S.H.M. is given by the equation y = asin t
Extreme position where y is the displacement of the body at any instant
x0 A t. a is the amplitude and is the angular frequency.
When x = 0, t1 = 0
m mean position
t1 = 0
When x = a/2, t2 = /6
Fmax kx0 kA
t2 =
2
6
2
= mg m A mg 4 mA
T2
When x = a, t3 = /2 t3 =
16. (a) Time period 2
Time taken for O to a/2 will be
x
(T ) 2
a t2 – t1 = = T1
6
If we displace the block by a distance ‘x’ upward then
the restoring force will be Time taken for a/2 to a will be
2
2kx t3 – t2 = = = = T2
Frestoring = 2kx a= 2 6 6 3
M
T1 /6 3 1
x xM M = = =
T 2 2 2 T2 /3 6 2
a 2kx 2k
T2 = 2T1
kx
or T1 < T2
18. (c) Let the spring is further extended by y when the
cylinder is given small downward push. Then the
kx restoring forces on the spring are,
SIMPLE HARMONIC MOTION 345

(i) Ky due to elastic properties of spring T


(ii) upthrust = yAdg = weight of liquid displaced

m1 T m2
k(0)=0

y m2 g
From above :
T = m2 g
Total restoring force m2 g
After x , m1 moves towards right till the total
= (K + Adg) y k
= M × a = – (K + Adg) y kinetic energy acquired does not converted to potential
Comparing with a = – 2y we get energy.
21. (b) Maximum tension in the string is at lowest position.
K + Adg
or = Mv 2
M Therefore, T = Mg + . To find the velocity v at
L
1 K + Adg the lowest point of the path, we apply law of
f = =
2 2 M conservation of energy i.e.
19. (b) Springs on the left of the block are in series, hence 1
Mv 2 = Mgh = MgL(1 cos )
their equivalent spring constant is 2

(2k)(2k)
K1 k
2k 2k x L
Springs on the right of the block are in parallel, hence
their equivalent spring constant is h a
k2 = k + 2k = 3k Mv2
Now again both K1 and K2 are in parallel L
Keq = k1 + k2 = k + 3k = 4k Mg
Hence, frequency is
[ h L x, h L L cos ]

1 Keq 1 4k or v2 2gL (1 cos )


f
2 M 2 M
20. (b) After the system is released, m2 moves down. The or v 2gL(1 cos )

extension in the spring becomes


m2 g T = Mg + 2 Mg(1 cos )
( m2 g k x0 ) ,
k
which is the new equilibrium position of the system. T = Mg 1 + 2 2 sin 2 ÷
2
For small ‘x’, restoring force on the system is F = kx
kx 2
a or T = Mg 1 + 4 ÷
m1 m2 2
For (m1 + m2 + spring) system)
[ sin ( / 2) / 2 for small amplitudes ]
x x(m1 m2 ) m1 m2
T 2 2 2
a kx k T = Mg[1 + 2
]
Angular frequency a
From figure =
2 k L
=
T m1 m2 2
a
T = Mg 1 + ÷
F.B.D. of m1 and m2 just after the system is released : L
346 IIT-JEE PHYSICS Challenger
22. (b) The maximum acceleration of the plateform Therefore P.E. is max. The graph I and III represent the
Amax = 2a = 4 2 n2 a = 4 × 9.86 × 4 × 0.5 above characteristics.
= 78.8 m/s2 27. (c) Net dipole moment = 3 q a
Maximum reading of the machine
| 1 – 2 |
m (g + Amax ) 60 (10 + 78.8) =– 3 qa I
2 0
= =
e 10
2
3q a | – | 2
60 88.8
1 2
=I ÷
= = 532.8 kg. 2 0 T
10
23. (c) Restoring torque is given by 2 0I
T 2
3q a | 1 2 |
kx ÷ I
2 28. (c) There is no effect of a and g on time period of spring
pendulum.
x=( /2) M 1 M 3 M
29. (c) T + =
k 2 k 2 k
k
30. (c) x = A sin ( t + )
A 5
t = 0, x = , = , .
2 6 6
m 2 3k 3k dU
k ( ) 31. (d) F = 20 x
2 2 12 m m dx
24. (b) The small block oscillates along the inclined plane with F 20 x
a= = = 20 x
an amplitude A. As a result the centre of mass of the M 1
system undergoes SHM along the horizontal direction: – 20 x = – 2 (x)

xcm =
mA sin t
cos 60º
1 m
A sin t
= 20 = 2 5
m+M 2 m+M
2 5 5
2 f= = = ÷
The acceleration of the C.M. is acm xcm , along 2 2
the horizontal while the net horizontal force is 32. (c) k2, k3 and k4 are in parallel in a way that they all support
= ( M + m) acm , which is equal to the force of friction mass m.
acting on it. k' = k + 2k sin2 = 3k/2
25. (d) F = 2 × S (3 – )g × x k1k ' 3
knet = k + k ' = 5 k
= 4S gx 1

33. (b) A = 0.1m, m = 0.1 kg, KEmax = 18 × 10–3 J, =


4
x 1 2 2 KEmax
x We have, K.Emax = kA k=
2 A2
3
36 10
k= = 3.6 ,
(0.1) 2
m= S + S3 =4 S
k 3.6
4S g g = = = 6 rad/s
= = m 0.1
4 S
26. (a) In S.H.M., at extreme position, P.E. is maximum when Equation is y = 0.1 sin 6t + ÷
t = 0, x = A. 4
i.e., at time t = 0, the particle executing S.H.M. is at its
extreme position.
SIMPLE HARMONIC MOTION 347

34. (a) For a torsion pendulum, T 2 I / K mg


40. (a) It will perform SHM with amplitude A =
In our case the torsion constant k
external torque Now a = 2x
K=
resulting angular displacement /////////////

5Nm
= = 23.9 N.m / rad kx
2 rad
(12 deg)
360 deg ÷
We then have after squaring the equation for T and mg
rearranging,
k mg
T
2
0.5 s
2 amax = 2A = =g
I= K= ÷ (23.9 Nm) = 0.151 kg-m
2 m k
2 2

1 41. (d)
35. (a) Total energy = m
2 2
a T .E. µ m.
2
1 2 1 P
36. (c) E1 = kx , E2 = ky 2 , P'
2 2 A
1 1 1 S max /6
E= k ( x + y )2 = kx 2 + ky 2 + kxy
2 2 2 /6
= E1 + E2 + 2 E1 E2 A
Q' Q
= 2 + 8 + 2 16 = 18J
1 2 75 1
37. (a) m ( A2 x2 ) m 2
A2
2 100 2 Figure shows the mapping of the two SHMs with
circular motions having phase difference
3 2
A2 x2 = A 2
4 t 1= rad
6 3
A
x= The maximum separation between the two particles is
2
Smax = 2 A sin
6

1
30° 30° or Smax 2 5 = 5cm.
2

3d d
42. (a) Initial stretch in both springs = d =
4 4
/////////////

360° 2 sec. d
////////////

2 1
30° 30 = sec.
360 6
38. (d) If it oscillates with maximum amplitude then tension d'
T = 3mg
d d
m1m2 Frestoring = k +x k x÷ = 2kx
39. (c) T 2 = 4 4
2k m1 + m2
m
10 2 5 10 1 Ta 2
T 2 = = =2 2k
7 200 7 7 140
d d sec
348 IIT-JEE PHYSICS Challenger

3d 1 3 d2y
x d sec d ÷ or = 2k ... (i) ( k = 1 m/s2 given)
4 cos 4
dt 2
Force towards equilibrium position (kx' sin )
or a = 2m / s 2
3sin
= dk tan ÷ due to one spring and
4
We know that T = 2
3sin g
net force = 2dk tan ÷
4
T12 g2
3 d =
For small , force = 2dk =k ÷ T22 g1
4 2
d = displacement from mean position T12 12 6
= =
kx 2m Ta 1 T22 10 5
F Tb 2 =
2 k Tb 2 [ g1 = 10 m/s2 and g2 = g + 2 = 12 m/s2]
43. (a) In equilibrium position
m Id 2
mg = FB = 4 = 1 + ay0 45. (d) k1b b cos k2 cos
0 (1 ay0 ) g
dt 2
4 0

3 m 2
y0 == 1.5 Here I = and as is small cos 1
2 3
Now displace it downward by y
m 2d 2
m + (k1b 2 + k2 2 ) =0
F mg 0 [1 + a ( y0 + y )] g 3dt 2
4 0

mg 3 ( k1b 2 + k2 2
)
= mg [1 + ay0 + a y ] =
4 m
mga ag
F y ; Acceleration = ÷ y
4 4

2 4 2
T= =2 = sec.
ag 5
44. (b) y = kt2
1 3 (k1b 2 + k2 2
)
dy and f =
= 2kt 2 m
dt

1. (c) Let mass be pulling the string by x. Force trying to


Comparing with a – 2x
move the mass back
T cos T cos T a+b
1 2
= ÷
m ab
Tx Tx a+b
= + = Tx ÷
a b ab extention x
3. (a) Net restoring force –
original length
Tx a + b
2. (d) Acceleration of mass = – ÷
m ab x
Restoring acceleration = –
m
SIMPLE HARMONIC MOTION 349

Extension of the spring when it is stretched to


x
– 2
x=– = equilibrium line is x'.
m m
mg = kx'

m 2 10
Þ T = 2p x = = 2.5 cm.
800

4. (b), 5. (c), 6. (b) Therefore, amplitude


A = x – x' = 2.5cm.
k If downward direction is taken as positive at t = 0, x =
= = 400
m A.
= 20 rad/s. Using x = A sin ( t + )
Maximum extension in the spring from natural position A = A sin
is x.
Then mg + ma = kx =
2
2 (10 + 10)m 7. (a) Just before collision, both P and Q arrive at their
x= = 5cm.
800 equilibrium position

A k A
///////////////////////// vp =
2 m 2
8. (b) Speed of Q just before collision is

k
vQ A= A
m

Eq. line 9. (a) The blocks shall meet after time t = T/4, where T is time
F=0 period of either isolated spring block system,
A
T 1 m m
v=0 t= = 2 =
4 4 k 2 k

1. (a) At the middle point velocity of the particle under SHM Now, V = a cos t
is maximum but acceleration is zero since displacement
is zero. So statement-1 is true. dv
a= = a 2 sin t
We know that x = a sin t ...(1) dt
where x is displacement & a is amplitude. Now, when velocity is maximum, from equation (ii) it is
dx clear that
Velocity = = a cos t
dt
t+ = t=0
2 2
a cos( t) a sin ( t )÷
If we put the value of wt = 0 in acceleration equation, it
2
become
a sin t+ ÷ ...(2) dv
2 a= = a 2
sin 0 = 0
dt
From (i) & (ii) it is clear that
So, when velocity is maximum, acceleration is minimum
Velocity is ahead of displacement (x) by angle. (zero).
2
350 IIT-JEE PHYSICS Challenger
2. (b) SHM is to and fro motion of an object and it is periodic. 6. (c) For a periodic function to be simple harmonic

v k2 x2 d2 y
y. But given function y = sin t + cos 2 t will
If x = 0, v has maximum value. At x = k, v has minimum dt 2
velocity. Similarly, when x = – k, v has zero value, all
not satisfy the same. So, it is periodic but not simple
these indicate to & fro movement.
harmonic.
3. (a) Time period of pendulum in a satellite is infinity. It means
it may not oscillate as apparent value of g is zero. So,
7. (d) The mean position of the particle in statement-1 is
1 x = –3 and the force is always proportional to
time period being µ µ .
g displacement from this mean position. The particle
executes SHM about this mean position. Hence
4. (c) The amplitude of an oscillating pendulum decreases
statement-1 is false.
with time due to friction of air. In absence of air, its
frequency and amplitude will remain constant. 8. (c) Speed = A2 x2

R as | x | increases speed decreases


5. (d) Time period becomes 2 . We can’t neglect the
g Acceleration is in direction of speed as it comes towards
mean position.
roundness of earth for the pendulum of infinite length.

1. (b, c) Amplitude is obtained for v = 0 P.E. at mean position = (160 – 100) J = 60 J


Max P.E. = (100 + 60 ) J = 160 J.
E
Amplitude =
A
Extreme Mean
Maximum velocity is obtained for x = 0 position position

K.E. = 0 K.E. = 100J


vmax = P. E. = 160J P. E. = 60J
B

vmax amplitude
T0
4. (a, d) T = + 2t = + 2t ..... (1)
A B 2 g
T 2
B A
t time to travel from O to angle ; = sin t
2. (c) During one complete oscillation, the kinetic energy will
become maximum twice. 1 T0
t sin 1
t= sin 1
÷
Therefore the frequency of kinetic energy will be 2f. 2
3. (b, c) The total energy of the oscillator
Putting value of t in eqn (1)
1 2
= kA = Max. K.E.
2
T= 2 + sin 1
÷
g 2
1
= 2 106 (0.01) 2 = 100 J
2
As total mechanical energy = 160 J 1
=2 cos ÷
g
The P.E. at equilibrium position is not zero.
SIMPLE HARMONIC MOTION 351

5. (a, c) From the equations of SHM we can show that


The particle is doing SHM because of the frictional T1 = T0–A/2 = T/12
force only. The contact force on the block is [ A/2 = A sin T1 or T1 = /6]
F = (mg ) + f
2 2
= m g +a = m g +(
2 2 2 2
x) 2 and T2 = TA/2–A = T/6
This is not proportional to x but proportional to m. The So that, T1 + T2 = T0–A = T/4
contact force on the platform is F only and this is 8. (b) Let us consider the wire also as a spring. Then the case
independent of m. becomes that of two spring attached in series. The
6. (d) When a force is applied on cubical block A in the equivalent spring constant is
horizontal direction then the lower block B will get
1 1 1
distorted as shown by dotted lines and A will attain a = +
keq k k '
new position (without distortion as A is a rigid body)
as shown by dotted lines.
where k' is the spring constant of the wire

kk '
keq =
A
k+k'
F
L F/A F L
L Now, Y =
L/L A L
L
L
F YA
B Fixed or =
L L

For cubical block B YA


or, K'=
L
F/A F L F L F
= = 2 = We know that time period of the system
L/L A L L L L L

F= L L m m(k + k ')
T= 2 2
L is a constant keq kk '
Force F µ L and directed towards the mean
position, oscillation will be simple harmonic in m k + YA / L m(kL + YA)
nature. T= 2 2
k YA / L kYA
Here, M 2= L
9. (a) U (x) = k | x |3
L
=
M [U ] ML2T 2
[k] = = = ML 1T 2
[ x3 ] L3
2 L
= Now time period may depend on T µ (mass) x
T M
(amplitude)y (k)z
[M0L0T] = [M]x [L]y [ML–1 T–2]z
M
T= 2 = [Mx + z Ly – z T – 2z]
L
Equating the powers, we get
7. (a, d)
– 2z = 1 or z = – 1/2
In SHM, velocity of particle also oscillates simple
y–z=0
harmonically. Speed is more near the mean position
or y = z = – 1/2
and less near the extreme positions. Therefore, the time
taken for the particle to go from 0 to A/2 will be less Hence T µ (amplitude) –1/2 µ a–1/2
than the time taken to go it from A/2 to A or T1 < T2
352 IIT-JEE PHYSICS Challenger
12. (a, c, d)
1
or Tµ The given equation is
a x = A sin2 t + B cos2 t + C sin t cos t
10. (a, c) From given equation, = 2 , A = 0.01m, Rearranging the equation in a meaningful form (for
interpretation of SHM)
2
T= = 1 sec. x=
A
(2 sin 2 t )
B
(2 cos 2 t )
C
(2sin t cos t )
2 2 2
amax = 2A = (2 )2 × 0.01 = 0.04 2 m/s2 A B C
= [1 cos 2 t ] + [1 + cos 2 t ] [sin 2 t ]
11. (a, c) 2 2 2
From superposition principle C
(a) For A = 0 and B = 0, x = sin (2 t )
y = y1 + y2 + y3 2
= a sin t + a sin ( t + 45°) + a sin ( t + 90°) The above equation is that of SHM with
= a[sin t + sin ( t + 90°] + a sin ( t + 45°) C
amplitude and angular frequency 2 . Thus
= 2a sin ( t + 45°) cos 45° + a sin ( t + 45°) 2
option (a) is correct.
= ( 2 + 1 ) a sin ( t + 45°) (b) If A = B and C = 2B then x = B + B sin 2 t
= A sin ( t + 45°) This is equation of SHM. The mean position of
Therefore resultant motion is simple harmonic of the particle executing SHM is not at the origin.
Option (b) is correct. [x = B = x' = b sin 2 t]
amplitude A = ( 2 + 1 ) a (c) A = – B, C = 2B; Therefore
and which differ in phase by 45° relative to the first. x = B cos 2 t + B sin 2 t
Let B = X cos = X sin then
1
Energy in SHM µ (amplitude)2 E mA2 2 x = X sin 2 t cos + X cos 2 t sin
2
This represents equation of SHM.
(d) A = B, C = 0 and x = A. This equation does not
2
Eresultant A represents SHM.
= ÷ = ( 2 + 1) = (3 + 2 2)
2
Esingle a

Eresultant = (3 + 2 2) Esingle

1. A-q, r, s; B-p, q, r, s; C-p, q; D-q, r, s 3. A-r; B-s; C-p; D-q


2. A-s; B-q, r; C-p; D-s (A) In frame of lift effective acceleration due to gravity is
v = A cos ( t + ),
g 3g
The velocity-time graph is (sinusoidal) g+ = downwards.
2 2
Relation between acceleration velocity is

v2 a2 2 (3 )
2
+ 4
= A2 which may be circle if = 1 and ellipse T 2
3g
if 1.
Acceleration-displacement graph is a straight line and m m m
(B) T 2 2 2 .
acceleration time graph is sinusoidal. keff k + 2k 3k
SIMPLE HARMONIC MOTION 353

reaches position 3 and travels an angle


2
m
I 2 3 3 3 2m 3m
(C) T 2 2 3 2 t= = =
mgd 3g 2 2 2 3k 2k
mg
2
(C) Acceleration for both is zero when both pass their
respective mean positions i.e. phase of B
m ( / 2) A (2 ) reaches position 2 and hence travels angle
(D) T 2 2 2
Ag Ag g
2m
4. A-r; B-p; C-q; D-s t= =
3k
vm 2
vm = A A= = (0.2) = 0.20m 6. A–p ; B–q, r ; C–p ; D–q, r
2
A p
( it is clear from graph that T = 0.20 sec)
2
Reason : For a simple harmonic motion v = a x 2 . On
2
m T k
T 2 m= 2
= 0.2kg
k 4 comparing it with v = c1 c2 x 2 we find the two
At t = 0.1 sec, acc. is maximum = – 2A = – 200 m/s2 comparable.
1 2 B q, r
Maximum kinetic energy = mvm = 4 J
2 Reason : v = – k x
5. A-p; B-r; C-q when x is positive; v is – ve, and as x decreases, v decreases.
Therefore kinetic energy will decreases. When x = 0, v = 0.
Therefore the object does not change its direction.
2 0/3 0/3 When x is negative, v is positive. But as x decreases in
magnitude, v also decreases. Therefore kinetic energy
m A 2m decreases. When x = 0, v = 0. Therefore the object does not
B
change its direction.
4
C p

1 Reason : When a = 0, let the spring have an extension x.


3
Then k x = mg.
When the elevator starts going upwards with a constant
2
acceleration, as seen by the observer in the elevator, the
object is at rest.
ma + mg = k x'
2u m u 2 m
vCM = =0 ma = k (x' – x) (Since a is constant)
3m
D q, r
k k .3m 3k
= = = The speed is 2 times the escape speed. Therefore the
m 2m 2m
object will leave the earth. It will therefore not change the
(A) Spring is maximum compressed when phase of block B direction and its kinetic energy will keep on decreasing.
reaches position 1, and travels an angle

2m
t= =
2 2 2 3k

(B) Spring is maximum elongated when phase of block B


354 IIT-JEE PHYSICS Challenger

1. 20. Using values, t = 20 ms


Velocity of the particle just before collision x = 20
2. 50.
4.5mg m
u 2g u 3g F = 20 – 10x = – 10 (x – 2)
k k
Hence, force constant is k = 10 ; m = 0.1 kg
Now it collides with the plate.
Now just after collision velocity (v) of system of “plate + k
particle” Angular frequency, 10 rad/s
m
u m
mu = 3mv v= =g vmax A 50 m/s
3 k
3. 9.8
When the platform reaches at the highest point, the
v
acceleration is maximum which acts towards the mean
mg/k position. At highest point, R – mg = – ma i.e., R = mg – ma
t y=0
The maximum acceleration of platform
a = 2A (A = amplitude)
R = mg – m 2A

For the object to rest on the platform, g should be less than


3m a.
Now system performs SHM with time period T 2
k The maximum value of a will give R = 0.
0 = mg – m 2A
mg
and mean position as distance below the point of
k g 9.8 T 2 9.8 4 2

collision. A= 2 = 2 = 2 = 9.8 metre.


(4 ) 4
Let the equation of motion, y A sin( t )
4. 0.67
mg
For t = 0, y = mg/K ; A sin .......... (1)
k For the equation, y1 10sin 4 t + ÷
2
Now for amplitude, v A2 x2 The amplitude a1 = 10 units and for the equation

m k m2g 2 y2 5sin [2 t 8 cos 2 t ]


g A2
k 3m k2
Multiplying and dividing by 1 + 8 = 3
2
mg m2g 2 2 mg
3 ÷ A2 2
A= .......... (2) 1 8
k k k y2 15 sin (2 t ) cos (2 t )
3 3
A T
By eq. (1) and (2), x to x = 0 t=
2 12 = 15 sin (2 t ) where tan = 8

T whose amplitude a2 is obviously 15 units.


x = 0 to x = A t ;
4 a1 10 2
The required ratio = = = = 0.67
a2 15 3
T T 2 3m
Total time =
12 4 3 k
SIMPLE HARMONIC MOTION 355

5. 1.6 Comparing it with = C

M 4k C 4k
We know that T = 2 ÷, We get C = I =
k m I m

where k = spring constant. We know that in rotatory motion

1 C 1 4k 1
M v= = = s 1
= 0.32 s–1.
In first case, 2= 2 ÷ ... (1) 2 I 2 m
k
7. 0.628

M +2 The force constant k of the spring is given by


In second case, 3 = 2 ÷ ... (2)
k F 0.5 kg Wt 0.5 10 Newton
k= = = = 25 Newton/metre
y 0.20 m 0.20 m
Squaring of eq. (1) and (2) and then dividing (2) by (1), we
have
m 0.25
9 M +2 2 Now T 2 ÷ 2 3.14 ÷ = 0.628 second.
= = 1+ k 25
4 M M
8. 0.75
Solving we get M = 1.6 kg.
We know that
6. 0.32
Let during oscillation at any instant of time, the balls are in 1 2
Total energy E m a2 ;
such a position that the radius vector makes an angle 2
with the diameter PQ as shown. The compression of top
1 2
spring is 2R and the elongation of bottom spring is 2R . Kinetic energy T m (a 2 y2 )
2
The force acting on ball B is due to both the springs. The
total force acting tangentially is :
1
m 2 (a 2 y 2 )
This force creates a tangential acceleration at Kinetic energy 2 a 2 y2 y2
= = =1
Total energy 1 a2 a2
m 2 a2
2

B A
K.E. (a / 2) 2 3
When y = a/2, 1 = = 0.75.
R Total Energy a 2 4

R R Q
9. 4
The displacement of a particle in S.H.M. is given by :
y = a sin ( t + f )

dy
velocity = = a cos ( t + f )
dt
4KR The velocity is maximum when the particle passes through
Ft = mat at =
m the mean position i.e.
But at = Ra dy
÷
4k dt max = a
= = angular acceleration
m The kinetic energy at this instant is given by
The torque acting on B
2
1 dy 1
I 4k m ÷ = m 2 a2 = 8 × 10–3 joule
=I = 2 dt max 2
m
356 IIT-JEE PHYSICS Challenger
11. 1
1
or × (0.1) 2 × (0.1)2 = 8 × 10–3 Since, the sphere is moved down through a distance x, then
2
elongation in the spring = x cos 30°
Solving we get =±4
Restoring force = k x cos230° = 3 k x/4
Positive vaolue of = 4 rad/s.
10. 3.14 d 2x –3
M 2
= kx
F = – mg sin = – mg ( is small) dt 4
= – mg (x/R)
d2x æ 3k ö÷
Þ = -çç
çè 4M ø÷÷
F = – kx, where k = (mg/R) x
2x 2
Now,, a = F/m = – kx/m = – dt
Motion is simple harmonic motion.
d2x 2
But x
2 R 2.5 dt 2
T= =2

= 2 × 3.14 × ÷ = 3.14 sec.
10
3k
= = 1 rad/s
4M
1. Two sound sources emitting sound each of wavelength 7. A train approaching a hill at a speed of 40 km/hour sounds
are fixed at points A and B. A listener moves with velocity u a whistle of frequency 580 Hz when it is at a distance of 1 km
from A to B. The number of beats heard by him per second is from the hill. Wind is blowing in the direction of the train
(a) 2u/ (b) u/ (c) u/3 (d) 2 /u with a speed of 40 km/h. Find the frequency of the whistle
2. If the fundamental frequency of a vibrating organ pipe is heard by an observer on the hill: (Velocity of sound in
200 Hz, then air = 1200 km/h)
(a) the first overtone is 200 Hz (a) 585 Hz (b) 575 Hz (c) 599 Hz (d) 589 Hz
(b) the first overtone may be 400 Hz 8. A siren placed at a railway platform is emitting sound of
(c) the first overtone is 300 Hz frequency 5kHz. A passenger sitting in a moving train A
(d) none of the above records a frequency of 5.5 kHz while the train approaches
3. Calculate the velocity of sound in a mixture of oxygen, the siren. During his return journey in a different train B be
nitrogen and argon at 0°C. The mixture consists of the gases records a frequency of 6.0 kHz while approaching the same
oxygen, nitrogen and argon in the mass ratio 2 : 7 : 1. (Given siren. The ratio of the velocity of train B to that of train A is
R = 8.3 J mol–1 K–1. Ratio of specific heats of the gases are (a) 242/252 (b) 2 (c) 5/6 (d) 11/6
argon 1.67, oxygen 1.4, nitrogen 1.4. The molecular weights 9. A note is produced when you blow air across the top of a
of the respective gases are 40, 32 and 28.) test tube. Two students were asked about the effect of
(a) 329.5 m/s (b) 219.0 m/s blowing harder.
(c) 422.0 m/s (d) 380.2 m/s
Student A : The pitch of sound would increase.
4. A listener moves towards a source with a speed of 10 ms–1.
Student B : The intensity of sound would increase.
If the source emits a frequency 200 Hz and velocity of sound
(a) A is correct, B is wrong
in air is 332 ms–1, the wavelength of the note received by the
(b) B is correct, A is wrong
listener is
(c) Both are correct
(a) 1.685 m (b) 1.71 m (c) 1.66 m (d) 2 m
(d) Both are wrong
5. A cylindrical hose open at both ends has a fundamental
frequency f in air. The hose is dipped vertically in water so 10. An object of specific gravity is hung from a thin steel wire.
that half of it is in water. The fundamental frequency of the The fundamental frequency for transverse standing waves
air column in now in the wire is 300 Hz. The object is immersed in water so that
(a) f / 2 (b) 3f / 4 (c) f (d) 2f one half of its volume is submerged. The new fundamental
frequency in Hz is
6. A wave represented by the equation y = a cos (kx t ) is
1/ 2 1/ 2
superposed with another wave to form a stationary wave 2 1 2
300 ÷÷ 300 ÷

such that point x = 0 is a node. The equation for the other (a) (b)
2 2
wave is
(a) a sin (kx t) (b) – a cos (kx t) 2 2 1
300 ÷ 300 ÷÷

(c) (d)
(c) – a cos (kx t) (d) – a sin ( kx t) 2 2

MARK YOUR 1. 2. 3. 4. 5.
RESPONSE 6. 7. 8. 9. 10.
358 IIT-JEE PHYSICS Challenger
11. Two vibrating strings of the same material but length L and 17. A source emitting sound of frequency f0 is moving in a
2L have radii 2r and r respectively. They are stretched under circle of radius R, having centre at the origin, with a uniform
the same tension. Both the strings vibrate in their speed = c/3, where c = speed of sound. Find the maximum
and minimum frequencies heard by a stationary listener at
fundamental modes, the one of length L with frequency f1 the point (R/2, 0).
and the other with frequency f 2 . The ratio f1 / f 2 is given 6 f0 6 f0 2 3 f0 2 3 f0
(a) , (b) ,
by 5 7 2 3 1 2 3 +1
(a) 2 (b) 4 (c) 8 (d) 1
3 f 0 3 f0
12. A wave disturbance in a medium is described by (c) , (d) None of these
2 5
18. A transverse wave in a medium is described by the equation
y ( x, t ) 0.02 cos 50 t ÷ cos(10 x) where x and y are y = A sin2( t – kx). The magnitude of the maximum velocity
2
of particles in the medium is equal to that of the wave
in metre and t is in second velocity, if the value of A is
(a) A node occurs at x = 0.15 m (a) /2 (b) /4
(b) An antinode occurs at x = 0.3 m (c) / (d) 2 /
(c) The speed of wave is 5 ms–1 19. A complex wave is represented by an expression of the form:
(d) The wavelength of wave is 0.3 m 1 2
13. String # 1 is connected with string # 2. The mass per unit y(t) = 1. sin x .sin 3 x , where = is the angular
2 T
length in string # 1 is 1 and the mass per unit length in frequency and T is the period of the wave. Which of the
string # 2 is 4 1. The tension in the strings is T. A travelling following sketches best represents the wave?
wave is coming from the left. What fraction of the energy in
the incident wave goes into string # 2 ? y(t)
incident (a)
2 =4 1
1

(a) 1/8 (b) 4/9 (c) 2/3 (d) 8/9


14. An open pipe is suddenly closed at one end with the result
that the frequency of third harmonic of the closed pipe is
y(t)
found to be higher by 100 Hz than the fundamental
(b)
frequency of the open pipe. The fundamental frequency of
the open pipe is
(a) 200 Hz (b) 300 Hz
(c) 240 Hz (d) 480 Hz
15. The extension in a string, obeying Hooke’s law, is x. The
y(t)
speed of sound in the stretched string is v. If the extension
(c)
in the string is increased to 1.5x, the speed of sound will be
(a) 1.22v (b) 0.61v
(c) 1.50v (d) 0.75v
2
16. The equation y = a cos (2 nt – 2 x/ ) represents a wave
with : y(t)
(a) amplitude a, frequency n and wavelength (d)
(b) amplitude a, frequency 2n and wavelength 2
(c) amplitude a/2, frequency 2n and wavelength
(d) amplitude a/2, frequency 2n and wavelength /2

MARK YOUR 11. 12. 13. 14. 15.


RESPONSE 16. 17. 18. 19.
WAVES 359

20. A whistle giving out 450 Hz approaches a stationary 27. The amplitude of a wave disturbance propagating in the
observer at a speed of 33 m/s. The frequency heard by the
1
observer in Hz is positive x–direction is given by y = at t = 0 and
(a) 409 (b) 429 (c) 517 (d) 500 1 + x2
21. Equations of a stationary and a travelling waves are as 1
follows y1 = a sin kx cos t and y2 = a sin ( t – kx). The y= at t = 2s, where x and y are in meter..
2 x2 2x
3
phase difference between two points x1 = and x2 = Assuming that the shape of the wave disturbance does not
3k 2k change during the propagation, the speed of the wave is
is 1 in the standing wave (y1) and is 2 in travelling wave (a) 0.5 m/s (b) 1 m/s (c) 1.5 m/s (d) 2 m/s
28. A composite string is made up by joining two strings of
1 different masses per unit length µ and 4µ. The composite
(y2), then ratio is string is under the same tension. A transverse wave pulse Y
2
(a) 1 (b) 5/6 (c) 3/4 (d) 6/7 = (6 mm) sin (5 t + 40 x) , where 't ' is in seconds and 'x' is in
22. A travelling wave in a stretched string is described by the metres, is sent along the lighter string towards the joint.
equation y = A sin (kx – t) The maximum particle velocity is The joint is at x = 0. The equation of the wave pulse reflected
(a) A (b) /k (c) d / dk (d) x / t from the joint is
23. The equation of plane progressive wave motion is (a) Y = (2 mm) sin (5 t – 40 x)
y = a sin2 / (vt – x). Velocity of particle is (b) Y = (4 mm) sin (40 x – 5 t)
(c) Y = – (2 mm) sin (5 t – 40 x)
dv dy dv dy (d) Y = (2 mm) sin (5 t – 10 x)
(a) y (b) v (c) – y (d) – v
dx dx dx dx 29. Two monotomic ideal gases 1 and 2 of molecular masses m1
24. A train moves towards a stationary observer with speed 34 and m2 respectively are enclosed in separate containers
m/s. The train sounds a whistle and its frequency registered kept at the same temperature. The ratio of the speed of sound
by the observer is f1. If the train's speed is reduced to 17 in gas 1 to that in gas 2 is given by
m/s, the frequency registered is f2. If the speed of sound is
m1 m2 m1 m2
340 m/s, then the ratio f1/f2 is (a) (b) (c) (d)
(a) 18/19 (b) 1/2 (c) 2 (d) 19/18 m2 m1 m2 m1
25. Three coherent sonic sources emitting sound of single 30. A sound source S, emitting a particular sound frequency
wavelength ' ' are placed on the x-axis at points can slide along a horizontal rough rod. It is tied with a string
(– 11 6 , 0) , (0 , 0) , ( 11 6 , 0) . The intensity (of total length ) that passes over a pulley. An observer O
is in a lift which is tied to the other end of the string as
reaching a point ( 0 , 5 6) from each source has the same shown in the figure The system is made to move with a
value I0 . Then the resultant intensity at this point due to constant speed. Considering that is not very large, as
the interference of the three waves will be : distance between the pulley and the source S varies from
(a) 6 I0 (b) 7 I0 (c) 4 I0 (d) 5 I0 to zero, the apparaent frequency perceived by the observer
26. Two pulses on the same string are described by the following
wave equations S
5 5
y1 = and y2 = ,
(3 x 4t ) + 2
2 (3 x 4t 6) 2 + 2
where symbols have usual meaning. Choose the incorrect
statement
(a) Pulse y1 and pulse y2 travel along +ve and –ve x-axis O
respectively
(b) At t = 0.75s, y displacement at all points on the string is (a) first increases, then decreases
zero (b) first decreases, then increases
(c) At x = 1m, y displacement is zero for all time (c) continously increases
(d) Energy of string is zero at t = 0.75s (d) continuously decreases

20. 21. 22. 23. 24.


MARK YOUR 25. 26. 27. 28. 29.
RESPONSE
30.
360 IIT-JEE PHYSICS Challenger
31. Two pulses in a stretched string whose centers are initially so that one half of its volume is submerged. The new
8 cm apart are moving towards each other as shown in the fundamental frequency (in Hz) is
figure. The speed of each pulse is 2 cm/s. After 2 seconds,
the total energy of the pulses will be 1 2
(a) 300 1– (b) 300
2 ÷ 2 –1÷

2 2 –1
(c) 300 (d) 300
2 –1÷ 2 ÷
8 cm
36. A sonometer wire resonates with a given tuning fork forming
(a) zero standing waves with five antinodes between the two bridges
(b) purely kinetic when a mass of 9 kg is suspended from the wire. When this
(c) purely potential mass is replaced by a mass M, the wire resonates with the
(d) partly kinetic and partly potential same tuning fork forming three antinodes for the same
32. A stationary observer receives sound wave from two tuning positions of the bridges. The value of M is
forks, one of which approaches and the other recedes with (a) 25 kg (b) 5 kg
the same velocity. As this takes place, the observer hears (c) 12.5 kg (d) 1/25 kg
beats with frequency 2 Hz. What is the velocity of each 37. How many frequencies below 1 kHz of natural oscillations
tuning fork if their oscillation frequency is 0 = 680 Hz and of air column will be produced if a pipe of length 1 m is
the velocity of sound in air is vs = 340 m/s closed at one end ? [velocity of sound in air is 340 m/s]
(a) 5 m/s (b) 0.5 m/s
(a) 5 (b) 6
(c) 15 m /s (d) 50 m/s
(c) 4 (d) 8
33. The temperature of a mono-atomic gas in an uniform
38. A sound source emits frequency of 180 Hz when moving
container of length L varies linearly from T0 to TL as shown
towards a rigid wall with speed 5 m/s and an observer is
in the figure. If the molecular weight of the gas is M0, then
moving away from wall with speed 5 m/s. Both source and
the time taken by a wave pulse in travelling from end A to
observer moves on a straight line which is perpendicular to
end B is
the wall. The number of beats per second heard by the
observer will be [Speed of sound = 355 m/s]
A B (a) 5 beats/s (b) 10 beats/s
(c) 6 beats/s (d) 8 beats/s
T0 TL 39. A police car moving at 22 m/s, chases a motorcyclist. The
L police man sounds his horn at 176 Hz, while both of them
2L 3M 3(TL – T0 ) move towards a stationary siren of frequency 165 Hz.

( )
(a) (b) Calculate the speed of the motorcycle, if it is given that he
TL + T0 5R 5 RM 0 L
does not observes any beats.

L 3M Police Car Motorcycle


M0
( )
(c) (d) L
TL – T0 5R 2 R (TL – T0 )
Stationary
22 m/s
34. The ends of a stretched wire of length L are fixed at x = 0 and V Siren
176 Hz (165 Hz)
x = L. In one experiment, the displacement of the wire is
y1 = A sin ( x/L) sin t and energy is E1 and in another (a) 33m/s (b) 22m/s
experiment its displacement is y2 = A sin (2 x/L) sin 2 t and (c) zero (d) 11m/s
energy is E2. Then 40. A string of length 0.3 m and mass 10 –2 kg is clamped at both
(a) E2 = E1 (b) E2 = 2E1 of its ends. The tension in the string is 1.2 N. When a pulse
(c) E2 = 4E1 (d) E2 = 16 E1 travels along the string, the shape of the string is found to
35. An object of specific gravity is hung from a thin steel be the same at times t and t + t. The minimum value of t is
wire. The fundamental frequency of transverse standing (a) 0.1 sec (b) 0.2 sec
waves in the wire is 300 Hz. The object is immersed in water, (c) 0.3 sec (d) 0.4 sec

MARK YOUR 31. 32. 33. 34. 35.


RESPONSE 36. 37. 38. 39. 40.
WAVES 361

41. A plane progressive simple harmonic sound wave of 47. In a standing wave formed as a result of reflection from a
angular frequency 680 rad/s moves with speed 340 m/ surface, the ratio of the amplitude at an antinode to that at
s in the direction which makes equal angle with each node is x. The fraction of energy that is reflected is
x, y and z-axis. The phase difference ( 1 – 2) between 2 2
the oscillations of the particle in the medium located at x 1 x
(a) (b)
the positions ( 3, 3, 3) and (2 3, 2 3, 2 3) is x x +1
(assume cos > 0) 2 2
(a) 2 radian (b) 3 radian x 1 1
(c) (d)
(c) 4 radian (d) 6 radian x +1 x
42. A pipe of length 1, closed at one end is kept in a chamber 48. An open pipe is in resonance in 2nd harmonic with
of gas of density 1. A second pipe open at both ends is frequency f1. Now one end of the tube is closed and
placed in a second chamber of gas of density 2. The frequency is increased to f2 such that the resonance again
compressibility of both the gases is equal. Calculate the occurs in nth harmonic. Choose the correct option
length of the second pipe if frequency of first overtone in 3 5
both the cases is equal (a) n = 3, f 2 = f1 (b) n = 3, f 2 = f1
4 4
4 2 4 1
(a) (b) 3 5
3
1
1 3
1
2 (c) n = 5, f 2 = f1 (d) n = 5, f 2 = f1
4 4
2
49. A string of length L is clamped at each end and vibrates in a
(c) (d) 1
1 1 standing wave pattern. The wavelengths of the constituent
1 2
traveling waves cannot be
43. An organ pipe of 3.9 m long, open at both ends is driven (a) 3L (b) 2L (c) 2L/3 (d) L/2
to third harmonic standing wave. If the amplitude of pressure 50. A loop of a string of mass per unit length µ and radius R is
oscillation is 1 % of mean atmospheric pressure ( 0 = 105 rotated about an axis passing through centre perpendicular
N/m2). The maximum displacement of particle from mean to the plane with an angular velocity . A small disturbance
position will be [given v = velocity of sound = 200 m/s and is created in the loop having the opposite sense of rotation.
The linear speed of the disturbance for a stationary observer
0 density of air 1.3 kg/m3)
is
(a) 2.5 cm (b) 5 cm (c) 1 cm (d) 2 cm (a) R (b) 2 R (c) 3 R (d) zero
44. A progressive wave is travelling in a medium such that 51. A massless rod of length L is suspended by two identical
frequency of oscillation and displacement amplitude of the strings AB and CD of equal length. A block of mass m is
particles of the medium are f and A respectively. The ratio of suspended from point O such that BO is equal to ‘x’. Further
their acceleration, amplitude and velocity amplitude is it is observed that the frequency of 1st harmonic in AB is
(a) 2 f (b) f equal to 2nd harmonic frequency in CD. ‘x’ is
2 f f
(c) (d)
A A A C
45. In a resonance tube with tuning fork of frequency 512 Hz,
first resonance occurs at water level equal to 30.3 cm and
second resonance occurs at 63.7 cm. The maximum possible
error in the speed of sound is
O
(a) 51.2 cm/s (b) 102.4 cm/s B D
x L
(c) 204.8 cm/s (d) 153.6 cm/s
46. A tube of length L1 is open at both ends. A second tube of m
length L2 is closed at one end and open at the other end.
Both tubes have the same fundamental frequency of L 4L
vibration of air in it. What is the value of L2 ? (a) (b)
5 5
L1 L1
(a) 4L1 (b) 2L1 (c) (d) 3L L
2 4 (c) (d)
4 4

41. 42. 43. 44. 45.


MARK YOUR 46. 47. 48. 49. 50.
RESPONSE
51.
362 IIT-JEE PHYSICS Challenger
52. A parachutist jumps from the top of a very high tower with length 1m. The intensity of sound at M due to source P
a siren of frequency 800 Hz on his back. Assume his initial above is I0. What will be the intensity of sound at point M
velocity to be zero. After falling freely for 12s, he observes when both the sources are on ?
that the frequency of sound heard by him reflected from
level ground below him is differing by 700Hz w.r.t. the original P
frequency. What was the height of tower. Velocity of sound
in air is 330 m/s, and g = 10 m/s2. Q M
(a) 511.5m. (b) 1057.5m.
(c) 757.5m. (d) 1215.5m.
53. There are cases when an explosion at a point A will be heard
at point B that is far away from A while in a certain region 3I0
(a) 4I0 (b)
located much closer to A than to B, the explosion is not 2
heard due to obstruction. This will be possible
9
(c) I0 (d) None of these
4
57. A sound source S emitting a sound of frequency 500Hz and
A zone of B receiver R of mass m are at the same point. R is performing
silence SHM with the help of a spring of force constant k. At a time
(a) if air temperature increases with altitude t = 0, R is at mean position and moving toward right as
(b) if air temperature decreases with altitude shown in figure. At the same time, source starts moving
away from the R with an acceleration 18.75m/s2. Find the
(c) if air is blowing from B to A
frequency (in Hz) registered by receiver at a time t = 10s.
(d) if air is blowing from A to B
m 100
54. In the experiment to determine the speed of sound using a Given that = 2 and amplitude of oscillation of
resonance column, k
(a) prongs of the tuning fork are kept in a vertical plane 150
(b) prongs of the tuning fork are kept in a horizontal R= m, vsound = 300 m/s.
plane
(c) in one of the two resonances observed, the length of y
the resonating air column is close to the wavelength
of sound in air x
(d) in one of the two resonances observed, the length of R k

/////////
the resonating air column is close to half of the
S m
wavelength of sound in air
55. A piece of wire is cut into two pieces A and B, and stretched
to the same tension and mounted between two rigid walls. (a) 320 Hz (b) 220 Hz
Segment A is longer than segment B. Which of the following (c) 420 Hz (d) 350 Hz
quantities will always be larger for waves on A than for 58. Sound waves of frequency 16 kHz are emitted by two
waves on B ? coherent point sources of sound placed 2m apart at the
centre of a circular train track of large radius. A person riding
(a) Amplitude of the wave
the train observes 2 maxima per second when the train is
(b) Frequency of the fundamental mode running at a speed of 36 km/h. Calculate the radius of the
(c) Wave velocity track. [Velocity of sound in air 320 m/s]
(d) Wavelength of the fundamental mode
1000 500
56. Two identical point like sound sources emitting sound in (a) m (b) m
same phase of wavelength 1m are located at points P and Q
as shown in figure. All sides of the polygon are equal and of 250 700
(c) m (d) m

MARK YOUR 52. 53. 54. 55. 56.


RESPONSE 57. 58.
WAVES 363

59. The fundamental frequency of a sonometer wire of length 63. A thin string is held at one end and oscillated vertically, so
is n0. A bridge is now introduced at a distance of (<< ) that y (x = 0, t) = 8 sin 4t cm. Neglect the gravitational force.
from the centre of the wire. The lengths of wire on the two The string’s density is 0.2 kg/m and its tension is 1N. The
sides of the bridge are now vibrated in their fundamental string passes through a bath filled with 1 kg water. Due to
modes. Then, the beat frequency nearly is friction, heat is transferred to the bath. The heat transfer
(a) n 0 / (b) 8n 0 / efficiency is 50%. Calculate how much time (approximately)
passes before the temperature of the bath rises one degree
(c) 2n 0 / (d) n0 /2
Kelvin.
60. Two identical pulses move in opposite directions with same
(a) 5.2 days (b) 4.2 days
uniform speeds on a stretched string. The width and kinetic
energy of each pulse is L and k respectively. At the instant (c) 3.2 days (d) 6.2 days
they completely overlap, the kinetic energy of the width L 64. In the experiment for the determination of the speed of sound
of the string is in air using the resonance column method, the length of the
air column that resonates in the fundamental mode, with a
k tuning fork is 0.1 m. When this length is changed to 0.35 m,
/////////////

L
/////////////

the same tuning fork resonates with the first overtone.


L Calculate the end correction.
k (a) 0.012 m (b) 0.025 m
(a) k (b) 2k (c) 0.05 m (d) 0.024 m
(c) 4k (d) 8k 65. In the diagram below (not to scale), each of the loudspeakers
61. A siren placed at a railway platform is emitting sound of emits a continuous sound of the same frequency.
frequency 5 kHz. A passenger sitting in a moving train A A microphone moved along the line PQ detects a series of
records a frequency of 5.5 kHz while the train approaches maximum and minimum sound intensities. Which one of the
the siren. During his return journey in a different train B he following actions on its own, will lead to an increase in the
records a frequency of 6.0 kHz while approaching the same distance between the maxima of sound intensity ?
siren. The ratio of the velocity of train B to that train A is
(a) 242/252 (b) 2
Q
(c) 5/6 (d) 11/6
Loudspeaker
62. Statement 1 : Due to the motion of listener, the frequency of
the sound waves (as received by listener) emitted by
stationary source is affected.
Statement 2 : Due to the motion of source, wavelength of
the sound waves (emitted by source) as received by Loudspeaker
stationary listener is affected.
P
Statement 3 : If receiver and source both are moving, the
observed frequency must be different from the original
frequency of source. (a) Decreasing the frequency of the sound emitted by the
loudspeakers.
Treat motion of source or listener always along a line joining
them for all above cases. Choose correct option. (b) Increasing the frequency of the sound emitted by the
loudspeakers.
(a) All the three statements are correct
(b) Only all three statements are wrong (c) Increasing the separation of the loudspeakers
(c) Only statements 1 and 2 are correct (d) Decreasing the distance of the loudspeakers from the
(d) Only statements 2 and 3 are correct line PQ.

MARK YOUR 59. 60. 61. 62. 63.


RESPONSE 64. 65.
364 IIT-JEE PHYSICS Challenger

PASSAGE-1

There is a sinusoidal standing wave created in a string of D


=0
length 150 cm and mass 1.5 kg tied at both ends. String has
tension of 36 N. There are 4 nodes in the string other than the
ends. Maximum displacement of particle at a distance 35 cm 4. The frequency of sound detected by detector D, before
man jumps of the plank is
from one end is 2mm.
1. Circular natural frequency of the wave is 332 330
(a) f0 (b) f0
324 322
2
(a) 20 10 per sec (b) 20 per sec
3 328 330
(c) f0 (d) f0
336 338
(c) 20 per sec (d) 200 per sec
5. The frequency of sound detected by detector D, after man
2. Maximum displacement of mid point of the string is jumps of the plank is
(a) 8 mm (b) 4 mm
332 330
(c) 2 mm (d) 1 mm (a) f0 (b) f0
324 322
3. Speed of a point of string at a distance 25 cm from one end,
when the displacement of the midpoint of the string is zero 328 330
(c) f0 (d) f0
336 338
(a) 40 mm/s (b) 80 mm/s
6. Choose the correct plot between the frequency detected by
2 2 detector VS position of the man relative to detector.
(c) 40 mm/s (d) 80 mm/s
3 3
frequency detected
PASSAGE-2
(a) f0

A man of mass 50 kg is running on a plank of mass 150 kg


with speed of 8 m/s relative to plank as shown in the figure position of man relative to detector
(both were initially at rest and the velocity of man with respect
to ground any how remains constant). Plank is placed on
smooth horizontal surface. The man, while running whistles frequency detected
with frequency f0. A detector (D) placed on plank detects
frequency. The man jumps off with same velocity from point (b) f0
D and slides on the smooth horizontal surface (Assume
coefficient of friction between man and horizontal surface is
zero]. The speed of sound in still medium is 330 m/s. position of man relative to detector

MARK YOUR 1. 2. 3. 4. 5.
RESPONSE 6.
WAVES 365

7. Find acceleration of system


frequency detected
2g 201
(a) (b) g
(c) 3 301
f0
(c) g (d) g/2
8. What among the following is the closest value of velocity
position of man relative to detector
of sound wave in lower string ?

YA
frequency detected (a) 100g (b)
m

(d) f0 10000g
(c) (d) none of these
301
position of man relative to detector 9. Wavelength corresponding to first harmonic of lower string
is
(a) (b) /2
(c) 2 (d) /4
PASSAGE-3

Figure shows an arrangement of 3 masses A, B, C of mass PASSAGE-4


100 m each and 2 strings. Block B is hollow having C hung
with rope of mass m inside it. Upper string is massless.
A sound source is attached to one end A of a rod of length
Lower string has mass m, length L, Young’s modulus Y, and
area of cross section A. All surfaces are frictionless. , which is performing uniform circular motion with angular
velocity about the other end B.
A

100 m massless

A O
B
2
100 m An observer O is stationary at a distance 2 from B in the
plane of circular motion. Angle ABO at any time is
represented by . = 30m, vsound = 300 m/s, = rad/s,
B /2 speed of source is << velocity of sound.
m, , y, A 10. Find the least time difference between two natural (original)
frequency perceptions by O.
/2 (a) 0.8 s (b) 1.2 s
(c) 1 s (d) 2 s
100 m 11. Distance traveled by the source during the time between a
C least frequency perception and the immediate next highest
frequency perception by the observer.
(a) 20 m (b) 30 m
(c) 40 m (d) 10 m

MARK YOUR
7. 8. 9. 10. 11.
RESPONSE
366 IIT-JEE PHYSICS Challenger
12. What will be value of at an instant, when the observer
PASSAGE-7
records least frequency. (Tick the closest answer)
(a) /2 (b) /4
(c) 2 /3 (d) Figure shows a clamped metal string of length 30cm. and
linear mass density 0.1 kg/m. Which is taut at a tension of
40N. A small rider (piece of paper) is placed on string at
PASSAGE-5 point P as shown. An external vibrating tuning fork is brought
near the string and oscillations of rider are carefully observed.

Waves y1 = A cos(0.5 x 100 t ) and y2 = A cos(0.46 x 92 t ) 30cm.


are travelling along x-axis. (Here x is in m and t is in second)
P
13. Find the number of times intensity is maximum in time interval 10cm.
of 1 sec.
(a) 4 (b) 6 19. At which of the following frequencies of tuning fork, rider
will not vibrate at all
(c) 8 (d) 10
14. The wave velocity of louder sound is 100
(a) Hz (b) 50 Hz
(a) 100 m/s (b) 192 m/s 3
(c) 200 m/s (d) 96 m/s
(c) 200 Hz (d) None of these
15. The number of times y1 + y2 = 0 at x = 0 in 1 sec is
20. At which of the following frequencies the point P on string
(a) 100 (b) 46
will have maximum oscillation amplitude among all points
(c) 192 (d) 96 on string

200
PASSAGE-6 (a)
3
Hz (b) 100 Hz

(c) 200 Hz (d) None of these


A wave described by the equation y = A sin ax + bt + ÷
2 21. Now if the tension in the string is made 160N, at which of
is reflected by an obstacle at x = 0. The intensity of the the following frequencies of tuning fork, rider will not vibrate
reflected wave is 64 % of the incident wave. As a result, the at all
incident and reflected wave superpose to give a resultant
wave 100
(a) Hz (b) 50 Hz
y = – 1.6 A sin ax sin bt + cA cos (bt + ax) 3

16. The amplitude of reflected wave is (c) 200 Hz (d) None of these
(a) 0.2 A (b) 0.8 A
(c) 0.6 A (d) 0.4 A
17. The value of c is PASSAGE-8

(a) 0.2 (b) 0.3


(c) 0.4 (d) 0.6 A sinusoidal wave is propagating in negative x-direction in
18. The maximum particle velocity is a string stretched along x-axis. A particle of string at x=2m is
(a) 1.36 bA (b) 1.66 bA found at its mean position and it is moving in positive y-
direction at t = 1 sec. The amplitude of the wave, the
(c) 1.8 bA (d) 2 bA

MARK YOUR 12. 13. 14. 15. 16.


RESPONSE 17. 18. 19. 20. 21.
WAVES 367

wavelength and the angular frequency of the wave are 0.1 straight track, with B ahead of A. The engines are at the
meter, /2 meter and 2 rad/sec respectively. front ends. The engine of train A blows a long whistle.
22. The equation of the wave is Assume that the sound of the whistle is composed of
(a) y = 0.1 sin (4 (t – 1) + 8 (x – 2)) components varying in frequency from f1 = 800 Hz to f2
(b) y = 0.1 sin ((t – 1) – (x – 2)) = 1120 Hz, as shown in the figure. The spread in the
(c) y = 0.1 sin (2 (t – 1) + 4 (x – 2)) frequency (highest frequency – lowest frequency) is
(d) None of these thus 320 Hz. The speed of sound in still air is 340 m/s.
23. The speed of particle at x = 2m and t = 1 sec is 28. The speed of sound of the whistle is
(a) 0.2 m/s (b) 0.6 m/s (a) 340 m/s for passengers in A and 310 m/s for passengers
(c) 0.4 m/s (d) 0 in B
24. The instantaneous power transfer through x = 2m and
t = 1.25 sec, is (b) 360 m/s for passengers in A and 310 m/s for passengers
(a) 10 J/s (b) 4 /3 J/s in B
(c) 2 /3 J/s (d) 0 (c) 310 m/s for passengers in A and 360 m/s for passengers
in B
PASSAGE-9 (d) 340 m/s for passengers in both the trains
29. The distribution of the sound intensity of the whistle as
A stationary observer is situated at A, which is at a distance observed by the passengers in train A is best represented
of 300m from a railway crossing O. A train moving with by
constant speed 54 km/h on a straight track perpendicular to
OA passes O by blowing horn continuously. The frequency
of sound is 400 Hz. The speed of sound on that particular Intensity
day was V = 300 m/s. (a)
u=54km/hr
O f1 f 2 Frequency
Intensity

300m
(b)

A f1 f 2 Frequency
25. When the train crosses the junction O, the frequency
observed by the observer at A approximately is
(a) 400 Hz (b) 399 Hz
Intensity

(c) 421 Hz (d) 401 Hz


26. The distance of the train from O at the time the observer (c)
hears actual frequency is
(a) zero (b) 15m right of O f1 f 2 Frequency
(c) 15m left of O (d) 30m right of O
27. As the train approaches the crossing at O and crosses it,
the frequency heard by the observer at A will
Intensity

(a) first decrease and then increase


(b) decrease continuously (d)
(c) first increase and then decrease
(d) increase continuously f1 f 2 Frequency

30. The spread of frequency as observed by the passengers in


PASSAGE-10 train B is
(a) 310 Hz (b) 330 Hz
Two trains A and B moving with speeds 20 m/s and 30 (c) 350 Hz (d) 290 Hz
m/s respectively in the same direction on the same

MARK YOUR 22. 23. 24. 25. 26.


RESPONSE 27. 28. 29. 30.
368 IIT-JEE PHYSICS Challenger

PASSAGE-11 PASSAGE-12

You are provided with three similar, but slightly different, A uniform string of length is fixed at both ends such that
tuning forks, when A and B are both struck, a beat frequency tension T is produced in it. The string is excited to vibrate
of fAB is heard. When A and C are both struck, a beat with maximum displacement amplitude a0.
frequency of fAC is heard. It was noticed that fAB < fAC. This 33. The maximum kinetic energy of the string for its fundamental
experiment is repeated after coating tuning fork A with a tone is
little wax. Now it is observed that values of both fAB and fAC
increase. a02 2T a02 2T
(a) (b)
31. Which tuning fork has the highest frequency ? 4
(a) A
(b) B a02 2T a02 2T
(c) (d)
(c) C 2 3
(d) The answer cannot be determined from the information 34. The maximum kinetic energy of the string for its first overtone
given is
32. Which tuning fork has the middle frequency ?
(a) A a02 2T a02 2T
(a) (b)
(b) B 4
(c) C
(d) The answer cannot be determined from the information a02 2T a02 2T
(c) (d)
given 2 3

MARK YOUR
31. 32. 33. 34.
RESPONSE

1. Statement - 1 : Sound travels faster in solids than gases. of pulses is longitudinal while the other
Statement - 2 : Solids possess greater density than gases. set in transverse.
4. Statement - 1 : When pressure of an ideal gas is increased,
wavelength
2. Statement - 1 : Speed of wave = the speed of sound in gas must increase.
time period
Statement - 2 : The speed of sound in ideal gas is directly
Statement - 2 : Wavelength is the distance between two proportional to square root of pressure of
nearest particles in phase. the gas.
3. Statement - 1 : When a beetle moves along the sand 5. Statement - 1 : The pitch of wind instruments rises and
within a few tens of centimeters of a sand that of string instruments falls as an
scorpion, the scorpion immediately turns orchestra warms up.
towards the beetle and dashes towards it. Statement -2 : When temperature rises, speed of sound
Statement - 2 : When a beetle disturbs the sand, it sends increases but speed of wave in a string
pulses along the sand's surface. One set fixed at both ends decreases.

MARK YOUR
1. 2. 3. 4. 5.
RESPONSE
WAVES 369

6. Statement - 1 : Two sound waves of same intensity in a 8. Statement - 1 : When a closed organ pipe vibrates, the
particular medium will have displacement pressure of the gas at the closed end
amplitude in ratio of 2 : 1 if they have remains constant.
frequency in the ratio 1 : 2. Statement - 2 : In a stationary wave system, displacement
Statement - 2 : Two wave of same velocity amplitude in a nodes are pressure antinodes, and
particular medium have equal intensity. displacement antinodes are pressure
7. Statement - 1 : Two identical pulses travel on a string in nodes.
the opposite direction as shown. When 9. Statement - 1 : When a wave propagates from a heavier
the pulses overlap, the entire energy is string to a lighter string amplitude of
potential. transmitted wave must be less than the
amplitude of incident wave.
Statement - 2 : Power of incident wave is divided into
reflected and transmitted waves.
Statement - 2 : The velocity of any point can be found by
superposition principle
v p = v p1 + v p2 .

MARK YOUR
6. 7. 8. 9.
RESPONSE

1. Displacement (y) of air column at position x from its mean 2. A wave equation which gives the displacement along the
position at any instant is given by graph shown in the figure. y-direction is given by y = 10–4 sin (60t + 2x) where x and y are
We can conclude from graph that in metres and t is time in seconds. This represents a wave
(a) travelling with a velocity of 30 m/s in the negative x
direction
(b) of wavelength m
(c) of frequency 30/ hertz

y (d) of amplitude 10 –4 m traveling along the negative


A x-direction
E
B 3. A solid is attached to the free end of a sonometer wire. The
D x wire has a frequency of 500Hz. Then the solid is immersed in
C water and it is found that the wire has a frequency of 460Hz.
When the solid is immersed in a liquid the frequency of the
wire is 480Hz. Choose the correct options
(a) Density of air is maximum at B (a) The specific gravity of the solid is 6.51
(b) Density of air is minimum at D (b) The specific gravity of the liquid is 0.51
(c) Density is maximum at D
(c) The specific gravity of the solid is 0.51
(d) Density is minimum at A
(d) The specific gravity of the liquid is 6.51

MARK YOUR
1. 2. 3.
RESPONSE
370 IIT-JEE PHYSICS Challenger
4. Velocity of sound in air is 320 m/s. A pipe closed at one end 10. A string is stretched along the x-axis. There is a transverse
has a length of 1 m. Neglecting end corrections, the air perturbation along the string :
column in the pipe can resonate for sound of frequency :
(a) 80 Hz (b) 240 Hz x
( x, t ) 4sin ÷ cos (10 t ) where x is measured in
(c) 320 Hz (d) 400 Hz 3
5. An air column in a pipe, which is closed at one end, will be centimeters and t in seconds. Both waves moving towards
in resonance with a vibrating tuning fork of frequency 264 each other create this disturbance. Assume the velocities
Hz if the length of the column in cm is : and amplitudes are equal for the two waves.
(a) 31.25 (b) 62.50
Choose the correct options
(c) 93.75 (d) 125
6. In a resonance-column experiment to measure the velocity (a) amplitude of each waves is 2cm.
of sound, the first resonance is obtained at a length 1 and (b) velocity of each waves is 40cm/sec
the second resonance at a length 2. Then which of the (c) distance between two adjacent junctions is 2cm.
following is (are) incorrect (d) distance between two adjacent junctions is 3cm.
(a) 2 >3 1 11. A transverse sinusoidal wave of amplitude a, wavelength
(b) 2 =3 1 and frequency f is travelling on a stretched string. The
maximum speed of any point on the string is v/10, where v is
(c) 2 <3 1 the speed of propagation of the wave. If a = 10–3m and
(d) May be any of the above, depending on the frequency v = 10 m s–1, then and f are given by
of the tuning fork used
7. A wave pulse moving to the right along the x-axis is (a) =2 10 2
m (b) = 10 3
m
2.0 (c) f 103 Hz /(2 ) (d) f = 104 Hz
represented by the wave function y (x, t) = ,
( x 3.0t ) + 1
2
12. y(x, t) = 0.8/[4x+5t)2+5] represents a moving pulse, where x
where x and y are in centimeters and t is in seconds. (The
and y are in meter and t in second. Then
maximum pulse height is defined as maximum displacement
along y-axis). Then (a) pulse is moving in + x direction
(a) The maximum pulse height is decreasing with time (b) in 2 s it will travel a distance of 2.5 m
(b) The maximum pulse height is constant with time (c) its maximum displacement is 0.16 m
(c) The speed of the pulse is 3.0 cm/s (d) it is a sysmmetric pulse
(d) The speed of the pulse is 0.33 cm/s
13. A metallic rod of length 1m is rigidily clamped at its midpoint.
8. Standing waves can be produced
Longitudinal stationary waves are set up in the rod in such
(a) on a string clamped at both the ends.
a way that there are two nodes on either side of the midpoint.
(b) on a string clamped at one end free at the other
The amplitude of an antinode is 2 × 10–6m.
(c) when incident wave gets reflected from a wall
(Young’s modulus = 2×1011Nm–2 and density = 8000kg m–3)
(d) when two identical waves with a phase difference of
are moving in the same direction Choose the correct options
9. As a wave propagates, (a) The equation of motion at a point 2cm. from the
(a) the wave intensity remains constant for a plane wave midpoint is y = 2 × 10–6 sin 5 x cos 25 × 103 t
(b) the wave intensity decreases as the inverse of the (b) The equation of one of the constituent waves in the
distance from the source for a spherical wave rod is y1 = (1 × 10–6) sin (5 x – 25 × 103 t)
(c) the wave intensity decreases as the inverse square of (c) The equation of one of the constituent waves in the
the distance from the source for a spherical wave
rod is (1 × 10–6) sin (5 x + 25 × 103 t)
(d) total intensity of the spherical wave over the spherical
(d) The equation of motion at a point 2cm. from the
surface centred at the source remains constant at all
midpoint is y = 2 × 10–6 sin 10 x cos 25 × 103 t
times.

MARK YOUR 4. 5. 6. 7. 8.
RESPONSE 9. 12. 11. 12. 13.
WAVES 371

14. Two identical straight wires are stretched so as to produce (b) all the particles must be at their positive extremes
6 beats per second when vibrating simultaneously. On simultaneously once in a time period
changing the tension slightly in one of them, the beat (c) in one time period all the particles are simultaneously
frequency remains unchanged. Denoting by T1 , T2 the at rest twice
higher and the lower initial tension in the strings, then it (d) all the particles are never at rest simultaneously
could be said that while making the above changes in 17. In a wave motion y = a sin (kx - t), y can represent
tension, (a) electric field (b) magnetic field
(a) T2 was decreased (b) T2 was increased (c) displacement (d) pressure
18. A sound wave of frequency f travels horizontally to the
(c) T1 was decreased (d) T1 was increased
right. It is reflected from a large vertical plane surface moving
15. The (x, y) co-ordinates of the corners of a square plate are to left with a speed v. The speed of sound in medium is c
(0, 0), (L, 0), (L, L) and (0, L). The edges of the plate are (a) The number of wave striking the surface per second is
clamped and transverse standing waves are set up in it. If
u(x, y) denotes the displacement of the plate at the point (c + v )
f
(x, y) at some instant of time, the possible expression(s) for c
u is (are) (a = positive constant)
c (c v )
(a) a cos ( x/2L) cos ( y/2L) (b) The wavelength of reflected wave is
f (c + v )
(b) a sin ( x/L) sin ( y/L)
(c) a sin ( x/L) sin (2 y/L) (c + v)
(c) The frequency of the reflected wave is f
(d) a cos (2 x/L) sin ( y/L) (c v)
16. In a standing wave on a string rigidly fixed at both ends.
(d) The number of beats heard by a stationary listener to
(a) all the particles must be at their positiveextremes
simultaneously once in half of the time period vf
the left of the reflecting surface is
c v

MARK YOUR
14. 15. 16. 17. 18.
RESPONSE

1. Match the column


Column I Column II
(A) Graphical representation of pressure variation in (p) Maximum kinetic energy at B
both end open organ pipe.
A B C

(B) Graphical representation of pressure variation in (q) Maximum potential energy at B


one end closed organ pipe.
A B
372 IIT-JEE PHYSICS Challenger
(C) Snapshot of string fixed at both ends (r) Maximum particle velocity at B

A B
////////////////

(D) Snapshot of a string fixed at one end and (s) Maximum particle acceleration at B
connected to a smooth massless ring that is
constrained to move vertically.

B (Extreme
////////////////

position)

////////////////

2. Each of the properties of sound listed in the column A primarily depends on one of the quantities in column B. Write down the
matching pairs from the two columns.
Column A Column B
(A) pitch p. Waveform
(B) quality q. frequency
(C) loudness r. intensity
3. A small pulse travelling with speed v in a string is shown at t = 0, moving towards free end. Select the shape of string from column
II at moments shown in column I. Match the columns
L/2
x=0
v x=L
t=0
s<<L free end
fixed end
L

Column I Column II

L
(A) t = (p)
v

2L
(B) t = (q)
v

3L
(C) t = (r)
v

(s)

1. 2. 3.

MARK YOUR
RESPONSE
WAVES 373

1. An observer standing at sea coast observes 54 waves two bridges be placed so that the fundamental frequencies
reaching the coast per minute. If the wavelength of the waves of the three segments are in the ratio of 1 : 3 : 15 ?
is 10m, find the velocity (in m/s). 7. AB is a cylinder of length 1m fitted with a thin flexible
2. Determine the change in volume (in cc) of 6 litres of alcohol diaphram C at middle and two other thin flexible diaphram A
if the pressure is decreased from 200 cm of Hg to 75 cm. and B at the ends. The portions AC and BC contain
[velocity of sound in alcohol is 1280 m/s, density of alcohol hydrogen and oxygen gases respectively. The diaphrams
= 0.81 gm/cc, density of Hg = 13.6 gm/cc and g = 9.81 m/s2]. A and B are set into vibrations of the same frequency. What
3. Speed of sound in air is 332 m/s at NTP. What will be the is the minimum frequency (in Hz) of these vibrations for
speed of sound (in m/s) in hydrogen at NTP if the density of which diaphram V is a node? Under the condition of the
hydrogen at NTP is (1/16) that of air ? experiment the velocity of sound in hydrogen is 1100 m/s
4. A uniform rope of length 12m and mass 6 kg hangs from a and oxygen 300 m/s.
rigid support. A block of mass 2 kg is attached to the free end 8. A column of air and a tuning fork produce 4 beats per second
of the rope. A transverse pulse of wavelength 0.06 m is when sounded together. The tuning fork gives the lower
produced at the lower end of the rope. What is the wavelength note. The temperature of air is 15ºC. When the temperature
(in m) of the pulse when it reaches the top of the rope ? falls to 10ºC, the two produce 3 beats per second. Find the
5. A source emitting sound of frequency 180 Hz is placed in frequency (in Hz) of the fork.
front of a wall at a distance of 2 m from it. A detector is also 9. A string of length 25 cm and mass 2.5 g is under tension. A
placed in front of the wall at some distance from it. Find the pipe closed at one end is 40 cm long. When the string is set
minimum distance (in m) between the source and the detector vibrating in its first overtone and the air in the pipe in its
for which the detector detects a maximum of sound. Speed of fundamental frequency, 8 beats per second are heard. It is
sound in air = 360 m/s observed that decreasing the tension in the string decreases
6. The length of a wire between the two ends of a sonometer is the beat frequency. If the speed of sound in air is 320 ms-1,
105cm. What is the sum (in cm) of the distances where the find the tension (in newton) in the string.

1. 2. 3. 4. 5. 6.

MARK
YOUR 7. 8. 9.
RESPONSE
374 IIT-JEE PHYSICS Challenger
10. A train approaching a hill at a speed of 40km/hr sounds a now moved away from the vibrating wire with a constant
whistle of frequency 580 Hz when it is at a distance of 1km speed and an observer standing near the sonometer hears
from a hill. A wind with a speed of 40km/hr is blowing in the one beat per second. Calculate the speed (in m/s) with
direction of motion of the train. Find the frequency (in Hz) of which the tuning fork is moved if the speed of sound in air
the whistle as heard by an observer on the hill.(velocity of is 300 m/s.
sound in air = 1200 km/hr) 15. A steel wire of length 1 m, mass 0.1 kg and uniform cross-
11. S1 and S2 are two coherent current sources of radiations sectional area 10–6 m2 is rigidly fixed at both ends. The
separated by distance 100.25 , where is the wavelength of temperature of the wire is lowered by 20° C. If transverse
radiation. S1 leads S2 in phase by /2. A and B are two points waves are set up by plucking the string in the middle,
on the line joining S1 and S2 as shown in figure. calculate the frequency (in Hz) of the fundamental mode of
The ratio of amplitudes of source S1 and S2 is 1 : 2. vibration.
16. Two tuning forks with natural frequencies of 340 Hz each
A S1 S2 B move relative to a stationary observer. One fork moves away
from the observer, while the other moves towards him at the
same speed. The observer hears beats of frequency 3 Hz.
Find the speed (in m/s) of the tuning fork.
Find the ratio of intensity at A to that at B. 17. A tuning fork of frequency 480 Hz resonates with a tube
12. In a car race sound signals emitted by the two cars are closed at one end of length 16 cm and diameter 5 cm in
detected by the detector on the straight track at the end fundamental mode. Calculate velocity (in m/s) of sound
point of the race. Frequency observed are 330 Hz and 360 Hz in air.
and the original frequency is 300 Hz of both cars. Race ends 18. A whistling train approaches a junction. An observer
with the separation of 100m between the cars. Assume both standing at junction observes the frequency to be 2.2 kHz
cars move with constant velocity and velocity of sound is and 1.8 kHz of the approaching and the receding train
330 m/s. Find the time (in second) taken by winning car. respectively. Find the speed (in m/s) of the train (speed of
13.. A copper wire is held at the two ends by rigid supports. At sound = 300 m/s).
30°C, the wire is just tant, with negligible tension. Find the 19. In a resonance tube experiment to determine speed of sound,
speed (in m/s) of transverse waves in this wire at 10°C. air column in the pipe is made to resonate with a given
14. A sonometer wire under tension of 64 newton vibrating in tuning fork of frequency 480 Hz. The diameter of the pipe is
its fundamental mode is in resonance with a vibrating tuning 5 cm and it is open at one end. The smallest resonating
fork. The vibrating portion of the sonometer wire has a length length is observed to be 16 cm. Calculate the speed of sound
of 10 cm and a mass of 1 gm. The vibrating tuning fork is (in m/s) from the given experimental data.

10. 11. 12. 13. 14. 15.

MARK
YOUR
RESPONSE 16. 17. 18. 19. 20.
WAVES 375

1 (a) 11 (d) 21 (d) 31 (b) 41 (d) 51 (a) 61 (b)


2 (b) 12 (c) 22 (a) 32 (b). 42 (b) 52 (b) 62 (c)
3 (a) 13 (d) 23 (d) 33 (a) 43 (a) 53 (a) 63 (b)
4 (b) 14 (a) 24 (d) 34 (c) 44 (a) 54 (a) 64 (b)
5 (c) 15 (a) 25 (b) 35 (a) 45 (c) 55 (d) 65 (a)
6 (c). 16 (d) 26 (d) 36 (a) 46 (c) 56 (c)
7 (c) 17 (a) 27 (a) 37 (a) 47 (c) 57 (d)
8 (b) 18 (b) 28 (c) 38 (a) 48 (d) 58 (a)
9 (b) 19 (b) 29 (b) 39 (b) 49 (a) 59 (b)
10 (a) 20 (d) 30 (d) 40 (a) 50 (d) 60 (c)

1 (c) 7 (b) 13 (a) 19 (c) 25 (d) 31 (c)


2 (b) 8 (b) 14 (c) 20 (d) 26 (b) 32 (b)
3 (a) 9 (a) 15 (d) 21 (c) 27 (b) 33 (a)
4 (a) 10 (a) 16 (b) 22 (c) 28 (b) 34 (b)
5 (c) 11 (c); 17 (a). 23 (a). 29 (a).
6 (a) 12 (a) 18 (c) 24 (d) 30 (a)

1 (b) 3 (a) 5 (a) 7 (a) 9 (d).


2 (a) 4 (d) 6 (a) 8 (d)

1 (a, b). 5 (a,c) 9 (a,c,d) 13 (a, b, c) 17 (a,b,c,d)


2 (a,b,c,d) 6 (a, b, d) 10 (a, b, d) 14 (b,c) 18 (b)
3 (a, b) 7 (b, c) 11 (a,c) 15 (b,c)
4 (a,b,d) 8 (a,b,c) 12 (b,c,d) 16 (b, c).

1. A-q; B-q; C-q; D-s 2. A-q; B-p; C-r 3. A-p ; B-s ; C-r

1 9 2 0.75 3 1328 4 0.12 5 3


6 175 7 1650 8 110 9 27 10 600
11 0.11 12 4 13 70 14 0.075 15 11
16 1.5 17 336 18 30 19 336
376 IIT-JEE PHYSICS Challenger

observer u 4. (b) The apparent frequency heard by the listener


A B
1. (a) x.......................> x v v0 332 10
A B n =n = 200 = 194 Hz
v 332
The observed frequency from A,
v
( vs u ) but n =
A =
vs So, wavelength of the note received by the listener
The observed frequency from B, v 332
= 1.71m
(vs + u ) n 194
B =
vs 5. (c) Frequency of the open organ pipe n = f.
Initial length of the pipe = l
2u
Number of beats = B ~ A = 2. .u = Final length =
l
s 2
2. (b) A vibrating organ pipe can be closed or open. For a closed We know that frequency of open organ pipe is
organ pipe fundamental frequency v
f1 =
v 2l
fc = 200Hz
4l and when it is dipped in water, it behaves as a closed end
pipe
3v
Its first overtone = 3 fc 600Hz v v v
4l Its frequency f2 = f
For an open organ pipe, fundamental frequency 4l2 4l/ 2 2l
v 6. (c) Stationary wave is produced when two waves travel in
f0 = 200Hz opposite direction.
2l
Now, y = a cos (k x – t) – a cos (k x + t)
2v y = 2a sin kx sin t is equation of stationary wave which
Its first overtone = 2 f 0 400Hz
2l gives a node at x = 0.
3. (a) The relation for the velocity of sound in a gas 7. (c) The air is blowing in the direction of train i.e., in the
direction of motion of sound. The frequency of sound
RT heard by observer
v=
M (v w ) v0
Considering the mixture of gas while all the constituents =
(v w) vs
of the mixture occupy the same volume their masses vary.
Let mO, m N, m A be the fractions of masses of the Here, = 580 Hz
respective gases and MO, MN, MA be their respective v + w = 1200 + 40 = 1240 km/h
molecular weights. Now the velocity of sound in the vs = 40 km/h
mixture can be given by the relation, v0 = 0
1/ 2 1240 0
O mO N mN A mA
So, = 580 599 Hz
v = RT + + 1240 40
MO MN MA
v + vA v
8. (b) fA
v
f , or v A
f
( fA f)
1/2
2 7 1 v + vB
f , or vB = ( f B
v
10
1.4
1.4
10
1.67
10 ÷ fB = f)
= 8.3 273 3
+ 3
+ 3÷
v f
32 10 28 10 40 10 ÷
vB f f 6.0 5 1
= B = = =2
= [8.3×273×1000 (8.75× 10–3 + 35 × 10–3 vA f A f 5.5 5 0.5
+ 4.175 × 10–3)]1/2
9. (b) Pitch is related with frequency (f) = v/ = const. But
= [8.3 × 273 × 47.925]1/2 = 329.5 m/s
intensity is related with particle energy which increases.
WAVES 377

1 T v
10. (a) f = ; 14. (a) Fundamental frequency of open pipe is f0 =
2 m 2
In air : T = mg = Vg v
Third harmonic of the closed pipe fc = 3 ÷
1 Vg 4
f= ... (i)
2 m v v v
In water : T = mg – upthrust Given : 3 = + 100 or = 200 Hz
4 2 2
V Vg 15. (a) According to Hooke's law FR µ x
=V g– g (2 ) [Restoring Force FR = T, tension of spring]
2 2
T
Vg Velocity of sound by a stretched string v =
(2 ) m
1 2
f '= where m is the mass per unit length
2 m
vµ T
1 Vg (2 )
=
2 m 2 Hence v µ T
v T
f' 2 or, =
= v' T'
f 2
T' 1.5x
1/ 2 or v' = v =v = 1.22 v
2 T x
f '= f ÷
2
2 x
16. (d) y = a cos2 (2 nt ) can be written as
1/ 2
2 1
= 300 Hz
2 2 x
1 cos 2 2 nt
1 T =a
11. (d) Fundamental frequency f = 2
2
a a 4 x
= cos 4 nt
f1 2 2
= 1 2
(since tension is same)
f2 2 1 Comparing with standard equation of harmonic wave
y = A cos ( t – x), we get
2L r2 a
= , A= , = 4 n = 2 f, where f = 2n
L 4r 2 2
(since the wires are of same material) 4 2 2
=1 Also k = = = ; '= /2
( / 2)
12. (c) Comparing it with
y (x, t) = A cos ( t + /2) cos k x 17. (a) For f to be greatest, should be minimum ABC, using
If k x = /2, a node occurs; the rule
10 x = /2 x = 0.05 m sin (90 ) sin sin
If k x = , an antinode occurs cos =
R/2 R 2
10 x = x = 0.1 m
Also speed of wave
50
= = = 5 m/s and R/2
k 10 B O
= 2 /k = 2 /10 = 0.2 m R
2 2
A
13. (d) Er = Ar V V
= 2 1 ÷ = 1/ 9
Ei Ai ÷ V1 + V2

Et For minimum sin should be max. = 90°


Therefore, = 8/9
Ei = 60°
378 IIT-JEE PHYSICS Challenger

c c dy
A cos (kx t)
÷ cos 60° =
22. (a) v =
approach velocity = dt
3 6
vmax = A
23. (d) Velocity of particle
dy 2 2 v
c ÷ 6 f0 = a cos (vt x) ......(1)
f max = f 0 = dt
c÷ 5
c ÷ Slope of curve,
6
dy 2 2
R/2 = a cos (vt x ) .....(2)
dx
60° 60° By equation (1) and (2)
R
dy / dt dy dy
c ÷ 6 f0 =–v =–v
and f min = f 0 = dy / dx dt dx
c÷ 7
c+ ÷ 340 10
6 24. (d) f1 = n0 = f0 ;
340 34 9
18. (b) The equation of the given transverse wave is given by 340 20
the displacement of the particle y.
f2 = n0 = f0 ;
340 17 19
y = A sin2 ( t – kx)
f1 10 19 19
dy =
Velocity of the particle, 2 A sin ( t kx ) f2 9 20 18
dt
Maximum velocity = 2A 25. (b) AB = AD 2 + BD 2 = = AC
2 So sound reaching from B and C will be in same phase
But the velocity of the wave = = Now AD = 5 /6
k 2 /
2 5 10 5
1 = =
If 2A = , 2A = = A= 6 6 3
k k 2 4
A
19. (b) Draw a sketch graph showing the two terms and use the
principle of superposition to find the final waveform.
5 /6

y(t) B D C
Resultant 11/6

x A = (2 A) 2 + ( A) 2 + 2 2 A A cos( / 3).

= 5 A2 + 2 A2 = A 7 I = 7I0
26. (d) A travelling wave equation y = f (x ± vt)
At t = 0.75 s,
20. (d) vs = 0 33 m/s
5
y1 = ,
(3 x 3) 2 + 2
0
Vs (Observer)
(Source) 5
y2 =
(3 x 3)2 + 2
v
'= y = y1 + y2 = 0 for any value of x
v vs At x = 1m,
5 5
= 450
330
= 500 Hz y1 = , y2 =
330 33 (3 4t ) + 2
2 (4t 3) 2 + 2
21. (d) x1 and x2 are in successive loops of stationary waves. y = y1 + y2 = 0, for any value of t.
At t = 0.75s, y = 0 for all x i.e. string is in its natural length
so, 1
hence its elastic potential energy is zero but kinetic energy
3 7 6 is not zero.
and 2 k ( x) k ÷
1
= Interference of waves does not destroy energy, it remains
2k 3k 6 2 7
conserved.
WAVES 379

1 1 y
27. (a) At t = 0, y = or x = = x1 5R T –T
1 + x2 y dx = C.dt = T0 + L 0 ÷ dt
3M 0 L
1 1
At t = 2s, y = =
2 x 2
2x 1 ( x 1)2 t=
2L 3M

1 y
( TL + T0 ) 5R
or ( x 1) =
2
or x = 1 +
1 y
= x2
y y 34. (c) E µ A2 2 where A = amplitude and = frequency.
Speed of the wave Also =2 µ
In case 1 : Amplitude = A and 1 =
x x2 x1 1
v= = = = 0.5 m/s In case 2 : Amplitude = A and 2=2
t t2 t1 2 0
E2 A2 2
= 2
=4
T T E1 A2 2
28. (c) v1 = ; v2 = 1
4
E2 = 4E1
v2 < v1 35. (a) The expression for fundamental frequency is
2nd medium is denser
The wave reflected from the denser medium has 1 T
=
phase change of . 2 µ
v1
v1 In air, T = mg = (V )g
v2 v1 2
Ar 6 6 2mm
v2 + v1 v2 1 V g
+ v1 =
2 2 µ
eqn of reflected wave pulse is When the object is half immersed in water
Y = –(2mm) sin (5t – 40x) T' = mg – upthrust
RT 1 V g – (V / 2) w g (V / 2) g (2 – w )
29. (b) Crms = ÷ Here Crms µ m
;
M The new fundamental frequency is

Crms1 m2 Vg
(2 – w)
= 1 T' 1
m1 ÷ '= = 2
Crms 2
2 2 µ
30. (d) v0 = speed of sound
u
' 2 – 1
w
' = 300 1– Hz
2 2 ÷
v – u sin
n = n0 ÷
v – u cos u
5 9g 3 Mg
36. (a) f0 = =
As varies from zero to 90°, n decreases continuously. 2 µ 2 µ
M = 25 kg
31. (b)
Using the formula of a vibrating string,
p T
f = where p = number of loops.
After two seconds pulses will overlap each other. 2
According to superposition principle the string will not In each case, the wire vibrates, in resonance with the
have any distortion and will be straight. same tuning fork. Frequency of wire remains same while
Hence there will be no P.E. The total energy will be only p and T change.
kinetic.
32. (b) Beats are produced due to the difference in apparent p1 T1 p T2
= 2 or p1 T1 = p2 T2
frequency of the two tuning forks. 2 2
dx
x T2 p
RT 5RT or = 1
33. (a) C = = T1 p2
M0 3M 0 A B
M g 5
= or M = 5 5 9 or M = 25 kg.
T0 TL 9 g 3 3 3
L
380 IIT-JEE PHYSICS Challenger

v 103 3.9
37. (a) L = (2n + 1) and = = (2n + 1)85Hz A=
4
3 4 104 1.3
38. (a)
= 0.025 m = 2.5 cm
44. (a) vmax = (2 f) A
amax = (2 f)2 A
O S
amax
(2 f )
vmax
Frequency heard by observer directly coming from
355 – 5 45. (c)
source 180 = 175 Hz.
355 + 5 l1 +e
l2 +e
f2 frequency heard by observer after reflection
355 355 – 5
180 = 180 Hz
355 – 5 355
f2 – f1 = 5 Hz
39. (b) f1= frequency of the police car heard by motorcyclist,
f2 = frequency of the siren heard by motorcyclist.
330 v 330 + v
f1 = 176; f2 = 165; For first resonance For second resonance
330 22 330
3
1+e = +e=
f1 – f2 = 0 v = 22 m/s
2
4 4
T But v =
40. (a) v = = 6 m/s
µ 4 3v
v= ( 2 + e) 2 +e=
2 0.3 3 4v
t= = 0.1 sec ...(i)
6
v
41. (d) y(r , t ) = A sin ( t – k . r ) v = 4( 1 + e) 1+e =
4
2 k ... (ii)
k (cos i cos j cos k ) = (i + j + k ) Subtracting (i) and (ii),
3
– = 2 (3) = 6 rad. v 2 ( 2 1)
1 2
42. (b) Frequency of first overtone in closed pipe, v= 2 ( 2 1)
3v P = 2 × 512 × (0.1 + 0.1) cm/s
= ... (i) = 204.8 cm/s
4 1 1 46. (c) Fundamental frequency for a tube open at both ends, v1
Frequency of first overtone in open pipe,
v
= .
1 P 2 L1
'= ... (ii)
2 2
From equation (i) and (ii)
4
2 = 1
1
3 2
43. (a) y (x, t) = A sin t cos kx [Assuming t = 0 y = 0] = L1 = L2
2 4
y
P (x) = B = + kBA sin t cos kx
x

B Fundamental frequency for a tube closed at one end,


[B is Bulk modulus of elasticity, v = ]
0 v
v2 =
4 L2
2 3 v2 0 A
Pmax 0 v 2
A = v1 = v2
L
2 L1 = 4 L2 L1 = 2 L2
WAVES 381

A1 + A2 A x 1 1 TAB 1 TCD
47. (c) =x ; 2 = ; Energy µ A2 =
A1 A2 A1 x + 1 2 m m

x 1
2 TAB
= TCD
÷ 4
x +1
TAB = 4TCD ... (i)
48. (d)
A B
nth harmonic

l l
AB
= CD =
= 2
v v TAB TCD
f1 = =
O
... (i) x D
B L–x
4
=
n m
L
v nv
f2 = = For rotational equilibrium of massless rod, taking torque
4
... (ii) about point O.
Here n is a odd number. From (i) and (ii) TAB × x = TCD (L – x) ... (ii)
For translational equilibrium,
n TAB + TCD = mg ... (iii)
f2 = f1
4 On solving, (i) and (iii), we get
5 mg
For first resonance, n = 5, f2 = f1 TCD = ;
4 5
49. (a) Cannot be greater than 2L. 4mg
For fundamental case ( /2 = L) TAB =
5
d Substituting these values in (ii), we get
50. (d) dm. 2R = 2T sin
2 4mg mg
x ( L x)
dm.v 2/r 5 5
4x = L – x
d d L
2 x=
5
52. (b) Let the sound observed by the parachutist at t0 = 12s be
produced at t1s. Velocity of source at the instant of sound
= gt1 and velocity of observer at the instant of observing
2R =
d
µRd 2T same sound = gt0. Hence the relation between apparent
2 frequency f ' and original frequency f will be
T 2 v + gt0
µ 2 R2 = T vw = R2 R f =f .
v gt1 ÷
also speed of string is R
The velocity of disturbance w.r.t. ground
= R– R=0
51. (a) Frequency of Ist harmonic of AB (1/2) gt12
(1/2) gt02
1 TAB
=
2 m h

1 TCD Reflected
Frequency of 2nd harmonic of CD = Incident
m sound
sound
Given that the two frequencies are equal.
382 IIT-JEE PHYSICS Challenger
Here f = 800 Hz, g = 10 m/s2, v = 330 m/s, t0 = 12s and vs = at = 18.75 × 8 = 150 m/s
f ' = 800 + 700 = 1500 Hz
300 + 15
Putting these, we get, t1 = 9s f = 500 ÷ = 350 Hz
300 + 150
Now the distance travelled by sound in (t0 – t1) sec is
58. (a) The wavelength of the sound wave in air is
1 2 1 2
v (t0 – t1) = h gt0 h gt1 ÷ 320
2 2 = 3
= 2 10 2 m .
16 10
Putting the values, we get, h = 1057.5m.
53. (a) This concept is same as that of mirage i.e. total internal
reflection of sound wave due to varying density of air of
atmosphere.
54. (a) As shown in the figure, the prongs of the tuning fork are S1
kept in a vertical plane.
d ?

S2

The positions of maxima on the circumference of the


circular track will be given by
d sin = n
55. (d) For fundamental mode, = L, = 2L When d is the separation between the sources and is
2
the angular position of nth maximum as shown in the
High L assures high . figure
56. (c) Wave from P reaches M in same phase as originated
n
wave from Q also reaches M in same phase as originated 2 sin = n (2 × 10–2) sin =
100
Hence both are in same phase at M and thus constructive
Since sin lies between 0 and 1 there are 400 maxima on
interference takes place.
the entire circle.
2 These 400 maxima will be heard by the person in the time
I 9
I = ( I1 + I 2 ) = 2
I0 + 0 ÷ = I0
4 4 400
t= = 200s
2
57. (d) Time period of oscillation of R
5
m 10 Speed of the train = 36 km/h = 36 = 10 m / s
T 2 2 = 20 s 18
k
From the obtained values so far we get length of the
At a time t = 10s, R will be at mean position and moving track
along negative x-axis. = (10 m/s) (200s) = 2000m
vR = A = 15m/s
2000 1000
The second which is received at t = 10s, is emitted at So radius of the track = = m
2
t = t0 s.
v v v
; n1 = 2 ( / 2 , n2 =
1 2
at0 v (10 t0 ) 59. (b) n0 = ) 2 ( /2 )
2 2
Beat frequency = n1 – n2
1
18.75t02 300 (10 t0 ) t0 = 8s
2 1 1
v
2 2
WAVES 383

( 2 ) ( 2 ) 4
=v 2 2 k = m 1
4 T 5

4 8 v 8 n0 The wave power is :


=v 2 2
= =
4 2 y y
P = Fy v y = T T kA2 cos 2 ( t kx ) ..... (2)
60. (c) The velocity profile of each elementary section of the x t
pulse is shown in figure 1 and figure 2. The average power over a period is :
When both the pulses completely overlap, the velocity
profiles of both the pulses in overlap region are identical.
1 1
By superposition, velocity of each elementary section P T kA2 2
A2 v .......... (3)
2 2
doubles. Therefore, KE of each section becomes four the energy density
times. Hence the K.E. in the complete width of overlap
where v = is the wave velocity. Note that eq. (3) was
becomes four times. k
pulse moving derived from the known result that the average over a
towards right
elementary 1
section whole period of cos2 x is . Substitution of the numerical
pulse moving 2
towards left

Velocity
values yield P = 0.0229 J/s
vector of
elementary The power transferred to the bath, P, is
section
P = 0.5 P = 0.01145 J/s ........... (4)
The temperature difference caused by the heat transfer
Velocity Velocity
profile profile Q to a substance with heat capacity C is
Figure-1 Figure-2 1 1
T Q (P t ) .......... (5)
C C
v +v
61. (b) Using the formula n ' = n A
{
÷
v In our case : C = 1k cal / ° K
T = 1° K
v A + v 5.5 VB + V 6
= and = Therefore, t = 3.6 × 105 s 4.2 day .......... (6)
v 5 V 5

vB 64. (b) 1+ x=
=2 4
vA
or, = 4( 1+ x)
V ± V0 V ± V0
f = ; f1 =
V ÷
62 (c) f f
V ± Vs ÷ ( 2 + x) =
3
4
V V ± VS
f2 = S =
V ± VS ÷ ; 2
f ÷ 4
= ( 2 + x)
V0 = velocity of listener, Vs = velocity of source. 3

But if Vs = V0 then f ' = f v v


1=
=
Thus statement-3 is not always correct. 1 4 ( 1 + x)
63. (b) The wave resulting from the oscillation of the end of the
string is defined by : v 3v
2=
=
2 4( 2 + x)

y (x, t) = 8sin 4t 4 x÷ cm ..... (1) Given


T 1= 2

Using eq. (1), we can immediately write : = 4s–1


384 IIT-JEE PHYSICS Challenger

65. (a)
Q
0.1m=l1 l1 +x 0.35m=l 2 l2 +x
x1
d x=n
x2

D P

v 3v x d n d
or, =
4( 1 + x ) 4 ( 2 + x)
d d
or, x = 0.025 m
n or n ÷ f
v

df
or nmax =
v
by decreasing f, number of maxima will decrease, thus,
their separation will increases.

1. (c); 2. (b); 3. (a)

F 36
v= = = 6 m/s
µ 1

2 v
= = 20 per sec.

30 cm

201mg 201g
150 cm = 60 cm =
301m 301
4. (a); 5. (c); 6. (a)
Y YA
Let velocity of plank is v and velocity of man is (v – 8) v= =
0 = 50 × (v – 8) + 150 v m
v = 2 m/s
=
f1 frequency of sound detected by detector D before man
10. (a); 11. (c); 12. (a)
jumps off the plank
Original frequencies will be produced, when AB lies along
330 + 2 332 BO. So time difference
= f0 = f0
330 – 6 324 2 2 30
f2 frequency of sound detected by detector D after man = 1–
– = 0.8 s
v 300
jump off the plank
Least and highest frequency perceptions will be as shown
330 – 2 328 in the figure.
= f0 = f0
330 + 6 336 Distance traveled
7. (b); 8. (b); 9. (a) 2 4
2 = = 40 m
3 3
WAVES 385

Wave velocity is same because it depends on the medium in


lowest which the wave is travelling.

90° Now, at x = 0,
y1 + y2 = (A cos 10 t) + (A cos 92 t) = 0
cos 100 t = – cos 92 t = cos (– 92 t)
2 = cos [(2n + 1) – 92 t
90° 2n + 1
t=
highest 192

Additional in the time that sound takes to reach from A 1


when t = 0, n = and when t = 1,
to O. 2

3 191
2 ÷ n= = 95.2
2 2
'
v
net amplitude is zero for n = 96 times (the nearest
3 30 3 answer).
= =
300 10 16. (b) I µ A2
= 0.1732 Intensity of reflected wave is 64% of incident wave
Ar = 0.8 A.

17. (a) yi = A sin (ax + bt + ) = A cos (ax + bt)


' 90° 2
and yr = – 0.8 A cos (ax – bt)
/3
y = yi + yr = – 1.6 A sin ax sin bt + 0.2 A cos (ax + bt)
c = 0.2.
1 3
'+ = +
3 3 10 ÷ dy
18. (c) ÷ = 1.8 bA.
= 0.5065 dt max
13. (a); 14. (c); 15. (d) 19. (c); 20. (d); 21. (c)
The equations are y 1 = A cos (0.5 x – 100 t) and
Wave velocity in string is
y2 = A cos (0.46 x – 92 t) represents two progressive wave
travelling in the same direction with slight difference in the
T 40
frequency. This will give the phenomenon of beats. v= = = 20m/s
0.1
Comparing it with the equation
y = A cos (kx – t), we get Fundamental frequency of string oscillations is
1 = 100 2 f1 = 100 f1 = 50 Hz and
v 20 100
2 n0 = = = Hz
K1 = 0.5 0.5 1=4m
2 0.6 3
1
Thus string will be in resonance with a turning fork of
frequency,
Wave velocity = 1f1 = 200 m/s [Alternatively use v = ]
K
100 200 400
nf = Hz, Hz,100Hz, Hz,.....
2 = 92 2 f2 = 92 f2 = 46 Hz 3 3 3
Therefore beat frequency = f1 – f2 = 4 Hz and
Here rider will not oscillate at all only if it is at a node of
2 200 stationary wave in all other cases of resonance and non-
K2 = 0.46 0.46 2= 46
2 resonance. It will vibrate at the frequency of turning fork. At
a distance /3 from one end node will appear at 3rd, 6th, 9th or
200 similar higher harmonics i.e. at frequencies 100Hz, 200Hz, ......
Wave velocity = ×46 = 200 m/s
46
If string is divided in odd no. of segments, these segments
386 IIT-JEE PHYSICS Challenger
can never resonate simultaneously, hence at the location of intensity of the whistle by the passengers in train A is
rider, antinode is never obtained at any frequency. uniform.
22. (c) The equation of wave moving in negative x-direction,
v v0 340 30 31
assuming origin of position at x = 2 and origin of time 30. (a) ' 1 = 800 = 800
(i.e. initial time) at t = 1 sec. v vs 340 20 32
y = 0.1 sin (2 t + 4x)
Shifting the origin of position to left by 2m, that is, to x v v0 31
'' 2 1120
= 0. Also shifting the origin of time backwards by 1 sec, v vs 32
that is to t = 0 sec.
y = 0.1 sin (2 (t – 1) + 4 (x – 2)) 31 31
'' ' = (1120 – 800) × 320 = 310 Hz.
23. (a) As given the particle at x = 2 is at mean position at 32 32
t = 1 sec.
Its velocity v = A = 2 × 0.1 = 0.2 m/s. 31. (c), 32. (b)
24. (d) Time period of oscillation Coating tuning fork A will decrease its frequency, beat
frequency increases means fB > fA and fC > fA
2 2
T= = = 1 sec. Given (fB – fA) < (fC – fA)
2
fC > fB > fA
Hence, t = 1.25 sec, that is, at T/4 seconds after t = 1
33. (a), 34. (b)
second, the particle is at rest at extreme position.
When a string oscillates, nodes are produced at its ends. In
Hence instantaneous power at x = 2 at t = 1.25 sec. is
case of fundamental tone, it vibrates in single loop. Hence,
zero.
wavelength of fundamental tone, 0 = 2 and in case of first
25. (d) The sound heard by observer must have been emitted
overtone it vibrates in two loops as shown in figure. Hence
sometime earlier at B so that by the sound reached at
wavelength of first overtone is 1 = .
A, train also reacted at O.

BO u 15 1
= = cos = =
BA v 300 20
The observed frequency

v
f = f0
v u cos
(Fundamental tone)
300
400 300
= 400 =
1 3
300 15 300
20 4

1
1 B O
= 400 1
÷ Hz (First overtone)
400

1
= 400 1 + ÷ Hz = 401 Hz When stationary performs SHM and displacement amplitude
400 A at a point distant x from one end is given by
26. (b) There will be no Doppler effect when sound is emitted 2 x
at O. By the time sound reaches A, the train will travel a = a0 sin ÷ ......... (1)
S = 15 × 1 = 15m.
where a0 is maximum displacement amplitude which occurs
27. (b) Decrease continuously
at antinode. Since, tension in string is T, therefore, velocity
28. (b) The speed of sound depends on the frame of reference
of the observer. T
of transverse wave is given by v = where m is mass
29. (a) Since all the passengers in train A are moving with a m
velocity of 20 m/s therefore the distribution of sound per unit length of string.
WAVES 387

v a02 2T
Fundamental tone : Since, frequency is n = , therefore, energy of the string in its fundamental tone =
4
frequency of fundamental tone of the string,
v 1 T
1 T First overtone : Frequency, n1 = =
n0 = 1 m
2 m
Considering an elemental length dx of string at a distance
Considering an elemental length dx of string at a distance x
from left end, its mass = m dx.
from left end, its mass = m dx.
1 2 x
1 2 2 Its oscillation energy = (mdx ) a02 .sin 2 ÷ (2 n1 )
2
Its oscillation energy = ( m dx ) a (2 n0 ) 2
2
2 2
a02 2
T 2 x 2 a0 T 2 x
2 = sin 2 ÷ dx
= 2
sin ÷ dx 2
2 2

Total oscillation energy of the string Total oscillation energy of the string

2 2
a02 2
T x a02 2T 2 a0 T 2 x
2
= sin 2 ÷ dx
= 2
sin ÷ dx = 2
2 0
4 0

Since, maximum kinetic energy of a particle performing SHM or maximum kinetic energy of string in its overtone
is equal to its oscillation energy, therefore, maximum kinetic
a02 2T
=

1. (b) Sound travels faster in solids than gases. It is because


v
the elasticity of solid is more than that of gases. Solids 5. (a) Pitch is related to frequency and f =
posses greater density than gases. Though density
has effect on the velocity of sound in the medium as 6. (a) Statement 1 : I µ A2 and I µ f 2
follows 2 2 2
I1 A f1 2
= 1 ÷ = ÷ =1
1 I2 A2 f2 1

Statement 2 : Velocity amplitude : Aw µ I
In case of solid, its elasticity far exceeds that of gas so therefore if velocity amplitude is same then intensity
its effect far exceeds the effect of density. will also be same.
2. (a) Since wavelength is distance between two nearest 7. (a) Both pulses overlap each other, as they are travelling
particles in phase and time period is time required by a is opposite directions.
wave to cover this distance.
dy dy
Using relation v = = vp
wave length dx dt
So, speed of wave =
time period We see that velocity of all particle is zero.
Net energy is potential.
3. (a) When beetle moves along the sand it sends two sets
8. (d) Closed end is pressure antinode therefore pressure is
of pulses, one longitudinal and the other transverse.
not constant. Statement 2 is true.
Scorpion has the capacity to intercept the waves. By
getting a sense of time interval between receipt of these 2
9. (d) At = 1
two waves, it can determine the distance of bettle also.
1 2
4. (d) If temperature of gas is constant change in pressure will µ1 > µ 2 At > Ai
not effect the speed of sound. Hence statement-1 is false. Statement 1 is false.
388 IIT-JEE PHYSICS Challenger

1. (a, b) The displacement of air column to the right of B is 5002 4802


negative (i.e. towards B) while displacement of air = = 0.51
column to the left of B is positive (i.e., towards B) 5002 4602
so density is maximum at B, similarly density is 4. (a,b,d)
minimum at D.
2. (a,b,c,d) y = 10–4 sin (60t + 2x)
Comparing the given equation with the standard 1m
wave equation travelling in negative x-direction
y = a sin ( t + k x) In general, we can write for a closed end pipe that
we get amplitude a = 10–4m (2n 1) c
v= where n = 1, 2, 3,....
Also, = 60 rad/s 4
2 f = 60
4
30 (2n – 1) = =
f= Hz 4 (2n 1)

Also, k = 2 c 3c 5c
= , , ,....
4 4 4
2
=2 = 80, 240, 400....
5. (a,c) The wavelengths possible in an air column in a
= m pipe which has one closed end is
30 4
We know that v = f = = 30 m/s
=
(2n + 1)
3. (a, b) The solid in air, in water and in liquid have three
So, c =
different weights due to the buoyancy.
Let its weight in air be W1, that in water be W2 and 4
300 = 264 ×
that in liquid be W3. 2n + 1
1 T = 264 Hz as it is in resonance with a vibrating
The frequency of the wire is given by f = turning fork of frequency 264 Hz.
2L m
and in the present case f = a constant × T 330 (2n + 1)
=
or, T = kf2
(k is a constant) 264 4
Now, W1 = k (500)2, W2 = k (460)2, W3 = k (480)2 For n = 1, = 0.3125 m = 31.25 cm
From the principle of Archimedes we have the For n = 2, = 0.9375 m = 93.75 cm
specific gravity of solid 6. (a, b, d) wavelength
Weight of solid in air W1 end correction
= =
Loss of weight in water W1 W2 3
1 = + ; 2 = +
4 4
k (500) 2
=
k (500)2 k (460)2 or, 2 3( 1 ) =3 1 2

5002 250000 7. (b, c) The maximum pulse height is 2.0 cm. when
= 2 2
= = 6.51 x – 3.0 t = 0. It is constant.
500 460 38400
Specific gravity of liquid Speed of the pulse

Loss of weight of solid in liquid coefficient of t


= = = 3.0 cm / s
Loss of weight of solid in water coefficient of x
8. (a,b,c) Standing waves are produced by two similar waves
k (500)2 k (480) 2
= superposing while travelling in opposite direction.
k (500)2 k (460) 2 This can happen in case (a), (b) and (c).
WAVES 389

9. (a,c,d) For a plane wave, intensity (energy crossing per 11. (a,c) For a transverse sinusodial wave travelling on a
unit area per unit time) is constant at all points. string, the maximum velocity is a .
But for a spherical wave, intensity at a distance r
v 10
from a point source of power (P), energy But maximum velocity is = = 1m/s
10 10
transmitted per unit area per unit time is given by
a =1
P
I= 10–3 × =1
4 r2
1 103
1 = = Hz
Iµ 2 2 10 3 2
r
The velocity v =
But the total intensity of the spherical wave over
the spherical surface centered at the source v 10
remains constant at all times. = = 3
=2 10 2 m
10 / 2
1
For line source I µ 0.8
r 12. (b,c,d) y=
(4x + 5t)2 + 5
10. (a, b, d) If we use the trigonometric identity,

y x y x 0.8
sin x + sin y = 2 sin cos ÷ .... (1) =
2 2 5
2
The transverse perturbation can be decomposed 16 x + t +5 ... (1)
4
into two transverse waves which move in different
directions : We know that equation of moving pulse is
y = f (x + vt) ... (2)
1 ( x, t ) A sin ( kx t)
....... (2) On comparing (1) and (2), we get
2 ( x, t ) = A sin ( kx + t )
5 2.5
Since 2A = 4, we immediately obtain A = 2cm. The v= ms 1
= ms 1
4 2
velocity of each wave is
So, the wave will travel a distance of 2.5 m in 2 sec.
10
v= = = 40 cm/s ....... (3) 0.8
k /3 y=
The distance between two adjacent junctions can
( 4 x + 5t )2 + 5
be derived from the condition sin (kx ) = 0, which
0.8
leads to: At x = 0, t = 0, y = = 0.16 m
5
kx = n , n = 0, 1, 2, ........ ....... (4)
maximum displacement is 0.16 m
or x = n= n ....... (5)
k 2
The wavelength can be calculated by considering y
the given (x, t) :

2 2
= = = 6cm. ....... (6)
k /3
Therefore, the distance between two adjacent
junctions O x
–x

(n = 1) is : = = 3 cm.
2 The graph for the given equation is drawn. This is
symmetric about y-axis.
390 IIT-JEE PHYSICS Challenger
13. (a, b, c) The equation of standing wave can be written as Only the choices (b) and (c) y
y = 2A sin kx cos t predict these displacements
R Q (L, L)
correctly. This is because (0, L)
2 2 v sin 0 = 0.
where k = and =
Option (a) :
The standing wave is obtained by adding the P
u ( x, y ) = 0 at x = L, y = L x
equation of two identical progressive waves (0, 0) (L, 0)

u ( x, y )
travelling in opposite directions
0 at x = 0, y = 0
y1 = A sin (kx – t), y2 = A sin (kx + t)
Option (b) :
In the present problem the length L of the rod = 1
metre. u ( x, y ) = 0 at x = 0, y = 0 [ sin 0 = 0]
5 2
i.e., L or = metre . u ( x, y ) = 0 at x = L, y = L [ sin = 0]
2 5
Option (c) :
Velocity of longitudinal wave is given by u ( x, y ) = 0 at x = 0, y = 0 [ sin 0 = 0]

Y 2 10 11 u ( x, y ) = 0 at x = L, y = L [ sin 0, sin 2 = 0 ]
v= = = 5 10 3 ms 1
8000 Option (d) :

2 2 u ( x, y ) = 0 at y = 0, y = L [ sin 0 = 0, sin = 0 ]
k= = = 5 metre 1
2/5
u ( x, y ) 0 at x = 0, x = L [ cos 0 = 1, cos 2 = 1]
2 v 5 103 2
= = = (25 103 ) s 1
2/5 16. (b, c) y = 2A sin kx . sin t.
Hence, equation of standing wave is dy
Vy = = 2 A sin kx.cos t
y = 2 × 10–6 sin 5 x cos 25 × 103 t dt
Equations of component waves are 2
Vy 0 t = T / 4, 3T / 4 T = ÷
y1 = (1 × 10–6) sin (5 x – 25 × 103 t)
y2 = (1 × 10–6) sin (5 x + 25 × 103 t) Hence, all the particles will be at rest at T/4 and
3T/4, i.e., two times in one time period.

14. (b,c) As, f =


1 T 17. (a,b,c,d) In the wave motion y a ( kx t ) , y can
2
represent electric and magnetic fields in
electromagnetic waves and displacement and
f µ T
pressure in sound waves.
Given that T1 > T2 18. (b) For such a case, we may consider that the observer
is standing along the stationary surface. Then
f1 > f 2
c+v
Initially beat frequency ( f1 f 2 ) = 6.
f '= f ÷
c
The beat frequency remains unchanged which is
c+v
possible when f2 increases and f1 decreases. Thus Frequency of reflected wave is f '' = f ÷
T2 increases and T1 decreases. c v

15. (b,c) Due to the clamping of the square plate at the


2v
edges, its displacements along the x and y axes Beat freq. = f '' – f = .
c 1
will individually be zero at the edges.
c c(c v)
Wavelength of reflected wave = =
f '' f (c + v)
WAVES 391

1. A-q; B-q; C-q; D-s 3. A-p ; B-s ; C-r


(A) and (B) B is displacement node
L
a = 0, v = 0 and E = 0 and deformation maximum At t = , pulse position will be same as at t = 0 but v
v
PE max.
(D) B is displacement antinode will be in opposite direction (not inverted).
a = max, KE = 0, PE = 0, V = 0 2L
2. A-q; B-p; C-r At t = , pulse position will be same as at t = 0 but
v
(A) Pitch q. frequency
inverted due to reflection at rigid end.
(B) quality p. waveform
(C) loudness r. intensity 3L
At t = , inverted moving towards fix end.
v

1. 9
As 54 waves reaches the shore per minute air 16
or vH = vair = 332 = 1328 m/s
H 1
54 9
f = = Hz 4. 0.12
60 10
And as wavelength of waves is 10 m Tension at the lower end, T1 = 2g N,
9 Tension at the upper end, T2 = 8gN
v=f = × 10 = 9 m/s If v1 and v2 are the speeds at the lower and the upper ends,
10
2. 0.75 respectively, then
For propagation of sound in liquid v2 T2 8
= = =2
v= (B / ) , i.e., B = v2 v1 T1 2

P P Further, if 1 and 2 are the wavelength at the lower and


But by definition B=–V –V = v2 ,
V V the upper ends, respectively, then, since the frequency is
constant,
V( P)
i.e., V= 2
v v2
2
= =2
Here P = H2 g – H1 g = (75 – 200) 13.6 981 1 v1
= –1.667 106 dynes/cm2
or 2 2 0.06 = 0.12 m
(6 )( ) = 0.75 cc
2 1
103 1.667 106
So V= 5. 3
(1.280 )
2
0.81 105 The situation is shown in figure. Suppose the detector is
placed at a distance of x metre from the sources. The wave
3. 1328
received from the source after reflection from the wall has
The velocity of sound in air is given by
traveled a distance of 2[(2) 2 + x 2 / 4]1/ 2 metre. The difference
P
v=
1/ 2
x2
In terms of density and pressure between the two waves is = 2 (2) 2 + x metre.
4
vH PH air air
= = [as Pair = PH]
vair H Pair H Constructive interference will take place when , 2 ,...
392 IIT-JEE PHYSICS Challenger
The minimum distance x for a maximum corresponds to
nH f0 150 3
= …(i) f = nH fH = n0 f0 i.e., =
n0 fH = 550 = 11
u 360 m/s
The wavelength is = = = 2m i.e., the third harmonic of hydrogen and 11th harmonic of
v 180s-1 oxygen or 6th harmonic of hydrogen and 22nd harmonic of
oxygen will have same frequency. So the minimum common
1/ 2
x2 S frequency
Thus, by (i), 2 (2) 2 x=2 f = 3 550 = 1650 Hz
4
8. 110
Let the frequency of the tuning fork be n Hz
x
2 1/ 2 Then frequency of air column at 15ºC = n + 4
x x
or 4+ = 1+ Frequency of air column at 10ºC = n + 3
4 2
According to v = n , we have
D v15 = (n + 4) and v10 = (n + 3)
x2 x2
or 4+ = 1+ +x or 3=x v15 n+4
4 4 =
Thus, the detector should be placed at a distance of 3m from v10 n+3
the sources. Note that there is no abrupt phase change. The speed of sound is directly proportional to the square-
6. 175 root of the absolute temperature.
From the law of length of stretched string, we have
n1 1 = n2 2 = n3 3 v15 15 + 273 288
= =
Here n1 : n2 : n3 = 1 :3 : 15 v10 10 + 273 283
n2 3 n3 15
1
= = 1
= = n+4 288 5
1/2
1
2 n1 1 and 3 n1 1 = = 1+ ÷ 1+ = 1 + 1/2 ×
n+3 283 283 n+3

2
1
=
and 3 =
1
5 5
3 15 =1+
283 566
The total length of the wire is 105 cm
Therefore, 1 + 2 + 3 = 105 1 5
= n + 3 = 113 n = 110 Hz
21 1 n + 3 566
1 1
or 1 + + = 105 or = 105
3 15 15 9. 27

105 15 1 75 1 1 T
1= 15
= 75 cm 2= 3
=
3
= 25 cm ; 3= 15
n (fundamental frequency of the string) =
2 m
75
= = 5 cm 1 T
15 or n = = 20 T
2 0.25 10 2
Hence the bridge should be placed at 75 cm and (75 + 25)
= 100 cm from one end. c
Required sum = 75 + 100 = 175 cm The fundamental frequency of a closed pipe n ' =
4
7. 1650
As diaphram C is a node, A and B will be antinode (as in an 320
n' = = 200Hz
organ pipe either both ends are antinode or one end node 4 0.40
and the other antinode), i.e., each part will behave as closed The frequency of the first overtone of the string
end organ pipe so that
= 2n = 40 T
v 1100
fH = H = = 550 Hz Since there are 8 beats/second, 2n – n' = 0 or 40 T 200 = 8
4 LH 4 0.5
Since on decreasing the tension, the beat frequency
v0 300 decreases,
and f0 = = = 150 Hz
4 L0 4 0.5 2n is definitely greater than n ' .
As the two fundamental frequencies are different, the system
40 T 200 = 8 or T = 27 N
will vibrate with a common frequency f such that
WAVES 393

10. 600 13.. 70


According to Doppler's effect, the apparent frequency when
Thermal stress =
both source and observer move along the same direction is
= 1.3 × 1011 × 1.7 × 10–5 × 20 = 44.2 × 106
(v w ) v0
n = (v w) vs T T
speed = =
m dAL /
( v + w)
Velocity of observer v0 = 0 n = v w vs n T Thermal stress
= =
Given v = 1200 km/hr, w = 40 km/hr, vs = 40 km/hr. and dA density
n = 580 Hz
1200 + 40 4.42 107
n = × 580 = 599.33 Hz = 600 Hz = 70m/s
(1200 40) 40 9 103
11. 0.11 14. 0.075
For interference at A : S2 is behind of S1 by a distance of Mass per unit length of sonometer wire,

100 + (equal to phase difference /2). m


=
0.001
4 µ= = 0.01 kg/m
0.1
Further S2 lags S1 by /2. Hence the waves from S1 and S2
interference at A with a phase difference of T 64 8
v= = = = 8 100
200.5 + 0.5 = 201 = . µ (0.01) 2 0.01
Hence the net amplitude at A is 2a – a = a
For interference at B : S2 is ahead of S1 by a distance of Also, = 0.1 = 0.2
2
100 + (equal to phase difference ).
4 2 v
8 100
f= = = 4000 Hz
0.2
Further S2 lags S1 by . Hence the waves from S1 and S2 Since tuning fork is in resonance, therefore, frequency of
2
interference at B with a phase difference of tuning fork is 4 Hz. The observer is hearing one beat per
200.5 – 0.5 = 200 = 0 . second when the tuning fork is moved away with a constant
Hence the net amplitude at A is 2a + a = 3a . speed b.
2
IA a 1
Hence = ÷ = = 0.11
1
IB ÷ 3a 9
12. 4 10cm
Let the velocities of car 1 and car 2 be v1 m/s and v2 m/s v
Apparent frequencies of sound emitted by car 1 and car 2
as detected at end point are
v v
f1 = f 0 f2 = f0
v v1 , v v2

330 330 The frequency of tuning fork as heard by the observer


330 = 300 ; 360 = 300
330 v1 330 v2 standing stationary near sonometer wire can be found with
the help of Doppler effect.
v2t
v1t c v0 vc
1 2 v' = v = v0 = 0 m/s
c + vs c + vs
A B
100m 4000 300
v' =
v1 = 30 m/s and v2 = 100 m/s 300 + vs
The distance between both the cars just when the Since the beat frequency is 1 and as the tuning fork is going
2nd car reaches point B (as shown in figure is) away from the observer, its apparent frequency is (normal
100m v2t v1t t = 4 sec frequency – 1)
394 IIT-JEE PHYSICS Challenger
= 4000 – 1 = 3999
3 c2
4000 300 v=
3999 = c v 2
300 + vs
3 340 340
3999 × 300 + 3999 vs = 4000 × 300 v= = 1.5m/s
340 340 2
vs = 0.075 m/s
17. 336
15. 11
v
T ( + 0.6 r ) =
4 4f
stress T/A
Also, Y = Y= v = 4f ( + 0.6 r ) = 336 m/s.
strain /
18. 30
T T T Let v be the actual frequency of the whistle. By Doppler's
Y= =
A A T A T effect
T = YA T vS
v' = v
The frequency of the fundamental mode of vibration. vS vt

1 T 1 YA T where vs = Speed of sound = 300 m/s (given)


v= = v' = 2.2 k Hz = 220 Hz (given)
2 m 2 m
300
11 6 5 2200 = v ... (i)
1 2 10 10 1.21 10 20 300 vt
=
2 1 0.1 WHILE THE TRAIN IS RECEDING
= 11 Hz vs
16. 1.5 v'' = v
v s + vt
The apparent frequency from tuning fork T1 as heard by the
observer will be Here, v' = 1.8 kHz = 1800 Hz (given)

v v 300
1800 = v ... (ii)
300 + vt
Dividing (i) and (ii)
T2 T 2200 300 300 + vt
Observer
1800 300 vt 300
c
v1 = v ... (i) 11 300 + vt
c v =
9 300 vt
The apparent frequency from tuning fork T2 as heard by the
observer will be 3300 – 11vt = 2700 = 9vt
600 = 20 vt
c vt = 30 m/s
v2 = v ... (ii)
c+v 19. 336
Given v1 – v2 = 3 The smallest resonating length 1, corresponds to the
1 1 c v c+v fundamental mode. The diameter D, of the pipe is not very
c×v = 3 = cv
c v c+v c2 v2 small compared to the resonating length 1 . So one should
account for the end correction which is generally taken to
c v 2v be in the range of 0.29 D to D/3. Taking the end correction to
3=
c + v2
2 be 0.3 D.
Since, v<<c
1 + 0.3D =
; where is the wavelength of the
c v 2v 4
3= fundamental mode. Then = 70 cm and the velocity of
c2
sound = 336 m/s
1. Consider a pair of insulating blocks with thermal resistances
R1 and R2 as shown in the figure. The temperature at the
boundary between the two blocks is
1
2 1 2
R2 R1

(a) ( 1 2 R1R2) / ( 1 + 2) (R1 + R2)


(b) ( 1R1 + 2R2) / (R1 + R2) 1 2 1 2
(a) (b)
(c) [( 1 + 2) R1R2] / (R12 + R22) 2 t1 1t 2 1t1 2t2
(d) ( 1 R2+ 2R1) / (R1 + R2)
2. One end of a uniform rod of length 1m is placed in boiling 1+ 2 1+ 2
(c) + (d)
water while its other end is placed in melting ice. A point P 2 t1 1t 2 1t1 + 2 t 2
on the rod is maintained at a constant temperature of 800°C. 5. A body cools in a surrounding which is at a constant
The mass of steam produced per second is equal to the temperature of 0. Assume that it obeys Newton’s law of
mass of ice melted per second. If specific latent heat of cooling. Its temperature is plotted against time t. Tangents
steam is 7 times the specific latent heat of ice, the distance are drawn to the curve at the points P( = 2 ) and
of P from the steam chamber must be Q( = 1). These tangents meet the time axis at angle of 2
(a) (1/7) m (b) (1/8) m and 1, as shown.
(c) (1/9) m (d) (1/10) m
3. Two thin walled spheres of different materials, one with
double the radius and one-fourth wall thickness of the other,
2 P
are filled with ice. If the time taken for complete melting of
ice in the sphere of larger radius is 25 minutes and that for 1 Q
smaller one is 16 minutes, the ratio of thermal conductivities 2 1
0 t
of the materials of larger sphere to the smaller sphere is
(a) 4 : 5 (b) 25 : 1
tan tan
(c) 1 : 25 (d) 8 : 25 (a)
2
= 1 0
(b) tan
2
= 2 0

4. In a vertical U-tube containing a liquid, the two arms are tan 1 2 0 1 1 0

maintained at different temperatures, t1 and t2. The liquid


tan tan
columns in the two arms have heights 1 and 2 respectively. (c)
1
= 1
(d)
1
= 2

The coefficient of volume expansion of the liquid is equal to tan 2 2 tan 2 1

MARK YOUR
1. 2. 3. 4. 5.
RESPONSE
396 IIT-JEE PHYSICS Challenger
6. The temperature of an air bubble while rising from bottom to 11. The density of water at 4°C is 1000.0 kg/m3 and at 100°C it is
surface of a lake remains constant but its diameter is doubled 958.4 kg/m3. The cubic expansivity of water between these
if the pressure on the surface is equal to h meter of mercury temperatures is
column and relative density of mercury is then the depth (a) 4.5 × 10–3 / K (b) 5.4 × 10–5 / K
of lake in metre is (c) 4.5 × 10–4 / K (d) 5.4 × 10–6 / K
(a) 2 h (b) 4 h 12. The power radiated by a black body is P, and it radiates
(c) 8 h (d) 7 h maximum energy around the wavelength 0 . If the
temperature of the black body is now changed so that it
7. Two ideal gases at temperature T1 and T2 are mixed. There is
radiates maximum energy around a wavelength 3 0/4, the
no loss of energy. If the masses of molecules of the two
power radiated by it will increase by a factor of
gases are m1 and m2 and number of their molecules are n1
(a) 4/3 (b) 16/9
and n2 respectively, the temperature of the mixture will be
(c) 64/27 (d) 256/81
T1 + T2 T1 T2 13. Two gases occupy two containers A and B the gas in A, of
(a) (b) + volume 0.10m3, exerts a pressure of 1.40 MPa and that in B
n1 + n2 n1 n2
of volume 0.15m3 exerts a pressure 0.7 MPa. The two
n1T1 + n2T2 containers are united by a tube of negligible volume and the
n2T1 + n1T2
(c) (d) gases are allowed to intermingle. Then if the temperature
n1 + n2 n1 + n2
remains constant, the final pressure in the container will be
8. A mixture of n1 moles of monoatomic gas and n2 moles of (in MPa)
(a) 0.70 (b) 0.98 (c) 1.40 (d) 210
Cp
diatomic gas has = 1.5 . 14. A diatomic ideal gas undergoes a thermodynamic change
Cv according to the P-V diagram shown in the figure. The total
heat given to the gas is nearly (use ln 2 = 0.7)
(a) n1 = n2 (b) 2n1 = n2
P
(c) n1 = 2n2 (d) 2n1 = 3n2
9. Two identical containers A and B have frictionless pistons. B
2P0
They contain the same volume of an ideal gas at the same isothermal
temperature. The mass of the gas in A is mA and that in B is
A
mB. The gas in each cylinder is now allowed to expand P0 C
isothermally to double the initial volume. The changes in
the pressure in A and B are found to be p and 1.5 p
respectively. V
(a) 4mA = 9mB (b) 2mA = 3mB V0 2V0
(c) 3mA = 2mB (d) 9mA = 4mB (a) 2.5 P0V0 (b) 1.4 P0V0
10. A monatomic ideal gas, initially at temperature T1, is enclosed (c) 1.1 P0V0 (d) 3.9 P0V0
in a cylinder fitted with a frictionless piston. The gas is 15. One mole of an ideal gas is taken from state A to state B by
allowed to expand adiabatically to a temperature T2 by three different processes, (i) ACB (ii) ADB (iii) AEB as shown
releasing the piston suddenly. If L1 and L2 are the length of in the P-V diagram. The heat absorbed by the gas is–
the gas column before and after expansion respectively, then
P
T1 B
T2 is given by
C D E
2/3
L1 L1
L2 ÷
(a) (b) L2 A V
(a) greater in process (ii) than in (i)
L2
2/3 (b) the least in process (ii)
L2
L1 ÷
(c) (d) (c) the same in (i) and (iii)
L1
(d) less in (iii) than in (ii)

MARK YOUR 6. 7. 8. 9. 10.


RESPONSE 11. 12. 13. 14. 15.
HEAT AND THERMODYNAMICS 397

16. Three rods of the same cross section and made of the same 19. When a system is taken from a state i to a state f along the
material from the sides of a triangle ABC as shown. The path i a f (as shown in the figure), Q = 50 cal and W = 20 cal.
points A and B are maintained at temperatures T and 2T Along path i b f Q = 36 cal.
respectively in the steady state. Assuming that only heat (i) What is W along path i b f ?
conduction takes place, the temperature at point C is (ii) If W = – 13 cal for the return path f i, what is Q for this
A path?
(iii) Take Eint i= 10 cal. What is Eint f ?
60°

C a f
B

2 2+ 3 3 2
(a) T (b) T
2+ 3 2+ 3

2 5 i b
(c) T (d) T
3 2 Volume
17. 5 moles of nitrogen gas are enclosed in an adiabatic (a) + 6, – 43, + 40 (b) + 20, + 40, – 20
cylindrical vessel. The piston itself is a rigid light cylindrical (c) + 13, + 30, – 20 (d) + 15, – 50, + 30
container containing 3 moles of Helium gas. There is a heater 20. The ratio C P /C V for a gas mixture consisting of
which gives out a power 100 cal to the nitrogen gas. A power 8g of helium and 16 g of oxygen is
of 30 cal is transferred to Helium through the bottom surface
(a) 24.2/15 (b) 15/23
of the piston. The rate of increment of temperature of the
(c) 27/17 (d) 17/27
nitrogen gas assuming that the piston moves slowly :
21. An ideal gas heat engine operates in a cycle between 227°C
and 127°C. It absorbs 6.0 × 104 cal at the higher temperature.
How much work per cycle is this engine capable of
He performing?
N2 (a) 1.2 × 103 cal (b) 1.2 × 104 J
(c) 12 × 104 J (d) 1.2 × 104 cal
CP
22. Find the value of for a mixture consisting of n1 moles
(a) 2K/sec (b) 4K/sec CV
(c) 6K/sec (d) 8K/sec of a monoatomic gas and n2 moles of a gas of diatomic
18. Pressure p, volume V and temperature T for a certain gas molecules:
AT BT 2 n1 5n1 7 n2
are related by p = , where A and B are constants. (a) (b)
V n2 3n1 5n2
The work done by the gas as its temperature changes from
3n1 5n2 7 n2 3n1
T1 to T2 while pressure remains constant is (c) (d)
5n1 7 n2 5n1 3n2
B
(a) A (T2 T1 ) 23. Two identical containers A and B with frictionless pistons
2
contain the ideal gas at the same temperature and the same
(b) A(T2 T1 ) B (T22 T12 ) volume V. The mass of the gas in A is mA, and in B is mB. The
gas in each cylinder is now allowed to expand isothermally
A 2 B 3 to the same final volume 2V. The changes in pressure in A
(c) (T2 T12 ) (T2 T13 )
T 3 and B are found to be P and 1.5 P respectively. Then:
(a) 4mA = 9mB (b) 2mA = 3mB
B
(d) A(T2 T1 ) 2 (T2 T1 )3 (c) 3mA = 2mB (d) 9mA = 4mB
3

MARK YOUR 16. 17. 18. 19. 20.


RESPONSE 21. 22. 23.
398 IIT-JEE PHYSICS Challenger
24. In a given process on an ideal gas, dW = 0 and dQ < 0. Then
for the gas
(a) the temperature will decrease 300 K 300 K
vacuum
(b) the volume will increase
(c) the pressure will remain constant 1m 1m
(d) the temperature will increase (a) 1.3 m (b) 1.5 m
25. An ideal gas can be expanded from an initial state to a certain (c) 1.1 m (d) 1.0 m
volume through two different processes, 28. A rod PQ of length l is pivoted at an end P and freely rotated
(A) PV2 = K and (B) P = KV2 , where K is a positive in a horizontal plane at an angular speed about a vertical
constant. Then, choose the correct option from the axis passing through P. If coefficient of linear expansion of
following. material of rod is , find the percentage change in its angular
(a) Final temperature in (A) will be greater than in (B) velocity if temperature of system is increased by T . is
(b) Final temperature in (B) will be greater than in (A) (a) ( T 100)% (b) (2 T 100)%
(c) Work done by the gas in both the processes would be (c) (3 T 100) 100% (d) (4 T 100)%
equal
29. The maximum attainable temperature of ideal gas in the
(d) Total heat given to the gas in (A) is greater than in (B) process
26. 0.5 mole of an ideal gas at constant temperature 27°C kept
inside a cylinder of length L and cross-section area A closed P P0 V2
by a massless piston. where P0 and are +ve constants.

1/ 2 1/ 2
2 P0 P0 P0 2 P0
(a) (b) ÷
L 3nR 3 ÷ 2nR 3 ÷

L 1/ 2 1/ 2
2nR 2 P0 2 P0 P0
(c) ÷ (d) ÷
The cylinder is attached with a conducting rod of length L , P0 3 ÷ nR 2 ÷
cross-section area (1/9) m2 and thermal conductivity k, whose
30. Suppose the pressure P and the density of air are related
other end is maintained at 0°C. If piston is moved such that
n
rate of heat flow through the conducing rod is constant as P/ = constant regardless of height (n is a constant
then velocity of piston when it is at height L/2 from the here). The corresponding temperature gradient is (M is the
bottom of cylinder is : molecular weight of air)
[ neglect any kind of heat loss from system ]
Mg (n + 1) Mg (1 n)
(a) (b)
k k R nR
(a) ÷ m / sec (b) ÷ m / sec
R 10R
Mg (1 n2 ) Mg (n 2 + 1)
(c) (d)
k k nR R
(c) ÷ m / sec (d) ÷ m / sec
100R 1000R 31. Three moles of an ideal gas being initially at a temperature
27. Consider the shown diagram where the two chambers T0 = 273 K were isothermally expanded = 5.0 time its
separated by piston-spring arrangement contain equal initial volume and then osochorically heated so that the
amounts of certain ideal gas. Initially when the temperatures volume and pressure in the final state become equal to that
of the gas in both the chambers are kept at 300 K. The in the initial state. The total amount of heat transferred to
compression in the spring is 1m. The temperature of the left the gas during the process equals Q = 80 kJ. The ratio
and the right chambers are now raised to 400 K and 500 K = C p / Cv for this gas is
respectively. If the pistons are free to slide,the compression
(a) 1.4 (b) 1.33
in the spring is about.
(c) 1.5 (d) 1.6

MARK YOUR 24. 25. 26. 27. 28.


RESPONSE 29. 30. 31.
HEAT AND THERMODYNAMICS 399

32. A certain amount of ideal monoatomic gas undergoes, 36. Three discs A, B and C having radii 2 m, 4m and 6 m
process given by UV1/2 = C where U is the internal energy respectively are coated with carbon black on their outer
of the gas. The molar specific heat of the gas for the process surfaces. The wavelengths corresponding to the maximum
will be intensity are 300 nm, 400 nm, 500 nm respectively. The power
(a) R/2 (b) 3R radiated by the are QA, QB and QC respectively. Then
(c) 5R/2 (d) –R/2 (a) QA is maximum (b) QB is maximum
33. Three pieces of metallic heat conductors X, Y, Z of equal (c) QC is maximum (d) QA = QB = QC
lengths and cross-sectional area are connected in series as 37. An ideal gas whose adiabatic exponent equals is expanded
shown in figure. Ends A and D are maintained at 0°C and so that the amount of heat transferred to the gas is equal to
100°C respectively. If Y is made up of same material as that the decrease of its internal energy. Then the equation of the
of Z, point B is at 20°C. What will be the temp of B, if Y is
process in the variables T and V
made up of a material same as that of X. Consider only
conduction and laterally insulated surfaces of conductors. ( –1) ( –2)

0°C 100°C (a) TV 2 =C (b) TV 2 =C

X Y Z ( –1) ( –2)

A B C D (c) TV 4 =C (d) TV 4 =C
(a) 25°C (b) 50°C 38. One mole of a diatomic gas is taken through the process
(c) 40°C (d) 20°C PV n = k, where n and k are constant. If the heat capacity of
34. 12 identical rods made of same material are arranged in the gas is negative, then the value of n may be
form of a cube. The temperature of P and R are maintained
5 5
at 90°C and 30°C respectively. Then the temperature of point (a) (b) –
V, when steady state is reached. 7 7
T U
9 9
(c) (d) –
P Q a 7 7
90°C 39. An ideal monoatomic gas is taken round the cycle ABCDA
as shown in the P – V diagram (see Fig.). The work done
W during the cycle is
V
30°C P
S R
(a) 65°C (b) 60°C 2P, V 2P, 2V
B C
(c) 20°C (d) 50°C
35. ABCDE is a regular pentagon of uniform wire. The rate of
heat entering at A and leaving at C is equal. TB and TD are A D
temperature of B and D. Find the temperature TC. P, V P,2V
TB V
B (a) PV (b) 2 PV
(d /dt )=1 (d /dt )=1
A C 1
(c) PV (d) zero
2
E D 40. A thermodynamical process is shown in the figure with PA =
TD 3 × Patm, VA = 2 × 10–4 m3, PB = 8 × Patm, VC = 5 × 10–4 m3, in
3TB + 2TD the process AB and BC, 600 J and 200 J heat are added to the
(a) (b) 3TD – 2TB system, respectively. Find the change in internal energy of
5
the system in the process CA. (1 Patm = 105 N/m2)
(c) 3TD + 2TB (d) can have any value

MARK YOUR 32. 33. 34. 35. 36.


RESPONSE 37. 38. 39.
400 IIT-JEE PHYSICS Challenger
44. Two adiabatic vessels, each containing the same mass m of
P water but at different temperatures, are connected by a rod
of length L, cross – section A, and thermal conductivity K.
B C The ends of the rod are inserted into the vessels, while the
PB
rest of the rod is insulated so that there is negligible loss of
heat into the atmosphere. The specific heat capacity of water
is s, while that of the rod is negligible. The temperature
PA 1
A difference between the two vessels reduces to of its
e
VA VC V original value after a time t. The thermal conductivity (K)
of the rod may be expressed by
(a) 560 J (b) – 560 J msL emsL
(c) – 40 J (d) + 40 J (a) (b)
A t A t
41. If a given mass of an ideal gas followed a relation VT =
constant during a process, then which of the following msL msL
(c) (d)
graphs correctly represents the process ? 2eA t 2A t
45. What is the equivalent thermal conductivity of the rods in
P P figure given below , if the length of each cylinder be and
area of cylinder having thermal conductivities K1 and K3 be
A while that of the middle cylinder having thermal
(a) (b) conductivity K2 be 2A?
T T
K2 Area 2A
K1 K3
P P
Area A

l l l
(c) (d)
T T 5 1
(a) (b)
1 1 1 1 1 1
42. A monoatomic ideal gas is taken through a reversible process 2 + + + +
whose equation is given by : K1 2 K 2 K 3 K1 2 K 2 K 3
1
2
p = kV 2 (c) (d) None
1 1 1
where p is the pressure and V is the volume of the gas. 5 + +
K1 2 K 2 K 3
The molar heat capacity of the gas in the above process, is
(a) CP + 2R (b) CV – 2R 46. Heat is supplied to a certain homogeneous sample of matter
(c) CV + 2R (d) CP + 2R at a uniform rate. Its temperature is plotted against time as
43. The heat (Q) supplied to a solid, which is otherwise thermally shown in the figure.
isolated from its surroundings, is plotted as a function of its
absolute temperature, . It is found that they are related by
the equation.
Q = a 2 + b 4. (a, b are constants).
The heat capacity of the solid is given by
Temp.
3 5
(a) a +b (b) a + b 3
3 5
3
(c) a +b (d) 2a + 4b 3 Time
3 5

MARK YOUR 40. 41. 42. 43. 44.


RESPONSE 45.
HEAT AND THERMODYNAMICS 401

Which of the following conclusions can be drawn? 50. Match the column according to the bulk modulus
(a) Its specific heat capacity is greater in the solid state dP
than in the liquid state ÷ for different processes, where symbols have
dV / V
(b) Its specific heat capacity is greater in the liquid state
than in the solid state usual meaning.
(c) Its latent heat of vaporization is smaller than its latent Process Bulk modulus
heat of fusion (A) isothermal process (i) 0
(d) None of these (B) adiabatic process (ii)
47. The value of Cp – Cv is 1.00R for a gas sample in state A and (C) isobaric process (iii) P
is 1.06R in state B. Let pA, pB denote the pressure and TA , (D) isochoric process (iv) P
TB denote the temperature of the states A and B respectively. (a) (A)-iv, (B)-iii, (C)-ii, (D)-i
Most likely – (b) (A)-iii, (B)-iv, (C)-ii, (D)-i
(a) pA < pB and TA > TB (c) (A)-iii, (B)-iv, (C)-i, (D)-ii
(b) pA > pB and TA < TB (d) (A)-iii, (B)-ii, (C)-iv, (D)-i
(c) pA = pB and TA < TB 51. The relation between internal energy U, pressure P and
(d) pA > pB and TA = TB volume V of an ideal gas in an adiabatic process is
U = 2 + 3PV. What is the value of the ratio of the molar
48. P
CP
a
CV ÷
specific heats ?
b
c (a) 2/3 (b) 4/3
d
(c) 3/2 (d) 1
e 52. The graph below shows the variation with temperature of
f the volume of a fixed mass of water.
V
The pressure-volume diagram shows six curved paths that
Volume/cm3

2.0004
can be followed by the gas (connected by vertical paths).
Which two of them should be part of a closed cycle if the 2.0003
net work done by the gas is to be its maximum positive
value? 2.0002

(a) af (b) ae 2.0001


(c) ac (d) ce
49. An ideal gas is initially at temperature T and volume V. Its 2.0000
274 275 276 277 278 279 280 281
volume is increased by V due to an increase in temperature Temperature/K
V
T, pressure remaining constant. The quantity = A thermometer is to be designed that measures temperatures
V T
in the range between 274K and 280K. Which one of the
varies with temperature as
following is the main reason why the variation with
Y Y temperature of the volume of the water is not suitable to use
in the thermometer ?
(a) Water is a colourless liquid
(a) (b)
o X o X (b) Water freezes at 273 K
T T+ T T T+ T (c) The change in volume is too small over the temperature
Temperature K Temperature K range
(d) The volume has the same value at more than one
Y temperature
Y
53. For a monoatomic ideal gas undergoing an adiabatic change,
the relation between temperature and volume is
(c) o X (d) TVx = constant where x is
T T+ T o X
T T+ T (a) 7/5 (b) 2/5
Temperature K
Temperature K (c) 2/3 (d) 1/3

MARK YOUR 46. 47. 48. 49. 50.


RESPONSE 51. 52. 53.
402 IIT-JEE PHYSICS Challenger
54. In the following P-V diagram of an ideal gas, two adiabates 59. If the molecules in a tank of hydrogen have the same RMS
cut two isotherms at T1 = 300K and T2 = 200K. The value of speed as the molecules in another tank of oxygen, we may
VB/VC is be sure that
(a) the pressure are the same
(b) the hydrogen is at the higher temperature
(c) the temperature are the same
(d) the oxygen is at the higher temperature
60. The state of an ideal gas is changed through an isothermal
process at temperature T0 as shown in figure. The work
done by gas in going from state B to C is double the work
done by gas in going from state A to B. If the pressure in the
state B is P0/2 then the pressure of the gas in state C is
(a) = VA/VD (b) < VA/VD P
(c) > VA/VD (d) 3/2
55. A 2 kW continuous flow geyser (instant geyser) has inlet
P0 A
water temperature of 10°C, while water is coming out from
the tap at the rate of 20 g/s. The temperature of the water P0 B
coming out from the tap must be about [Assume no loss of 2 C
energy in other forms. Take sw = 4.2 J/g°-C] T0
(a) 45°C (b) 33.8°C
(c) 23.8°C (d) None of these V
56. The quantity of gas in a closed vessel is halved and the (a) P0/2 (b) P0/4
velocities of its molecules are doubled. The final pressure (c) P0/6 (d) P0/8
of the gas will be 61. A system is taken from a given initial state to a given final
(a) P (b) 2P state along various paths represented on a P–V diagram.
(c) P/2 (d) 4P The quantity that is independent of the path is
57. Soap bubbles filled with helium float in air. Which has the (a) amount of heat transferred Q
greater mass : the wall of a bubble or the gas enclosed within (b) amount of work done W
it (Assume pressure inside and outside the bubble to be (c) (Q – W)
same) ? (d) none of these
(a) the wall of bubble (b) the enclosed gas 62. P-V plots for two gases during adiabatic processes are
(c) mass of both are equal shown in the figure. Plots 1 and 2 should correspond
(d) any one can be greater depending on size of bubble respectively to
58. Three moles of an ideal monoatomic gas perform a cycle P
shown in figure. The gas temperatures in different states
are T1 = 200K, T2 = 400K, T3 = 1600 K, and T4 = 800K. The
work done by the gas during the cycle is 1
(Take R = 25/3 J/mol-K) 2
V
P
2 3 (a) He and O2 (b) O2 and He
(c) He and Ar (d) O2 and N2
63. Why does the pressure of an ideal gas increase when it is
1 heated at constant volume ?
4
(a) The gas molecules expand
T (b) The molecules move at the same speed, but hit the
walls more often
(a) 5 kJ (b) 25 kJ
(c) 15 kJ (d) 20 kJ (c) The molecules move faster and hit the walls more often
(d) The number of molecules of gas increases

MARK YOUR 54. 55. 56. 57. 58.


RESPONSE 59. 60. 61. 62. 63.
HEAT AND THERMODYNAMICS 403

64. Which of the following will have maximum total kinetic comfortable, that is why the electric stove with power of
energy at temperature 300K ? 1kW is also plugged in. Together these two devices maintain
(a) 1 kg H2 (b) 1 kg He the room temperature of 22°C. Determine the thermal power
of the heater.
1 1 1 3
(c) kg H 2 + kg He (d) kg H 2 + kg He (a) 3 kW (b) 4 kW
2 2 4 4
(c) 5 kW (d) 6 kW
65. When an ideal monoatomic gas is heated at constant 70. A black plane surface at a constant high temperature TH, is
pressure, the fraction of the heat energy supplied which parallel to another black plane surface at constant lower
increases the internal energy of the gas is temperature TL. Between the plates is vacuum. A heat shield
(a) 2/5 (b) 3/5 consisting of a thin black plate is placed between the warm
(c) 3/7 (d) 5/7 and the cold surfaces and parallel to these. After some time
66. A mass of material exists in its solid form at its melting stationary conditions are obtained. By what factor is the
temperature 10°C. The following processes then occur to stationary heat flow between the plane surface reduced due
the material : to the presence of the heat shield ? Neglect end effects due
Process 1 : An amount of thermal energy Q is added to the to the finite size of the surfaces.
material and 3/4 of the material melts. (a) 16 (b) 2
Process 2 : An identical additional amount of thermal energy (c) 4 (d) 21/4
Q is added to the material and the material is now a liquid at 71. Certain perfect gas is found to obey PV3/2 = constant during
50°C. an adiabatic process. If such a gas at initial temperature T is
What is the ratio of the latent heat of fusion to the specific adiabatically compressed to half the initial volume, its final
heat of the liquid for this material ? temperature will be
(a) 80°C (b) 60°C (a) (b) 2T
2T
(c) 40°C (d) none of these
67. Temperature of a body is only on manifestation of the mean (c) 2 2T (d) 4T
(a) total mechanical energy of a molecule of the body 72. If in defining the specific heat, temperature is represented in
(b) potential energy of a molecule of the body °F instead of °C then the value of specific heat will
(c) rotational kinetic energy of a molecule of the body (a) be converted to heat capacity
(d) translational kinetic energy of a molecule of the body (b) remain unchanged
68. The expansion of a perfect gas of mass 0.01 kg at constant (c) decrease
pressure is shown in the figure where x and y respectively (d) increase
represent
73. At very high temperatures vibrational degrees also becomes
y
active. At such temperatures an ideal diatomic gas has a
molar specific heat at constant pressure, Cp is
O (a) 3R/2 (b) 5R/2
x
(c) 6R/2 (d) 9R/2
74. The relation between U, P and V for an ideal gas in an
adiabatic process is given by relation U = a + bP V. Find the
(a) temperature (°C) and volume
value of adiabatic exponent ( ) of this gas.
(b) temperature (K) and volume
(c) volume and temperature (°C) b +1 b +1
(a) (b)
(d) volume and temperature (K) b a
69. The room heater can maintain only 16°C in the room when
a +1 a
the temperature outside is –20°C. It is not warm and (c) (d)
b a+b

64. 65. 66. 67. 68.


MARK YOUR
69. 70. 71. 72. 73.
RESPONSE
74.
404 IIT-JEE PHYSICS Challenger

constant pressure till the volume is tripled. It then undergoes an


PASSAGE-1
adiabatic change until the temperature returns to its initial value.
The system shown in figure consists of three springs and two
4. Final volume of the gas will be
rods as shown. The temperature of the rods is increased by T.
The springs are initially relaxed. There is no friction. The coefficient (a) 6 3 litres (b) 9 3 litres
of linear expansion of the material of rods is equal to .
2/5 2/5
3 3
(c) 6 ÷ litres (d) 9 ÷ litres
2 2
L L/2
5. Final pressure of the gas will be
k 2k 3k
S1 S2 S3 8.31 104 8.31 104
(a) N/m2 (b) N/m2
3 3/ 2

1. The energy stored in spring S1 is 8.31 104 8.31 104


(c) N/m2 (d) N/m2
81 27 (3) 2 / 5 [3 / 2]2 / 5
(a) k L2 2
( T )2 (b) k L2 2
( T )2
484 242 6. Work done by the gas in the whole process will
approximately be
81 49
(c) k L2 2
( T ) 2 (d) k L2 2
( T )2 (a) 1496 joule (b) 2244 joule
242 484
(c) 3740 joule (d) zero
2. The energy stored in spring S2 is

81 27
(a) k L2 2
( T )2 (b) k L2 2
( T )2 PASSAGE-3
484 242
A diathermic piston divides adiabatic cylinder of volume V0 into
81 49 two equal parts as shown in the figure. Both parts contain ideal
(c) k L2 2
( T ) 2 (d) k L2 2
( T )2
242 484 monoatomic gases. The initial pressure and temperature of gas in
left compartment are P0 and T0 while that in right compartment are
3. The energy stored in spring S3 is
2P0 and 2T0. Initially the piston is kept fixed and the system is
81 27 allowed to acquire a state of thermal equilibrium.
(a) k L2 2
( T )2 (b) k L2 2
( T )2
484 242

81 49
(c) k L2 2
( T ) 2 (d) k L2 2
( T )2
242 484
P0 , T0 2 P0 , 2T0

PASSAGE-2

0.3 mole of a monoatomic gas ( = 5/3) are initially at a temperature


of 27°C and has a volume 1 litre. The gas is first expanded at

MARK YOUR 1. 2. 3. 4. 5.
RESPONSE 6.
HEAT AND THERMODYNAMICS 405

7. The pressure in left compartment after thermal equilibrium


is achieved is P B
C
3
(a) P0 (b) P0 (b)
2
4 A
(c) P0 (d) none of these
3 V
8. The heat that flown from right compartment to left
compartment before thermal equilibrium is achieved is P C
3
(a) P0V0 (b) P0V0
4 (c)
3 2 A B
(c) P0V0 (d) P0V0
8 3
9. If the pin which was keeping the piston fixed is removed V
and the piston is allowed to slide slowly such that a state of (d) None of these
mechanical equilibrium is achieved. The volume of left 11. The total amount of heat absorbed by the system for cyclic
compartment when piston is in equilibrium. process is
3 V0
(a) V0 (b) 10 10
4 4 (a) In 2.5 – 2 ÷ U 0 (b) In 0.4 – 2 ÷ U 0
3 5
V0 2
(c) (d) V0
2 3 (c) 50 U0 (d) None of these
12. The work done in process AB is
PASSAGE-4 (a) – U0 (b) – 2U0
Figure shows the variation of the internal energy U with the (c) – 5U0 (d) None of these
density of one mole of ideal monoatomic gas for a
thermodynamics cycle ABCA. Here process AB is a part of
rectangular hyperbola PASSAGE-5

U In the figure n mole of a monoatomic ideal gas undergo the process


A C ABC as shown on the adjacent P-V diagram. The process AB is
5U0
isothermal and BC is isochoric. The temperature of the gas at A is
T0. Total heat given to the gas during the process ABC is measured
to be Q.
2U0 B

A C
P
2 0 5 0

10. The P-V diagram of above process


P C
B
(a)
2V 6V
B A

MARK YOUR 7. 8. 9. 10. 11.


RESPONSE 12.
406 IIT-JEE PHYSICS Challenger
13. Temperature of the gas at C is equal to
PASSAGE-7
(a) T0 (b) 3 T0
(c) 6 T0 (d) 2 T0 One mole of a diatomic gas is heated under a “Kibolinsky process”
14. Heat absorbed by the gas in the process BC in which gas pressure is temperature controlled according to law
(a) 3nRT0 (b) nRT0 4
P = CT 3/ 2 where C is Kibolinsky constant. In this process, gas
(c) 2nRT0 (d) 6nRT0. 5
15. The average molar heat capacity of the gas in process ABC is heated by 300K temperature.
is 19. The amount of work done by the gas under above heating
is
Q Q
(a) (b) (a) 600R (b) 750R
nT0 2nT0
(c) –150R (d) None of these
Q 2Q 20. The amount of heat supplied to the gas under above heating
(c) (d) is
3nT0 nT0
(a) 600R (b) 750R
(c) –150R (d) None of these
PASSAGE-6 21 Under above process the molar specific heat of gas is
In a thermally insulated tube of cross sectional area 4cm2 there is (a) R/2 (b) R
a liquid of thermal expansion coefficient of 10–3K–1 flowing. Its (c) 2R (d) None of these
velocity at the entrance is 0.1 m/s. At the middle of the tube a
heater of a power of 10kW is heating the liquid. The specific heat
capacity of the liquid is 1.5 kJ/(kg K), and its density is 1500 kg/m3 PASSAGE-8
at the entrance.
16. The rise in temperature of the liquid as it pass through the
A body cools in a surrounding of constant temperature 30°C.
tube is
Its heat capacity is 2J/°C. Initial temperature of the body is
1000 1 40°C. Assume Newton’s law of cooling is valid. The body
(a) °C (b) °C cools to 36°C in 10 minutes.
9 9
22. In further 10 minutes it will cool from 36°C to
500 (a) 34.8°C (b) 32.1°C
(c) °C (d) None of these
9 (c) 32.8°C (d) 33.6°C
17. What is the density of liquid at the exit ? 23. The temperature of the body in °C is denoted by . Then the
(a) 1450 kg/m3 (b) 1400 kg/m3 variation of versus time t is best represented by
(c) 1350 kg/m3 (d) None of these 40°C
18. How much bigger is the volume rate of flow at the end of the
tube than at the entrance in cubic meters ?
(a) 30°C
1
(a) 9 × 10–5 (b) × 10–5
3
(0,0) t
4
(c) × 10–5 (d) None of these
9

13. 14. 15. 16. 17.


MARK YOUR
18. 19. 20. 21. 22.
RESPONSE
23.
HEAT AND THERMODYNAMICS 407

40°C 27. The summation of work done by the gas and the change in
the internal energy of the gas is
(a) 1 (b) –1
(b) 30°C (c) (d) 0

(0,0) t PASSAGE-10

40°C
A quantity of an ideal monoatomic gas consists of n moles
initially at temperature T1. The pressure and volume are
then slowly doubled in such a manner so as to trace out a
(c)
straight line on P-V diagram.

W
(0,0) t 28. For this process, the ratio is equal to (where W is
nRT1
40°C
work done by the gas)
(a) 1.5 (b) 3
(c) 4.5 (d) 6
(d) 30°C
Q
29. For the same process, the ratio is equal to (where Q
nRT1
(0,0) t
is heat supplied to the gas)
24. When the body temperature has reached 36°C it is heated
(a) 1.5 (b) 3
again so that it reaches to 40°C in 10 minutes. Assume that
the rate of loss of heat at 38°C is the average rate of loss for (c) 4.5 (d) 6
the given time. The total heat required from a heater by the 30. If C is defined as the average molar specific heat for the
body is process then C/R has value
(a) 7.2 J (b) 0.728 J (a) 1.5 (b) 2
(c) 16 J (d) 32 J (c) 3 (d) 6

PASSAGE-9 PASSAGE-11

A monoatomic ideal gas is filled in a non-conducting


A brass ball of mass 100g is heated to 100°C and then dropped
container. The gas can be compressed by a movable
into 200g of turpentine in a calorimeter at 15°C. The final
nonconducting piston. The gas is compressed slowly to
12.5% of its initial volume. temperature is found to be 23°C. Determine the specific heat of
25. The final temperature of the gas if it is T0 initially is turpentine. Take specific heat of brass as 0.092 cal/g°C and water
(a) 4T0 (b) 3T0 equivalent of calorimeter as 4g.
(c) 2/3 T0 (d) T0 31. Heat lost by the ball is approximately
26. The initial adiabatic bulk modulus of the gas is Bi and the (a) 810 cal (b) 610 cal
final value of the adiabatic bulk modulus of the gas is Bf . (c) 710 cal (d) 510 cal
Then 32. Specific heat of the brass is
(a) 32Bi = Bf (b) Bi = Bf (a) 0.62 cal/g°C (b) 0.52 cal/g°C
(c) Bi = 32Bf (d) Bi = 4Bf (c) 0.42 cal/g°C (d) 0.32 cal/g°C

MARK YOUR 22. 23. 24. 25. 26.


RESPONSE 27. 28. 29. 30.
408 IIT-JEE PHYSICS Challenger

1. Statement - 1 : The equivalent thermal conductivity of Statement - 2 : Peak emission wavelengths of a black body
two plates of same thickness in contact is is proportional to the fourth-power of
less than the smaller value of thermal temperature.
conductivity. 6. Statement - 1 : Temperature of real gases often changes
Statement - 2 : For two plates of equal thickness in when undergoing a free expansion under
contact the equivalent thermal adiabatic condition whereas that of an ideal
conductivity is given by gas does not.
Statement - 2 : First law of thermodynamics holds for ideal
1 1 1
= + . gases only.
K K1 K 2

Cp
2. Statement - 1 : The isothermal curves intersect each other 7. Statement - 1 : The ratio of for an ideal diatomic gas
at a certain point. Cv

Statement - 2 : The isothermal change takes place slowly, is less than that for an ideal monoatomic
so, the isothermal curves have very little gas (where C p and C v have usual
slope. meaning).
Statement - 2 : The atoms of a monoatomic gas have less
3. Statement - 1 : Melting of solid causes no change in degrees of freedom as compared to
internal energy. molecules of the diatomic gas.
Statement - 2 : Latent heat is the heat required to melt a
unit mass of solid. 8. Statement - 1 : The Carnot cycle is useful in
understanding the performance of heat
engines.
4. Statement - 1 : The total translational kinetic energy of all
the molecules of a given mass of an ideal Statement-2 : The Carnot cycle provides a way of
diatomic gas is 2.5 times the product of its determining the maximum possible
pressure and its volume. efficiency achievable with reservoirs of
given temperatures.
Statement - 2 : The molecules of a gas collide with each
other and the velocities of the molecules
change due to the collision. 9. Statement - 1 : When temperature rises the coefficient of
5. Statement - 1 : For higher temperature the peak emission viscosity of gases decreases.
wavelength of a blackbody shifts to lower Statement - 2 : Gases behave more like ideal gases at
wavelengths. higher temperature.

MARK YOUR 1. 2. 3. 4. 5.
RESPONSE 6. 7. 8. 9.
HEAT AND THERMODYNAMICS 409

10. Gas in cylinder with light piston is slowly heated till all the Statement - 2 : According to first law of thermodynamics
mercury spills over. (Patm = 76cm of Hg column) dQ = dU + dW where symbols have their
usual meaning.
11. Statement - 1 : When a glass of hot milk is placed in a
room and allowed to cool, its entropy
decreases.
76cm
Statement - 2 : Allowing hot object to cool does not
violate the second law of thermodynamics.

76cm 12. Statement - 1 : Equal moles of helium and nitrogen gases


(considering both to be ideal) are given
equal quantities of heat at constant
volume. There will be a greater rise in the
temperature of helium as compared to that
Statement - 1 : The temperature of gas continuously of nitrogen.
increases. Statement - 2 : The molecular weight of nitrogen is more
than the molecular weight of helium.

MARK YOUR
10. 11. 12.
RESPONSE

1. A cubical rigid container of edge l = 10 cm, thickness of wall


R
3
(a) is Cv + ÷ if the gas undergoes the process
d = 4mm and thermal conductivity K 8.31 10 W/mK V
has one mole of a monoatomic gas at temperature
T = T0 e V
T0 = 400 K . If the temperature of the surrounding is
2R
Ts = 300 K , then (b) is Cv + ÷ if the gas undergoes the process
V
(a) the time when pressure becomes 5/6 of initial pressure
is 110 sec approx. T = T0 e V

(b) the temperature at the instant when pressure becomes


R
5/6 of initial pressure is 333.33K. (c) is Cv + if the gas undergoes the process
(1 V)
(c) the time when pressure becomes 5/6 of initial pressure
is 70 sec appx:
P = T0 e V
(d) the temperature at the instant when pressure becomes
5/6 of inital pressure is 111.11 K. R
(d) is Cv + ÷ if the gas undergoes
2. An ideal gas has a molar heat capacity Cv at constant V
volume. The molar heat capacity of this gas as a function of
its volume, V process P = T0 e V

MARK YOUR
1. 2.
RESPONSE
410 IIT-JEE PHYSICS Challenger
3. Two bodies A and B ahave thermal emissivities of 0.01 and
0.81 respectively. The outer surface areas of the two bodies
are the same. The two bodies emit total radiant power of the
B
same rate. The wavelength B corresponding to maximum
spectral radiancy in the radiation from B shifted from the A
wavelenth corresponding to maximum spectral radiancy in V0 P0
the radiation from A, by 1.00 m. If the temperature of A is Q
5802 K :
V
(a) the temperature of B is 1934 K
Chamber of volume V
(b) B = 1.5 m
(a) its internal energy is equal to 7 P0V0
(c) the temperature of B is 11604 K
(b) its temperature is equal to 1.4 T0
(d) the temperature of B is 2901 K
1.4 P0V0
4. One mole of an ideal gas is taken through the cyclic process (c) its pressure is equal to
V
shown in the V-T diagram, where V = volume and T = absolute
temperature of gas. Which of the following statemens is/are (d) None of these
6. For an ideal gas :
correct?
(a) the change in internal energy in a constant pressure
V process from temperature T1 to T2 is equal to nCv
A B (T2 – T1), where Cv is the molar specific heat at constant
2V0
volume and n the number of moles of the gas.
(b) the change in internal energy of the gas and the work
V0 C done by the gas are equal in magnitude in an adiabatic
D
process.
(c) the internal energy does not change in an isothermal
T0 2T0 T process.
(d) no heat is added or removed in an adiabatic process.
(a) Heat is given out by the gas in the whole process
7. For any monoatomic gas the quantity/quantities
(b) Heat is absorbed by the gas in the complete cycle independent of the nature of the gas at the same temperature
(c) The magnitude of the work by the gas is RT0 ln 2 is/are
(a) the number of molecules in one mole
(d) The magnitude of the work done by the gas is V0T0
(b) the number of molecules in equal volume and pressure.
5. In the figure shown, a horizontal cylindrical tube connected (c) the translational kinetic energy of one mole
to the vertical wall of a chamber of volume V, contains V0 (d) the kinetic energy of unit mass
volume of air at atmospheric pressure P0 and temperature 8. Choose the correct statement(s)
T0 by means of a movable piston ‘A’ and a valve ‘B’. In the (a) In an adiabatic expansion the product of pressure and
beginning valve is closed and the chamber is evacuated. volume decreases.
The valve gets opened and air starts leaking into the (b) The rms translational speed for all ideal gas molecules
chamber very slowly. The volume V >> V0. (Take the molar at the same temperature is not the same but it depends
on the mass.
5 (c) Temperature of an ideal gas is doubled from 100°C to
heat capacity of air at constant volume equals R, where
2 200°C. The average kinetic energy of each particle is
also doubled.
R is the universal gas constant) When air occupies the total
(d) The rms translational speed for all ideal gas molecules
volume of the chamber. All walls and piston are made up of at the same temperature is not the same and does not
insulating material. depend on the mass.

MARK YOUR 3. 4. 5. 6. 7.
RESPONSE 8.
HEAT AND THERMODYNAMICS 411

9. One mole of a gas is subjected to two processes AB and BC, (d) After thermal equilibrium, the level of liquid in
one after the other as shown in the figure. BC is represented thermometer in contact with body A is more its initial
by PVn = constant. level while the level of liquid in thermometer in contact
We can conclude that (where T = temperature, W = work with body B may be same as its initial level.
done by gas, V = volume and U = internal energy). 12. A black body of temperature T is inside chamber of T0
temperature initially. Sun rays are allowed to fall from a hole
P
in the top of chamber. If the temperature of black body (T)
A B and chamber (T0) remains constant, then

C
T0
V T
V0 2V 0 3V 0

(a) TA = TB = TC (b) VA < VB, PB < PC


(c) WAB > WBC (d) TA < TB (a) Black body will absorb more radiation
10. A bimetallic strip is formed out of two identical strips one of (b) Black body will absorb less radiation
copper and the other of brass. The coefficients of linear (c) Black body emit more energy
expansion of the two metals are c and B. On heating, the
(d) Black body emit energy equal to energy absorbed by it
temperature of the strip goes up by T and the strip bends
to form an arc of radius of curvature R. Then R is. 13. Hydrogen gas and oxygen gas have volume 1cm3 each at
NTP.
(a) proportional to T
(a) Number of molecules is same in both the gases
(b) inversely proportional to T
(b) The rms velocity of molecules of both the gases is the
(c) proportional to | B – C|
same
(d) inversely proportional to | B – C|
(c) The internal energy of each gas is the same.
11. Graph in figure shows the variation in density of a liquid
substance with temperature. It can be seen that density of (d) The average velocity of molecules of each gas is the
this substance is minimum at temperature TC. This liquid is same.
used as a thermometer fluid in bulb of a thermometer and 14. An ideal gas is taken from the state A (pressure P, volume V)
two such identical thermometers are used to measure to the state B (pressure P/2, volume 2V) along a straight line
temperature of two bodies A and B which are at temperature path in the P-V diagram. Select the correct statement (s)
T1 and T2. Now in an isolated system A and B are brought in from the following:
contact so that they will be in thermal equilibrium. Which of (a) The work done by the gas in the process A to B exceeds
the following statements is/are correct. the work that would be done by it if the system were
taken from A to B along the isotherm.
(b) In the T-V diagram, the path AB becomes a part of a
parabola
(c) In the P-T diagram, the path AB becomes a part of a
hyperbola
1 (d) In going from A to B, the temperature T of the gas first
2
T increases to a maximum value and then decreases.
T1 Tc T2 15. A thermos bottle contains coffee. The thermos bottle is
vigorously shaken. Consider the coffee as the system.
(a) Initially liquid level in thermometer of body A is lower
than that of B. Choose the correct statement (s).
(b) After thermal equilibrium, the level of liquid in both (a) Its temperature would rise
thermometer are less than initial level of both. (b) Heat has been added to it
(c) After thermal equilibrium, the level of liquid in both (c) Work has been done on it
thermometer may be more than initial level of both. (d) Its internal energy has changed

MARK YOUR 9. 10. 11. 12. 13.


RESPONSE 14. 15.
412 IIT-JEE PHYSICS Challenger
16. An ideal gas can be expanded from an initial state to a certain 17. During the melting of a slab of ice at 273 K at atmospheric
volume through two different processes (i) PV2 = constant pressure,
and (ii) P = KV2 where K is a positive constant. Based on (a) positive work is done by the ice-water system on the
the given situation, choose the correct statement(s). atmosphere.
(a) Final temperature in (ii) will be greater than in (i) (b) positive work is done on the ice- water system by the
(b) Final temperature in (ii) will be less than in (i) atmosphere.
(c) Total heat given to the gas in case (ii) is greater than in (i) (c) the internal energy of the ice-water system increases.
(d) Total heat given to the gas in case (ii) is less than in (i) (d) the internal energy of the ice-water system decreases.

MARK YOUR
16. 17.
RESPONSE

1. Match the following :


Column-I Column -II
(A) Cyclic process (p) Q = 0
(B) Isobaric process (q) Q = W
(C) Isochoric process (r) W = nR T
(D) Adiabatic expansion (s) Q= U
2. A gas undergoes a process according to the graph. P is pressure, V is volume, W is work done by the gas, U is change in internal
energy of the gas and Q is heat given to the system.

C B
P

D A
1/V
Column-I Column -II
(A) For process AB (p) U > 0, Q > 0
(B) For process BC (q) U < 0, Q < 0
(C) For process CD (r) Q× U×W=0
(D) For process DA (s) Q× U<0

1. 2.

MARK YOUR
RESPONSE
HEAT AND THERMODYNAMICS 413

3. Column I Column II
(A) Work done by an ideal gas in a closed system (p) Path dependent
(B) Internal energy of an ideal gas in a closed system (q) State function
(C) Molar heat capacity of an ideal gas in a closed system (r) May be constant (other than zero)
(D) Heat absorbed or rejected by an ideal gas in a closed system (s) May be zero.
4. With reference to the following P-V diagram, match the following
P N O

P
M
V
Column I Column II
(A) Process MN (p) W = 0
(B) Process NO (q) W < 0
(C) Process OP (r) U< 0
(D) Process PM (s) U> 0
5. Some processes are mentioned in Column I. Column II lists the resulting effects. Match the entries in Column I with those in
Column II.
Column I Column II
(A) Isothermal expansion (p) Internal energy increases
(B) Adiabatic expansion (q) Temperature decreases
(C) Isobaric heating (r) Volume increases
(D) Isochoric cooling (s) Pressure decreases
6. An ideal gas undergoes two processes A and B. One of these is isothermal and the other is adiabatic.
Column I Column II
P
A

(A) B (p) Heat supplied during curve A is positive

V
P

(B) A (q) Work done by gas in both processes is positive

B
V
P

A
B
(C) (r) Internal energy increases in adiabatic process

V
(s) Temperature of gas in process B is constant

3. 4. 5. 6. p q r s
A p q r s
MARK YOUR B p q r s
RESPONSE C p q r s
414 IIT-JEE PHYSICS Challenger
7. An ideal gas undergoes a cyclic process as shown in P-V graph. Match each process given in column-I with the corresponding
results given in column II.

P(N/m²)

J
30

20 M

10 L
K

10 20
V(m³)

Column I Column II
(A) Process J-K (p) W>0
(B) Process K-L (q) W<0
(C) Process L-M (r) Q>0
(D) Process M-J (s) Q<0
8. When a sample of a gas is taken from state i to state f along the path iaf, heat supplied to the gas is 50 cal and work done by the
gas is 20 cal. If it is taken by path ibf, then heat supplied is 36 cal. Then match the column

a f
Pressure

i b

Volume
Column I Column II
(A) Work done by the gas along path ibf is (p) 6 cal
(B) If work done upon the gas is 13 cal for the return path (q) 18 cal
‘fi’, then heat rejected by the gas along path ‘fi’ is
(C) If internal energy of the gas at state i is 10 cal, then (r) 40cal
internal energy at state ‘f’ is
(D) If internal energy at state ‘b’ is 22 cal and at ‘i’ is 10 cal (s) 43 cal
then heat supplied to the gas along path ‘ib’ is

7. 8.

MARK YOUR
RESPONSE
HEAT AND THERMODYNAMICS 415

9. The figures given below show different processes (relating pressure P and volume V) for a given amount for an ideal gas. W is
work done by the gas and Q is heat absorbed by the gas.

P P

adiabatic

1/V V
fig. (i) fig. (ii)

P P

V V
fig. (iii) fig. (iv)

Column I Column II
(A) In figure (i) (p) Q>0
(B) In figure (ii) (q) W<0
(C) In figure (iii) (r) Q<0
(D) In figure (iv) (for complete cycle) (s) W>0

9.

MARK YOUR
RESPONSE
416 IIT-JEE PHYSICS Challenger

1. From what height (in meter) must a block of ice be dropped 6. A 10 kW drilling machine is used to drill a bore in a small
into a well of water so that 5% of it may melt. Given : both ice aluminium block of mass 8.0 kg. How much is the rise in
and water are at 0°C, L = 80 cal g–1, J = 4.2 J cal–1 and g = 980 temperature (in °C) of the block in 2.5 minutes assuming
cm s–2. 50% of power is used up in heating the machine itself or lost
2. A clock which keeps correct time at 25ºC has a pendulum to the surroundings. Specific heat of aluminium = 0.91 Jg–
made of brass whose coefficient of linear expansion is 1°C–1.
0.000019. How many seconds a day will it gain if the 7. A room is maintained at 20°C by a heater of resistance of 20
temperature fall to 0ºC. ohms connected to 200 volts mains. The temperature is
3. The coefficient of volume expansion of glycerine is uniform throughout the room and the heat is transmitted
49 × 10–5 °C–1. What is the fractional change in the density through a glass window of area 1 m2 and thinkness 0.2 cm.
for a 30°C rise in temperature. Calculate the temperature (in °C) outside. Thermal
4. A brass boiler has a base area of 0.15 m2 and thickness 1.0 conductivity of glass is 0.2 cal/m-sec-°C and mechanical
cm. It boils water at the rate of 6.0 kg/min when placed on a equivalent of heat is 4.2 joules/cal.
gas stove. Estimate the temperature (in °C) of the part of the 8. A lead bullet just melts when stopped by an obstacle.
flame in contact with the boiler. Thermal conductivity of brass Assuming that 25 per cent of the heat is absorbed by the
= 109 Js–1m–1°C–1. (L = 2256 × 103 J kg–1) obstacle, find the velocity of the bullet (in metrer/second) if
5. A brass rod of length 50 cm and diameter 3.0 mm is joined to its initial temperature is 27°C.
a steel rod of the same length and diameter. What is the (Melting point of lead = 327°C, specific heat of lead = 0.03
change in length (in cm) of the combined rod at 250 °C, if the calories /gm/°C, latent heat of fusion of lead = 6 calories /
original lengths are at 40.0 °C ? The ends of the rod are free gm, J = 4.2 joules /calorie).
to expand (Co-efficient of linear expansion of brass = 2.0 × 9. An electric heater is used in a room of total wall area 137 m2
10–5 K–1, steel = 1.2 × 10–5 K–1 ). to maintain a temperature of + 20°C inside it, when the outside

1. 2. 3. 4. 5. 6.

MARK
YOUR 7. 8. 9.
RESPONSE
HEAT AND THERMODYNAMICS 417

temperature is – 10°C. The walls have three different layers 12. An ideal gas is taken through a cyclic thermodynamic
materials. The innermost layer is of wood of thickness process through four steps. The amounts of heat involved
2.5 cm, the middle layer is of cement of thickness 1.0 cm and in these steps are Q1 = 5960 J, Q2 = –5585 J, Q3 = –2980J and
the outermost layer is of brick of thickness 25.0 cm. Find the Q4 = 3645J, respectively. The correspending quantities of
power (in watt) of the electric-heater. Assume that there is work involved are W1= 2200J,W2=–25J, W3 = – 1100 J and W4
no heat loss through the floor and the ceiling. The thermal respectively. What is the efficiency (in percentage) of the
conductivities of wood, cement and brick are 0.125, 1.5 and cycle ?
1.0 watt/m/°C respectively. 13. A 5 m long cylindrical steel wire with radius 2 10-3 m is
10. An ideal gas has a specific heat at constant pressure suspended vertically from a rigid support and carries a bob
of mass 100 kg at the other end. If the bob gets snapped,
5R calculate the change in temperature of (in °C) the wire
CP = . The gas is kept in a closed vessel of volume
2 ignoring radiation losses.
0.0083 m3, at a temperature of 300 K and a pressure of 1.6 × (For the steel wire: Young’s modulus = 2.1 1011 Pa;
106 N/m2. An amount of 2.49 × 104 joules of heat energy is Density = 7860 kg/m3; Specific heat = 420 J/kg-K).
supplied to the gas. Calculate the final pressure (in mega 14. Hot oil is circulated through an insulated container with a
pascal) of the gas. wooden lid at the top whose conductivity
11. An ideal monoatomic gas is confined in a cylinder by a K = 0.149 J/(m-ºC-sec), thickness t = 5 mm, emissivity = 0.6.
spring-loaded piston of cross-section 8.0 × 10–3 m2. Initially Temperature of the top of the lid is maintained at T = 127ºC.
the gas is at 300 K and occupies a volume of 2.4 × 10–3 m3 If the ambient temperature Ta = 27ºC, calculate the rate of
and the spring is in its relaxed (unstretched, uncompressed) heat loss per unit area (in watt/m2) due to radiation from the
state, fig. The gas is heated by a small electric heater until lid.
the piston moves out slowly by 0.1 m. Calculate the final Te = 127°
temperature (in kelvin) of the gas. The force constant of the
spring is 8000 N/m, atmospheric pressure is 1.0 × 105 Nm–2.
The cylinder and the piston are thermally insulated. The
piston is massless and there is no friction between the piston
and the cylinder. Neglect heat loss through lead wires of the T0 Ta = 27ºC
heater. The heat capacity of the heater coil is negligible. Hot Oil

Assume the spring to be massless.

17
(Given = 10 8 )
3
Open atmosphere
15. 0.05 kg steam at 373 K and 0.45 kg of ice at 253K are mixed in
Rigid an insulated vessel. Find the equilibrium temperature (in
Heater support
kelvin) of the mixture. Given, Lfusion = 80 cal/g = 336 J/g,
Lvaporization = 540 cal/g = 2268 J/g, Sice = 2100 J/kg K = 0.5 cal/
gK and Swater = 4200 J/kg K = 1 cal/gK

10. 11. 12. 13. 14. 15.

MARK
YOUR
RESPONSE
418 IIT-JEE PHYSICS Challenger

1 (d) 14 (d) 27 (a) 40 (b) 53 (c) 66 (a)


2 (c) 15 (d) 28 (b) 41 (a) 54 (a) 67 (d)
3 (d) 16 (a) 29 (a) 42 (c). 55 (b) 68 (c)
4 (a) 17 (a) 30 (b) 43 (d) 56 (b) 69 (d)
5 (b) 18 (b) 31 (a) 44 (d) 57 (a) 70 (b)
6 (d) 19 (a) 32 (d) 45 (a) 58 (c) 71 (a)
7 (d) 20 (a) 33 (a) 46 (a) 59 (d) 72 (c)
8 (a) 21 (b) 34 (d) 47 (a) 60 (d) 73 (d)
9 (c) 22 (b) 35 (b) 48 (b). 61 (c) 74 (a)
10 (d) 23 (c) 36 (b) 49 (c) 62 (b)
11 (c). 24 (a) 37 (a) 50 (c) 63 (c)
12 (d) 25 (b) 38 (c) 51 (b) 64 (a)
13 (b) 26 (c) 39 (a) 52 (d). 65 (b)

1 (c) 7 (b) 13 (b) 19 (c) 25 (a) 31 (c)


2 (a) 8 (c) 14 (a) 20 (a) 26 (a) 32 (c)
3 (b) 9 (c) 15 (b) 21 (c) 27 (d)
4 (b) 10 (a) 16 (a) 22 (d) 28 (a)
5 (a) 11 (a) 17 (c) 23 (d) 29 (d)
6 (c) 12 (b) 18 (c) 24 (c) 30 (b)

1 (a) 3 (d ) 5 (c) 7 (a) 9 (d ) 11 (b )


2 (d ) 4 (d ) 6 (c) 8 (a) 10 (d ) 12 (b )

1 (a, b) 4 (a, c) 7 (a, b , c) 10 (b, d). 13 (a, c, d) 16 (a, c)


2 (a, c) 5 (b, c) 8 (a, b) 11 (a, c, d) 14 (a, b, d) 17 (b, c)
3 (a, b) 6 (a, b, c, d) 9 (c, d) 12 (a, c, d) 15 (a, c, d)

1. A-q; B-r; C-s; D-p 2. A-p, r; B-p; C-q, r; D-q 3. A-p, s; B-q, r; C-p, r, s; D-p, s
4. A-p, s; B-s; C-p, r; D-q, r 5. A-r, s; B-q, r, s; C-p, r; D-q, s 6. A-p, q; B-r, s; C-p, q
7. A-s; B-p, r; C-r; D-q, s 8. A-p; B-s; C-r; D-q 9. A-p, s; B-q; C-q, r; D-p, s

1 1714 2 20.5 3 0.014 4 238 5 346

6 103 7 15.2 8 13 9 9000 10 3.6

11 800 12 10.8 13 0.002 14 595 15 273


HEAT AND THERMODYNAMICS 419

1. (d) Rate of transmission of heat


mL 4 K 4 3
= m r
Temperature difference time (t / r 2 ) 3
=
Thermal Resistance
L K
÷ constant
dQ d
= time tr
dt R
( t/4
Here,
dQ 2) 1 t
dt R2 R1
R 2R
2 1
R2 R1
tR
time µ
R1 R1 2 R2 1 R2 K
( R1 R2 ) R2 1 R1 2 t
2 RK s KL
25 4 1 KS 8
R2 1 R1 2 = = ; =
16 tRK L 2 KL K S 25
R1 R2
2. (c) Let Q1 and Q2 amounts of heat flow from P in any time 4. (a) Let 0, 1 and 2 be the densities of the liquid at
t. Let m be the masses of steam formed and ice melted temperature 0, t1 and t2 respectively
in time t. Let k and A be the thermal conductivity and To balance pressure, 1 1g = 2 2g
the area of cross-section respectively of the rod.
0 0
t1 ÷ t2 ÷
800 100 or 1 = 2
Q1 = kA ÷ t = mLsteam 1 1
x
Solving we get
800 0
Q2 = kA ÷ t = mLice 1- 2
1 x r=
2t1 - 1t2
700 1 x
Dividing, ÷ =7 dQ
x 800
5. (b) For -t plot, rate of cooling = = slope of the curve.
dt
1
or 1 – x = 8x or x = m
9 dQ
AT P, = |tan(180°– 2)| = tan 2 = k( 2 – 1)
dt
r2
dr 1 r2 r1
3. (d) R = dR = 2
= where k = constant.
r1 4 r K 4 K r2 r1 At Q,
[R = thermal resistance] dQ
=| tan(180° 1) | tan 1 k( 1 0)
dt
4 K 4 K
Q= =
r2 r1 t tan

( r1 = r2) 2
= 2 0
r1r2 ÷ r tan 1 1 0

6. (d) From Boyle's law (T = constant)


r1 P1 V1 = P2 V2
r2
420 IIT-JEE PHYSICS Challenger

4 3 pA
(Hdwaer + h dmercury) g ( r ) for A, p A p or p A 2 p
3 2

4 pB
= hdmercury g( (2r)3) for B, pB 1.5 p
3 2
Hdwater = 8hdmercury – hdmercury or pB 3 p
d mercury pA 2
H = 7h =
d water pB 3
H = 7h
mA m
3 Also, p AV = RT and pBV = B RT
M M
7. (d) Total energy of molecules of first gas = n1kT1
2
pA mA
=
3 pB mB
Total energy of molecules of second gas = n2kT2
2
mA 2
3 =
Total energy of molecules of mixture = K(n1T1 + mB 3
2
n 2 T2 ) or 3mA = 2mB.

3 3 10. (d) Here TV –1 = constant


(n1 + n2) KT = K(n1 T1 + n2T2)
2 2
5
n T + n2T2 As = , hence TV2/3 = constant
T= 1 1 3
( n1 + n2 )
Now T1 L12 / 3 = T2 L22 / 3 ( V µ L);
8. (a) The average number of degrees of freedom per
molecule, 2/3
T L
total number of degree of freedom Hence, 1 = 2 ÷
f = T2 L1
total number of molecules
11. (c) V = V0 ( 1 + ) where is the difference in
n1 N A f1 + n2 N A f 2 temperature, is the coefficient of volume expansion.
= n1 N A + n2 N A V V0 1
i.e., =
where NA = Avogadro constant. V0
Here, f1 = 3, f2 = 5
m m
n1 f1 + n2 f 2 3n1 + 5n2
f = = d 2 d1 1 (d1 d 2 ) d1
n1 + n1 n1 + n2 = .
m/d1 (d1d 2 )
2
Also, = 1+ = 1.5 or f = 4 (d1 d 2 ) 1
f = .
d2
3n1 + 5n2
=4 n1 = n2
1000 958.4
n1 + n2 = 4.5 10 4
per degree K.
958.4 96
9. (c) Let pA, pB be the initial pressures in A and B
12. (d) Let T0 = initial temperature of the black body.
respectively. When the gases double their volumes at
0T0 = b (constant)
pA Power radiated = P0 = C.T04.
constant temperature, their pressures fall to and
2 (C = constant)
pB Let T = new temperature of black body.
2 3 4T0
0
T =b= 0T0 or T =
4 3
HEAT AND THERMODYNAMICS 421

Power radiated
2 2+ 3
T = ÷T
2+ 3 ÷
4
4 256
= c.T 4 = (cT04 ) = P0 ÷
3 81
17. (a) Net power given to N2 gas = 100 – 30 = 70 cal/sec
13. (b) We know that The nitrogen gas expands isobarically.
PAVA = nRT, PBVB = nBRT
dT 7 dT
and Pf (VA + VB) = (nA + nB) RT Q = nCP or 70 5 R
dt 2 dt
Pf (VA + VB) = PAVA + PBVB
dT
PAV A + PBVB = 2K / sec
dt
V A + VB ÷
Pf =

AT BT 2
1.4 0.1 0.7 0.15 18. (b) p=
= MPa = 0.98MPa V
0.1 + 0.15
U AB + W AB AT BT 2
14. (d) Q AB or V=
p
W AB = 0
AdT B 2TdT
f f dV = , since p is constant
U AB nR T ( PV ) P
2 2 or pdv = (A – BT) dT
5
U AB ( PV ) QAB = 2.5 P0V0 T2 T2
2 W = pdV = ( A 2 BT )dT
Process BC : T1 T1
QBC U BC + WBC = 0 + 2 P0V0 ln 2 = 1.4 P0V0
A(T2 T1 ) B (T22 T12 )
Qnet = Q AB + QBC = 3.9 P0V0
19. (a) Along the path i a f
15. (d) Heat absorbed by gas in three processes is given by
50 = U + 20
Q ACB U + W ACB U = 30 cal.
QADB U The value of U remain same for the path i b f,
So, 36 = 30 + W
Q AEB U + W AEB
W = 6 cal
The change in internal energy in all the three cases is For the path f i, U = – 30 cal
same and WACB is +ve, WAEB is –ve.
Q = – 3 – 13 = – 43 cal
Hence QACB > QADB > QAEB
As U = Uf – Ui = 30
Uf = 30 + Ui = 30 + 10 = 40 cal.
16. (a) T
20. (a) for He = 5/3; for O2 = 7/5
60° 1
/2 8 g of He is equal to 2 moles of He and 16 g of O 2 =
2
30° mole of O2 gas. Total one has 2½ moles.
T'
2T 3 The weighted average is
2 5 7 1 2 10 7 2
= 2
3 5 2 5 3 10 5
Sum of heat current at C is zero.
121 2 24.2
(T T ') A ( 2T T ') A
+ =0 30 5 15
( 3 / 2)
21. (b) The efficiency of heat engine,

( 2T T ) Work done T2 W T2
(T T ) + 2 =0 1 , or 1
3 Heat taken T1 Q T1
422 IIT-JEE PHYSICS Challenger
Given, T1 = 227°C = 227 + 273 = 500 K
RT
T2 = 127°C = 127 + 273 = 400 K and PB PB PB mB
2 MV
Q = 6.0 × 104 cal
But PA = P and – PB = 1.5 P
W 400
1 PA 1 PB
6.0 104 500 So, PA
PB 1.5 1.5
500 400
or W 6.0 104 RT mB RT
500 mA
2 MV 1.5 2 MV
100
= 6.0 104 = 1.2 × 104 J 10 m B
500 or mA 3mA = 2mB
15
22. (b) From the relation 24. (a) From the first law of thermodynamics
n1 n2 n1 n2 dQ = dU + dW
1 1 1 Here dW = 0 (given)
1 2
dQ = dU
5 Now since dQ < 0 (given)
For monoatomic gas, 1
3 dQ is negative
dU = – ve
7
For diatomic gas, 2 dU decreases.
5 Temperature decreases.
n1 n2 n1 n2 nRT 2 1
25. (b) Process (A). V = K ; TV = constant ; T µ
1 5 7 V V
1 1
3 5
nRT
Process (B) = KV 2 ;
n1 n2 3 5 V
or n1 n2
1 2 2 T
3
= constant ; T µ V 3
V
n1 n2 3n1 5n2
or Q W ka
1 2 26. (c) = = work done per unit time=
t t L
2n1 2n2 dW dv a nRT
1 =P =k
3n1 5n2 , P=
dt dt L V
2n1 2n2 5n1 7 n2 .5 R(300) d ka
or 1 A. =
3n1 5n2 3n1 5n2 V dt L
23. (c) The initial pressure in the two containers will be .5 R (300) ka
A.v =
n A RT L L
RT A.
PA = mA 2
V MV
ka 27 k
nB RT RT v= ÷=
PB = mB R 300 100 R
V MV
27 (a) Let l1 and l2 be the final lengths of the two parts, then
After isothermal expansion, pressure will be
from gas equation :
n A RT RT
P 'A = mA
2V 2 MV
400 K 500 K
nB RT RT Vaccum
P 'B = mB
2V 2 MV
1 2
RT
– PA = PA – PA mA
2 MV
HEAT AND THERMODYNAMICS 423

P0 Al0 PAl1 PAl2 29. (a) We have from the gas law
= = ...(i)
T0 T1 T2 (P0 V 2 ) V = nRT
Considering the equilibrium of piston in initial and final
states, we have (P0 V 2 )V
T= = f (V )
P0 A = kx0 and PA = kx nR
From the maximum temperature
P x
or = ...(ii)
P0 x0 dT
=0
dV
Decrease in length of spring = total increase in the
lengths of the two chambers.
or P0 3 V 2 = 0
x x0 = l1 + l2 2l0 ...(iii)
1/ 2
From eq. (i), P0
V=
P0l0T1 Pl T 3 ÷
l1 = and l2 = 0 0 2
PT0 PT0 Therefore, corresponding pressure
Using eq. (ii), P0 2 P0
P P0 =
x0 l0T1 xlT 3 3
l1 = and l2 = 0 0 2
xT0 xT0 Hence, the sought maximum temperature from the gas
law becomes
Putting these in eq. (iii)
x0 l0 1/ 2
P0 P0
x x0 = [T1 + T2 ] 2l0
3 3 ÷
2 1/ 2
xT0 PV 2 P0 P0
T= = =
Substituting the values and solving for x, we get nR nR 3nR 3 ÷
x 1.3 m. 30. (b) The temperature gradient is given by
28 (b) If the temperature of surrounding increases by T , the dT dT d dP
new length of rod becomes = . . ...(i)
dh d dP dh
l = l (1 + T)
From, dP gdh
Due to change in length, moment of inertia of rod also
changes and it is about an end P and is given as dP
g ...(ii)
2 dh
Ml
lP =
3
Given that p / = c (constant)
n
As no e xternal force or torque is acting on rod thus its
angular momentumremainsconstantandduringheating, n
P c
thus we have
IP IP ' dP
cn n 1 ...(iii)
d
[If is the final angular velocity of rod after heating].
We know that
M l2 M l 2 (1 T )2
or R
3 3 P T
M
or (1 2 T)
n R
[using binomial expansion for small ]. or c T
M
Thus percentage change in angular velocity of rod due
to heating can be given as M n 1
T c
R
100%
dT M
2 T 100% or c (n 1) n 2 ...(iv)
d R
424 IIT-JEE PHYSICS Challenger
From eqs. (ii), (iii) and (iv) substituting these values in 33. (a) Slope of temperature gradient in a material is ratio of
eq. (i) and solving, we get : temp. gradients in two material will be constant.
dT Mg ( n 1) Ratio for Z and X
dh nR 100 – 20
31. (a) In isothermal process, the heat transferred to the gas is 2 = 2, n = 2
given by 20 – 0
Q1 = nRT0 log e (V2 / V1 ) = nRT0 log e ...(i) The dotted line represents the new temperature gradi-
ent.
[ = (V2 / V1 ) = ( P1 / P2 ) ]
In isochoric process, A D
Q2 U [ W =0] C
nk1
Nk2
B
nR
Q2 nCv T T ...(ii)
1 k2
20°C
P T P k1
Now, 2 = 0 or , T = T0 1 ÷ = T0
P1 T P2 100°C

T T0 T0 ( 1) T0 ..(iii) k1 + nk12 = (100 –0)


Substituting the value of T from eq. (iii) in eq. (ii), we 100 100 20
k1 = = =
get (1 + 2n) 5
R Similarly,
Q2 n ÷( 1) T0
1
100 25
k2 = =
R (2 + n)
Q n RT0 log e n ÷( 1) T0
1 New temp. of B = k2 = 25°C
34. (d) Equivalent circuit is
Q 1
or log e =
nRT0 1 2RT V RT

( 1)
or 1=
Q
log e
n RT0
P (90°C) RT R (30°C)
( 1)
= 1+
Q
log e 90 – V V 30
n RT0 = V = 50°C
2 RT RT
Substituting the given values, we get
TE + TD
(5 1) 35. (b) TB =
= 1+ 2
80 103 2TB = TE + TD ...(i)
log e 5
3 8.3 273
TE + TC
Solving we get = 1.4 TD =
2
32. (d) The process is equivalent to TV 1/2 = C TE = 2TD – TC ...(ii)
Compare with TV –1 = C x = 3/2
2TB = 2TD – TC + TD
R R R R TC = 3TD – 2TB
C= + = +
– 1 1 – X 2 / 3 1 – (3 / 2)
36. (b) Q = Q AT 4 , Here A = r2
3 1 According to Wien’s law
= R – 2R = – R
2 2 =b
mT
HEAT AND THERMODYNAMICS 425

or, T = (b/ m) dV V
2 4 ÷ =2
Q ( r )(b / m) dT proc T

r2 dQ dV
or, Qµ 4
= CV + 2. p ÷ = CV + 2R.
m
dT proc dT proc

QB > QC > QA 43. (d) The heat capacity of the solid is


37. (a) Q = U + W. dQ
Given that Q = – U, 2a 4b 3 .
d
–1 44. (d) Suppose that the temperature of the water in the first
÷
After solving, TV 2
=C vessel is 1 (t) and that of the second is 2 (t), then
R R
38. (c) C= + for a polytropic process ms
d 1 KA
(
–1 1– n 1 2) ... (i)
dt L

5R R 7 d 2 KA
ÞC = + ( r= for a diatomic gas) and ms ( 2) ... (ii)
2 1- n 5 dt L
1

From (i) and (ii), we get


9
For C to be negative, the possible value of n will be .
7 d 2 KA
,
39. (a) The work done during the cycle = area enclosed in the dt msL
curve. where 1 2.
The time, in which the temperature difference reduces
40. (b) P
1
B to of its initial value, is given by
PB C e

msL
t= .
2 KA
PA
A
45. (a) + + = + +
K1 A 2K2 A K3 A KA 2K A KA
VA VC V
5 1 1 1
= + +
K1 2 K 2 K 3 ÷
WAB = 0
2K
QAB = VAB + WAB
600 = UAB
5
QBC = UBC + WBC Keq =
1 1 1
200 = UBC + 8 × 105 × 3 × 10–4 2 + +
K1 2 K 2 K 3 ÷
UBC = – 40 J
UCA = – 560 J
dQ dT dQ
41. (a) VT = constant 46. (a) = ms ; = constant
dt dt dt
PV = nRT
1 1
nRT sµ µ
V= dT / dt Slope of temp.-time graph
P
Slope of graph is lesser in the solid state hence higher
nRT heat capacity. The transition from solid to liquid state
.T = constant
P takes lesser time, hence latent heat is smaller.
P µ T2. 47. (a) For ideal gas Cp – Cv = R
42. (c) The equation of the process may be written as : If Cp – Cv = 1.06 R,
TV1/2 = constant, from which, we get, then gas will be real gas. Thus pressure is high and
temperature is low for real gas.
426 IIT-JEE PHYSICS Challenger
48. (b) Work done in expansion is positive and in compression 57. (a) VL + VHe = Va
it is negative. Va = volume of displaced air
49. (c) We know that V/T = constant If the bubble floats mHeg + mLg = mag
V V V
=
T T T
or VT + T V = VT + V T
or T V=V T
V 1 VHe VL
or =
V T T

dP
50. (c) B
dV / V
Let PV x = constant B = xP
(A) isothermal, x = 1 B = P He VHe LVL aVa
(B) adiabatic, x = B= P Now, He a
(C) isobaric, x = 0 B = 0
VHe < Va
(D) isochoric, x = B=
51. (b) U = 2 + 3 nRT mHe is much smaller than ma and hence we can
also say that
dU = n3RdT = nCVdT Cv = 3R
ma
Cp = 3R + R ; m He < mL > mHe
2
Cp 3 +1 4 58. (c) During processes 1 2 and 3 4, pressure is directly
= =
Cv 3 3 proportional to T which is possible only when volume
of the gas is constant. Hence, process 1 2 and
52. (d) The volume has the same value at more than one process3 4 are isochoric process.
temperature, hence it cannot be used for thermometer.
W12 = 0, W34 = 0, W23 = nR (T3 – T2)
53. (c) For an adiabatic change in case of a monoatomic gas,
= 3R (1600 – 400) = 3600 R
TV –1 = constant. In this case x itself is ( – 1) and =
5/3 giving the value of x. W41 = nR (T1 – T4) = 3R (200 – 800) = –1800 R
W = (3600 – 1800) R = 1800 R = 15 kJ
54. (a) PAV A = PBV B ; PBVB = PCVC
3RT
59. (d) vrms =
PC VC = PDVD ; PDVD = PAV A M
Solving above eqs. T
Same Vrms, implies same.
VB V A M
=
VC VD Hence, T µ M
M for oxygen is more than that of hydrogen, therefore,
dm oxygen is at higher temperature than hydrogen.
55. (b) sw =P
dt 60. (d) Work done by gas in going isothermally from state A to
20 × 4.2 × (T – 10) = 2000 T = 33.8°C B is

3P 3PV m (vrms )2 W AB = nRT ln


PA
= nRT ln 2
56. (b) vrms = = P= ... (1)
m 3V PB

If P be the final pressure then Work done by gas in going isothermally from state B
to C is
m
(2vrms )2 WBC = nRT ln
PB P
= nRT 0
2 ... (2)
P' PC 2 PC
= 3V =2 P ' = 2 P.
P m(vrms ) 2 It is given that WBC = 2 WAB
3V P0
ln = ln(2)2
2 PC
HEAT AND THERMODYNAMICS 427

P0 1
PC = That is, Q = m÷ L + mc (40°).
8 4
61. (c) Q – W = E
3 1
Change in internal energy E is a state function which Equating the energy, mL = m÷ L + mc (40°)
does not depend on the path. 4 4
62. (b) For adiabatic process PV = constant
1 L
L = c (40°) = 80°
Cp 2 c
Also for mono atomic gas = = 1.67
CV 67. (d) Mean translational K.E. of any molecule whether
monoatomic, diatomic or polyatomic, at a given
Also for diatomic gas = 1.4
3
temperature T is kT .
2
68. (c) V vs T graph is a straight line for isobaric process,
P volume cannot be –ve, hence x represent volume and y
1 temperature.
2 y is –ve when x = 0 i.e. absolute zero hence the scale is
not kelvin.
69. (d) Rate of heat loss with only room heater
V
Q
Since, diatomic < mono atomic Ph = = C (16 + 20) , C = constant
t
Pdiatomic > Pmono atomic
while for both heater and stove it is
Graph 1 is for diatomic and graph 2 is for mono
atomic.
Q
63. (c) As temperature increases, velocity of molecules Ph + Ps = ÷ = C (22 + 20)
t
increases and hence the pressure.

f Ph 36
64. (a) Total Kinetic Energy = U = nRT 7 Ph = 6 Ph + 6 Ps
2 Ph + Ps 42
In case of H2 degree of freedom is greatest and number Ph = 6Ps = 6kW
of moles n is highest.
So this is the case of maximum kinetic energy. 70. (b) A(TH4 T4) A(T 4 TL4 )
65. (b) By first law,
Q= U+ W TH4 + TL4
T4 =
2
f
= nRT + P dV
2 A (TH4 TL4 )
= =2
f A (TH4 T 4)
= nRT + PV
2 71. (a) PV3/2 = constant
f 3 5 PV = nRT
Q= nRT + nRT = nRT + nRT = nRT
2 2 2 1/ 2
V
V1/2 T = const. ÷ T = const.
(3 / 2) nRT 3 2
Fraction = (5 / 2) nRT = 5
T =T 2
3 72. (c) Corresponding to 1°C change in temperature, the
66. (a) The first energy results in Q = m÷ L .
4
9
change is in Fahrenheit temperature. Therefore, if
The contribution of energy melts the remaining material 5
and then increases the temperature. specific heat is defined in terms of °F, then it will
decrease.
428 IIT-JEE PHYSICS Challenger
73. (d) At high temperature for diatomic gas f = 7
nR
U= dU = dT
fR 9R 1
Cp = +R=
2 2
nR PV
74. (a) U = a + bPV ......(1) or U = T +a= + a ......(2)
In adiabatic change, 1÷ 1

where a is the constant of integration.


nR nR
dU = – dW = (T2 T1 ) = (d T ) Comparing (1) and (2), we get
1 1
1 b +1
b= = .
1 b

For (Qs. 1-3) 1. (c) 2. (a) 3. (b)


When the temperature of rods is increased, there will For (Qs. 4-6)
be increase in their lengths and thereby the springs are
TV –1 = constant
compressed. Let x1 , x2 and x3 be the compresion in
the three springs respectively. Then –1 = 3T0 (3V0) –1
T0 (V )
L T ( L / 2) T = x1 + x2 + x3
V = 3V0 (3)1/ –1 = 9 3 litres
3
or L T = x1 + x2 + x3 ...(i) P0V0 = PV
2 (1) (P0, V0, T0)
The free body diagrams of the rods are shown in figure.
L L/2 (2) (P0, 3V0, 3T0)
kx1 2kx2 2kx2 3kx3
(3) (P, V, T0)

Considering the equilibriums of rods, we have


x1 = 2 x2 = 3x3 ...(ii)
From eqs. (i) and (ii), we get
P0V0 P0 nRT0
x1 +
x1 x1 3
+ = L T P = = =
2 3 2 9 3V0 9 3 9 3V0

or 11 3 0.3 8.314 300 8.314


x1 L T = 104 N/m2
4 2 9 3 10 –3
3
9
or x1
L T nR (T2 – T3 )
11 Work done = nR (T2 – T1) +
Energy stored in spring of spring constant (k). –1

2 2nRT0
1 2 1 9
E1 = k x1 = k L T = = 3739.5 Joule
2 2 11 –1

81 4. (b) 5. (a) 6. (c)


k L2 2
T2 7. (b) n1Cv (T – T0) + n2 Cv (T – 2T0) = 0
242
3
Similarly, E2
81
k L2 2
T2 T = T0
484 2

27 PT 3
and E3 k L2 2
T2 Pf =
i f
= P0
242 Ti 2
HEAT AND THERMODYNAMICS 429

8. (c) Q = n1Cv (T f – T0 ) 14. (a) Heat absorbed during BC is given by


Q = nCv T
P0V0 3 3 3 3R
R T0 – T0 ÷ = P0V0 =n (TC TB )
2 RT0 2 2 8 2
9. (c) Let V is change in volume in any compartment then 3R
=n (2T0 )
V0 2
Pf – V÷ = 3nRT0.
PV 2
n1 = 0 0 = and 15. (b) Heat capacity is given by
2 RT0 RT f
1 dQ
C=
V0 n dT
Pf V÷
2P V 2
n2 = 0 0 = V =0 1 Q
2 RT0 RT f C=
n 2T0
10. (a) Process AB, U = constant For (Qs. 16-18)
1 v1 A 1 = 2 v2 A 2
P RT
= and U µ t m = 1500 kg/m3 × 0.1 m/s × 4 (cm)2
M
ms T = 10000
P = const 1500 × 0.1 × 4 × 10–4 × 1500 × T = 10000
Process BC isochoric
10000 1000
Process CA isothermal T= = °C
90 9
11. (a) Q = QAB + QBC + QCA
1500
10U 0 2 = 1
= = 1350 kg/m3
Q = – 5U0 + 3U0 + ln 2.5 (1 T) 3 1000
3 1 1 10 ÷
9
12. (b) WAB = QAB – UAB = – 5U0 – (–3U0) = – 2U0
13. (b) AB is an isothermal process then 2 v 2 A2 1v1 A1
P × 2V = PB × 6V 1350 × v2 = 1500 × 0.1
P v2 = 1/9 m/s
PB = Volume rate of flow at the end of tube
3
1
P = A2v2 = 4 × 10–4
9
A C 4 40
P = 10 4 m3 = 10 5 m3
9 9
Volume rate of flow at the entrance = A1v1
= 0.1 × 4 × 10–4 = 4 × 10–5m3
Hence, difference of volume rate of flow at the two
B ends

V 40 5 4
2V 6V = 4÷ 10 = 10 5 m3
9 9
Now BC is an isochoric process then 16. (a) 17. (c) 18. (c)
PB PC For (Qs. 19-21)
=
TB TC 4
We have P = CT 3/ 2 ......(i)
5
P P
= Also, PV = nRT ......(ii)
3T0 TC From (i) and (ii)
TC = 3T0. PV 3 = constant
430 IIT-JEE PHYSICS Challenger
Thus molar specific heat of gas is 2/3
V0
T = T0 = 4T0
V0 / 8 ÷
R R 5R
C = CV + = CV = 2R as CV =
1 3 2 2
Heat supplied to gas in temperature increment by T = dP
300 K in this process is 26. (a) Adiabatic bulk modulus B = V P
dV
Q = nC T = n (2R) (300) = 600R
[ PV = const.]
Change in internal energy of gas in this process is
d d
5R P (V ). dV + ( P )dP.V = 0
U = nCV T = 300 = 750R dV dP
2
Thus work done by the gas is dP nRT
V = P =
W = Q – U = – 150 R dV V
19. (c) 20. (a) 21. (c) Bi T0 V T0 V0 / 8 1
=
40 36 40 + 36 Bf V0 T V0 4T0 32
22. (d) k( 30) ......(i)
10 2 27. (d) For adiabatic process dQ = 0
Let the body cools to x°C in further 10 min dU + dW = 0
36 x 36 + x 28. (a) W = Area under the curve
k 30÷
10 2 1
= (V2 V1 ) ( P2 P1 ) + (V2 V1 ) ( P2 P1 )
2
4 8
= ; from (i)
36 x x 3
30 + 18÷ = (V2 V1 ) ( P2 P1 )
2 2

x 5x 3 3
12 72 2 x ; = 72 + 12 = (2V1 V1 ) (2 P1 P1 ) = P1V1
2 2 2 2
5x 2 84 168
= 84 ; x = = = 33.6 P2
2 5 5

d kA P1
23. (d) (T T0 )
dt ms
( V2 = 2V1 , P2 = 2 P1 )
Magnitude of slope will decrease with time.
V1 V2
40 36 40 + 36 4 1
24. (c) k( 30) k= = and P1V1 = nRT1
10 2 10 8 20
When the block is at 38°C and room temperature is at W (3/ 2) PV 3
30°C the rate of heat loss
Therefore, = 1 1
= = 1.5
nRT1 PV
1 1 2
d
ms ms k (38 30) 29. (d) Q = dU + dW
dt
dU = nCvdT
Total heat loss in 10 minutes
For final state P2V2 = 2P1 2V1 = 4P1V1 = nR (4T1)
dQ = ms k (38 – 30) × 10
Hence final temperature is 4T1
1 3 9
=2×
20
× 8 × 10 = 8 J dU = n R3T1 = nRT1
2 2
Now heat gained by the object in the said 10 minutes, 3 9 Q
Q = nRT1 + nRT1 = 6 nRT1 ; =6
Q = ms =2×4=8J 2 2 nRT1
Total heat required = 8 + 8 = 16 J 30. (b) nC T = Q nC T = 6nRT1
25. (a) Let initial volume be V0. Since the process is adiabatic, dT = 4T1 – T1 = 3T1
the first temperature and volume is TV 1
= T0V0 1
C
( = 5/3 for monoatomic gas) n.C.3T1 = 6nRT1 ; =2
R
HEAT AND THERMODYNAMICS 431

For (Qs. 31-32) Heat lost by the ball is


Let c be the specific heat of turpentine Mc2 (T2 – T) = 100 (0.092) (100 – 23)
Mass of the solid, M = 100g = 708.4 cal.
Mass of turpentine m = 200g According to the principle of calorimetry
Water equivalent of calorimeter, W = 4g Heat gained = Heat lost
Initial temperature of calorimeter, T1 = 15°C (200c + 4) 8 = 708.4
Temperature of ball, T2 = 100°C 1600c + 32 = 708.4
Final temperature of the liquid, T = 23°C 708.4 32
or c = = 0.42 cal/g°C
Specific heat of solid, c2 = 0.092 cal/g°C 1600
Heat gained by turpentine and calorimeter is 31. (c) 32. (c)
mc (T – T1) + W (T – T1) = 200c (23 – 15) + 4 (23 – 15)
= (200c + 4) 8

1. (a) For equivalent thermal conductivity, the relation is 3. (d) Melting of solid causes change in its internal energy.
1 1 1 Latent heat is the heat required to melt one unit mass
= +
K R K1 K 2 ; If K1 = K 2 = K of solid.
4. (d) Translational degree of freedom is three.
1 1 1 2 K
= + = KR =
KR K K K 2 3
Translational K.E. = PV = 1.5 PV
Which is less than K. 2
If K1 > K 2 , suppose K1 = K 2 + x 5. (c) According to Wein’s displacement law,

1 K2 + K2 + x K 22 + K 2 x mT = constant
= K= Naturally, when T increases decreases. is peak
K (K 2 + x)K 2 2K 2 + x m m
emission wavelength
K 22 + K 2 x and E = T4
Now, K 2 K = K 2
2K 2 + x Here E is energy being radiated per unit area per unit
K2 > K 1 time.
P 6. (c) In free expansion pressure outside is zero. So no work
2. (d) Isothermal curves have slope which is equal to . It
V is done by ideal gas but in real gas work may be done
can be calculated as follows against internal forces existing between molecules.
PV = RT
Cp
Differentiating, 7. (a) If f be the degree of freedom, then the ratio C ( )
PdV + VdP = 0 v

dP P 2
= is given by = 1+ .
dV V f
Now, if they cut each other at certain point, they will
have different slope at the same point (for same value For monoatomic gas, f = 3;
of P & V). So, they can not cut each other at some 2 5
point. = 1+ = = 1.67
3 3
P For diatomic gas , f = 5
Reason is true, slope is .
V
2 7
= 1+ = = 1.4.
P 5 5
For adiabatic curve slope is times .
V < monoatomic
diatomic
432 IIT-JEE PHYSICS Challenger
8. (a) Carnot cycle represents process of an ideal heat engine 11. (b) A body cools, its entropy decreases as
which has maximum efficiency of conversion of heat dQ
energy into mechanical energy. So A is right. The dS = and dQ is –ve, dS is also –ve.
T
efficiency of a carnot cycle depends only on the
Statement -2 is also true. Second law states that entropy
temperature of heat reservoirs is source and sink. So of the universe increases. Universe includes both
reason is right and reason explains assertion. system and surroundings. Statement -2 does not explain
9. (d) As temperature increases coefficient of viscosity of Statement -1 .
gases increases.
n.3R
10. (d) Initial condition 12. (b) For Helium, Q = THe
2
Volume = 76cm × A, where A is the area of cross-section
Pressure = (76 + 76) dg = 152dg n.5 R
For Nitrogen, Q = TN 2
Final condition 2
Volume = 152 cm × A THe TN 2
Pressure = 76dg
P1 V1 = P2 V2
Both points lie on the same isothermal line
i.e. both have same T.

1. (a, b)
P 5
From the first law of thermodynamics = , A = 6l2
P0 6
dQ = dU + dW
As volume is constant. Therefore, dW = 0,
3KA 4 Kl 2 4 8.31 10 3 (0.1) 2
dQ dT = =
or Cv ÷ ...(i)
3Rd Rd (8.31) (4 10 3 )
dt dt
(for n = 1) = 10 2 s 1
Further from the equation of heat conduction Substituting the values in eq. (iv)
dQ KA (T Ts )
÷= ...(ii) 5 1 10 2 t
dt d = ÷ [300 + (100)
e
]
6 400
From eqs. (i) and (ii)
Solving this equation, we get
dT KA(Ts T ) 2 KA (Ts T )
= = t 110s
dt d Cv 3R d
t
(b) P µ T (V = constant)
T dT 2 KA
= dt
or T0 Ts T 3R d 0 Temperature will also remain 5 / 6th of its initial
value.
T Ts (Ts T0 ) e 2 KAt / 3 Rd ...(iii)
5 5
As the volume remains constant T = T0 = ÷ (400) = 333.33K
6 6
P P0
= 2. (a, c)
T T0
According to first law of thermodynamics,
P d Q = dU + PdV (for one mole)
or P = 0 ÷T
T0 Now molar specific heat is given by
dQ dU + PdV Cv dT + ( RT / V )dV
P0 2 KAt / 3 Rd
C= = =
or P ÷ [Ts (Ts T0 ) e ] ...(iv) dT dT dT
T0
RT dV
Given, = Cv + …(i)
V dT
HEAT AND THERMODYNAMICS 433

Given, T = T0 e V 4. (a, c)
Now work is done in the processes AB and CD
or, dT v
T0 e dV
dV 1 WDA = RT0 In 2
= …(ii)
dT T0 e V V0
Substituting the value of (dV/dT) from eq. (ii) in eq. (i), WBC = 2 RT0 In = –2 RT0 In 2
2V0
we have
Total work done by the gas in the cycle = W
RT 1
C = Cv + = – RT0 In 2
V T0 e V As U = 0, Q = W = – RT0 In 2
i.e., heat is given out.
V
RT0 e R 5. (b, c)
C = Cv + = Cv + …(iii)
V V
VT0 e Using the 1st law, we get,

(c) Given that P = P0 e V 5


Q=0= nR (T Ta ) P0V0
2
RT P0
= P0 e V
or T= Ve V
V R P0V0 ÷ 2
= T
or, T = T0 = 5 ÷ 5 0
Now C = Cv +
RT dV nR ÷
2
V dT
T = 1.4 T0
R
= Cv + ( P0 e V
) 1.4 P0V0
P0 e V (1 V) PV = nRT = 1.4 nRT0 or P =
V
R 6. (a, b, c, d)
= Cv + …(iv)
(1 V) (a) For all thermal processes.
3. (a, b) U nCv T where T (T2 T1 )
Energy emitted per second by body A = A
TA4 A where (b) According to first law of thermodynamics.
A is the surface area Q U W
4
Energy emitted per second by body B = B TB A In an adiabatic process Q = 0.
Given that power radiated are equal
or, 0 U W
A
T A4 A = B TB4 A
or, U W
T4 4
A A = B TB (c) In the isothermal process, T = 0.
1/ 4 U=0
A
TB = ÷ TA = 1934 K (d) In the adiabatic process, Q = 0.
B
7. (a, b , c)
According to Wein's displacement law
(a) No. of molecules in 1 mole = 6.023 × 1023
1
m µ PV
T (b) No. of moles are same n =
RT
Since temperature of A is more therefore ( m)A is less (c) Translational KE of N-moles
( m)B – ( m)A = 1 × 10–6 m (given) ... (i)
Also according to Wein's displacement law 3 3
= NRT RT / mole
( m)A TA– ( m)B TB 2 2
(d) Unit mass contains different no. of moles.
( m )A T 5802
= B = ... (ii) different no. of molecules and hence different KE.
( m )B TA 1934
8. (a, b)
On solving (i) and (ii), we get (a) In an adiabatic expansion, internal energy decreases
–6
B = 1.5 × 10 m. and hence temperature decreases.
434 IIT-JEE PHYSICS Challenger
From equation of state of ideal gas PV = nRT, 12. (a, c, d)
the product of P and V decreases. Since sun rays fall on the black body, it will absorb
more radiation and since, its temperature is constant it
3kT
(b) vrms = will emit more radiation. The temperature will remain
m same only when energy emitted is equal to energy
(c) The temperature in kelvin scale is not doubled. absorbed.
9. (c, d) 13. (a, c, d)
PV = nRT V µT (at const. P) At NTP same volume means same molecules
TB > TA f1 = f2, so same temperature means same internal
WAB = P0 (2V0 – V0) = P0V0 f
WBC = RT ln (3/2) = 2 P0V0 [ln 3 – ln 2] energy U = nRT ÷ . Average velocity is zero.
2
= 2 P0V0 (2.303) (0.477 – 0.30)
= 0.8142 P0V0 < WAB 14. (a, b, d)
10. (b, d) The work done by the gas in the process A to B exceeds
Brass the work that would be done by it if the system were
taken from A to B along the isothermal line. This is
Copper because the work done is the area under the P-V
d
indicator diagram. As shown by the diagram the area
under the graph in first diagram will be more than in
second diagram. When we extrapolate the graph shown
R in figure (i). Let P0 be the intercept on P-axis and V0 be
the intercept on V-axis. The equation of the line AB can
be written as
P0
Co-efficient of linear expansion of brass is greater than P= V + P0 ... (i)
V0
that of copper i.e., B C.
[ y = mx + c ]
LB = L0 (1 + B T)
To find a relationship between P and T, we use
or, ( R d ) = L0 (1 + B T) RT
PV = RT V= ... (ii)
Again, LC = L0 (1 + C T ) P

R = L0 (1 + T)
P
or, C P
Po
isothermal
(R d ) 1 T P A process
= B
P
R 1 C T
p B p
R+d
or, = (1 + B T ) (1 C T ) , by binomial 2 2
R
V 2V Vo V V 2V V
theorem.
Fig (i) Fig (ii)
d
or, 1+ = 1+ ( B C ) T Smaller terms From (i) and (ii)
R
P0 RT
d d + P0
or, ( C) T or R = P=
R
B
( B C ) T V0 P
P2V0 – PP0V0 = – P0RT ... (iii)
1 1
Rµ and R µ . Relation between P and T is the equation of a parabola.
T B C Also, PV = RT
11. (a, c, d)
RT
1 2 A < B P=
V
After thermal equilibrium density will be minimum. ... (iv)
Hence level of A increases and level of B may increase
From (i) and (iii)
or remain same.
HEAT AND THERMODYNAMICS 435

15. (a, c, d)
RT P0
V + P0 Heat is energy transfer by virtue of temperature
V V0
difference.
16. (a, c)
P0 2
V + P0V B

KV ²
RT = ... (v)
V0

PV
P

P=
²=
[Note : The above equation is of a parabola (between T and

C
V)]
A
C
P0 2 P0
Equation T = V + V
V0 R R V
From equation PV = nRT
Differentiating the above equation w.r.t.V, we get
PC < PB, VC = VB and TB > TC
dT P0 P0 WAB > WAC
2V +
dV V0 R R Also, (TB – TA) > TC – TA
By 1st law, Q = U + W
dT
When = 0 then 1
dV Q nR T + W ; QAB > QAC
2
P0 P0 V0
2V = V= 17. (b, c)
V0 R R 2 There is a decrease in volume during melting of an ice
Also, slab at 273 K. Therefore, negative work is done by ice-
water system on the atmosphere or positive work is
d 2T 2P0 done on the ice-water system by the atmosphere.
2
= = – ve Hence, option (b) is correct.
dV V0 R
Secondly heat is absorbed during melting (i.e. dQ is
V0 positive) and as we have seen, work done by ice-water
V= is the value of maxima of temperature system is negative (dW is negative.) Therefore, from
2
first law of thermodynamics
Also,
dU = dQ – dW
PA VA = PB VB
change in internal energy of ice-water system, dU will
TA= TB (From Boyle's law)
be positive or internal energy will increase.
In going from A to B, the temperature of the gas first
V0
increase to a maximum (at V = ) and the decreases
2
and reaches back to the same value.

1. A-q; B-r; C-s; D-p For process


2. A-p, r; B-p; C-q, r; D-q J K : W = 0 and U < 0
3. A-p, s; B-q, r; C-p, r, s; D-p, s K L : W > 0 and U > 0
4. A-p, s; B-s; C-p, r; D-q, r L M : W = 0 and U > 0
5. A-r, s; B-q, r, s; C-p, r; D-q, s M J : W < 0 and U < 0
6. A-p, q; B-r, s; C-p, q 8. A-p; B-s; C-r; D-q
(A) Isothermal process, V = 0, W = +ve Q = +ve Wiaf + Uif = Hiaf
Since volume is increasing in both processes 20 + Uij = 50 ; Uif = 30
(B) A is adiabatic with –ve work done U = +ve and Wibf + Uif = Hibf
B is isothermal Wibf + 30 = 36 ; Wipf = 6
(C) Same as A (A) Wibf = 6
7. A-s; B-p, r; C-r; D-q, s (B) Hfi = Wfi + Ufi = 13 + 30 = 43
436 IIT-JEE PHYSICS Challenger
(C) Uif = 30, Ui = 10, Uf = 40 Q= U+ W>0
(D) Hib = Wib + Uib = 6 + (22 – 10) = 18 (B) Q=0
9. A-p, s; B-q; C-q, r; D-p, s pdV = W = negative
(A) PV = nRT (C) PV = nRT
As volume decreases. T also decreases i.e., U > 0
1 1
P = (nRT) = (constant) PdV = W < 0 so Q < 0
V V
(D) For cyclic process U = 0
T = constant i.e. isothermal process
W > 0 (clockwise), Q > 0
V increases, W is positive.

1. 1714 Hence the gain in time in one day


Let m be the mass of ice. Let h be the height from which block
24 60 60
of ice is dropped. Work done, = 2 × 0.000237 × = 20.5sec.
2
W = mgh = m × 980 × h erg
3. 0.014
5
Mass of ice to be melted = m
100 Here, 49 10 5
/ C, t = 30°C

5 V = V (1 + t ) = V (1 + 49 10 5
30)
Heat required, Q = × m × 80 cal or Q = 4 m cal
100 = V (1 + 1470 × 10–5)
[ L = 80 cal g–1] = 1.01470 V
Now, m × 980 × h = J × 4 m [ W = JQ] Now density,
4.2 107 4m M M M
h= cm = 1714.3 m 1714 m = and = =
m 980 V V 1.0147V
[ J = 4.2 × 107 erg cal–1]
1
2. 20.5 or
1.0147
Let L0 and L25 be the length of pendulum at 0ºC and 25ºC
respectively.
1
We know that ÷
Fractional change in density = 1.0147
L25 = L0(1 + T) =
= T0(1 + 0.000019 × 25) = 1.000475 L0
If T25 and T0 be the time periods at 25ºC and 0ºC respectively,
(1.014 1)
then = = 0.0138 0.014
1.014
L25 L0 4. 238
g ÷ g÷
T25 = 2 and T0 = 2
Here A = 0.15 m2, 1.0 10 2 m

m 6 kg 6kg 1 kg
T25 L25 1.000475L0 = = =
t min 60 sec 10 sec
T0 = L0 ÷ ÷
=
L0
K = 109 Js–1m–1°C–1, L = 2256 × 103 J kg–1, 2 = 1000°C
= (1.000475) = 1.000237 KA ( 2) x
From formula, Q =
1

T25 T0
Now T0 = 0.000237 This heat supplied to the water is used in boiling it. Let
temperature of the part of the flame in contact with boiler is
Gain in time for one vibration = 2 × 0.000237 sec.
1.
Number of vibration in one day
KA ( 2) t mL
24 60 60 mL = 1
or 1 2 =
= (T = 2 sec.) KAt
2
HEAT AND THERMODYNAMICS 437

mL 1
or 1 = ÷+ 2 (0.75) × mv 2 = Mc T + ML
t AK 2

1 2256 103 1 10 2 1 2
= + 100 (0.75) × v = (0.03 × 300 + 6) × 4.2
10 109 0.15 2

= 137.9 + 100 = 237.9° = 238°C [4.2 to convert into S.I. system]


5. 346 v = 12.96 m/s 13 m/s
9. 9000
Here 1 50cm, 1 2.10 10 5
/ °C
K1 K2 K3
t (250 40) = 210°C , 1.2 10 5
/ °C
2 QA = 20°C Wood Cement Brick B( 10°C)
Change in length d1 d2 d3
L1 L1 1 t 50 2.10 10 5
210 = 0.22 cm.
Heat flown per second
Similarly, L2 L2 2 t 50 1.2 10 5
210 = 0.126 cm.
( A B )A
Total change in length H=
d1 d 2 d3
+ +
L L1 L2 = 0.22 + 0.126 = 346 cm. k1 k 2 k 3
6. 103
Here, P = 10 kW = 104 W, m = 8.0 kg = 8.0 × 103 g [20 ( 10)] 137
= = 9000 W
t = 25 min = 2.5 × 60 = 150s, Sp. heat = 0.91 J/g°C 2.5 1 25
+ + 10 2
Q = P × t = 104 × 150 = 15 × 105 J 0.125 1.5 1

50 This rate of heat has to be supplied by the heater to keep


Heat absorbed by block = 15 × 105 × = 7.5 × 105 J the temperature constant.
100
From formula Q = ms t 10. 3.6
We know that PV = nRT
Q 7.5 105
t= = 103°C PV 1.6 106 0.0083 16
m.s 8 103 0.91 n= = = = 5.33 moles
RT 8.3 300 3
7. 15.2
Heat entering per second to the room, According to Mayer's Relationship
5R 3R
V 2 200 200 Cv = Cp – R = R=
= = = 2000 J 2 2
R 20
When 2.49 × 104 J of heat energy is supplied at constant
H KA ( T ) volume then we can use the following relationship to find
=
t change in temperature
Q = nCv T
0.2 4.2 1 (20 T )
2000 =
0.2 10 2 Q 2.49 10 4
T= = = 375 K
nCv 3
2 5.33 8.3
2000 0.2 10 2
20 – T = = 4.76
0.2 4.2
Therefore, the final temperature = 300 + 375 = 675 K
T = 15.2°C Applying Gay Lussac's Law to find pressure,
8. 13
P1 P2
Lead bullet just melts when stopped by an obstacle. Given =
that 25% of the heat is absorbed by the obstacle. Therefore, T1 T2
75% heat is used in melting of lead.
Initial Temperature = 27°C P PT 1.6 106 675
P2 = 1 = 1 2 =
M.P. = 300°C T1 T1 300
(0.75) K.E. = Heat utilised in increasing the temperature +
P2 = 3.6 × 106 Nm–2 = 3.6 MPa
heat utilised to melt lead at 300°C
438 IIT-JEE PHYSICS Challenger
11. 800
1 (Mg / r 2 )2 1 M 2g2
The final pressure on the Gas = Atm pressure + Pressure = r2 = ... (i)
due to compression of spring 2 Y 2 r 2Y

kx This energy is released in the form of heat, thereby raising


P2 = Patm + the temperature of the wire
A
Q = mc T ... (ii)
8000 0.1 From (i) and (iii), since U = Q, therefore
P2 = 105 + 3
8 10
1 M 2g2
= 2 × 105 N/m2 mc T =
2 r 2Y
The final volume

1 M 2g2
T1=300K Atm pressure=105 N/m2 T=
2 r 2 cm Y
Ideal
Heater Monoatomic Here, m = mass of string = density × volume of string
Gas
k=8000N/m = × r2

1 M 2g2
T=
= V2 = V1 + xA = 2.4 × 10–3 + 0.1 × 8 × 10–3 2 ( r 2 ) 2 Yc
= 3.2 × 10–3 m3
1 (100 10) 2
PV PV P2V2 T1 =
Applying 1 1 = 2 2 T2 = 2 (3.14 2 10 3 ) 2 2.1 1011 420 7860
T1 T2 P1V1
= 0.00157°C 0.002 °C
5 3 14. 595
2 10 3.2 10 300
T2 = = 800 K.
10 5
2.4 10 3 The rate of heat loss per unit area per second due to radiation
is given by Stefan's-Boltzmann law
12. 10.8
4
The process is cyclic, therefore U = 0 E= (T4 – T0 )
According to first law of thermodynamics
17
Q= U+ W = 0.6 × 10 8 [(400)4 (300)4 ]
3
Q= W
= 595 watt/m2
Q1 + Q2 + Q3 + Q4 = W1 + W2 + W3 + W4
15. 273
5960 – 5585 – 2980 + 3645 = 2200 – 825 – 1100 + W4
(1) Heat lost by steam at 100°C to change to 100°C water
W4 = 765 J
mLvap = 0.05 × 2268 × 1000 = 1,13,400 J
Work done W + W2 + W3 + W4 (2) Heat lost by 100°C water to change to 0°C water = 0.05
= = 1
Heat supplied Q1 + Q4 × 4200 × 100 = 21,000 J
(3) Heat required by 0.45 kg of ice to change its temperature
1040 from 253 K to 273 K = m × Sice × T = 0.45 × 2100 × 20 = 18,900 J
= = 10.8%
9605 (4) Heat required by 0.45 kg ice at 273 K to convert into
13. 0.002 0.45 kg water at 273 K = mLfusion = 0.45 × 336 × 1000 = 151,200 J
When the mass of 100 kg is attached, the string is under From the above data it is clear that the amount of heat
tension and hence in the deformed state. Therefore, it has required by 0.45 kg of ice at 253 K to convert into 0.45 kg of
potential energy (U) which is given by the formula water at 273 K (1,70,100 J) cannot be provided by heat lost
by 0.05 kg of steam at 273 K to convert into water at 273 K.
1 Therefore, the final temperature will be 273 K.
U= × stress × strain × volume
2

1 (stress) 2
= r2
2 Y
ELECTROSTATICS 439

1. A solid sphere of radius R carries a uniform volume charge ur ur ur ur q + q + q3


density r. The magnitude of electric field inside the sphere (a) Ñòs
( E1 + E 2 + E 3 ).d A = 1 2
Î0
at a distance r from the centre is

R3r ur ur ur ur ur q1 + q2 + q3 +
Ñò s (E1 + E 2 + E3 + E 4 ).d A
rr Rr R 2r =
(a) (b) (c) (d) (b) Î0
3e 0 3e 0 r e0 r 2 e0
ur ur ur ur ur q1 + q2 + q3 + q4
Ñò s (E1 + E 2 + E3 + E 4 ).d A
2. Two identical metallic blocks resting on a frictionless
horizontal surface are connected by a light metallic spring (c) =
Î0
having the spring constant 100 N/m and an unstretched
length of 0.2m, as shown in figure 1. A total charge Q is (d) none of the above
slowly placed on the system, causing the spring to stretch 4. Two thin flat metal plates having large surface area are
to an equilibrium length of 0.3m, as shown in figure 2. The charged separately to acquire charge densities + s and – s.
value of charge Q, assuming that all the charge resides on The plates are then brought near to each other and held
the blocks and that the blocks are like point charges, is parallel to each other (Fig.) :

K
m m A B C

Figure 1

K –vely charged
m m
+vely charged
Figure 2
If EA, EB and EC denote the electric fields at the points A, B
(a) 10 µC (b) 15 µC (c) 20 µC (d) 30 µC and C respectively, then which of the following will be true :
3. q1, q2, q3 and q4 are point charges located at point as shown
s
in the figure and S is a spherical Gaussian surface of radius (a) E A = EC =
R. Which of the following in true according to the e0
Gauss’s law : s
(b) E A = E B = EC =
S e0
s
(c) E A = EC = 0, EB =
q1 R e0
q4
2s
q2 q3 (d) E A = EC = 0, EB =
e0

MARK YOUR
1. 2. 3. 4.
RESPONSE
440 IIT-JEE PHYSICS Challenger
p particle before coming to rest and acceleration of particle at
5. Two point dipoles pkˆ and kˆ are located at (0, 0, 0) and that moment are respectively :
2
(1m, 0, 2m) respectively. The resultant electric field due to 2A qA
the two dipoles at the point (1m, 0, 0) is (a) ,0 (b) 0, -
B m
9p ˆ -7 p ˆ
(a) 32p Î k (b) k 2 A qA -2 A qA
0 32p Î0 (c) ,- (d) ,-
B m B m
7p ˆ
(c) k (d) None of these 9. In the figure shown, A is a fixed charged B (of mass m) is
32p Î0 given a velocity v perpendicular to line AB. At this moment
6. Four charges are rigidly fixed along the Y-axis as shown. the radius of curvature of the resultant path of B is
A positive charge approaches the system along the X-axis
v
with initial speed just enough to cross the origin. Then its
total energy at the origin is –
Y
A r B
+q +q
2 2q
(a) 0 (b) infinity
–q
4p Î0 r 2 mv 2
(c) (d) r
v Q X q2
–1 –q
10. A system consists of a uniform charged sphere of radius R
–2 2q and a surrounding medium filled by a charge with the volume

(a) zero (b) positive a


density r = , where a is a positive constant and r is the
(c) negative (d) data insufficient r
7. A particle of charge – q and mass m moves in a circle of distance from the centre of the charge. The charge of the
radius r around an infinitely long line charge of linear charge sphere for which the electric field intensity E outside the
density +l. Then time period will be – sphere is independent of r is –
(a) pR2a (b) 4pR2a
+l (c) 2pR a 2 (d) 3pR2a/4
11. Three identical positive charges Q are arranged at the
vertices of an equilateral triangle. The side of the triangle is
–q a. Find the intensity of the field at the vertex of a regular
tetrahedron of which the triangle is the base.
KQ KQ
(a) 6 2 (b) 2 2
a a
KQ
(c) 3 2 (d) None of these
1 a
[where k = ]
4p Î0 12. A network of six identical capacitors, each of value C is
made as shown in the figure. Equivalent capacitance between
m 4p 2 m 3 points A and B is
(a) T =p
2 r (b) T2 = r
2k lq 2klq
A
1 2klq 1 m
(c) T= (d) T=
2pr m 2pr 2klq
8. A particle of charge q and mass m moves rectilinearly under
B
the action of electric field E = A – Bx, where A and B are
positive constants and x is distance from the point where
(a) C/4 (b) 3C/4
particle was initially at rest then the distance traveled by the
(c) 4C/3 (d) 3C

MARK YOUR 5. 6. 7. 8. 9.
RESPONSE 10. 11. 12.
ELECTROSTATICS 441

13. Three concentric charged metallic spherical shells A, B and 17. If the electrostatic potential were given by
C have radii a, b and c; charge densities s , – s and s and f = f0 (x2
+ + y2 where f0 is constant, then the charge
z2),
potentials VA, VB and VC respectively. Then which of the
density giving rise to the above potential would be.
following relations is correct?
(a) 0 (b) – 6 f0 e0
s
(a) V A = (a + b + c ) 6f0
e0
(c) – 2 f0 e0 (d) –
e0
æ a2 ös 18. A conducting disc of radius R about its axis with an angular
(b) VB = ç - b + c÷
velocity w. Then the potential difference between the centre
è b ø e0
of the disc and its edge is (no magnetic field is present)
æ a 2 + b2 ös me w 2 R 2
(c) VC = ç + c÷ (a) zero (b)
è b ø e0 2e

s me w R 3 eme w R 2
(d) V A = VB = VC = ( a + b + c ) e
(c) (d)
0
3e 2
19. Four identical particles each of mass m and charge q are
14. A capacitor is charged to a potential difference of 100 V and
kept at the four corners of a square of length L. The final
is then connected across a resistor. The potential difference
velocity of these particles after setting them free will be
across the capacitor decays exponentially with respect to
time. After 1s, the potential difference between the plates of 1/ 2 1/ 2
é Kq 2 ù é Kq 2 ù
the capacitor is 80 V. After 2 s, the potential difference (a) ê (5.4) ú (b) ê (1.35) ú
between the plates will be ëê mL úû ëê mL úû
(a) 40 V (b) 56 V
1/ 2
(c) 60 V (d) 64 V é Kq 2 ù
15. Four identical charges are placed at the four vertices of a (c) ê (2.7)ú (d) Zero
êë mL úû
square lying in YZ plane. A fifth charge is moved along X
axis. The variation of potential energy (U) along X axis is 20. Three charges –q, +q and +q are situated in X-Y plane at
correctly represented by points (0, –a), (0, 0) and (0, a) respectively. The potential at
a point distantr (r > a) in a direction making an angle q from
U U Y-axis will be
Kq æ 2a cos q ö
(a) ç1 - ÷
(a) (b) r è r ø

–X O +X –X O +X 2kq cos q
(b)
r2
U U
Kq
(c)
r
(c) (d)
Kq æ 2a cos q ö
(d) ç1 + ÷
–X O +X –X O +X r è r ø
16. A charge of 3 coulombs moving in a uniform electric field 21. A large hollow metallic sphere A (of radius R) is positively
experiences a force of 3000 newtons. The potential difference charged to a potential of 100 V and a small sphere B (of
between the two points situated in a field at a distance of 1 radius R/5) is also positively charge to a potential of 100 V.
cm from each other will be : Now B is placed inside A and they are connected by a wire.
(a) 100 volt (b) 5000 volt The final potential of A will be :
(c) 10 volt (d) 50 volt (a) 200 V (b) 150 V
(c) 120 V (d) none of above

MARK YOUR 13. 14. 15. 16. 17.


RESPONSE 18. 19. 20. 21.
442 IIT-JEE PHYSICS Challenger
22. A charge +q fixed at each of the points x = x 0 , 26. Two parallel conducting plates 5mm apart are held
x = 3x0, x = 5x0, ... upto ¥ on X-axis and charge – q is fixed horizontally one above the other. The upper plate is
on each of the points x = 2x0, x = 4x0, ... upto ¥ . Here x0 is maintained at a positive potential of 15 kV while the lower
a positive constant. Take the potential at a point due to a plate is earthed. If a small oil drop of relative density 0.92
and of radius 5 µm remains stationary between the plates,
Q
charge Q at a distance r form it to be 4pe r , then the then the charge on the drop will be
0 (a) 10e (b) 8e
potential at the origin due to above system of charges (c) 5e (d) 3e
will be: 27. A parallel plate capacitor of area ‘A’ plate separation ‘d’ is
(a) zero (b) infinite filled with two dielectrics as shown. What is the capacitance
of the arrangement ?
q q log e 2 A/2 A/2
(c) 8pe0 x0 log e 2 (d) 4pe0 x0
23. A beam of electron moving with velocity vo ẑ is incident on K d
a metallic sphere of radius R. Initially the electrons stick to d
K
the sphere and change it. In the steady state, the charge on 2
the sphere(under the assumption that the charge is 4K e0 A
3K e 0 A
uniformaly distributed on the surface)is (a) (b)
4d 3d
(a) –2p mv02 e R / e (b) – (2p mv02 e 0 R )
( K + 1) e 0 A K ( K + 3) e 0 A
(c) (d) .
(c) – mv02 e 0 R / e (d) – mv02 e 0 R 2d 2 ( K + 1) d
24. Two metallic bodies separated by a distance 20 cm, are given 28. Two equal point charges are fixed at x = – a and x = + a on
equal and opposite charges of magnitude of 0.88µC. The the x-axis. Another point charge Q is placed at the origin.
component of electric field along the line AB, between the The change in the electrical potential energy of Q, when it is
plates, varies as Ex = 3x² + 0.4 N/C, where x (in meters) is the displaced by a small distance x along the x-axis, is
distance from one body towards the other body as shown : approximately proportional to–
(a) x (b) x2
C (c) x 3 (d) 1/x
x 29. The electric charge required to expand a soap bubble to
A B twice its dimension is
20cm (a) 8p e0 r 3 (7 Pr + 12T ) (b) 8p e0 r 2 (7 Pr + 12T )

(c) 8p e0 r 3 (6 Pr + 12T ) (d) 8p e0 r 3 ( Pr + T )


(a)The capacitance of the system is 10µF
(b)The capacitance of the system is 20µF 30. Consider two small balls carrying equal and opposite
(c)The potential difference between A and C is 0.088 volt charges. If one is secured and other is released it can do
(d)The potential difference between A and C is cannot be
determined from the given data work W1 against the repulsive force (as it moves away). If
25. A charge Q is distributed over two concentric hollow spheres before the second ball is released, they are connected by a
of radii r and R (R > r) such that the surface densities are conductor for some time, the second ball can do work W2
1 while moving away. The heat liberated in the conductor
equal. The potential at the common centre is times – when the balls are connected, the energy at the expense of
4pe 0
which heat is liberated and mechanical work changed is.
é r+R ù Q æ r+R ö
(q1 + q2 )2 æ 1 1 ö (q1 + q2 ) æ 1 1 ö
2 çè r 2 + R 2 ÷ø
(a) Qê 2 ú (b)
ë r + R2 û (a) ç - ÷ (b) ç - ÷
4 èr Lø 2 èr Lø
æ r+R ö
2Q ç 2 (q1 + q2 ) 2 æ 1 1 ö (q1 + q2 )2 æ 1 1 ö
(c) è r + R 2 ÷ø (d) zero
(c) ç 2 - 2 ÷ (d) ç - ÷
4 èr L ø 4 èr Lø

MARK YOUR 22. 23. 24. 25. 26.


RESPONSE 27. 28. 29. 30.
ELECTROSTATICS 443

31. Two small equally charged conducting balls are suspended


from long threads secured at one point. Their charges and ¥
masses are such that they are in equilibrium when distance l/2
between them is a = 10 cm : (Length of the thread L >> a).
One of the balls is then discharged. What will be the distance
‘b’ between the balls when equilibrium is restored ?

a a2 ¥
(a)
3
(b) ¥
2 3
4

(c) a (d)
a2
3
4 3
2 l/2

32. The capacitance of a parallel plate condenser whose plates ¥


are not quite parallel, the separation at one edge being (d +
a) and at opposite one being (d – a). (assume d >>> a and me 0 2me0
neglect edge effects) is. (a) p (b) 2p
3sl 3sl
Ae0 æ 1 a 2 ö Ae0 æ 1 a 2 ö
(a) ç1 + ÷ (b) ç1 + ÷ p me 0 me 0
d çè 2 d 2 ÷ø d 2 çè 3 d 2 ÷ø (c) (d) 4p
2 3sl 3sl
35. The left plate of the capacitor shown in the figure carries a
Ae0 æ 1 a 2 ö Ae0 æ 1 a ö
(c) ç1 + ÷ (d) ç1 + ÷ charge +Q while the right plate is uncharged at t = 0. The total
d çè 3 d 2 ÷ø d2 è 2 d ø charge on the right plate after closing the switch will be

33. What amount of heat will be generated in the circuit, after +Q 0


the switch is shifted from position 1 to position 2 ? C

C C0 C
e

Q Q
1 2 (a) + Ce (b) – Ce
2 2
E
Q
(c) – (d) –Ce
2
CC0V CC0V 2
(a) (b) 36. A solid sphere of radius R, and dielectric constant k has
C0 + 2C 2C0 + C
spherical cavity of radius R/4. A point charge q1 is placed in
the cavity. Another charge q2 is placed outside the sphere
CC0V 2 CC0V
(c) (d) at a distance of r from q. Then coulombic force of interaction
C0 + 2C C0 + 2C 2 between them is found to be F1. When the same charges are
34. An infinite non-conducting sheet having charge density s separated by same distance in vacuum then the force of
and there is a small hole as shown in the figure and there is interaction beween them is found to be F2 then
rod of length l having linear charge density l and hinged (a) F1 = F2/k (b) F2 = F1/k
in the hole as shown. If the mass of the rod is m then time 1
period of oscillation for small angular displacement is (c) F1. F2 = (d) F1 = F2
k

MARK YOUR 31. 32. 33. 34. 35.


RESPONSE 36.
444 IIT-JEE PHYSICS Challenger
37. Two conducting spheres of radius r and 2r at very large qd
separation. Each sphere is having charge Q. These sphere
are connected with a conducting wire of resistance R. Then,
the current flowing in the wire at t = 2 µs. (where r = 30 cm,
R = 90 kW and Q = 10 µC)
100 50
(a) amp (b) amp qb qc
3e 3e
50 25
(c) amp (d) amp
e 3e
38. Two batteries of emf E1 and E2, a capacitor of capacitance
C, and a resistor R are connected in a circuit as shown in (a) F1 < F2 < F3 (b) F1 = F2 < F3
figure. Then, the amount of heat Q liberated in the resistor (c) F1 = F2 > F3 (d) F1 > F2 > F3
after switching the key K will be
41. Three infinite plane have a uniform surface charge
R distribution s on its surface. All charges are fixed. On each
of the three infinite planes, parallel to the y-z plane placed at
K
x = – a, x = 0 and x = a, there is a uniform surface charge of
the same density s . The potential difference between A
E1 E2 C and C is
y
B C
A
C ( E1 + E2 )2 + + +
(a) C (E1 + E2)2 (b) + + +
2 + + + + + +
+ + +
+ + +
C 1 + s +
(c) ( E1 + E2 ) 2 (d) CE12 + +
+
+ + x
4 2 + s +
s
39. Three identical particles, each possessing the mass m and
charge + q, are placed at the corners of an equilateral triangle z
with side r0. The particles are simultaneously set free and
s s
start flying apart symmetrically due to Coulomb’s repulsion (a) a (b) a
foces. 2 Î0 Î0
The work performed by Coulomb’s forces acting on each
particle until the particles fly from one another to a very sa
(c) (d) none
large distance is (where k = 1/4pe0.) 4 Î0

3k q 2 k q2 42. Two identical charged spheres are suspended by strings of


(a) (b) equal length. The strings makes an angle of 30° with each
r0 r0
other. When suspended in a liquid of density 0.8 gm/cc, the
3k q 2 k q2 angle remains the same. The dielectric constant of the liquid
(c) (d) is [density of the material of sphere is 1.6 gm/cc]
2r0 2r0
(a) 2 (b) 4
40. A spherical conductor A contains two spherical cavities. (c) 2.5 (d) 3.5
The total charge on the conductor itself is zero. However,
43. The charge Q and – 2Q are placed at some distance. The
there is a point charge qb at the centre of one cavity and qc
locus of points in the plane of the charges where the
at the centre of the other. A considerable distance r away
from the centre of the spherical conductor, there is another potential is zero will be
charge qd. Force acting on qb, qc and qd are F1 F2 and F3 (a) straight line (b) a circle
respectively. [Assume all charges are positive) (c) a parabola (d) an ellipse

MARK YOUR 37. 38. 39. 40. 41.


RESPONSE 42. 43.
ELECTROSTATICS 445

44. A uniformly charged and infinitely long line having a linear


charge density l is placed at a normal distance y from a U U
point O. Consider an imaginary sphere of radius R with O as
centre and R > y. Electric flux through the surface of the
sphere is (a) (b)
h h
+ h=R h=R
B ++
+
+ U U
+
+
+ y
+
+ O
+ (c) (d)
A+ +
+ R h h
+ h=R h=R
+
47. The figures represent charged metallic plates. Find the
incorrect distribution of charge.
2lR
(a) zero (b) e0 +s –s – s2 + s1 – s1 – s2
+ – – + – –
2l R 2 – y 2 l R2 + y 2
(c) (d)
e0 e0 (a) (b)
45. Find equivalent capacitance between A and B. [Assume
each conducting plate is having same dimensions and
+ – – + – –
e0 A
neglect the thickness of the plate, = 7 µF where A is
d
area of plates, A > > d] + s 2 – s1 + s 1 – s2 + s1 – s2 + s2 + s1
+ – + – + – + +

d A
(c) (d)
d
d
+ – + – + – + +
2d
48. Four identical square plates of side a are arranged as shown.
d The equivalent capacity between A and C

B B D

(a) 7 µF (b) 11µF A C


(c) 12 µF (d) 14 µF
46. A positively fixed charged ring is placed on a horizontal
surface on the earth. A positively charged particle q is d d 2d
released from a height R on its axis. The variation of total
potential energy (gravitational + electrostatic) as a function 3e0 a 2 3e0 a 2
of separation between the particle and the ring is correctly (a) (b)
2d 5d
depicted by, if charge particle just reaches at the centre
of ring 3e0 a 2 5e 0 a 2
(c) (d) .
3d 3d

MARK YOUR
44. 45. 46. 47. 48.
RESPONSE
446 IIT-JEE PHYSICS Challenger
49. An electric dipole is placed in an electric field whose 52. Two identical blocks are kept on a frictionless horizontal
direction is fixed but whose magnitude varies with distance. table connected by a spring of stiffness k and of original
The incorrect option is length l 0 . A total charge Q is distributed on the block such
(a) it experiences no net force and no torque that maximum elongation of spring at equilibrium is equal to
(b) it experiences a net force but no torque x. Value of Q is
(c) it experiences a net force and a torque
(a) 2l 0 4pe 0 k (l 0 + x ) (b) 2 x 4pe 0 k (l 0 + x )
(d) it experiences no net force but a torque
50. In the circuit shown, A and B are equal resistance. When S (c) 2(l 0 + x ) 4 pe 0 kx (d) (l 0 + x ) 4 pe 0 kx .
is closed, the capacitor C charge from the cell of emf e reaches
53. Electrical field intensity is given as
a steady state. The correct option is ur
E (2x + 1)yi$ + x(x + 1)$j. The potential of a point (1, 2) if
=
C potential at origin is 2 volts is,
(a) 2 volts (b) 4 volts
B (c) – 2 volts (d) 0 volts
54. A point charge 50 µC is located in the x-y plane at a point
A S r
+ whose position vector is r = (2iˆ + 3 ˆj ) m . Then electric field
e
r
at the point whose position vector is r = (8iˆ - 5 ˆj ) m
(a) During charging, more heat is produced in A than in B
(in vector form) will be
(b) In the steady, heat is produced at the different rate in A
and B (a) 90 ( -3iˆ + 4 ˆj ) V/m (b) 900 (3iˆ - 4 ˆj ) V/m

1 2 (c) 90 (3iˆ - 4 ˆj ) V/m (d) 900 ( -3iˆ + 4 ˆj ) V/m


(c) In the steady state, energy strored in C is Ce
4 55. Two capacitors of capacitances C1 and C2 are connected in
series, assume that C1 < C2. The equivalent capacitance of
1 2
(d) In the steady state, energy stored in C is Ce this arrangement is C, where
2
(a) C < C1/2 (b) C1/2 < C < C2/2
51. Three identical capacitors A, B and C are charged to the (c) C1 < C < C2 (d) C2 < C < 2C2
same potential and then made to discharge through three 56. Figure shows a system of three concentric metal shells A, B
resistanceRA, RB and RC, where RA > RB > RC. Their potential and C with radii a, 2a and 3a respectively. Shell B is earthed
differences (V) are plotted against the time t, giving the and shell C is given a charge Q. Now if shell C is connected
curves 1, 2 and 3. The correlation between A, B, C and 1, 2, to shell A, then the final charge on the shell B, is equal to
3 is

3
V 3a a
A
2
1 2a B
C
t

(a) 1 ® B, 2 ® C,3 ® A (b) 1 ® C, 2 ® B,3 ® A


(a) – 4Q/13 (b) – 8Q/11
(c) 1 ® A, 2 ® B,3 ® C (d) 1 ® B, 2 ® A,3 ® C (c) – 5Q/3 (d) – 3Q/7

MARK YOUR 49. 50. 51. 52. 53.


RESPONSE 54. 55. 56.
ELECTROSTATICS 447

57. The electric field intensity at all points in space is given by 60. A battery, or batteries, connected to two parallel plates
r produce the equipotential lines between the plates as shown.
E = 3iˆ - ˆj volt/metre. The nature of equipotential lines in
Which of the following configurations is most likely to
xy-plane is given by produce these equipotential lines ?
High potential Low potential
y y

30° 30°
(a) x (b) x
–2V –1V 0V 1V 2V

Low potential High potential (a) (b)


+ – + – – +
Low potential 2V 2V 2V
High potential
y y

60°
(c) (d)
60°
(c) x (d) x
– + – + + – – +
2V 2V 2V 2V
61. The force on an electron located on the 0-volt potential line
(in previous problem) is
Low potential High potential
(a) 2N, directed to the right
(b) zero
58. Both capacitors are initially uncharged and then connected
(c) directed to the right, but its magnitude cannot be
as shown and switch is closed. What is the potential
determined without knowing the distance between the
difference across the 3µF capacitor ?
lines
(d) directed to the left, but its magnitude cannot be
19V
determined without knowing the distance between the
lines
r uur q
62. Consider Gauss’s law Ñ ò E.dA = Î0 . Which of the following
2µF 9V 3µF 15V
is true ?

(a) 30V (b) 10V


(c) 25V (d) None of these + – +
59. Three charges +q, +2q and +4q are connected by strings as
shown in the figure. What is ratio of tensions in the strings
AB and BC ? r
A B (a) E must be the electric field due to the enclosed charge
r
C
d d (b) If net charge inside the Gaussian surface = 0, then E
must be zero everywhere over the Gaussian surface.
+q +2q +4q (c) If the only charge inside the Gaussian surface is an
(a) 1 : 2 (b) 1 : 3 electric dipole, then the integral is zero.
(c) 2 : 1 (d) 3 : 1 r uur
(d) E is parallel to dA everywhere over the Gaussian
surface.

MARK YOUR 57. 58. 59. 60. 61.


RESPONSE 62.
448 IIT-JEE PHYSICS Challenger
63. An electrostatic field line leaves at angle a from point charge
q1, and terminates at point charge –q2 at an angle b
(shown in figure). Then the relationship between a and b is
+100µC –50µC
C=5µF C=20µF
–100µC +50µC
+q1 –q 2

a b (a) 100 µC (b) 90µC


(a) q1 sin 2 a = q2 sin 2 b (b) q1 sin 2 = q2 sin 2 (c) 10µC (d) None of these
2 2
67. Three identical large metal plates of area A are arranged at
(c) q1 tan a = q2 tan b (d) q1 cos a = q2 cos b distances d and 2d from other. Top metal plate is uncharged,
64. Two very long line charges of uniform charge density +l while other metal plates have charges +Q and –Q. Top and
and –l are placed along same line with the separation bottom metal plates are connected by switch S through a
between the nearest ends being 2a, as shown in figure. The resistor of unknown resistance. What energy (in mJ) is
electric field intensity at point O is dissipated in the resistor when switch is closed ?

a Î0 A

a
+ (Given : = 6µF, Q = 60mC )
d
O ++++++
A
l
(a) (b) 0 R 2d
2pe 0 a
A
+Q
l l d
(c) (d) S
pe0 a 4pe 0 a A –Q

65. Three capacitors C1, C2 and C3 are connected to a battery (a) 0.1 mJ (b) 0.5 mJ
as shown in the figure. The three capacitors have equal (c) 1.0 mJ (d) 2.0 mJ
capacitances. Which capacitor stores the most energy ? 68. The figure shows the electric field lines in the vicinity of
two point charges. Which one of the following statements
C2 concerning this situation is not true ?

+
V – C1 C3
q1
q2

(a) C2 or C3 as they store the same amount of energy


(b) C2
(c) C1
(a) q1 is negative and q2 is positive
(d) All three capacitors store the same amount of energy
(b) The magnitude of the ratio (q2/q1) is less than one
66. For the configuration of capacitors shown, both switches (c) Both q1 and q2 have the same sign of charge
are closed simultaneously. After equilibrium is established, (d) The electric field is strongest midway between the
what is the charge on the top plate of the 5µF capacitor ? charges.

MARK YOUR 63. 64. 65. 66. 67.


RESPONSE 68.
ELECTROSTATICS 449

69. A point charge +Q is positioned at the center of the base of (a) t1 = t2 (b) t1 > t2
a square pyramid as shown. The flux through one of the (c) t1 < t2
four identical upper faces of the pyramid is (d) No definite relation between t1 and t2 may be predicted
by the given data
72. Two identical electric dipoles are arranged on x-axis as
shown in figure. Electric field at the origin will be

+Q

Q Q
(a) (b) x
16e 0 4e 0 45° 45°
Q
(c) (d) None of these
8e 0 r r
70. A dielectric slab is attached to a string of mass per unit
length µ, whose other end is fixed to a wall. Capacitor has
square plates of side b and separation between the plates is kp 2 ˆ
(a) Zero (b) j
d. Find the fundamental frequency of vibration of the string. r3
(Dielectric slab remains in equilibrium)
- kp 2 ˆ 2kp ˆ kp ˆ
i- 3 j
(c) j (d)
r3 r3 r
b
V L
////////////

73. A thin insulator rod is placed between two unlike point


d k charges +q1 and –q2 (figure). How will the forces acting on
the charges change ?
//////////////////////
+ –
q1 q2
(a) The total force acting on the charge +q1 will increase.
1 Î0 bV 2 (k - 1)m 1 Î0 bV 2 ( k + 1)m
(a) (b) (b) The total force acting on the charge +q1 will decrease.
2L 2d 2L 2d
(c) The total force acting on the charge +q 1 remain
unchanged
1 Î0 bV (k - 1) 1 Î0 bV 2 (k - 1)
(c) (d) (d) None of these
2L 2d m 2L 2d m
74. Two small balls of mass m, bearing a charge q each, are
71. A dipole is kept in front of a conducting sphere containing connected by a nonconducting thread of length 2l. At a
a total charge Q. If the dipole is released from rest, it reaches certain instant, the middle of the thread starts moving at a
the sphere in time t1. If the same experiment is repeated with constant velocity v perpendicular to the direction of the
an insulating sphere (Îr = 1) with same charge Q distributed thread at the initial instant. Determine the minimum distance
uniformly over its surface, the dipole reaches in time t2. d between the balls.
Then
Q 2 lq 2 2lq 2
(a) (b)
q 2 + 8pe 0 mv 2 l q 2 + 4pe 0 mv 2 l
P
2 lq 2 3lq 2
(c) (d)
q 2 + 3pe 0 mv 2 l q 2 + 8pe 0 mv 2 l

MARK YOUR 69. 70. 71. 72. 73.


RESPONSE 74.
450 IIT-JEE PHYSICS Challenger
75. Two identical beads each have a mass m and charge q. When
placed in a hemispherical bowl of radius R with frictionless, 1 q1q2 l 2 2 q1q2 l 2
(a) (b)
nonconductive walls, the beads move, and at equilibrium 2 pe 0 d 4 3 pe 0 d 4
the distance between them is R (Fig. ). Determine the charge
on each bead. 3 q1q2 l 2 1 q1q2 l 2
(c) (d)
2 pe 0 d 4 4 pe 0 d 4
R R 79. A parallel-plate capacitor is filled by a dielectric whose
permittivity varies with the applied voltage according to the
m m law e = aV, where a = 1 V–1. The same (but containing no
R dielectric) capacitor charged to a voltage V0 = 156V is
connected in parallel to the first “nonlinear” uncharged
capacitor. Determine the final voltage V across the capacitors.
1/ 2 1/ 2
æ mg ö æ mg ö (a) 6V (b) 12V
(a) Rç ÷ (b) Rç ÷ (c) 8V (d) 4V
è ke 3 ø è ke 2 ø
r
80. A uniform electric field E = (aiˆ + bjˆ) N/kg exists in a region.
1/ 2 1/ 2
æ mg ö æ 2mg ö Work done in displacing a particle along a straight line
(c) Rç ÷ (d) Rç ÷
è ke 3 ø represented by ax + by = c will be (a, b and c are contants)
è ke 2 3 ø
76. The minimum strength of a uniform electric field which can (a) a x2 + y2 (b) a 2 + b2 x 2 + y 2
tear a conducting uncharged thin-walled sphere into two (c) ax + by (d) zero
parts is known to be E0. Determine the minimum electric 81. Three small identical neutral metal balls are at the vertices of
field strength E1 required to tear the sphere of twice as large an equilateral triangle. The balls are in turn connected to an
radius if the thickness of its walls is the same as in the isolated large conducting sphere whose centre is on the
former case. perpendicular erected from the plane of the triangle and
E0 E0 passing through its centre. As a result, the first and second
(a) (b) balls have acquired charges q1 and q2 respectively. Determine
2 2
the charge q3 of the third ball.
(c) 2E0 (d) E0
77. Two small identical balls lying on a horizontal plane are 2q22 q22
connected by a weightless spring. One ball is fixed at point (a) (b)
q1 2 q1
O and the other is free. The balls are charged identically, as
a result of which the spring length increases twofold. Find q22
3q22
the ratio of the frequency of harmonic vibrations of the (c) (d)
2q1 q1
system after and before charging.
(a) 1/4 (b) 82. Why do electrons and not ions cause collision ionization of
2
atoms although both charges acquire the same kinetic energy
(c) 3 2 (d) 1/2 mv2/2 = eDf (e is the charge of the particles, and Df is the
78. Two parallel-plate capacitors are arranged perpendicular to potential difference) in an accelerating field ? Assume that
the common axis. The separation d between the capacitors an atom to the ionized and a particle impining on it have
is much larger than the separation l between their plates approximately the same velocity after the collison.
and than their size. The capacitors are charged to q1 and q2 (a) Because the energy of the ion required for the ionization
respectively (figure). Find the force of interaction between must be twice as high as the energy of the electron.
the capacitors. (b) Because the energy of the ion required for the ionization
q1 q2 must be thrice as high as the energy of the electron.
(c) Because the energy of the ion required for the ionization
d must be four times as high as the energy of the electron.
(d) None of these

MARK YOUR 75. 76. 77. 78. 79.


RESPONSE 80. 81. 82.
ELECTROSTATICS 451

83. A metal sphere having a radius r1 charged to a potential V1 86. Small identical balls with equall charges are fixed at the
is enveloped by a thin-walled conducting spherical shell of vertices of a right polygon with side ‘a’, At a certain instant,
radius r2 (figure). Determine the potential V2 acquired by one of the balls is released, and a sufficiently long time
the sphere after it has been connected for a short time to the interval later, the ball adjacent to the first released ball is
shell by a conductor. freed. The kinetic energies of the released balls are found to
differ by K at a sufficiently long distance from the polygon.
Determine the charge q of each ball.
r2 (a) 2pe0 Ka (b) pe 0 Ka
V1 (c) 3pe 0 Ka (d) 4pe0 Ka
r1
87. Two small balls having the same mass and charge and
located on the same vertical at heights h1 and h2 are thrown
in the same direction along the horizontal at the same
velocity v. The first ball touches the ground at a distance l
2r1 r1 from the initial vertical. At what height H2 will the second
(a) V1 (b) V1
r2 2 r2 ball be at this instant ? The air drag and the effect of the
charges induced on the ground should be neglected.
r1 r2
æ lö
(c) V1 (d) V1 2
r2 r1 (a) h1 + h2 - g ç ÷
è vø
84. The thickness of a flat sheet of metal foil is d, and its area is
S. A charge q is located at a distance l from the centre of the
æ lö
2
(b) h1 - h2 - g ç ÷
sheet such that d << S << l . Determine the force F with è vø
which the sheet is attracted to the charge q, assuming that
æ lö
1/ 2
the straight line connecting the charge to the centre of the
(c) h1 + h2 - g ç ÷
sheet is perpendicular to the surface of the sheet. è vø
(approximately)
æ lö
2
h1 + h2
q 2 Sd q 2 Sd (d) -gç ÷
(a) (b) 2 è vø
8p 2 e 0 l5 4p 2 e 0 l5
88. A particle having charge q = +2.00 µC and mass m = 0.0100
2
q Sd 2
2q Sd kg is connected to a string that is L = 1.50 m long and is tied
(c) (d) to the pivot point P in Figure. The particle, string and pivot
6p e 0 l
2 5
3p 2 e 0 l5 point all lie on a frictionless horizontal table. The particle is
85. Where must a current source be connected to the circuit released from rest when the string makes an angle = 60.0°
shown in figure in order to charge all the six capacitors with a uniform electric field of magnitude E = 300V/m.
having equal capacitances ? Determine the speed of the particle when the string is parallel
to the electric field (point a in Fig.).
A
m,q

B C L
D

q a E

(a) AB (b) AC (a) 0.150 m/s (b) 0.200 m/s


(c) AD (d) Not possible (c) 0.250 m/s (d) 0.300 m/s

MARK YOUR 83. 84. 85. 86. 87.


RESPONSE 88.
452 IIT-JEE PHYSICS Challenger

PASSAGE-1 y

–s
A sphere of radius R contains charge density r (r ) = A( R - r ),
for 0 < r < R. The total electric charge inside the sphere is Q. Plate
1. The value of A in terms of Q and R is
L
2Q 2 3Q a
(a) (b) v0
pR 4 pR 4
x
3Q 2 3Q A
(c) (d)
pR 3 pR
4. At what minimum distance, L0 , should the electron be
2. The electric field inside the sphere is
ejected so as to miss the plate ?
12Q é 1 æ r ö 1 æ r ö

(a) ê ç ÷- ç ÷ ú K 0 cos 2 a K 0 cos a
R 2 êë 3 è R ø 4 è R ø úû (a)
4pes
(b)
4pes

12Q 2 é 1 æ r ö 1 æ r ö ù
2
K 02 cos 2 a K 0 cos2 a
(b) ê ç ÷- ç ÷ ú (c) (d)
R 3 êë 3 è R ø 4 è R ø úû 4ps 4pe2 s
5. The acceleration of the electron (mass = me) along y-axis is
12Q é 1 æ r ö 1 æ r ö

(c) ê ç ÷- ç ÷ ú 4pse 2 - 4 ps e
5 R 2 êë 4 è R ø 3 è R ø úû (a) (b)
me me

12 é 1 æ r ö 1 æ r ö ù
2
-4pes 2 -4ps
(d) ê ç ÷ - ç ÷ ú (c) (d)
R 2 Q êë 3 è R ø 4 è R ø úû me eme

3. The electric field outside the sphere is 6. The ejected electron’s trajectory for plate L > L0 is
Q Q (a) Parabolic (b) Circular
(a) (b) (c) Straight (d) Elliptical
r r2

Q Q2 PASSAGE-3
(c) (d)
r 3 r2
Two identical capacitors A and B, both having same capacitance
C, are connected with battery of emf E as shown in the figure.
PASSAGE-2
Initially both switches S1 and S2 are open and initially, capacitor B
has a charge of CE coulomb as shown in the figure and capacitor
An electron with initial kinetic energy K 0 is ejected at point A as A is uncharged. Now switch S1 is closed and S2 remain open till
shown in the figure. the capacitor A becomes fully charged. Then S1 is open and S2 is
closed till the flow of charge through battery is stopped. This
process constitutes one cycle. The cycle is repeated for a number
of times.

MARK YOUR 1. 2. 3. 4. 5.
RESPONSE 6.
ELECTROSTATICS 453

11. Potential at A is
– CE +
B
Q Q
(a) (b)
4pe 0 R 16pe 0 R
P E
Q
A S2 (c) (d) None of these
20pe0 R
12. Potential at B is
Q
Q Q
(a) (b)
E S1 4pe 0 R 16pe 0 R
7. Charge on the capacitor B after 2nd cycle is
5Q
7CE 7CE (c) (d) None of these
(a) (b) 48pe0 R
8 4
CE CE
(c) (d) PASSAGE-5
2 4
8. Total work done by the battery in 5 cycles is
A parallel plate condenser consists of two plates of area A and
127 31 2 separation d. A slab of thickness t and dielectric constant k is
(a) CE 2 (b) CE
64 16 inserted between the plates with its faces parallel to the plates
and having the same surfaces area as that as shown in the figure
63 2 31 2
(c) CE (d) CE given k = 2.
32 32
9. Potential difference across capacitor for B if cycle is repeated C1 C2
for infinite time is
3E
(a) E (b)
2
5E
(c) 2 E (d)
2

PASSAGE-4
t d–t
d
Three concentric spherical conductors A, B and C of radii R, 2R
and 4R respectively. A and C is shorted and B is uniformly charged 13. The capacitance of the system is
(charge + Q)
C e0 A e0 A
(a) (b)
+ B + + d – (t / 2) d + (t / 2)
+ +
A
+ +
(c) e0 A (d) e0 A
+ +
4R + d –t d +t
+ 14. For what value of t/d will the capacitance of the system be
R +
+
2R + (3/2) times that of the condenser with air filling the full space
+ + (a) 2/3 (b) 3/2
+ + +
(c) 1 (d) 1/3
15. The ratio of energy in two cases account for the energy
10. Charge on conductor A is change is
(a) Q/3 (b) – Q/3 (a) 1/2 (b) 2/3
(c) 2Q/3 (d) None of these (c) 1 (d) 3/2

MARK YOUR 7. 8. 9. 10. 11.


RESPONSE 12. 13. 14. 15.
454 IIT-JEE PHYSICS Challenger

PASSAGE-6 E E

Three concentric conduction shells A, B and C are shown in figure.


Radius of the shells are R, 2R and 3R respectively. Initial charge (c) y (d) y
given to the shell are Q, – Q and 3Q respectively.

C
B 20. The variation of electric potential (V) at any point on y-axis
A as a function of its y-coordinate is best represented by
R (assume reference potential at infinity to be zero)

2R V
3R

16. If VA and VB are potentials of shells A and B respectively (a) y


then
(a) VA = VB (b) 2VA = 3VB
(c) 3 VA = 2VB (d) VA = 2VB.
17. If potential difference between A and B is VAB and between
V
B and C is VBC then
(a) VAB = 0 (b) VBC = 0
(c) VAB = VBC (d) 2VAB= 3VBC. (b) y
18. If shell B is connected to earth then charge given by earth
to the shell B is
(a) – 2Q (b) – Q
(c) Q (d) 2Q. V

PASSAGE-7
(c) y

Q Q V
x
(–a,0) (+a,0)
(d) y

Two point charges having positive charge Q each are fixed on x- 21. Now a third negatively charged particle is released from rest
axis as shown. at a point P (0, 2a) on y-axis. Under the electrostatic influence
19. The variation of electric field (E) at any point on y-axis as a of both fixed charges, the motion of negatively charged
function of its y-coordinate is best represented by
particle is described by
(take electric field in positive y-direction as positive).
(a) motion of negatively charged particle will be oscillatory
E E but not SHM
(b) the direction of velocity of negatively charged particle
will never change
(a) y (b) y (c) motion of negatively charged particle will be SHM
(d) the negatively charged particle will stop at origin

MARK YOUR 16. 17. 18. 19. 20.


RESPONSE 21.
ELECTROSTATICS 455

PASSAGE-8 PASSAGE-9
A thin, homogeneous stick of mass m and length L may rotate in
the vertical plane around a horizontal axle pivoted at one end of Capacitor C3 in the circuit is variable capacitor (its capacitance
the stick. A small ball of mass m and charge Q is attached to the can be varied). Graph is plotted between potential difference V1
opposite end of this stick. The whole system is positioned in a (across capacitor C1) versus C3. Electric potential V1 approaches
mg on asymptote of 10 volts as C3 ® ¥.
constant horizontal electric field of magnitude E = . The stick
2Q V1
is held horizontally at the beginning. 10
8
× Q,m C1
6
E V
C2 C3 4
22. What is the acceleration of the small ball at the instance of 2
releasing the stick ? C3
(a) 3g/2 (b) 3g/4 2 4 6 8
(c) 9g/8 (d) None of these
C1
25. The ratio of the capacitance will be
23. What is the speed of ball when rod becomes vertical ? C2
(a) 2 / 3 (b) 4 / 3
3gl
(a) (b) 2gl (c) 3 / 4 (d) 3 / 2
2
26. The value of C3 for which potential across C1 will become
3gl
(c) (d) None of these 8V is
4
24. Magnitude of acceleration of the end of the stick when it (a) 1.5C1 (b) 2.5C1
swings through the vertical position is (c) 3.5 C1 (d) 4.5 C1
3 5g 3 17 g 27. The ratio of energy stored in capacitor C1 to that of total
(a) (b)
2 8 energy when C3 ® ¥ is

3 2g (a) zero (b) 1/3


(c) (d) None of these (c) 1 (d) Data insufficient
4

MARK YOUR 22. 23. 24. 25. 26.


RESPONSE 27.
456 IIT-JEE PHYSICS Challenger

1. Statement - 1 : Two concentric charged of Statement - 2 : Disc can be supposed to be made up of


outer shell are given. many concentric rings. Also electric field
at the centre of uniformly charged ring is
zero.
The potential difference
between the shells depends 6. Two charged concentric spherical conductors have electric
on charge of inner shell. potential V1 and V2 respectively. Then
Statement - 2 : Potential due to charge of outer shell Statement - 1 : The potential at centre is V1 + V2.
remains same at every point inside the Statement - 2 : Potential is scalar quantity.
sphere. 7. Statement - 1 : The electrostatic force between the plates
2. Statement - 1 : In a given situation of arrangement of of a charged isolated capacitor decreases
charges, an extra charge is placed outside when dielectric fills whole space between
the Gaussian surface. In the Gauss theorem plates.
r r Statement - 2 : The electric field between the plates of a
Ñò E.ds = Îin0 , Qin remains
Q
unchanged charged isolated capacitance decreases
r when dielectric fills whole space between
whereas electric field E at the site of the
plates.
element is changed.
r 8. Statement - 1 : The separation between the plates of a
Statement - 2 : Electric field E at any point on the parallel-plate capacitor is made double
Gaussian surface is due to inside charge while it remains connected to cell, the cell
only. absorbs some energy.
3. Statement - 1 : The electric potential and the electric field Statement - 2 : If the separation between the plates of a
intensity at the centre of a square having parallel-plate capacitor, connected to a cell,
four fixed point charges at their vertices as is made double, then the electric field in
shown in figure are zero. the region between the plates remains
+q –q same.
9. Statement - 1 : A charge q is placed at the centre of a
metallic shell as shown in the figure. Electric
field at point P on the shell due to charge q
is zero.
–q +q
Statement - 2 : If electric potential at a point is zero then
the magnitude of electric field at that point P
q
must be zero.
4. Statement - 1 : Electric field of a dipole can’t be found
using only Gauss law. (i.e. without using
superposition principle) Statement - 2 : Net electric field within a conductor under
the electrostatic conditions is zero.
Statement - 2 : Gauss law is valid only for symmetrical
charged distribution. 10. Statement - 1 : In a region where uniform electric field
exists, the net charge within volume of any
5. Statement - 1 : A uniformly charged disc has a pin hole at
size is zero.
its centre. The electric field at the centre of
the disc is zero. Statement - 2 : The electric flux within any closed surface
in region of uniform electric field is zero.

MARK YOUR 1. 2. 3. 4. 5.
RESPONSE 6. 7. 8. 9. 10.
ELECTROSTATICS 457

1. A dielectric cylinder of radius a is infinitely long. Its volume (b) The electric field is zero at all points inside S
charge density r varies directly as the distance from the (c) At a point just outside S, the electric field is double the
cylinder. If r is zero at the axis and is rs on the surface, the field at a point just inside S.
electric intensity due to it is (d) At any point inside S, the electric field is inversely
proportional to the square of its distance from C
rs a 3 4. A small electric dipole is placed in a uniform electric field as
(a) at a point distant ‘r’ outside it
3e 0 r shown in the diagram.

rs r 2 +q
(b) at a point distant ‘r’ inside it
3e0 a
E
rs a
(c) at a point on its surface –q
3e 0
rs Considering the situation above, choose the correct
(d) at a point on its surface
3e0 a 2 statement(s).
2. Three concentric conducting spherical shells have radii r, (a) The torque on the dipole points into the plane of the
2r & 3r and charge q1, q2 & q3 respectively as shown in the paper
figure. Select the correct alternatives
(b) If allowed to rotate freely about its center, the dipole
would initially swing counter-clockwise
(c) Work done by the electric field on the dipole, in rotating
3r it from q = 90° to q = 30° is positive
(d) The potential energy of the dipole is maximum when
q1 q2 q3 the electric field is perpendicular to the dipole moment.
2r 5. For the situation shown in the figure (assume r >> length of
r
dipole) mark out the correct statement(s).

p (Small dipole)

Q
q r
(a) q1 + q3 = – q2 (b) q1 = – 2
4
(a) Force acting on the dipole is zero
q3 q3 1
(c) =3 (d) =– pQ
q1 q2 3 (b) Force acting on the dipole is approximately
4p Î0 r 3
3. A thin-walled, spherical conducting shell S of radius R is and is acting upward
given charge Q. The same amount of charge is also placed
pQ
at its centre C. Which of the following statements are (c) Torque acting on the dipole is in clockwise
correct? 4p Î0 r 2
direction.
Q
(a) On the outer surface of S, the charge density is pQ
2 pR 2 (d) Torque acting on dipole is in anti-clockwise
4p Î0 r 2
direction.

MARK YOUR
1. 2. 3. 4. 5.
RESPONSE
458 IIT-JEE P HYSICS Challenger
6. How does the total energy stored in the capacitors in the (c) The speed of particle P is
circuit shown in the figure change when first switch K1 is 1/ 2
éæ -1
closed (process-1) and then switch K 2 is also closed mö æ ql æ Röö ù
êç1 - ÷ ç 2 gR + ln ç1 - ÷ ÷ ú
(process-2). Assume that all capacitors were initially êëè M ø è pe 0 m è r ø ø úû
uncharged.
mv 2
C C (d) The tension in the string is mg -
C R+r
8. A very small earthed conducting sphere is at a distance ‘a’
C K2 from a point charge q1 and at a distance ‘b’ from a point
charge q2 (a < b). At a certain instant, the sphere starts
expanding so that its radius grows according to the law,
K1
V R = vt. Choose the correct options for the time dependence
I (t) of the current in the earthing conductor, assuming that
(a) Increases in process-1 the point charges and the centre of the sphere are at rest,
(b) Increases in process-2 and in due time the initial point charges get into the expanding
(c) Decreases in process-2 sphere without touching it (through small holes).
(d) Magnitude of change in process-2 is less than that in
æq q ö a q2 a b
process-1 (a) -v ç 1 + 2 ÷ , t < (b) -v , £t<
èa bø v b v v
7. Figure shows a smooth thin nonconducting rod of radius r
charges with uniform linear charge density l, fixed
b æq q ö a
horizontally. A neutral and smooth ring Q of mass M can (c) 0, t ³ (d) -v ç 1 + 2 ÷ , t >
v è a bø v
slide freely on the rod which happens to just fit in it, and P
is non-conducting particle having a mass m and charge q, 9. Two balls of charge q1 and q2 initially have a velocity (v) of
attached to the ring Q by means of a non-conducting and the same magnitude and direction. After a uniform electric
inextensible string of length R. Choose the correct options field has been applied during a certain time, the direction of
when string becomes vertical, if P is released from the the velocity of the first ball changes by 60°, and the velocity
position shown in figure. magnitude is reduced by half. The direction of the velocity
of the second ball changes thereby by 90°. The velocity of
Q the second ball changes to v2. The magnitude of the charge-
+ + + + + + + + +
to-mass ratio for the second ball is x if it is equal to k for the
+ + + + + +P + + + first ball. The electrostatic interaction between the balls
R
should be neglected. Choose the correct options
(a) The speed of particle P is
v 4
(a) v2 = (b) x= k1
-1 1/ 2 3 3
éæ mö æ ql æ Röö ù
êç1 + ÷ ç 2 gR + ln ç 1 + ÷ ÷ ú
êëè M ø è pe0 m è r ø ø úû
v 2
(c) v2 = (d) x= k1
2 3
2
mv
(b) The tension in the string is mg +
R+r
where v is the speed of particle.

MARK YOUR
6. 7. 8. 9.
RESPONSE
ELECTROSTATICS 459

1. Match the electric field lines with the charge configuration


Column-I Column -II
(A) Three equal positive (p)
charges placed at
the vertices of an
equilateral triangle

(B) In thin, circular disc (q)


of uniform + ve charge
distribution

(C) Two point charges of (r)


+q and – 2q separated
by a small distance

(D) A conductor of finite (s)


length with uniform
– ve charge distribution

2. When a dielectric slab is inserted between the plates of one of the two identical capacitors shown in the figure then match the
following.

A
E
B

Column-I Column -II


(A) Charge on A (p) increases
(B) Potential difference (q) decreases
across A
(C) Potential difference (r) remains constant
across B
(D) Charge on B (s) will change

1. 2.

MARK YOUR
RESPONSE
460 IIT-JEE PHYSICS Challenger
3. A non-conducting ring has linear charge density l. Match the following column regarding this ring.

q
O x

Column-I Column -II


(A) l = l0 C/m (p) Electric field at centre of
r
the ring is in the direction of – j ( )
(B) l = l0 cos q C/m, (q) Electric field at centre of
0 £ q £ 2p the ring is zero
(C) l = l0 sin q C/m, (r) Electrostatic potential at
0 £ q £ 2p centre of the ring is zero
(D) l = l0 (1 – cos 2q) C/m (s) Electric field at centre of
r
0 £ q £ 2p the ring in the direction ( –i)
4. A parallel plate capacitors of capacity C0 is charged by a battery. After charging battery is disconnected a separation between the
plates is reduced and a dielectric is filled between the plates. Column I represents a quantity and Column II represents the effect
of changes made to capacitor
Column –I Column – II
(A) P.E stored by capacitor (p) increase
(B) Potential differences between plates (q) decrease
(C) Capacity of capacitor (r) remains same
(D) Charge on capacitor (s) may increase or decrease
5. Column I Column II
(A) if separation between two charge particle is (p) Increase
increased P.E between the charge
(B) If separation between particle (q) Decreases
is decreases force exerted by
one particle on other particle
(C) On changing separation (r) Remains unchanged
between the charges; number
of electric field lines from the
charge
(D) If the separation between (s) May increase or
two unequal charge particles decrease
decreases. Then potential of
a point which lies on a line
which is perpendicular bisector
to the line joining charge

3. 4. 5.

MARK YOUR
RESPONSE
ELECTROSTATICS 461

6. Find the equivalent capacitance of the arrangement in Column – I and match it with the option in column – II
Column I Column II

A B

Long e 0 A( b + 2 a )
(A) a b (p)
2ab
Conducting
Metallic wire Metallic
sphere sphere

B
A
é ab ù
(B) a (q) 4pe 0 ê
ë b - a úû
b

Concentric spherical shell of radius a and b


Inner one is given a charge and outer is earthed.

A B
a b
(C) (r) 4pe0(a + b)

Concentric spherical shells of radius a and b


Outer one is given a charge and inner one is earthed

a
– 4pe 0 b2
(D) b (s)
+ (b - a )
a

Area of the plates is A and their separation is as indicated

6.

MARK YOUR
RESPONSE
462 IIT-JEE PHYSICS Challenger
7. Match the columns :
Column I Column II
(A) Removal of dielectric when battery is present (p) Potential difference between plates increases.

(B) Insertion of dielectric when battery is present (q) Capacitance increases.

(C) Removal of dielectric when battery is not present (r) Stored energy increases.

(D) Insertion of dielectric when battery is not present (s) Charge present on plates decreases.

(t) Electric field remain constant

7.

MARK YOUR
RESPONSE
ELECTROSTATICS 463

8. For the situation shown in the figure below, match the entries of column I with entries of column II.

q1 q2
Hollow neutral
conductor

Column I Column II

(A) In the situation shown (p) Distribution of charge on inner surface

of conductor is uniform

(B) If outside charge is not present (q) Distribution of charge on inner surface

of conductor is non-uniform

(C) If we displace the outside charge while the inside (r) Distribution of charge on outer surface

charge remains at centre of conductor is uniform

(D) If the inside charge is displaced by small amount from (s) Distribution of charge on outer surface

centre then of conductor is non-uniform

8.

MARK YOUR
RESPONSE
464 IIT-JEE PHYSICS Challenger
9. In each situation of column-I, some charge distributions are given with all details explained. In column-II, the electrostatic potential
energy and its nature is given in colunm II. Then match situation in column I with the corresponding results in column II.
Column I Column II

1 Q2
(A) A thin shell of radius a and having a charge –Q (p) in magnitude
8p Î0 a 2

uniformly distributed over its surface as shown.


–Q

5a 3 Q2
(B) A thin shell of radius and having a charge –Q (q) in magnitude
2 20p Î0 a
uniformly distributed over its surface and a point charge
–Q placed at its centre as shown.
–Q

–Q
5a
2
2 Q2
(C) A solid sphere of radius a and having a charge –Q (r) in magnitude
5p Î0 a
uniformly distributed throughout its volume as shown.

–Q

(D) A solid sphere of radius a and having a charge –Q (s) Positive in sign
uniformly distributed throughout its volume. The solid
sphere is surrounded by a concentric thin uniformly
charged spherical shell of radius 2a and carrying charge
–Q as shown
–Q

–Q

a 2a

27Q 2
(t) in magnitude
80p Î0 a

9.

MARK YOUR
RESPONSE
ELECTROSTATICS 465

10. Column I enlists some cases of capacitors and column II enlists nature of energy density. Match column I with column II.
Column I Column II
(A) Energy density between plates of a parallel plate (p) zero
capacitor
(B) Energy density between the shells of a spherical (q) non-zero
capacitor when inner sphere is given a charge +q and
outer shell is earthed.
(C) Energy density between the shells of a spherical (r) uniform
capacitor when outer shell is given a charge +q and
inner shell is earthed.
(D) Energy density outside the outer shell in the arrangement (s) non-uniform
described in part (c) above
(t) inversely proportional to 4th power of distance from
centre

10.

MARK YOUR
RESPONSE
466 IIT-JEE PHYSICS Challenger

1. A thin half ring of radius R = 20 cm is uniformly charged with


a total charge q = 0.70 millicoloumb (mC). Find the magnitude
of electric field strength (in V/m) at the curvature centre of
this half ring. K=4

2. A 10 mF capacitor is charged to a potential difference of


50 V and is connected to another uncharged capacitor in
parallel. Now the common potential difference becomes 20 6. A circuit is shown in fig.
volt. What is the capacitance (in µF) of second capacitor ?
3. A parallel plate capacitor is maintained at a certain potential
A B
difference. When a 3 mm slab is introduced between the plates;
2m F
in order to maintain the same potential difference, the distance 3 mF
between the plates is increased by 2.4 mm. Find the dielectric
constant of the slab. 5 F
4. Two insulated metal spheres of radii 10 cm and 15 cm charged
4m F
to a potential of 150 V and 100 V respectively, are connected
by means of a metallic wire. What is the charge on the first + –
sphere (in e.s.u.) ? 6V

5. Consider a parallel plate capacitor of capacity 10 µF with air


Find the charge (in µC) on the condenser having a capacity
filled in the gap between the plates. Now one half of the
of 5 mF.
space between the plates is filled with a dielectric of dielectric
7. A pendulum bob of mass 80 mg and carrying a charge of
constant K = 4 as shown in figure. What is the new 2×10–8 coulomb is at rest in a horizontal uniform electric
capacitance (in µF) of the capacitor ? field of 20,000 volt/metre. Find the tension (in milli newton)
in the thread of the pendulum.

1. 2. 3. 4. 5. 6. 7.

MARK
YOUR
RESPONSE
ELECTROSTATICS 467

8. A thin fixed ring of radius 1 metre has a positive charge 1 × 10–5 The moving charge, when it reaches the point C at a
coulomb uniformly distributed over it. A particle of mass 0. distance of 4 m from O, has a kinetic energy of 4 joules.
9 gm and having a negative charge of 1 × 10–6 coulomb is Calculate the distance (in metre) of the farthest point D
placed on the axis at a distance of 1 cm from the centre of the which the negative charge will reach before returning
ring. Calculate the time period (in second) of oscillations. towards C.
9. The figure shows two identical parallel plate capacitors 11. Three particles, each of mass 1 gm and carrying a charge q,
connected to a battery with the switch S closed . The switch are suspended from a common point by insulated massless
is now opened and the free space between the plates of the strings, each 100 cm long. If the particles are in equilibrium
capacitors is filled with a dielectric of dielectric constant (or and are located at the corners of an equilateral triangle of
relative permittivity) 3. Find the ratio of the total electrostatic side length 3 cm, calculate the charge q (in nano coulomb)
energy stored in both capacitors before and after the on each particle. (Take g = 10 m/s2).
introduction of the dielectric. 12. Two square metal plates of side 1 m are kept 0.01 m apart like
a parallel plate capacitor in air in such a way that one of their
S edges is perpendicular to an oil surface in a tank filled with
an insulating oil. The plates are connected to a battery of
emf 500 V. The plates are then lowered vertically into the oil
V A C B C
at a speed of 0.001 ms –1 . Calculate the current
(in nano ampere) drawn from the battery during the process.
(Dielectric constant of oil = 11, e0 = 8.85 × 10–12C2N–1m–1)
13. Four point charges +8µC, –1µC, –1µC, and +8µC are fixed at
10. Two fixed, equal, positive charges, each of magnitude 5
×10–5 C are located at points A and B separated by a distance 27 3 3 27
of 6 m. An equal and opposite charge moves towards them
the points - m,- m, + m and + m
2 2 2 2
along the line COD, the perpendicular bisector of the line
respectively on the y-axis. A particle of mass 6 × 10–4 kg and
AB.
charge +0.1 mC moves along the x-direction. Its speed at
x = + ¥ is v0. Find the least value of v0 (in m/s) for which the
A +q
particle will cross the origin. Assume that space is gravity

O –q 1
free. Given = 9 ´ 109 Nm 2 / C 2 .
D C 4pe 0

B +q

8. 9. 10. 11.
1. 12. 13.

MARK
YOUR
RESPONSE
468 IIT-JEE PHYSICS Challenger

1 (a) 16 (c) 31 (c) 46 (d) 61 (c) 76 (a)


2 (c) 17 (b) 32 (c) 47 (c) 62 (c) 77 (b)
3 (b) 18 (b) 33 (c) 48 (c) 63 (c) 78 (c)
4 (c) 19 (c) 34 (b) 49 (a). 64 (a) 79 (b)
5 (b) 20 (d) 35 (b) 50 (a) 65 (c) 80 (d)
6 (b) 21 (c) 36 (d) 51 (b) 66 (c) 81 (d)
7 (a) 22 (d) 37 (b) 52 (c) 67 (a) 82 (a)
8 (c) 23 (a) 38 (b) 53 (c) 68 (b) 83 (c)
9 (c) 24 (a) 39 (b) 54 (b) 69 (c) 84 (a)
10 (c) 25 (a) 40 (b) 55 (b) 70 (d) 85 (d)
11 (a) 26 (a) 41 (d) 56 (b) 71 (c) 86 (d)
12 (c) 27 (d) 42 (a) 57 (c) 72 (c) 87 (a)
13 (b) 28 (b) 43 (b) 58 (b) 73 (a) 88 (d)
14 (d) 29 (a) 44 (c) 59 (b) 74 (a)
15 (b) 30 (a) 45 (b) 60 (c) 75 (a)

1 (b) 6 (a) 11 (b) 16 (b) 21 (a) 26 (b)


2 (a) 7 (b) 12 (c) 17 (b) 22 (c) 27 (c)
3 (b) 8 (b) 13 (a) 18 (a) 23 (a)
4 (a) 9 (c) 14 (a) 19 (c) 24 (b)
5 (b) 10 (b) 15 (d) 20 (d) 25 (a)

1 (a) 3 (c) 5 (a) 7 (d) 9 (d)


2 (c) 4 (c) 6 (d) 8 (c) 10 (a)

1 (a, b, c) 3 (a, c, d) 5 (b, c) 7 (a, b) 9 (a, b)


2 (a, b, c) 4 (a, c) 6 (a, b, d) 8 (a, b, c)

1. A-p; B-q; C-r; D-q 2. A-p, s; B-p, s; C-q, s; D-p, s 3. A-q; B-r, s; C-p, r; D-q
4. A-s; B-q; C-p; D-r 5. A-s; B-p; C-r; D-s 6. A-r; B-q; C-s; D-p
7. A-s, t; B-q, r, t; C-p, r; D-q 8. A-p, s; B-p, r; C-p, s; D-q, s 9. A-p, s; B-q, s; C-q, s; D-s, t
10. A-q, r; B-q, s, t; C-q, s, t; D-q, s, t

1 100 2 15 3 5 4 4 5 25 6 9 7 0.9
8 0.63 9 0.6 10 8.48 11 3.19 12 4.42 13 3
ELECTROSTATICS 469

1 qr q r r´ r 6. (b) There exists a point P on the x-axis (other than the


1. (a) E= = 4 ´ = origin), where net electric field is zero. Once the charge
4pe 0 R 3 3e 3e 0
pR 3 0
Q reaches point P, attractive forces of the two -ve charge
3
will dominate and automatically cause the charge Q to
1 (Q / 2) (Q / 2)
2. (c) . = (100 N / m) (0.1m) cross the origin.
4p Î0 (0.3m 2 ) Now if Q is projected with just enough velocity to reach
4 ´ 100 ´ (0.1) ´ (0.3) 2 P, its K.E. at P is zero, but while being attracted towards
Q2 = origin it acquires KE and hence its net energy at the
9 ´ 109
Þ Q = 20 × 10–6 C = 20 µC origin is positive. (P.E. at origin = zero)
7. (a) We have centripetal force equation
r uur q
3. (b) Ñò
E.ds = in
e0 æ 2kl ö mv
qç =
2

è r ÷ø r
Where E ® field due to all the charges in space.
s –s 2kql
so v=
m
4. (c)
A. .B .C 2pr m
Now, T = = 2pr
v 2kql

1
where k =
s s 4pe 0
EA = - =0
2e0 2e0 8. (c) F= qE = q (A – Bx)
EC = 0, ma = q (A – Bx)
s s s q
EB = + = . a= (A – Bx) ...(1)
2e0 2e0 e0 m
r
p vdv q q
5. (b). The given point is at axis of dipole and at equatorial = ( A - Bx ) ; vdv = ( A - Bx) dx
2 dx m m
r r r
line of p dipole so that field at given point is E1 + E2 0
q
x 2
ò vdv = ò ( A - Bx ) dx ; Ax - Bx = 0
0
m0 2
2A
x = 0, x = ...(2)
B
From eq. (1) and (2)
q qæ 2 Aö
( A - Bx) = ç A - B ´ ÷
m mè Bø

q - qA
= ( A - 2 A) = .
r 2 K ( p / 2) Kp m m
E1 = = (+ kˆ)
23 8 Kq 2 Kq 2 mv 2
9. (c) F= Þ =
r Kp ˆ r2 r2 RC
E2 = (- k )
1
r r mv 2 r 2 4 pe 0 v 2 r 2 m
7 Þ RC = Þ RC =
E1 + E2 = - Kp( - kˆ) = - 7 p kˆ Kq 2 q2
8 32p Î0
470 IIT-JEE PHYSICS Challenger

Q + Q¢ Therefore, the sought intensity of the field is


10. (c) Ñò E p dS = e0
where Q' is the charge outside the
KQ
E= 6
sphere a2
12. (c) The network is equivalent to
Q

A C C B

R P C
C
C C

r r
a
Q1¢ = ò dV = ò r
´ 4pr 2 dr Therefore equivalent capacitance
= [2C series C] // [C series 2C]
R R

æ r2 ö
r
æ r 2 R2 ö æ 2C ´ C ö 4C
= 4pa ç 2 ÷ = 4pa ç 2 - 2 ÷ = 2pa ( r - R )
= 2 çè ÷=
2C + C ø 3
2 2
è øR è ø
13. (b) Potential due to a charged shell at an outside point
Q + 2pa (r - R )
2 2
E p ´ 4pr 2 = 1 Q
e0 = 4 pe ´ r
0
Q a aR 2
Ep = + - 2 sR
4pr 2 e0 2e0 2r e0 while that inside the shell = , where R is the radius
e0
Q aR 2 of the shell.
E is independent of r if - =0
4pr e0 2
2r e 0
2 Add potential due to all the shells at a given point.
Q = 2pR2a 14. (d) During discharging of a capacitor,
11. (a) Each charge creates at point D a field intensity of q, the charge after a time t = q0e–lt
But q = CV. i.e. q µ V, C × 100ve–l1s = 80V. C
KQ
E1 = . The total intensity will be the sum of three \ e–l = 0.8
a2 After 2 s, V2 = 100 e–2l = 100 (e–l)2 = 100 × 0.64
vectors (fig.). The sum of the horizontal components \ V after 2 s = 64 V.
of these vectors will be zero, since they are equal in
15. (b) At the centre of the square net force on the charge is
magnitude and form angles of 120° with each other.
The vectors from angles of 90° – a with the vertical, zero. However, in this position the charge is in unstable
where a is the angle between the edge of the tetrahedron equilibrium and hence its potential energy is maximum
and the altitude h of triangle ABC. and finite. The four charges are assumed to be placed
symmetrically about the origin in YZ plane.
D 16. (c) Force on a charge in an electric field is
F=qE
a a V
a h
But E =
d
a qV
A B \ F=
d
h
a E
Fd 300N ´ 1 ´ 10-2 m
or V= = = 10 volt
C q 3C
The vertical components are identical each being equal
17. (b) E = -Ñf = -f0 2[ xiˆ + yiˆ + zxˆ]
KQ
to sin a . It follows from triangle ADE that
a2 = e0Ñ.E = –2e0f0Ñ.( xiˆ + yiˆ + zxˆ)
2 n = –6f0 e 0
sin a = .
3
ELECTROSTATICS 471

18. (b) e E = me w2r


1 éq q q ù
2 R 22. (d) V = 4 pe ê + + + ...ú
me w ë x0 3 x0 5 x0 û
ò E dr = ò rdr
0
Þ
e 0 1 é ( - q ) ( - q) ( - q) ù
+ ê + + + ...ú
4pe0 ë 2 x0 4 x0 6 x0 û
e
1 q é 1 1 1 1 1 ù
r = ê1 - + - + - + ...ú a
4pe0 x0 ë 2 3 4 5 6 û
1 q q log e 2
= log e (1 + 1) =
4pe0 x0 4 pe0 x0
me w 2 R 2 23. (a) Let V be the steady state potential of sphere, then
Þ V =
2e 1 2
19. (c) Potential energy of the system eV = mv0
2

K æq + q + q ö mv02
2 2 2
Kq 2 Þ V =
=4× ç ÷ = 5.4
2 è L L L 2ø L 2e
= Final kinetic energy of the system
\ Q = CV = 4pe 0 R ´
mv02
= –
2pe 0 R mv02 ( )
1 2e e
= 4 × mv2 = 2 mv2
2 negative sign has been included because of electron.
1/ 2
24. (a) Ex = 3x² + 0.4 N/C
é Kq 2 ù 0.2
\ v= ê (2.7)ú V = ò Ex dx = ò (3x + 0.4) dx = [ x 3 + 0.4 x]0.2
2

ëê mL úû 0
0

V = (0.2)³ + 0.4 × 0.2 = 0.088 volt


Kq.2a cos q Kq
20. (d) V = + Q 0.88
r2 r C= = = 10mF
V 0.088
Kq é 2a cos q ù 25. (a) qr + qR = Q ......... (1)
= ê + 1ú
r ë r û
qr 4 pr 2 s r 2
21. (c) The charge on sphere A and B are: = =
q R 4 pR 2 s R 2
1 q A
V = 4pe r qr r2
0 R/5 Þ = ........ (2)
qR R 2
From (1) and (2)
\ qA = (4pe0R)V Qr 2 QR 2
qr = and q R =
R R +r
2 2
R2 + r 2
and qB = (4 pe 0 ) V
5 qr qR
When they are connected by conducting were, the So, V = +
4pe 0 r 4pe 0 R
entire charge will flow to outer sphere, so that their
potentials become equal. Thus Q æ r+R ö
Þ V =
6R 4pe 0 çè r 2 + R 2 ÷ø
q = qA + qB = (4pe0 ) .V
5 26. (a) Electric field intensity between plates
The final potential of A now,
V 15 ´ 103
æ 6R ö E= = = 3 × 106 V/m
(4pe 0 ) ç V ÷ d 5 ´ 10 –3
1 q è 5 ø
V' = = 4 3
(4pe 0 ) R 4pe 0 R Weight of the oil drop = pa rg
3
6V 600 4
= = = 120V. = p (5 × 10–6)3 × (0.92 × 103) (9.8)
5 5 3
472 IIT-JEE PHYSICS Challenger
For equilibrium qE = mg. 30. (a) Let the initial charges on the balls be q1 and q2 .
mg 4 ´ p ´ 125 ´ 0.92 ´ 9.8 ´ 10 –15
q1q2
\ q= = C \ W1 =
E 3 ´ 3 ´ 106
L
4 ´ p ´ 1 2 5 ´ 0 .9 2 ´ 9 .8 ´ 1 0 – 15 where L = distance between the balls
= = 10 e
9 ´ 1 0 6 ´ 1 .6 ´ 1 0 –1 9
After the balls are connected, their charges become
electronic charge. identical.
A / 2e 0 Ae 0 Ae Ae
27. (d) c1 = = , c2 = K 0 , c2 = K 0 i.e., q = q1 + q2 \ W2 = 1 2
(q + q ) 2
d /2 d d 2d 2 4L
According to law of conservation of energy, total
c1 amount of energy in the balls should be same in both
c3 cases.
K d
d Hence, W1 + We = W2 + Q + We
K c2 1 2
2
q2 q2
where We1 = 1 + 2 = intrinsic energy of balls
c1 ´ c2 ( 3 + K ) KAe 0 2r 2r
\ ceq. = + c3 =
c1 ´ c2 2 d ( K + 1) before connection
(Q C1 and C2 are in series and resultant of these two q2 q2
in parallel with C3) and We2 = + = intrinsic energy of balls
2r 2r
2KQq after redistribution of charges
28. (b) Ui = ; The energy liberated as heat is
a
Q = We1 - We2 + W1 - W2
KQq KQq 2KQqa
Uf = + =
a + x a - x (a 2 - x 2 ) (q1 + q2 )2 æ 1 1 ö
q q = çè - ÷ø
4 r L
(–a,0) Q (a,0) 31. (c) Let the charge and mass of each ball before contact be
é a 1ù e and m and let the length of the threads be L.
DU = U f - U i = 2KQq ê 2 - ú
ëê (a - x ) a ûú
2

é x2 ù
= 2KQq ê a
2 ú
ëê a (a - x ) ûú
2

2KQqx 2
= (since a 2 - x 2 » a 2 ) L
a3
29. (a) Let P1 = internal pressure inside the bubble
r = radius of the bubble F1
F2
T = surface tension a
P = atmospheric pressure
Then, P1 = P + 4T / r mg
When the bubble expands after charging, its radius Now, L >>>> a and b
becomes 2r, hence, volume becomes 8 times the initial Force of interaction between the charges
volume. Therefore pressure inside the bubble reduces
e2 e2
to P1 / 8 due to mechanical outward pressure resulting F1 = and =
L2 a 2
from charge q given to the bubble. Resultant of the forces of gravity and tension in the
\ Total outward pressure = Total inward pressure thread

1æ 4T ö q2 4T F2 = mg tan a
çè P + ÷ø + = P+ As per the conditions of equilibrium we have
8 r 32e0 p (2r )
2 4 r
F1 = F2
q = 64p e0 r [7 P r + 12T ]
2 2 3
e2 a
Þ = mg tan a = mg
or q = 8p e0 r (7 P r + 12T )
3
a 2 2L
ELECTROSTATICS 473

On contact, the charge e will be equally distributed


1
between both balls. Hence, now we have qL = a, bL = A
2
e2 b e2b Ae0 æ 1 a 2 ö
= mg tan b = mg =
2 2 L a3 \ C= ç1 + ÷
4b d çè 3 d 2 ÷ø

a
Hence, b = 33. (c)
3
4
+q +q0 q – q0
32. (c) Let L and b be the length and breadth of each plate. If
C C0 C
plate AB makes angle q with plate CD, then, –q –q0 q + q0

qL = 2 a
1 2
Let this capacitor be divided into large number of
different capacitors. Consider one such capacitor at a
distance x from the middle point of the plate and of E
(a)
length dx. The capacitance of this differential capacitor
is given by :
q0 q0
dC = e0 b dx /(d + x q)
– q – q0 –q0 –q
As these capacitors are parallel, hence their capacitance C C0 C
q + q0 +q0 +q0
is given as

1 2
B q0 q0
E
q (b)
A d dx The charges on the capacitors in position 1 are shown
in Fig. (a). We have
x q q0 q + q0
C D = =
L C C0 C + C0

æ 1 1ö
e0 b and (q + q0 ) ç + ÷ =V
ò
c = dC = ò d + xq
dx è C + C0 C ø

C (C + C0 )V
+L / 2 or q + q0 =
ò
dx C0 + 2C
= e0 b
-L / 2 d + x q

C 2V CC0V
Hence, q = , q0 =
e0 b C0 + 2C C0 + 2C
= [log e ( d + x q)]+- LL // 22
q
After the switch is thrown to position 2, the charges
change as shown in (Fig.b). A charge q0 has flown in
æ 1 ö
e0 b ç d + 2 qL ÷ the right loop through the two capacitors and a charge
= log e ç ÷
q 1 q0 through the cell. Because of the symmetry of the
çç d - qL ÷÷
è 2 ø problem there is no change in the energy stored in
capacitors.
Thus, H = Heat produced = Energy delivered by the cell
æ1+ y ö æ y2 ö
As loge ç ÷ = 2 y çç 1 + ÷
è 1- y ø è 3 ÷ø CC0V 2
= DqV = q0V =
C0 + 2C
474 IIT-JEE PHYSICS Challenger

l/2 K (2Q – q) kq dq
æ s ö s - = R
34. (b) t = 2ç ldx÷ x sin q = l ò xdx sin q 37. (b)
r 2r dt
è 2e 0 ø e0
0
K q dq
(s/2e0)ldx
ò rR dt = òQ 3q
2Q –
2
q 2Q–q
2r
r
R
q
dx -3kt
x 4Q – 3q = Q e 2rR
x
Differentiating w.r.t. time
dx 3kt
3dq 3 KQ – 2rR
– =– e
dt 2 rR
3t
(s/2e0)ldx Q –
i= e 8pe 0rR
8pe 0 rR
s l2
t= l sin q 38. (b) Before switching
2e 0 4
q 1 = E 1C
t = –I a R

s l 2 12
a = –t / I = l . sin q
2e0 4 ml 2
– q1
E1 E2
æ 3sl ö + q2
a = –ç q (for small angle)
è 2me0 ÷ø
After switching k
2me 0 q 2 = E 2C
T = 2p
3sl R
35. (b) Electric field between the capacitor plates

s1 ( – s 2 )
= + + q1
2e 0 2e 0 E1 E2
– q2
Q+x –x
C So, work done by battery
E2 = (q1 + q2) E2
Heat liberated

æ q 2 q 2 ö C ( E1 + E2 )2
e Q = E2 (q1 + q2 ) – ç 2 – 1 ÷ =
è 2C 2C ø 2
Q+ x x 1
E= + =
2 Ae 0 2 Ae0 2 Ae 0
[Q + 2 x] 39. (b) Since the given system is closed, the increase in KE is
equal to decrease in P.E.
d
Þ Potential difference Ed =
2 Ae 0
[Q + 2 x] = e Þ
3 2 2k q 2 3k q 2
mv = –
2 r0 r
Q + 2x Q
Þe= Þ – x = – Ce 2kq 2 (r – r0 )
2C 2 Þv= , v will be max when r ® ¥
36. (d) Coulombic force between them remains same. mrr0
ELECTROSTATICS 475

[ s and r are densities of liquid and sphere


2k q 2 respectively]
Þ vmax =
m r0 q'=q
r 1.6
The work performed by the interaction force during k= = =2
the variation of the system’s configuration is equal to r – s 1.6 – 0.8
the decrease in the potential energy P (x, y)
2 43. (b)
3k q
W = U1 – U 2 =
r0
(–a, 0) (a, 0)
2
kq
\ Work done per particle = +Q O –2Q
r0
kQ –2kQ
VP = + =0
k (qb + qc )qd
40. (b) F3 = ( x + a) 2 + y 2 ( x – a) 2 + y 2
r2
F1 = F2 = zero 44. (c) Charge on length AB = 2 R 2 – y 2 ´ l

qb + qc qd ur uur 2l R 2 – y 2
Electric flux = Ñò E . ds =
e0

+
–qb –qc B ++
+
+
qb qc +
+
+ y
+
+ O
+
A ++
B ur + R
41. (d) VV = - ò E.d l = zero +
+
A
42. (a) Initially
4C
T cos q = mg ....(i) ´C
11C
and T sin q = F ....(ii) 45. (b) Ceq = 3 +C =
7C 7
From (i) & (ii)
3
F
tan q =
mg C
When suspended in a liquid of density s and dielectric
constant k. C C
A B
C/2
q

T T
F F
+q +q C

mg mg 46. (d) When the particle is a height y, the total potential energy

F
F'= +q
k
mg' = mg – v s g y
+ +
F' F + +
tan q ' = =
mg ' æ sö R
kmg ç 1 – ÷ +
è rø +
+ +
+
476 IIT-JEE PHYSICS Challenger
UTotal = UElectrostatic + Ugravitational 52. (c) For maximum elongation charges on the blocks must
Qq be equal to Q/2 on each block.
= + mgy
4 pe 0 R 2 + y 2 QQ
1 2 2
dU \ = kx
Force on the particle will be zero where F = – =0 4 pe 0 (l 0 + x ) 2
dy
47. (c) The field at any point inside any one of the plate should
Q = 2 ( l 0 + x ) 4 pe 0 kx .
be zero. Option (a), (b), (d) satisfies the above criteria
but (c) does not.
ur r
v (1,2)
48. (c) Equivalent circuit is shown
3e0 a 2
53. (c) ò dV = - ò E.d r
\ Ceq = 2 (0,0)
5d
r
a2 dr = dxi$ + dy $j
C1 = Ceq
a2 d ur r
C1 = Ceq E.dr = (2xy + y) dx + (x2 + x) dy = d (x2y + xy)
d
B v (1,2)
A D C \
ò dV = ò ( x 2 y + xy )
2 (0,0)
a2
C1 = Ceq
d V – 2 = – [12 × 2 + 1 × 2) – 0]
49. (a) Following 3 diagrams explain why b, c, d are correct V – 2 = – 4, V = – 2 volts.
and a is incorrect.
r kQ r 9 ´ 109 ´ 50 ´ 10-6 r r
E = E0y E = E0y E = E0y 54. (b) E= 3 r = r r ´ (r - r0 )
qE1 r | r - r0 |3
qE1
r r
+ where r - r0 = (8$i - 5 $j ) - (2iˆ + 3 $j ) = 6$i - 8 ˆj
+

– r
E = 900 (3iˆ - 4 ˆj ) V/m
–qE2
qE2 55. (b) C1 < C2
F = 0, t ¹ 0 E1 > E2 here E1 > E2 C1 1 C2 1
\ (d) is correct \ qE1 > qE2 \ Fnet¹0 & tnet¹0 \ < and >
C1 + C2 2 C1 + C2 2
Þ Fnet ¹ 0 \ (c) is correct
but t = 0
\ (b) is correct C1C2 C2 C
C= = C1. > 1
and option (a) is not possible for any position of the dipole. C1 + C2 C1 + C2 2
e
50. (a) In steady state potential difference across C is C2
2 Similarly, C <
2
2
1 æ eö C e2 56. (b) From given conditions,
\ Energy stored in C is = Cç ÷ =
2 è 2ø 8 VA = VC and VB = 0
\ (c) and (d) are incorrect.
k (Q - q1 ) kq 2 kq1
In steady state current through resistors A and B will Þ VB = + + =0
be same. So heat will be produced at the same rate in 3a 2a 2a
both A and B as their resistances are same. Þ 2Q + q1 + 3q2 = 0 ......... (1)
\ (b) is also incorrect.
Before the current reaches the steady state, current in Q–q1
A is more than that in B. So the heat dissipated in A is q2
more than that in B.
q1
51. (b) Charge on the three capacitors is same initially and
during discharging the potential difference across them 3a a
A
will be given by 2a B
C
VA = V0 e t / R AC
,VB = V0 e t / R BC

VC = V0 e t / R C .C Using VA = VC
AlsoRA > RB > RC.
ELECTROSTATICS 477

k ( Q - q1 ) kq 2 kq1 kq1 k (Q - q1 ) kq 2 64. (a) The field at O due to small element dx is


+ + = + +
3a 3a 3a a 3a 2a 1 ldx O
dE = .
q 4pe 0 x 2 dx x
Þ q1 = - 2 ......... (2)
4 Hence, due to one wire,
8 ¥
Using it in (1), q2 = - Q 1 ldx
11 E1 = ò . 2
4pe0 x
57. (c) Electric field is perpendicular to equipotential surfaces a
from high to low potential.
58. (b) Let q charge flow through the circuit, then using 1 l
E1 = . towards left.
Kirchoff’s law 4pe0 a
q q
19 + 15 + - 9 + = 0 Electric field at O due to other wire,
3 2
1 l
25 +
5q
= 0 Þ q = 30µC E2 = . towards left
6 4pe 0 a

P.D. across 3µF is V =


30µC 1 l l
= 10V \ Net field at O is E = 2 ´ =
3µF 4pe 0 a 2pe0 a

kq.2q kq.4q 3kq 2 65. (c) Potential drop across C1 is maximum.


59. (b) TAB = 2
+ 2
= Hence, energy stored in C 1 is maximum as
d (2d ) d
energy µ (potential drop) 2.
k .4q.2q kq.4q 9kq 2
TBC = + = q
d2 (2d ) 2 d2 66. (c)

60. (c)
d
–2V E +2V
100 –q –50 +q
F
–100+q +50 –q

+ – – +
–2V 2V 0 +2V
2V

æ 4V ö 100 - q 50 - q
F = eE = ç ÷ -6
+ = 0 Þ q = 90 × 10–6 C
è d ø 5 ´ 10 20 ´ 10-6
\ Final charge on 5µF top plate is 10µC.
61. (c) Force on electron is directed to the right but its
magnitude cannot be determined as E is unknown. Q2d
r 67. (a) Ui = then VC = VA (At equilibrium condition)
62. (c) E is due to all charges. Net flux is due to enclosed 2e 0 A
charge only. VAB = VBC
63. (c) Flux passing through the cone will remain same for Q1d (Q - Q1 )(2d )
both. =
e0 A e0 A

+q1 –q2
+Q
Thus, f1 = f2
–Q
q1 q
or (1 - cos a ) = 2 (1 - cos b) Q1 = 2 (Q – Q1)
2 Î0 2 Î0 3Q1 = 2Q
(using solid angle)
a b Q12 d (Q - Q1 )2 2d Q2 d
q1 sin 2 = q2 sin 2 Uf = + =
2 2 2e0 A 2e 0 A 3e0 A
478 IIT-JEE PHYSICS Challenger
X 72. (c) The dipoles can be resolved along x and y axes as
C
–(Q–Q1) shown in figure.

R y
Q – Q1
B
– Q1
Q1 p/ 2 p/ 2
A
X
x
Q2 d Q 2 d Q 2 d (60) 2
DU = - = = = 0.1 mJ p/ 2 p/ 2
2e 0 A 3e 0 A 6e 0 A 6(6)
r r
68. (b) As more lines emit from q1
\ Its magnitude is greater than magnitude of q2. The formulae for electric field at axial and equatorial
| q1 | > | q2 | and q1 is (+)ve q2 is (–)ve points of a dipole are :
Q 2 Kp
69. (c) Flux going in pyramid = . E (axial) = along the dipole
2e 0 r3
which is divided equally among all 4 faces.
Kp
Q E (equatorial) = opposite to the direction of dipole.
\ Flux through one face = r3
8e 0 Electric field at centre due to horizontal components of
dipoles will cancel out.
1 T
70. (d) Fundamental frequency, f = Total electric field at centre due to vertical components
2L m
T = force on dielectric slab. Kp / 2
will be E = ´ 2 in vertically downward
To calculate force on slab consider capacitor as r3
combination of two capacitors one with slab and one
direction.
with air.
- Kp 2 ˆ
dU 1 é k Î0 bx Î0 b(b - x ) ù 2 \ E= j
F =- ; U = ê + úV r3
dx 2ë d d û 73. (a) Due to polarisation of the insulator rod AB, the point
dU 1 é k Î0 b Î0 b ù 2 1 Î0 bV 2 charge +q1 will be acted upon, in addition to the point
= - V = (k - 1)
dx 2 êë d d úû
charge –q2, by the polarisation charges formed at the
2 d ends of the rod (figure).
A B
1 Î0 bV 2 (k - 1)
Thus, f = + –– – +
+
2L 2d m q1 q2
71. (c) In case of the insulating sphere, charge distribution on The attractive force exerted by the negative charge
sphere remains uniform irrespective of motion of the induced at the end A will be stronger than the repulsive
dipole. force exerted by the positive charge induced at the end
In case of the conducting sphere, charge distribution B. Thus, the total force acting on the charge +q1 will
on sphere does not remain uniform throughout motion increase.
of dipole, because electric field inside the conducting 74. (a) Let us go over to the inertial reference frame fixed to
sphere must be zero. the moving centre of the thread. Then the balls have
The charge distribution is shown in the figure for the the same velocity v at the initial instant. The energy
conducting sphere. stored initially in the system is
++++
++ + q2 2mv 2
+ W1 = +
4pe 0 .2l
+
+

2
++
++ ++

P At the moment of the closest approach, the energy of


+++++

E=0 the system is

q2
++

W2 = .
+

4pe 0 d
+ ++
+ ++ + + +
This type of charge distribution reduces the attractive Using the energy conservation law, we find that
force between the sphere and dipole. Therefore the
2lq 2
time taken by dipole to reach the sphere is lesser in d=
case of conducting sphere i.e. t1 < t2. q + 8pe 0 mv 2 l
2
ELECTROSTATICS 479

75. (a) The bowl exerts a normal force N on each bead, directed we obtain the following expression for the length l of
along the radial line or at 60.0° above the horizontal. the unstretched spring :
Consider the free-body diagram of the bead on the left
q2
with the electric force Fe applied: l3 = ,
y 16pe0 k
N where k is the rigidity of the spring and q are the
charges of the balls.
Let us suppose that the free balls is deflected from the
equilibrium position by a distance x which is small in
comparison with l. The potential energy of the system
Fe 60°
depends on x as follows :
x
1 q2 5
W ( x) = k (l - x 2 ) + » k l 2 + kx 2
2 4pe 0 (2l - x) 2
Here we have taken into account the relation between
mg q1, k, and l obtained above and retain the terms of the
order of [x/2l)]2 in the expression
SFy = N sin 60° - mg = 0 1 1 1 é æ xö æ xö
2 ù
= = ê1 + ç ÷ + ç ÷ + ......ú
2l - x 2l (1 - x / 2l) 2l êë è 2l ø è 2l ø úû
mg
Þ N=
sin 60° Thus, it is as if the stretched spring has double the
rigidity, and the ratio of the frequencies of harmonic
S Fx = - Fe + N cos 60° = 0
n2 2k
ke q 2 vibrations of the system is =
= 2
mg mg n1
Þ 2 = N cos 60° = = k
R tan 60 ° 3 78. (c) Let us consider the case when the capacitors are
oriented so that the plates with like charges face each
1
ke = » 9.0 ´ 109 Nm 2 / C 2 . other (figure). The field produced by the first capacitor
4pe 0 on the axis at a distance x from the positive plate is
1/ 2
æ mg ö q1 é1 1 ù 2q1l
Thus, q = R ç ÷ E ( x) = ê 2- 2ú
» ( x >> l)
è ke 3 ø 4pe 0 ëê x ( x + l) ûú 4pe 0 x3
76. (a) The density of charges induced on the sphere is The force acting on the second capacitor situated at a
proportional to the electric field strength : s µ E. The distance d from the first is
force acting on the hemispheres is proprotional to the
field strength : q1q2l é 1 1 ù 3q1q2l2
F = q2 [ E (d ) - E (d + l) = ê 3- ú»
F µ s SE µ R2E2, 4pe 0 ëê d (d + l)3 ûú 2pe0 d 4
where S is the area of the hemisphere, and R is its Therefore, the capacitor will repel each other in this
radius. As the radius of the sphere changes by a factor case.
of n, and the field strength by a factor of k the force will A similar analysis can be carried out for the case when
change by a factor of k2n2. Since the thickness of the the capacitors are oriented so that the plates with unlike
sphere walls remains unchanged , the force tearing the charges face each other. Then the capacitors will attract
sphere per unit length must remain unchanged, each other with the same force
k 2 n2 1 1 3 q1q2 l 2
i.e. = 1 and k = = , F=
n n 2 2 pe0 d 4
Consequently, the minimum electric field strength
capable of tearing the conducting shell of twice as large 79. (b) The capacitance of the nonlinear capacitor is
C = e C0 = aVC0,
E0 where C0 is the capacitance of the capacitor without a
radius is E1 = .
2 dielectric. The charge on the nonlinear capacitor is
77. (b) From the equality of the electric and elastic forces acting qn = CV = aC0V2, while the charge on the normal
capacitor is q0 = C0V. It follows from the charge
on the free ball,
conservation law
q2 qn + q0 = C0V0
= kl
4pe 0 .4l 2 4aV0 + 1 - 1
that the required voltage is V = = 12V .
2a
480 IIT-JEE PHYSICS Challenger
80. (d) The straight line is perpendicular to the direction of 83. (c) The charge q1 of the sphere can be determined from
r r the formula
the electric field. Hence W = F .DS = 0
81. (d) Let l be the distance from the large conducting sphere q1 = 4pe 0V1r1
to each of the small balls, d the separation between the After the connection of the sphere to the envelope,
balls, d the separation between the balls, and r the the entire charge q1 will flow from the sphere to the
radius of each ball. If we connect the large sphere to envelope, and will be distributed uniformly over its
the first ball, their potentials become equal : surface. Its potential V2 (coinciding with the new value
1 æ Q q1 ö of the potential of the sphere) will be
+ =V
4pe 0 çè l r ÷ø ........ (1)
q1 r
Here Q is the charge of the large sphere, and V is its V2 = = V1 1
4pe 0 r2 r2
potential. If the large sphere is connected to the second
ball, we obtain a similar equation corresponding to the 84. (a) Since the sheet is metallic, the charges must be
equality of the potentials of the large sphere and the redistributed over its surface so that the field in the
second ball bulk of the sheet is zero. In the first approximation, we
can assume that this distribution is uniform and has
1 æ Q q1 q2 ö
+ + ÷ =V density –s and s over the upper and the lower surface
4pe 0 çè l d rø .......... (2)
respectively of the sheet. According to the
(We assume that the charge and the potential of the superposition principle, we obtain the condition for
large sphere change insignificantly in each charging the absence of the field in the bulk of the sheet :
of the balls.) When the large sphere is connected to
the third ball, the first and second balls being charged, q s
- =0
the equation describing the equality of potentials has 4pe 0 l 2 e0
the form
Let us now take into consideration the nonuniformity
1 æ Q q1 q2 q3 ö of the field produced by the point charge since it is the
+ + + ÷ = V ......... (3)
4pe 0 çè l d d rø single cause of the force F of interaction. The upper
surface of the sheet must be attracted with a force
The charge q3 can be found be solving the system of
equations (1)-(3) : sSq
, while the lower surface must be repelled with
q2 4pe 0 l 2
q3 = 2
q1 sSq
a force .
82. (a) According to the momentum conservation law, 4pe 0 (l + d ) 2
mv = (m + M) u,
where m is the mass of the accelerated particle, M is the Consequently, the force of attraction of the sheet to
mass of the atom, and u is their velocity immediately
sSq é 1 ù q 2 Sd
after the collision. the charge is F = ê1 - ú »
We denote by Wion the ionization energy and write the 4pe 0l2 ëê (1 + d / l)2 ûú 8p 2 e0l5
energy conservation law in the form, 85. (d) It can easily be seen that the circuit diagram proposed
mv 2
(m + M ) u 2 in the problem is a “regular” tetrahedron whose edges
= Wion + contain six identical capacitors.
2 2
D
Eliminating the velocity u from these equations, we
obtain

mv 2 æ mö
= Wion ç 1 + ÷ .
2 è Mø
If m is the electron mass, then m/M << 1, and the kinetic A B
energy required for the ionization is

mv 2
; Wion
2 C
When an atom collides with an ion of mass m » M, mv2/ Therefore, we conclude from the symmetry
2 » 2Wion, i.e. the energy of the ion required for the considerations that irrespective of the pair of points
ionization must be twice as high as the energy of the between which the current source is connected, there
electron. always exists an uncharged capacitors in the circuit
ELECTROSTATICS 481

(the capacitor in the edge crossed with the edge Then the centre of mass will move along a parabola
containing the source). For example, if the current characterized by the following equation :
source is connected between points A and B in figure, 2
h1 + h2 æ g ö æ xö
the capacitor between points C and D will be uncharged h= -ç ÷ çè ÷ø ......... (1)
since the potentials of points C and D are equal. 2 è 2ø v
86. (d) The kinetic energy of the first ball released at infinity where x is the horizontal coordinate of the centre of mass,
(after a long time) can be determined from the energy and h is its vertical coordinate. At the moment the first ball
conservation law : touches the ground at a distance x = l, the height H of the
centre of mass, according to expression (1), is
mv12 q2 æ 1 1 1 ö
= ç + + .... +
aN -1 ÷ø , h1 + h2 æ g ö æ l ö
2
2 4pe 0 è a1 a2
H= -ç ÷ ç ÷ .
2 è 2 ø è vø
where a1, a2, .........., aN–1are the distances from the first
ball (before it was released) to the remaining balls in Since the masses of the balls are equal, the second ball
the circle, a1 and aN–1 being the distances to the nearest must be at a height H2 = 2H at this instant.
neighbours, i.e. a1 = aN–1 = a (N = 1977).
æ lö
2
Analyzing the motion of the second ball, we neglect H 2 = h1 + h2 - g ç ÷
the influence of the first released ball. Then è vø
88. (d) Arbitrarily take the potential V = 0 at point P. Then the
mv22 q2 æ 1 1 1 ö
= ç + + .... + potential at the original position of the charge is
2 4pe 0 è a1 a2 aN - 2 ÷ø – E . s = – EL cos q
i.e. one of the nearest neighbours is missing in the At the final point a, V = – EL. Suppose the table is
parentheses. Therefore, frictionless. Then the law of conservation of energy is
given by:
mv12 mv22 q2 (K + U)i = (K + U)f , where K is kinetic energy and U is
K= - = or q = 4pe 0 Ka
2 2 4pe 0 a potential energy of the particle
87. (a) Let us consider the two charged balls to be a single 1 2
mechanical system. The coulomb interaction between 0 - qEL cos q = mv - qEL
2
the balls is internal, and hence it does not affect the
motion of the centre of mass. The only external force 2qEL (1 - cos q)
v=
acting on the system is the force of gravity. It is only m
this force that will determine the motion of the centre
of mass of the system. Since the masses of the balls are 2 (2.00 ´ 10-6 C) (300 N / C) (150m) (1 - cos 60°)
equal, the initial position of the centre of mass is at a =
0.0100 kg
height (h1 + h2)/2, and its initial velocity v is horizontal.
= 0.300 m/s

òsphere r dV
r
(b) Net charge inside the sphere =
1. Þ 4 p r 2 E (r ) = 4p.4pA ò0 r 2 ( R - r )dr
Due to spherical symmetry, we get
R R æ r3R r 4 ö
Q= ò0 4pr 2rrdr = 4pA ò0 r 2 ( R - r ) dr = 4p.4pA ç
ç 3
è
- ÷
4 ÷ø
æ R4 R4 ö
= 4pA ç - ÷ æ rR r 2 ö
ç 3 4 ÷ø Hence, E (r ) = 4pA ç - ÷ , for 0 < r < R
è
è 3 4ø
3Q
\ A= 3Q
pR 4 But A =
2. (a) According to Gauss law pR 4
uur uur
Ñò s
E.dS = 4pr 2 E (r )
\
é 2ù
We get, E (r ) = 12Q ê 1 æç r ö÷ - 1 æç r ö÷ ú
4 R 2 ëê 3 è R ø 4 è R ø ûú
= 4p ò0 4pr r (r ) dr
2
482 IIT-JEE PHYSICS Challenger
3. (b) The electric field outside the sphere is given by :
2K0
Q x (t ) = t sin a
E (r ) = , for r ³ R me
r2
4. (a) The electric field in the region y < L is given as : 2K0 æ 2p e s ö 2
ur y (t ) = t cos a - ç ÷t
E = 4ps $y me è me ø
When potential at y = 0 is zero, the potential energy in Eliminating t we get :
the region y < L is given as :
pes
y
$y.$y dy y=- x 2 + x cot a
U ( y ) = -(-e) 4ps ò0 K0 sin 2 a

= 4p e s y Hence, the trajectory of the electron is parabolic.


7. (b); 8. (b); 9. (c)
According to the principle of conservation of energy
When S1 is closed & S2 is open
K 0 = K + 4p e s y
(K = kinetic energy) - (2CE + q1 ) + (2CE + q1 )
A
1
Now, K = me (vx2 + v 2y )
2
1 1
Also : me vx2 = me (v0 sin a )2 = K 0 sin 2 a
2 2
E
1
\ K 0 cos 2 a = me v 2y + 4p e s y
2 When S2 is closed & S1 is open
If L > L0 for which v y = 0, the electron will not reach CE
– +
the plate. B
K0 cos 2 a
Hence, L0 =
4p e s
+ q1 - q1
5. (b) Electric field in region y < L A
ur
E = 4p s $y
Using KVL
Hence, electric force on the electron is given as
ur ur 2CE + q1 q1 – CE
F = -(e) E = -4 p e s $y E– – = 0 Þ q1 =
C C 2
From Newton’s second law, we get
3CE
d2x Q1 = 2CE + q1 =
ax = =0 2
dt 2
3E
d2y -4 p e s VB1 (potential difference across B) =
ay = = = constant 2
dt 2 me
6. (a) \ Trajectory of the electron is that of a projectile in æ 5CE ö æ 5CE ö
-ç + q2 ÷ + ç + q2 ÷
a homogeneous gravitational field. è 2 ø è 2 ø
2K0
Hence, vx (0) = v0 sin a = - sin a
me

+ q1 - q2
A
2 K0
v y (0) = v0 cos a = cos a
me After second cycle, using KVL
and
5CE
x (0) = y (0) = 0 + q2 q
E– 2 – 2 =0
Thus, by integrating (for L > L0 ), we get C C
ELECTROSTATICS 483

–3CE 2
Þ q2 = q2 , U = q
4 15. (d) Ui = f
2Cair 2Ceq
5CE 5CE 3CE 7CE
Q2 = + q2 = – =
2 2 4 4 Ui Ceq d 3
= = =

=
(2 n +1
–1 ) E = çæ 2 n +1
-
1ö æ 1ö
E = ç2- ÷ E
Uf Cair t 2

VB 2
2 n
è 2 n
2 ø è 2n ø 16. (b), 17. (b)

1 1 æ Q Q 3Q ö
For n ® ¥, ®0 VA = ç - + ÷
2 n 4pe 0 è R 2 R 3R ø
\ VB2 = 2 E.
1 æ Q Q 3Q ö
VB = ç - + ÷
Work done by battery Q in n-cycle 4pe 0 è 2 R 2 R 3R ø
CE 2 CE 2 æ 2n –1ö
= CE 2 + + ¼+ =ç CE 2
n –1 n –1 ÷ 1 æ Q
-
Q 3Q ö
+
2 2 è 2 ø VC = ç ÷
4pe 0 è 3R 3R 3R ø
10. (b) q1 + q2 = 0
q2 \ 2VA = 3VB
Q VBC = VB – VA = 0.
C
q1 18. (a) Q' ® final charge on B
B
1 æ Q Q ' 3Q ö
A \ VB = ç + + ÷ =0
4pe 0 è 2R 2R 3R ø

Q Q'
+ +Q=0
2 2
kq1 kQ kq2 Q' = – 3Q
VA = + +
R 2R 4R \ charge given by earth
kq1 kQ kq2 DQ = – 2Q.
VC = + +
4R 4 R 4R 19. (c) The electric field at points on positive y-axis will be
VA = VC positive and negative at all points on negative y-axis.
Þ q1 = – Q/3 and q2 = Q/3 Hence the correct options is (c).
é –Q Q Q ù Q
11. (b) VA = k ê + + ú = E
ë 3R 2 R 12 R û 16pe0 R

é –Q Q Q ù 5Q
12. (c) VB = k ê + + ú = y
ë 6 R 2 R 12 R û 48pe0 R
e 0 kA e A
13. (a) C1 = , C2 = 0 20. (d) The potential is positive and decreases as the point of
t d –t
Two capacitors are in series observation is moved away from origin along the y-
axis. Hence the correct option is
1 1 1 t d –t
\ = + = +
C C1 C2 e 0 kA e0 A V

e0 A
C= y
d – (t / 2)
3
14. (a) Ceq = Cair
2 21. (a) Since the distance of released particle from mean
e0 A 3 e0 A t 2 position is comparable to distance between the fixed
Ceq = Þ = charges, motion of negatively charged particle will be
d – (t / 2) 2 d d 3
oscillatory but not SHM.
484 IIT-JEE PHYSICS Challenger
a
22. (c) E 25. (a)

mg mg C1 C2+C3

l æ ml 2 ö 9g
t = mg + mg l = ç + ml 2 ÷ a ; a = V
2 è 3 ø 8l
9g 9g
a= ´l =
8l 8 V (C 2 + C 3 )
VC1 =
23. (a) E C1 + (C2 + C3 )
Q
m Initially C3 = 0

VC2
So VC1 = =6 ........... (1)
C1 + C2

Now, at VC1 = 10, C3 = ¥

V (C2 + C3 )
Þ 10 =
l 1 æ ml 2 ö C1 + (C2 + C3 )
EQl + mg + mg l = ç + ml 2 ÷ w 2
2 2è 3 ø
V
mg 3mg l 2ml 2 2 Þ 10 = Þ 10 = V .......... (2)
- l+ = w æ C1 ö
2q 2 3 çè C + C + 1÷ø
2 3

2 ml 2 2
mg l = w Eq. (1) and (2),
3

3g l 10 æC ö
v= = 6 Þ 5 = 3 ç 1 + 1÷
2 æ C1 ö è 2 ø
C
çè C + 1÷ø
2
4ml 2
24. (b) EQl = a ;
3 C1 5 2
Þ = -1 =
mg l 4ml 2 3g C2 3 3
= a ; =a
2 3 8l
10 (C2 + C3 )
26. (b) Now, VC1 = =8
4M l 2 mg l 4ml 2 3g (C1 + C2 + C3 )
EQl = µ = µ =a
3 2 3 8l
æC C ö
at =
3g 10 ç 2 + 3 ÷
è C1 C1 ø
8 Þ = 8 Þ C = 2.5C
æ C2 C3 ö 3 1
3g çè 1 + C + C ÷ø
ar = w2 r = 1 1
2l
1 1 1
27. (c) + » (where C3 ® ¥ )
C3 + C2 C1 C1
EQ
\ Total energy = energy in C1
\ Required ratio = 1
2 2
æ 3g ö æ 3g ö 3g 1 3g
a = ç ÷ +ç ÷ = +1 = 17
è 8ø è 2ø 2 16 8
ELECTROSTATICS 485

1. (a) zero.
A
4. (c) Statement 1 : As net charge of dipole is zero.
B Statement 2 : Gauss law is valid for all charge
Q1 Q2
R1 distributions.
5. (a) The electric field due to disc is superposition of electric
R2 fields due to its constituent rings as given in statement-2.
statement-1 is true, statement-2 is true. Statement-2 is a
1 Q1 + Q2 correct explanation for statement-1.
VA =
4pe 0 R2 6. (d)

1 æ Q1 Q2 ö V1
VB =
4pe 0 çè R + R ÷ø V2
1 2

1 æ 1 1 ö
Q1 ç -
è R1 R2 ÷ø
VB – VA =
4 pe 0 Statement-1 Potential inside shell 2 is V2 alone and not
r V1 + V2.
2. (c) E at any point on the Gaussian surface is due to all
charges. Statement-2 Potential is work done per unit charge
3. (c) Net potential at centre hence it is scalar.
7 (d) For isolated capacitor Q = constant, F = constant. But
kq kq kq kq
+ - + - =0 s
a/ 2 a/ 2 a/ 2 a/ 2 E= , hence E decreases.
K Î0
and field is zero due to symmetry.
\ Statement -1 is false and Statement - 2 is true.
+q a –q
8. (c) V = const. d ­ C ¯ energy ¯ E ­
9. (d) Inside shell,
a a r r
Edue to q + Einduce charge = 0

a q
–q +q 10. (a) f = net
Î0
If electric potential at a point is zero then the magnitude
In uniform electric field fclosed surface = 0
of electric field at that point is not necessarily to be

1. (a, b, c) Let us divide the charged cylinder into a large


q 1
number of co-axial thin cylindrical shells. Consider E 2prL = = r dq
e0 e0
one such co-axial shell of radii x and x + dx and
Since charge inside the Gaussian surface is the
of length L. If the thickness dx is very small, so
charge on the cylinder, hence, we get :
that the charge density can be assumed to be
a
constant at each point on it, then the charge on ca3 rs a3
ò
1
the shell is given by : E= 2pLcx 2 dx = =
2prLe0 3r e 0 3r e 0
dq = 2pxLdxr 0
At the points inside the cylinder, E is given by :
As r varies directly as the distance x, hence,
r= cx q¢ 1
r

ò ò
1
where c = constant which can be given as : E 2prL = = dq = 2pLcx 2 dx
e0 e0 e0
0
c a = rs or c = rs /a
The field E at a point P distant r outside the cr 2 rs r 2
cylinder is given by : Þ E= =
3e0 3ae0
486 IIT-JEE PHYSICS Challenger
At the point on the surface of the cylinder :
5C
r a Equivalent capacitance =
E= s 3
3e0 When K2 is closed, total energy stored
In all three cases field is directed radially outwards.
1 5C 2 5CV 2
2. (a, b, c) Potential on innermost shell is zero = V =
2 3 6
q1 q2 q3
+ + =0
r 2r 3r 5CV 2 1 1
Change in process 2 = - CV 2 = CV 2
Þ 6q1 + 3q2 + 2q3 = 0 ....(1) 6 2 3
Potential on outermost shell is zero
1
q1 q2 q3 Change in process 1 = CV 2
+ + = 0 Þ q1 + q3 = – q2...(2) 2
3r 3r 3r 7. (a, b) The loss in gravitational and electrostatic potential
3. (a, c, d) Charge – Q must be induced on inner surface. energy of the system appears kinetic energy. Since
+ 2Q the work done by the tension in the string is zero.
–Q \ Loss in gravitational potential energy + loss in
+Q electrostatic potential energy = gain in kinetic
C energy ...... (1)
Now, loss in gravitational potential energy = mgR
and loss in electrostatic potential energy
So, + 2Q charge appears on the outer surface. r+ R
r r r æ ldx ö ql æ Rö
4. (a, c) t = p´E =q ò çè 2pe x ÷ø = 2pe ln çè 1 + r ÷ø
0 0
r

p If v' and v be the velocities attained by the ring Q


and particle P, when the string becomes vertical,

E 1 1
r r Mv ¢ 2 + mv 2
the gain in kinetic energy =
Þ p ´ E points inside plane of paper according 2 2
to right hand thumb rule. Also, according to conservation of linear
If allowed, dipole would swing clockwise momentum,
As it swings from 90° to 30° it gains K.E. mv
\ work done by E is positive. Mv' = mv or v¢ =
M
When field is orthogonal to dipole moments, PE
is taken zero. mv 2 é mù
Gain in kinetic energy =
2 êë1 + M úû
p
5. (b, c) F = QEdue to dipole = Q acting upward
4p Î0 r 3 From eq. (1),
1 Q ql æ Rö 1 æ mö
ton dipole = pE sin q = p [Q q = 90°] mgR + ln ç 1 + ÷ = mv 2 ç 1 + ÷
4p Î0 r 2 2pe0 è rø 2 è Mø
r r r Solving we get,
The direction of t is clockwise using t = p ´ E
-1 1/ 2
6. (a, b, d) Equivalent circuit éæ mö æ ql æ Röö ù
v = êç1 + ÷ ç 2 gR + ln ç 1 + ÷ ÷ ú
C êëè M ø è pe 0 m è r ø ø úû

C mv 2
C K2 T = mg +
R+r
C 8. (a, b, c) We shall write the condition of the equality to
zero of the potential of the sphere, and hence of
K1 any point inside it (in particular, its centre), by the
V
moment of time t. We shall single out three time
intervals :
1
When K1 is closed, energy stored = CV 2
2 a a b b
(1) t < , (2) £ t < , (3) t ³ .
When K2 is also closed, v v v v
ELECTROSTATICS 487

Denoting the charge of the sphere q (t), we obtain 9. (a, b) Let v1 and v2 be the velocities of the first and
the following expression for an instant t from the second balls after the removal of the uniform
q1 q2 q (t ) electric field. By hypothesis, the angle between
first time interval : + + = 0, the velocity v1 and the initial velocity v is 60°.
a b vt Therefore, the change in the momentum of the
æ q1 q2 ö first ball is
whence q (t ) = - v ç + ÷ t
èa bø Dp1 = q1 E Dt = m1v sin 60° .
Here we use the condition that v1 = v/2, which
æq q ö implies that the change in the momentum Dp1 of
I1 (t ) = -v ç 1 + 2 ÷
èa bø the first ball occurs in a direction perpendicular to
For an instant t from the second time interval, we the direction of its velocity v1.
find that the fields inside and outside the sphere Since E || Dp1 and the direction of variation of the
are independent, and hence second ball momentum is parallel to the direction
of Dp1, we obtain for the velocity of the second
q (t ) + q1 q q
= - 2 , I 2 (t ) = - v 2 ball (it can easily be seen that the charges on the
vt b b balls have the same sign)
Finally, as soon as the sphere absorbs the two
v
point charges q1 and q2, the current will stop v2 = v tan 30° =
flowing through the “earthing” conductor, and 3
we can write I3 (t) = 0. Thus, The corresponding change in the momentum of
the second ball is
ì æ q1 q2 ö a
ï -v çè a + b ÷ø , t< m2 v
v Dp2 = q2 E Dt =
ï cos 30°
ï q2 a b
I (t ) = í - v b , v
£t<
v q1 m1 sin 60°
ï Hence we obtain, q = m / cos 30° ,
ï b 2 2
ï 0 , t³
î v q2 4 q1 4
= = k1
m2 3 m1 3

1. A-p; B-q; C-r; D-q


2. A-p, s; B-p, s; C-q, s; D-p, s 1 Q 2 (d0 - vt ) (d 0 + vt ) 1 Q 2 [d0 + vt ]2
uf = 2 ´ + ´
3. A-q; B-r, s; C-p, r; D-q 4d02 ´ e0 A 2 4d02
4. A-s; B-q; C-p; D-r
Let q 1 and q 2 be the instantaneous charge on the (d0 - vt )
´
capacitors. Since they are in parallel. e0 A
q1 q2 e0 A e A
\ = and C1 = , C2 = 0
C1 C2 d 0 + vt d 0 - vt Q2 (d 2 - v2t 2 )
=
2e 0 A 2 d 0
q1 C1 d0 - vt
\ = = Þ q1 + q2 = Q
q2 C2 d 0 + vt Q 2 d0
ui = ´2
é d - vt ù 2e 0 A
Þ q2 ê 0 ú + q2 = Q
ë d0 + vt û
uf d 2 - v 2t 2
\ = 0 is < 1
Q [d 0 + vt ] Q [d 0 - vt ] ui d02
\ q2 = and q1 =
2d 0 2d 0
\ u reduces.
dq1 dq2 Qv 5. A-s; B-p; C-r; D-s
i= - or i.e., i = (constant) 6. A-r; B-q; C-s; D-p
dt dt 2d 0
488 IIT-JEE PHYSICS Challenger
7. A-s, t; B-q, r, t; C-p, r; D-q
Q2
(A) At constant voltage removal of dielectric decreases (Using U = , C = 4p Î0 a)
capacity, 2C
\ Charge decreases. V = Ed, E remains constant. (B) Electrostatic potential energy
(B) Insertion of dielectric increases capacity hence charge é ( -Q ) ´ ( -Q )
1 ( -Q ) 2 ù 3 Q 2
and energy increases at constant voltage. V = Ed, E = ê + ú=
remain constant 4p Î0 ëê 5a / 2 2 (5a / 2) ûú 20 p Î0 a
(C) At constant Q removal of dielectric decreases capacity
(C) Electrostatic potential energy
hence voltage increases, and hence energy increases.
(D) Insertion increases capacity.
1 3Q 2 3 Q2
8. A-p, s; B-p, r; C-p, s; D-q, s = =
4p Î0 5a 20 p Î0 a
(A) Due to q1 distribution of charge on inner surface of
conductor is uniform. Due to q2 distribution of charge
(D) Electrostatic potential energy
on outer surface of conductor is non-uniform.
(B) Due to q1 distribution of charge on inner surface of 1 é 3Q 2 (-Q)2 (-Q) ´ (-Q) ù 27Q 2
conductor is uniform. Due to q2 distribution of charge = ê + + ú=
4p Î0 êë 5a 2(2a ) 2a úû 80p Î0 a
on outer surface of conductor is uniform.
(C) Due to q1 distribution of charge on inner surface of
10. A-q, r; B-q, s, t; C-q, s, t; D-q, s, t
conductor is uniform. Due to q2 distribution of charge
on outer surface of conductor is non-uniform. 1
Energy density = Î0 E 2 . E is uniform between
(D) If the inside charge is displaced by small amount from 2
centre then symmetry will be distorted hence plates of parallel plate capacitor.
distribution due to q1 will not be uniform.
1
9. A-p, s; B-q, s; C-q, s; D-s, t For spherical shell E µ 2 .
r
(A) Electrostatic potential energy
1
Energy density µ 4
1 ( -Q ) 2 Q2 r
= =
4p Î0 2a 8p Î0 a

1. 100 1 dq
Consider a small element of length dl of the wire with centre dE = . 2
4p Î0 R
at O, as shown in Fig. The charge on this element is
From symmetry, net field at O is
q d l q ´ ( R d q ) qd q
dq = = = E = ò d Ex
pR pR p
+ + (Since the net electric field along y-axis vanishes as each
+ current element nulify the effect due to diametrically opposite
similar current element)
+

dl +p/2
1 dq cos q 1 q cos q d q

4 p Î0 -pò/ 2 p R 2
+ + + + +

dq =
R q dEx 4 p Î0 R 2
X
O
1´ q
= (sin q)p-p/ 2/ 2
dEy 4 p 2 Î0 R 2
dE
+

+ 2q 2 ´ 7 ´ 10 -10 ´ 9 ´ 109
+ + E= =
4 p Î0 R
2 2 22
(0.2) 2
The electric field due to the current element at O is 7
E = 100 V/m
ELECTROSTATICS 489

2. 15 6. 9
q1 = 10 ´ 50 = 500 m C , C1 = 10m F , C2 = ? , q2 = 0 Potential difference between A and B = 6 volts. The
q1 + q2 condensers 2 mF and 5 mF are in parallel. Their effective
As V =
C1 + C 2 capacitance, C = 2 + 5 = 7 mF.
The capacitance between A and B is given by
q + q2 500 + 0
\ C1 + C2 = 1 = = 25 m F
V 20 C ´ 3 7 ´ 3 21
C' = = = mF
C2 = 25 - C1 = 25 - 10 = 15 μ F C + 3 7 + 3 10
3. 5
The capacity of a parallel plate capacitor in air is given by 21 63
Total charge Q = CV = ´ 6 = mC
e0 A 10 5
C= ...(1)
d Total potential difference across 3 mF is
By introducing a slab of thickness t, the new capacitance C´
becomes Q 63 1 21
V1 = = ´ = volts
e0 A 3 5 3 5
C'= ...(2)
d '- t (1 - 1/ K)
Hence the common potential difference across the
The charge (Q = CV) remains the same in both the cases.
condensers in parallel is
Hence
eo A eoA æ 1ö 21 9
= or d = d '- t ç 1 - ÷ V2 = 6 - = V
d d '- t (1 - 1/ K) è Kø 5 5
d ' = d + 2.4 ´ 10-3 m, t = 3 mm = 3 × 10–3 m. So, the charge on 5 µF condenser is
Substituting these values, we have
9
-3 1ö -3 æ Q = 5 ´ V2 = 5 ´ = 9 μC
d = d + (2.4 ´10 ) - 3 ´10 ç1 - ÷ 5
è Kø
7. 0.9
-3 -3 æ 1ö
or (2.4 ´10 ) = 3 ´ 10 ç1 - ÷ m = 80 × 10–3 g = 80 × 10–6 kg,
è K ø
q = 2 × 10–8 C;
Solving it, we get K = 5.
4. 4 E = 20,000 V/m
Here, r1 = 10 cm, r2 = 15cm T sin q = qE ... (i)
V1 = 150 V, V2 = 100 V
Common potential T cos q = mg ... (ii)
C1V1 + C 2V2 4 p Î0 ( r1 V1 + r2 V2 )
V = = = 120 volt.
C1 + C 2 4 p Î0 ( r1 + r2 )
q
10-1 T T cos q
q1 = C1V = 4 p Î0 r1 V = ´ 12 C
9 ´ 10 9
q
12 qE
= ´ 3 ´ 109 esu = 4esu T sin q
9 ´ 109
5. 25 mg
The arrangement is equivalent to three capacitors in parallel
e o A / 4 10
C1 = = = 2.5 mF ;
d 4 qE 2 ´ 10-8 ´ 2 ´ 104
\ tan q = ´ = 0.5
K eo A / 2 10 mg 8 ´ 10-6 ´ 9.8
C2 = = 4 ´ = 20 mF ;
d 2 \ q = 27° 2'
e o A / 4 10
C3 = = = 2.5 mF Substituting the value of q in (i), we get
d 2
T sin (27° 2') = 4 × 10–4
\ C p = C1 + C2 + C3 + = 2.5 + 20 + 2.5 = 25μF
\ T = 9 × 10–4 N = 0.9 mN.
490 IIT-JEE PHYSICS Challenger
8. 0.63 9. 0.6
r = 1m Situation 1 : Total Energy = Energy in A + Energy in B
Q = 10–5 C 1 1
m = 0.9 × 10–3 kg = mass of particle =
CV 2 + CV 2 = CV 2
2 2
q = – 10–6 C = charge on particle.
Situation 2 : Total Energy = Energy in A + Energy in B
We know that the electric field due a uniformly. Charged
ring of radius r at a point distant x from its center on its axis 1 3
Energy in A = (KC )V 2 = CV 2
is given by 2 2
Q
+ +
+ 1 q 2 1 (CV 2 ) CV 2
+
+ r Energy in B = = ´ =
+ 2 KC 2 KC 6
+ +
+ 3 1 5
+ x \ Total Energy = CV 2 + CV 2 = CV 2 ... (ii)
+ + –q 2 6 3
+
CV 2
+
+ Total Energy initially 3
= = = 0.6
+
+ 5
+
+ Total Energy finally 2 5
Fixed CV
3
Ring
Qx 10. 8.48
E= k Total energy of the system of three charges when the charge
(r 2 + x2 )3/ 2 – q is at C = P.E. + K.E.
\ Force on the negative charge q will be F = qE
é Kq ´ q K(q)(- q) Kq(- q) ù
= ê + + +4
5 úû
- kQq ... (i)
\ F= ´x ë 6 5
(r 2 + x2 )3 / 2
2 +q
2 +3 5m
-k Q q x
Þ mA = ´x 3m
(r 2 + x2 )3 / 2 B x
A
4m -q
3m
Öx 2 5m
Qq +3 2
Þ A=–k ´x +q
m (r + x )
2 2 3/ 2
Final energy of the system of three charges when – q is at D
Þ The motion is simple harmonic in nature. Comparing it and momentarily at rest
will A = – w2 x = P.E. + K.E.
kQq
We get w2 = é Kq ´ q Kq ( - q ) Kq ( - q ) ù
2
m (r + x 2 )3 / 2 =ê + + ú
êë 6 x +3
2 2
x 2 + 32 úû
In this case x < < r
kQq Kq ´ q 2Kq (- q )
\ w2 = = + ... (ii)
mr 3 6 x 2 + 32
By the principle of conservation of energy from (i) and (ii),
kQq
Þ w= we get
mr 3
kq ´ q 2kq (- q ) kq ´ q 2kq(- q)
+ +4= +
2p kQq 6 5 6 x 2 + 32
\ =
T mr 3
é1 1 ù
2 = kq2 ê - ú
mr 3 êë 5 x 2 + 32 úû
T = 2p
kQq
2 1 1
= -5 -5
= -
é 0.9 ´ 10-3 ´ 13 ù
1/ 2 9 ´ 10 ´ 5 ´ 10
9
´ 5 ´ 10 5 x 2 + 32
T = 2 × 3.14 ê -5 -6
ú
êë 9 ´ 10 ´ 10 ´ 10 úû
9
20 1 1
= = -
= 6.28 [0.01]1/2 = 6.28 [0.1] 9´5´5 5 x + 32
2

T = 0.628 sec ; 0.63 sec


ELECTROSTATICS 491

12. 4.42
1 1 4 9-4 5 1
= = - = = = The adjacent figure is a case of parallel plate capacitor. The
x 2 + 32 5 45 45 45 9 combined capacitance will be
\ x2 + 9 = 81
V
\ x = 8.48 m
11. 3.19 + -

Each mass will be in equilibrium under the act of three forces +


namely tension of string, weight, resultant electrostatic force 1-x
of the two other charges. Out of these three forces, F and
mg are perpendicular. x 1m
d

M C = C1 + C2
M
q ke 0 ( x ´ 1) e 0 [(1 - x) ´ 1)
+
1m

q =
A B d d
O O
m Tcosq
m 60
T 60
q T
O e0
0.03 nq C= [ dx + 1 - x] ... (i)
C C Tsi d
F After time dt, the dielectric rises by dx. The new equivalent
FCB mg
FCA capacitance will be
F
C + dC = C1' + C2'
Let T make an angle q with the vertical e0 e [(1 - x - dx) ´ 1]
= [( x + dx ) ´ 1] + 0
d d
2
OC = (0.03) 2 - (0.015)2 = 0.0173 m
3 e0
= [ kx + kdx + 1 - x - dx ] ... (ii)
\ OM = 0.9997 d
Resolving T in the direction of mg and F and applying the Charge of capacitance in time dt
condition of equilibrium, we get e0
dC = [kx + kdx + 1 – x – dx – kx – 1 + x]
T cos q = mg; T sin q = F d
F e0
\ tan q = ... (i) = (k – 1) dx
mg d
dC e 0 dx e0
F= 2
FCA + FCB
2
+ 2 FCA FCB cos a = (k - 1) = (k - 1)v ... (iii)
dt d dt d
1 dx
\ F= 2
FCA + FCA
2
+ 2 FCA
2
´ where v=
2 dt
We know that q = CV
kq 2
F= 3FCA = 3 ´ ... (ii) dq dC
(CA) 2 =V ... (iv)
dt dt
[where FCB = Force on C due to B
e0
FCA = Force on C due to A Þ I= V (k - 1)v
ur ur d
| F CB | = | F CA | and a = 60° ] From (i) and (ii)

OC 0.0173 500 ´ 8.85 ´ 1012


Also, tan q = = .... (iii) I= (11 – 1) × 0.001= 4.425 × 10–9 A
OM 0.9997 0.01
From (i), (ii) and (iii) = 4.42 nA
Alternatively :
0.0173 3 ´ 9 ´ 109 ´ q 2 We can differentiate eq. (1) w.r.t 't ', we get
=
0.9997 (0.03) 2 ´ 10-3 ´ 9.8 dC e 0 dx
= (K - 1) and then proceed further..
On solving, we get q = 3.19 × 10–9 C = 3.19 nC. dt d dt
492 IIT-JEE PHYSICS Challenger
13. 3
æ 3ö 27
Let the particle be, at some instant, at a point P distant x Þ 4 ç x2 + ÷ = x2 +
è 2ø 2
from the origin. As shown in the figure, there are two forces
of repulsion acting due to two charges of + 8 mC. The net 27 12
force is 2F cos a towards right. Þ 3x2 = -
2 2
Similarly, there are two forces of attraction due to two charges
of – 1 mC. The net force due to these force is 2F' cos b 5
towards left. Þ x=±
2
This means that we need to move the charge from – ¥ to
27 +8µC
+
2 5
. Thereafter the attractive forces will make the charge
3
–1µC 2
+ F' F
2 move to origin. Thereafter the attractive forces will make the
+0.1µC Fcosa
F'cosb b charge move to origin.
O X
F'cosb
x P a Fcosa
3 F 5
- F' The electric potential of the four charges at x = is
2 –1µC 2
27
-
2 +8µC 2 ´ 9 ´ 109 ´ 8 ´ 10 -6 2 ´ 9 ´ 109 ´ 10 -6
V= -
5 27 5 3
+ +
The net force on charge 0.1 µC is zero when 2 2 2 2
2F cos a = 2F' cos b
é8 1ù
= 2 × 9 × 109 × 10–6 ê - ú = 2.7 ´ 10 V
4
K ´ 8 ´ 10 -6 ´ 0.1 ´ 10-6 x ë 4 2 û
Þ ´
27 27
x +
2
x2 + Kinetic energy is required to overcome the force of repulsion
2 2
5
from ¥ to x = .
-6 -6 2
K ´ 1 ´ 10 ´ 0.1 ´ 10 x
= ´
3 3 The work done in this process is W = q (V)
x2 + x2 +
2 2 5
where V = p.d between ¥ and x = .
2
Þ 8 1
3/ 2
= 3/ 2 \ W = 0.1 × 10–6 × 2.7 × 104 = 2.7 × 10–3 J
é 2 27 ù é 2 3ù
êë x + 2 úû êë x + 2 úû 1
By energy conservation mv02 = 2.7 × 10–3
2
3/ 2 3/ 2
ìï 3 é 2 3 ù 3 / 2 üï ìï é 27 ù üï
3/ 2 1
Þ Þ ´ 6 ´ 10 -4 v02 = 2.7 ´ 10 -3 Þ v0 = 3 m/s
í2 ê x + ú ý = íê x2 + ú ý 2
ïî ë 2û ï
þ ïî ë 2û ï
þ
1. In the circuit shown in the given figure the resistances R1
b
and R2 are respectively
R1
0.5 A 220 V.
20 c
10

R2 V

1A a
(a) 32 V (b) 36 V
69 V (c) 40 V (d) 42 V
(a) 14 and 40 (b) 40 and 14
4. What is the voltage across resistor A in the following circuit?
(c) 40 and 30 (d) 14 and 30 .
Each resistor has a resistance of 2 M and the capacitors
2. A copper wire and an iron wire, each having an area of
have capacitances of 1 F. The battery voltage is 3V.
cross-section A and lengths L1 and L2 are joined end to end.
The copper end is maintained at a potential V1 and the iron
end at a lower potential V2. If 1 and 2 are the conductivities
of copper and iron respectively, then the potential of the
junction will be
1V1 2V2
(a)
( 1 / L1 ) ( 2 / L2 )

1V1 2V2 (a) 0 V (b) 0.5 V


L1 L2 (c) 0.75 V (d) 1.5 V
(b) 5. Calculate the drift velocity of electrons in silver wire with
( 1 / L1 ) ( 2 / L2 )
cross-sectional area 3.14 × 10–6 m2 carrying a current of 20
( A. Given atomic weight of Ag = 108, density of silver
1 / L1 ) ( 2 / L2 )
(c) = 10.5 × 103 kg/m3.
1V1 2V2 (a) 2.798 × 10–4 m/sec. (b) 67.98 × 10–4 m/sec.
(c) 0.67 × 10–4 m/sec. (d) 6.798 × 10–4 m/sec.
1V1 2V2
(d) 6. All the edges of a block with parallel faces are unequal. Its
( 1 / L1 ) ( 2 / L2 ) longest edge is twice its shortest edge. The ratio of the
3. A potential difference of 220 V is maintained across a 12000 maximum to minimum resistance between parallel faces is
ohm rheostat, as shown in the figure. The voltmeter has a (a) 2
resistance of 6000 ohm and point c is at one-fourth of the (b) 4
distance from a to b. Therefore, the reading of the voltmeter (c) 8
will be (d) indeterminate unless the length of the third edge is
specified

MARK YOUR 1. 2. 3. 4. 5.
RESPONSE 6.
494 IIT-JEE PHYSICS Challenger
7. A wire of length L and 3 identical cells of negligible internal
resistances are connected in series. Due to the current, the 12
temperature of the wire is raised by T in a time t. A number
N of similar cells is now connected in series with a wire of 7 4
the same material and cross-section but of length 2L. the R
temperature of the wire is raised by the same amount T in
the same time t. The value of N is
(a) 4 (b) 3
(c) 8 (d) 9 24 V
8. Two resistances equal at 0° C with temperature coefficient
of resistance 1 and 2 joined in series act as a single (a) 12.5 (b) 15.5
resistance in a circuit. The temperature coefficient of their (c) 17.5 (d) 19.5
single resistance will be :
12. A moving coil galvanometer of resistance 100 is used as
(a) (b) 1 2 an ammeter using a resistance 0.1 . The maximum deflection
1 2
1 2 current in the galvanometer is 100 A. Find the minimum
1 2 1 2 current in the circuit so that the ammeter shows maximum
(c) (d)
2 2 deflection
9. The current density varies with radial distance r as J = a r2, (a) 100.1 mA (b) 1000.1 mA
in a cylindrical wire of radius R. The current passing through
(c) 10.01 mA (d) 1.01 mA
the wire between radial distance R/3 and R/2 is :
13. Calculate the relaxation time and mean free path in Cu at
65 a R 4 25 a R 4 room temperature 300 K, if number density of free electrons
(a) (b)
2592 72 is 8.5 × 1028 /m³ and resistivity = 1.7 × 10–8 mho-m.
65 a 2 R 3 81 a 2 R 4 Given k = 1.38 × 10–23 J/K.
(c) (d)
2938 144 (a) 25 Å (b) 20 Å
10. A simple ohm meter is made by connecting a 1.5 V battery in (c) 5 Å (d) 30 Å
series with a resistance R and a ammeter that reads from 0–
14. In the diagram shown, all the wires have resistance R. The
1.0mA, as shown below. The resistance R is adjusted so that
equivalent resistance between the upper and lower dots
when the leads A and B shortened together, the meter reads 1
shown in the diagram is :
mA. What external resistance across A, B results in (1) 10% (2)
50% (3) 90% of the full scale deflection ?

mA

B (a) R/8 (b) R


R (c) 2R/5 (d) 3R/8
15. The charge flowing through a resistance R varies with time
(a) (1) 17,500 (2) 9500 (3) 365
(b) (1) 15000 (2) 7500 (3) 1500 t as Q = at – bt2. The total heat produced in R by the time
(c) (1) 13500 (2) 1500 (3) 167 current ceases is
(d) (1) 11, 500 (2) 1300 W(3) 65 (a) a3R/6b (b) a3R/3b
11. In the circuit shown, for what value of R, will the ideal battery
a3 R a3 R
transfer energy at the rate of 60 W ? (c) (d)
2b b

MARK YOUR 7. 8. 9. 10. 11.


RESPONSE 12. 13. 14. 15.
CURRENT ELECTRICITY 495

16. A wire has linear resistance (in Ohm/m). Find the resistance
R between points A and B if the side of the larger square is 20. Ends of two wires A and B having resistivity A
3 10 –5
'd'.
A m and B
6 10 –5 m of same cross section area are
joined together to form a single wire. If the resistance of the
joined wire does not change with temperature, then find the
ratio of their lengths, given that temperature coefficient of

B resistivity of wire A and B is A 4 10 –5 / °C and


(a) d/ 2 (b) 2 d
B –6 10–6 / °C. Assume that mechanical dimensions
(c) 2 d (d) None of these do not change with temperature.
17. A potentiometer wire AB as shown is 40 cm long of
resistance 50 /m, free end of an ideal voltmeter is 3 10
(a) (b)
touching the potentiometer wire. What should be the 7 3
velocity of the jockey as a function of time so that reading 3 1
(c) (d)
in voltmeter is varying with time as (2 sin t). 10 2
10 Q 10 21. A resistance coil, wired to an external battery is placed inside
a thermally insulated cylinder fitted with a frictionless piston
V and containing an ideal gas. A current I = 240 mA flows
A B through the coil which has resistance R= 490 . At what
constant speed v must the piston of mass m = 12 kg move
4V upward in order that the temperature of the gas remains
unchanged? [Assume that 40% energy is utilised to do work
(a) 10 sin t cm/s (b) 10 cos t cm/s
by the gas, and neglect the work done by atmospheric
(c) 20 sin t cm/s (d) 20 cos t cm/s
pressure].
18. For the shown circuit the effective resistance between the v
points A and B will be
2R m
Gas
R R R g
R/2 R/2
2R 2R
R R
A B
4R (a) 0.141 m/s (b) 1.41 m/s
R R R R (c) 0.24 m/s (d) 2.4 m/s
22. In the circuit shown in the figure, find the current in 45 .
2R
(a) 2 R (b) 4 R
(c) R (d) R/2
180V 90 45
19. An n sided regular polygon is inscribed in a circular region
of radius R using a wire of cross sectional radius r and 90
resistivity . If the magnetic field in the circular region has
a time rate of change given by B then the current in the wire
100 100
loop at any time is given by 50 50
2 2
RB r cos( / n) RB r cos( / n)
(a) (b)
2 2 100 50
(a) 4 A (b) 2.5A
RB r 2 sin( / n) RB r 2 cos( / n) (c) 2 A (d) none
(c) (d)
2 3

MARK YOUR 16. 17. 18. 19. 20.


RESPONSE 21. 22.
496 IIT-JEE PHYSICS Challenger
23. Figure shows a 2.0 V potentiometer used for the (a) decreasing the resistance R and decreasing V
determination of internal resistance of a 1.5 V cell. The (b) decreasing the resistance R and increasing V
balance point of the cell in open circuit is 76.3 cm. When a (c) increasing the resistance R and increasing V
resistor of 9.5 is used in the external circuit of the cell, the (d) increasing the resistance R and decreasing V.
balance point shifts to 64.8 cm length of the potentiometer 26. A meter bridge with two resistances connected in the two
wire. The internal resistance of the cell is approximately
gaps has a balance point at 40 cm. If a resistance 10 is
connected in series with the smaller resistor, the balance
2.0 V
point shifts by 20 cm. Neglecting any corrections, the larger
resistance is
A B
(a) 8 (b) 40
1.5 V h
h (c) 60 (d) 12
27. For maximum power from battery the internal resistance of
battery r is
9.5
(a) 1.7 (b) 1.5
(c) 1.2 (d) None of these R R
24. The incorrect option in the given circuit if the reading of R
ammeter is zero is
R R
R R
1 r
R R
R r

2R
A
4R
(a) 10 R (b)
9
(R + r)
(a) The value of 1 will be R R 10 R
(c) (d)
8 9
1
(b) Current in R is
r+R 28. As shown in circuit diagram an ideal voltmeter and an ideal
(c) Value of 1 will be ammeter reading are zero. Sum of potential drop across box
(d) Potential across 2R is zero. and 5 resistances will be (given that there are no fault in
25. A potentiometer circuit shown in the figure is set up to voltmeter and ammeter)
measure emf of cell E. As the point P moves from X to Y the
galvanometer G shows deflection always in one direction, Box
but the deflection decrease continuously until Y is reached.
The balance point between X and Y may be obtained by 7 V
12 Volt
V R 5
A

X P Y (a) 7 volt
(b) 5 volt
(c) 12 volt
G (d) Data insufficient
E

MARK YOUR 23. 24. 25. 26. 27.


RESPONSE 28.
CURRENT ELECTRICITY 497

29. In the diagrams, all light bulbs are identical, all cells are ideal
and identical. In which circuit (a, b, c, d) will the bulbs be
dimmest ? R1 Rx
A A
C D
(a)

(a) (b) R1
A

C D
Rx
A
(c) (d) (b)

R1 n2 n1 2R1n2 n1
30. A hank of uninsulated wire consisting of seven and a half (a) (b) n1 n2
n1 n2
turns is stretched between two nails hammered into a board
to which the ends of the wire are fixed. The resistance of the R1 n2 n1 3 R1 n2 n1
circuit between the nails is determined with the help of (c) 2 n1 n2 (d) 2 n1n2
electrical measuring instruments. Determine the proportion
in which the resistance will change if the wire is unwound 33. Determine the resistance RAB between points A and B of the
so that the ends remain to be fixed to the nails. frame formed by nine identical wires of resistance R each
(a) 225 (b) 15 (figure).
(c) 240 (d) 250
31. The diagram below shows a junction with currents labeled R R B
I1 to I6. Which of the following statements is correct?
R R R R R

I3 A R R

2 9
(a) R (b) R
11 11
I6 I4
15 1
(c) R (d) R
11 11
34. For the circuit shown, a shorting wire of negligible resistance
I1 I5 I2 is added to the circuit between points A and B. When this
(a) I1 + I3 = I6 + I4 (b) I1 + I2 = I6 + I4 shorting wire is added, bulb-3 goes out. Which bulbs (all
(c) I4 + I3 = I6 (d) I2 = I6 + I4 identical) in the circuit brighten ?
32. Given two different ammeters in which the deflections of
the pointers are proportional to current, and the scales are 1 A
uniform. The first ammeter is connected to a resistor of
resistance R1 and the second to a resistor of unknown 3
resistance Rx. At first the ammeters are connected in series 2 B
between points C and D (as shown in figure a ). In this case,
4
the readings of the ammeters are n1 and n2 . Then the
ammeters are connected in parallel between C and D (as
shown in figure b) and indicates n1' and n2'. Determine the (a) only bulb 2 (b) only bulb 4
unknown resistance Rx of the second resistor.
(c) only bulbs 1 and 4 (d) only bulbs 2 and 4

MARK YOUR 29. 30. 31. 32. 33.


RESPONSE 34.
498 IIT-JEE PHYSICS Challenger
35. A perfect voltmeter and a perfect ammeter are connected in (a) 1/9 (b) 2/9
turn between points E and F of a circuit whose diagram is (c) 1/4 (d) 10/9
shown in figure. The readings of the instruments are V0 and 38. Which one of the following modifications may increase the
I0. Determine the current I through the resistor of resistance
sensitivity of moving coil galvanometer ?
R connected between points E and F.
1st Way : By using spring of smaller torsion constant.
A 2nd Way : By using a smaller coil.
3rd Way : By using a stronger magnet.
4th Way : By using a coil having fewer number of turns
F (a) 1st and 4th ways only
(b) 1st and 3rd ways only
E
(c) 1st, 2nd and 3rd ways only
(d) 2nd and 4th ways only
39. A circuit is connected as shown in the figure with the switch
B S open. When the switch is closed, the total amount of
charge that flows from Y to X is
V0 I 0 V0 I 0 3 F 6 F
(a) (b) X
RI 0 + V0 RI 0 V0

2V0 I 0 V0 I0
(c) (d) S
RI 0 + V0 2RI0 + V0
36. Five identical resistors (coils for hot plates) are connected 3 6
as shown in the diagram. What will be the change in the Y
voltage across the right upper spiral upon closing the key
K? 9V
(a) 0 (b) 54 C
(c) 27 C (d) 81 C
40. The resistance RAB between points A and B of the frame
K made of thin homogeneous wire (figure), assuming that the
number of successively embedded equilateral triangles (with
sides decreasing by half) tends to infinity, if side AB is equal
to a, and the resistance of unit length of the wire is
~
(a) Increases (b) Decreases
(c) no change (d) Nothing can be said
37. The voltage across a load is controlled by using the circuit
diagram shown in figure. The resistance of the load and of
the potentiometer is R. The load is connected to the middle
of the potentiometer. The input voltage is constant and equal
to V. The change in the voltage across the load if its A B
resistance is doubled is
a

a ( 7 1) a ( 7 + 1)
(a) (b)
3 3
V R R
2a ( 7 + 1)
(c) (d) a ( 7 + 1)
3

MARK YOUR 35. 36. 37. 38. 39.


RESPONSE 40.
CURRENT ELECTRICITY 499

41. A concealed circuit (black box) consisting of resistors has


four terminals (figure). If a voltage is applied between clamps C
1 and 2 when clamps 3 and 4 are disconnected, the power
liberated is N1 = 40W, and when the clamps 3 and 4 are
R R
connected, the power liberated is N2 = 80W. If the same R
source is connected to the clamps 3 and 4, the power
liberated in the circuit when the clamps 1 and 2 are R R
disconnected is N3 = 20W. The power N4 consumed in the B
A
circuit when the clamps 1 and 2 are connected and the same V
voltage is applied between the clamps 3 and 4 is R
R
R
1 3
2 4
D

(a) 40W (b) 20W (a) 5V/7R (b) 5V/2R


(c) 10W (d) 60W (c) 10V/7R (d) 15V/7R
44. Two wires of the same length but of different square cross-
42. If the switch at point P is opened (shown in the figure),
sections are made from the same material. The sides of the
choose the correct option
cross sections of the first and second wires are d1 = 1 mm
P and d2 = 4 mm. The current required to fuse the wire is I1 =
R2
10A. Determine the current I2 required to fuse the second
wire, assuming that the amount of heat dissipated to the
X ambient per second obeys the law Q = kS (T – Tam), where
S is the cross-sectional area of the wire, T is its temperature,
R3 Tam is the temperature of the ambient away from the wire,
and k is the proportionality factor which is the same for the
R1 R4 two wires.
(a) 40A (b) 80A
E (c) 90A (d) 60A
45. A 1µF capacitor is connected in the circuit shown below.
The emf of the cell is 3 volts and internal resistance is
(a) the current in R1 would not change 0.5ohms. The resistors R1 and R2 have values 4 ohms and 1
(b) the potential difference between point X and the ground ohm respectively. The charge on the capacitor in steady
would increase state must be
3V 0.5
(c) the current provided by the battery would increase
(d) the emf produced by the battery (assumed to have no 1µF R1
internal resistance) would change
4
43. The resistance of each resistor in the circuit diagram shown
in figure is the same and equal to R. The voltage across the R2
terminals is V. The current I in the leads if their resistance 1
can be neglected is (a) 1 µC (b) 2 µC
(c) 1.33 µC (d) zero

MARK YOUR
41. 42. 43. 44. 45.
RESPONSE
500 IIT-JEE PHYSICS Challenger
46. In the figure ammeter A1 reads a current of 10mA, while the
voltmeter reads a potential difference of 3V. What does
ammeter A2 in mA read ? The ammeters are identical, the E1 R1
internal resistance of the battery is negligible.
(Consider all ammeters and voltmeters as non-ideal.)

C K 1
100 A1 2
4V
A2
E2
100 V
R2

(a) 6.67 mA (b) 3.12 mA C ( E2 R1 E1 R2 ) C ( E2 R1 + E1 R2 )


(c) 1.12 mA (d) 5.14 mA (a) R1 + R2 (b) R1 + R2
47. All the wires on the front and the back face have resistance
R. All the wires along the length have resistance 2R. Find 2C ( E2 R1 + E1 R2 ) C ( E2 R1 + E1 R2 )
the equivalent resistance between P and Q. (c) R1 + R2 (d) 2 ( R1 + R2 )
49. Six identical resistors are connected as shown in the figure.
The equivalent resistance will be
P
R
P Q

R R
R
R R

Q R

(a) Maximum between P and R


(b) Maximum between Q and R
(c) Maximum between P and Q
23R 15R (d) all are equal
(a) (b) 50. What will be the change in the resistance of a circuit
20 20
consisting of five identical conductors if two similar
13R 3R conductors are added as shown by the dashed line in figure.
(c) (d)
20 20
48. The key K (figure) is connected in turn to each of the
contacts over short identical time intervals so that the
change in the charge on the capacitor over each connection
is small. What will be the final charge qf on the capacitor ? (a) becomes 1/5 times (b) becomes 3/5 times
(c) becomes 2/5 times (d) becomes 1/2 times

MARK YOUR
46. 47. 48. 49. 50.
RESPONSE
CURRENT ELECTRICITY 501

51. The key K in circuit diagram shown in figure can be either in 54. In a certain particle accelerator, electrons emerge in pulses
position 1 or 2. The circuit includes two d.c. sources, two at the rate of 250 pulses per second. Each pulse is of duration
resistors, and an ammeter. The emf of one sources is E1 and of 200ns and the electrons in the pulse constitute a current
of the other is unknown. The internal resistance of the of 250mA. The number of electrons delivered by the
sources should be taken as zero. The resistance of the accelerator per pulse is
(a) 8.00 × 1010 (b) 5.00 × 108
resistors is also unknown. One of the resistors has a varying 11
(c) 3.13 × 10 (d) 9.60 × 1010
resistance chosen in such a way that the current through
55. A wire frame in the form of a tetrahedron ADCB is connected
the ammeter is the same for both positions of the key. The to a d.c. source (figure). The resistances of all the edges of
current is measured and is found to be equal to I. Determine the tetrahedron are equal. Indicate the edge of the frame
the resistance denoted by Rx in the diagram. that should be eliminated to obtain the maximum change in
the current Imax in the circuit, neglecting the resistance of
1
the leads.
D
K
Rx

2 E1 A
C
I

A
B
I
(a) 2E1/I (b) E1/2I
(c) 3E1/I (d) E1/I (a) AB (b) AD
(c) AC (d) BC
52. The total momentum of electrons in a straight wire of length
56. The circuit diagram shown in figure consists of a very large
1000m carrying a current of 70A is closest to
(infinite) number of elements. The resistances of the resistors
(a) 40 × 10–8 N-sec in each subsequent element differ by a factor of k from the
(b) 30 × 10–8 N-sec resistances of the resistors in the previous elements.
(c) 50 × 10–8 N-sec Determine the resistance RAB between points A and B if the
(d) 70 × 10–8 N-sec resistances of the first element are R1 and R2. (k = 1/2)
53. The current in the circuit shown is constant when the switch R1 kR1
is closed. The energy dissipated in the internal resistance r A C
of the battery is 15J when a charge of 40C passes through it.
For the same amount of charge, 45J of energy is transferred
in the resistor R. Which of the following gives the emf of the R2 kR2
battery ?
r
B D

R1 R2 + R12 + R22 + 6 R1 R2
(a)
2

R R1 + R2 + R12 + R22 + 6 R1 R2
(b)
2
15 30
(a) V (b) V
40 40 R1 R2 R12 + R22 + 6 R1R2
(c)
2
45 60
(c) V (d) V (d) None of these
40 40

MARK YOUR 51. 52. 53. 54. 55.


RESPONSE 56.
502 IIT-JEE PHYSICS Challenger
57. The circuit shown in figure is made of a homogeneous wire
3 4
of constant cross section. Find the ratio Q12/Q34 of the
amounts of heat liberated per unit time in conductors 1-2
and 3-4.

1 2

(a) 11 + 2 2 (b) 8+6 2


(c) 11 + 5 2 (d) 11 + 6 2

MARK YOUR
57.
RESPONSE

PASSAGE-1 3. The effective conductivity when a third layer of


cross-sectional area A1 + A2 , length L and specific

Two conducting layers of length L each, specific resistivities resistivity 3 is attached at one of the ends of the given
and 2 and cross-sectional areas A1 and A2 respectively
1 system is
are connected to a voltage source V such that current flow
(a) ( 1 2) 3
lines do not cross from one layer to the other.
1 2 3x
1. The equivalent resistance of the system is

L( A1 + A2 ) L
(b)
2( 1 2 x) 3
(a) (b) A1 A2
1 2 1 2 1 3 ( 2 3 )x

L 1 2 L( 2) 2 ( 1x
(c) (d) 1
(c) 2) 3
A1 + A2 A1 A2 ( 1 3)x 2

2. The total current in the system is


2( 1 2 ) 3x A
V L (d) (where x = 2 )
(a) ( 1 A1 2 A2 ) (b) ( 1 A2 2 A1 )
( 1 2) 3x A1
L V

V V2
(c) ( 1 A1 2 A2 ) (d) ( 1 A2 2 A1 )
L2 L

MARK YOUR
1. 2. 3.
RESPONSE
CURRENT ELECTRICITY 503

7. The current through battery just after closing the switch is


PASSAGE-2
(a) 6.75 A (b) 8.75 A
(c) 2 A (d) 4 A
A circuit containing a two positions switch S is shown in figure.
8. The time after which the charge on capacitor becomes 63.2%
of maximum charge is
R3 (a) 20 s (b) 30 s
C
(c) 10 s (d) 15 s.
2 2 F 9. Maximum charge on plates of capacitor is
R5 (a) 45 C (b) 120 C
R1 E1
(c) 30 C (d) 90 C
1 2 12 V R2
A B
S PASSAGE-4
2 E2 R4
L A fixed resistor is in parallel with a variable resistor, both are
connected to a real battery (internal resistance is not
10 mH negligible). Originally the fixed and variable resistors have
the same resistances.
4. The switch S is in position 1. The potential difference 10. As the resistance of the variable resistor is decreased, the
current through the fixed resistor
VA VB in R1 is.
(a) increases
(a) – 5 V (b) + 5 V (b) decreases
(c) – 7 V (d) + 7 V (c) remains the same
5. The rate of production of joule heat in R1 is (d) cannot be determined without more information
(a) 21.5 J/s (b) 20 J/s 11. As the resistance of the variable resistor is decreased, the
(c) 24.5 J/s (d) 26.5 J/s rate at which energy is transferred to the fixed resistor
6. If now the switch S is put in position 2 at t = 0, the steady (a) increases
current in R4 is (b) decreases
(a) 0.6 A (b) 0.6 mA (c) remains the same
(c) 0.6 kA (d) 0.6 daA (d) cannot be determined without more information
12. If the resistance of the variable resistor is decreased slightly,
the rate at which energy is transferred to the variable resistor
PASSAGE-3 (a) increases
(b) decreases
In the circuit shown at t < 0 the switch S is open and (c) remains the same
capacitor is uncharged. The switch S is closed at t = 0. (d) cannot be determined without more information
Then

PASSAGE-5
B
3 6 In the connection shown in the figure initially the switch K
A C
5µF is open and the capacitor is uncharged. Then the switch is
4 4 closed and the capacitor is charged up to the steady state
S
and the switch is opened again.
D
[Given : V0 = 30V, R1 = 10k , R2 = 5k ]

36 volts

MARK YOUR 4. 5. 6. 7. 8.
RESPONSE 9. 10. 11. 12.
504 IIT-JEE PHYSICS Challenger

PASSAGE-7

C
A 6V battery of negligible internal resistance is connected
across a uniform wire AB of length 100cm. The positive
terminal of another battery of emf 4V and internal resistance
R2 1 is joined to the point A as shown in figure. Take the
R1
A potential at B to be zero.

6V

A B
V0 K D C

4V 1
13. Determine the values indicated by the ammeter just after
closing the switch 19. What are the potentials at points A and C
(a) 2mA (b) 3mA (a) 6V, 2V (b) 8V, 4V
(c) 0mA (d) none of these (c) 6V, 4V (d) 8V, 3V
14. Determine the values indicated by the ammeter a long time
20. At which point D of the wire AB, the potential is equal to the
after the switch was closed
potential at C ?
(a) 2mA (b) 3mA
(a) AD = 60 cm. (b) AD = 50 cm.
(c) 6mA (d) none of these
(c) AD = 40 cm. (d) None of these
15. Determine the values indicated by the ammeter just after
reopening the switch 21. If the points C and D are connected by a wire, what will be
(a) 2mA (b) 3mA the current through it ?
(c) 6mA (d) none of these (a) zero (b) 1A
(c) 2A (d) 3A

PASSAGE-6
PASSAGE-8
The figure shows part of a certain circuit
1 2 4 2A In the circuit shown in figure V1 and V2 are two voltmeters
C 5A 12V 3V B of resistances 3000 and 2000 respectively. In addition
5 6
R1 = 2000 , R2 = 3000 and E = 200V.
4A
16. Power dissipated in 5 resistance is E
(a) 512 W (b) 480 W (c) 120 W (d) 605 W
17. Potential difference VC – VB is
(a) 12V (b) 6V (c) 10V (d) 8V V1 V2
18. Which battery is being charged
R1 S R2
(a) 12V (b) 3V
(c) Both (a) and (b) (d) None of these

MARK YOUR 13. 14. 15. 16. 17.


RESPONSE 18. 19. 20. 21.
CURRENT ELECTRICITY 505

22. The reading of voltmeter V1 when switch S is open is 24. The reading of voltmeter V1 when switch S is closed is
(a) 120V (b) 80V (a) 120V (b) 80V
(c) 100V (d) 60V (c) 100V (d) 60V
23. The reading of voltmeter V2 when switch S is open is
(a) 120V (b) 80V
(c) 100V (d) 60V

MARK YOUR
22. 23. 24.
RESPONSE

1. Statement - 1 : Current is passed through a metallic wire, 5. Statement - 1 : When two conducting wires of different
heating it red. When cold water is poured resistivity having same cross section area
on half of its portion then rest of the half are joined in series, the electric field in them
portion becomes more hot. would be equal when they carry current.
Statement - 2 : Resistance decreases due to decrease in Statement - 2 : When wires are in series they carry equal
temperature and so current through wire current.
increases. 6. Statement - 1 : In the circuit shown,
2. Statement - 1 : A larger dry cell has higher emf. Vab or Va – Vb = 0, if
Statement - 2 : The emf of a dry cell is independent of its I = 2A.
size.
3. Statement - 1 : Ohm’s law cannot be applied to a.c
a 4V 2 b
circuit.
Statement - 2 : Resistance offered by capacitor for a.c Statement - 2 : Potential difference across the terminal of
source is dependent upon the frequency a non ideal battery is less than its emf when
of the source. a current flows through it.
4. Statement - 1 : In a simple battery circuit, the point of the 7. Statement - 1 : In case of potentiometer experiment
lowest potential is negative terminal of the (without rheostat) if emfs of known and
battery. unknown both batteries (ideal) are made
Statement - 2 : The current flows towards the point of the two times, null point length will remain
higher potential, as it does in such a circuit unchanged.
from the negative to the positive terminal. Statement - 2 : Null point length does not depend on emf
of either of the two batteries.

MARK YOUR 1. 2. 3. 4. 5.
RESPONSE 6. 7.
506 IIT-JEE PHYSICS Challenger

1. A cable AB of length L of uniform wire develops a leak at a (a) The reffective resistance between E and F is R/2
certain point. The resistance between A and the earth through (b) The effective resistance between A and C is R/2
the cable when B is earthed is found to be that of a length a (c) The effective resistance between A and D is R/2
of the wire and that between B and the earth, when A is (d) If a battery is connected between D and F then the
earthed that of length b. potential of points A and C will be same
3. The network shown in the figure is part of some bigger
C circuit. Charge on capacitor (C = 1 F) at any time t
A B is q = 3(1 – e –t ) in coulombs. If at t = 0 the potentials
x L–x
of points A and B is 20 V, then at t = 0 choose the
r following

20V A
C i +q
A B C = 1F
x L–x –q
1
r B 1 R
20V 2V P
Earth
(a) The point at which the fault exists divides AB in the
1/ 2 (a) current in branch AP is 3 A
ratio [a( L b) / b( L a)] (b) current through R is 7 A
[ a (1 ) L] (c) value of R is 2
(b) the resistance of the leak is r L 2 (d) value of R is 1
(1 ) ( L a) 4. A charged particle having a positive charge q approaches a
grounded metallic sphere of radius R with a constant speed v
[a L]
(c) the resistance of the leak is r L as shown in the figure. Now choose the correct alternative (s).
2
(1 ) ( L a)
(d) the point at which the fault exists divides AB in the
a(L b) q
ratio (where x/ L x ) v
R
(L a)2
2. Consider 12 resistors arranged symmetrically in shape of
bipyramid ABCDEF. The resistance of each resistor is R.

E C (a) As the charge draws nearer to the surface of the sphere,


a current flows into the ground.
(b) As the charge draws nearer to the surface of the sphere,
R a current flows out of the ground into the sphere.
B
D (c) As the charged particle draws nearer, the magnitude of
current flowing in the connector joining the shell to
F the ground increases.
(d) As the charged particle draws nearer, the magnitude of
current flowing in the connector joining the sphere to
A the ground decreases.

MARK YOUR
1. 2. 3. 4.
RESPONSE
CURRENT ELECTRICITY 507

5. The potential difference across a 2H inductor as a function (c) 2.5 mA range with 2.5 resistance in parallel
of time is shown in the figure. At time t = 0, current is zero. (d) 2.5 mA range with 4 resistance in parallel
Choose the correct statement 8. The adjacent figure shows a resistance network with value
of each resistance mentioned. Choose the correct options
V (volt)

A
10

R R
R
0 2 4 t (sec) R B R
C D
R/2
(a) Current at t = 2 sec is 5A R/2 R/2
(b) Current at t = 2 sec is 10 A E
(c) Current vs time graph across inductor will be

iL (a) RCD = R/2 (b) RAD = (9/5) R


(c) RAC = (5/9) R (d) RAD = (5/9) R
9. A circuit consists of a current source of emf E and internal
resistance r, capacitors of capacitance C1 and C2, and
resistors of resistance R1 and R2. (Figure). Choose the
t
correct options
(d) Current vs time graph across inductor will be

C1 R1 C2
ic

t R2

6. In experiments using the metre bridge, post-office box, and E, r


potentiometer, a galvanometer is used. Which property of
the galvanometer makes it suitable for these experiments?
(a) It has a relatively high coil resistance
(b) It measures the magnitude of the current ER2 C2
(a) Voltage V1 across C1 is
(c) It can indicate currents flowing through it in either ( r + R2 ) (C1 + C2 )
direction ER2 C1
(d) It is very sensitive to small currents. (b) Voltage V2 across C2 is
( r + R2 ) (C1 + C2 )
7. A galvanometer has a resistance of 96 and full scale
ER2 C1
deflection of 100 µA. It can be used as ammeter provided a (c) Voltage V1 across C1 is
resistance is added to it. Pick up the correct range and ( r + R2 ) (C1 + C2 )
resistance combination(s). ER2 C2
(d) Voltage V2 across C2 is
(a) 1.3 mA range with 25k resistance in parallel ( r + R2 ) (C1 + C2 )
(b) 1.3 mA range with 8k resistance in parallel

MARK YOUR
5. 6. 7. 8. 9.
RESPONSE
508 IIT-JEE PHYSICS Challenger
10. Two conducting spheres of radii r and 2r are placed at very 12. Choose the correct options for the circuit shown.
large separation. Each spheres possesses charge Q. These Earth
R
spheres are connected with a conducting wire of resistance
R. Then, which of the following is true ? 3R R
P
Q 4V 3V
(a) Initial current is 8
0 rR
Earth
(a) Potential of the point P is 7V
Q (b) Current in the 3R resistance is 7/R
(b) Initial current is 4
0 rR (c) Current in the 3R resistance is 7/3R
(d) Potential of the point P is 3V
(c) Current reduces to half the initial current after time
13. Choose the correct options if a current source E2 is short-
12 0 rR ln 2 circuited. The parameters of the elements of the circuit should
(d) Current reduces to half the initial current after time be assumed to be specified.

8 0 rR ln 2
3 E1 E2

11. A battery of emf E0 = 5V and internal resistance 5 is


connected across a long uniform AB of length 1m and R
resistance per unit length 5 m–1. Two cells of E1 = 1V and (a) Current may increase (b) Current may decrease
E2 = 2 V are connected as shown in the figure. (c) Current must increase (d) Current must decrease
14. There are four resistors of 12 ohm each. Which of the
following values is/are possible by their combinations ?
5V (series and/or parallel).
5
(a) 9 ohm (b) 16 ohm
(c) 12 ohm (d) 30 ohm
15. For the circuit shown in figure.

P
A B
1V 1 5 B 8

G
10 2
2 25V
2V 4

(a) The null point is at A A 15 2


(b) If jockey is touched to point B the current in the
galvanometer will be going towards B
(a) potential difference between points B and A is 20V
(c) When jockey is connected to point A no current is (b) potential difference between points B and A is 10V
flowing through 1V battery (c) the current in the 4 resistor is 0.25 A
(d) The null point is at distance of 8/15m from A (d) the current in the 4 resistor is 0.5 A

MARK YOUR 10. 11. 12. 13. 14.


RESPONSE 15.
CURRENT ELECTRICITY 509

16. The diagram shows a modified meter bridge, which is used 17. Choose the correct options for the circuit shown.
for measuring two unknown resistance at the same time.
When only the first galvanometer is used, for obtaining the
balance point, it is found at point C. Now the first E K
galvanometer is removed and the second galvanometer is
used, which gives balance point at D. Using the details
given in the diagram, find out the value of R1 and R2. C1

R1 R2 C2
A B
R R1 R2

R3
G G

A B (a) The current through the battery in the circuit


C D
immediately after the key K is closed is E/R1
AB = L, AC = L/4 and AD = 2L/3 (b) The current through the battery in the circuit a long
time interval after the key K is closed is E/(R1 + R3)
(c) The current through the battery in the circuit
(a) R1 = 5R/3 (b) R2 = 4R/3 immediately after the key K is closed is E/R3
(c) R1 = 4R/3 (d) R2 = 5R/3 (d) The current through the battery in the circuit a long
time interval after the key K is closed is E/(R1 + R2)

MARK YOUR
16. 17.
RESPONSE
510 IIT-JEE PHYSICS Challenger
1. In the circuit shown in the figure the electromotive force of the battery is 9V and its internal resistance is 15 . The two identical
voltmeters and the ammeter can all be considered ideal (i.e. voltmeter is of very large resistance and ammeter is of negligible
resistance)
Column I Column II
(A) Voltmeter (Left) reading (in volts) when K is open (p) 4.5
V V

(B) Voltmeter (right) reading (in volts) when K is closed (q) 3 K


10 20

(C) Ammeter reading (in amperes) when K is closed (r) 4


(s) 0.2 9V 15
A
2. Regarding a conducting wire, match the following
Column I Column II
(A) With increase in temperature (p) Drift velocity increases
(B) With increase in length (q) Resistance increases
(C) With increase in area of cross-section (r) Resistance decreases
(D) With increase in volume (s) Number of conduction electrons increases

1. 2.

MARK YOUR
RESPONSE
CURRENT ELECTRICITY 511

1. A heater is designed to operate with a power of 1000 watts


in a 100 volt line. It is connected in a combination with a
1A
resistance of 10 ohm and a resistance R to a 100 volt mains
as shown in the figure. What should be the value of R
3
(in ohm) so that the heater operates with a power of 62.5
watt ? 4V 3 5

Heater 2A
100 R C 1
4 F
3V 1
100V 2A 2 4
2. If a copper wire is stretched to make it 0.1% longer, what
is the percentage change in its resistance? 3

3. In the circuit shown in figure, a voltmeter reads 30 volts


when it is connected across 400 ohm resistance. Calculate 1A
what the same voltmeter will read when it is connected across
the 300 ohm resistance?
5. An electrical circuit is shown in Fig. Calculate the potential
difference across the resistor of 400 ohm, as will be measured
by the voltmeter V of resistance 400 ohm, either by applying
300 400
Kirchhoff’s rules or otherwise.

60V
V

4. A part of circuit in a steady state along with the currents 400


flowing in the branches, the values of resistances etc., is
shown in the figure. Calculate the energy (in mJ) stored in 100 100 200
the capacitor C (4µF ). I2
100
I1

I 10V

1. 2. 3. 4. 5. 6.

MARK
YOUR
RESPONSE
512 IIT-JEE PHYSICS Challenger

1 (a) 7 (b) 13 (a) 19 (a) 25 (b) 31 (b) 37 (d) 43 (d) 49 (c) 55 (a)
2 (b) 8 (d) 14 (d) 20 (c) 26 (d) 32 (a) 38 (b) 44 (b) 50 (b) 56 (a)
3 (c) 9 (a) 15 (a) 21 (a) 27 (b) 33 (c) 39 (c) 45 (b) 51 (d) 57 (d)
4 (d) 10 (c) 16 (a) 22 (c) 28 (c) 34 (c) 40 (a) 46 (a) 52 (a)
5 (d) 11 (d) 17 (d) 23 (a) 29 (c) 35 (a) 41 (a) 47 (a) 53 (d)
6 (b) 12 (a) 18 (c) 24 (c) 30 (a) 36 (c) 42 (b) 48 (b) 54 (c)

1 (b) 4 (a) 7 (b) 10 (b) 13 (c) 16 (d) 19 (a) 22 (a)


2 (a) 5 (c) 8 (a) 11 (b) 14 (a) 17 (b) 20 (d) 23 (b)
3 (b ) 6 (a) 9 (c) 12 (d) 15 (a) 18 (c) 21 (a) 24 (c)

1 (a) 2 (c) 3 (d) 4 (c) 5 (d) 6 (c) 7 (c)

1 (a,b) 4 (a, c) 7 (b, d) 10 (a, d) 13 (a, b) 16 (a, b)


2 (a, b, c, d) 5 (a, c) 8 (a, c, d) 11 (a, b) 14 (a, b, c, d) 17 (a, b)
3 (a, b, c) 6 (c, d) 9 (a, b) 12 (a, c) 15 (a, c)

1. A-q; B-r; C-s 2. A-q, s; B-q, s; C-r, s; D-s

1 5 2 0.2 3 22.5 4 0.8 5 6.67


CURRENT ELECTRICITY 513

1. (a) Potential difference across 20 = 20 × 1 = 20 volt


6.023 1026
= potential difference across R2. n 10.5 103
Current in R2 = 0.5 A. R2 = 20/0.5 = 40 . 108
Potential difference across R1 = 69 – 20 = 49 Volts.
vd =
20 neA
Current in R1 = 0.5 A + 1A = 3.5A
10
20
108
49 6.023 10 26 10.5 103 1.6 10 19
3.14 10 6
R1 14
3.5 = 6.798 × 10–4 m/sec.
L1 L2 6. (b) Let the edges be 2 , a and , in decreasing order..
2. (b) R1 , R2 .
1A 2A 2 2
2Rmax =
V = potential at junction. a a

V1 V V V2 Rmax
Rmin = ; R =4
R1 R2 2 a 2a min

I [(3 ) 2 /( L / A)] ( LA ) s T
3. (c) ×12000 /6000=2000, 7. (b) 2
=
4 [( N ) /( 2 L / A)] (2 L A ) s T

220 (using V2/R = ms T)


I= A = resistivity; s = specific heat capacity of material of
9000+2000
the wire. A = area of cross section
Potential difference across voltmeter = 40 V.
4. (d) 1.5 V 9 1
2
= N2 = 9 N = 3.
N 2 2
8. (d) R1 = R0(1 + 1t) + R0(1 + 2t)

= 2 R0 1 + 1 2

2
Comparing with R = R0 (1+ t)

A = 1 2
2
9. (a) Given ; J = ar2.
This is a DC circuit because the battery is the only 2 R/ 2
source of voltage. Hence, the capacitors behave like i= J 2 r dr ar 2 2 r dr
open circuits. An equivalent circuit is then two parallel 1 R/3

sets of two identical series resistors, see figure. The R/2


voltage drop across each parallel branch must be the R/2 r4
=2 a r 3dr 2 a
battery voltage of 3V. Since the resistors are identical R/3 4
R/3
there is an equal voltage drop of 1.5 V across each
resistor. In particular there is a drop of 1.5 V across 4 4
a R R
resistor A. = ÷
2 2 3
6.023 1026
5. (d) Number of electrons per kg of silver =
108 aR 4 65 65 aR 4
= =
Number of electrons per unit volume of silver 2 81 16 2592
514 IIT-JEE PHYSICS Challenger

1.5 Ig
10. (c) The value of R = 3
1.5 103 G
1 10 I
Let R0 be the required resistance.
Then for 10% of the full scale deflection current, we I–Ig
have
S
mA
G
I = Ig + 1÷
A S
R0
100
1.5 V B 100 10 6
+ 1÷
0.1
= 100 × 10–6 × 1001
R = 100.1 mA
1.5 m m
0.1 × 10–3 = 13. (a) Relaxation time = =
R + R0 ne 2
ne2
1.5
R + R0 = 15 103 31
0.1 10 3 9.1 10
R0 = 15 × 103 – R =
8.5 1028 (1.6 10 19 2
) 1.7 10 8
= 15 × 103 – 1.5 × 103
= 13.5 × 103 = 2.5×10–14 sec

11. (d) 12 3kT 3 1.38 10 23 300


vrms = = = 105 m / s
7 m 9.1 10 31
4 So mean free path
= vrms × = 105 × 2.5 × 10–14 = 25 Å
R
14. (d) Points 1, 2 , 3 , ........ are equipotential

24 V

3R R/8
The equivalent resistance = 7 +
3+ R
The power consumed R/8
=
V2
= 60 R/8
3R
7+ ÷
3+ R

24 2
or = 60 and 1 , 2 , 3 , ........ are also equipotential.
3R
7+ ÷
3+ R
15. (a) Q at bt 2
3R
7+ = 9.6 dQ
or
(3 + R) i= = a 2bt
dt

or
3R
= 2.6 i =0 for t = t0 = a / 2b , i.e., current flow from
3+ R t = 0 to t = t0.
3R = 7.8 + 2.6 R
t0
or R = 19.5
The heat produced = i 2 R dt. Putting the value of i
12. (a) IgG = (I – Ig) S
0
Here, Ig = 100 × 10–6 A
G = 100
a3 R
S = 0.1 we get heat produced = .
6b
CURRENT ELECTRICITY 515

16. (a) Let each half side has resistance r (= d/2)


R
A
r r
R/2 R/2
r r
r
R R R
r A R B
R R R
r r R
r r R R
B
19. (a) Area of the polygon

r r = A= R sin 2 R cos ÷ n
2 2 2
A r B 1 2 1 2
r R n sin = R 2 n sin ÷
2 2 n
r r
r
dx
1 (2r )(r 2)
R= 2r + =r 2
2 (2 + 2)r
(on solving)
R d/ 2

d 1 2
20 + x = = BA = BR 2 n sin ÷
17. (d) vP = 4 dt 2 n
40
vQ = 2 Resistance R ' =
A
10 Q 10
Now, R sin ÷ .2 n, A r2
V 2
A B
PL n 2 R sin ( / n) 2 R n sin ( / n)
20 cm x R' = =
4v r2 r2

1 2 r2
x I= = BR n sin ( 2 / n ).
vP – vQ = R' 2 2 R n sin ( / n)
10
x r 2 RB cos ( / n)
2 sin t = =
10 2
x = 20 sin t
20. (c) RA' = RA (1 + A T)
dx
= (20 cos t) cm/s.
dt RB' = RB (1 + B T)
18. (c) R' = RA (1 + A T ) + RB (1 + B T)
2R
From given condition,
RA (1 + A T ) + RB (1 + B T ) = RA + RB
R/2 R/2
2R 2R
A A
A
B B
B =0
4R AA AB
A B
R R R R 3
A
=
2R B 10
516 IIT-JEE PHYSICS Challenger
21. (a) mgh = 0.6 I 2 R t point may be obtained. The current may be increased
also by increasing V.
0.6 I 2 Rt dh 0.6 I 2 R
h v= = = 0.141 m/s R1 40
mg dt mg 26. (d) =
R2 100 40
22. (c)
R1 + 10 60
and =
180V 90 45 R2 100 60

90 R2 = 12 .
4R
27. (b) r =
9
40 50
R

R By Symmetry method

i 4R
180V 90 45
The internal resistance must be equal to external
i resistance for maximum power transfer.
45 4R
The Req for circuit =
9
180
i= = 2A 4R 4R
90 Thus, Req = . Thus r =
9 9
76.3
23. (a) r 9.5 –1÷ 1.7
64.8 28. (c) Since voltmeter is ideal and reading of voltmeter is zero.
So, either fault is in box or 5 resistance.
1 So, sum of potential difference across box and 5
24. (c) S R
i ir resistance will be 12 volt.
29. (c) Power = I2R. Brightness order, d > a > b > c
Q P 30. (a) Let the resistance of half the turn be R. Then in the
R iR
former case, we have fifteen resistors of resistance R
M A N connected in parallel, the total resistance being R/15.
2R In the latter case, we have the same fifteen resistors
Reading in Ammeter is zero, so the current (i) will connected in series, the total resistance being 15R.
only flow in the upper circuit. Therefore, as a result of unwinding, the resistance of
the wire will increase by a factor of 225.
1 31. (b) By using Kirchhoff’s first law
where I =
R+r I1 + I5 = I6 ... (i)
= ir = iR, I2 = I4 + I5 ... (ii)
(b) is correct. Adding (i) and (ii), we get
Here, vQ – vR = vO – vP = vM – vN = I1 + I2 = I6 + I4
1R 32. (a) In the former case, the condition I1 = I2 is fulfilled,
also vQ – vR = – rir = – r.
1
=
1 1 R+r R+r where I1 = 1n1 and I2 = 2n2. Consequently, 1n1 =
= 2n2. In the latter case, I1'R1 = I2'Rx, where Rx is the
resistance of the second resistor. Besides, I'1 = 1n'1
(R + r) and I'2 = 2n'2, and hence
1=
R R1 1 n1 Rx 2 n2
(a) is correct and (c) is incorrect
no current is flowing through 2R R1 n1 R x n2
Finally, we obtain =
potential difference across 2R = 0 n1 n2
(d) is also correct.
25. (b) Decreasing R increases current in XY and there by R1 n2 n1
increases the potential drop across XP and the balance Therefore, Rx =
n1 n2
CURRENT ELECTRICITY 517

33. (c) In order to simplify the solution, we present the circuit


V
in a more symmetric form (figure I). satisfies the equation R AB = , we obtain
I1 + I 2
V V 5V
A B RAB = = =
I1 + I 2 I1 + (6 / 5) I1 11 I1
Taking into account the relation obtained above, we
get the following expression for the required resistance:
15
V R AB = R
11
(I) 34. (c) I1 and I4 increases.
The obtained circuit cannot be simplified by connecting 35. (a) We shall mentally connect in series two perfect (having
or disconnecting junctions (or by removing some zero internal resistance) current sources of emf’s equal
conductors) so as to obtain parallel- or series- to –V0 and V0 between points A and F. Obviously, this
connected subcircuits. However, any problem will not introduce any change in the circuit. The
involving a direct current has a single solution, which dependence of the current through the resistor of
we shall try to “guess” by using the symmetry of the resistance R on the emf’s of the sources will have the
circuit and the similarity of the currents in the circuit. form
Let us apply a voltage V to the circuit and mark currents I = E – V0 + V0 ,
through each element of the circuit. We shall require where E is the emf of the source contained in the circuit,
not nine values of current (as in the case of arbitrary and the coefficients and depend on the resistance
resistances of circuit elements) but only five values I1, of the circuit.
I2, I3, I4, and I5 (figure II). If we connect only one perfect source of emf equal to –
C I5 E V0 between A and F, the potential difference between A
I1 I2 and B will become zero. Therefore, the first two terms
A I3 B in the equation for I will be compensated : I = V0. The
I3
I4 1
I2 I1 coefficient is obviously equal to , where
R + R eff
D I5 F
Reff is the resistance between A and B when the resistor
V R is disconnected. This formulas is also valid for the
case R = 0, which corresponds to the connection of the
(II)
V0
For such currents, Kirchhoff’s first rule for the junction ammeter. In this case, I 0 =
C, I1 = I3 + I5 R eff
and for the junction D V0 I 0
Consequently, the required current is I =
I2 + I3 = I4 + I5 RI 0 + V0
will automatically be observed for the junctions E and 36. (c) It can easily be noted from symmetry considerations
F (this is due to the equality of the resistances of all that the potentials of points A and C (figure) at any
resistors of the circuit). Let us now write Kirchhoff’s instant of time will be the same. Therefore, the closure
second rule in order to obtain a system of five of the key K will not lead to any change in the operation
independent equations: of the circuit, and the coils AC will not be heated.
(I2 + I5 + I1) R = V A
(I3 + I4) R = I5R
(I1 + I3) R = I2R,
where R is the resistance of each resistor. Solving this
system of five equations, we shall express all the
currents in terms of I1 : K
C
6 I 3 4
I2 = I1 , I3 = 1 , I 4 = I1 , I5 = I1
5 5 5 5
4 6 ~
Besides, V = I1 + I1 + I1 ÷ R 37. (d) The potentiometer with the load is equivalent to a
5 5
resistor of resistance
V
Consequently, =R R RR / 2 5
3I1 R1 = + = R
Considering that the resistance RAB of the circuit 2 R+ R/2 6
Hence the total current in the circuit will be
518 IIT-JEE PHYSICS Challenger
V 6V We replace the “inner” triangle consisting of an infinite
I1 = = number of elements by a resistor of resistance RAB/2,
(5 / 6) R 5 R
where the resistance RAB is such that RAB = Rx and
The voltage across the load will be RAB = a . After simplification, the circuit becomes a
R 2 system of series and parallel connected conductors. In
V11 V I1 = V
2 5 order to find Rx, we write the equation
If the resistance of the load becomes equal to 2R, the 1
total current will be RR x / 2 RR x / 2
Rx = R R + R+R+
V 10 V R + Rx / 2 R + Rx / 2 ÷
I2 = =
R ( R / 2) (2 R ) 9 R Solving this equation, we obtain
+
2 R / 2 + 2R
The voltage across the load will become R ( 7 1) a ( 7 1)
R AB = Rx = =
R 4 3 3
V21 V I 2 = V 41. (a) Let us make use of the fact that any “black box” circuit
2 9
Thus, the voltage across the load will change by a consisting of resistors can be reduced to a form (figure a),
factor of k = V21/V11 : where the quantities R1, R2, ............ R5 are expressed in
terms of the resistances of the initial resistors of the “black
V21 10 box” circuit (this can be verified by using in the initial
k= =
V11 9 circuit the transformations of the star-delta type and
reverse transformations).
BNA
38. (b) Sensitivity = ; so by increasing B (stronger
k
magnet) and decreasing torsion constant, k (weaker 1 3
R1 R3
springs) sensitivity of a moving coil galvanometer can
be increased.
R2
39. (c) When steady state is reached, the current I coming
from the battery is R4 R5
9 = I (3 + 6) 2 4
I = 1A
3 F 6 F (a)
+Q1 –Q1 +Q2 –Q2
By hypothesis, equal currents pass each time through
3 6 resistors of resistance R1 and R4 and also through R3
and R5 (or there is no current through them when the
clamps are disconnected). Using this circumstance, we
can simplify the circuit as shown in figure b.
9V
Potential difference across 3 resistance = 3V and
potential difference across 6 resistance = 6V 1 3
R'1 R'3
p.d. across 3 µF capacitor = 3V
and p.d. across 6 µF capacitor = 6V
R'2
Charge on 3 µF capacitor Q1 = 3 × 3 = 9 µC
Charge on 6 µF capacitor Q2 = 6 × 6 = 36 µC
Charge (– Q1) is shifted from the positive plate of 6 µF 2 4
capacitor. The remaining charge on the positive plate
of 6 µF capacitor is shifted through the switch.
(b)
Charge passing the switch = 36 – 9 = 27 µC
40. (a) It follows from symmetry considerations that the initial Then, by hypothesis,
circuit can be replaced by an equivalent one (figure).
V2
N1 = ;
R1 + R2
R/2 R/2
V2
Rx/2 N2 = ;
R1 + R2 R3 /( R2 + R3 )
R/2 R/2
V2 V2
R N3 = ; N4 =
B R2 + R 3' R3 + R1 R2 /( R1 + R 2 )
A
CURRENT ELECTRICITY 519

It can easily be verified that where is the length of the wires. Tmelt is the melting
N1N4 = N2N3 point of the wire material , and I1 and R1 are the current
Consequently, and resistance of the first wire.
N 2 N3 The resistance of the second wire is R2 = R1/16.
N4 = = 40W Therefore, the current I2 required for melting the second
N1
wire must satisfy the relation
42. (b) If P is disconnected Req of circuit increases hence
less current is drawn. I 22 R2 k .4d 2 (Tmelt Tam )
The potential difference between the point X and the
Finally, we obtain
ground is x given by
I2 > 8I1 = 80A
0 + E – iR1 = x
as i decreases, x increases 45. (b) In steady state current in the branch containing the
43. (d) It follows from symmetry considerations that the capacitor is zero and hence emf E is shared between r
potentials of points C and D are equal. Therefore, the and R2 in the ratio of their resistances. Voltage across
circuit can be replaced by an equivalent one (we ER2
combine the junctions C and D, figure). R2 is R + r = 2 Volts = Voltage across capacitor..
C(D) 2

Q = CVC = 1 F × 2V = 2µC.
46. (a) RA = resistance of ammeter
R/2 R/2
4 V1 V1 1 V1 0
R/2 = + ......... (1)
100 RA 100
R R 1V – 0V = (10mA) RA
B RA = 100 ......... (2)
A
V 0V 0V 0V

R
R 100 A 10mA
R
4V
V1 A 1V
D
The resistance between points A and B of the circuit 100 V 3V(given)
can be determined from the formulas for parallel and
series connection of conductors :
4V 4V 4V
R / 2 (R / 2 + R) 3 R 3
RC ( D ) B = = = R 4 V1 V1 1 V1 0
R/2+ R/2+ R 4 2 8 = + [By using eq. (1) and (2)]
100 100 100
R 3 7
R AC ( D ) B = + R= R, V1 = (5/3) V
2 8 8
V1 1
R (7 / 8) R 7 = (Current in ammeter (II))
R AB = = R RA
R + (7 / 8) R 15
Thus, the current I in the leads can be determined from 5/3 1
the formula = 6.67mA
100
V 15 V 47. (a) Compressing the hexagonal box.
I= =
(7 /15) R 7 R
44. (b) The condition required for heating and melting the wire
is that the amount of Joule heat liberated in the process
must be larger than the amount of heat dissipated to
the ambient :
I2R > kS (T – Tam) P Q
By hypothesis, the current required for melting the first
wire must exceed 10A. Therefore,
k .4d1 (Tmelt Tam ) = I12 R1 ,
Folding about the line PQ
520 IIT-JEE PHYSICS Challenger
P
P 51. (d) Let the emf of the second source be E2. Then, by
5R/16 hypothesis,
R/2 5R/8 E1 + E2 E2
5R/2 R/8 I= =
R R + Rx R ,
where R is the resistance of the varying resistor for a
R/2 R/2 R/2 R/2 constant current. Hence we obtain the answer
R E1
Rx =
I
R/2 5R/2 R/8 5R/8 52. (a) No. of electron in the wire = nA (n = e– density)
5R/16 me I me
and momentum (nA ) mevd = neAvd =
Q e e
Q
(Using star-delta transformation) 70 1000 31
= 9.1 10 = 40 × 10–8 N-sec
23R 1.6 10 19
RPQ = 53. (d) For constant current, Q = it and for resistance,
20
i2Rt = 45 and i2rt = 15
48. (b) Let us consider the steady-state conditions when the
i2 (R + r) t = 60 or i (R + r) (it) = 60 or E (Q) = 60
voltage across the capacitor practically does not
change and is equal on the average to Vst. 60 60
or E = = V
When the key is switched to position 1, the charge on Q 40
the capacitor will change during a short time interval 54. (c) The charge in a pulse dq = Idt = 5.00 × 10–8C.
t ( E1 Vst )
t by q 5 10 8 C
R1 n= = = 3.12 1011
e 1.6 10 19 C
When the key is switched to position 2, the charge will
55. (a) The total resistance RAB of the frame can easily be
t ( E2 Vst ) calculated by noting from symmetry considerations that
change by there is no current in the edge CD . RAB = R/2, where R
R2
During cycle, the total change in charge must be zero : is the resistance of an edge. Therefore,
2V
( E1 Vst ) ( E2 Vst ) I= , where V is the applied voltage.
+ =0 R
R1 R2
The total current can be changed in two ways : (1) if we
Hence the voltage Vst and the capacitor charge qst in remove one of the edges AD, AC, BC or BD, the change
the steady state can be found from the formulas in the current will be same, (2) if we remove the edge
E R + E1 R2 AB, the change will be different. In the first case, the
Vst = 2 1 change in the current will be I = – (2/5) V/R = – I/5,
R1 + R2
and in the second case, the total resistance will be R,
C ( E2 R1 + E1R2 ) and hence
qst = CVst = I = – V/R = – I/2 = Imax.
R1 + R2
56. (a) It follows from symmetry considerations that if we
49. (c) For various combinations equivalent resistance is remove the first element from the circuit, the resistance
maximum between P and Q. of the remaining circuit between points C and D will be
50. (b) After adding two conductors, the circuit will acquire RCD = kRAB. Therefore, the equivalent circuit of the
the form shown in figure. From symmetry infinite chain will have the form shown in figure.
considerations, we calculate that the central conductor
R1 C
will not participate in electric charge transfer. A

R2 k RAB

Therefore, if the initial resistance R1 of the circuit was B D


5r, where r is the resistance of a conductor, the new
Applying to this circuit the formulas for the resistance
resistance of the circuit will become of series and parallel resistors, we obtain
2r R 3 R1 + R2 kR AB
R2 = 2r + = 3r Therefore, 2 = R AB =
2 R1 5 R2 + kR AB
CURRENT ELECTRICITY 521

Solving the quadratic equation for RAB, we obtain (in By hypothesis,


particular, for k = 1/2) R12 = R13 = R34 = R24 = r,

R1 R2 + R12 + R22 + 6 R1 R2 r
R AB = R15 = R25 = R36 = R46 = ,
2 2
57. (d) Let us represent the central junction of wires in the Let V be the voltage between points 1 and 2. Then the
form of two junctions by the wire 5-6 as shown in amount of heat liberated in the conductor 1-2 per unit
figure.
3 V2
4 time is Q12 = .
r
From ohm’s law, we obtain the current through the
6 conductor 3-4 :
5 V
C I34 =
r ( 2 + 3)
The amount of heat liberated in the conductor 3-4 per
1 2
unit time is
Then it follows from symmetry considerations that there
V2
is no current through this wire. Therefore, the central Q34 = I34
2
r=
junction can be removed from the initial circuit, and we r ( 2 + 3)2
arrive at the circuit shown in figure. Therefore, the required ratio is
3 4 Q12
= ( 2 + 3)2 = 11 + 6 2
Q34
6
5

1 2

1. (b), 2. (a)
L 1
Resistance of the 2 layers are = , where = ÷
1 A1 2 A2
L
R1 1
A1 V V
I= = ( 1 A1 2 A2 )
L Req L
R2 2
A2 3. (b) Here, the total resistance is given by
Hence, total resistance across the voltage source is
R1R2
Req = + R3
1 A1 R1 + R2
2 A2
L L
A1 1
A 3 = A 1 + A 2,
A2 2 3

I V
2
1 2L
R1R2 A1 A2 L
Req = = =
R1 + R2 A1 A2
1 + 2
÷L + I V
A1 A2 1 2
522 IIT-JEE PHYSICS Challenger
Potential difference between A and B
L L
= + VA VB i2 R2 E2 ( 1) 2 3 5V
1 A1 2 A2 3 ( A1 + A2 )
Production of heat in R1 = i12 R1
L [( 3 ) A1 ( 2 3 ) A2 ]
= 1
3 ( 1 A1 A
2 2 ) ( A +
1 A2 )
(3.5)2 2 =24.5 J/s
Hence, 6. (a) Now we consider the situation in which the switch is
put in position 2. (see figure)
V ( 1 A1 2 A2 ) ( A1 + A2 )V
I= = 3 E2
Req ( 1 3 1 ( 2
) A 3 ) A2 L A R2 B
S – +
The electric field in the layers is E = (V / 2 L) 3V 2

2( 1 A1 2 A2 ) 3 O 3
I= ( A1 + A2 ) E R4
( 1 3 ) A1 ( 2 3 ) A2
Effective current density L
P Q
I 2( 1 A1 2 A2 ) 3 E 10 mH
J eff = =
A1 A2 ( 1 3 ) A1 ( 2 3 ) A2 Total resistance of the circuit
R = R2 + R4 = 5
J 2( 1 A1 2 A2 ) 3
Now, eff = eff =
E ( 1 3 ) A1 ( 2 3 ) A2 L = 10 mH = 10 10 3
H = 10 2
H
Steady current i0 = E / R = 3/ 5 = 0.6 A
2( 2 x) 3 A
= 1
where x = 2 ÷ ( In steady state, there is no role of inductor).
1 3 ( 2 3)x A1 7. (b) Just after crossing the switch capacitor is short circuited
4. (a), 5. (c)
When the switch is in position 1. Let the currents in 3 6
the branches R1 and R2 be i1 and i2 respectively
(see figure). The capacitor offers infinite resistance to 4 4
dc. Therefore current in capacitor branch is zero. The
current in resistance R3 will be (i1 + i2 ) .
R3 30 volts
K L
2 12 24
Req = + .
i1 + i2 7 10
R1 E1
i1 – + 144 36
M Req = ,i = = 8.75 A
2 N 35 R eq
8. (a) = Req C
E2 R2 Here Req is taken across capacitor
i2 – + Req = 4
S A 2 B = 20 µs.
Applying Kirchhoff’s law, to mesh KMNLK, we have, 9. (c) At steady state capacitor is open circuit.
VBD = 6 volts
i1 2 + (i1 + i2 ) 2 = 12
q = CVBD = 30 µC.
or 2i1 + i2 = 6 ...(i) 10. (b) As Rv decreases, I 2 increases. Therefore, I1 will
decrease.
Applying Kirchhoff’s law to mesh KSBNLK
I1
i2 2 + (i1 + i2 ) 2 = 3
or 2i1 + 4i2 = 3 ...(ii) I2
Solving eqs. (i) and (ii), we get Rv
i1 = 3.5 A and i2 = 1 A
E r
CURRENT ELECTRICITY 523

11. 2
(b) Rate of energy (= Power = I1 R) decreases as I1 In both the batteries, current is entering from positive
decreases. terminal. Therefore, both batteries are being charged.
19. (a) Potential at A = 6V
2R
E 2 Rv
12. (d) Rate of energy = Power = I2 v=
VA – VC = 4
(r + Rv ) 2 VC = 2V
V AD V AC AD 4 2
P 20. (d) = = = = ;
V AB V AB AB 6 3

200
e
as

AD = cm.
cre

Inc 3
rea
De

se 21. (a) D is balance point, hence no current

22. (a) When switch is open,


Rv
Rv=r
E
V1 = ÷ RV1
Thus, we cannot determine the variation of power. RV1 + RV2
13. (c) Just after closing, capacitor behaves as short circuit
and all current flows through it hence ammeter reads
200
zero. 3000 = 120 volt
14. (a) After long time capacitor behaves as open circuit and 5000
no current flows through it. 23. (b) When switch is open,
v0 30
Therefore i = = = 2mA E
R1 + R2 10 + 5 V2 = ÷ RV2
RV1 + RV2
15. (a) Just after reopening, potential difference across R2
remain same initially as charge on capacitor does not
200
change initially hence current remains same. 2000 = 80 volt
16. (d), 17. (b), 18. (c). 5000
P5 = i2R = (11)2 (5) = 605 W 24. (c) When switch is closed, RV1 and RV are also in
2
VC + 5 + 12 – 12 – 3 – 8 = VB
VC – VB = 6 volt parallel. Their net is same. Therefore, 200V will be
equally distributed among them.
1 12V 2 3V 2A 4
200
C 5A 6A B V1 = V2 = = 100 volt
5 6 2
11A 4A

1. (a) Both the statement – 1 and statement – 2 are true and 3. (d) Statement – 1 is false and statement–2 is correct
statement-2 is the correct explanation of statement –1. 4. (c) Positive terminal of a battery is point of highest
When cold water is poured on half portion of the wire, potential and current flows from highest to lowest
its resistance decreases due to decrease in temperature. potential i.e. from +ve to –ve potential.
As a result of this total resistance of the circuit
5. (d) E j as area and current are same j is same but are
decreases, i.e., current through each portion of wire
increases, i.e., rest of the half portion becomes still different.
more hot. E is different.
2. (d) The e.m.f. of a dry cell is dependent upon the electrode 6. (c) V = E – ir = 4 – 2 × 2 = 0, During charging V > E.
potential of cathode and anode which in turn is
dependent upon the reaction involved as well as Ep
7. (c) E , = (assuming Rh = 0, r = 0)
concentration of the electrolyte. It has nothing to do L
with size of the cell. If E and Ep double remain same.
So, statement-1 is false and statement-2 is true. Statement -1 is True, Statement-2 is False.
524 IIT-JEE PHYSICS Challenger

1. (a,b) Let be the resistance per unit length of the Potential drop across capacitor C = 3V
wire. Let the fault exist at a distance x from the end Potential drop across resistor = (1 × 3) = 3V
A and r be the resistance of the leak. Potential at point P (Vp) = 20 – (3 + 3) = 14 V
(a) When B is earthed, resistance between A and the V P– 0 = 14 V
earth through cable is given as Now, current through R is given by
1 Vp 0 14
R x + = a (given) i= = = 7A.
1/ r 1/( L x) R 2
1 1 1 1 Q 1 q
or = + ...(i)
4. (a, c) + =0
(a x) r ( L x) 4 0 R 4 0 ( R0 – vt )
(b) When A is earthed, resistance between B and earth where R0 is the initial distance of the charged
through cable is given as particle.
1
R ( L x) + =b (given) Q=
Rq
1/ r 1/ x R0 – vt
1/ 2
x a ( L b) dQ Rqv
or = ...(ii) =i=
L x b( L a ) dt ( R0 – vt )2
For second part of the question, Let us assume
5. (a, c) Area in (VL – t) graph = L I
that
1
x L 2(i f – 0) 10 2 i f = 5A
or x = ...(iii) 2
L x (1 )
6. (c, d) In the three experiments, the jockey is moved
Substituting (iii) in (i), we get : according to the direction of deflection of the
1 1 1 galvanometer.
= +
[a L /(1 )] r [L L(1 )] 7. (b, d) (i – ig) R = ig × 96

1 L L /(1 ) a L /(1 )
R
=
r [a L / (1 )][ L L(1 )] (i – 10–4) R = 96 × 10–4
[a (1 ) L] G
or r L ig 96
(1 )2 ( L a)
8. (a, c, d) Across all, we will get a balanced wheatstone
2. (a, b, c, d)
bridge and use series and parallel combination.
Consider the symmetry about various planes e.g.
9. (a, b) A direct current cannot pass through the
ABCD etc.
3. (a, b, c) At t = 0 capacitors of capacitance C1 and C2. Therefore,
in the steady state, the current through the current
dq
q = 3 C, iAP = = 3A E
dt I=
r + R2
q
VC = =3V Since the capacitors are connected in series, their
C
charges q will be equal, and
VA = 20 V q q
+ = IR2
i +q C1 C2
C = 1F
–q ER2 C1C2
1 Consequently, q =
1 R ( r + R2 ) (C1 + C2 )
B
P The voltages V1 and V2 across the capacitors can
VB = 20 V 20V
Earth be calculated from the formulas
CURRENT ELECTRICITY 525

where r1 and r2 are the internal resistances of the


q ER2 C2
V1 = = current sources. After the short-circuiting of the
C1 ( r + R2 ) (C1 + C2 ) ,
second source of emf E2, the current through the
q ER2 C1 resistor of resistance R can be determined from
V2 = = the formula
C2 ( r + R2 ) (C1 + C 2 ) respectively..
E1
I2 =
10. (a, d) Initial PD
Q Q
=
Q R + r1
4 0 r 8 0 r 8 0 r
E1 E + E2
Q Obviously, if > 1 ,
Initial pressure = PD = R + r1 R + r1 + r2
8 0 rR
we will say current increases.
Current reduces to half in time , RCeq ln 2 Thus, if the inequality E1 (R + r1 + r2) > (E1 + E2) (R
(Using i = i0e–t/RC) + r1) is satisfied, and hence E1r2 > E2 (R + r1), the
8 r current in the circuit increases. If, on the contrary,
Ceq = 0
(Both spherical capacitor are in E1r2 < E2 (R + r1), the short-circuiting of the current
3
source leads to a decrease in the current in the circuit.
series)
14. (a, b, c, d)

11. (a, b)
9

A B 12
i

G
C D
i 16

For null point current flow in the loop CD only.


3V
i= = 1A
2 1
VCD = 1V – 1 (1) = 0
option (a) is correct. 30
When Jockey touches to B current flow from A to
B to increase the P.D. across the secondary circuit.
option (b) is correct.
15. (a, c)
12. (a, c) Earth
R 1A 5 B 0.5A 8
3R R 0.5A 0.25A 0.25A
7V P
4V 3V 10 2
25V
4
Earth 4V
0V
1A
VP = 7V ; A 15 2 0.25A
7 0 7
i= =
3R 3R Net resistance of the circuit is 25 . Therefore
13. (a, b) The current through the circuit before the source current through the battery is 1A. The distribution
of emf E2 is short-circuited satisfies the condition of current in difference resistors is as shown in
figure.
E1 + E2
I1 = VB – 0.5 × 10 – 15 × 1 = VA
R + r1 + r2 VBA = 20 volt
526 IIT-JEE PHYSICS Challenger
17. (a, b) At the first moment after the connection of the
R L/4
16. (a, b) = key K, the capacitors are not charged, and hence
R1 + R2 3 L / 4 the voltage between points A and B is zero (in
3R = (R1 + R2) Question figure). The current in the circuit at this
instant can be determined from the condition
R + R1 2 / 3
=
R2 1/ 3 E
I1 = . Under steady-state conditions, the
3R = R2 + R1 R1
R = 2R2 – R1 current between points A and B will pass through
4R = 3R2 the resistors R1 and R2. Therefore, the current
passing through these conductors after a long
5R 4R
R1 = , R2 =
3 3 E
time is I 2 =
R1 + R3

1. A-q; B-r; C-s 2. A-q, s; B-q, s; C-r, s; D-s


9 1 T R n ; R n ; A R
K open, i = = = 0.2 amp . n
45 5 Volume n (Volume may be due to
1
Voltage across 10 and 20 , V = (10 + 20) = 6 V, V1eft A or A or A , so variation in
5
1 resistance is not certain)
= 3V, K closed, Vright = 20 = 4 volt
5

1. 5
A B Heater
The resistance of the heater is
R
V 2 100 100
R= = = 10
P 100 100 V

The power on which it operates is 62.5 W Applying Ohm's law across R, we get
25 = 5 × R R=5
V= R P' 10 62.5 = 625 = 25
2. 0.2
V 75
The current in AB = I = = = 7.5 A f i f i
R 10
Rf Ri Af Ai Af Ai
100 = = ... (i)
This current divides into two parts. Let I1 be the current that Ri i i
passes through the heater. Therefore Ai Ai

25 = I1 × 10
Let the initial length of the wire be 100 cm, then the new
I1 = 2.5 A
0.1
Curent through R in 5A. length is 100 + 100
100

f = 100.1 cm ...(ii)
CURRENT ELECTRICITY 527

Let Ai and Af be the initial and find area of cross-section. IV × RV = (I – IV) 300
Then IV × 1200 = (I – IV) 300
4IV = I – IV ... (1)
100 ×Ai = 100.1 Af Also 300 (I – IV) + 400 I = 60 700 I – 300 IV = 60
100 60 3
Af = Ai ... (iii) 3500 IV – 300 IV = 60 IV = = A
100.1 3500 160
From (i), (ii) and (iii) 3
Reading of voltmeter = 1200 = 22.5 V..
160
4. 0.8
(100.1) 2 100
Applying Kirchoff's first law at junction M, we get the current
Rf Ri 100 Ai Ai
100 100 i1 = 3A
Ri 100
Ai
1A

(100.1) 2 (100)2 200.1 0.1 3


= 2
100 100
(100) 100 100 4V 3 M i1 5
N
= 0.2 % 2amp
3. 22.5 C 4µF 1
For situation 1 3V 1
2A P O
V
i2 2 4
IV 3
I
300 A 400 B
1A
60 V

Let the resistance of voltmeter be RV and the current flowing Applying Kirchoff's first law at junction P, we get current
i2 = 1A
through it be IV.
Moving the loopfrom MNO to P
Then IV RV = 30 and (I – IV) × 400 = 30 VM – 5 × i1 – 1 × i1 – 2 × i2 = VP
Also 300 × I = 30 I = 0.1 A VM – VP = 6i1 + 2i2 = 6 × 3 + 2 × 1 = 20 V
Energy stored in the capacitor
3
0.1 – IV = 1 1
40 = CV 2 = × 4 × 10–6 × 20 × 20 = 8 × 10–4 J = 0.8 mJ
2 2
5. 6.67
3 Applying Kirchoff's law in loop JMGDJ, we get
IV = 0.1 –
40
I2/3 L
4 3 1 K V
IV = = A
40 40 I3/2
J M
1 I3 400 I3
RV = 30 RV = 1200 . I2 –I1
40 C E I1+I2–I3
G
I2 100 100 200
For situation 2
B F
I1 100 I1 + I2
V
IV
A
I
I 300 400 10V
60 V
528 IIT-JEE PHYSICS Challenger
Applying Kirchoff's law ABFEGHA
I3
× 400 + (I1 + I2 – I3) 200 + (I2 – I3) 100 = 0 – 100 I1 – 200 (I1 + 22 – I3) + 10 = 0
2
– 3I1 – 2I2 + 2I3 + 0.1 = 0 ... (iv)
– 200 I3 + 200 I1 + 300 I2 – 300 I3 = 0
Multiplying (ii) by (3) and adding it with (iv)
2I1 + 3I2 – 5I3 = 0 ... (i)
– 8I2 + 5I3 + 0.1 = 0 – 8I3 + 5I3 + 0.1 = 0
Applying Kirchoff's law in CDEFBC
– 100 I2 – 100 (I2– I3) + 100 I1 = 0 0.1
3I3 = 0.1 I3 =
I1 – 2I3 + I3 = 0 ... (ii) 3
Multiplying eq. (ii) by 2 and subtracting from (i) Potential difference across JM
2I1 + 3I2 – 5I3 – 2I1 + 4I2 – 2I3 = 0
0.1/ 3 0.1 20
7I2 – 7I3 = 0 = 400 400 = = 6.67V
2 2 3 3
I2 = I3 ... (iii)
1. Two electron beams having their velocities in the ratio 1: 2
(a) (1, 3) (b) (1, 3)
are subjected to identical magnetic fields acting at right
angles to the direction of motion of electron beams. The 1 3 3 1
2 ÷
ratio of deflection produced is: (c) , (d) , ÷
2 2 2
(a) 2 : 1 (b) 1: 2
5. A long straight wire along the z-axis carries a current I in the
(c) 4 : 1 (d) 1: 4
2. A current I flows in the anticlockwise direction through a negative z direction. The magnetic vector field B at a point
square loop of side a lying in the xoy plane with its center at having coordinates (x, y) in the z = 0 plane is –
the origin. The magnetic induction at the center of the square
loop is I ( yiˆ xjˆ)
0 I ( xiˆ + yjˆ)
0
(a) (b)
2 2 0I 2 2 0I 2 ( x2 + y 2 ) 2 ( x2 + y 2 )
(a) eˆx (b) eˆz
a a I ( xjˆ yiˆ) I ( xiˆ yjˆ)
0 0
2 2 0I 2 2 0I (c) (d)
(c) 2
eˆz (d) eˆx 2 ( x2 + y 2 ) 2 ( x2 + y 2 )
a a2
3. A particle of charge q and mass m moves in a circular orbit 6. An infinitely long wire carrying current I is along Y axis
of radius r with angular speed . The ratio of the magnitude such that its one end is at point A (0, b) while the wire
of its magnetic moment to that of its angular momentum extends upto + .
depends on The magnitude of magnetic field strength at point (a, 0).
(a) and q (b) , q and m
(c) q and m (d) and m
4. A uniform magnetic field of magnitude 1T exists in region
y 0 is along kˆ direction as shown. I
y A
B=1T

x (0,0) (a,0)

0I b 0I b
(a) 1+ ÷ (b) 1 ÷
(– 3–1) 4 a a + b2
2 4 a a + b2
2

A particle of charge 1C is projected from point ( 3, 1)


towards origin with speed 1 m/sec. If mass of particle is 1 kg, 0I b
then co-ordinates of centre of circle in which particle moves (c) 1 ÷ (d) None of these
4 a a + b2
2
are –

MARK YOUR 1. 2. 3. 4. 5.
RESPONSE 6.
530 IIT-JEE PHYSICS Challenger
7. A battery is connected between two points A and B on the 12. A charged particle is released from rest in a region of steady
circumference of a uniform conducting ring of radius r and and uniform electric and magnetic fields which are parallel
resistance R. One of the arcs AB of the ring subtends an to each other. The particle will move in a
angle at the centre.The value of the magnetic induction at (a) straight line (b) circle
the centre due to the current in the ring is (c) helix (d) cycloid
(a) proportional to 2 (180° – ) 13. A cyclotron is operated at an oscillator frequency of 24
(b) inversely proportional to r MHz and has a dee radius R= 60cm. What is magnitude
(c) zero, only if = 180° of the magnetic field B (in Tesla) to accelerate deuterons
(d) zero for all values of (mass = 3.34 × 10–27) kg?
8. A small current element of length d and carrying current is (a) 9.5 (b) 7.2
placed at (1, 1, 0) and is carrying current in ‘+z’ direction. If (c) 5.0 (d) 3.2
14. A thin rod is bent in the shape of a small circle of radius r. If
magnetic field at origin be B1 and at point (2, 2, 0) be B2 the charge per unit length of the rod is , and if the circle is
then: rotated about its axis at a rate of n rotations per second, the
magnetic induction at a point on the axis at a large distance
(a) B1 = B2 (b) | B1 | = | 2 B2 |
y from the centre
(c) B1 B2 (d) B1 2 B2
(a) 0 r 3n (b) 2 0 r 3n
9. Two long parallel wires carry equal current i flowing in the y3 y3
same direction are at a distance 2d apart. The magnetic field
B at a point lying on the perpendicular line joining the wires
and at a distance x from the midpoint is – (c)
0
r 3n 3 (d) 0
r 3n 3
4 y 2 y
0id 0ix 15. An infinitely long conductor is bent at a point O at an angle
(a)
(d 2
+x2
) (b)
( d 2 2
x ) as shown. The magnetic field at a point P distant r along
the angle bisector is
0ix 0id i
(d ) (d )
(c) (d)
2
+x 2 2
+ x2
P
10. A long straight wire of radius R carries current i. The
magnetic field inside the wire at distance r from its centre is
O
expressed as : i
µ0 i 2µ0 i (a) ( 0 / 4 ).(2i / r )[1 cos( / 2)] / sin( / 2)
(a) .r (b) ÷ .r
2÷ R2
R (b) ( 0 / 4 ).(i / r ) cot( / 2)
µ0 i µ0 i (c) ( 0 / 2 ).(i / r ) cot( / 2)
2 R÷
(c) (d) .r

.r
2 R (d) ( 0 / 4 ).(i / r )sin( / 2)
11. An electron, charge –e, mass m, enters a uniform magnetic 16. Two particles A and B of masses mA and mB respectively
field B = Bi with an initial velocity v = v x i + v y j . What is and having the same charge are moving in a plane. A uniform
magnetic field exists perpendicular to this plane. The speeds
the velocity of the electron after a time interval of t second?
of the particles are vA and vB respectively and the trajectories
e are as shown in the figure. Then
(a) vxiˆ + v y ˆj + v y Bt kˆ
m
e A
(b) vxiˆ v y ˆj v Bt kˆ
m y
e B
(c) vxiˆ + v y + v y B t ÷ ˆj
m
e
(d) vxiˆ + v y + v y Bt ÷ iˆ + v y ˆj (a) mAvA < mBvB (b) mAvA > mBvB
m (c) mA < mB and vA < vB (d) mA = mB and vA = vB

MARK YOUR 7. 8. 9. 10. 11.


RESPONSE 12. 13. 14. 15. 16.
MAGNETIC EFFECT OF CURRENT 531

17. Three infinitely long wires are placed equally apart on the (a) angle subtended by charged particle at the centre of
circumference of a circle of radius a, perpendicular to its circular path is 2 .
plane. Two of the wires carry current I each, in the same (b) the charge will move on a circular path and will come
direction, while the third carries current 2I along the direction out from magnetic field at a distance 4d from the point
opposite to the other two. The magnitude of the magnetic of insertion.
indution B at a distance r from the centre of the circle, (c) the time for which particle will be in the magnetic field
for r > a, is is 2 / b.
2 0 I (d) the charged particle will subtend an angle of 90° at the
(a) 0 (b) centre of circular path
r
21. An equilateral triangular loop is made up of uniform wire as
I 2 0
2 0 Ia
(c) – (d) shown in figure.
r r2
18. A long cylindrical wire kept along z-axis carries a current of
density j = J 0 rkˆ , where J0 is a constant and r is the radial
distance from the axis of the cylinder. The magnetic
induction B̂ inside the conductor at a distance d from the
axis of the cylinder is

ˆ 0 J0d ˆ
(a) 0 J0 (b)
2
2 3 i i
0 J0 d ˆ 0 J0d ˆ
( c) (d) – A current I enters through one of the vertices of triangle
3 4
and exits from other vertices of the triangle of side ‘a’ as
19. A circular arc QTS is kept in an external magnetic field B0 as shown.
shown in figure. The arc carries a current I. The magnetic The magnitude of magnetic field at the centre of the
field is directed normal and into the page. The force acting equilateral triangle loop due to itself is
on the arc is 3 0I 9 0I
T (a) (b)
2 a 2 a
× B× × × × × × × × × × × ×
0 I 3 3 0I
× × × × × × × × × × × × × (c) (d) none of these
× × ×
Q
× × × × × × × × S× × 2 a
60° R
× × × × × × × × × × × × × 22. An ionized gas contains both positive and negative ions. If
R it is subjected simultaneously to an electric field along the
(a) 2IB0 Rkˆ (b) IB0 Rkˆ +x-direction and a magnetic field along the +z-direction, then
(c) –2 IB0 Rkˆ (d) – IB0 Rkˆ (a) positive ions deflect towards +y-direction and negative
ions towards -y direction
q (b) all ions deflect towards +y-direction
20. If a charged particle of charge to mass ratio is
m (c) all ions deflect towards –y-direction
entering in a magnetic field of strength B at a speed
(d) positive ions deflect towards –y-direction and negative
v = (2 d) (B), then which of the following is correct
ions towards + y-direction.
4d
23. An particle is moving along a circle of radius R with a
× × × × ×× × × constant angular velocity . Point A lies in the same plane
× × × × ×× × ×
× × × × ×× × × at a distance 2R from the centre. Point A records magnetic
× × × × ×× × × field produced by particle. If the minimum time interval
q/ m= × × × × ×× × ×
× × × × ×× × × between two successive times at which A records zero
× × × × ×× × ×
× × × × ×× × × magnetic field is t, the angular speed , in terms of t is –
× × × × ×× × ×
× × × × ×× × × (a) 2 /t (b) 2 /3t
× × × × ×× × ×
× × × × ×× × × (c) /3t (d) /t
× × × × ×× × ×

MARK YOUR 17. 18. 19. 20. 21.


RESPONSE 22. 23.
532 IIT-JEE PHYSICS Challenger
24. Three infinite current carrying conductors are placed as
shown in figure. Two wires carry same current while current B
in third wire is unknown. The three wires do not intersect
with each other and all of them are in the plane of paper. i1
Which of the following is correct about a point P which is
also in the same plane
P P m. Q

OP=x
i i A –
2mg mg
(a) (b)
µ0 i1 2µ0 i1

mg mg
O Third wire (c) (d)
4µ0 i1 µ0 i1

27. A non-planar loop of conducting wire carrying a current I is


placed as shown in the figure. Each of the straight sections
of the loop is of length 2a. The magnetic field due to this
(a) magnetic field intensity at P is zero for all values of x loop at the point P(a, 0, a) points in the direction
2I z
(b) if the current in the third wire is (left to right)
sin
then magnetic field will be zero at P for all values of x y
I
2I
(c) if the current in the third wire is (right to left)
sin
then magnetic field will be zero at P for all values of x
x
(d) none of these
25. A positive charged particle of mass m and charge q is projected 2a

( ) ( )
with velocity v as shown in the figure. If radius of curvature
of charged particle in magnetic field is R (2d < R < 3d), then
1 ˆj + kˆ 1 ˆj + kˆ + iˆ
(a) (b)
time elapse by charged particle in magnetic field regions is 2 3

× × × × × × (c)
3
(
1 ˆ ˆ ˆ
i + j+k ) (d)
1 ˆ ˆ
2
i +k( )
× × × × × ×
× × × × × × 28. In the figure shown a coil of single turn is wound on a
B B sphere of radius R and mass M. The plane of the coil is
× × × × × × parallel to the plane and lies in the equatorial plane of the
v
× × × × × × sphere. Current in the coil is i. The value of B if the sphere is
90° in equilibrium is
× × × × × ×
d d d
B
m m 2d
(a) (b) sin –1 ÷
2qB qB R

m m d
(c) (d) sin –1 ÷
qB qB R
26. A metallic rod PQ is hinged at point P and it can rotate about
point P in vertical plane as shown in the figure. If mass of mg cos mg
(a) (b)
rod is m and length , then the current in PQ, such that it iR iR
remains in equilibrium as shown. (Separation between P and mg tan mg sin
current currying conductor AB is very small) (c) (d)
iR iR

MARK YOUR
24. 25. 26. 27. 28.
RESPONSE
MAGNETIC EFFECT OF CURRENT 533

29. A line charge per unit length is pasted uniformly on to the (a) a (b) b
rim of a wheel of mass m and radius R. The wheel has light (c) c (d) d
non-conducting spokes and is free to rotate about a vertical 32. The magnetic force acting on the rod ABC in the presence
axis as shown in figure. A uniform magnetic field extends of external magnetic field as shown in the figure is :
over a radial region given by
B = – B0 k (r a; a < R) B
= 0 (otherwise) z
60°
l l
I I
× × × ×
× × × ×
R A C
What is the angular velocity of the wheel when this field is
suddenly switched off (a) BI (b) 2BI

–2B a 2 r – B a2r (c)BI 3 (d) Zero


(a) k (b) k
mR 3mR 33. A current carrying circular coil of single turn of mass m is
hanging by two ideal strings as shown in the figure. A
– B a2 –B a 2
(c) k (d) k constant magnetic field B is set up in the horizontal
2mR mR
direction. Then the ratio of tension (T1/T2) in the string will
30. A particle of mass m and charge q moves with a constant
velocity v along the positive x-direction. It enters a region mg
be [given BIR = and = 45º]
containing a uniform magnetic field B directed along the 4
negative z-direction, extending from x = a to x = b. The
minimum value of v required so that the particle can just
enter the region x > b is
z
qbB q (b a ) B
(a) (b)
m m

qaB q (b + a ) B T1 T2
(c) (d)
m 2m B
31. An electron is released from rest at origin O in a region of
space where a uniform electric field E exists along y-axis I
and uniform magnetic field B is pointing out of the page.
Which path in the figure best represent the motion of the
O
electron after it is released x
Y
b

–y
a c

X (a) 2 : 1 (b) 5 : 3
O
d (c) 4 : 1 (d) 1 : 2

MARK YOUR
29. 30. 31. 32. 33.
RESPONSE
534 IIT-JEE PHYSICS Challenger
34. For a positively charged particle moving in a x-y plane initially 36. The magnetic field at the mid- point along the length of a
along the x-axis, there is a sudden change in its path due to solenoid is B0. If the solenoid is stretched to double its
the presence of electric and/or magnetic fields beyond P. length and current through the coil is also doubled. Then
The curved path is shown in the x-y plane and is found to be the new magnetic field at the new mid point is close to
non-circular. Which one of the following combinations is
possible? B0
(a) B0 (b)
2

y (c) 2B0 (d) 4B0

37. A uniform and constant magnetic field ‘B0’ is kept at 30°


with X-axis. Now a square loop is kept in the position as
P x shown in figure. If the current in loop is in anti clockwise
sense, the torque acting on the loop is along
y
D C

B0
(a) E = 0; B = biˆ + ckˆ (b) E = aiˆ; B = ckˆ + aiˆ
30°
x
(0, 0)
(c) E = 0; B = cjˆ + bkˆ (d) E = aiˆ; B = ckˆ + bjˆ

35. The magnetic field at point ‘C’ due to current flowing in ‘M’
shape figure is : A B
2m

(a) x-axis
B l D (b) y-axis

30° 30° (c) AC

(d) a line at 120° with + x-axis


i
C 38. A charged particle moves in a uniform magnetic field with
i
30° its velocity vector making an angle of 120° with the direction
30°
of the magnetic field. The path of the particle will be (ignore
A E any effects due to radiation)

(a) a circle

(b) a helix with uniform pitch


0 3i 0 i
(a) . (b) . 3 (c) a helix with non – uniform pitch
2
(d) none of the above.

0 i
(c) zero (d) .
4 3

MARK YOUR
34. 35. 36. 37. 38.
RESPONSE
MAGNETIC EFFECT OF CURRENT 535

39. A magnetic field B = B0 Jˆ , exists in the region a < x < 2a,

and B = – B0 ˆj , in the region 2a < x < 3a, where B0 is a


positive constant. A positive point charge moving with a
B
velocity v = v0iˆ , where v0 is a positive constant, enters the
magnetic field at x = a. The trajectory of the charge in this
region can be like (a) clockwise, smaller, counter-clockwise, larger
(b) counter-clockwise, smaller, clockwise, larger
B0 (c) clockwise, larger, counter-clockwise, smaller
(d) counter-clockwise, larger, clockwise, smaller
41. A non-conducting non-magnetic rod having circular cross-
0 x section of radius R is suspended from a rigid support as
a 2a 3a
shown in the figure. A light and small coil of 300 turns is
wrapped tightly at the left end of the rod where uniform
B0 magnetic field B exists in vertically downward direction. Air
of density hits the half of the right part of the rod with
z velocity v as shown in the figure. What should be current in
clockwise direction (as seen from O) in the coil so that rod

(a) x 2 RB
a 2a 3a remains horizontal ? Given = 1A 1/ 2 .
Lv
/////////////////// air

z v
O
B
(b) x L L/2 L/2
a 2a 3a (a) 10mA (b) 20 mA
(c) 15mA (d) 5mA
42. Consider six wires into or out of the page, all with the same
current. Rank the line integral of the magnetic field (from
z most positive to most negative) taken counterclockwise
around each loop shown.
loop C
(c) x loop B
a 2a 3a

a 2a 3a
(d) x
loop A loop D
(a) B > C > D > A (b) B > C = D > A
(c) B > A > C = D (d) C > B = D > A
40. An electron and a proton each travel with equal speeds 43. Two particles A and B having equal charges after being
around circular orbits in the same uniform magnetic field as accelerated through the same potential difference, enter a
indicated (not to scale) in the diagram. The field is into the region of uniform magnetic field and describe circular paths
page of the diagram. The electron travels ............... around of radii R1 and R2 respectively. The ratio of mass of A to that
the ............ circle and the proton travels ............... around the of B is
............... circle. (a) (R1/R2)2 (b) (R1/R2)1/2
(c) (R2/R1) 3/2 (d) (R1/R2)

MARK YOUR
39. 40. 41. 42. 43.
RESPONSE
536 IIT-JEE PHYSICS Challenger
44. A conducting loop carrying a current I is placed in a uniform
47. A charged particle is moving in the presence of electric ( E)
magnetic field pointing into the plane of the paper as shown.
The loop will have a tendency to and magnetic ( B ) fields. The directions of E and B are
B Y such that the charged particle moves in a straight line and
its speed increases. The relations amongst E, B and
x
velocity V must be such that
I
(a) E.B = 0, V is arbitrary

(b) E , B and V are all parallel to each other


(a) contract (b) expand
(c) move towards +ve x-axis(d) move towards -ve x-axis.
(c) E.V = 0, BV
. = 0 but E.B 0
45. A direct current flows in a long solenoid of radius R,
(d) V is parallel to E and perpendicular to B
producing a magnetic field of magnitude B0 inside the
solenoid. Now half of the solenoid is removed and a “semi- 48. A slightly divergent beam of non-relativistic charged
infinite” portion remains as shown in figure. Which of the particles accelerated by a potential difference V passes
following statements is/are correct regarding its end face ? through a point A at the axis of a long straight solenoid. The
beam is brought to focus at a distance from A at two
successive values of magnetic induction B1 and B2. Find
the specific charge q/m of particles.
P B0

R V
A

(a) Magnetic flux becomes half because magnetic field at


P becomes B0/2
(b) Magnetic flux becomes half because magnetic field at 2 2V 4 2V
(a) 2 (b)
all points of disc become B0/2 ( B2 B1 )2 2
( B2 B1 )2
(c) Magnetic field at all points becomes B0/2 but it does
not point along axis of solenoid. 8 2V 8 2V
(d) Magnetic flux becomes half of that due to infinite (c) (d)
solenoid due to symmetry consideration.
2
( B2 + B1 ) 2 2
( B2 B1 )2
46. The figure shows two long wires carrying equal currents I1 49. An electron travelling with a speed u along the positive x-
and I2 flowing in opposite directions. Which of the arrows
labeled A to D does correctly represent the direction of the axis enters into a region of magnetic field where B = –B0 k̂
magnetic field due to the wires at a point located at an equal (x > 0). It comes out of the region with speed v then
distance d from each wire ?

I1 I2 ×B
y
d
e– u
B x
d d
A C

D
(a) v = u at y > 0 (b) v = u at y < 0
(a) A (b) B
(c) v > u at y > 0 (d) v > u at y < 0
(c) C (d) D

MARK YOUR 44. 45. 46. 47. 48.


RESPONSE 49.
MAGNETIC EFFECT OF CURRENT 537

50. Four free-particles with the same negative charge and the 52. The figure shows four different set of wires that cross each
same initial speed are incident on the same region of constant other without actually touching. The magnitude of the
magnetic field pointed in the +y-direction (as shown). Rank current is the same in all four cases, and the directions of
the z-component of the force due to the magnetic field on current flow are as indicated. For which configuration will
the particle from most positive to most negative. (The +z the magnetic field at the center of the square formed by the
direction points out of the page.) wires be equal to zero ?

I I
v4
v1
I I
I I
y B
(a) (b)
I I
x v3

I I
v2

(a) Fzv1 = Fzv4 > Fzv3 > Fzv2 I


I
I
I
(c) (d)
(b) Fzv2 > Fzv3 > Fzv1 > Fzv4
I I
(c) Fzv1 > Fzv2 > Fzv3 > Fzv4
53. A long straight wire along the Z-axis carries a current I in the
(d) Fzv4 > Fzv3 > Fzv2 > Fzv1
negative X-direction. The magnetic vector field B at a point
51. When a charged particle moving with velocity v enters a
having coordinates (x, y) in the Z = 0 plane is
region containing a perpendicular magnetic field, it moves
along a semi-circular path of radius r as shown in the figure.
Consider the following two statements 0I ( yiˆ xjˆ ) 0I ( xiˆ + yjˆ )
(x )
(a) (b)
(I) The radius r of the semicircle is proportional to the
initial speed v
2 (x 2
+ y2 ) 2 2
+ y2

( xjˆ )
(II) The time required for the particle to transverse the
semicircle is independent of v. 0I yiˆ 0I ( xiˆ yjˆ )
(x ) (x )
(c) (d)
Then, 2 2
+y 2
2 2
+y 2

V
q 54. The current density J inside a long, solid, cylindrical wire
of radius a = 12 mm is in the direction of the central axis, and
2r its magnitude varies linearly with radial distance r from the

J r 105
axis according to J = 0 , where J 0 = A/m 2 . Find the
a 4

(a) I is true, but II is incorrect a


magnitude of the magnetic field at r = in µT..
(b) I is incorrect, while II is true 2
(c) I is true, II is also and the two statements are (a) 10µT (b) 4µT
independent
(c) 5µT (d) 3µT
(d) I is true, II is also true and II is the cause of I.

MARK YOUR
50. 51. 52. 53. 54.
RESPONSE
538 IIT-JEE PHYSICS Challenger
55. A positively charged disk is rotated clockwise as shown in
NI b 0 IN a
the figure. What is the direction of the magnetic field at lno
(d) 2 b a ln b
point A in the plane of the disk.
(c)
2 (b a ) a ( )
57. A uniform magnetic field is directed out of the page. A
charged particle, moving in the plane of the page, follows a
+ + + + + clockwise spiral of decreasing radius as shown.
+ + + + + +
+ + + + + + + A
+ + + + + + + + Particle
+ + + + + + +
+ + + + + +
+ + + + +

(a) (into the page) (b)


(c) (d) (out of the page)
56. A coil having N turns is wound tightly in the form of a spiral B
with inner and outer radii a and b respectively. When a
A reasonable explanation is
current I passes through the coil, the magnetic field at the
(a) the charge is positive and slowing down
center is
(b) the charge is negative and slowing down
NI
o 2 o NI (c) the charge is positive and speeding up
(a) (b)
b a (d) the charge is negative and speeding up

MARK YOUR
55. 56. 57.
RESPONSE

2. The magnetic field at a distance r from the conductor when


PASSAGE-1
a < r < b is

(a) 0 2ir (b) 0 2ir


A coaxial line carries the same current i up the inside conductor of . .
2
radius a as down the outer conductor of inner radius b and outer 4 a2 4 b a2
radius c. The magnetic induction field at distance r from the i 2i
(c) 0 (d) 0
conductor . . .
2 r a b2 4 2

3. The magnetic field at a distance r from the conductor when


1. The magnetic field at a distance r from the conductor when
b<r< c
r < a is
0i (r 2 b 2 ) 0i (r 2 b 2 )
0 2ir 2 0 2ir (a) 1 (b) 1+
(a) . (b) . 2 r (c 2 b 2 ) 2 r (c 2 b 2 )
4 a2 4 a2

2i 2 r 0i (r 2 b2 ) 0i (c 2 r2 )
0 2i 2 r 2 0 (c) 1 (d) 1
(c) . (d) . 2
2 r 2
(c 2 b 2 ) 2 r (r 2 b2 )
4 a2 4 a

MARK YOUR
1. 2. 3.
RESPONSE
MAGNETIC EFFECT OF CURRENT 539

7. If the magnetic field is increased by 2%, the speed will


PASSAGE-2
(a) increase by 2% (b) increase by 1%
(c) decrease by 2% (d) decrease by 1%
A conducting wire is bend into a loop as shown in the figure. The 8. In the above question, the charge will
segments AOB is parabolic given by the equation y2 = 2x while (a) increase by 2% (b) increase by 1%
segment BA is a straight line parallel to the y-axis. (c) decrease by 2% (d) decrease by 1%
9. Choose the correct option
The magnetic field in the region is B = –8k and the current in the
(a) Both charge and mass can be constant
wire is 2A. (b) If mass is constant, charge must change
(c) If mass is constant, charge must increase
y
(d) None of these are correct.
A

O C x PASSAGE-4
2m

B As we know that the magnetic force on any current carrying wire

is given by F (
i d B ) id B sin . A magnetic field is
4. The torque on the loop will be
present as B 0 (i ) in the space of cube and a cube is made up of
(a) 16 2 Nm (b) 16 Nm
rods in which currents are flowing as shown in figure. The magnetic
(c) 18 2 Nm (d) Zero
flux for any closed figure is given by = B.S = | B || S | cos where
5. The field created by the current in the loop at point C will be
µ0 µ0 ‘ ’ is angle between B and area vector S .
(a) – k (b) – k
4 2 Y
F i B0i
µ0 2 X
G
(c) – k (d) None of these
E B
6. If the loop were rotated about the z-axis with an angular H
velocity of 1 rad/sec. then magnitude of emf induced across C
the straight segment AC will be A
(a) 32 V (b) 8 V D Z
i
(c) 16 V (d) 4 V
10. The magnetic force acting on the wire AD is along
PASSAGE-3 (a) i (b) j

(c) k (d) j + k
The central cross-section of a long cylindrical region containing
11. The magnetic flux through the diagonal plane AEGC is (if
uniform but time varying magnetic field B is shown in the figure. A
particle of unknown constant mass and variable positive charge side length of cube is 2m)
moves in a plane in such a way so that the radius of the circles (a) zero (b) 4B0
remains constant. (c) 4 2B0 (d) 2 2B0
v
12. The magnetic force on the rod segment EFG will be (when
all the 12 rods have equal resistance)
(a) 2B0i( j) (b) 2B0 i(k)
q, m
2 2 2B0
(c) B0i( j) (d) i( j)
3 3

MARK YOUR 4. 5. 6. 7. 8.
RESPONSE 9. 10. 11. 12.
540 IIT-JEE PHYSICS Challenger
15. What height each of the rod will rise if the discharge current
PASSAGE-5
is for a very small time
We know that any current carrying wire produces a magnetic
(a) 10 cm (b) 20 cm
field in the space surrounding it. If another current carrying wire
is present they will experience a magnetic force given by (c) 15 cm (d) 5 cm

2i1i2
F= 0
on a unit meter of each wire. Now capacitor of
4 d PASSAGE-6
C = 10–7 F and Q0 = 2 Coulomb is connected across the two ends
of a infinite long rod arrangement as shown in figure. If d = 1 m A proton, an particle, a doubly ionised Lithium atom (6Li)2+ and
and linear mass density of each rod is = 2 units. The charged triply ionised carbon atom (12C)3+ are projected in a region having
capacitors discharges in a very small time through these rods
uniform magnetic field. Subsequently they exhibit helical motion
each having resistance R 1 . The connecting wires are loose
(or circular motion depending on direction of projection) with
and have no resistance. [assume the ‘d’ remains constant during
radius r1, r2, r3, r4 and pitch P1, P2, P3, P4 respectively.
the time current flows in the circuit] .

d 16. If each is projected perpendicular to the magnetic field with


same kinetic energy, then
(a) r1 = r2 = r3 = r4 (b) r4 > r3 > r2 = r1
+– (c) r1 > r2 > r3 > r4 (d) r3 > r4 > r2 = r1
Capacitor 17. If each is projected with same velocity each at an angle of
13. Then find the initial speed v0 with which the two rods will 45° with respect to the magnetic field, then
move (a) P1 = P2 = P3 = P4 (b) P1 > P2 > P3 > P4
1 (c) P4 > P3 > P2 = P1 (d) P4 > P3 > P2 > P1
(a) 1 m/s (b) m/s
2 18. If each particle were accelerated through the same potential
(c) 2 m/s (d) zero difference and then projected at 45° to the magnetic field
14. The instantaneous force between the two rods will be then
(a) attractive (b) repulsive (a) r4 = r3 = r2 = r1 (b) r1 > r2 > r3 > r4
(c) zero
(c) r4 > r3 > r2 > r1 (d) r4 > r3 > r2 = r1
(d) may be attractive or repulsive

MARK YOUR 13. 14. 15. 16. 17.


RESPONSE 18.

1. Statement - 1 : The magnetic field inside a solenoid of 2. Statement - 1 : For a charged particle to pass through a
uniform electro-magnetic field without
finite length is always less than 0 ni
. change in velocity, its velocity vector must
2 be perpendicular to the magnetic field.
Statement - 2 : The magnetic field inside a solenoid of Statement - 2 : Net Lorentz force on the particle is given
infinite length is 0ni.
by F = q [ E + v B] .

MARK YOUR
1. 2.
RESPONSE
MAGNETIC EFFECT OF CURRENT 541

3. Statement 1 : If the current in a solenoid is reversed in Statement 2 : The electric field exerts no force on the particle
direction while keeping the same magnitude, the magnetic but the magnetic field does.
field energy stored in the solenoid decreases. 5. Statement 1 : A cyclotron cannot accelerate neutrons.
Statement 2 : Magnetic field energy density is proportional Statement 2 : Neutrons are neutral.
to square of magnetic field.
6. Statement 1 : Free electrons always keep on moving in a
4. Statement 1 : If a charged particle is released from rest in a conductor even when no magnetic force act on them in
region of uniform electric and magnetic fields parallel to magnetic field unless a current is passed through it.
each other, it will move in a straight line.
Statement 2 : The average velocity of free electron is zero.

MARK YOUR
3. 4. 5. 6.
RESPONSE

(a) the direction of magnetic field at O due to the wire


1. A particle is released from the origin with a velocity v i . The

electric field in the region is Ei and magnetic field is Bk . frame will be along
–i – j
÷
Then 2
(a) If v = 0 and E = 0 then particle will execute a circular (b) the direction of magnetic field at P (0, a, 0) due to wire
path.
CD and EF will be along ( – j )
(b) If v = 0 and E 0 and particle is positively charged then
it execute clockwise circle as seen from + z direction. (c) the direction of magnetic field at Q (a, 0, 0) due to wire

(c) If v = 0 and E 0 then the particle will execute a cycloidal EF and CD will be along ( – j )
motion in the x-y plane. (d) the direction of magnetic field at P (0, a, 0) due to wire
(d) If v > 0 and E = 0 and the particle is positively charged AF and CB will be along ( –i )
and B > 0 then the particle will execute a clockwise
3. A large plate with uniform surface change density is
circle as seen from + z direction.
moving with constant speed v as shown in the figure. The
2. A wire frame ABCDEF is kept as shown in figure. I is the magnetic field at a small distance from plate is
current in the wire frame.

Y
a B v
A
P
2a C D
O µ0 v
(a) µ0 v in magnitude (b) in magnitude
Q 2a X 2
Z F 2a E
(c) perpendicular to plate (d) parallel to plate

MARK YOUR
1. 2. 3.
RESPONSE
542 IIT-JEE PHYSICS Challenger
4. A particle with charge + q and mass m, moving unde rthe 6. A plane sheet AB having charge density has two holes at
M and N at a separation of d and is placed normal to the
influence of a uniform electric field E i and a uniform
plane of the paper. Two metallic boxes of very large size
magnetic, field Bk , follows a trajectory from P to Q as having magnetic field B are kept on both sides of the sheet
at a distance as shown in figure. A charge particle q at N is
shown. The velocities at P and Q are vi and –2v j given small velocity towards right. Holes on the plane sheet
and holes on boxes are on the same level. The charge particle
y E enters in right box, comes out through the holes and enters
P in the left box at the upper hole. Again comes out from the
lower hole and continues to revolve. Neglect gravity. Choose
B the correct options.

O x
Q

3 mv 2
(a) E= ÷
4 qa

(b) The rate of work done by the electric field at P is

3 mv3
÷ (a) The magnetic field inside the boxes is
2m q
4 a
qd 0m

(c) The rate of work done by the electric field at P is 0


m 0 d
(d) The rate of work done by the both the fields at Q is 0 (b) The time period of revolution is 8+ ÷
q
5. A charged particle with velocity v = xiˆ + yjˆ moves in a

magnetic field B = yiˆ + xjˆ . The magnitude of magnetic force m q


(c) The magnetic field inside the boxes is
qd 0m
acting on the particle is F. Which one of the following
statements are correct ?
m 0 d
(a) No force will act on particle if x = y (d) The time period of revolution is 4+ ÷
q
(b) F µ (x2 – y2) if x > y

(c) The force will act along z-axis if x > y

(d) The force will act along y-axis if y > x

MARK YOUR
4. 5. 6.
RESPONSE
MAGNETIC EFFECT OF CURRENT 543

7. Figure shows a square loop in x, y plane carrying current I (a) x-component of velocity of the proton will be
present in the magnetic field which is given by qBt
vox cos ÷
m
B0 z ˆ B0 y ˆ
B= j+ k where B0 is positive constant. Which
L L qBt
(b) y-component of velocity will be voy cos ÷
m
of the following statement(s) is/are correct ?
qBt
(c) z-component of velocity will be voy sin ÷ along
m
Y
(+ kˆ) direction
I (L,L)
(0,L) (d) x-component of velocity will remains unchanged

I I 9. Two long thin, parallel conductors are kept very close to


each other without touching. One carries a current i and the
X other has charged per unit length. An electron moving
(0,0) I (L,0)
parallel to the conductors is undeflected. Let c = velocity of
light. Then

B0 IL ˆ c2
v=
2 ÷
(a) Force on side (0, 0) to (0, L) is i (a)
i

(b) Force on side (0, L) to (L, L) is – B0 ILjˆ i


(b) v=
(c) Net magnetic force on loop is zero

(d) force on side (L, 0) to (0, 0) is zero i


(c) c=
8. A proton of charge ‘e’ and mass m enters a uniform and

constant magnetic field B = B0iˆ with an initial velocity (d) the electron may be at any distance from the conductor

v = vox iˆ + voy ˆj . Which of the following will be correct at

any later time ‘t’

MARK YOUR
7. 8. 9.
RESPONSE
544 IIT-JEE PHYSICS Challenger
1. Consider no gravity. For a neutral metallic conductor MN, match the following. (The magnetic field exist in a circular region
centered at M.)
Column-I Column -II

B = B0et
(A) (p) Constant (including zero) external work is required

N
V = V0

B = B0e t
(B) (q) Will generate a constant non-zero potential difference between M and N

N
V=0

= 0

B = B0
(C) (r) Will generate a time varying potential difference between M and N.

B = B0
(D) (s) Time varying external work is required.

N
V = V0et

1.

MARK YOUR
RESPONSE
MAGNETIC EFFECT OF CURRENT 545

2. A charged particle having charge q and mass m is to be subjected to a combination of constant uniform magnetic field ( B ) and

a constant uniform gravitational field (G). Apart from these field forces there exists no other force. Now match the column.
Column-I Column -II
(A) The charged particle moves without change (p) It is possible that both B and G are zero.
in its direction.
(B) The charged particle moves without change (q) It is possible that both B and G are non zero.
in its velocity.
(C) The charged particle takes a circular path (r) It is possible that B is zero and G is not zero.
(D) The charged particle takes a parabolic path (s) It is possible that B is non zero and G is zero.
3. A frame ABCD is rotating with angular velcoity about an axis which passes through the point O perpendicular to the plane of
Paper as shown in the figure. A uniform magnetic field B is applied into the plane of the paper in the region as hown. Match the
following.
A D

L × L

B O C
L L
Column-I Column -II
(A) Potential difference between A and O (p) zero
(B) Potential difference between O and D (q) B L2/2
(C) Potential difference between C and D (r) B L2
(D) Potential difference between A and D (s) constant
4. A square loop of side a and carrying current i as shown in the figure is placed in gravity free space having magnetic field
B = B0 k . Now match following.
y
i

Column-I Column -II


(A) Torque on loop (p) is zero
(B) Net force on loop (q) is in direction (– k )
(C) Potential energy of loop (r) has minimum magnitudes
(D) Magnetic moment of loop (s) has maximum magnitudes

2. 3. 4.

MARK YOUR
RESPONSE
546 IIT-JEE PHYSICS Challenger
5. Find correct match for the figure in column I as shown with the items given in column II :
Column I Column II
B
I

(A) (p) F 0
I

B
(B) (q) F 0, 0
I

R 2B

R
(C) I (r) F 0, = 0
R
I

2B B

(D) (s) F 0, = 0
R
I

6. Positively charged particles are moving in x-y plane, where magnetic field is B0 ( k ). Then as per the shown condition
+j
B0(–k)

v1 v3 v4(+k)
v2

+q +q +q +q
(1) (2) (3)
Column I Column II
(A) For v1 (p) Straight line
(B) For v2 (q) Path is semicircular
(C) For v3 (r) Path is minor arc in B
(D) For v4 (s) Path is major arc in B

5. 6.

MARK YOUR
RESPONSE
MAGNETIC EFFECT OF CURRENT 547

7. The entries in Column I depict certain current distributions, while the entries in Column II depict the variation of the magnetic
field (B) as one moves along the x– axis for each of these distributions, but in a different order. Match the entries in Column I with
the proper entries in Column II.
Column I Column II

B
O x
(A) (perpendicular (p)
to wire)
x
Straight current
carrying wire

B
x
(B) O (axis of wire) (q)

Circular current x
carrying wire

B
x
O i (perpendicular to the
(C) (r)
plane of the wires;
i
O being equidistant) x
Parallel current carrying
wires in the same plane

i
B
x
(D) (parallel to one (s)
i of the wires)
x
Two perpendicular current
carrying wires in the same plane

7.

MARK YOUR
RESPONSE
548 IIT-JEE PHYSICS Challenger
8. Column II gives four situations in which three or four semi infinite current carrying wires are placed in xy-plane as shown. The
magnitude and direction of current is shown in each figure. Column I gives statements regarding the x and y components of
magnetic field at a point P whose coordinates are P (0, 0, d). Match the statements in column I with the corresponding figures in
column II.
Column I Column II

i/3
i/3
(A) The x-component of magnetic field at point P is zero in (p) x
i
i/3

y
i/2

45° x
(B) The z-component of magnetic field at point P is zero in (q)
i 45°

i/2

i/2
0i
(C) The magnitude of magnetic field at point P is in (r) x
4 d i
i/2

0i i
(D) The magnitude of magnetic field at point P is less than in (s) x
2 d 45° 45°
i/2
i/2

8.

MARK YOUR
RESPONSE
MAGNETIC EFFECT OF CURRENT 549

9. A charged particle having non-zero velocity is subjected to certain conditions given in column I, column II gives possible
trajectories of the particle. Match the conditions in column I with the results in column II
Column I Column II
(A) In only uniform electric field (p) the path of the charged particle may be a straight line
(B) In only uniform magnetic field (q) the path of the charged particle may be a parabola
(C) In uniform magnetic and uniform electric field (r) the path of the charged particle may be circular
(D) Subjected to a net force of constant magnitude (s) the path of the charged particle may be helical with
uniform or non uniform pitch.
10. A square loop of uniform conducting wire as shown in figure. A current-I (in amperes) enters the loop from one end and leaves
the loop from opposite end as shown in figure. The length of one side of square loop is metre. The wire has uniform cross
section area and uniform linear mass density. In four situations of column-I, the loop is subjected to four different uniform and
constant magnetic field. Under the conditions of column I, match the column-I with corresponding results of column-II. (B0 in
column I is a positive non-zero constant).
y
I/2 I

I/2 I/2

x
I/2
I
Column I Column II
(A) B = B0iˆ in tesla (p) magnitude of net force on loop is 2B0 I newton

(B) B = B0 ˆj in tesla (q) magnitude of net force on loop is zero

(C) B = B0 (iˆ + ˆj ) in tesla (r) magnitude of net torque on loop about its centre is zero

(D) B = B0 kˆ in tesla (s) magnitude of net force on loop is B0I newton


(t) Net force perpendicular to the plane of loop

9. 10.

MARK YOUR
RESPONSE
550 IIT-JEE PHYSICS Challenger
1. A beam of protons with a velocity 4 × 105
m/sec enters a at a height of 0.01 m over another parallel long wire CD
uniform magnetic field of 0.3 tesla at an angle of 60° to the which is fixed in a horizontal plane and carries a steady
magnetic field. Find the radius (in m) of the helical path current of 30A, as shown in figure . When AB is slightly
taken by the proton beam. depressed, then find the period of oscillations (in second).
2. Two long parallel wires carrying current 2.5 amperes and I
A B
ampere in the same direction (directed into the plane of the 20 A
paper ) are held at P and Q respectively such that they are
perpendicular to the plane of paper. The points P and Q are C D
30 A
located at a distance of 5 metres and 2 metres respectively
from a collinear point R (see figure). An electron moving 5. A circular loop of radius R is bent along a diameter and
with a velocity of 4 × 105 m/s along the positive x- direction given a shape as shown in the figure. One of the semicircles
experiences a force of magnitude 3.2 × 10–20 N at the point (KNM) lies in the x-z plane and the other one (KLM) in the
R. Find the value of I (in ampere). y-z plane with their centres at the origin. Current I is flowing
through each of the semi circles as shown in figure.
5m
2m
P Q R
O
X OX ............. x L
2.5A IA
L y
3. An electron gun G emits electrons of energy 2keV travelling M
in the positive x-direction. The electrons are required to hit
L
N x
the spot S where GS = 0.1m, and the line GS makes an angle
of 60° with the x-axis as shown in the fig. A uniform magnetic z
K I
field B exists parallel to GS in the region outside the electron
gun. Find the minimum value of B (in mT) needed to make
the electrons hit S. A particle of charge q is released at the origin with a velocity

S v v0 i. Find the magnitude of instantaneous force F


on the particle if µ0qv0I =8 R. Assume that space is gravity
B free.
90° 6. A proton and an -particle are accelerated with same
x potential difference and they enter in the region of constant
v G
magnetic field B perpendicular to the velocity of particles.
Find the ratio of radius of curvature of proton to the radius
4. A long horizontal wire AB, which is free to move in a vertical of curvature of - particle.
plane and carries a steady current of 20A, is in equilibrium

1. 2. 3. 4. 5. 6.

MARK
YOUR
RESPONSE
MAGNETIC EFFECT OF CURRENT 551

1 (b) 7 (d) 13 (d) 19 (b) 25 (b) 31 (d) 37 (b) 43 (a) 49 (b) 55 (d)
2 (b) 8 (c) 14 (a) 20 (b) 26 (d) 32 (a) 38 (b) 44 (b) 50 (d) 56 (c)
3 (c) 9 (b) 15 (a) 21 (d) 27 (d) 33 (b) 39 (a) 45 (d) 51 (d) 57 (a)
4 (c) 10 (c) 16 (b) 22 (c) 28 (b) 34 (b) 40 (a) 46 (b) 52 (c)
5 (a) 11 (a) 17 (a) 23 (b) 29 (d) 35 (b) 41 (a) 47 (b) 53 (a)
6 (b) 12 (a) 18 (c) 24 (c) 30 (b) 36 (a) 42 (c) 48 (d) 54 (a)

1 (b) 3 (a) 5 (d) 7 (b) 9 (b) 11 (a) 13 (a) 15 (d) 17 (d)


2 (c) 4 (d) 6 (c) 8 (d) 10 (b) 12 (c) 14 (b) 16 (d) 18 (c)

1 (d) 2 (d) 3 (d) 4 (c) 5 (a) 6 (a)

1 (c,d) 2 (a,b,c,d) 3 (b,d) 4 (a,b,c) 5 (a,b,c) 6 (a,b) 7 (a,b,d) 8 (b,d) 9 (a,d)

1. A-r, s; B-p; C-p, q; D-r, s 2. A-p, q, r, s; B-p, q, s; C-s; D-r


3. A-r, s; B-r, s; C-p, s; D-p, s 4. A-p, s; B-p; C-s; D-q
5. A-q; B-r; C-p; D-s 6. A-q; B-s; C-r; D-p
7. A-s; B-q; C-p; D-r 8. A-p, q, r; B-p, q, r, s; C-r; D-p, q, r, s
9. A-p, q; B-p, q, r; C-p, s; D-p, q, r, s 10. A-r, s, t; B-r, s, t; C-q, r; D-p, r

1 0.012 2 4 3 4.74 4 0.2


5 2 6 0.7
552 IIT-JEE PHYSICS Challenger

1. (b) Since, magnetic field is acting at right angles to the Coordinates of the centre are (r cos 60°, – r sin 60°)
direction of motion of electron beams, so their paths where r = radius of circle
will be circular.
mv 1 1 1 3
mv r1 v1 = = 1 i.e., 2 ,
= 2 ÷
r= or r µ v = 1/2 =
qB r2 v2 Bq 1 1
or r1 : r2 = 1: 2 5. (a) Magnetic field
0I I yiˆ xjˆ I ( yiˆ xjˆ)
2. (b) Field due to one side of loop at O = (2sin 45°) 0 0
a =
2 (x + y )
2 2
x +y
2 2
2 ( x2 + y 2 )

4

0 I 0 I
Field at O due to all four sides is along unit vector eˆz 6. (b) B= [sin 90° + sin( )] = (1 sin )
4 a 4 a
I a

45° 45°
I
I I A
O

I (0,0) (a,0)
0I 2 2 0I
Total field = 4. (2 sin 45°) = 0 I b
a a = 4 a 1 ÷
4 ÷ a 2 + b2
2
3. (c) The angular momentum L of the particle is given by
L = mr2 where = 2 n. 7. (d)
D
q
Frequency n = ; Further i = q × n = I2
2 2
2 -
q
Magnetic moment, M = iA = r2;
2 A B

qr 2
M= C
2

M qr 2 q I
So, = 2
= E
L 2mr 2m
4. (c) The centre will be at C as shown :
Magnetic field at the centre due to current in arc
y ABC is
B=1T 0 I1
B1 = (Directed upwards)
4 r
Magnetic field at the centre due to current in arc
x
ADB is
60° 30°
(– 3–1) µ0 I 2
C
B2 = (2 ) (Directed downwards)
4 r
MAGNETIC EFFECT OF CURRENT 553

Therefore net magnetic field at the centre 11. (a) Force on the electron
0 I1 I20 F = e (v B)
B= (2 )
4 r 4 r
= – e [(vx i v y j) Bi] = evyB k
E E EA
Also, I1 = = = Thus acceleration of the electron
R1 1 / A r
ev y B
E E EA az = k
and I2 = = = m
R2 2/A r (2 )
ev y B
EA EA (2 ) Velocity vz = azt = t
B=
0
=0 m
4 r r r (2 ) r
ev y Bt
id r v = vx i + v y j + kˆ
8. (c) B= 0
3 for B1 , r ( iˆ ˆj ) m
4 r
12. (a) FE = qE (Force due to electric field)
i
B1 0
kˆ ( iˆ ˆj ) ............... (1) FB = evB sin = qvB sin 0 = 0
4 2 2
(Force due to magnetic field)
for B2, r = iˆ + ˆj
E
ikˆ (iˆ + ˆj )
B2 = 0
............... (2)
4 2 2 +q
From (1) and (2) B

B1 B2 and | B1 | = | B2 | Force due to electric field will make the charged particle
released from rest to move in the straight line (that of
9. (b) The magnetic field due to two wires at P
electric field). Since the force due to magnetic field is
d zero, therefore, the charged particle will move in a
M P straight line.
i x i 13. (d) Given: f = 24 × 106 Hz, R = 0.60 m

mv
We know that, R =
qB
0i 0i
B1 = ; B2 =
2 ( d + x) 2 (d x) mv
B= ...(i)
Both the magnetic fields act in opposite direction. qR

0i 1 1 where v = R = 2 f R
B B2 B1
2 d x d+x = 2 × 24 × 106 × 0.60 = 9.04 × 107 m/s
From (i),
0i d x d+x 0ix
. 27
= 2 2 = 2 2 (3.34 10 )(9.04 107 )
2 d x (d x ) B= = 3.2 T..
19
1.6 10 0.60
10. (c) Using Ampere's law, we have
2 r
14. (a) Current, I = 2 r n.
r /n
B . d = µ0iin R
2 2
0 Ir 0 Ir
B 2 2 3/ 2 3
2( y r ) 2y
i
or B × 2 r = µ0 2
r2 If y > > r.
R
2
0r 0 r3 n
µ0 i r B 2
(2 r n)
B= . 2 2y y3
2 R
554 IIT-JEE PHYSICS Challenger
15. (a) Note that an expression for magnetic field at a point
18. (c) Current density, J = j0 rkˆ
distant x from a conductor of finite length is to be used.
In this case the integration is to be carried out between Current within a distance d,
angles (90° – /2) and 90°. d d2
2
I= J .ds ( J 0 r.rd dr )(kˆ.kˆ)
i =0
r =0 0 0
ˆ
[ ds krd dr ]
P
r
/2
x d3
O = 2 J0
i 3
Referring to the figure, the magnitude of the magnetic
field at point P will be d3
From Ampere’s Law, B.d 02 J0
B = 2 ( 0/4 )(i/x) [sin (90° – /2) + sin 90°] 3
c
The multiplying factor 2 appears in the above
Here loop c is a circle of radius r, B is the magnetic field
expression as the magnetic fields due to the two parts
at r.
of the conductor are in the same direction. The
perpendicular distance x is obviously (r sin /2) and z
^
hence the result. d3 k
16. (b) When a charged particle is moving at right angles to 2 r.B 0 2 J0 . y
3
the magnetic field then a force acts on it which behaves
as a centripetal force and moves the particle in circular
2
motion. 0 J0d
Br ( =d ) =
3
m Av 2A
= q.v A B 19. (b) We know, F I( B)
rA
or dF IBd sin
mAv A
= qB 120°
rA F IB0 d sin d where, d = rd
60°
mB vB 120° 120°
= qB F IB0 R sin d IBR [ – cos ]60°
rB 60°

1 1
B
= IB0 R – – + ÷ = IB0 R
2 2

Hence, force acting on the arc is IBR kˆ .


20. (b) Electromagnetic force will provide the necessary
A centripetal force.

m Av A mB vB
=
rA rB 2r r
Since rA > rB
mAvA > mBvB
17. (a) From ampere circuital law

B.dl 0i
mv 2
eBv =
r
Where, i = Net current passing through the loop
Now i = I + I – 2I = 0 mv v (2 d )( B)
r= = = = 2d
B. 2 r = 0 eB B (B )
B=0 i.e., the electron will move out after travelling on a
semicircular path of radius r = 2d.
MAGNETIC EFFECT OF CURRENT 555

23. (b) Point A shall record zero magnetic field (due to -


R i
21. (d) iABC = i= particle) when the -particle is at position P and Q as
2R + R 3 shown in figure. The time taken by -particle to go
from P to Q is
2R 2i
iAC = i=
2R + R 3
Magnetic field at point O (centre) P
B

60° 2R A
O 60°
i/3
O
Q
60° 60°

C C 12 2
OD = d D t= or =
2i/3 3 3t
i i
24. (c) Magnetic field at P due to wires (1) and (2) is:
0 (i / 3)
(sin 60° + sin 60°) B1 = 0I
+ 0I
=
2 0I
4 d 2 ( x sin ) 2 ( x sin ) 2 ( x sin )
(out of the paper)
0 (i / 3)
+ (sin 60° + sin 60°) P
4 d
xs
in in
0 (2i / 3)
(sin 60° + sin 60°) = 0 xs
4 d x
22. (c) Case of positively charged particle : (2)
Two forces are acting on the positively charged particle (1)
(a) due to electric field in the positive x-direction.
(b) Force due to magnetic field.

F q (v B )

F q(vi Bk )
2I
Now if a current of is flowing in the third wire
F qvB ( j ) sin
then the magnetic field due to the same will be
This forces will move the positively charged particle
2I
towards y-axis. B2 = 0
÷ , which will cancel B1 if it is inside
Case of negatively charged particle. 2 x sin
Two forces are acting on the negatively charged particle 2I
the paper which is possible if the current in the
(a) due to electric field in the negative x-direction. sin
(b) due to magnetic field third wire is from right to left.
2 m m 2d
F q (v B ) 25. (b) t= = sin –1 ÷
qB 2 qB R
F q[v( i ) B (k )]
× × × × × ×
R
F qvB [i k ] × × × × × ×
× × × × × ×
F qvB ( j ) × × × × × ×
v 2d
Same direction as that of positive charge. × × × × × ×
q
(c) is the correct answer. × × × × × ×
2d
556 IIT-JEE PHYSICS Challenger
Due to symmetry the y-components and x-components
µ0 i1
26. (d) dF = i2 dx. will cancel out each other.
2 x
Similarly the magnetic field due to current in ADEFA
µ0 i1 will be in x-direction.
d = i2 dx.
2 x÷
x The resultant magnetic field will be

1
B B= (i + k ).
2

dF 28. (b)
i1
x 2
M = i( R )
P dx i2 Q
P P
mgcos mg sin mg sin
A – mgcos

µ0
mg ( / 2) = i2 i1 dx
2 For rotational equilibrium of the sphere about point P.
0
(mg sin ) R = M (B sin )
µ0 or, mgR sin = i R2 B sin
mg ( / 2) = i2 i1
2 B = mg/ iR

mg 1 d
i2 = 29. (d) I =0= dtk QRdtkz
µ0 i1 2 R dt

27. (d) If we take individual length for the purpose of – R d –B a 2 Rk


calculating the magnetic field in a 3-Dimensional figure
then it will be difficult. –B a 2
Here a smart choice is divide the loop into two loops. = k
mR
One loop is ADEFA in y-z plane and the other loop will
30. (b)
be ABCDA in the x – y plane.

Z q X
E Y x=a v x=b

I
I
D C Width of the magnetic field region (b – a) < R; where 'R'
F is its radius of curvature inside magnetic field,
X
I mv (b a) qB
R= (b – a) vmin =
A B qB m

31. (d) E = E0 j
We actually do not have any current in the segment
AD. By choosing the loops we find that in one loop we
B = B0 k
have to take current from A to D and in the other one
from D to A. Hence these two cancel out the effect of Therefore, an electric force will act in –ve Y-axis and
each other as far as creating magnetic field at the the magnetic force will tend the electron to move in a
concerned point A is considered. circular path.
The point (a, 0, a) is in the X-Z plane. 32. (a) Force = BI
The magnetic field due to current in ABCDA will be in Here, equivalent length '
+ ve Z-direction.
= 2
+ 2 2 . cos60° 2 2 2
=
MAGNETIC EFFECT OF CURRENT 557

33. (b) z
µ0 i
35. (b) B due to AB = 3
4 d

T1 T2 µ0 i µ0i
= 3 = 2 3
4 /2 4
B
µ0 i µ i
Net magnetic field = 3 2= 0 3
I 2
36. (a) B0 = µ0nI
O n
x B' = µ0n'I' = µ0 ÷ 2I = µ0 nI = B0
2

37. (b) M B0
–y
38. (b) A charged particle projected perpendicular to the
direction of a magnetic field moves along a circular
A R 2 (– sin i cos k ) path, while one projected at an angle with the magnetic
field moves along a helical path with a uniform pitch.
m BIA(– sin i cos k ) ( –i ) 39. (a) For a < x < 2a;

B = B0 ˆj
= – BIA cos j = – BI R2 cos j
Facing torque about O The initial velocity is v = v0 i
mgR cos – 2RT2 cos – BIR2 cos =0
Z
mg
mg – 2T2 =0 Y'
4 F B0
3mg 5m g
T2 = and T1 =
8 8 X
x=a V0
34. (b) The velocity at P is in the X-direction (given).

Let v = mi. From the diagram it is clear that the force on the particle
is towards positive Z-axis (apply Fleming's left hand
After P, the positively charged particle gets deflected
rule) at x = a, which shifts the particle as shown in the
in the x – y plane toward – y direction and the path is
X-Z plane.
non-circular.
For 2a < x < 3a
Now, F q (v B )
B B0 j
F q [mi (ck + ai )] for option (b) The direction of velocity is shown at x = 2a. Again
using Fleming's left hand rule, we get the direction of
= q [mci k mai i ] force. This changes the profile of the particle as shown
by the dotted line.
= mcq ( j )
Alternatively,
Use the vector form of B and v in the formulae
Since in option (b), electric field is also present E = ai,
therefore it will also exert a force in the + X direction. F q (v B) to get the instantaneous direction of
The net result of the two forces will be a non-circular force at x = a and x = 2a.
path.
mv
Only option (b) fits for the above logic. For other option, 40. (a) r= r µ m (v = same for both charge)
we get some other results. qB
558 IIT-JEE PHYSICS Challenger

L 2 m
41. (a) Force exerted by air on the rod = 2R÷ v 2 LRv 2 Similarly, =
2 2 B2 q

Balancing torque about point O, 4 mv


B2 = ........ (2)
q
3L
NI ( R 2 ) B LRv 2
4 2 mv
B2 – B1 =
q
3 v 2 L2
300 IBR = 2 2
4 2 m 2Vq
m ÷
(B2 – B1)2 =
q ÷
L2 v 2
I= = 0.01 A = 10mA
400 BR q 8 2V
=
2
m ( B2 B1 ) 2
42. (c) loop B = µ0 (2i – i) = B.d
49. (b) The force acting on electron will be perpendicular to
loop C = µ0 (i – 2i) = B.d the direction of velocity till the electron remains in the
magnetic field. So the electron will follow the path as
given.
loop A = µ0 (3i – 3i) = B.d
y X B
loop D = µ0 (0 – i) = B.d
e
So, B > A > C = D u x

mv 2
43. (a) = qvB
r

mv 2m( KE ) 2mqV 2mV 1 50. (d) Force for v4 & v3 is along +ve z, for v2 it is zero. Force
r= = = = on v1 is –ve. Between v4 & v3, force on v4 is greater in
qB qB qB q B
magnitude as v4 B .
R1 M1 M 1 R12
= 51. (d) If r is proportional to v, t is independent of r
R2 M2 M 2 R22 52. (c) Magnetic field due to clockwise currents = Magnetic
field due to anticlockwise currents
44. (b) Use Fleming's left hand rule, we find that a force is
53. (a) The wire carries a current I in the negative z-direction.
acting in the radially outward direction throughout the
We have to consider the magnetic vector field
circumference of the conducting loop.
45. (d) Magnetic flux becomes half of that due to infinite B at (x, y) in the z = 0 plane.
solenoid due to symmetry consideration. Magnetic field B is perpendicular to OP.
46. (b) I1 I2 B B sin iˆ B cos ˆj
d
B y x I
d d sin , cos = B = 0
B2 B1 r r 2 r
30° 30° y
B 0I
2 r2
(
yiˆ xjˆ ) P(x, y)
47. (b) E increases the speed
r
( yiˆ ).
V and E are parallel and V || B
0I xjˆ x
B= O×
( )
48. (d) Charged particles are focused at the same point on the or
screen when pitches of the helical paths followed by 2 x +y 2 2

them in magnetic fields B1, B2 are and /2 respectively 54. (a) Current in the element = J (2 r. dr)
where is the distance of the screen from the source of Current enclosed by ampere loop of radius a/2
the charged particles.
a/2
J0r 2 J0 a 3 J 0 a3
2 mv I .2 r.dr = ÷ =
B1 = ........ (1) 0
a 3a 2 12
q
Applying ampere’s law
MAGNETIC EFFECT OF CURRENT 559

N
dr Number of turns per unit length =
b a
r N
Number of turns in thickness dx = dx
b a
Small amount of magnetic field is produced at O due to
thickness dx of the wire.
a J0a2 J a
B.2 . 0. B= 0 0 NI dx
2 12 12 0
dB =
On putting values, B = 10µT 2 (b a) x
On integrating, we get,
b 0 NI dx NI
55. (d) For circular wire B= a = 0
× 2 b a x 2 (b a)

b dx NI
= 0
[log e x]ba
a x 2 (b a)
56. (c) Let us consider a thickness dx of wire. Let it be at a
distance x from the centre O.
0 NI b
B= log e
2 (b a) a

mv
dx 57. (a) R= ; F = q (v B)
qB
x As R decreases, v will decrease.
a Clockwise motion for upward magnetic field +ve
O
b

(For Sol. 1-3) 1. (b) When r < a


According to the figure let the current be distributed Consider a co-axial circular path of radius r. Let B be
uniformly over the cross-sections of outer and inner the magnitude of magnetic field at this distance, then
B.2 r 0 current enclosed by path
i
conductors. Current density in inner conductor =
a2 i 0 ir
2
0 r2 ÷ = ,
a2 a 2

(using Ampere’s law)

0 2ir weber
B
a 4 a 2 metre 2
2. (c) When a < r < b
b c In this case the circular path of radius will enclose the
current passing through inner conductor. Using
Ampere’s law.

i 0i
Current density in outer conductor = B2 r oi or B = weber/metre2
(c 2 2
b ) 2 r÷
560 IIT-JEE PHYSICS Challenger
3. (a) When b < r < c
dv dq dB
Here, current enclosed by co-axial circular path of radius = +
r. v q B
Current passing through inner conductor-current
dq 1 dB
passing through portions of outer conductor lying =– = 1%
q 2 B
between r = b and r = c (–ve sign is used because the
currents in two conductors are in opposite direction. 10. (b) The magnetic force is along the plane containing x and
By Ampere’s law z-axes; i.e. along +ve y-axis.
11. (a) Take the area component which is perpendicular to
i
B2 r o i (r 2 b2 ) B0 . Therefore, = BS cos 90° = 0
(c 2 b2 )
12. (c) F (i B)
0i (r 2 b2 )
B 1
2 r (c 2 b 2 ) µ0 Q02 10 7
4
13. (a) v0 = . = 7
= 1m/s
4 2 Rcd 2 10 1 2
4. (d) Since M B Torque is zero.
14. (b) Apply F | B | i | | sin .
5. (d) The field must be in + k direction.
15. (d) v2 = 2gh
6. (c) The emf is the difference between emf across straight
12 = 2 × 10 × h h = 5 cm
segment OA and OC.
7. (b); 8. (d); 9. (b) mv 2m ( KE )
16. (d) r= =
d qB qB
E 2 R (B R2 )
dt
p+ 2+ 6
Li 2 + 12
C3+
R dB
E= m 1 4 6 12
2 dt
q +1 +2 +2 +3
dv
Now, qE = m
dt m
rµ ; r1 = r2 < r4 < r3
q
q dB dv 1 m
R. =m dB = dv
qR ÷
17. (d)
2 dt dt 2
2 mv cos m
dv 1 dB P Pµ P1 < P2 < P3 < P4
= mv qB q
....(i) [using R = ]
v 2 B qB 18. (c)
1
2% = 1% mv sin 2mqv m
2 r= = sin rµ
qB qB q
mv
Also, R = r1 < r2 < r3 < r4
qB

1. (d) The magnetic field inside a solenoid of finite length is proportional to the square of the magnitude of the
µ0 ni magnetic field.
B= [sin sin ]
2 4. (c) Due to electric field, the force is F = qE in the direction
2. (d) No net force will act on charged particle if of E . Since E is parallel to B , the particle velocity v
F = q [ E + v B] = 0 (acquired due to force F ) is parallel to B . Hence B will
E v B v need not to be perpendicular to B not exert any force since v B = 0 and the motion of the
3. (d) Reversing the direction of the current reverses the
particle is not affected by B .
direction of the magnetic field. However, it has no effect
on the magnetic-field energy density, which is 5. (a) Neutrons are neutral.
MAGNETIC EFFECT OF CURRENT 561

6. (a) In the absence of the electric current, the free electrons magnetic field. On passing the current, the free electrons
in a conductor are in a state of random motion, like acquire drift velocity in a definite direction, hence
molecule in a gas. Their average velocity is zero. i.e. they magnetic force acts on them, unless the field has no
do not have any net velocity in a direction. As a result, perpendicular component.
there is no net magnetic force on the free electrons in the

1. (c, d) Refer to the theory of motion of charged particles


3 mv 2 3 mv3
in magnetic fields and simultaneous electric and = qE × v = qv 4 qa ÷ = 4 a ÷
magnetic fields.
F = q ( E + v B) At Q, v is perpendicular to E & B and hence
no work done by either field.
2. (a, b, c, d) Direction of magnetic field at P due to CD
5. (a, b, c) If x = y then v || B , i.e. Fm = 0 .

i
( j + kˆ ) kˆ – j Hence, the option (a) is correct.
2 2 F q (v B )
Direction of magnetic field at P due to EF
= q [( xiˆ + yjˆ) ( yiˆ + xjˆ)] = ( x 2 y 2 ) kˆ

–i
( j – kˆ) – kˆ – j Now if x > y, F ( x 2 y 2 ) and F is along +ve z-
2 2 axis. But if y > x, force will be along negative z-
axis.
Hence direction will be along – j The option (b) and (c) are also correct.
Direction of magnetic field at O due to upper frame 6. (a, b) For the charge particle to continue to revolve,
radius of circular path inside the magnetic field
is along –i should be
Direction of magnetic field at O due to horizontal d mv 2mv
r= = B=
frame will be along – j 2 qB qd
dx Electric field due to plane sheet is E =
3. (b, d) Current flowing per unit length v 2 0
dt
There will be no electric field inside the box
because box is metallic. Velocity of charge particle
when it reaches to the surface of the box.
qE q
v= 2 =
m m 0
and time taken by charge particle to move a
By Ampere’s law
distance in electric field is
µ0 v 2 4m 0
B + B = µ0 v B= t1 = =
2 qE q
4. (a, b, c) In going from P to Q increases in K.E. m
1 1 1 Time taken by charge particle to move semicircle
= m(2v 2 ) – mv 2 = m(3v 2 ) inside the box is
2 2 2
= work done by electric field. r d 0m
t2 = =
[ Magnetic field never do any work on moving v 2 q
charged particle.] Time period of revolution is
m 0 d
3 2 3 mv 2 T = 4t1 + 2t2 = 8+ ÷
or, mv q[ E 2a ] E = ÷ q
2 4 qa
The rate of work done by E at P = force due to 2mv 2m q
and magnetic field is B = =
E × velocity qd qd 0m
562 IIT-JEE PHYSICS Challenger
y-component of velocity is voy cos t and z-
7. (a, b, d) dF (d B)
component of velocity along negative z-direction
L
B0 y IB0 L2 B0 IL ˆ qB
F I dy = = (i ) at an any time t is voy sin t, where =
0
L L 2 2 m
(0,L) 9. (a, d) At P, the electric field

E= (to the right)


(0,0) 2 0x
i v
dF2 I ( L B) I L B0 and magnetic field
B0LI in (–)ve y-direction.
0i
8. (b, d) The x-component of velocity, being parallel to B= (into the paper) x P
magnetic field, shall remain unchanged. 2 x
The component of For no deflection
velocity perpendicular E
to x-axis will always E = vB or v =
B
have magnitude voy, and
at any time t it shall 2 x 1 c2
make an angle = t or v = . =
voy 2 0x 0i i 0 0 i
with y-axis as shown.

1. A-r, s; B-p; C-p, q; D-r, s µ0i/2


µ0i/2
Because as potential difference increased self energy (r)
4 d 4 d
increases, which comes from external work.
As P.D. = Bv
2. A-p, q, r, s; B-p, q, s; C-s; D-r
µ0i
3. A-r, s; B-r, s; C-p, s; D-p, s
4. A-p, s; B-p; C-s; D-q 4 d
5. A-q; B-r; C-p; D-s
0i
B= ( j)
dF Id B; µ B 4 d
µ0i
6. A-q; B-s; C-r; D-p µ0i/2
4 d 4 2 d
F q (v B )
7. A-s; B-q; C-p; D-r (s)
8. A-p, q, r; B-p, q, r, s; C-r; D-p, q, r, s
µ0i/2 µ0i
µ0i
i/3 4 d 4 d
4 d
i/3 0i 0i /3
(p) x B= + ( ˆj) ;
i 4 d 4 d÷ 0i
( iˆ) ( ˆj) 0i
4 2 d 4 d
i/3 9. A-p, q; B-p, q, r; C-p, s; D-p, q, r, s
x, z component zero (A) If velocity is along electric field path will be straight
line otherwise parabolic.
(B) In magnetic field straight line, parabolic, circular paths
are possible.
µ0i/2 µ0i/2 (C) If qE = qvB then path will be straight line and if electric
0i 0i
(q) B= 2 cos 45°÷ + field and magnetic field parallel to each other then path
4 d µ0i 4 d 8 d 4 d will be helical.
4 d (D) If any net constant force like gravity, electric force etc.
then path could be straight line, parabola, circular and
0i 1 helical (helical path is possible if magnetic field is
1 ( j)
4 d 2 applied).
MAGNETIC EFFECT OF CURRENT 563

10. A-r, s, t; B-r, s, t; C-q, r; D-p, r (C) Since net displacement of current from entry point in
(A) Because the magnetic field is parallel to x-axis, the force the loop to exit point in the loop is along the diagonal
on wire parallel to x-axis is zero. The force on each wire of the loop. The direction of external uniform magnetic
field is also along the same diagonal. Hence net force
I
parallel to y-axis is B0 . Hence net force on loop is on the loop is zero. Since force on each wire on the
2 loop passes through centre of the loop net torque about
B0I , to loop plane (downward). Since force on each centre of the loop is zero.
wire parallel to y-axis passes through centre of the loop (D) The net displacement of current from entry point in
net torque about centre of the loop is zero. the loop to exit point in the loop is along the diagonal
(B) Because the magnetic field is parallel to y-axis, the force
on wire parallel to y-axis is zero. The force on each wire (of length 2 ) of the loop. The direction of external
uniform magnetic field is also perpendicular to the
I same diagonal. Hence magnitude of net force on the
parallel to x-axis is B0 . Hence net force on loop is
2
loop is B0 I 2 . Since force on each wire on the loop
B0I , to loop plane (upward). Since force on each
wire parallel to x-axis passes through centre of the passes through centre of the loop net torque about
loop, net torque about centre of the loop is zero. centre of the loop is zero.

1. 0.012 The force experienced by the electron


v1 is responsible for horizontal motion of proton
F = qvB sin
v2 is responsible for circular motion of proton
µ0
= evB sin 90° = 1.6 × 10–19× 4 × 105 × (1 + I)
4
mv22 But F = 3.2 × 10–20 N (Given)
= qv2 B
r 3.2 × 10–2 = 1.6 × 10–19 × 4 × 105 × 10–7 (1 + I)
I = 4 amp.
27 5 3. 4.74
mv2 1.76 10 4 10 3
r= = 19 = 0.012 m
qB 1.6 10 0.3 2 Let us resolve the velocity to rectangular component v1 (= v
2. 4 cos ) and v2 (v sin 60°), v1 component of velocity is
The magnetic field (due to current in wire P) at R responsible to move the charged particle in the direction of
the magnetic field whereas v2 component is responsible for
µ0 2I p µ0 2 2.5 rotating the charged particle in circular motion. The overall
= path is helical. The condition for the charged particle to
4 rPR 4 5
strike S with minimum value of B is
µ0 Pitch of Helix = GS
= [in the plane of paper downwards]
4
2 m 1
2.5amp Iamp 5 T × v1 = GS × v cos 60° = 0.1 mv2 = E
e v=4×10 m/s qB 2
P Q R +X
2m 2E
v= S B
rPR=5m m
Similarly the magnetic field (due to current is wire Q) at R
µ 2 I µ0 r=0.1m
= 0 = I [in the plane of paper downward] 2 mv cos 60 ° v1=v cos 60°
4 2 4 B=
q 0.1 v
The total magnetic field at R [due to P and Q]
G
v2=v sin 60°
µ0 µ0 µ 2 m 2E
B= + I = 0 (1 + I ) B= cos 60°
4 4 4 q 0.1 m
[in the plane of paper downwards]
564 IIT-JEE PHYSICS Challenger

2 g
= 2mE cos 60° =
q 0.1 r
2 3.14 2 g
= 19 =
1.6 10 0.1 T r
31 1 r 0.01
= 2 9.1 10 2 103 1.6 10 19
T=2 2 = 0.2 sec .
2 g 9.8
149.8 5. 2
= 19
×0.316 × 10–23 = 47.37 × 10–4
10 Magnetic field ( B ) at the origin = Magnetic field due to
= 4.737 × 10–3 T = 4.737 mT 4.74 mT semicircle KLM + Magnetic field due to other semicircle KNM.
4. 0.2 µ0 I µ I
When AB is steady, Therefore, B ( i) + 0 ( j )
4R 4R
Weight per unit length = Force per unit length
µ0 I µ I µ I
µ0 2I1I2 B i + 0 j = 0 ( i + j)
Weight per unit length = ... (i) 4R 4R 4R
4 r
When the rod is depressed by a distance x, then the force µ I
[ B due to a circular current carrying loop is 0
acting on the upper wire increases and behaves as a 2R
restoring force For semicircle it is half]
Fmag Therefore, magnetic force acting on the particle,
A B µ0 I
I1=20A x
F q (v B ) q ( v0 i ) ( i j)
A' 4R
r = 0.01m mg B' µ0 qv0 I
= k
4R
C I2=30A D µ qv I 8R
F = 0 0 = = 2 units
µ0 2 I1I 2 µ0 2 I1I 2 4R 4R
Restoring force/length = 6. 0.7
4 r x 4 r
1 2 1
µ0 1 1 eV = mv p and eV = mv 2
= 2I1I 2 2 2
4 r x r V is the potential difference
vp = velocity of proton
µ0 r (r x) v = velocity of -particle
Restoring force/length = 2 I1I 2
4 ( r x) r m = mass of proton, mass of -particle = 4 m
2eV 2eV
µ0 2I1I 2 x vp = ,v =
= m 4m
4 r ( r x)
Now when the particles enter in magnetic field, the Lorentz
when x is small i.e., x << r then r – x r force on proton is
µ0 2 I1I 2
Restoring force/length F = x mv 2p
4 r2 evpB =
rp
Since, F µ x
The motion is simple harmonic mv p
or rp =
µ0 2I1I 2 eB
= (mass per unit length) 2 ... (ii)
4 r2 m
r =
µ0 2I1I 2 eB
From (i), (Mass per unit length) × g =
4 r 2eV 1 2mV 1 4mV
= and r =
µ0 2I1I 2 m B e B e
Mass per unit length = ... (iii) rp
4 rg 1
= = 0.7
From (ii) and (iii), r 2
µ0 2 I1I 2 µ0 2I1I 2 2
4 r2 4 rg
1. An ac source of angular frequency is fed across a resistor
R and a capacitor C in series. The current registered is I. If
5. The equation of AC voltage is E = 220 sin ( t+ / 6) and
now the frequency of source is changed to /3 (but the A.C. current is I = 10 sin ( t / 6) . The average power
maintaining the same voltage), the current in the circuit is
dissipated is
found to be halved. Then the ratio of reactance to resistance
at the original frequency is (a) 150 W (b) 550 W
(c) 250 W (d) 50 W
(a) 3/ 5 (b) 5/3
6. A uniform coil of self-inductance 1.8 × 10–4 H and resistance
(c) 2/3 (d) 3/ 2 6W is broken up into two identical coils. These two coils are
2. If the readings of V1 and V3 are 100 volt each then reading then connected in parallel across a 12 V battery. The circuit
of V2 is time constant and steady state current through the battery
respectively are :
L C
R (a) 30 s, 8 A (b) 30 ms, 8 mA
(c) 30 s, 8 A (d) 300 s, 800 A
7. Current in an ac circuit is given by i = 3 sin t + 4 cos t
V1 V2 V3 then
(a) rms value of current is 5 A
(b) mean value of this current in one half period will be 6/
~
200V,50Hz
(c) if voltage applied is V = Vm sin t then the circuit must
(a) 0 volt be containing resistance and capacitance.
(b) 100 volt (d) if voltage applied is V = Vm sin t, the circuit may
(c) 200 volt contain resistance and inductance.
(d) cannot be determined by given information. 8. An LCR series circuit with 100 resistance is connected to
3. A beam of light of intensity 12 watt/cm² incident on a totally an AC source of 200 V and angular frequency 300 radians
reflecting plane mirror of area 1.5 cm². The force in newton per second. When only the capacitance is removed, the
acting on the mirror will be- current lags behind the voltage by 60°. When only the
(a) 2.4 × 10–6 (b) 1.2 × 10–7 inductance is removed, the current leads the voltage by 60°.
(c) 3.6 × 10 –8 (d) 5.6 × 10–5 Then the current and power dissipated in LCR circuit are
4. If i1 = 3 sin t and i2 = 4 cos t, i3 = i1 + i2 then i3 is respectively
(a) 5 sin ( t + 53°) (b) 5 sin ( t + 37°) (a) 1A, 200 watt (b) 1A, 400 watt
(c) 5 sin ( t + 45°) (d) 5 cos ( t + 53°) (c) 2A, 200 watt (d) 2A, 400 watt

MARK YOUR 1. 2. 3. 4. 5.
RESPONSE 6. 7. 8.
566 IIT-JEE PHYSICS Challenger
9. In an L-R circuit, the value of L is (0.4/ ) henry and the 15. The charge on a capacitor decreases times in time t, when
value of R is 30 ohm. If in the circuit, an alternating emf of it discharges through a circuit with a time constant
200 volt at 50 cycles per second is connected, the impedance (a) t = (b) t = ln
of the circuit and current will be :
(a) 11.4 ohm, 17.5 ampere 1
(c) t = (ln – 1) (d) t ln 1 ÷
(b) 30.7 ohm, 6.5 ampere
(c) 40.4 ohm, 5 ampere 16. A uniform magnetic field of induction B is confined to a
(d) 50 ohm, 4 ampere. cylindrical region of radius R.
10. What is the amount of power delivered by the ac source in
the circuit shown (in watts).
XC=12 R1=5 B
R
XL=8 R2=6
P

Rrms=130V dB
The magnetic field is increasing at a constant rate of
dt
(a) 500 watt (b) 1014 watt
(tesla/second). An electron of charge q, placed at the point
(c) 1514 watt (d) 2013 watt
P on the periphery of the field experiences an acceleration.
11. Calculate the power factor of L–C–R circuit at resonance ?
(a) 0.1 (b) 1/4 BR
(a) towards left
(c) 1/2 (d) 1 ( 2 + 1) m
12. For a LCR series circuit with an A.C. source of angular 1 eR dB
frequency . (b) towards left
2 m dt
1
(a) circuit will be capacitive if > (c)
eR dB
towards left
LC m dt
1 (d) zero
(b) circuit will be inductive if = 17. A wooden stick of length 3 is rotated about an end with
LC
(c) power factor of circuit will by unity if capacitive constant angular velocity in a uniform magnetic field B
reactance equals inductive reactance perpendicular to the plane of motion. If the upper one third
of its length is coated with copper, the potential difference
1
(d) current will be leading voltage if > across the whole length of the stick is
LC
× × × × × × × × × ×
13. An observer is at a distance of 18 meter from a point source × × × × × × ×copper
× × ×
of light. The output power of the source is 250 watt. The × × × × × × ×coat× × ×
R.M.S. value of electric field at the position of the observer × × × × × × × × × ×
will be in volt/meter. × × ×3 × × × × × × ×
× × × × × × × × × ×
(a) 0.48 (b) 4 8 × × × × × × × × × ×
(c) 4.8 (d) 4 80 × × × × × × × × × ×
14. A coil has an inductance of 0.7 henry and is joined in series × × × × × × × × × ×
with a resistance of 220 . When the alternating emf of 220 × × × × × × × × × ×
V at 50 Hz is applied to it then the phase through which 9B 2
4B 2
current lags behind the applied emf and the wattless (a) (b)
2 2
component of current in the circuit will be respectively
(a) 30°, 1 A (b) 45°, 0.5 A 2 2
5B B
(c) (d)
(c) 60°, 1.5 A (d) None of these 2 2

MARK YOUR 9. 10. 11. 12. 13.


RESPONSE 14. 15. 16. 17.
ELECTROMAGNETIC INDUCTION & AC CURRENT 567

18. A superconducting loop of radius R has self inductance L. 22. A coil of area 7 cm2
and of 50 turns is kept with its plane
A uniform and constant magnetic field B is applied normal to a magnetic field B. A resistance of 30 ohm is
perpendicular to the plane of the loop. Initially current in connected to the resistance-less coil. B is 75 exp (– 200t)
this loop is zero. The loop is rotated by 180°. The current in gauss. The current passing through the resistance at t = 5
the loop after rotation is equal to ms will be-
(a) 0.64 mA (b) 1.05 mA
B R2
(a) zero (b) (c) 1.75 mA (d) 14 2.60 mA
L
23. A short-circuited coil is placed in a time-varying magnetic
2B R 2 B R2 field. Electrical power is dissipated due to the current induced
(c) (d) in the coil. If the number of turns were to be quadrupled and
L 2L
the wire radius halved, the electrical power dissipated would
19. PQ is an infinite current carrying conductor. AB and CD are
be
smooth conducting rods on which a conductor EF moves
with constant velocity v as shown. The force needed to (a) halved (b) the same
maintain constant speed of EF is (c) doubled (d) quadrupled
24. The figure shows a conducting loop consisting of half circle
P R
A C of area A = 0.06 m2 and three straight segments. The half
circle lies in a uniform changing magnetic field B = 4r2 + 2t +
5 (SI unit), where t is the time in second. An ideal battery E
I E F = 2 V is connected as shown and the total resistance of the
wire is 2 . The net current in the loop is at t = 5 second is:
v
V

a B
B
Q B D

2 2
1 0 Iv b 0 Iv a 1 – +
(a) ln ÷ (b) ln ÷
vR 2 a 2 b vR
(a) 1 A (b) 1.5 A
2 2 (c) 0.26 A (d) 0.10 A
0 Iv b v v 0 Iv a
(c) ln ÷ (d) ln ÷ 25. A current of 1.5 A flows through a solenoid of length 20.0
2 a R R 2 b
cm, cross-section 20.0 cm2 and 400 turns. The current is
20. A copper rod of length 0.19 m is moving parallel to a long suddenly switched off in a short time of 1.0 millisecond.
wire with a uniform velocity of 10 m/s. The long wire carries Ignoring the variation in the magnetic field the ends, the
5 ampere current and is perpendicular to the rod. The ends average back emf induced in the solenoid is:
of the rod are at distances 0.01 m and 0.2 m from the wire. (a) 0.3 V (b) 9.6 V
The emf induced in the rod will be- (c) 30.0 V (d) 3.0V
(a) 10 µV (b) 20 µV 26. A rod PQ of length L moves with a uniform velocity v parallel
(c) 30 µV (d) 40 µV to a long straight wire carrying a current i, the end P
21. The current in an L–R circuit builds up to remaining at a distance r from the wire. The emf induced
(3/4)th of its steady state value in 4 seconds. The time across the rod is
constant of this circuit is
2 2
0 iv
2
r+L 0i v r2 + L
1 2 (a) ln ÷ (b) ln ÷
(a) sec (b) sec 2 r 2 r ÷
ln 2 ln 2

3 4 0 iv r+L 0iv r 2 + L2
sec sec (c) ln ÷ (d) ln ÷
(c) ln 2 (d) ln 2 2 r 2 L2 ÷

MARK YOUR 18. 19. 20. 21. 22.


RESPONSE 23. 24. 25. 26.
568 IIT-JEE PHYSICS Challenger
27. In the given figure MNPQ which falls through the magnetic B = 1 T, perpendicular into the paper. Find the current in the
field has conductivity and mass density . The frame’ss loop at the moment, when the heating starts. Resistance of
terminal velocity assuming it to be small enough so that it the loop is 10 at any temperature. Coefficient of linear
reaches its final velocity before leaving the region occupied expansion = 10–6/°C.
by the magnetic field is

B × ×
45° 45°
× ×
× × 1m
M × × N

a a 1m
(a) 1.5 × 10–6 A anticlockwise
(b) 1.5 × 10–6 A clockwise
Q a P (c) 0.75 × 10–6 A anticlockwise
v (d) 0.75 × 10–6 A clockwise
30. An equilateral triangular loop having a resistance R and
12c 2 g 16c 2 g
(a) (b) length of each side is placed in a magnetic field which is
eB 2 eB 2
dB
varying at = 1 T/s. The induced current in the loop will
16c 2 2 g 16c g dt
(c) (d)
eB e2 B be
28. The given assembly made of a conducting wire is rotated
with a constant angular velocity about a vertical axis MO
as shown in the figure. The magnetic field B exists vertically
upwards as shown in the figure. Find the potential difference
between points M and N, |Vm – VN| (only the magnitude)

M
R

2
3 2 4
R (a) (b)
4 R 3 R
3 R 4 R
(c) (d)
N O 4 2 3 2
B 31. A resistor of resistance R, capacitor of capacitance C and
inductor of inductance L are connected in parallel to AC
B R2 R2 power source of voltage 0 sin t . The maximum current
(a) (b) B
2 4 through the resistance is half of the maximum current through
the power source. Then value of R is
R2
(c) B 2R2 – ÷ (d) zero 3
4 1
(a) (b) 3 – L
1 C
29. A straight conducting metal wire is bent in the given shape C–
L
and the loop is closed. Dimensions are as shown in the
figure. Now the assembly is heated at a constant rate dT/dt 1
(c) 5 – L (d) none of these
= 1°C/s. The assembly is kept in a uniform magnetic field C

MARK YOUR
27. 28. 29. 30. 31.
RESPONSE
ELECTROMAGNETIC INDUCTION & AC CURRENT 569

32. In the circuit shown switch k2 is open and switch k1 is (a) maximum charge in the capacitor can be 6C
closed at t = 0. At time t = t0 switch k1 is opened and switch (b) maximum charge in the capacitor can be 8C
k2 is simultaneously closed. The variation of inductor
(c) charge in the capacitor will be maximum after time 2
current with time is
sin–1 (2/3) sec
k2
(d) None of these
34. In the circuit shown, the key (K) is closed at t = 0, the current
L through the key at the instant t = 10–3 ln 2, is

k1 E 20V

i L = 5mH

Et0 K
(a)
L
6 C = 0.1 mF
t0 t
i (a) 2A (b) 3.5A
(c) 2.5A (d) zero
Et0 35. Find the current passing through battery immediately after
(b)
L key (K) is closed. It is given that initially all the capacitors
are uncharged. (given that R = 6 and C = 4µF)
t0 t
i
R C

Et0
(c) K R
L
R C
t0 t
i C
E = 5V
L
Et0
(d) C
L R

t0 R
t
33. In an L-C circuit shown in the figure, C = 1F, L = 4H. At time
t = 0, charge in the capacitor is 4C and it is decreasing at a
(a) 1 A (b) 5 A
rate of 5 C/s. Choose the correct statements.
(c) 3 A (d) 2 A
q
+ – 36. The magnetic flux in a closed circuit of resistance 10
varies with time as = (2t –4t2 +1). The current in the loop
C
will change its direction after a time of
(a) 0.25 sec (b) 0.5 sec
(c) 1 sec (d) None

MARK YOUR
32. 33. 34. 35. 36.
RESPONSE
570 IIT-JEE PHYSICS Challenger
37. A magnet is being moved towards the axis of a circular coil
of radius ‘a’ such that (x >> a); then the induced current in (c) source emf leads current by
4
the loop coil is proportional to

x (d) source emf leads current by


3
40. For the circuit as shown in figure; the applied current in
A.C. circuit is zero ampere and IC = 10A. Then the magnitude
v
S N of current IL is
a

IC = 10 A
1 1
(a) (b) 2 µF = C
x x2

1
(c) 4
(d) x2 4 mH = L
x
~
38. A current is passed through a spring connected between
the two terminal of a battery. Taking spring as an inductor
(a) 4 A (b) 10 A
coil, the effective spring constant of spring will
(c) 5 A (d) undefined
(a) increase (b) decrease
41. The current in a coil of L = 4H is varying with time at
(c) unchanged (d) may increase or decrease
I = 4 sin (2t2). Then the energy stored in the coil when
39. In the given AC circuit, when switch S is at position 1, the
current change from 0 to 4A is
(a) 16 J (b) 32 J
source emf leads current by . Now, if the switch is at
6 (c) 8 J (d) 4 J
position 2, then 42. A rectangular loop is present in the magnetic field region of
an infinite long wire. Now the loop is being rotated as shown
L = 3 mH 1 in the figure.

20 V S

2
100 µF A B
C= I
3
~
V = V0 sin 1000 t
D C

(a) current leads source emf by


4
Then the induced current in side AD will be
(a) along DA (b) along AD
(b) current leads source emf by
3 (c) zero (d) None of these

MARK YOUR 37. 38. 39. 40. 41.


RESPONSE 42.
ELECTROMAGNETIC INDUCTION & AC CURRENT 571

43. In the figure ab and cd are two long conduction wires kept
mv02 mv02
parallel to each other at a separation in a uniform time (a) (b)
L 2L
varying magnetic field. The ends a and c are connected
together by a resistor of resistance R. The magnetic
mv02
induction B perpendicular to the plane of the wires varies (c) (d) zero.
with time according to the relation B = B0t, where B0 is a 4L
positive constant with proper unit and t is the time in second. 45. The two capacitors, shown in the circuit, are initially
A conducting wire PQ placed on the wire ab and cd and uncharged and the cell is ideal. The switch S is closed at t =
dragged on the wires with constant speed v along the length 0. Which of the following functions represents the current
i(t), through the cell as a function of time?
of the wires by applying a constant force F. Find the value
of F in terms of the other given parameters. At time t = 0, PQ
i(t) C 2C
is very close to ac. Ignore any resistance (electrical as well
R R
as mechanical) other than R. R

S
P
a b
R
(a) i (t ) = i0 + i1e t/
; 3C
F 3
R
(b) i (t ) = i0 + i1e t/
+ i2 e t/2
; = RC
c d
Q R
(c) i (t ) = i1 + i1e t/
; 3C
3
B02 2
vt 2 2B02 2
vt 2 (d) i(t) = i0 + i1e–t/ ; = 3RC
(a) (b) Here i0, i1, i2 are constants.
R R
46. Time constant of L–R circuit will be
4B02 2
vt 2 8B02 2
vt 2
(c) (d) 2R
R R
44. In a uniform and constant magnetic field of induction B, two L R
long conducting wires ab and cd are kept parallel to each
other at distance with their plane perpendicular to B. The K 2R
ends a and c are connected together by an ideal inductor of
E
inductance L. A conducting slider wire PQ is imparted a
speed v0 at time t = 0. The situation is shown in the figure. L 2L
(a) (b)
R R
mL
At time t = , the value of current I through the wire
4B L
(c) (d) None of above
PQ is (ignore any resistance, electrical as well as mechanical) 2R
47. Two identical circular loops of metal wire are lying on a table
P without touching each other. Loop-A carries a current which
a b increases with time. In response, the loop-B
(a) remains stationary
L v
(b) is attracted by the loop-A
(c) is repelled by the loop-A
c d (d) rotates about its CM, with CM fixed
Q

MARK YOUR
43. 44. 45. 46. 47.
RESPONSE
572 IIT-JEE PHYSICS Challenger
48. As shown in figure, value of inductive reactance XL will be
if source voltage is 100 volt y

XL=30 R2=10 XC=40


R1=20 b

a x
I = 2A
xL
c
z

(1) B1 = 3iˆ + 7 ˆj 5kˆ


(a) 40 (b) 30
50
(2) B2 5tiˆ 4 ˆj 15kˆ
(c) (d) can have any value
49. In circuit diagram capacitance of capacitor C1= 3 F and (3) B3 2iˆ 5tjˆ 12kˆ
C2 = 1 F. It is given that time constant of circuit between A
and B is 3 millisecond. Value of R will be where B is in milli tesla and t is in seconds. If the induced
current in the loop due to B1 , B2 and B3 are i1, i2 and i3
A
respectively, then
C2 R (a) i1 > i2 > i3 (b) i2 > i1 > i3
C1
(c) i3 > i2 > i1 (d) i1 = i2 = i3
C1
C2 B 52. In the LC circuit, the current is in the direction shown and
C2 the charges on the capacitor plates have the signs shown.
C1 At this time
R
C
I
(a) 1 (b) 10
(c) 100 (d) 1000
+Q – Q
50. In figure given below if ZL = Zc and reading of ammeter is
1A. Find value of source voltage V. L

ZL ZC

(a) I is increasing and Q is increasing


L=i/ H
(b) I is increasing and Q is decreasing
(c) I is decreasing and Q is increasing
R = 80 (d) I is decreasing and Q is decreasing
~ A 53. Resonance occurs in a series L-C-R circuit when the
V, 30 Hz frequency of the applied emf is 1000 Hz. Then
(a) when f = 900 Hz, the circuit behaves as a capacitative
(a) 80 Volt (b) 60 Volt circuit.
(c) 100 Volt (d) None of these (b) the impedance of the circuit is maximum at f = 1000 Hz
51. In given figure, a wire loop has been bent so that it has three (c) at resonance the voltage across L and voltage across
segments : segment ab (a quarter circle), bc (a square corner), C differ in phase by 180°
and ca (straight). Here are three choices for a magnetic field (d) if the value of C is doubled resonance occurs at
through the loop : f = 2000Hz.

MARK YOUR 48. 49. 50. 51. 52.


RESPONSE 53.
ELECTROMAGNETIC INDUCTION & AC CURRENT 573

54. A conducting square loop is placed in a magnetic field B (a) R1 = R2, L1 > L2 (b) R1 > R2, L1 = L2
with its plane perpendicular to the field. The sides of the
(c) R1 > R2, L1 < L2 (d) R1 = R2, L1 < L2
loop start shrinking at a constant rate . The induced emf in
the loop at an instant when its side is ‘a’ is 57. In the circuit shown, assuming all ammeters to be ideal, if
(a) 2a B (b) a2 B readings of the hot wire ammeters A1 and A2 are i1 and i2
(c) 2a B2 (d) a B respectively then reading of the hot wire ammeter A3 is
55. An infinitely long solenoid passes through the circuit as
shown. The magnetic field of the solenoid, directed into the
plane of the page, is weakening which produces a constant R
A1
emf of magnitude E for a closed loop outside of the solenoid.
Once equilibrium is established in this circuit, what is the A3
potential difference across the switch S ? L
A2

R ~
×
× × AC source
E R ×
Solenoid
R (a) equal to (i1 + i2) (b) greater than (i1 + i2)
S
(c) less than (i1 + i2) (d) equal to 2 (i1 – i2) |

1 58. In the circuit shown, the symbols have their usual meanings.
(a) 0 (b) E The cell has emf E. X is initially joined to Y for a long time.
3
Then, X is joined to Z. The maximum charge on C at any later
(c) 4 (d) cannot be determined time will be
E
3
56. Refer to the circuit diagram and the corresponding graphs. L
The current rises when key K is pressed. With R = R1 and L
= L1 the rise of current is shown by curve (1), while curve (2) R + – Y
shows the rise of current when R = R2 and L = L2. The
maximum current is same for the both curves, then
L X
R
C Z

E ER
(a) (b)
K R LC 2 LC
V
i
E LC E LC
(c) (d)
2R R
i0
59. A coil of inductance 8.4 mH and resistance 6 is connected
(1) to a 12 V battery. The current in the coil is 1.0 A at
(2) approximately the time
(a) 500 s (b) 25 s
(c) 35 ms (d) 1 ms
O Time (t)

MARK YOUR 54. 55. 56. 57. 58.


RESPONSE 59.
574 IIT-JEE PHYSICS Challenger
60. A rectangular loop of wire with dimensions shown in Up
coplanar with a long wire is carrying current I. The distance
between the wire and the left side of the loop is r. The loop N S Left Right
is pulled to the right as indicated. What are the directions of
A Down
the induced current in the loop and the magnetic forces on
the left and right sides of the loop as the loop is pulled?
r N S

I b (a) to the right (b) to the left


(c) upwards (d) equal to zero
a 64. A very long solenoid perpendicular to the page generates a
downward magnetic field whose magnitude increases with
Induced current Force on Force on
the time. A conducting wire loop around the solenoid
left side right side
contains two identical bulbs A and B which are glowing.
(a) Counterclockwise To the left To the left Two points diametrically opposite on the wire loop are
(b) Counterclockwise To the right To the left shorted with another wire lying to the right of bulb B in the
(c) Clockwise To the right To the left plane of the page as shown in figure. After the shorting wire
(d) Clockwise To the left To the right is inserted

61. y

A B
10

(a) bulb A goes out and bulb B dims


0 0.1 0.2 0.3 0.4 (b) bulb A goes out and bulb B gets brighter
t (Second)
(c) bulb B goes out and bulb A dims
(d) bulb B goes out and bulb A gets brighter
Calculate the rms value of the function shown in figure if it
is given that for 0 < t < 0.1, y = 10 (1 – e–100t) and for 65. The circuit shown has been operating for a long time. The
instant after the switch in the circuit labeled S is opened,
0.1 < t < 0.2, y = 10e–50(t–0.1).
what is the voltage across the inductor VL and which labeled
(a) 6.2 (b) 5.3
point (A or B) of the inductor is at a higher potential ?
(c) 4.1 (d) 6.9
TakeR1 = 4.0 , R2 = 8.0 and L = 2.5 H.
62. In a series LCR circuit, the difference of the frequencies at
L S
1
which current amplitude falls to of the current amplitude
2 A B
at resonance is
R1 R2
R R E = 12V
(a) (b)
2 L L

2R 3R
(c) (d)
L 2 L
(a) VL = 12V, point A is at the higher potential
63. A bar magnet was pulled away from a hollow coil A as shown. (b) VL = 12V, point B is at the higher potential
As the south pole came out of the coil, the bar magnet next (c) VL = 6V, point A is at the higher potential
to hollow coil B experienced a magnetic force (d) VL = 12V, point B is at the higher potential

MARK YOUR 60. 61. 62. 63. 64.


RESPONSE 65.
ELECTROMAGNETIC INDUCTION & AC CURRENT 575

66. The four wire loops shown have vertical edge lengths of 69. A conducting disc of conductivity has a radius ‘a’ and
either L, 2L or 3L. They will move with the same speed into thickness ‘t’. If the magnetic field B is applied in a direction
a region of uniform magnetic field B directed out of the perpendicular to the plane of the disc changes with time at
page. Rank them according to the maximum magnitude of dB
the induced emf, greatest to least. the rate of . Calculate the power dissipated in the
dt
disc due to the induced current.

t a4 2 t a4 2
(a) (b)
B 8 4

1 2 3 4
t a4 2 2 t a4 2
(c) (d)
2 3
(a) 1 and 2 tie, then 3 and 4 tie
70. In the circuit shown in Figure, how does the potential
(b) 3 and 4 tie, then 1 and 2 tie difference between the points A and B depend on the value
(c) 4, 2, 3, 1 of the variable resistor (R) ?
(d) 4 then, 2 and 3 tie and then 1
67. When a resistance R is connected in series with an element C
A, the electric current is found to be lagging behind the r
voltage by angle 1. When the same resistance is connected
in series with element B, current leads voltage by 2. When V ~ A B
R, A, B are connected in series, the current now leads voltage
by . Assume same AC source is used in all cases, then r R
(a) = 2– 1 (b) tan = tan 2 – tan 1

(c) = 1 2
(d) None of these (a) V/2 (b) V
2
68. A rod lies across frictionless rails in a uniform magnetic field (c) V/3 (d) None of these

B as shown. The rod moves to the right with speed v. For 71. If you hold a strong permanent magnet inside a metal ring
the induced emf in the circuit to be zero, the magnitude of and then quickly remove that magnet, you create an electric
the magnetic field should field in the metal ring and an electrical current flows through
the metal ring. If you repeat this procedure with a glass ring,
v no current flows in the glass ring because
(a) although removing the magnet creates an electric field
× × × ×
in the glass ring, the glass ring has no mobile electric
charges to carry an electrical current
× × B ×
(b) removing the magnet from the glass ring does not create
an electric field
× ×
× (c) the glass ring creates its own magnetic field in the
opposite direction, opposing the field of the permanent
magnet and preventing any current flow
(a) not change
(d) the glass ring is made of a non-magnetic material. No
(b) increase linearly with time
magnetic fields can be present inside the ring so
(c) decrease linearly with time
removing the magnet has no effect on the ring.
(d) decrease non-linearly with time.

MARK YOUR 66. 67. 68. 69. 70.


RESPONSE 71.
576 IIT-JEE PHYSICS Challenger
72. A wire is being wound around a rotating wooden cylinder 74. A metallic ball of radius r is moving with constant velocity v
of radius R. One end of the wire is connected to the axis of in an uniform magnetic field B. Find max. The angle
the cylinder, as shown in the figure. The cylinder is placed between B and v is .
in a uniform magnetic field of magnitude B parallel to its axis [ = /4, B = 1T, v = 1 m/s and r = 1 cm]
and rotates at N revolutions per second. What is the
potential difference between the open ends of the wire? (a) 8mV (b) 24mV
(c) 14mV (d) 12mV
75. The source of alternating current in the circuit shown in Fig.
a produces a signal represented in Fig. b. Values of r, R, C,
V, and T are known. Voltage across the capacitor changes
very slightly during one period. Calculate the voltage across
the capacitor in a great number of periods.

B
r

V R C
(a) 0 (b) 2 NBR
(c) NBR2 (d) None of these
73. An equilateral triangular loop ABC made of uniform thin
wires is being pulled out of a region with a uniform speed v,
where a uniform magnetic field B perpendicular to the plane V
of the loop exists. At time t = 0, the point A is at the edge of
the magnetic field. The induced current (I) vs time (t) graph
will be as
V0

A v
×××××××××××××
×××××××××××××
×××××××××××××
×××××××××××××
×××××××××××××
× × × ×B× × × × ×C× × × × t
××××××××××××× T

I I
2R
(a) V0
R + rT + r
(a) (b)
t t R
(b) V0
R + rT + r

I I R
(c) V0
R rT r

(c) (d) R
t t
(d) V0
R rT r

MARK YOUR
72. 73. 74. 75.
RESPONSE
ELECTROMAGNETIC INDUCTION & AC CURRENT 577

PASSAGE-1
b 2 b
0i0 ln ÷ v 2 0 i0 ln ÷
A long straight wire carries a current i0 . At distances a and b from a a v
(c) (d)
it there are two other wires, parallel to the former one, which are 2 R2 2 R
interconnected by resistance R. A connector slides without friction
along the wires with a constant velocity v. Assuming the resistance
of the wires, the connector, the sliding contacts and the self
inductance of the frame to be negligible, find PASSAGE-2
Two long parallel conducting rails of zero resistance separated by
a distance L are joined to a cell of emf E at one end. An external
uniform magnetic field B is applied normal to the plane and into
the plane of the rails as shown in the figure. A conducting bar of
R v mass m and resistance R is placed across the rails. The bar can
b
slide freely parallel to itself always remaining perpendicular to the
rails.

a
i0
+ × B
L E

1. The magnitude of the emf induced in the connectors is

0 i0 v b 0 i0 v b
(a) ln ÷ (b) ln ÷
4 a 2 a 4. Find the terminal velocity of the bar if at t = 0, velocity is
zero
2 2 2
0i0 v b 0 i0 v b
(c) ln ÷ (d) ln ÷ (a)
E
(b)
E
2 a 2 a
B BL
2. The magnitude of current induced in the connectors is
2E EL
(c) (d)
0 i0 vb 0i0 v b a BL B
(a) ln ÷ (b) ln ÷
2 R a 2 R a 5. Find current flowing through resistance at terminal velocity
vT
2
0 i0 v a 0i0 v b
(c)
2
ln ÷ (d) 2
ln ÷ B BLvT
2 R b 2 R a (a) (b)
R R
3. The force required to maintain the connector’s velocity
(c) zero (d) None of these
constant is
6. The energy supplied by cell at any instant is
2 2 (a) Dissipated only as joule heating in the circuit.
b b
0i0 ln ÷ 0i0 ln ÷ v2
(b) Partly dissipated as joule heating in the circuit and the
a v a
(a) (b) rest is used up in moving the bar.
2 R 2 R2 (c) All used is moving bar
(d) Zero.

MARK YOUR 1. 2. 3. 4. 5.
RESPONSE 6.
578 IIT-JEE PHYSICS Challenger

PASSAGE-3 1
10. Ratio of is
Two capacitors of capacitance C and 3C are charged to potential 2
difference V0 and 2V0 respectively and connected to an inductor (a) 2/3 (b) 1/1
of inductance L as shown in the figure. Initially the current in the (c) 1/3 (d) 3/1
inductor is zero. Now the switch S is closed. 11. What fraction of the heat generated during discharging is
lost in R1?
V0 2V0
+ – + – 1 1
(a) (b)
6 3
C 3C
S 1 2
(c) (d)
L 2 3
12. If the battery as shown in the figure has an internal resistance
7. The maximum current in the inductor is R
r= then the time varying expression of the current in
2
3V0 3C 3C the capacitor is
(a) (b) V0
2 L L
t 4t
E RC 2E 5RC
(a) e (b) e
3C C R 5R
(c) 2V0 (d) V0
L L
4t 4t
8. Potential difference across capacitor of capacitance C when 2 E 5RC 3E 5 RC
(c) e (d) e
the current in the circuit is maximum. 3R 5R
CV0 3CV0
(a) (b) PASSAGE-5
4 4
Faraday’s Law says that the change of magnetic flux with any
5CV0 closed loop produces induced emf and current in the loop.
(c) (d) None of these
4
e d 1
9. Potential difference across capacitor of capacitance 3C when Magnitude of current produced i = = ÷ . The cause
R dt R
the current in the circuit is maximum
of induced current is the rate of change of flux.
CV0 3CV0 Then using this concept in the situation where a wire frame of
(a) (b)
4 4 area 4 ×10–4 m2 and resistance 10 is suspended from a thread 2
m long in a vertical plane. Now a horizontal magnetic field of 1
5CV0
(c) (d) None of these tesla is introduced and the arrangement is made to oscillate from
4 its equilibrium position as shown in the figure.

PASSAGE-4 O

The capacitor is charged by closing the switch S. The switch is


then opened and the capacitor is allowed to discharge. t1 and t2
be the time constant of the circuit during the charging and l
discharging of the capacitor. Take R1 = R2 = R3 = R Q
B
S P
E R1 R2 C 13. The magnetic flux with loop will be maximum in the position
(a) P (b) Q
R3 (c) between P and Q (d) = 45°

MARK YOUR 7. 8. 9. 10. 11.


RESPONSE 12. 13.
ELECTROMAGNETIC INDUCTION & AC CURRENT 579

14. For small angle , such that maximum displacement of


x0 (c) dP/dt (d) dP/dt
x0 = 1 cm and 0 = ÷ ; find the induced e.m.f as a

function of time ‘t’ when t = 0 is equilibrium position


(a) 5 ×10 –9 sin (10t) (b) 9
5 5 10 sin (2 5t) t t
(c)
5sin ( 10t) (d) 10 –6 sin (5t)
PASSAGE-7
15. The maximum induced current in the loop is
The conducting connector of mass m and length L can freely slide
(a) 5 × 10 – 8A (b) 2 5 10 9 A
on horizontal long conducting parallel rails connected by capacitor
(c) 5 5 10 10 A (d) 10 –7A C at one end as shown in figure. A non conducting light spring
(spring constant K) is connected to the connector ab and is in a
PASSAGE-6 relaxed state. The whole system is placed in uniform magnetic
field of strength B directed into the plane of rails (figure). Now at
A resistor and inductor are connected in series through a battery. time t = 0, connector is suddenly given velocity V0 in rightward
The switch S is closed at time t = 0. direction. If resistance and self inductance of circuit is negligible
then
R L
a

B
spring
C L
V S
V0
16. The rate of Joule heating (P) in resistor varies with the time b
‘t’ is best represented by the graph. x=0
P P
19. The magnitude of acceleration of connector as a function of
x is
(a) (b) Kx Kx
(a) (b)
t t m m B 2 L2C
P P
(c) Kx (d) None of these
m + B 2 L2C
(c) (d) 20. The maximum compression in the spring is
t t
m B 2 L2 C m + B 2 L2 C
17. What is the magnitude of current flowing when the rate of (a) V0 (b) V0
increase of magnetic energy in the inductor is maximum K K
(a) I = V/R (b) I = V/2R
m
(c) I = V/4R (d) I = (V/R) ln 2 (c) V0 (d) None of these
18. Which of the following graph best represent rate of change K
of power dissipated in resistor as a function of time. 21. The rod will execute (after projecting it at t = 0)
(a) SHM
(a) dP/dt (b) dP/dt (b) harmonic motion but not SHM
(c) the rod will come at rest at certain position then
afterward it will not move
(d) None of these
t t

MARK YOUR 14. 15. 16. 17. 18.


RESPONSE 19. 20. 21.
580 IIT-JEE PHYSICS Challenger
25. Which plot does correctly represents rms current against
PASSAGE-8 frequency ?
In a series L-R circuit, connected with a sinusoidal as source, the
maximum potential difference across L and R are respectively 3
volts and 4 volts. I Woofer
22. At an instant the potential difference across resistor is 2
Tweeter
volts. The potential difference in volt, across the inductor
(a)
at the same instant will be f
(a) 3 cos 30° (b) 3 cos 60°
(c) 3 cos 45° (d) None of these Tweeter
23. At the same instant, the magnitude of the potential difference I
in volt, across the ac source may be
Woofer
4+3 3 (b)
(a) 4+3 3 (b) f
2

3 3 Woofer
(c) 1 + (d) 2+ I
2 2

24. If the current at this instant is decreasing the magnitude of Tweeter


potential difference at that instant across the ac source is (c)
(a) increasing (b) decreasing f
(c) constant (d) cannot be said

PASSAGE-9 I Tweeter
A loudspeaker system uses alternating current to amplify sound
of certain frequencies. It consists of 2 speakers. Woofer
Tweeter-which has smaller diameter produces high frequency (d)
f
sounds. Woffer-which has larger diameter produces low frequency
26. What is the frequency which is sounded loudly in equal
sound. For purpose of circuit analysis, we can take both speakers amounts by both speakers ?
to be of equal resistance R. The equivalent circuit is shown in the
figure. The 2 speakers are connected to the amplifier via 1 R2 1 1 4R2 1
(a) (b)
capacitance and inductance respectively. The capacitor in tweeter 2 L2 LC 2 L2 LC
branch blocks the low frequency sound but passes the high
frequency. The inductor in woffer branch does the opposite. 1 1 R2 1
(c) 2 (d)
2 LC 4L 2 LC
R C 27. For a combination of L, R and C the current in woofer and
Tweeter tweeter are always found to have a phase difference of /2.
What is the relation between L, R and C ?
R
Woofer L (a) L = 2R2C (b) L = 2 R 2C

~ R 2C
(c) L = R2C (d) L=
V0sin( t) 2

MARK YOUR 22. 23. 24. 25. 26.


RESPONSE 27.
ELECTROMAGNETIC INDUCTION & AC CURRENT 581

1. Statement - 1 : In series LCR circuit resonance can take Statement - 2 : The direction of induced emf is such that
place. it tends to oppose the cause of its
Statement - 2: Resonance takes place if inductance and production.
capacitive reactances are equal and
8. Statement - 1 : The resistance offered by an inductor in a
opposite.
d.c circuit is always constant.
2. Statement - 1 : X-rays travel with the speed of light.
Statement - 2 : The resistance of inductor in steady state
Statement - 2 : X-rays are electromagnetic rays.
is zero.
3. Statement - 1 : At resonance, LCR circuit have a minimum
current. 9. Statement - 1 : When an inductor coil is connected to a

Statement - 2 : At resonance, in LCR circuit, the current cell, initial current in it is zero.
and emf are in phase with each other. Statement - 2 : When an inductor coil is connected to a cell,
4. Statement - 1 : The phenomenon of self induction is the initial emf induced in it is zero.
helpful in working of a choke coil. 10. Statement - 1 : Peak voltage across the resistance can be
Statement - 2 : A choke coil is used for reducing energy greater than the peak voltage of the source
loss in the circuit. in a series LCR circuit.
5. Statement - 1 : Capacitor serves as a block for D.C. and
Statement - 2 : Peak voltage across the inductor can be
offers an easy path to A.C.
greater than the peak voltage of the source
Statement - 2 : Capacitive reactance is inversely
in an series LCR circuit.
proportional to frequency.
11. Statement - 1 : A conducting circular disc of radius R
6. Statement - 1 : Faraday’s laws are consequences of
rotates about its own axis with velocity
conservation of energy.
in a uniform magnetic field B0 directed
Statement - 2 : In a purely resistive A.C. circuit, the current
along axis of the disc. There will be no emf
lags behind the e.m.f. in phase.
induced in the disc.
7. Statement - 1 : In electromagnetic induction the Lenz's law
tells us only about the direction of induced Statement - 2 : Whenever a conductor cuts across
emf while Faraday’s law gives us only the magnetic lines of flux, an emf is induced in
magnitude of e.m.f. the conductor.

1. 2. 3. 4. 5.
MARK YOUR
6. 7. 8. 9. 10.
RESPONSE
11.
582 IIT-JEE PHYSICS Challenger

1. Two inductors having self inductances L1 and L2 and mutual 3. There are three wire MO, NO and PQ, wires MO and NO are
inductance M are arranged in parallel as shown in the figure. fixed and perpendicular to each other. Wire PQ moves with
The sense of the helix is same for both the inductors. Then a constant velocity v as shown in the figure and resistance
L1 per unit length of each wire is and magnetic field exists
perpendicular and inside the paper then

L2 M

× × × × × × ×
P
× × × × × × ×
B
× × × × × × ×
(a) The effective inductance of the combination will be
× × × × × × ×
L1L2 × × × × × × ×
L1 + L2 irrespective of the value of M. × × × × × × ×
O
(b) If the separation between the coils is large then effective Q N
L1L2
inductance of the combination will be close to (a) current in loop is anticlockwise
L1 + L2
because then M will become negligible. Bv
(b) magnitude of current in the loop is
(c) Effective inductance of the combination is
( 2 +1 )
2
L1 L2 – M
(c) current in the loop is independent of time.
2M + L1 + L2
(d) magnitude of current decreases as time increases.
(d) Effective inductance of the parallel combination is 4. In the circuit shown in the figure, if both the bulbs B1 and B2
L1 L2 – 2M 2 are identical
M + L1 + L2
C = 500 µF B1
2. In the circuit shown in the figure switch S is closed at time
t = 0. Select the correct statement.
C 2R
L = 10 mH B2

2C R

~
320 V, 50 Hz
S
E
(a) Rate of increase of charge is same in both the capacitor. (a) Their brightness will be the same
(b) Ratio of charge stored in capacitors C and 2C at any (b) B2 will be brighter than B1
time t would be 1 : 2. (c) As frequency of supply voltage is increased brightness
(c) Time constant fo both the capacitors are equal. of B1 will increase and that of B2 will decreases
(d) Steady state charge in capacitors C and 2C are in the (d) Only B2 will glow because the capacitor has infinite
ratio 1 : 2. impedance.

MARK YOUR
1. 2. 3. 4.
RESPONSE
ELECTROMAGNETIC INDUCTION & AC CURRENT 583

5. A nonconducting ring of mass m and radius R has a charge


Q uniformly distributed over its circumference. The ring is L1 R3
placed on a rough horizontal surface such that the plane of
the ring is parallel to the surface. A vertical magnetic field
B = B0t2 tesla is switched on. After 2 second from switching G
L2 R4
on the magnetic field the ring is just about to rotate about
vertical axis through its centre. Then
(a) the induced electric field is quadratic in time t
(b) the force tangential to the ring is B0QRt
(c) until 2 seconds, the friction force does not come into K
play
(d) the friction coefficient between the ring and the surface
L1 R L1 R
2B0 RQ (a) = 3 (b) = 1
is L2 R4 L2 R2
mg
6. Two metallic square frames P and Q of side and 2 R1 R
respectively move in opposite directions with speed 2v and (c) = 3 (d) R1 R4 R2 R3
R2 R4
v on a conducting surface. There is a uniform magnetic field
B perpendicular to the planes of the frame. Then 9. A conductor moves with a velocity 12 m/s at an angle of (i)
90°, (ii) 30° to a magnetic field produced between two square
faced poles of side length 2cm. Then the flux leaving the
2
pole face is 5µWb. Which of the following statements are
correct ?
Q 2 v
2v P (a) emf induced in case (i) is 3mV
S S' (b) emf induced in case (ii) is 2.5mV
(a) the emf induced in each of the vertical sides of frame P (c) the emf induced is independent of the length of the
will be 2B v conductor
(b) the upper point of P will be at a higher potential (d) if the velocity component for which no emf is induced
(c) the emf’s of the two coils are additive
v
(d) the upper point of Q will be at a low potential is , then the conductor moves at an angle 60° to the
2
7. A resistance of 10 3 , a capacitance of 10–6 F and an magnetic field
inductance of 2 Henry are connected in series with a source
of 200 sin (1000 t) volt. Which of the following are correct ? 10. A current of 4A flows in a coil when connected to a 12V DC
source. If the same coil is connected to a 12V, 50rad/s AC
(a) rms current = 0.1 A
source a current of 2.4A flows in the circuit. Choose the
(b) impedance = 1414 correct options.
(c) the voltage leads the current by 45°
(a) The inductance of the coil is 0.08 Henry
(d) power factor = 0.707
(b) The power developed in the circuit if a 2500µF capacitor
8. Two inductors of self-inductances L 1 and L 2 and of
is connected in series with the coil is 17.28 W
resistances R1 and R2 (not shown here) respectively, are
connected in the circuit as shown in figure. At the instant t (c) The inductance of the coil is 0.4 Henry
= 0, key K is closed. Choose the correct options for which
(d) The power developed in the circuit if a 2500µF capacitor
the galvanometer will show zero deflection at all times after
is connected in series with the coil is 25.28 W
the key is closed.

MARK YOUR 5. 6. 7. 8. 9.
RESPONSE 10.
584 IIT-JEE PHYSICS Challenger
11. A thermocole vessel contains 0.5 kg of distilled water at 14. A closed conducting loop, having resistance R, is being
30°C. A metal coil of area 5 × 10–3 m2, number of turns 100, rotated about an axis perpendicular to the magnetic field
mass 0.06 kg and resistance 1.6 is lying horizontally at the such that the flux of the magnetic field through the closed
bottom of the vessel. A uniform, time varying magnetic field conducting loop is continuously changing according to the
is set up to pass vertically through the coil at time t = 0. The graph shown in the adjacent figure. Then, which of the
field is first increased from zero to 0.8T at a constant rate following statement(s) is/are correct
between 0 and 0.2s and then decreased to zero at the same
rate between 0.2 and 0.4s. This cycle is repeated 12000 times. +10
Assume that no heat is lost to the vessel or the surroundings. Flux
Specific heat of the metal = 500 J kg–1 K–1 and the specific (in T-m²) Time
1 2 3 4 5 6 (in s)
heat of water = 4200 Jkg–1 K–1. Neglect the inductance of
–10
the coil. Then
(a) Induced emf is –2 V (a) The electric current through the loop is minimum (zero)
(b) Power dissipated is 3.5W at t = 1s, 3s and 5s.
(c) Heat dissipated during 12000 cycles is 12 kJ (b) The electric currrent through the loop is maximum at t =
0, 2s, 4s and 6s.
(d) Final temperature of the water under thermal equilibrium
is nearly 41°C (c) Total charge flown through any cross-section of a
closed conducting loop between 0 and 6s is zero.
12. Key is in position 2 for time t. Thereafter, it is in position 1.
(d) Total work done in rotating the loop in the magnetic
Resistance of the bulb and inductance of inductor we are
field is zero.
marked in the figure choose the correct alternative.
15. The arrangement shown is confined in a vertical plane has
two rails inclined at angle with horizontal. A horizontal
Bulb 2 rod of length moves on the rails with constant speed v,
in the region with transverse field B. Choose the correct
R2
Bulb 1 alternative(s). The rod starts moving at time t = 0.
L
R1
E

Key
1 2

(a) Bulb 2 dies as soon as key is switched into position 1


(b) Time in which brightness of bulb 1 becomes half its
maximum brightness does not depend on t

LE 2
(c) In t = , total heat produced in bulb 1 is
2 R22
2 B v sin
(d) Ratio of maximum power consumption of bulbs (a) At t = 0, current in the circuit is
depends on time r
13. Which of the following statements is/are correct regarding B v sin
(b) At t = , current in the circuit is
the electric field produced by a changing magnetic field? r
(a) It is non-electrostatic in nature. (c) At any time t (except at t = 0) point A is at higher
(b) It is non-conservative in nature. potential than point B
(c) Potential can be defined corresponding to this field (d) At any time t (except at t = 0) point D is at lower potential
(d) The lines of this field are closed curves than C

MARK YOUR
11. 12. 13. 14. 15.
RESPONSE
ELECTROMAGNETIC INDUCTION & AC CURRENT 585

16. A circular metal plate falls straight vertically in a region of


uniform magnetic field of intensity B, as shown in Figure 1
(a) The acceleration ‘a’ of the plate is g
2
(side view). The magnetic field is parallel to the surface of B d R2
the Earth and to the plane of the plate, the acceleration of 1+ 0
m
free fall is g, the plate has mass m, and its thickness d is
much smaller than its radius, R. Choose the correct options
1
d (b) The acceleration ‘a’ of the plate is g
B d 0 R2
2
1+
2m

R
B 2
|a g| 0B R2 d
(c)
g m
g
2
|a g| 0B R2 d
(d)
g 2m

MARK YOUR
16.
RESPONSE

1. A circular loop is kept in a space of magnetic field lines


B
perpendicular to its plane. If B is constant throughout the
surface area and its magnitude varies with time as shown in the (+)
figure; then t6
Column I Column II (–) t1 t2 t3 t4
(A) The magnetic flux is maximum (p) t4 t5
at time
(B) Induced current is maximum at time (q) t6
(C) Induced current is zero at time (r) t3
(D) Induced magnetic field supports (s) t2
the present field at

1.

MARK YOUR
RESPONSE
586 IIT-JEE PHYSICS Challenger
2. Consider the circuit shown in figure given below :

L = 4mH R=10 10 µF

~
V = 20 sin t

Column I Column II
(A) For = 5,000 rad/sec (p) The current in circuit leads the voltage
(B) For = 2,500 rad/sec (q) The current and voltage in circuit are in same phase
(C) For = 75,00 rad/sec (r) The peak current in circuit is less than 2A

(D) For = 5,000 rad/sec and R 20 in place of 10 (s) Voltage in circuit leads the current.

3. The two circular coils are arranged co-axially with the directions of current as shown in the figure. Then if

I1 I2
R R

front front
P Q

Column I Column II
(A) Coil P is moved away from Q (p) Current I2 increases
(B) Coil P is moved towards Q (q) Current I1 increases
(C) Coils P and Q are moved towards each other (r) Current I2 decreases
(D) Coils P and Q are moved away from each other (s) Current I1 decreases

2. 3.

MARK YOUR
RESPONSE
ELECTROMAGNETIC INDUCTION & AC CURRENT 587

4. Column I [Current in inductor coil (with time)] Column II [Voltage in inductor coil (with time)]

I + V
+
(A) t (p)
t
– –

I + V
+
(B) t (q)
– –

I + V
(C) (r)
+
t t
– –

I + V
t +
(D) (s) t

– –

5. In the circuit shown in the figure, switch S is closed at t = 0

C
2R

2C R
S

Column I Column II
(A) Ratio of the charge in capacitor 2C and C at any time (p) 1 : 2
(B) Time constants of both the capacitor are in the ratio (q) 1 : 1
(C) Steady state charge in C and 2C are in the ratio of (r) 2 ln 2 : 1

t 1
(D) Ratio when the current in the C is th (s) 2 : 1
4
of the maximum current in it

4. 5.

MARK YOUR
RESPONSE
588 IIT-JEE PHYSICS Challenger
6. Column I gives some incomplete statements. Column II gives some completing statements. Match them correctly.
Column I Column II
(A) A rod rotates in a uniform transverse magnetic field (p) may be zero
as shown, about hinge at O. Potential difference
between points A and B
B
× × × × ×

× × × × ×

× × × × ×A
O

× × × × ×
(B) A conducting loop is moved in a region of transverse (q) must be zero
constant magnetic field (infinite region), downward the
plane of paper, as shown.
Value of induced current i ..........
× × × × ×

× × × × ×

× × × × ×V

× × × × ×

× × × × ×
(C) If a constant force F is acting on the wire, rate of (r) may be negative
dq
change of charge q stored by the capacitor, ......
dt
× × × × ×

× +q × × × ×

× × × × F ×
–q
× × × × ×

× × × × ×
(D) A square loop is rotated about diagonal in a region of (s) may be positive
uniform magnetic field as shown. (t) must be positive
Value of i at an instant .........

× × × ×
i
× × × ×

× × × ×

× × × ×

× × × ×

6.

MARK YOUR
RESPONSE
ELECTROMAGNETIC INDUCTION & AC CURRENT 589

7. Let XL, XC be the inductive reactance and capacitive reactance and R be the resistance in each of the circuits given in column I.
Let VL, VC and VR be the rms voltage drop in each case and i be the rms current from mains. The supply voltage is 10V. XL = 10 ,
XC = 10 and R = 10 . Match the column correctly.
Column I Column II

L R
C

(A) (p) i = 1A

~
L

(B) (q) i = 0

~
L

(C) (r) VL = 10V


R

~
10

L C

(D) (s) VC = 10V

R
~
(t) VR = 10V

7.

MARK YOUR
RESPONSE
590 IIT-JEE PHYSICS Challenger
8. Match the column I and column II.
Column I Column II
(A) A charged capacitor is connected to the ends (p) A constant current flows through the wire
of the wire
(B) A wire is moved perpendicular to its length with (q) Thermal energy is generated in the wire
a constant velocity in a uniform magnetic field
perpendicular to the plane of motion
(C) A wire is placed in a constant electric field (r) A constant potential difference develops between the
that has a direction along the length of the wire ends of the wire
(D) A battery of constant emf is connected to the (s) Charge of constant magnitude appear at the ends of
ends of the wire the wire

MARK YOUR 8.
RESPONSE

1. Two parallel vertical metallic rails AB and CD are separated A horizontal metallic bar L of mass 0.2 kg slides without
by 1 m. They are connected at two ends by resistances
friction vertically down the rails under the action of gravity.
R1 and R2 as shown in figure.
A R1 C There is a uniform horizontal magnetic field of 0.6 tesla
L perpendicular to the plane of the rails. It is observed that
when the terminal velocity is attained, the powers dissipated
in R1 and R2 are 0.76 watt and 1.2 watt respectively. Find the
terminal velocity (in m/s) of the bar L. E
R2
B D

1.

MARK
YOUR
RESPONSE
ELECTROMAGNETIC INDUCTION & AC CURRENT 591

2. A pair of parallel horizontal conducting rails of negligible 3. An inductor of inductance 2.0 mH is connected across a
resistance shorted at one end is fixed on a table. The distance charged capacitor of capacitance 5.0 mF, and the resulting
between the rails is L. A conducting massless rod of LC circuit is set oscillating at its natural frequency. Let Q
denote the instantaneous charge on the capacitor, and I the
resistance R can slide on the rails frictionlessly. The rod is
current in the circuit. It is found that the maximum value of Q
tied to a massless string which passes over a pulley fixed to
is 200 mC. When Q = 100 mC, the value of |dI/dt| is found to
the edge of the table. A mass m, tied to the other end of the
be 10x A/s. Find the value of x.
string hangs vertically. A constant magnetic field B exists
4. A coil having 100 turns and area of 0.001 metre2 is free to
perpendicular to the table. If the system is released from
rotate about an axis. The coil is placed perpendicular to a
rest, calculate the acceleration (in m/s2) of the mass at the
magnetic field of 1.0 weber/metre2. If the coil is rotated rapidly
instant when the velocity of the rod is half the terminal
through an angle of 180°, how much charge (in coulomb) will
velocity. flow through the coil? The resistance of the coil is 10 ohm.

5. Two coils are wound on the same iron rod so that the flux
generated by one also passes through the other. The primary
has 100 loops and secondary has 200 loops. When a current
of 2 A flows through the primary, the flux in it is 25 x 10–4
L R
Wb. Determine value of M (in mH) between the coils.

6. Find the self inductance (in H) of a coil in which an e.m.f. of


10 V is induced when the current in the circuit changes uni-
m
formly from 1 A to 0.5 A in 0.2 sec.

2. 3. 4. 5. 6.

MARK
YOUR
RESPONSE
592 IIT-JEE PHYSICS Challenger

1 (a) 11 (d) 21 (b) 31 (a) 41 (b) 51 (b) 61 (d) 71 (a)


2 (c) 12 (c) 22 (a) 32 (a) 42 (a) 52 (b) 62 (a) 72 (a)
3 (b) 13 (c) 23 (b) 33 (a) 43 (c) 53 (a) 63 (a) 73 (b)
4 (a) 14 (b) 24 (c) 34 (c) 44 (b) 54 (a) 64 (d) 74 (c)
5 (b) 15 (b) 25 (a) 35 (a) 45 (b) 55 (c) 65 (d) 75 (b)
6 (a) 16 (b) 26 (c) 36 (a) 46 (c) 56 (d) 66 (d)
7 (c) 17 (c) 27 (b) 37 (c) 47 (c) 57 (c) 67 (b)
8 (d) 18 (c) 28 (a) 38 (a) 48 (c) 58 (d) 68 (d)
9 (d) 19 (a) 29 (a) 39 (c) 49 (d) 59 (d) 69 (a)
10 (c) 20 (c) 30 (a) 40 (b) 50 (c) 60 (d) 70 (a)

1 (a) 5 (c) 9 (c) 13 (a) 17 (b) 21 (a) 25 (b)


2 (a) 6 (b) 10 (a) 14 (b) 18 (d) 22 (a) 26 (d)
3 (a) 7 (a) 11 (a) 15 (c) 19 (c) 23 (b) 27 (c)
4 (b) 8 (c) 12 (b) 16 (c) 20 (b) 24 (d)

1 (a) 3 (d) 5 (a) 7 (d) 9 (c) 11 (d)


2 (a) 4 (b) 6 (c) 8 (d) 10 (d)

1 (b, c) 4 (b, c) 7 (a, b, c, d) 10 (a, b) 13 (a, b, d) 16 (a, c)


2 (b, c, d) 5 (b, d) 8 (a, b, c, d) 11 (a, c, d) 14 (a, b, c)
3 (a, b, c) 6 (a, c) 9 (a, d) 12 (a, b, c) 15 (a, c)

1. A-r; B-p; C-s; D-p, q 2. A-q; B-r, p; C-r, s; D-q, r


3. A-p, q; B-r, s; C-r, s; D-p, q 4. A-q; B-p; C-s; D-r
5. A-s; B-q; C-p; D-r 6. A-p, r, s; B-q; C-t; D-p, r, s
7. A-p, r, s, t; B-q, r, s; C-p, r, s, t; D-p, r, s, t 8. A-q; B-r, s; C-s; D-p, q, r

1 1 2 5 3 4 4 0.01
5 25 6 4
ELECTROMAGNETIC INDUCTION & AC CURRENT 593

1. (a) According to given problem,


E0
V V 5. (b) We know that, Z = I
I= = 2 ....... (1) 0
Z [R (1/ C ) 2 ]1/ 2
Given, E0 = 220 and I0 = 10
I V 220
and = ....... (2) so Z= = 22 ohm
2 [R2 (3 / C ) 2 ]1/ 2 10
Substituting the value of I from eq. (1) in (2), LM FG IJ OP =
4 R2 +
1
= R2 +
9 N6 H 6KQ 3
2 2÷ 2 2
C C E0 I0 220 10
pa cos = cos
1 3 2 2 2 2 3
i.e., = R2
C2 2 5 = 550 W
1/ 2 6. (a) When coil is broken into two identical parts, then
3 2 resistance of each part
R ÷
X (1/ C ) 5 3
So that = = = R 6
R R R 5 R' = = =3
2 2
2. (c) Resultant voltage = 200 volt
Inductance of each part
Since V1 and V3 are 180° out of phase, the resultant
voltage is equal to V2. L 1.8
L' = = 10 –4 0.9 10 –4 H
V2 = 200 volt 2 2
p L'
3. (b) According to Newton’s Second law, F = for Now, time constant =
t R'
perfectly reflecting surface
L L 1.8 10 –4
2 U 2 = = = = 30 s
p = = .A. t R R 6
c c 2
2
p 2 A
F= = Now, effective resistance when both coils are connected
t c in parallel
2 12 104 1.5 10 4 R' R'
R" =
= 8 R '+ R '
3 10
= 1.2 x 10–7 newton So, maximum current drawn from battery
4. (a) From Kirchoff’s current law, E 12
i= = = 8A
i3 = i1 + i2 = 3sin t 4sin ( t + 90°) R" 6 / 9
3 4
= 32 + 42 + 2(3)(4) cos 90° sin ( t ) 7. (c) i = 3 sin t + 4 cos t = 5 sin t cos t
5 5
4sin 90° 4
where tan = = 5 [sin ( t )] ...... (1)
3 + 4cos 90° 3
i1 i2 T2
i dt
5 T1
rms value = mean value = T2
i3 2
i3 = 5 sin ( t + 53°) dt
T1
594 IIT-JEE PHYSICS Challenger
initial value is not given hence the mean value will 12. (c) The circuit will have inductive nature if
be difference for various time intervals.
1 1
If voltage applied is V = Vm sin t then i given by eq. > L> ÷
LC LC
(1) indicates that it is ahead of V by where 0 < < 90°
which indicates that the circuit contains R and C. Hence (a) is false. Also if circuit has inductive nature
8. (d) When capacitance is removed the current will log behind voltage. Hence (d) is also
false.
L
tan = or L = 100 tan 60° ....... (1) 11
R If L=
÷ the circuit will have
when inductance is removed LC C
resistance nature. Hence (b) is false.
1
tan = Power factor
( C ) ( R)
R 1
1 cos = = 1 if L= .
or = 100 tan 60° ....... (2) 1
2 C
C R2 L ÷
C
so z = R = 100
13. (c) The intensity of light at a distance r from source is
I = v / R = 200 /100 = 2A
P
Power P = I 2 R = 4 100 = 400W given by =
4 r2
0.4 From electromagnetic theory
9. (d) Here, X L L 2 fL 2 50 40
= cu = c 0 E 2 RMS
R = 30
2
P ERMS 0 cP
Z = R 2 + X L2 = 302 + 402 = 50 I= = ERMS = 2
4 r2 0c 4 r
Vrms 200
I rms = = = 4A 7
Z 50 10 3 108 250
or ERMS = = 4.8 V/m 2
18
Erms 130
10. (c) i1rms = = = 10A 14. (b) L = 0.7H, R = 220 , E0 = 220V, v = 50 Hz.
X C2 + R12 13
This is an L-R circuit
Phase difference,
Erms
i2rms = = 13A XL L 2 vL
X L2 + R22 tan =
R R R
i1 R1 22
[XL= 2 vL = 2 × × 50 × 0.7 = 220 ]
7
i2
R2 220
= = 1 or, = 45°
220
Wattless component of current
Power dissipated I0 1 E0
= I0 sin = .
= i21rms R1 + i22rms R2 = 102 × 5 + 132 + 6 2 2 Z
= power delivered by battery
= 500 + 169 × 6 = 1514 watt 1 220 1 220
= .
11. (d) We know that, 2 X L2 R2 2 220 2 2202
R
cos = 1
2 = = 0.5 A
1 2
R2 L ÷
C 15. (b) Q = Q0 e t/
= Q0 /

1 1
at resonance L = . or e t/
=
C
R t
So cos = =1 or et / or ln
R
ELECTROMAGNETIC INDUCTION & AC CURRENT 595

16. (b) If we consider the cylindrical surface to be a ring of 20. (c) The magnetic field at a distance x from the wire
radius R, there will be an induced emf due to changing
field. 0i
B (x) =
2 x
d dB
E.d = = A EMF induced in an element of length dx
dt dt
at a distance x from wire = Bvdx
dB dB Total EMF induced in the rod
E (2 R ) A R2
dt dt
0.2 0.2 0.2
R dB 0iv 0 iv
1
E= E= Bv dx = 2 x dx = 2 x dx
2 dt 0.01 0.01 0.01
Force on the electron
0 iv
F Ee
eR dB E=
2
[loge x ]0.2
0.01
2 dt
1 eR dB 0 iv
acceleration = = [log10 (0.2) – log10 (0.01)] × 2.303
2 m dt 2
As the field is increasing being directed inside the paper, 4 10 –7 5 10
hence there will be anticlockwise induced current (in E= [1.301] × 2.303
2
order to oppose the cause) in the ring (assumed). Hence
there will be a force towards left on the electrons. = 2.99 × 10–5 V 30µV
17. (c) When the rod rotates, there will be an induced current 21. (b) I I 0 (1 e t/
)
in the rod. The given situation can be treated as if a rod
where time constant
‘A’ of length 3 rotating in the clockwise direction,
while an other rod say B of length 2 rotating in the 3 t/
I0 I 0 (1 e )
anticlockwise direction with same angular speed . 4
1 3 1
As e B 2
l e t/
e t/
=
2 4 4
1 1 t 1 4
For A : e A B (3 ) 2 and eB B( ) (2 )2 ln e = ln 2 ln 2
2 2 4
Resultant induced emf will be
2
1 5 =
e = eA + eB = B 2 (9 4) or e B 2 ln 2
2 2
18. (c) Flux can’t change in a superconducting loop. dB
= 2 R2.B 22. (a) E = – NA ÷
dt
Initially current was zero, so self flux was zero.
Finally Li = 2 R2 × B E NA d
i= = (75e –200t) × 10–4
R R dt
2 R2 B
i= NA
L = [75 × (–200)e–200t] × 10–4
R
b b
0I
19. (a) Induced emf = Bvdx = vdx 50 7 10 8
a a
2 x =+ [15000e–1]
30
0 Iv b 5 5
Induced emf = ln ÷ 175 10 175 10
2 a = = = 0.64 × 10–3
e 2.73
E2 A = 0.64 mA
Power dissipated =
R
2
E2 r2 d r2 d
E2 23. (b) P= = ÷ = ( NBA)2
Also, power = F. v F= R dt dt
vR
2 2
1 0 Iv b r2 dB
F= ln ÷ = N 2 A2 ÷
vR 2 a dt
596 IIT-JEE PHYSICS Challenger

N 2r 2 B= 0
.
i
Pµ 2 x
...(ii)

From eqs. (i) and (ii),


N 2r 2
Case 1 : P1 µ 0i
de = v dx ...(iii)
2 x
(4 N )2 ( r / 2) 2 The emf e induced in the entire length of the rod PQ is
Case 2 : P2 µ given by
4
Note : When we decrease the radius of the wire, its Q
0 i
e = de = v dx
length increases but volume remains the same] P 2 x
P1 1 r+L
= 0 i
P2 1 = v dx
r 2 x
Power remains the same.
r + L dx
d d dB = 0
iv
24. (c) eind = ( BA) = A 2 r x
dt dt dt
0 iv
d = [log e x]rr + L
= A (4t + 2t + 5) = A (8t + 2)
2 2x
dt
0 iv
= 0.06(8t + 2) = 2.52 at t = 5 = [log e ( r + L) log e r ]
2
2 eind
i= = 0.26 A iv r+L
2 = 0 log e ÷
2 r
di µ0 N 2 A 27. (b) Since MN is perpendicular to the magnetic field as well
25. (a) eback= L where L =
dt l as velocity ‘v’, hence, the induced emf developed along
MN is given as
µ N 2 A 1.5 0
eback = 0 v
l 1 103 E = eBa , (directed from M to N)
c
4 10 7 (400) 2 20 10 4 The induced emf results due to decrease of flux as the
= (1.5 103 ) = 0.3 V
20 10 2 frame leaves the region occupied by the magnetic field.
According to Lenz’s law, the direction of induced
26. (c) current is clockwise. All this results in magnetic force
v acting on the loop which is equal and opposite on
P Q
r sides MQ and NP, hence cancel each other.
Net force is the magnetic force on MN given by
L
I
F= B a (directed upwards)
x c
x + dx The constant terminal falling velocity is achieved when
F is able to completely balance the force of gravity on
Figure shows a rod PQ of length L which moves with a the loop.
uniform velocity v parallel to a long straight wire Let A = cross-section area of the loop/frame
carrying a current i. Here a magnetic field is produced Then mass of the frame is given as
by the current carrying wire and the rod moves in this
1 a
field. Consider a small element of length dx of the rod m 4a A and R = 4
at a distance x and (x + dx) from the wire. A
The emf induced across the element E v eBv A
I= = eBa A=
de = B v dx ..(i) R c 4a 4c
We know that magnetic field B at a distance x from a
I eB 2 a v A
wire carrying a current i is given by Hence F = Ba =
c 4 c2
ELECTROMAGNETIC INDUCTION & AC CURRENT 597
Equating F to mg, we get
33. (a) i = 5A
eB 2 a v A 16 c 2 g 2
4a A g = v= qm q2 1 2
4 c2 e B2 = + Li qmax = 6C
2C 2C 2
(v is independent of geometrical dimensions of the t
frame.) 20 3
34. (c) I1 = 1– e 5 10 4 ÷ = = 1.5 A
10 ÷ 2
d R2
28. (a) = B.
dt 2

R2
where is area swept in unit time perpendicular to
2
the magnetic field. L = 5mH I1 20V
29. (a) Rate of change of area of the loop
dA dT dT 3
A, A.(2 ) 2 10 –6 1
dt dt dt 4
= 11.5 × 10–6m2/s 6

d .dA
emf =– = –1.5 10 –6 V
dt dt
current in the loop = 1.5 × 10–6A
The direction will be anticlockwise as the induced
20V I2
current will try to negate the increase in fluix due to
increase in area.
3
30. (a) = 2
B
4
C = 0.1 mF
d 3 dB
= = 2
20
t
dt 4 dt I2 = e 10 3 = 1.0 A
10
2
3 From superposition, I = I1 + I2 = 2.5A
i= =
R 4R 5R 6E
35. (a) Req I= = 1A
iR0 1 6 5R
31. (a) =
(iR0 ) + (ic0 – iL0 )
2 2 2 d
36. (a) e = [ 2 + 8t + 0]
dt
0
1
iC iR 8t=2 t= sec = 0.25 sec
0 0 4
t d
µ0 3M
37. (c) B = . 3 on the axis of a magnet ; e = –
4 x dt
iL 38. (a) Spring coil will attract each other; spring will contract
0

1
0 /R
=
1 kµ
2 2
( 0 / R)
2
0 C–
0

XL 1
39. (c) tan = =
6 R 3
3 R=3
R=
1 XC
C– ÷ tan = =1
L R
di Et
32. (a) L =E or i 0 t t0 =
4
dt L
598 IIT-JEE PHYSICS Challenger
40. (b) I = IC + (– IL)
Because IC and IL are 180° out of phase with each 48. (c) V = I (20 + 10)2 + ( X L + X L X C )2
other.
1 2 1 100 = 2 (30)2 + (30 + X L 40)2
41. (b) dU = LIdi = Li ;U = 4 16 = 32J
2 2 (50)2 = (30)2 + (XL – 10)2
42. (a) Apply Lenz's law such that the net force on loop will (XL – 10)2 = 80 × 20 : XL – 10 = 40; XL = 50
be towards the infinite long wire. 49. (d) Equivalent circuit can be drawn as below
43. (c) emf, 1 (due to varying B) = B0 vt . A
to motion of PQ) = B0 vt 1µF
2 (due R
= 1µF
1+ 2= 2B0 vt 1µF
1µF
1µF B
2
4B02 2 2 2
v t
Fv = =
R R 1µF R
4 B 02 2 v t2
F= A
R R
m B
44. (b) I(t) = v0 sin t where w = 3µF B
L mL

mv02 R
I=
2L RC = 3 × 10–3 sec
45. (b) The three branches of the circuits carry currents R × 3 × 10–6 = 3 × 10–3
i = i0, i = i1 et/RC and i =i2 2e–t/2RC respectively. R = 1000
The current through the cell, i (t) can be found by 50. (c) If ZL = ZC current will be same,
using Kirchhoff 's current law (or mode law).
46. (c) A circuit equivalent to the given circuit can be drawn So, VZ L = VZ C ;
as follows :
Req 1
VL = 1 × 2 × 30 × = 60 Volt

VR = 80 × 1 = 80 Volt;
E
V = VL + VR = (80) + (60) = 100 Volt
2 2 2 2
L
51. (b) Here, areasquare > areaquarter circle > areatriangle
K
d
so of B2 > B1 > B3
dt
2R 2R
Where Req = R + = R+R 52. (b) I C
4R q
Req = 2R I
I
L
Time constant = ;
2R
47. (c) When the current in the loop A increases, the magnetic L
lines of force in loop B also increases as loop A is I
placed near loop B. This induces an emf in B in such a I I
direction that current flows opposite in B (as compared
to A). As it can be easily seen by the direction of I that Q is
Since currents are in opposite direction, the loop B is decreasing thus, energy of capacitor is decreasing and
repelled by loop A. 1 2
hence, energy of inductance is increasing or LI ÷
2
gives that I is increasing.
53. (a) At resonance XL = XC and Z = Zmin = R
1
B A XL = L and = XC
C
ELECTROMAGNETIC INDUCTION & AC CURRENT 599

If ‘f’ is decreased, will decrease and hence XC will


V L
increase therefore at f < fr, circuit behaves as capacitive. But I 0 = and =
R R
VL and VC always differ in phase by 180° at any
frequency. V 12 6t /8.4 10 3
I (1 e Rt / L ) I e
54. (a) At any time t, the side of the square a = (a0 – t), R 6
where a0 = side at t = 0. = 1 (given)
At this instant, flux through the square : t = 0.97 ×10 – 3 s 1ms
= BA cos 0° = B (a0 – t)2 r
60. (d) A B
d
emf induced E = –
dt I v
E = – B.2 (a0 – t) (0 – ) = +2 aB
D C
(O) E As flux decreases the flux current in loop is clockwise.
Force on DA due to long wire and BC is towards left
R while on wire BC is towards right.
0.1 0.2
1
55. (c) E R 61. (d) Y2 = y 2 dt + y 2 dt
0.2 0 0.1
R
0.1
S 200t 100t
100 (1 e 2e )dt
(+iR) 1 0
= 0.2
0.2
E E 4E + 100e 100 (t 0.1)
dt
i= ; VS = E + iR = E + VS = 0.1
3R 3 3
56. (d) Consider the expression for the current rising 0.1
200t 100t
exponentially in the LR circuit. The time constant is (L/ 100 (1 e 2e )dt
1 0
R). In this case the curve (1) is rising faster than curve = 0.2
0.2
(2) indicating that (L1 / R1) < (L2 / R2). However, in + 100e 100 (t 0.1)
dt
both the cases the maximum current is the same and 0.1
equal to (V/R1) or (V/R2), which means R1 = R2.
100t 0.1
57. (c) t 0.005e 200t
+ 0.02e
= 500 0
i2 100 (t 0.1) 0.2
0.01e
0.1
= 500 {[0.1 – 0.005 e–20 + 0.02 e–10) – (0 – 0.005
+ 0.002)] + [(– 0.01e–10) – (0.01)]}= 500 × 0.095 = 47.5
i1 Y = 47.5 = 6.9
62. (a) At resonance
i3 = i12 + i22 and i3 < (i1 + i2 )
E0 IR E0 E0
IR = =
E R 2 2R 2
58. (d) Current in inductor = 1
R R2 L ÷
C
1 LE 2 1 1
its energy =
2 R2 1L R and 2L = +R
1C 2C
Same energy is later stored in capacitor
1 1
L( 1 2)
1 2
÷C 1 2 =
2 2 LC
Q 1 LE E 1 2
Q = LC
2C 2 R2 R
1
1) + ÷ C = 2R
2 1
59. (d) Using I = I0 (1 –e–t/ ) and L ( 2
1 2
600 IIT-JEE PHYSICS Challenger
Putting value of 1 2 we get
XC XL
R R tan =
2 1 f 2 f1 = R
L 2 L tan = tan 2 – tan 1
63. (a) Induced current will be such that it will try to oppose 68. (d) For induced emf to be zero, flux at any time t should be
cause of its own production. constant.
c
B vt = constant, B=
vt
N S N S
69. (a) Consider an elemental circle of thickness dr.
A
t

S N N S dr
r
B

64. (d) Due to variation in magnetic field there will be an


induced current. Due to short circuiting bulb B goes
out and resistance of circuit decreases hence current The induced emf in the circular path of radius r is
in A increases.
d
65. (d) Current in the inductor before opening ‘S’ ( r 2 B) r2
dt
L I S The resistance of circular path is
12 r
R=
tdr
R1 =4 R2 =8
12V The length of the path being 2 r and tdr is the cross
sectional area of current flow. For the element the power
dissipated inside the path is
2
t
dP = = 2 3
r dr
12 9 R 2
I= = = 4.5A
4 8 2 The total dissipated power P is
÷
4+8 a
t 2 t a4 2
Since current in inductor does not change instantly, P r 3dr
2 8
therefore, just after opening ‘S’, 0

70. (a) The vector diagram of the potential differences across


A L B I the elements of the circuit shown in question is
presented in Figure. Note that since the same current
– VL + flows through R and C, the vectors corresponding to
the voltages on R and on C are perpendicular,
R1
12V VR VC .

VR VC
12 + VL – IR1 = 0 VAB
12 + VL – (4.5) (4) = 0
1 A 1
VL = 6V with ‘B’ at higher potential. Vr = 2 V Vr = 2 V
66. (d) Perpendicular length is more, so induced emf is more.
The two resistors r are equal, and the current through
X X
67. (b) tan 1 = L ; tan 2 = C 1
R R them is the same, hence | V r | = | V | . The potential
2
ELECTROMAGNETIC INDUCTION & AC CURRENT 601

difference between the points A and B is the magnitude


qE qv B = 0 ....... (1)
of the vector V AB . Since the angle at vertex B is 90°, a where q is the charge of the electron. From (1), which
well known geometrical property tells us that states that the Lorentz’s force is 0, we obtain
| V AB | = | V r | = | V / 2 | , and A is the center of the E B v ....... (2)
circle circumscribing the triangle of vectors. Therefore we conclude that inside the ball the electric
field is uniform and has the magnitude
Hence the magnitude of V AB does not depend on the
| E | | B | | v | sin ....... (3)
value of R, since it always equals the radius of the
The maximum value of the potential difference max
circumscribed circle, which is half of | V | . can be found by taking the longest path inside the ball,

As R changes the vector | V AB | will rotate around A, that is the diameter parallel to the vector E . Finally, we
get
although it’s magnitude will not change. Therefore, this
circuit is suitable for changing the phase between the max = | E | 2r 2r | B | | v | sin ....... (4)
With = /4, B = 1T, v = 1 m/s and r = 1 cm, the
input voltage V and the one picked up in between A
maximum potential difference is max 14mV, which
and B. is easily detectable.
71. (a) The glass ring has no mobile electric charges to carry 75. (b) When the circuit comes to equilibrium, voltage VC and
an electrical current. charge qC of capacitor will not change any more. For
72. (a) As the magnetic field is parallel to the axis of the the source of alternating current it means that the
cylinder, the number of lines of forces cut by the wires charge increase q in capacitor during the charging
is zero. Therefore the potential difference developed is time interval will be compensated by the charge
zero. decrease during discharging. Apparently, we have a
capacitor with big capacitance.
73. (b) According to the problem situation, current through
A v the circuit alters very slightly during the charging
h process, and can be calculated as an average value
×× × × × ×b× × × × × ××
×× ××××××××× ××
×× ××××××××× ×× q0
××
××
× ×B× × × × ×C× ×
×××××××××
××
×× I1 =
×× ××××××××× ××
×× ××××××××× ××
Voltage across the capacitor during charging is given
by
dA
B. qR q0
I= =
| d / dt |
= dt VC = I R R = R and VC V0 r
R R R
qR q r q
= V0 r + ÷ = V0 VC r ....... (1)
d 1
B h.b÷ Bb dh R
dt 2 dt µ Bbv
= µ If we assume that during discharging time T no current
R R leaks through the source, charge from capacitor leaks
bµt through the second resistor with resistance R, with
Iµt average current I2
74. (c) Because of the motion of the metal ball in the uniform q
I2 =
T
magnetic field B the electrons are subjected to the
Voltage across capacitor during discharging is given
action of the magnetic force. As a result the free
by
electrons spread on the exterior of the ball, in such a
way that the electric field created by the charge q
VC = R ....... (2)
distribution inside the ball is uniform and compensates T
the effect of the magnetic force. Once the equilibrium Combining Eqs.(1) and (2)we can calculate the unknown
is achieved, the redistribution of the electrons stops. voltage, as follows:

The intensity of the electric field E in such R


VC = V0
circumstances obeys R + rT + r
602 IIT-JEE PHYSICS Challenger

For (Qs. 1-2) For (Qs. 7-9)


Magnetic field due to straight current carrying V V
conductor is given by – + + –

0 i0 C 3C
B
2 r S
L
Voltage induced de = B v dr
The total voltage induced can be obtained by
integrating the above expression. Thus di
When current is maximum =0
e b b dr
dt
0 io 0 i0 v
de v dr = emf across L = 0 and potential difference across the
0 a 2 r 2 0 r
capacitor will be same.
i v b From conservation of charge
e = 0 0 log e ÷ 3CV + CV = 6CV0 – CV0
2 a
5CV0
e i v b V=
i = = 0 0 log e ÷ 4
R 2 R a
Loss in energy of capacitor = energy stored in inductor
1. (a)
2 (a) 3V0 3C
I max =
3. (a) Force required for constant velocity 2 L

F = iB l , 7. (a)
8. (c)
where B is the average magnetic field. 9. (c)
10. (a) When S is closed, 1 = RC
b dr
0i0
0 i0 log e (b / a )
3R
B= 2 a r = When S is open 2 = C
b 2 (b a) 2
dr
a
C1 2
=
and l (b a) C2 3
11. (a) Energy stored in C before discharging starts Ui =
0 i0 v log e (b / a) 0 i0 log e (b / a)
F (b a) 1
2 R 2 (b a) CE 2 . When discharging starts, heat production in
2
2 the equivalent circuit is
0 i0 log e (b / a ) v
=
2 R 2
i i2 R
For (Qs. 4-6) H1 = ÷ R=
2 4
dv iLB BL
= = ( E BLv ) i
2
i2 R
dt m mR H2 = ÷ R=
2 4
E B 2 L2t / mR
u 1 e ÷ H3 = i2R
BL
H1 1
E =
vT = at t H1 + H 2 + H 3 6
BL
4. (b) R 5R
12. (b) When battery has a resistance , = C , current
5. (c) 2 4
6. (b) through the capacitor is maximum at t = 0.
ELECTROMAGNETIC INDUCTION & AC CURRENT 603

6E dP V2 R 2R
The current through the battery at t = 0 is and =0 e Rt / L
+ e 2 Rt / L
=0
5R dt R L L
2E
through the C at this instant is 1
5R e Rt / L
=
2
2E 4t / 5 RC Rt L V V
I = f (t) = e ln 2 t= ln 2 ; I (1 e Rt / L
)=
5R L R R 2R
13. (a) In 'P'

= B A cos ° = BA(max)
18. (d)
dPR 2V 2
dt L
1 e Rt / L ( )
Power will be maximum at certain time and then
decreases.
14. (b) trestoring = mg sin mg
19. (c) iLB + Kx = ma
mg g dq dq dV
= 2 ÷ i= ; q = CBLV i= = CBL = BLCa
I m dt dt dt
B2L2Ca + Kx = ma
g 10
T= 2 , = = = 5 rad / sec Kx
g 2 a=
m + B 2 L2C
0 sin t ... (1)
dV Kx
20. (b) V
Now, e =
d d
( BA) cos = BA sin dx m + B 2 L2C
dt dt
e = BA sin ( t) V2 V02 Kx 2

d
2 2 2 ( m + B 2 L2 C )
e = BA. ÷ .sin t
dt
m + B 2 L2 C
e = BA ( 0 cos t) sin t from (1) For xmax, V = 0 ; xmax = V0
K
= BA ( 0 cos t) ( 0 sin t) taking (sin ) 21. (a) a – x SHM
For (Qs. 22-24)
2
BA. 0 .sin 2 t
e= V L0 = 3, V R0 = 4 e0 = VL20 + VR20 = 5 volts
2
4 If i = i0 sin t, VR = 4 sin t,
1 4 10 sin (2 5t) 10 4
e= VL = 3 cos t
2 4 e = 5 sin ( t + 37°)
4 sin t = 2 t = /6
2 5
= sin(2 5t ) 10 8 VL = 3 cos t = 3 cos 30°
2
e = 5 sin ( t + 37°)
= 5 5 sin (2 5t ) 10 9 volt. 4+3 3
5 sin (37° + 30°) =
emax 10 2
15. (c) i = 5 5 10 A
R i is decreasing i.e. sin t is decreasing
e = 5 sin ( t + 37°)
V2 Nothing can be said about it.
16. (c) PR = i2R = (1 e RT / L )2
R 22. (a) 23. (b) 24. (d)

1 2 V
17. (b) E= LI 25. (b) I woofer f XL I woofer
2 R + X L2
2

Rt Rt
dE 2 LI dI LV V
= P 1 e L e L ÷ I tweester
V
,f XC I tweeter
dt 2 dt R L ÷
R + X C2
2
604 IIT-JEE PHYSICS Challenger
26. (d) I rms1 = I rms2 in inductor current lags voltage by

1 R
1 1 cos
= X L2 + R2
2
( L) 2
R 2
(1/ C ) + R 2 2

1 2 90 cos ( 1 2) =0
1
f =
2 LC tan 1 tan 2 =1
27. (c) Voltage is same for both. In capacitor current leads
XC XL
=1
1 R R R
voltage by cos ,
X C2 + R 2 L = CR2

1. (a) In series resonance circuit, current becomes maximum So this is infinite for D.C. (f = 0) and has a very small
because total impedance becomes zero. In case of LC value for A.C. Therefore, a capacitor blocks D.C.
circuit,
6. (c) Faraday’s laws of electromagnetic induction are
1 consequences of conservation of energy. It involves
Total impedance = L =0
C only transformation of energy into electrical energy.In
purely resistive circuit, current and voltage are in the
1 1 same phase.
L 2
=
C LC
d
2. (a) All electromagnetic waves have same speed in vacuum. 7. (d) The – ve sign is – signifies the direction of induced
dt
X-rays is a high energy electromagnetic wave.
emf.
1
3. (d) At resonance XL = XC or L= . Because of this 8. (d) Resistance offered by an inductor in a d.c. circuit at
C
t = 0 is infinity, which decreases to zero at steady state
impedance of LCR circuit become equal to resistance

of circuit ( Z = R 2 + ( X L X C ) 2 ). 9. (c) i i0 (1 e t/
) at t = 0, i = 0

E E
Therefore, from I = = , at resonance, current in di i0 L
Z R e L = e t/
at t = 0, e 0
dt
LCR circuit is maximum. Correspondingly phase angle
is also equal to zero. Therefore, emf and current are in 10. (d) Peak voltage across the resistance can be equal to the
phase in LCR circuit.
voltage of source at resonance condition in LCR series
circuit. At resonance condition circuit behave as
4. (b) The power factor in coil is Pav = Ev I v cos is zero
2 amplifier. i.e. Peak voltage across the inductor is greater
because of which energy loss in coil is reduced. than the peak voltage of the source in an series LCR
5. (a) The capacitive reactance of capacitor is given by circuit.

1 1
XC = = 11. (d) Emf will be induced between centre and rim of disc.
C 2 fC
ELECTROMAGNETIC INDUCTION & AC CURRENT 605

1. (b, c)
xBv Bv
Effective circuit is shown in the figure : i= =
Applying Kirchhoff loop law equations, we get : 2 (x 2x ) (1 + 2)
di1 di 4. (b, c)
L1 –M 2
dt dt Let I1 & I2 be current through B1 and B2
di2 di R2 + X c2 = 220
L2 –M 1 I1
dt dt
Solving these we get :
I1 R 2 + X c2 R2 (1/ C )2 R 2 + 40
di2 ( M L1 ) = = =
= I2 R 2 + X c2 R 2 + ( L) 2 R 2 + 9.87
dt L1L2 – M 2
I2 > I1
di1 ( M L2 ) B2 will be brighter than B1
=
dt L1L2 – M 2 As frequency increases, XC decreases, XL increases.
Solving these we get : So, I2 becomes less than I1. Hence brightness of B1 will
increases and that of B2 will decreases.
di ( M L1 ) ( L + L2 + 2M )
= 2
= 1 5. (b, d)
dt L1L2 – M L1L2 – M 2
Magnitude of induced electric field due to change in
magnetic flux is given by
di L1 L2 – M 2 di
= Leff = ÷
dt L1 + L2 + 2 M ÷ dt E.d =
d
=A
dB
( N = 1 and cos = 1)
dt dt
L1 L2 – M 2
Leff = dB
L1 + L2 + 2M or E. R 2 (2 B0t ) = 2 B0t ÷
dt
If M is negligible
Here, E = induced electric field due to change in
LL
Leff = 1 2 magnetic flux
L1 + L2 E (2 R) = 2 R2B0t
2. (b, c, d) or E= B0Rt
C1 C2 = 2CR = C Hence, F = QE = B0QRt
This force is tangential to ring. Ring starts rotating
q1 = ( EC )(1 – e – t / C )
when torque of this force is greater than the torque
and q1 = (2 EC )(1 – e – t / C ) due to maximum friction (fmax = µmg)
q1/q2 = 1/2 or when F fmax
Ratio of steady charge is also 1 : 2
dq1 EC – t / dq1 2 EC – t / Taking the limiting case, F fmax or
÷ = e C
and ÷ = e C
dt C dt C F.R = (µmg) R
It is given that ring starts rotating after 2 seconds.
dq1 dq2
dt dt ÷ So, putting t = 2 seconds, we get µ =
2B0 RQ
mg
B x x
3. (a, b, c) = BA = 6. (a, c)
2 2 2
The emf induced in each of the vertical sides of frame P
d 2 x dx x will be E1 = B ( ) 2v, with upper point at a lower
= = B = Bv
dt 4 dt 2 potential, while the emf induced in each of the vertical
d 2 x dx x d (2 y ) sides of frame Q will be E2 = B (2 ) v, with upper point
= = B= B at a higher potential. The emf induced are as shown
dt 4 dt 2 dt
606 IIT-JEE PHYSICS Challenger

R4 di2
L1 L2 = 0 as 0
R3 dt
– +
E2 L1 R
E1 = 3 ....... (4)
L2 R4

S S' di2 E
At t = , = 0 , i2 = = constant
7. (a, b, c, d) dt R2 + R4 ÷
= 1000, XL = L = 2000
R4 R1 R1 R
1 106 R2 =0 = 3 ....... (5)
XC = = = 1000 , R = 1000 R3 R2 R4
C 1000
L1 R R
From equations (4) and (5), = 1 = 3
Z (XL X C ) 2 + R 2 = 1000 2 L2 R2 R4

R 1 9. (a, d)
Power factor cos = = = 0.707 In the figure, two pole faces are shown.
Z 2
2cm
V 200 / 2
I rms = rms = = 0.1 A
Z 1000 2 2cm
XL – XC positive inductive circuit voltage leads
8. (a, b, c, d)
Since there is no current through BD therefore points
B and D are at the same potential Flux through each face = 5µWb
VB = VD
5
Potential difference, VAB = VAD B × (2 × 10–2)2 = 5µWb B= 10 2 T
4
di1 di
L1 + i1 R1 = L2 2 + i2 R2 ....... (1) (i) Now a conductor moves at 90° with B
dt dt
Emf induced = B v
Similarly, VBC = VDC
i1R3 = i2R4 ....... (2) 5
= 10 2 ÷ (2 10 2 ) 12 = 3mV
L1 R1 4
B R3
i1
× × ×
G v
L2 R2 R4 2cm. × × ×
A C
i2 D × × ×

(ii) The conductor moves at 30° with B . Hence


K
velocity perpendicular to B is v sin 30°.
From equations (1) and (2),
emf induced = B (v sin 30°) = 1.5mV
di1 di There will be no induced emf due to velocity
R3 = 2 R4
dt dt v
component v cos = , = 60°
di R di R 2
or 1 = 4 2 and i1 = i2 4 Hence the conductor moves at an angle of 60° to
dt R3 dt R3
the magnetic field.
R4 di2 RR di 10. (a, b)
L1 + i2 1 4 = L2 2 + i2 R2
R3 dt R3 dt In case of a coil as Z = R2 2 2
L

R4 di2 R1 R4 V V
L1 L2 ÷ i2 R2 ....... (3) i.e., I = =
R3 dt R3 Z R 2
2 2
L
At t = 0, i2 = 0 So when DC is applied as = 0,
ELECTROMAGNETIC INDUCTION & AC CURRENT 607

V 12 Heat dissipated in 12000 cycles is


I= , i.e. R = =3 H = 2.5 × 0.4 × 12 × 103 = 12 × 103 J = 12 kJ ... (1)
R 4
Heat gained by water = mwsw + mwireswire
and when AC is applied,
i.e. H = 0.5 × 4200 + 0.06 (500) .... (2)
V V 12
I= , i.e., Z = = ÷ =5
From eq. (1) and (2), 11.26°
Z I 2.4
f i 30°C + 11°C 41°C
or R 2 + X L2 = 52 (as Z = R + 2
X L2 ) 12. (a, b, c) i2 i1
So, X L2 52 R2 52 32 = 42
i.e. XL = 4 . R2

XL
4
but as XL = L, L = =
= 0.08 Henry R1 L
50
Now when the capacitor is connected to the above
circuit in series,
1 1 103
as XC = = 6
= =8
C 50 2500 10 125 t
R
E L 2÷
At time t, i1 = 1 E
so, Z = R 2 + ( X L X C )2 = 32 + (4 8)2 5 R2 ÷

V 12 (t )
and hence, I = = = 2.4 A R1
So, i1 = i2 = i2 max ; i2 (t) = i1e L
Z 5
R i1
So, Pav Vrms I rms cos ( I rms Z ) I rms ÷ Now for power become half i2 (t ) =
Z 2
i.e., Pav = I rms
2
R = (2.4)2 3 = 17.28 W (t )
i1 L 1
R
11. (a, c, d) = i1 e (don’t depend on t)
2
Here = B.A
E
When t , i1 =
R2
So power in R2 generated
B0.8
2
1 1 E L E2
L [i1 (t )] = L
2
=
2 2 R2 ÷ 2 R22
13. (a, b, d)
t
O 0.2 0.4 As field is non-conservative potential is not defined.
14. (a, b, c)
d dB 0.8 3
AN 5 10 100 d
dt dt 0.2
= – 2 volt dt
1d
I= = 1.25 amp i= =
R R R dt
P = I2R = ((1.25)2 × 1.6) = 2.5 W dq 1d
or
dt R dt
d
At t = 1, 3, 5, 0 i=0
dt
2.5W
d
At t = 0, 2, 4, 6, max i = max
dt
1
dq d
t R
O
608 IIT-JEE PHYSICS Challenger
of the plate in the magnetic field the electrons are
| |
q= subject to Lorentz’s force FL = evB (where the e is the
R magnitude of the charge of the electron). As a
Total 0, q = 0 , induce emf is such that it tries to consequence the electrons will move in the plate until
oppose the cause of its own production the electric force FE = eE (where E is due to the
redistribution of charge in the plate) balances Lorenz’s
W 0
force. Therefore E = vB.
15. (a, c) A The thickness of the plate is much smaller than its
C
radius, therefore the electric field inside the plate may
be considered uniform (neglect the edge effects). The
plate behaves as a plane capacitor, with the potential
r difference, V = Ed = Bvd, and with the charge

0S
Q = CV = Bvd = 0SBv
d
r Here S = R2 is the area of one of the plate’s faces. We
D B
notice that the charge depends on the velocity v. If the
velocity changes so does the charge, that is a current
will flow. We have
Q v
I= = 0 SB 0 SBa
t t
E=B v sin
where ‘a’ is the acceleration of the plate. The force on
the plate due to the interaction of the current I with the
V
magnetic field B is F = BId and is oriented upward
×B (opposite direction to the direction of motion). From
Newton’s second law we now have
ma = mg – BId
At t = 0 ma = mg – 0B2a R2d
E B v sin The acceleration of the plate is therefore

E 2 B v sin 1
i= = a=g 2
r/2 r B d R2
1+ 0
When capacitor is charged, then m
VA – VB = B v sin . For the numerical estimation we compute the ratio
16. (a, c)
2
Let us consider for the beginning the simpler case of a |a g| 0B R2 d
plate falling with constant velocity v. Due to the motion g m

1. A-r; B-p; C-s; D-p, q Time constant of the capacitor 2C and C are 1= R (2C) and
2. A-q; B-r, p; C-r, s; D-q, r 2 = (2R) C.

1 1
Resonant frequency is 2
c = .
1
= ; q1 = CE 1 e t / 2 RC
LC 2 1
(i) if < c, capacitor dominates
t / 2 RC
(ii) if > c, inductor property dominates q 2CE 1 e
2
3. A-p, q; B-r, s; C-r, s; D-p, q
q2 2
4. A-q; B-p; C-s; D-r =
q1 1
di
Use e = – L ( q1 ) steady state CE 1
dt At steady state = =
5. A-s; B-q; C-p; D-r ( q2 ) steady state 2CE 2
ELECTROMAGNETIC INDUCTION & AC CURRENT 609

Current in C is given by i = imax et/2RC. iC


imax
When i = then In case (B); VL or VC
4

imax t iL
= imax et / 2 ln 2 =
4 iC = iL i=0
6. A-p, r, s; B-q; C-t; D-p, r, s VR = 0 ; VL = VC = 10V
In case (C); i = 1, VL = VC = VR = 10V
d
e . In case (A), flux changes wrt time in case (B), flux VL
dt
is constant. iL or iC
In case (D),
In case (C), e = Bv , q = CBv ,
VC
dq dv
= CB = + ve const.
dt dt VL = VC = 10V
Net voltage across inductor and capacitor = 0
In Case D, Angle between B and A changes with time
10
hence i changes with time. i= = 1 amp , VR = 10
10
7. A-p, r, s, t; B-q, r, s; C-p, r, s, t; D-p, r, s, t
8. A-q; B-r, s; C-s; D-p, q, r
XL = XC resonance condition
(A) Total thermal energy generated in wire
10 = energy stored in capacitor
In case (A) : i = =1
10 (B) e = Bv
VL = 1 × 10, VC = 1 × 10 In opposite phase ; (C) F = qE
(D) V = iR, Heat = i2Rt
VR = 1 × 10

1. 1 B (i1 + i2 ) × = mg
We can understand the direction of flow of induced currents mg 0.2 9.8 49
if a fictitious battery is attached between EF. The direction i1 + i2 = = = amp. ....(iii)
B 0.6 1 15
of induced current can be found with the help of Lenz’s law.
From (ii) and (iii),
R1
0.76 49
A
i1
C i2 + i2 = i2 = 2 amp.
1.2 15
B(i1+i2)
19 0.76
e i1 = amp. e= = 0.6V
15 19 /15
E L i1+i2 F The induced emf across L due to the movement of bar L in a
mg
magnetic field
i2
B D e = BvT L
R2
e 0.6
vT = = = 1m/s
[P.d. across parallel combinations remains the same] BL 0.6 1
Also, P1 = ei1 = 0.76 W 2. 5
and P2 = ei2 = 1.2 W If v is the velocity of the rod at any time t, induced emf BvL
and so induced current in the rod
i1 0.76 0.76
i1 = i2 … (ii) Induced e.m.f. BvL
i2 1.2 1.2 I= =
R R
The horizontal metallic bar L moves with a terminal velocity. Due to this Current, the rod in the field B will experience a
This means that the net force on the bar is zero. force
610 IIT-JEE PHYSICS Challenger
From (1) 100 = 200 cos t
B 2 L2 v
F = BIL = (opposite to its motion) … (1)
R 1
or cos ( t) = , From eqution (3) :
So equation of motion of rod will be, 2
T – F = O × a i.e., T = F [as rod is mass less]
dI 1
T (2.0 10 4 C ) (10 4 s 1 )2 ÷
mg – T = ma i.e.a = g dt 2
m
dI
F B 2 L2 v = 10+4 A/s
or a=g g … (2) dt
m mR
So rod will acquire terminal velocity when its acceleration is dI
zero i.e., But = 10 x A/s
dt
B 2 L2VT mg R x=4
g = 0 i.e. VT = 2 2 ;
mR B L 4. 0.01
VT The flux linked with the coil when the plane of the coil is
For the case when velocity is perpendicular to the magnetic field is
2
= nAB cos = nAB.
VT mg R the change in flux on rotating the coil by 180° is d = nAB –
v= =
2 2 B 2 L2 (–nAB) = 2nAB
Substituting this value of velocity in eq. (2) we get
d
2 2
induced charge =
B L 1 mgR 1 g 10 R
a g g g= = = 5ms –2
mR 2 B 2 L2 2 2 2
2nAB 2 100 0.001 1
3. 4 = =
dt 10
This is a problem of L-C oscillations.
Induced charge = 0.01 coulomb.
Here Q0 = maximum value of
5. 25
Q = 200 µ C = 2 × 10–4 C
d s di p
=
1
=
1
= 10 4 s 1 es = N s and es = M ;
dt dt
LC (2 10 3 H ) (5.0 10 6 F )
Let at t = 0, Q = Q0 then d s di p
Ns =M
Q (t) = Q0 cos t … (1) dt dt
dQ
I (t ) = = Q0 sin t … (2) d s 200(2.5 10 4 0)
dt or M = N s = = 2.5×10–2 = 25mH
di p (2 0)
L=2.0mH
6. 4

dI 1 0.5 0.5
Given : e = 10 V and = = = 2.5 A/s
dt 0.2 0.2

e 10
Self inductance of coil L= = = 4H
C=5.0mF dI / dt 2.5

dI (t ) 2
Q0 cos ( t ) … (3)
dt

Q0
For Q = 100 µC or 2 ÷
1. A fish looking up through the water sees the outside world 4. r and r' denote the angles inside an equilateral prism, as
contained in a circular horizon. If the refractive index of usual, in degrees.
water is 4/3 and the fish is 12 cm below the surface, the
radius of this circle (in cm) is –
(a) 36 5 (b) 4 5
r r'
(c) 36 7 (d) 36 / 7
2. A mango tree is at the bank of river and one of the branch of
tree extends over the river. A tortoise lives in river. A mango
falls just above the tortoise. The acceleration of the mango
falling from tree appearing to the tortoise is (Refractive index
Consider that during some time interval from t = 0 to t = t, r'
of water is 4/3 and the tortoise is stationary)
varies with time as r' = 10 + t2. During this time r will vary as
(a) g (b) 3g/4
(c) 4g/3 (d) None of these (assume that r and r' are in degree)
3. The given lens is broken into four parts and rearranged as (a) 50 – t2 (b) 50 + t2
shown. If the initial focal length is f then after rearrangement
the equivalent focal length is – (c) 60 – t2 (d) 60 + t2
5. A triangular prism of glass is inside water. A ray, incident
normally, on one of the faces, is totally reflected from face
BC. Then the minimum refractive index of glass is –

1 2
A C
3 4

in air B
3 5
(a) f (b) f / 2 (a) (b)
2 3
(c) f / 4 (d) 4 f
2 2 4 2
(c) (d)
5 3

MARK YOUR
1. 2. 3. 4. 5.
RESPONSE
612 IIT-JEE PHYSICS Challenger
6. A ray of light is incident on a thick slab of glass (thickness
1
t) as shown below. The emergent ray is parallel to the incident
sin i + cos B
2 2 cos i sin B
ray but displaced sideways by a distance d. If the angles are (a) 1+ (b)
sin B cos B
small then d is:
t

i (c)
(cos i + cos B )2 (d)
sin i cos B
sin B sin B
r

10. On the earth a child focuses the sun light on a screen, using
a convex lens of focal length f and radius of aperture r. What
d
is the intensity of light at the focus point (S = solar constant)
(a) t (1–i/r) (b) r t (1–i/r) f

r r
(c) i t 1 ÷ (d) t 1 ÷
i i
7. An object is moving with speed v0 towards a spherical mirror r
with radius of curvature R, along the central axis of the focus
mirror. The speed of the image with respect to the mirror is point
(U is the distance of the object from mirror at any given time
t):
2
R R
(a) + 2
÷ v0 (b) ÷ v0
U 2R R 2U Sr 2 d 2 Sr 2 d 2
(a) (b)
2R2 f 2 4R2 f 2
2
R R
(c) ÷ v0 (d) + 2
÷ v0
2U 2R 2U 2 Sr 2 d 2 2Sr 2 d 2
(c) (d)
8. A point source of light is placed at a depth of h below the R2 f 2 R2 f 2
surface of water of refractive index . A floating opaque disc
11. In a thick glass slab of thickness and refractive index n1 a
is placed on the surface of water so that light from the
cuboidal cavity of thickness m is carved as shown in the
sources is not visible from the surface. The minimum diameter
figure and is filled with liquid of R.I. n2 (n1 > n2). The ratio of
of the disc is
(a) 2h/ ( 2 – 1)1/2 (b) 2h ( 2 – 1)1/2 / m , so that shift produced by this slab is zero when an
2
(c) h/ [2( – 1) ] 1/2
(d) h( 2 – 1)1/2 observer A observes an object B with paraxial rays is
9. A ray of light PQ is incident at angle i on a prism face. (See A
figure) After 2 refractions it leaves the prism at a grazing air
n1 (1)
angle.

B n2 (2) m

i Q air
B
P n1 n2 n1 n2
(a) n2 1 (b) n2 ( n1 1)
If is the refractive index and B is the refractive index and B
is the refracting angle of the prism, the refracting index ( )
n1 n2 n1 n2
is: (c) (d)
n1 1 n1 ( n2 1)

MARK YOUR 6. 7. 8. 9. 10.


RESPONSE 11.
RAY OPTICS 613

12. A pendulum of length is free to oscillate in vertical plane


above point O. An observer is viewing the bob of the
pendulum directly from above. The pendulum is performing
small oscillations in water (refractive index is µ) about its (a)
equilibrium position. The equation of trajectory of bob as 2f f 2f
seen by observer is

O x-axis

(b)
f 2f
2f

(a) µ2x2 + y2 = 2 (b) x2 y2


2
+ 2 2
=1

(c)
2 2 2 2f f 2f
x y
(c) x +y =
2 2
(d) + =1
÷ 2
( / )2

13. A planoconcave lens is placed on a paper on which a flower


is drawn.

Radius of
curvature =20cm.
(d)
air 2f f 2f
µ=3/2
t=20cm.

Paper 15. Choose the correct ray diagram of a thin equi-convex lens
which is cut as shown in the figure.
How far above its actual position does the flower appear to
be ?
(a) 10 cm. (b) 15 cm. (a)
(c) 50 cm. (d) None of these
14. The figure shows a straight small object kept in front of a
convex lens as shown in the figure. Which among the given
options shows the right image of the object?
(b)

(c)
2f f O 2f

(d)

MARK YOUR
12. 13. 14. 15.
RESPONSE
614 IIT-JEE PHYSICS Challenger
16. A biconvex thin lens of radius of curvature R is made up of 19. A plano convex lens fits exactly into a plano concave lens.
| y| Their plane surface are parallel to each other. If the lenses
variable refractive index µ = 2 1 + ÷ . Assume 2d << R. are made of different materials of refractive indices µ1 & µ2
d
There are infinite images of the point object O (which is and R is the radius of curvature of the curved surface of the
placed at a distance R on the principal axis from the lens as lenses, then focal length of combination is
shown in the figure) spreaded over the length R 2R
Y (a) (b)
µ1 – µ2 µ1 – µ2
R R R
(c) (d)
d 2 ( µ1 – µ2 ) 2 – ( µ1 + µ2 )
20. A glass prism of refractive index 1.5 is immersed in water
X
O P (refractive index 4/3). A light beam incident normally on the
R d face AB is totally reflected to reach the face BC, if
R
B A

R 2R
(a) (b)
5 5
3R 4R C
(c) (d)
5 5
17. The plane surface of a plano-convex lens of focal length 20 (a) sin 8/9 (b) 2/3 < sin 8/9
cm is silvered. It will behave as (c) sin 2/3 (d) sin 8/9
(a) plane mirror
(b) convex mirror of focal length 40 cm 21. f = –10 cm
(c) concave mirror of focal length 10 cm
(d) None of these f = 20 cm
18. A rectangular glass slab ABCD of refractive index n1 is Principal axis
immersed in water of refractive index n2(n1 > n2). A ray of of concave lens
(0,0) 5 mm
light is incident at the surface AB of the slab as shown. The Principal axis
maximum value of the angle of incidence max such that the of convex lens
ray comes out only from the other surface CD is given by

A D 30 cm
n1 n2
max
If the optic axis of convex and concave lenses are separated
B C by a distance 5 mm as shown in the figure. Find the coordinate
of the final image formed by the combination if parallel beam
of light is incident on lens origin is at the optical centre of
n n
(a) sin 1 1 cos sin 1 2 ÷ ÷ convex lens.
n2 n1 (a) (25 cm, 0.5 cm) (b) (25 cm, 0.25 cm)
(c) (25 cm, – 0.5 cm) (d) (25 cm, – 0.25 cm
1 22. Convex surface of thin concavo – convex lens of refractive
(b) sin 1 n1 cos sin 1
n2 ÷ ÷
index 1.5 is silvered as shown. A small object is kept in air at
30 cm left of the lens on its principal axis. The distance of
the final image is
1 n1
(c) sin r1= 60 cm
n2 ÷

n2 r2= 20 cm
(d) 1
sin
n1 ÷ (a) 20 cm (b) 30 cm
(c) 10 cm (d) 15 cm

MARK YOUR 16. 17. 18. 19. 20.


RESPONSE 21. 22.
RAY OPTICS 615

23. A concave lens of glass, refractive index 1.5 has both 27. The maximum deviation produced by a prism of material of
surfaces of same radius of curvature R. On immersion in a refractive index µ is
medium of refractive index 1.75, it will behave as a
(a) convergent lens of focal length 3.5 R 1 1 1 1 1
sin
µ÷
(a) 2sin (b)
(b) convergent lens of focal length 3.0 R 2 µ÷ 2
(c) divergent lens of focal length 3.5 R
(d) divergent lens of focal length 3.0 R 1 1 1 1
2sin 2sin
µ÷
(c) (d)
24. A equilateral prism is made of a transparent material of µ÷
refractive index 2 . A ray of light AB is incident at 45° as
28. A ray of light travelling along the positive z-axis is reflected
shown. The net deviation in the path of ray when it comes
twice :
out of prism is
(i) for the first time, by a mirror whose normal is along

(i + k)
60° (ii) for the second time, by a mirror whose normal is along
Silvered
45° B (i + k + j), where the symbols have their usual
A meanings. The final ray is along
60° 60°
(a) j+ k (b) k +i
(a) 135° (b) 120°
(c) 30° (d) 150° (c) 2 j 2k i (d) ( j k 2i).
25. Two lenses of focal length f1 = 10 cm and f2 = – 20 cm are 29. A composite glass slab is manufactured so that its refractive
kept as shown. The resultant power of combination will be index varies along its thickness according to the relation :
f2
f1 x
Silvered ( x) = 1 + ÷,
t
where t is the thickness of the slab.
The optical path, introduced, by the slab when it is placed
(a) –10D (b) 5 D in the path of light passing normally through it, is given by
(c) 0 (d) 10 D µ (x)
26. A plane mirror is kept parallel to y-axis. A point object is
approaching the mirror with velocity u = (10i + 10 j) m/s. The x
O
magnitude of relative velocity of objective w.r.t image is
equal to

y (a) ( 1 + ) t (b) 1+ ÷t
2
Plane t t
u mirror (c) (d) ln (1 )
1

x 30. A point object is moving with velocity u = 2iˆ + ˆj kˆ m/s


O
in front of a stationary plane mirror. The magnitude of relative
velocity of the image with respect to object is maximum if
the normal of the plane mirror will be along

(a) 20 2m / s (b) 20 m/s (a) 2iˆ + ˆj + kˆ (b) 2iˆ ˆj kˆ

(c) 10 2m / s (d) 10 m/s (c) 2iˆ ˆj kˆ (d) 2iˆ ˆj kˆ

MARK YOUR 23. 24. 25. 26. 27.


RESPONSE 28. 29. 30.
616 IIT-JEE PHYSICS Challenger
31. A stationary observer O looking at a fish F in water 35. The following data are given for a crown glass prism ;
(µw = 4/3) through a converging lens of focal length 90.0cm. refractive index for violet light nv = 1.521
The lens is allowed to fall freely from a height 62.0cm with refractive index for red light nr = 1.510
its axis vertical. The fish and the observer are on the principal refractive index for yellow light ny = 1.515
axis of the lens. The fish moves up with constant velocity Then the dispersive power of a parallel glass slab made of
100 cm/s. Initially it was at a depth of 44.0cm. Find the the same material is
velocity (in cm/s) with which the fish appears to move with (a) 0.01 (b) 0.03 (c) 0 (d) 0.02
respect to lens to the observer at t = 0.2 sec. 36. The inner surface of a cone coated by a reflecting layer
(take g = 10 m/s2) forms a conical mirror. A thin incandescent filament is
O stretched in the cone along its axis.
Determine the minimum angle of the cone for which the
Lens
rays emitted by the filament will be reflected from the conical
surface not more than once
air 62cm (a) 120° (b) 90° (c) 150° (d) 135°
37. Two identical thin planoconvex lenses of refractive index n
are silvered, one on the plane side and the other on the
convex side. The ratio of their focal lengths is
water 44cm n n 1 n +1
(a) (b) (c) (d) n
n 1 n n
F 38. Two converging lenses are mounted at the ends of a tube
(a) 1475 cm/s (b) 2615 cm/s with a blackened inner lateral surface. The diameters of the
(c) 2475 cm/s (d) 2271 cm/s lenses are equal to the diameter of the tube. The focal length
32. A point object P moves towards a stationary convex mirror of one lens is twice that of the other lens. The lenses are at
with a constant speed v, along the optical axis. The speed of such a distance from each other that parallel light rays
the image incident along the axis of the tube on one lens emerge from
(a) is always less than v the other lens in a parallel beam. When a wide light beam is
(b) may be greater than, equal to or less than v, depending incident on the lens with the larger focal length, a bright
upon the position of P spot of illuminance E1 is formed on the screen. When the
(c) is always greater than v tube is turned through 180°, the bright spot formed on the
(d) none of these screen has an illuminance E2 . Determine the ratio of
33. A hollow double concave lens is made of very thin illuminances on the screen.
transparent material. It can be filled with air or either of two (a) 1/4 (b) 1/2 (c) 1/8 (d) 1/16
liquids L 1 or L 2 having refractive indices 1 and 2 39. In the case of minimum deviation for an equilateral flint glass
respectively ( 2 > 1 > 1). The lens will diverge a parallel prism, the angle of refraction is 30° as shown in figure. What
beam of light if it is filled with should be the angle of refraction of light (for ray incident on
(a) air and placed in air (b) air and immersed in L1 surface AB) satisfying the condition of minimum deviation
(c) L1 and immersed in L2 (d) L2 and immersed in L1 in case of an equilateral crown glass prism?
34. A thin convex lens of focal length 10 cm and refractive index [Given µflint > µcrown]
1.5 is cut vertically into two equal pieces. They are placed
as shown with a liquid of refractive index 3 between them. A
What is the focal length of the combination ?

30°

B C

(a) –10 cm. (b) –10/4 cm. (a) > 30° (b) < 30°
(c) –10/3 cm. (d) None (c) 30° (d) information insufficient

MARK YOUR 31. 32. 33. 34. 35.


RESPONSE 36. 37. 38. 39.
RAY OPTICS 617

40. A convex lens is in contact with concave lens. The magnitude


of the ratio of their focal length is 2/3. Their equivalent focal
length is 30 cm. What are their individual focal lengths? O
(a) –15, 10 (b) –10, 15
(c) 75, 50 (d) –75, 50
41. A ray of light is incident at an angle on the boundary (a) 1.5 (b) 5/3
separating two transparent media and it refracts. When angle (c) 9/8 (d) None of these
is increased very slightly, the ray suffers a total internal 46. A glass porthole is made at the bottom of a ship for
reflection. The difference between the angles of deviation observing sea-life. The hole diameter D = 40 cm. is much
in two cases is nearly larger than the thickness of the glass. Determine the area S
(a) 90° – (b) 180° – (approximately) of the field of vision at the sea bottom for
(c) 180° – 2 (d) 2 the porthole if the refractive index of water is nw = 1.4, and
42. The image of a point source S' lying at a distance b from a the sea depth is h = 5m.
transparent sphere is formed by a small diaphragm only by (a) 41m2 (b) 55m2 (c) 82m2 (d) 164m2
rays close to the optical axis (figure). 47. A point object is located at a distance 15cm. from the pole of
a concave mirror of focal length 10cm on its principal axis is
moving with a velocity (8iˆ + 11 ˆj ) cm/s and velocity of
S S' mirror is (4iˆ + 2 ˆj ) cm/s as shown. If v is the velocity of
b image. Then find the value of | v | in (cm/s).
y

Where will the image be after the sphere is cut into two
^ ^

////
parts perpendicular to the horizontal axis, and the plane (8i + 11j) cm/s x

/////// ////////////
surface of the left half is silvered ?
^
(a) At a distance b from the sphere (4i + 2^j) cm/s
////
(b) At a distance b/2 from the sphere. O
15cm
(c) At a distance b/3 from the sphere.
(d) None of these
/

43. Two rays are incident on a spherical mirror of radius R = 5 (a) 20 (b) 30 (c) 10 (d) 40
cm. parallel to its optical axis at distances h1 = 0.5 cm. and 48. I is the image of a point object O formed by spherical mirror,
h2 = 3 cm. Determine the distance x (approximately) then which of the following statement is incorrect? (Take
between the points at which these rays intersect the optical real or virtual objects at finite distances from pole)
axis after being reflected at the mirror. (a) If O and I are on the same side of the principal axis,
(a) 0.2cm. (b) 1.5cm. then they have to be on opposite sides of the mirror
(c) 0.6cm. (d) 1.0 cm. (b) If O and I are on opposite sides of the principal axis,
44. In an experiment to determine the focal length then they have to be on same side of the mirror
(f ) of a concave mirror by the u - v method, a student places
(c) If O and I are on opposite sides of the principal axis,
the object pin A on the principal axis at a distance x from the
then they have to be on opposite sides of the mirror as
pole P. The student looks at the pin and its inverted image
well
from a distance keeping his/her eye in line with PA. When
(d) If O is on principal axis, then I has to lie on principal
the student shifts his/her eye towards left, the image appears
axis only
to the right of the object pin. Then,
49. A person walks at a velocity v in a straight line forming an
(a) x < f (b) f < x < 2f
angle with the plane of a mirror. Determine the velocity
(c) x = 2f (d) x > 2f
45. Behind a thin converging lens having both the surfaces of vrel at which he approaches his image, assuming that the
the same radius 10cm, a plane mirror has been placed. The object and its image are symmetric relative to the plane of
image of an object at a distance 40cm. from the lens is formed the mirror.
at the same position. What is the refractive index of the (a) 2v sin (b) 2v cos
lens? (c) v sin (d) v cos

MARK YOUR 40. 41. 42. 43. 44.


RESPONSE 45. 46. 47. 48. 49.
618 IIT-JEE PHYSICS Challenger
50. A point light source S is outside a cylinder on its axis near
2d sin 2d sin
the end face (base). Determine the minimum refractive index (a) (b)
n of the cylinder material for which none of the rays entering n 2 sin 2 1
sin 2
the base will emerge from the lateral surface. n2
(a) 1 / 2 (b) 2 (c) 1/2 (d) 1
51. A transparent sphere of radius R has a cavity of radius R/2 2d n 2 sin 2
(c) (d) None of these
as shown in figure. Find the refractive index of the sphere if sin
a parallel beam of light falling on left surface focuses at 54. Rays of light from Sun falls on a biconvex lens of focal
point P. length f and the circular image of Sun of radius r is formed
on the focal plane of the lens. Then
(a) area of image is r2 and area is directly proportional of f
(b) area of image is r2 and area is directly proportional to f 2
P (c) intensity of image increases if f is increased
O (d) if lower half of the lens is covered with black paper area
will become half
55. A parallel beam of light falls on two coaxial lens system
consisting of a thin converging lens of focal length 10cm.
and a thin diverging lens of focal length 5cm. as shown in
3+ 5 3 5
(a) = (b) = figure. The beam is also parallel to the principal axis and all
2 2 the rays in the beam are paraxial. Both lenses are surrounded
by air. The intensity of incident light is I0. If the complete
1+ 5
(c) = 3+ 5 (d) = light is transmitted through each lens (that is, there is no
2 absorption of light and neglecting partial reflection at any
52. A point light source is moving with a constant velocity v surface of lens), the intensity of light emergent from diverging
inside a transparent thin spherical shell of radius R, which is lens is
filled with a transparent liquid. If at t = 0 light source is at the f1 =10cm. f2=10cm.
centre of the sphere, then at what time a thin dark ring will
be visible for an observer outside the sphere. The refractive
index of liquid with respect to that of shell is 2.
R R R R
(a) (b) (c) (d)
2V 2V 3V 3V
53. A ray of light incident from air on a glass plate of refractive 5cm.
index n is partly reflected and partly refracted at the two (a) 4 I0 (b) I0 (c) 2 I0 (d) I0/2
surfaces of the glass. The displacement y0 in the figure is 56. A glass hemisphere of radius R and material having refractive
index 1.5 is silvered on its flat face as shown in the figure. A
small object of height h is located at a distance 2R from
surface of hemisphere. The final image will form
///////////////////////////////////////

y0 h
2R

d (a) at a distance of R from silvered surface, on right side


(b) on the object itself
(c) at the hemispherical surface
(d) at a distance of 2R from the silvered surface on left side

MARK YOUR 50. 51. 52. 53. 54.


RESPONSE 55. 56.
RAY OPTICS 619

57. A ray is incident normally on a right angle prism whose


refractive index is 3 and prism angle = 30°, after crossing
prism ray passes through glass sphere. It strikes the glass
(a) 40 cm (b) 160/3 cm
R (c) 200/3 cm (d) none of these
sphere at distance from principal axis, as shown in
3 61. A fish sees the smiling face of a scuba diver through a
figure sphere is half polished. Find the net angle of deviation bubble of air between them, as shown. Compared to the
of incident ray. face of the diver, the image seen by the fish will be

//
///
///////////////
R/ 3

/
/

Fish
Diver Bubble
(a) 120° (b) 150°
(a) smaller and erect (b) smaller and inverted
(c) 90° (d) 180°
(c) larger and erect (d) larger and inverted
58. A prism placed in air made up of flint glass is such that any
incident ray on one surface does not emerge from the second 62. The width of man’s face is 10cm. The distance between the
surface. Critical angle for flint glass is 36° in air. Then, eyes of the man is 4cm. Then the minimum width of plane
refracting angle A may be mirror to see his full face, is
(a) 37° (b) 54° (a) 5 cm (b) 4 cm
(c) 71° (d) 73° (c) 3 cm (d) 10 cm
59. A small straight rod is placed at an inclination with the optical 63. Five spheres are lined up in front of a plane mirror as shown.
axis of a thin lens as shown in the figure. The base of the rod The observer will be able to see the reflection of
is on the optical axis and at a distance 2f (f = focal length of
the lens) from the lens. The image of the rod would be Observer 4 2

5 3 1

2f
Mirror

(a) 1 only (b) 5 only


(a) a straight line leaning towards the lens (c) 1 and 3 only (d) 1, 2, 3 and 4 only
(b) a straight line leaning away from the lens 64. A scuba diver in an empty swimming pool uses a magnifier
(c) a curve leaning towards the lens (n = 1.25) to enlarge the print on a plastic instruction sheet.
(d) a curve leaning away from the lens If the pool is filled with water (n = 1.33), what happens to the
60. Half part of the rear surface of a thin equiconvex lens of magnification of the print ?
focal length 40cm and refractive index 1.5 is silvered. If parallel (a) It increases and is greater than one.
rays are incident on this lens, then the distance between the (b) It stays the same.
two focii is (c) It decreases, but is still greater than one.
(d) It decreases and is less than one

MARK YOUR 57. 58. 59. 60. 61.


RESPONSE 62. 63. 64.
620 IIT-JEE PHYSICS Challenger

PASSAGE-1 6. The length of the cylindrical tube is


(a) 16 cm (b) 18 cm
A point object is situated in front of a spherical glass surface (c) 20 cm (d) 22 cm
as shown in the figure.
PASSAGE-3
Plane mirror
= 1.2 = 1.5
O P
x A curved surface of radius R separates two media of
R =20 cm refractive indices µ1 and µ2 as shown in figures (A) and (B)
100 cm
respectively.
1. The position of image from P when x = 70 cm
(a) 70 cm right to P (b) 80 cm left to P µ1 µ2 µ1 µ2
(c) 90 cm left to P (d) 90 cm right to P O P O P
2. Position of image from P when x = 80 cm x x
(a) 80 cm right to P (b) 80 cm left to P Figure (A) Figure (B)
(c) 90 cm left to P (d) 70 cm right to P For a real object a image is virtual if the light rays diverge
3. The position of image from P when x = 90 cm after refraction and real if they converge or become parallel
(a) 80 cm right to P (b) 90 cm cm left to P
after refraction.
(c) 90 cm right to P (d) 70 cm right to P
7. A point object is kept at point O in figure (A) then
(a) real image is formed for all position of object if
PASSAGE-2
µ2 > µ1
(b) real image is formed only when x > R
(c) real image will form irrespective of µ 1 and µ2

µ1 R
1 w 2 (d) real image is formed if x > for µ2 > µ1.
µ2 µ1
O 8. If the point object is kept at O at a distance x in figure (A)
L1 L2
then
(a) virtual image is formed for any position of O for µ 2 < µ1
A cylindrical tube filled with water (µw = 4/3) is closed at its (b) virtual image can be formed only if x > R and µ2 < µ1
both ends by two silvered plano convex lenses as shown in (c) virtual image can be formed only if x < R and µ2 < µ1
the figure. Refractive index of lenses L1 and L2 are 2.0 and (d) virtual image can never be formed.
1.5 while their radii of curvature are 5 cm and 9 cm 9. If an object is kept at O at distance x from pole of figure B
respectively. A point object is placed somewhere at a point
then
O on the axis of cylindrical tube. It is found that the object
and image coincide each other. (a) If µ2 < µ1 then virtual image is formed for any value of x
4. The position of object w.r.t lens L1 is
µ1 R
(a) 8 cm (b) 10 cm (b) If µ2 < µ1 then virtual image is formed for x <
(c) 12 cm (d) 14 cm µ1 µ2
5. The position of object w.r.t lens L2 is (c) If µ2 < µ1 then real image is formed for any value of x
(a) 8 cm (b) 10 cm
(d) If µ2 < µ1 then real image is always formed.
(c) 12 cm (d) 14 cm

MARK YOUR 1. 2. 3. 4. 5.
RESPONSE 6. 7. 8. 9.
RAY OPTICS 621

PASSAGE-4 PASSAGE-5

A concave mirror M is mounted on an optical bench. Two A concave mirror of radius of curvature 20cm is shown in
pins A and B are placed on bench such that their tips also lie the figure. A circular disc of diameter (1.0 + 0.2t) cm is placed
on the principal axis of the mirror as shown. The image of tip on the principal axis of mirror with its plane perpendicular to
of one tip is formed on the tip of the other pin. Now if pin B the principal axis at a distance 15cm from the pole of the
is moved 80cm. towards left and pin A is moved 80cm. mirror.
towards right, image of tip of one pin is again on the tip of

/////
the other pin. The magnitude of transverse magnification of

////////////////////
the larger image is three. [All the images are formed by
paraxial rays].

/
20cm
A B
13. The image formed by the mirror will be in the shape of a
/////////////////////////////////////// (a) circular disc
10. The focal length of the mirror is (b) elliptical disc with major axis horizontal
(a) 30cm. (b) 60cm. (c) elliptical disc with major axis vertical
(c) 40 cm. (d) 15 cm. (d) distorted disc
11. The distance of the nearest pin from pole initially is 14. In the above question, the area of the image of the disc at
(a) 20cm. (b) 60cm. t = 1 second is
(c) 80 cm. (d) 40 cm. (a) 1.2 cm2 (b) 1.44 cm2
12. The magnification of the image of the pin B in second case (c) 1.52 cm2 (d) None of these
is 15. What will be the rate at which the horizontal radius of image
(a) 3 (b) 4.5 will be changing ?
(c) 1/4.5 (d) 1/3 (a) 0.2 m/s increasing (b) 0.2 cm/s decreasing
(c) 0.4 cm/s increasing (d) 0.4 cm/s decreasing

MARK YOUR 10. 11. 12. 13. 14.


RESPONSE 15.

1. Statement - 1 : When light travels from denser to rarer Statement - 2 : In case of a plane mirror, distance of object
medium the critical angle of incidence have and its image is equal from any point on
different values for different wavelengths the mirror.
of light. 3. Statement - 1 : It is not possible to see a virtual image by
Statement - 2 : Refractive index of a medium varies with eye.
wavelength of light. Statement - 2 : The rays that seem to emanate from a
2. Statement - 1 : Keeping a point object fixed, if a plane virtual image do not in fact emanate from
mirror is moved, the image will also move. the image.

MARK YOUR
1. 2. 3.
RESPONSE
622 IIT-JEE PHYSICS Challenger
4. Statement - 1 : Position of image approaches focus of a Statement - 2: In different surroundings, power of a given
lens, only when object approaches infinity. lens has different values but the same sign.
Statement - 2 : Paraxial rays incident parallel to principal 6. Statement - 1 : As the distance x of a parallel ray from axis
axis intersect at the focus after refraction increases, angle of incidence decreases.
from lens. ///
5. Statement - 1 : A lens L (shown in the figure), kept in

///
/////
surrounding medium X, has a power +10D.
x

//////////////////
If the same lens is kept in a surrounding
medium Y, its power is found to be +12.5D. P A C axis
Also if the same lens is placed in a
surrounding medium Z, its power is now
///

//
measured to be –3.5D, then µz > µx > µy.
L Statement - 2 : As x increases, the distance of point of
intersection of the reflected ray with axis
from pole decreases.

MARK YOUR
4. 5. 6.
RESPONSE

1. A certain mirror placed at the origin has i as the normal x


(a) f = (b) m1m2 =1
vector to its reflecting surface. The mirror begins to translate m1 m2

with a velocity i + j + k at t = 0, at the same time an object


D2 x 2
(c) f = (d) D 4f.
which was initially at i + j starts moving with a velocity 4D
3. Two thin convex lenses are separated as shown. The focal
(i + j ) m/s. Now choose the correct options. length of lens L1 is 15 cm and that of lens L2 is 30 cm. An
object is placed at distance 30 cm from lens L1. The location
(a) Initial position of the image will be –i + j of image formed finally will be at
(b) The velocity of the image will be i + j L1 L2
O
(c) The velocity of the image relative to the object will be
zero 30cm 30cm

(d) The velocity of the image relative to the mirror will be (a) infinity

–k . (b) 15 cm behind the lens L1

2. A lens of focal length ‘f ’ is placed in between an object and (c) optical centre of lens L2
screen at a distance ‘D’ apart.The lens forms two real images (d) 60 cm away from object
of the object on the screen for two of its different positions, 4. Which of the following quantities related to a lens depend
a distance ‘x’ apart. The two real images have magnifications on the colour/wavelength of the incident light ?
m1 and m2 respectively (m1 > m2). (a) Focal length (b) Power
(c) Image distance (d) Chromatic aberration

MARK YOUR
1. 2. 3. 4.
RESPONSE
RAY OPTICS 623

5. Remote objects are viewed through a converging lens with


a focal length F = 9 cm. placed at a distance a = 36cm. in C
front of the eye. Assume that the radius r of the pupil is A
approximately 1.5 mm. Choose the correct options O
(a) The minimum radius of the screen that should be placed B
behind the lens so that the entire field of view is covered
D
is 0.5mm. 11 Screen
(b) The minimum radius of the screen that should be placed
behind the lens so that the entire field of view is covered (a) intensity of light will be the same everywhere on the
is 1.0 mm. screen
(c) The screen must be placed in the plane S with its centre (b) intensity in region AB will be smaller than what it would
at point B. be in the absence of the lens
(d) The screen must be placed perpendicular to the plane (c) in the region AC and BD, the intensity will be smaller
S with its centre at point B. than what it would be in the absence of the lens
6. Let us suppose that a person seating opposite to you at the (d) in the region AC and BD, the intensity will be greater
table wears glasses. Choose the correct options if you wish than what it would be in the absence of the lens
to determine whether he is short sighted or long-sighted ? 9. P1 and P2 are identical prisms arranged as shown in figure.
Naturally, being a polite person, you would not ask him to A ray of white light incident on face of P1 undergoes
let you try his glasses and in general would make no mention dispersion and falls on one face of P2. The facing surfaces
of them. of the prisms are parallel to each other. Then
(a) Behind a diverging lens, the eye will look smaller.
(b) Relative to other parts of the face : if visible contour of A
the face is displaced inwards, the lenses are diverging, Red
White P2
and your companion is short-sighted.
P1 Violet
(c) Relative to other parts of the face : if visible contour of
the face is displaced outwards, the lenses are A
converging, and the person is long-sighted.
(d) Behind a converging lens, the eye will look larger. (a) light emerging from P2 will be white
7. A glass prism is immersed in a hypothetical liquid. The (b) in the light emerging from P2 dispersion will be greater
curves showing the refractive index n as a function of (c) the direction of light emerging from P2 will be parallel
wavelength for glass and liquid are as shown in the figure. to the direction of ray incident on P1
When a ray of white light is incident on the prism parallel to (d) the ray emerging from P2 will be white even if prisms
the base P1 and P2 have identical geometry but different material
n Vertex
Liquid 10. A right-angled prism is made up of a material of refractive
index µ. It is desired that a light ray incident normally on PQ
emerges parallel to the incident direction after suffering two
total internal reflections. In which of the following conditions
is this possible ?
Glass
Base P
O
yellow
µ
(a) yellow ray travels without deviation
(b) blue ray is deviated towards the vertex R Q
(c) red rays is deviated towards the base
(d) there is no dispersion (a) = 2 (b) 2/ 3
8. A concave lens is placed in the path of a uniform parallel (c) µ = 1.3 (d) never possible.
beam of light falling on a screen as shown. Then

MARK YOUR 5. 6. 7. 8. 9.
RESPONSE 10.
624 IIT-JEE PHYSICS Challenger

1. A convex lens of refractive index 2 is kept in a medium of refractive index 1 as shown.


y
µ2
µ1
x x'

y'
Column I Column II
(A) On increasing value of 1 lens will be (p) | f | increases and converging
(B) If 1 > 2 (q) | f | may decrease or increase and lens will be diverging
(C) When lens is cut into two parts along yy', then (r) | f | increases and nature of lens remains unchanged
for any one part
(D) 1 is increased but 1 < 2 (s) | f | increases then decreases

2. Refractive index of lenses is 3/2. Then the final position of


the image from the pole is
Column-I Column -II

(C) (r) 40/3 cm, right to P


O 40 cm P
(A) (p) 40/7 cm, left to P
O 40 cm P

R = 20cm

R = 20cm
R = 20cm

(D) (s) 40 cm, left to P


O 40 cm P

(B) (q) 40/9 cm, right to P


O 40 cm P R = 20cm

R = 20cm R = 20cm

1. 2.

MARK YOUR
RESPONSE
RAY OPTICS 625

3. A mirror faces the negative x-axis. (Normal to its reflecting surfaces is i ), while a particle starts moving such that its image is

formed in the mirror. At a certain instant the velocity of the particle is 3i + 4 j + 5k and that of the mirror is i j + k . Now match
the following :
Column I Column II
(A) Magnitude of relative velocity of the image (p) 42
w.r.t. mirror
(B) Magnitude of relative velocity of image w.r.t. object (q) 45
(C) Magnitude of relative velocity of object w.r.t. mirror. (r) 4
(D) Absolute velocity of the image w.r.t. ground (s) 43
4. A ray of light strikes at the boundary separating two media at angle . µ1 and µ2 are refractive indices of media with (µ2 > µ1).
µ1

µ2

Column I Column II

µ1 µ1
(A) When < sin 1
then deviation in the (p) sin 1
µ2 ÷ 2 µ2 ÷
path of ray is

1 µ1
(B) Maximum deviation in the path of ray for refraction (q) 2sin
µ2 ÷
at boundary
(C) Maximum deviation in the path of ray for reflection (r) Zero
at the boundary
µ2 1
(D) Deviation in the path at grazing angle of incidence (s) sin sin ÷
µ1
5. Column I Column II
(A) Rainbow (p) Refraction
(B) Mirage (q) Dispersion
(C) Twinkling of stars (r) Scattering
(D) Blue sky (s) Total internal reflection
6. In column I, some optical instruments are mentioned, while in column II, the description about nature of image they can form for
real objects are given. Match the entries of column I with the entries of column II.
Column I Column II
(A) Concave mirror (p) Real, erect
(B) Convex mirror (q) Virtual, magnified
(C) Diverging lens (r) Real, diminished
(D) Converging lens (s) Virtual, diminished

3. 4. 5. 6.

MARK YOUR
RESPONSE
626 IIT-JEE PHYSICS Challenger
7. A bird in air is diving vertically over a tank with speed 6 cm/s. Base of the tank is silvered. A fish in the tank is rising upward along
the same line with speed 4 cm/s [Take µwater = 4/3]

///////////////////////////////////////////////

Column I Column II
(A) Speed of the image of fish formed after reflection (p) 12
from the mirror as seen by the bird
(B) Speed of image of bird relative to the fish looking (q) 4
upwards
(C) Speed of image of bird relative to the fish looking (r) 9
downwards in the mirror.
(D) Speed of image of bird relative to fish looking upwards (s) 3
in the mirror.
8. Match the following for light ray.
Column I Column II
(A) In reflection from denser medium (p) Amplitude may change
(B) In refraction into denser medium (q) Phase changes
(C) In reflection from rarer medium (r) No phase change
(D) In refraction into rarer medium (s) Velocity decreases
9. Match the columns :
Column I Column II
(A) Object is between optic centre and 1st principal (p) Image is inverted
focus in a diverging lens
(B) Object is between optic centre and 1st principal (q) Image is erect
focus of a converging lens
(C) Object is between optic centre and 2nd principal (r) Image is of greater size than the object
focus of a diverging lens
(D) Object is between optic centre and 2nd principal (s) Image is of smaller size than the object
focus of a converging lens
(t) Image is real

7. 8. 9.

MARK YOUR
RESPONSE
RAY OPTICS 627

10. Match the entries of column I with the entries of column II, which describes the angle of deviation of ray with angle of incidence.
Column I Column II

(A) A ray is falling on a plane smooth mirror (p)


i

(B) A ray is going from a rarer to denser medium (q)


i

(C) A ray is going from a denser to rarer medium (r) i

(D) A ray is falling on a prism (s) i

11. Light rays are incident on devices which may cause either reflection or refraction or both. The natures of the incident light and
the devices are described in column I. Some possible results of this on the rays are given in column II. Match the columnd
correctly.
Column I Column II
(A) A ray of white light is incident on one face of an (p) Divergent beam
equilateral glass prism
(B) A ray of white light is incident at an angle on a thick (q) Total internal reflection
glass sheet
(C) A ray of white light passes from an optically denser (r) Lateral shift
medium to an optically rarer medium
(D) A parallel beam of monochromatic light passes (s) Dispersion
symmetrically through a glass lens. (t) Refraction

10. 11.

MARK YOUR
RESPONSE
628 IIT-JEE PHYSICS Challenger
12. Match the columns correctly.
Column I Column II
(A) Concave mirror, real object (p) Real image
(B) Convex mirror, real object (q) Virtual image
(C) Concave lens, real object (r) Magnified image
(D) Convex lens, real object (s) Diminished image
(t) Same size
13. A real object is kept in front of a lens. The object is a linear extended object with its length perpendicular to the optic axis of lens.
With reference to different cases of image formation by lenses, match the columns correctly.
Column I Column II
(A) The image has a magnification –2.5 (p) Image is virtual
(B) Magnification of the image is +0.5 (q) Image is real
(C) Length of image is the same as that of object (r) Power of lens is positive
(D) Length of image is four times the length of the object (s) Power of lens is negative

12. 13.

MARK YOUR
RESPONSE
RAY OPTICS 629

1. A ray of light is incident at an angle of 60° on one face of 6. Light is incident at an angle on one planar end of a
prism which has an angle of 30°. The ray emerging out of transparent cylindrical rod of refractive index µ. Determine
the prism makes an angle of 30° with the incident ray. the least value of µ so that the light entering the rod does
Calculate the refractive index of the material of the prism. not emerge from the curved surface of rod irrespective of
2. An object is placed 21 cm in front of a concave mirror of the value of .
radius of curvature 10 cm. A glass slab of thickness 3 cm
and refractive index 1.5 is then placed close to the mirror in µ
the space between the object and the mirror.
Find the image distance (in cm) of the final image formed.
(You may take the distance of the near surface of the slab 90
from the mirror to be 1 cm).
3. A telescope has an objective of focal length 50 cm and an
eye piece of focal length 5 cm. The least distance of distinct 7. A thin plano-convex lens of focal length f is split into two
vision is 25 cm. The telescope is focussed for distinct vision halves: one of the halves is shifted along the optical axis .
on a scale 200 cm away from the objective. Calculate the The separation between object and image planes is 1.8 m.
separation (in cm) between the objective and the eye-piece. The magnification of the image formed by one of the half-
4. The convex surface of a thin concavo-convex lens of glass lenses is 2. Find the focal-length of the lens (in m).
of refractive index 1.5 has a radius of curvature 20 cm. The
concave surface has a radius of curvature 60 cm. The convex
side is silvered and placed on a horizontal surface.

8. Two plane mirrors A and B are aligned parallel to each other,


as shown in the figure. A light ray is incident at an angle of
At what distance (in metre) should a pin be placed on the 30° at a point just inside one end of A. The plane of incidence
optic axis such that its image is formed at the same place? coincides with plane of the figure. Find the maximum number
Calculate your answer in cm. of times the ray undergoes reflections (including the first
5. A plano convex lens has a thickness of 4 cm . When placed one) before it emerges out.
on a horizontal table, with the curved surface in contact
with it, the apparent depth of the bottom most point of the 2 3m
lens is found to be 3 cm. If the lens is inverted such that the
plane face is in contact with the table, the apparent depth of B
the centre of the plane face is found to be 25/8 cm. Find the
focal length (in cm) of the lens. 0.2m
30 °

1. 2. 3. 4. 5. 6.

MARK
YOUR
RESPONSE

7. 8.
630 IIT-JEE PHYSICS Challenger

1 (d) 7 (b) 13 (a) 19 (a) 25 (d) 31 (c) 37 (a) 43 (c) 49 (a) 55 (a) 61 (a)
2 (c) 8 (a) 14 (b) 20 (a) 26 (b) 32 (a) 38 (d) 44 (b) 50 (b) 56 (b) 62 (c)
3 (b) 9 (a) 15 (c) 21 (b) 27 (d) 33 (d) 39 (c) 45 (c) 51 (a) 57 (d) 63 (a)
4 (a) 10 (c) 16 (d) 22 (c) 28 (c) 34 (c) 40 (a) 46 (c) 52 (a) 58 (d) 64 (d)
5 (d) 11 (b) 17 (c) 23 (a) 29 (b) 35 (d) 41 (a) 47 (a) 53 (a) 59 (c)
6 (c) 12 (d) 18 (a) 24 (d) 30 (c) 36 (a) 42 (a) 48 (c) 54 (b) 60 (b)

1 (c) 3 (d) 5 (a) 7 (d) 9 (b) 11 (d) 13 (a) 15 (a)


2 (a) 4 (b) 6 (b) 8 (a) 10 (a) 12 (d) 14 (b)

1 (a) 2 (d) 3 (d) 4 (d) 5 (c) 6 (d)

1 (a, b, c, d) 3 (c, d) 5 (a, c) 7 (a, b, c) 9 (a, c)


2 (a, b, c, d) 4 (a, b, c, d) 6 (a, b, c, d) 8 (b, d) 10 (a, b, c)

1. A-s; B-q; C-r; D-p 2. A-q; B-p; C-s; D-r


3. A-q; B-r; C-q; D-p 4. A-s; B-p; C-q; D-r
5. A-p, q, s; B-p, s; C-p; D-r 6. A-q, r; B-s; C-s; D-q, r
7. A-r, B-s; C-p; D-q 8. A-p, q; B-p, r, s; C-p, r; D-p, r
9. A-q, r, t; B-q, r; C-q, s; D-q, s, t 10. A-s; B-q; C-p; D-r
11. A-p, q, s, t; B-r, t; C-p, q, s, t; D-p, q, t 12. A-p, q, r, s, t; B-q, s; C-q, s; D-p, q, r, s, t
13. A-q, r; B-p, s; C-q, r; D-p, q, r

1 1.73 2 7.67 3 70.8 4 15 5 75 6 1.41 7 0.4 8 30


RAY OPTICS 631

For small angle i, angle r is also small, and so


1 r
1. (d) sin ic = = d = t (i – r) = t i (1 – r/i)
r 2 + h2 7. (b) For concave mirror
Using h = 12 cm, µ = 4/3 2 1 1
= +
r R v u
2 1 1
ic or = +
h R v u
ic
1 1 2 R 2U
=
Fish v U R UR
36 RU
We get cm. or v =
7 R 2U
In spherical mirror, image velocity
x xrel
2. (c) xrel x 2
1 v2 RU v0
vi = v0
2
u R 2U U2
d 2 xrel 2 d 2x
arel g 2
dt 2 dt 2 R
v0
3. (b) Cutting a lens in transverse direction doubles their R 2U
focal length i.e. 2f. 8. (a) The figure shows incidence from water at critical angle
Using the formula of equivalent focal length,
c for the limiting case.
1 1 1 1 1
= + + + disc r
f f1 f2 f3 f4 air
We get equivalent focal length as f/2. water
4. (a) In a prism : r + r' = A r = A – r' h
r = 60° – (10 + t2) = 50 – t2

light
source
45° Now, sin c = 1/ so that tan c =1/ ( 2 –1)1/2 which is
5. (d)
also equal to r/h where r is the radius of the disc.
45° Therefore, diameter of the disc is
2r = 2h tan c.
9. (a) A ray of light incident on face AB will just suffer
For T.I.R. 45° > C internal reflection at the other face BC. If it gets
incident on face BC at critical C angle for the material
sin 45° > sin C of the prism. If critical angle for material is C, then
1 4/3 1
> sin C = …(i)
2 n

4 2 B
n>
3
6. (c) The lateral displacement is given by i r1r2
Q
t
d= sin (i r )
cos r A C
632 IIT-JEE PHYSICS Challenger
Now, for prism we have
r1 + r2 = B or r1 + C = B
r1 = B – C
sin i O x-axis
At face AB , =
sin r1
y µ
sin i sin i
= =
sin( B – C ) sin B cos C – cos B sin C
x
sin i The x-co-ordinate of bob as seen by eye
=
sin B 1– sin 2 C – cos B sin C = x sin

Using relation (i) , we get x2 y2


The equation of trajectory is 2
+ =1
( / )2
13. (a) Considering refraction at the curved surface,
sin i µ2 = 1
= u = – 20,
1 1 µ1 = 3/2 , R = + 20
sin B 1 – 2
– cos B. ÷
Applying 2 1
= 2 1
v u R
1 1 1 3/ 2 1 3/ 2
sin i 1–
2 ÷ sin B – cos B ÷ v 20 20
v 10

i.e., 10 cm below the curved surface or 10 cm above


the actual position of flower.
1 14. (b) Firstly the image has to be inverted. Secondly as the
sin i + cos B = 1 – 2 ÷ sin B object point moves from 2f to f images moves from 2f
to infinity on the other side very fast.
(Take three points on the object, find their images,
sin i + cos B 2 see if they are in a straight line)
–1
sin B 15. (c)
2 16. (d) For y = 0, µ = 2
sin i + cos B
2
= 1+
sin B ÷
or 1 1 2(2 –1)
– = v=R
v –R R
2 1/ 2
sin i + cos B For y = d, µ = 4
= 1+
sin B 1 1 2(4 –1) R
– = v' =
v ' –R R 5
Rf
10. (c) Image radius (r1 ) = 4R
d Spreading = v – v ' =
Power collected by rays = (S) r² 5
1 1 1 1
(S ) r 2 Sr 2 d 2 17. (c) = ( µ –1) – = ( µ – 1) ÷
Intensity at focus = 2
= 2 2 f R R
Rf R f
÷ When one surface is silvered
d

1 1
11. (b) Shift = ( m) 1 m 1 =0
n1 n2 ÷
12. (d) For a small oscillations the eye is almost vertically
above the bob.
They y co-ordinate of bob as seen by eye
1 –2( µ –1)
= , f eq = – f / 2
cos feq R
y
concave mirror of focal length = 10 cm
RAY OPTICS 633

18. (a) See figure. The ray will come out from CD if it suffers
1.5 1
total internal reflection at surface AD, i.e., it strikes the Now, =
4/ 3 sin C
surface AD at critical angle C ( the limiting case).
8
P D or sin C = sin 8/9
A 9
C n n2 21. (b) Refraction at convex lens
1
Q 90°–C v1 = 20 cm
Refraction at concave lens u = – 10 cm
1 1 1
B C – =– v = – 5 cm
v –10 –10
Applying Snell's law at P
5mm
n height = = 2.5 mm = 0.25 cm
n1 sin C = n2 or sin C = 2 2
n1 Hence co-ordinate of the final image
Applying Snell's law at Q = (25 cm, 0.25 cm)
n2 sin = n1 cos C 22. (c) Power of combinations

n n 1 1 1
sin = 1 cos sin 1 2 ÷ P = 2 (µ – 1) + ÷
n2 n1 60 20 20 / 2

2 1
n1 n2 P= =
= sin 1
cos sin 1 15 f
n1 ÷
or
n2
Resultant focal length
1 1 1 15
19. (a) = + f=
F f1 f 2 2
From mirror formula
2
1 1 1
+ =
1 30 v f
Solving v = – 10 cm.
Alternatively
Let x be the distance at which object and image
coincide, then
1 1 1 1 1 µ – µ2
= ( µ1 – 1) + + ( µ2 –1) – ÷= 1 1.5 1 1.5 1
F R –R R = x = 15 cm
20 x 60
R
F= 15
µ1 – µ2 feq (of the mirror) =
= 7.5 cm
2
20. (a) Since ray MN is incident normally on face AB,
Now, from mirror formula
hence MNQ = C
v = – 10 cm.

M I
B A O
O
23. (a) Use lens maker's formula
P
N
1
f
( m
gµ 1) 1
R1
1
R2 ÷

C
sin sin C gµ 1.5
Now, g µ = =
m
mµ 1.75
634 IIT-JEE PHYSICS Challenger
For concave lens as shown in figure in this case
(I.N) N
R1= – R and R2 = + R R 2 +I
(N.N)
g
m Using this expression twice, we get the result.

2 j 2k i
R1 R2 R= .
3
MEDIUM
m 29. (b) The optical path of light passing through the slab is
given by
1 1.5 1 1 0.25 2
÷=+
t t x
1
f 1.75 R R 1.75 R = µ dx = 1+ ÷ dx = t 1 + 2 ÷
0 0 t
f = + 3.5 R
30. (c) The magnitude of the velocity of the image will be
The positive sign shows that the lens behaves as
convergent lens. same as that of the object. The difference will be
24. (d) = (45° – 30°) + (180° – 60°) + (45° – 30°) maximum when the velocity of the image is opposite
= 150° clockwise. to that of object. The velocity must be along normal
to the mirror.
31. (c) At t = 0.2 sec,
velocity of lens v1 = gt = 2 m/s (downwards)
30° For lens the fish appears to approach with a speed
45° of

30° 30° 3 11
2+ 1 ÷ = m/s
4 4

45° 24
At a distance 42 + = 60cm.
(4 / 3)
100 100
25. (d) P = 2 + +0 Image of fish from lens,
10 20
P = 10 dioptre. 60 90
V= = 180cm.
26. (b) Velocity of image in mirror 60 + 90
v 10i + 10 j Velocity of image w.r.t. lens
2
v rel u v = 20i. V 2 dU 180 11
= ÷ ÷
27. (d) For maximum deviation U 2 dt 60 4
i1 = i2 = 90°
99
= m/s = 2475 cm/s
4

v 2 du
c 32. (a) Image speed = ÷
u 2 dt

v
For convex mirror <1
1 u
r1 = r2 = = sin 1
µ÷
c
Image speed is always less than v.
A = r1 + r2 = 2 c
1 µ2 1 1
1
R1 R2 ÷
1 1 33. (d)
= – A = – 2 sin . f µ1
µ÷ 1 2 3
If 2 > 1, the concave lens
28. (c) When an incident ray I is reflected by a mirror whose maintains its nature otherwise the
nature of the lens will be
normal is N, the reflected ray is given by the
reversed.
following So, the lens should be filled with L2 and immerse in L1.
RAY OPTICS 635

1 2 2
34. (c)
1
( 1)
1 1
; R = 10 cm.
37. (a) = +
f R1 R2 ÷ f fe f m

////////////////

////////////////
////
R
fm =
2
fm = 0
1 n 1
f=20 f' f=20 =
fe R
n 1
1 1 1 10 fe =
(3 1) ÷, f R
f 10 10 4
R
f =
1 1 4 1 2 4 10 2 2n
+ = feq
f eq 20 10 20 20 10 3 R
f1 =
2 (n 1)
35. (d) Dispersive power ( )
f1 n
nv nr 1.521 1.510 Ratio =
= = = 0.02 f2 n 1
ny 1 1.515 1
38. (d) By hypothesis, the foci of the two lenses are made to
36. (a) Let us consider a certain luminous point A of the coincide, i.e., the separation between the lenses is 3f,
filament and an arbitrary ray AB emerging from it. We where f is the focal length of a lens with a lower focal
draw a plane through the ray and the filament. It power.
follows from geometrical considerations that with all

////////////////////////////
//////////////////////////////
possible reflections, the given ray will remain in the
constructed plane (figure).

D //////////////////////////////
//

B A
//////
/

In the former case, all the rays entering the tube will
////
////

emerge from it and form a circular spot of radius r/2,


///

where r is the radius of the tube (figure). In the latter


// ////

case, only the rays which enter the tube at a distance


/
///
////

/2 smaller than r/2 from the tube axis will emerge from
/2
///

the tube. Such rays will form a circular spot of radius


//

/////////////////////////// C
O r on the screen (figure).
////////////////////////////

A' //////////////////////////////
After the first reflection at the conical surface, the ray
AB will propagate as if it emerged from point A', viz.
the virtual image of point A. The necessary condition
so that none of the rays emerging from A ever gets on
the mirror is that point A' must not be higher than the //////////////////////////////
straight line OC, viz. the second generator of the cone, Thus, if J is the luminous intensity of the light
lying in the plane of the ray (point O is the vertex of entering the tube, the ratio of the illuminances of the
the conical surface). This will be observed if spots before and after the reversal of the tube is
J J /4
E1 = , E2 = ,
A OD AOD AOC 3 180° ( r / 2) 2 r2
2
Consequently, 120° E2 1
min =
E1 16
636 IIT-JEE PHYSICS Challenger
39. (c) For minimum deviation, the ray must be symmetric to and
the prism. The angle of refraction is 30°. It is BOC = ABO as the alternate-interior angles.
independent of refractive index. Hence
A OD = DB = R/2. From the triangle ODC, we obtain
R R2
60° x= =
2 cos 2 R2 h2
30° (C is the point of intersection of the ray reflected by
the mirror and the optical axis).
60°
For a ray propagating at a distance h1, the distance
B C
x1 R/2, with an error of about 0.5% since h12 << R 2 .
| P1 | 2 f2 2
40. (a) = = ...(i) For a ray propagation at a distance h2, the distance
| P2 | 3 f1 3 x2 = 3.125cm. Finally, we obtain x = x2 – x1
Focal length of their combination x 0.6cm.
1 1 1 1 1 1 3 44. (b) As shown in the figure, when the object (O) is placed
between F and C, the image (I) is formed beyond C. It
f f1 f2 30 f1 2 f1 from (i)
is in this condition that when the student shifts his
1 1 3 1 1 eyes towards left, the image appears to the right of the
1 ÷ object pin.
30 f1 2 f1 2
f1 = – 15 cm
Movement towards left
2 2
f2 f1 15 = 10 cm
3 3 F
41. (a) For refracted ray, angle of refraction = 90° and
hence 1 = 90 – I C O
For reflected ray, deviation 2 = – 2

2 1
2
42. (a) It follows from symmetry considerations that the image 45. (c) “O” act as focal point.
of the point source S will also be at a distance b from
1 1 1
the sphere but on the opposite side (figure). ( 1)
f R1 R2 ÷
////////////////////////////

S S' O
1 1 1
b ( 1) + ÷
40 10 10
43. (c) Let O be the centre of the spherical surface of the
1 2 9
mirror, ABC is the ray incident at a distance BE from ( 1) ÷ =
the mirror axis, and OB = R (figure). 40 10 8
////////// 46. (c) An observer on the ship can see only the rays for
/// /////
////
A //
B which sin < 1/ ng1 (if sin > 1/ ng , such a ray
1
///
////

undergoes total internal reflection and cannot be seen


/////

D by the observer, figure). For the angle , we have the


////////////////////////

h
relation
O C
E
x /
///////////////////

///////////////////
////

R
/

D
/ /////
///

///
////
//////// ////

From the right triangle OBE, we find that sin = h/R. h


The triangle OBC is isosceles since
ABO = OBC according to the law of reflection,
//////////////////////////////
RAY OPTICS 637

ng1 50. (b) None of the rays will emerge from the lateral surface
nw sin ng1 sin , sin sin of the cylinder if for a ray with an angle of incidence
nw
where n g1 is the refractive index of glass. Since (figure), the angle of incidence on the inner
2
| sin | < 1/ ng1 , | sin | < 1/ nw .
surface will satisfy the relation sin > 1/n. In this
Therefore, the observer can see only the objects case, the ray will undergo total internal reflection on
emitting light to the porthole at an angle of incidence
the lateral surface.
sin 1 (1/ nw ) . Figure shows that the radius of a
circle at the sea bottom which is accessible to
observation is R h tan , and the sought area

h2
( h tan >> D / 2) is S R2 82m2
nw2 1
47. (a) vobj , mirror = 4iˆ + 9 ˆj
dx dy
= 4, =9 ; u=–x S
dt dt
It follows from geometrical considerations that
1 1 1 10 x
= + ; V = 1
10 V x x 10 sin 1 sin 2 , sin =
n
2
dV 10 dx
v1x = = ÷ ;v1x = – 16 Thus, nmin = 2
dt x 10 dt
51. (a) Let refractive index of glass be µ.
V 10 y0 Let after first refraction, image distance be v then
m = = 1
x x 10 y
1 1 R
10 dy v=
v1 y ÷ ; v1y = –18 v R 1
x 10 dt
Now second refraction will take place.
vimage, mirror 16iˆ 18 ˆj ; vimage 12iˆ 16 ˆj So distance of first image from O is
| v | = 20 cm/s
R R
48. (c) Use the concepts related to image formation by u1 R=
spherical mirrors. 1 1
49. (a) We resolve the velocity vector v of the person into and image is formed at R
two components, one parallel to the mirror, v|| and 1 ( 1) 2 (1 )
=
the other perpendicular to the mirror, v , i.e. R R R
v = v|| + v (figure).
3+ 5
2
3 + 1 = 0 So, =
2
52. (a) This dark ring will be visible if ray from source gets
////////////////////////////////////////////

total internal reflection from the spherical shell.


Q

v|| v||
v'
v
O P
vt

v
v
Let the source at any instant be at point P then at
The velocity of the image will obviously be point Q ray will be totally reflected if is equal to or
v v|| v . Therefore, the velocity at which the greater than critical angle. If QP is equal to x, then
person approaches his image is defined as his velocity
relative to the image. From the formula R2 x 2 v 2t 2
z cos =
vrel = 2v = 2v sin . 2 Rx
638 IIT-JEE PHYSICS Challenger
For to be minimum
f1=10cm. f2=10cm.
dz 2 x (2 Rx ) 2 R ( R 2 x 2 v 2t 2 )
= =0
dx 4R2 x2

x R2 v 2t 2

2 (R2 v2t 2 ) R2 v 2t 2
So, cos = =
2R R2 v 2t 2 R 5cm.
10cm.
1
For no light come out, sin or 45°
2 56. (b) Here, three optical phenomena take place-first
refraction, then reflection, and finally refraction.
R2 v 2t 2 1 R For refraction at 1st.
= ; t=
R 2 2V 1.5 1 1.5 1
=
v 2R R
y0 sin
53. (a) = d tan r ; ÷ =n 1.5
2 sin r 0 v
v
sin 57. (d) µ sin 30° = sin
y0 = 2d ÷
n 2 sin 2
//

///
///////////////
C
R/ 3
R
r
r /

/
r y0
r
1
3. sin = 60°, = 30°
2
R
Point P where it strikes is
d 3

R
AB tan 30° = x=R
54. (b) From the figure in ABC, tan = x 3
AC
AB = AC tan Ray strikes normal to the spherical surface. It
2r = f tan retraces the path.
Area of image = r2µ f 2 Angle of deviation = 180°

58. (d) A
Parallel
Rays from
Focal object placed
plane at infinity i
r1 r2
f
A
on focal plane
Image formed

2r C
B

Biconvex lens For ray to not emerge from second surface, r2 > C
r2 min C
A – r1 max C
55. (a) The diameter of emergent rays is half of the incident but r1 max = C when i = 90°
one. Hence the intensity will be 4-times. A 2C A 72°
RAY OPTICS 639

59. (c) The top portion of the rod is at a distance > 2f. So v is For the unsilvered part :
less than 2f for this portion. So the image would lean Flens = + 40cm.
towards the lens. The image would not be a straight 40 160
line as the lateral magnification is not the same as the So, distance between focii = 40 + = cm.
3 3
longitudinal magnification. 61. (a) The bubble behaves like a diverging lens, since the
60. (b) For equiconvex lens index of refraction of the air in the bubble is less than
1 2( 1) the index of refraction of the water. Thus the image
; 1.5 R= f
f R will be smaller and inverted.
A thin silvered lens is equivalent to a spherical mirror 62. (c) Min. width of plane mirror to see full face
of equivalent focal length : D d 10 4
1 1 2 = = = 3cm.
2 2
Feq Fm F 63. (a) Use the first law of reflection. Only sphere 1 is in a
where Fm = focal length of the silvered surface . position such that light leaving it can reflect from the
F = focal length of the unsilvered lens. mirror to the observer.
Here, for the silvered half, 64. (d) Since the water has a greater index of refraction than
the lens, the magnifier now acts like a diverging lens,
1 1 2 resulting in an image that is smaller than the object.
Feq = – 40/3
Feq 40 40

1. (c); 2. (a); 3. (d) and for x = 90 cm


v = 70 cm
4. (b) For lens L1, ray must move parallel to the axis after
refraction
O
µ1 µ µ –µ
I + = 1 x = 10cm
70 cm x R1
5. (a) For lens L2, image must form at centre of curvature of
the curved surface after refraction through plane part
700 cm 100 cm 700 cm
µ2 µ
+ =0
µ2 µ1 µ2 – µ1 – R2 x '
– = , when x = 70 cm
v u R x = 8 cm
1.5 1.2 1.5 –1.2 6. (b) Length of tube = x + x = 18 cm
– =
v –70 20 µ1 µ2 µ2 µ1
7. (d) + =
x v R
20 70 1.5
v=
–1.2 20 0.3 70 µ2 µ2 µ1 µ1
v = – 700 cm v R x
µ2 µ1 µ2 – µ1 For real image v > 0
– =
v u R µ2 µ1 µ1
or >
1.2 1.5 1.2 – 1.5 R x
– =
v 900 –20 µ1 R
if µ2 > µ1 then x > to form real image.
900 200 1.2 µ2 µ1
v=
1.5 200 – 900 3
µ2 (µ1 µ2 ) µ1
v = – 90 cm 8. (a) +
Similarly, for x = 80 cm v R x
v = 80 cm if µ2 < µ1 then, v is –ve for any value of x.
640 IIT-JEE PHYSICS Challenger

µ1 µ2 µ2 µ1 12. (d) The pin are interchanged and so will be object.


9. (b) + = Image distances
x v R
40 1
µ2 µ2 µ1 µ1 m ÷
+ 120 3
v R x
Thus 1
magnification =
(a) for µ2 > µ1 virtual image is always formed 3
µ1R
(b) for µ2 < µ1 virtual image is formed if x < . uf
µ1 µ2 hi v ri v u f f
13. (a) = = = = =

/////
10. (a) As the magnification is three h0 u r0 u u u f

/////////
x + 80
So only dimension of image changes and not the shape.
x
///////////////
(v is same for every part)
( x + 80) I O
3 x = 40cm. ri v v r0 f
x 14. (b) ri r0 = 1.2cm.
/////

From mirror formula r0 u u u f


/

1 1 1 1 1 1
+ = + = f 30 cm. Area of image ri2 (1.2)2 1.44 cm 2
v u f 120 40 f

v 80 + x dri
11. (d) m 3 x = 40cm. 15. (a) = +0.2
u x dt

4. (d) For lens two focus can be considered.


B
1. (a) µ = A + 2 Statement-1 is false, Statement-2 is true.

L
1 5. (c) P 1÷ ;
and = sin 1
.
µ÷
c M

2. (d) If the mirror is shifted parallel to itself such that the P M y x


velocity of the mirror is parallel to its surface, the image
MIf L lens nature changes.
shall not shift. Hence statement-1 is false.
3. (d) Using a plane mirror we see our virtual image, hence µz > µx > µy.
statement-1 is false. 6. (d) As x increases angle of incidence increases.

1. (a, b, c, d) Motion of mirror perpendicular to its area normal 3. (c, d) From first lens,
will not made any difference to the velocity of its 1 1 1
image. =
v u f
2. (a, b, c, d)
v + u = D and v – u = x 1 1 1
=
D x D x v 30 +15
v= ,u =
2 2 1 1
1 15 30 1
D2 x2 = =
and f = v 30 30
4D
v = + 30 cm
D x D x Thus the first lens will form image at optical centre
m1 = , m2 =
D x D+ x of lens L2 and lens L2 will form image at its optical
itself because object distance is close to zero.
RAY OPTICS 641

4. (a, b, c, d) µ depends on wavelength hence focal length, contour of the face behind the glasses relative to
power, image distance and chromatic aberration other parts of the face : if it is displaced inwards,
depends on the colour/wavelength of the incident the lenses are diverging, and your companion is
light. short-sighted, if its is displaced outwards, the
5. (a, c) Let us first neglect the size of the pupil, assuming lenses are converging, and the person is long-
that it is point-like. Obviously, only those of the sighted.
beams passing through the lens will get into the 7. (a, b, c) n for liquid = n for Glass/yellow light but n for
eye which have passed through point B before liquid < n for glass (red light) deviated toward
they fall on the lens (figure). This point is base
conjugate to the point at which the pupil is located. 8. (b, d) The intensity of light in the region AB (when the
lens is absent) now gets distributed over the region
CD. In the regions AC and BD light intensity is
S
due to both the direct beam and the diverged light
from the lens.
9. (a, c)
B
r
R

a b

(1) (2)

The distance b from the lens to point B can be


calculated by using the formula for a thin lens :
Deviation, = i + e – A
1 1 1 aF dispersion, = ( V – R) = (nV – nR) A
= + , b= = 12cm.
F a b a F
1=– 2
It is clear now that the screen must coincide with 1+ 2=0
the real image of the pupil in the plane S, figure
Final ray will be parallel to initial ray.
shows that the minimum radius of the screen is
But 1 2 1+ 2 0
b
R= r 0.5 mm , and the screen must be placed
a 10. (a, b, c)
in the plane S with its centre at point B.
90°
6. (a, b, c, d) Short-sighted persons use concave (diverging)
glasses which reduce the focal power of their eyes,
while long-sighted persons use convex
(converging) glasses. It is clear that behind a sin 1 1/
diverging lens, the eye will look smaller, and
sin 2 1/ , 1 2 = 90° cos 1 1/
behind a converging lens the eye will look larger.
If, however you have never seen your companion
Remember it is not given that 1 2 = 45°
without glasses, it is very difficult to say whether
his eyes are magnified or reduced, especially if
3 1 2
the glasses are not very strong. The easiest way If 1 = 30° , 1.15
is to determine the displacement of the visible 2 3
642 IIT-JEE PHYSICS Challenger

1. A-s; B-q; C-r; D-p

1 µ2 1 1
1 + .
f µ1 R1 R2 ÷ Image (erect
(B) enlarged)

2. A-q; B-p; C-s; D-r F1

3. A-q; B-r; C-q; D-p


4. A-s; B-p; C-q; D-r
5. A-p, q, s; B-p, s; C-p; D-r F2
(C)
6. A-q, r; B-s; C-s; D-q, r
Image (erect
Concave mirror can form virtual, magnified and real, diminished)
diminished image like convex (converging) lens. Convex
mirror form virtual diminshed image like concave
(diverging) lens.
F2
7. A-r, B-s; C-p; D-q
(D)
Vertual object
3 Image (real, diminished, erect)
(A) Velocity of the image of fish in air = 4 3cm /s ,
4

Velocity of image of fish w.r.t bird = 3 + 6 = 9 cm/s 10. A-s; B-q; C-p; D-r

(B) Velocity of image of fish after reflection from mirror in (A) = 180° 2i

3 sin i
air = 4 3m /s
(B)
4 i r ; ; Increasing i increases .
sin r
w.r.t bird = – 3 + 6 = 3 m/s
sin i 1
4 (C) i r ; sin r = ; Increasing i increases upto
(C) Velocity of the image of bird in water = 6 8 cm /s
3
i < ic ; i > ic TIR occur and = 180° – 2i
w.r.t fish = 8 + 4 = 12 cm/s
(D) For Prism = (i1 + i2 ) A , on increasing i, first
(D) Velocity of the image of bird in water after reflection
decreases attain minimum value and then increases.
from mirror = 8
11. A-p, q, s, t; B-r, t; C-p, q, s, t; D-p, q, t
w.r.t fish = 8 – 4 = 4 cm/s
(A) When a ray of white light is incident on one face of an
8. A-p, q; B-p, r, s; C-p, r; D-p, r equilateral glass prism it will disperse, a divergent beam
In refraction phase remains same. is obtained after refraction and if angle of incidence is
In denser medium speed decreases. greater then critical angle then TIR occurs.

9. A-q, r, t; B-q, r; C-q, s; D-q, s, t (B) When a ray of white light is incident at an angle on a
thick glass sheet, a lateral shift occurs after refraction.
Image (real (C) When a ray of white light passes from an optically
enlarged, erect)
denser medium to an optically rarer medium, it will dis-
(A)
Virtual F perse, a divergent beam is obtained after refraction and
1
object
if angle of incidence is greater than critical angle then
TIR occurs.
RAY OPTICS 643

(D) When a parallel beam of monochromatic light passes 13. A-q, r; B-p, s; C-q, r; D-p, q, r
symmetrically through a glass lens, a divergent beam Convex lens (power positive) can form enlarged (or of
is obtained after refraction and if angle of incidence is same size) real, inverted image.
greater than critical angle then TIR occurs.
Concave lens (power negative) can form virtual, erect,
12. A-p, q, r, s, t; B-q, s; C-q, s; D-p, q, r, s, t diminished image.
For real object, concave mirror & convex lens can form Convex lens (power positive) can form enlarged, vir-
real , virtual , magnified , diminished or of same size tual, erect image.
image. For real object, convex mirror & concave lens
can form virtual and diminished image.

1. 1.73
1
The situation can be shown as in the figure. AA ' = shift = t 1
µ÷
A
Therefore, the object distance

1
u = OA ' = OA AA ' 21 t 1
i
r r'
i' µ÷

B C 1
Here, i = 60°, A = 30°, = 30°, i' = ?
21 3 1 ÷ = 20cm
1.5
We know that, A + = i + i' ....(i)
Also, A = r + r' ....(2) uf 20 5 20
V = = = cm
From (1), u f 20 5 3
i' = A + – i = 30° + 30° – 60° = 0
The reflected rays again through the glass slab the image
As the angle of emergence (i') is 0, hence the emergent ray
should have formed at B is the absence of glass slab. But.
is normal to the face from which it emerges.
due to its presence the image is formed at B'.
When i' = 0, r' = 0
From (2), A = r = 30°.
From Snell's law, R.I. of prism,

sin i sin 60° 3/2


µ= = = = 3 = 1.73. O B
sin r sin 30° 1/ 2 B'

2. 7.67
The rays originating from A (the point object) suffer
refraction before striking the concave mirror.
For the mirror the rays are coming from A' Such that Therefore, image distance = OB + BB'

20 1
t 1
3 µ÷

O
A' A 20 23
+1 = = 7.67 cm
3 3
644 IIT-JEE PHYSICS Challenger
3. 70.8 Alternatively :
f0 = + 50 cm; fe = + 5 cm ; D = 25 cm ; u1 = – 200 cm µ1 µ2 µ2 µ1
For objective lens + =
u v R

1 1 1 1 1 4 1 3
= + = = = 1
+
1.5 1.5 1
=
v0 f 0 u0 50 200 200 200 x = 15 cm
x 20 60
5. 75
200
v0 = e Here R = µ i.e., plane surface is the refracting surface
3
For eyepiece lens

1 1 1 1 1 6 v
u
ue ve fe 25 5 25 I
O
25
ue = cm
6 µ1 µ2 µ2 µ1
=
[ve is take negative because the image is virtual] u v R
Separation between objective and eyepiece µ1 µ 2
+ =0
= | v0 | + | u e | u 3
200 25 400 + 25 425
= + = = = 70.8cm µ2 3
= ...(i)
3 6 6 6 µ1 4
4. 15
Again applying
This silvered concavo-convex lens behaves like a mirror
whose focal length can be calculated by the formula µ1 µ2 µ2 µ1
+ =
u v R

µ1 µ2 / µ1 µ2 / µ1 1
+ =
u v R

R1=60 cm 1 3/ 4 3/ 4 1
+ =
4 25 / 8 R
R2=20 cm

u v
1 2 1
= + I
f f1 f2 O
f1 = focal length of concave surface.
f2 = focal length of concave mirror On solving we get R = – 25cm.
Applying Len's maker formula,
1 2 1 4
= + =
f 30 10 30 1 1 1
(µ 1)
f R1 R2 ÷
f = – 7.5 cm
Using mirror formula
1 4 1 1
1 ÷
1 1 1 1 1 1 f 3 25 µ
= + = + x = 15 cm
f v u 7.5 x x
f = 75cm
RAY OPTICS 645

6. 1.41 The magnification of lens (i) is 2


The light entering the rod does not emerge from the curved
v2 + d
surface of the rod when the angle 90 – r is greater than the 2= ... (ii)
critical angle. u2 d

1 1.8 u2 + d
i.e., µ < where C is the critical angle.
sinC 2= 2u2 – 2d = 1.8 – u2 + d
u2 d
Here C = 90 – r
3u2 = 1.8 + 3d
1 1 u2 = 0.6 + d
µ< µ<
sin (90 r ) cos r
v2 = 1.8 – 0.6 – d

1 v2 = 1.2 – d
As a limiting case µ = ... (i)
cos r Applying lens formula for lens (1)
Applying Snell's law at A
1 1 1
+ = .... (iii) for lens (2)
sin sin v2 d u2 d f
µ= sin r = ... (ii)
sin r µ
1 1 1
The smallest angle of incident on the curved surface is when + = ... (iv)
v2 u2 f

= . This can be taken as a limiting case for angle of From (iii) and (iv)
2
incidence on plane surface. 1 1 1 1
+ = +
From (ii), v2 d u2 d v2 u2

sin / 2 1
sin r = µ= ... (iii) 1 1 1 1
µ sin r + = +
1.2 d + d 0.6 + d d 1.2 d 0.6 + d
From (i) and (ii) sin r = cos r
r = 45° 1 1 0.6 + d + 1.2 d
+ =
1.2 0.6 (1.2 d ) (0.6 + d )
1 1
µ= =
cos 45° 1/ 2
3 1.8
=
µ= 1.2 (1.2 d ) (0.6 + d )
2 = 1.41
This is the least value of the refractive index of rod for light (1.2 – d) (0.6 + d) = 0.6 × 1.2
entering the rod and not leaving it from the curved surface.
(1.2 × 0.6 + 1.2 d – 0.6 d – d2 = 0.6 × 12)
7. 0.4
d (d – 0.6) = 0 d = 0.6 m
Substituting this value in (iv)
1
u2–d v2+d
1 1 1
+ =
1.2 0.6 0.6 + 0.6 f
O
Object
plane d 1 1 1 1.2 + 0.6 1.8
+ = = =
Image 0.6 1.2 f 0.6 1.2 0.6 1.2
u2 2
v2 plane

0.6 1.2
f= = 0.4 m
Given u2 + v2 = 1.8 m ... (i) 1.8
646 IIT-JEE PHYSICS Challenger
8. 30
DE 0.2
's ACE and DCE are congruent, therefore, DE = AE In CDE, tan 30° = or = DE
CE 3
AD = 2DE
2 0.2
AD =
3
2 3m
C B
2 3 2 3 3
No. of reflection on one face = = = 15
30 °

3 0° AD 2 0.2
0.2m
3 0° Total number of reflections on two faces is

D = 15 + 15 = 30
E A
WAVE OPTICS 647

1. Two sources S1 and S2 emitting coherent light waves of


wavelength in the same phase are situated as shown. The
distance OP, so that the light intensity detected at P is equal I0 I0
to that at O is (a) (b)

P
2
S1 S2
O
I0 I0
D (c) (d)
Screen

(a) D 2 (b) D/2 5. Two coherent monochromatic light beams of intensities I


and 4 I are superposed. The maximum and minimum possible
(c) D 3 (d) D/ 3 intensities in the resulting beam are
2. In Young’s double-slit experiment, the separation between (a) 5I and I (b) 5I and 3I
the slits is halved and the distance between the slits and the (c) 9I and I (d) 9I and 3I
screen is doubled. The fringe width is 6. In Fresnel's biprism experiment the width of 10 fringes is
(a) unchanged. (b) halved. 2cm which are formed at a distance of two 2 meter from the
slit. If the wavelength of light is 5100 Å then the distance
(c) doubled (d) quadrupled between two coherent sources will be
3. Two coherent light sources each of P B (a) 5.1 × 10–4 m (b) 5.1 × 104 cm.
wavelength are separated by a distance –4
(c) 5.1 × 10 mm (d) 10.1 × 10–4 cm
3 . The maximum number of minima 7. A beam of light of wave length 600 nm from a distance source
formed on line AB which runs from – to falls on a single slit 1 mm wide and a resulting diffraction
+ is pattern is observed on a screen 2m away. The distance
between the first dark fringes on either side of central bright
fringe is
(a) 2 (b) 4 (a) 1.2 cm (b) 1.2 mm
Q
(c) 6 (d) 8 A (c) 2.4 cm (d) 2.4 mm
4. In a YDSE experiment if a slab whose refraction index can be 8. When a plastic thin film of refractive index 1.45 is placed in
varied is placed in front of one of the slits then the variation the path of one of the interfering waves then the central
of resultant intensity at mid-point of screen with ‘ ’ will be fringe is displaced through width of five fringes. The
best represented by ( 1). [Assume slits of equal width thickness of the film, if the wavelength of light is 05890Å,
and there is no absorption by slab; mid point of screen is will be
the point where waves interfere with zero phase difference (a) 6.544 × 10–4 cm (b) 6.544 × 10–4 m
(c) 6.54 × 10 cm–4 (d) 6.5 × 10–4 cm
in absence of slab]

MARK YOUR 1. 2. 3. 4. 5.
RESPONSE 6. 7. 8.
648 IIT-JEE PHYSICS Challenger
9. The equation of two light waves are (a) /4 (b) /2
y1 = 6 cos t, y2 = 8 cos ( t + f ).
(c) 2 (d)
The ratio of maximum to minimum intensities produced
by the superposition of these waves will be 14. Figure shows two coherent sources S1 – S2 vibrating in
(a) 49 : 1 (b) 1 : 49 same phase. AB is an irregular wire lying at a far distance
(c) 1 : 7 (d) 7 : 1 = 10 3 .
from the sources S1 and S2. Let
10. In an interference arrangement similar to Young’s double- d
slit experiment, the slits S1 and S2 are illuminated with BOA = 0.12 ° . How many bright spots will be seen on
coherent microwave sources, each of frequency 106 Hz. The the wire, including points A and B.
A
sources are synchronized to have zero phase difference. S1
The slits are separated by a distance d = 150.0 m. The
intensity I ( ) is measured as a function of , where is d O
defined as shown. If I0 is the maximum intensity, then I ( ) S2
B
for 0 90° is given by (a) 2 (b) 3
(c) 4 (d) more than 4
S1 15. Yellow light is used in a single slit diffraction experiment
with slit width of 0.6 mm. If yellow light is replaced by
d/2 X– rays, then the observed pattern will reveal,
(a) that the central maximum is narrower
d/2 (b) more number of fringes
S2 (c) less number of fringes
(d) no diffraction pattern
16. Two slits separated by a distance of 1 mm are illuminated
(a) I( ) I0 / 2 for = 30° with red light of wavelength 6.5 × 10–7 m. The interference
fringes are observed on a screen placed 1 m from the silts.
(b) I( ) I0 / 4 for = 90° The distance of the third dark fringe from the central fringe
(c) I( ) I0 for = 0° will be equal to :
(a) 0.65 mm (b) 1.30 mm
(d) I ( ) is constant for all values of . (c) 1.62 mm (d) 1.95 mm
17. A diffraction grating 1 cm wide has 1000 lines and is used to
11. Interference fringes were produced using white light in a third order. Find the difference between the diffraction angles
double slit arrangement. When a mica sheet of uniform
for 400 nm and 600 nm light.
thickness of refractive index 1.6 (relative to air) is placed in
(a) 2° (b) 3.4°
the path of light from one of the slits, the central fringe
moves through some a distance. This distance is equal to (c) 4° (d) 6°
the width of 30 interfernece bands if light of wavelength 18. A thin slice is cut out of a glass cylinder along a plane
4800 Å is used. The thickness (in 1 µm) of mica is parallel to its axis. The slice is placed on a flat glass plate as
(a) 9 0 (b) 12 shown in figure. The observed interference fringes from
(c) 14 (d) 24 this combination shall be
12. In Young's experiment the wavelength of red light is
7.5 × 10–5 cm and that of blue light 5.0 × 10–5 cm. The value of
n for which (n +1)th the blue bright band coincides with nth
red band is
(a) 8 (b) 4
(c) 2 (d) 1 (a) straight
13. Consider Fraunhoffer diffraction pattern obtained with a (b) circular
single slit illuminated at normal incidence. At the angular (c) equally spaced
position of the first diffraction minimum the phase difference (d) having fringe spacing which increases as we go
(in radians) between the wavelets from the opposite edges outwards
of the slit is

MARK YOUR 9. 10. 11. 12. 13.


RESPONSE 14. 15. 16. 17. 18.
WAVE OPTICS 649

19. A broad source of light (I = 680 nm) illuminates normally two 22. A beam of light consisting of two wavelength 6500Aº &
glass plates 120 mm long that touch at one end and are 5200Aº is used to obtain interference fringes in a young's
separated by a wire 0.034 mm in diameter at the other end. double slit experiment. The distance between the slits is 2.0
The total number of bright fringes that appear over the 120 mm and the distance between the plane of the slits and the
mm distance is – screen is 120 cm. What is the least distance from the central
maximum where the bright fringes due to both the wave
incident light
length coincide ?
(a) 0.156 cm (b) 0.152 cm
t 0.034mm (c) 0.17 cm (d) 0.16 cm
23. A thin sheet of mica ( 12 × 10–7 m thick) is placed in the path
120mm of one of the interfering beams in a biprism arrangement. It
(a) 50 (b) 100 is found that the central bright band shifts a distance equal
(c) 200 (d) 400 to the width of a bright fringe. The refractive index of mica
20. In the figure shown in a YDSE, a parallel beam of light is (Given = 6 × 10–7 m) is
incident on the slits from a medium of refractive index n1. (a) 1.0 (b) 1.5
The wavelength of light in this medium is 1. A transparent (c) 1.75 (d) 1.25
slab of thickness t and refractive index is put infront of one 24. Two beams of light having intensities I and 4I interfere to
slit. The medium between the screen and the plane of the produce a fringe pattern on a screen. The phase difference
slits is n2. The phase difference between the light waves between the beams is /2 at point A and at point B. Then
reaching point O (symmetrical, relative to the slit) is
the difference between the resultant intensities at A and B is
n1 (a) 2I (b) 4I
n3
n2 (c) 5I (d) 7I
25. From a medium of index of refraction n1, monochromatic
O
light of wavelength is incident normally on a thin film of
uniform thickness L (where L > 0.1 ) and index of refraction
n2. The light transmitted by the film travels into a medium
with refractive index n3. The value of minimum film thickness
2 2
(a) ( n3 n2 ) t (b) ( n3 n2 ) t when maximum light is transmitted if ( n1 < n2 < n3 ) is
n1 1 1

n1 n1
2 n1 n3 2 n1 (a) (b)
(c) 1÷ t (d) ( n3 n2 ) t 2 n2 4 n2
n2 1 n2 1

21. In a double slit experiment instead of taking slits of equal


widths, one slit is made twice as wide as the other. Then, in (c) (d)
4n2 2n2
the interference pattern
(a) the intensities of both the maxima and the minima 26. Interference fringes were produced in Young's double slit
increase experiment using light of wave length 5000 Å. When a film
(b) the intensity of the maxima increases and the minima of material 2.5 × 10–3 cm thick was placed over one of the
has zero intensity slits, the fringe pattern shifted by a distance equal to 20
(c) the intensity of the maxima decreases and that of the fringe width. The refractive index of the material of the film
minima increases is -
(d) the intensity of the maxima decreases and the minima (a) 1.25 (b) 1.33
has zero intensity (c) 1.4 (d) 1.5

MARK YOUR 19. 20. 21. 22. 23.


RESPONSE 24. 25. 26.
650 IIT-JEE PHYSICS Challenger
27. In a Young’s double slit experiment, 12 fringes are observed
to be formed in a certain segment of the screen when light of
wavelength 600 nm is used. If the wavelength of light is
S1
changed to 400 nm, number of fringes observed in the same
segment of the screen is given by
d
(a) 12 (b) 18 S0
O
(c) 24 (d) 30
28. A plastic sheet (refractive index = 1.6) covers one slit of a S2
double slit arrangement meant for the Young's experiment.
When the double slit is illuminated by monochromatic light D
(wavelength in air = 6600 Å), the centre of the screen appears
dark rather than bright. The minimum thickness of the plastic (a) Larger the wavelength, larger will be the fringe width.
sheet to be used for this to happen is: (b) If white light is used, violet colour forms its first maxima
(a) 3300 Å (b) 6600 Å closest to the central maxima.
(c) 2062 Å (d) 5500 Å (c) The central maxima of all wavelength coincide.
29. The intensity of a point source of light, S, placed at a distance (d) All of the above
d in front of a screen A, is I0 at the center of the screen. Find 32. Monochromatic light of wavelength 400 nm and 560 nm are
the light intensity at the center of the screen if a completely incident simultaneously and normally on double slits
reflecting plane mirror M is placed at a distance d behind the apparatus whose slits separation is 0.1 mm and screen
source, as shown in figure. distance is 1m. Distance between areas of total darkness
will be
M A
(a) 4 mm (b) 5.6 mm
s (c) 14 mm (d) 28 mm
d × d 33. Consider the YDSE arrangement shown in figure. If d = 10
then position of 8th maxima is

27 I0 25 I0 y
(a) (b) d
9 9

17 I0 10 I0
(c) (d) D >>d
9 9
30. In the ideal double-slit experiment, when a glass-plate D D
(a) y = (b) y =
(refractive index 1.5) of thickness t is introduced in the path 10 3
of one of the interfering beams (wave-lenght ), the
4 4D
intensity at the position where the central maximum occurred (c) y = D (d) y =
5 3
previously remains unchanged. The minimum thickness of
the glass-plate is 34. In a Young’s double slit experiment, the fringes are displaced
by a distance x when a glass plate of refractive index 1.5 is
(a) 2 (b) 2 /3
introduced in the path of one of the beams. When this plate
(c) /3 (d) is replaced by another plate of same thickness, the shift of
31. The figure shows a schematic diagram for Young’s double fringes is (3/2) x. The refractive index of second plate is
slit experiment. Given d << , d << D, / D << 1. Which of (a) 1.75 (b) 1.50
the following is a right statement about the wavelength of (c) 1.25 (d) 1.00
light used?

MARK YOUR 27. 28. 29. 30. 31.


RESPONSE 32. 33. 34.
WAVE OPTICS 651

35. In Young’s double slit experiment intensity at a point is (1/4) 39. The sky is blue because
of the maximum intensity. Angular position of this point is (a) most polluting gases and dust particles in the air are
(a) sin–1( /d) (b) sin–1( /2d) bluish in colour and lend their colour to that of the sky.
–1
(c) sin ( /3d) (d) sin–1( /4d) (b) air molecules absorb red light more efficiently than they
36. A Young’s double slit experiment is conducted in a liquid of do blue light because of their electron orbitals.
refractive index 1 and a glass plate of thickness t and
(c) tiny particles in the air are more efficient at scattering
refractive index 2 is placed in path of one slit. The magnitude
short wavelength light than they are at scattering long
of the optical path difference at centre of screen will be
wavelength light.
2 1 (d) air molecules absorb blue light more efficiently than
(a) 1÷ t (b) 1÷ t
1 2 they do red light because of their electron orbitals
40. At night approximately 500 photons per second must enter
(c) | ( 2 – 1) t | (d) | ( 2 – 1) t |
an unaided human eye for an object to be seen. A light bulb
37. A thin convex lens of focal length f = 0.6m is a cut into two
unequal parts L1 and L2. One part is shifted along the cutting emits about 5.00 × 1018 photons per second uniformly in all
plane axis as shown in figure. A monochromatic line source directions. The radius of the pupil of the eye is about 4×10–3
S, perpendicular to the plane of paper, emitting light of meters. What is the maximum distance from which the bulb
wavelength = 600nm, is placed on the cutting plane axis. A could be seen ?
screen with slits where the images of S is formed by these (a) 2.0 × 104 m (b) 2.0 × 105 m
two pieces of the lens separately is placed perpendicular to (c) 2.0 × 102 m (d) 5.0 × 103 m
the optical axis from the source at 4.9m. There is an another 41. A certain region of a soap bubble reflects red light of
screen placed at distance 0.6m normal to optical axis where
vaccum wavelength = 650nm. What is the minimum thick-
fringes are observed due to interference of the light passes
through the holes. Find the position of central maximum ness that this region of the soap bubble could have? Take
from P. [Dotted line represent the principal axis of lens L1] the index of reflection of the soap film to be 1.41.
(a) 1.2 10–7m (b) 650 10–9m
Screen Screen (c) 120 107m (d) 650 105m
42. A double slit arrangement produces fringes for = 5890 Å
L1
that are 0.4° apart. What is the angular width if the entire
6mm arrangement is immersed in water ? (µ w = 4/3)
P
S (a) 0.3° (b) 2.3°
L2
(c) 0.8° (d) 1.3°
43. Screen S is illuminated by two points source A and B.
4.9m 0.6m
Another source C sends a parallel beam of light towards
the point P on the screen. Line AP is normal to the screen
and line AP, BP and CP are in one plane. The distance AP,
(a) 24.5mm. (b) 12.5mm.
BP and CP are 3m, 1.5m and 1.5m respectively. The radiant
(c) 18.5mm. (d) 6.5mm.
powers of source A and B are 90 and 180W respectively
38. A physics professor wants to find the diameter of a human and the beam from C is of the intensity 20W/m2. Calculate
hair by placing it between two flat glass plates, illuminating
the intensity at P on the screen.
the plates with light of vaccum wavelength = 552nm
and counting the number of bright fringes produced along W W
the plates. The Professor find 125 bright fringes between (a) 14 2 (b) 10
m m2
the edge of the plates and the hair. What is the diameter of
the hair?
m2 m2
(a) 525 10–9m (b) 344 10–3m (c) 10 (d) 14
W W
(c) 3.44 10–5m (d) none of the above

MARK YOUR 35. 36. 37. 38. 39.


RESPONSE 40. 41. 42. 43.
652 IIT-JEE PHYSICS Challenger
44. A ray of light travels through a slab as shown.

////////////////////////////////////////////////
x=0 x = 1m

S2

The refractive index of the material of the slab varies as µ = S d

S1
x2
1.2 + , where 0 x 1m. What is the equivalent optical
2 2D D
path of the glass slab ?
D 2D
(a) 1.212 m (b) 1.367 m (a) (b)
d d
(c) 0.123 m (d) 2.124 m
45. White light used to illuminate the two slits in Young’s 3D 4D
(c) (d)
double slit experiment. The separation between the slits is d d
d and the distance between the screen and the slit is D 49. A possible means for making an airplane invisible to radar is
(>>d). At a point on the screen in front of one of the slits, to coat the plane with an antireflective polymer. Radar waves
have a wavelength of 3.00 cm and the index of refraction of
certain wavelengths are missing. The missing wavelengths
the polymer is n = 1.50. How thick would you make the
are
coating?
(a) 1.50 cm (b) 3.00 cm
d2 (2n + 1)d 2
(a) = (b) = (c) 0.50 cm (d) None of these
( 2n + 1)D D 50. In a two-slit experiment, with monochromatic light, fringes
are obtained on a screen placed at some distance from the
d2 =
(n + 1)D slits. If the screen is moved by 5 × l 0 –2 m towards the slits,
(c) = (d)
(n + 1)D d2 the change in fringe width is 10–3 m. Then the wavelength
of light used is (given that distance between the slits is 0.03
46. Young’s double slit experiment is made in a liquid. The 10th mm)
bright fringe lies in liquid where 6th dark fringe lies in vacuum. (a) 4000 Å (b) 4500 Å
The refractive index of the liquid is approximately (c) 5000 Å (d) 6000 Å
(a) 1.8 (b) 1.5 51. Specific rotation of sugar solution is 0.5 deg m2/kg. 200 kgm–3 of
impure sugar solution is taken in sample polarimeter tube of
(c) 1.3 (d) 1.6
length 20 cm and optical rotation is found to be 19º. The
47. The wavelength of visible light in air can be percentage of purity of sugar is
(a) 5 nm (b) 5 mm (a) 20% (b) 80%
(c) 50 µm (d) 500 nm (c) 95% (d) 89%
52. If white light is used in the Newton’s rings experiment, the
48. A double slit, S1 – S2 is illuminated by a light source S colour observed in the reflected light is complementary to
emitting light of wavelength . The slits are separated by a that observed in the transmitted light through the same point.
distance d. A plane mirror is placed at a distance D in front This is due to
of the slits and a screen is placed at a distance 2D behind (a) 90º change of phase in one of the reflected waves
the slits. The screen receives light reflected only by the (b) 180º change of phase in one of the reflected waves
plane mirror. The fringe-width of the interference pattern on (c) 145º change of phase in one of the reflected waves
the screen is (d) 45° change of phase in one of the reflected waves

MARK YOUR 44. 45. 46. 47. 48.


RESPONSE 49. 50. 51. 52.
WAVE OPTICS 653

53. An unpolarised beam of intensity I0 is incident on a pair of (b) reflected and refracted beams are partially polarised
nicols making an angle of 60° with each other. The intensity (c) the reason for (a) is that almost all the light is reflected
of light emerging from the pair is (d) all of the above
(a) I0 (b) I0/2 56. As a result of interference of two coherent sources of light
(c) I0/4 (d) I0/8 energy is :
54. The width of the diffraction band varies (a) redistributed and the distribution does not vary with
time
(a) inversely as the wavelength (b) increased
(b) directly as the width of the slit (c) redistributed and that distribution changes with time
(c) directly as the distance between the slit and the screen (d) decreased
(d) inversely as the size of the source from which the slit is 57. Waves that can not be polarised are :
illuminated (a) electromagnetic waves
55. When unpolarised light beam is incident from air onto glass (b) light waves
(n = 1.5) at the polarising angle : (c) longitudinal waves
(a) reflected beam is polarised 100 percent (d) transverse waves

MARK YOUR
53. 54. 55. 56. 57.
RESPONSE

1. The y co-ordinates of central maxima at any time t is


PASSAGE-1
For the Young’s double slit experiment a monochromatic D sin D cos
(a) (b)
source light is used whose wavelength is strikes on the n0 + kt n0 + kt
slits, separated by distance d as shown in the figure.
D sin D cos
(c) (d)
P'
( n0 + kt ) 2
( n0 + kt )2
2. The velocity of central maxima at any time t as a function of
y time t is
2 n = (n0 + kt)
–2kD sin – kD sin
O (a) (b)
d (n0 + kt ) 2
(n0 + kt )2
S1
–2kD sin – kD sin
(c) (d)
(n0 + kt ) (n0 + kt )
D

MARK YOUR
1. 2.
RESPONSE
654 IIT-JEE PHYSICS Challenger
3. If a glass plate of small thickness b is placed in front of S1.
How should its refractive index vary with time so that central
PASSAGE-3
maxima is formed at O.
In a Young’s double slit experiment a parallel beam containing
2d sin 2d sin
(a) n0 + kt + (b) n0 + kt – wavelengths 1 = 4000 Å and 2 = 5600 Å incident at an
b b angle = 30° on a diaphragm having narrow slits at a
separation d = 2mm. The screen is place at a distance D =
d sin d sin
(c) n0 + kt – (d) n0 + kt + 40cm. from slits. A mica slab of thickness t = 5mm is placed
b b
in front of one of the slits and whole the apparatus is
submerged in water. If the central bright fringe is observed
PASSAGE-2 at C, which is equidistant from both the slits then
In YDSE two slits S1 and S2 are giving light of wavelength
and intensity I0 and are in the same phase. Two pin holes
S3 and S4 are made in screen 1 as shown in figure. S1

Screen2
Screen1 C
S1 S3
D
3d
S2
d Screen
D O2
4d
S2 7. The refractive index of the slab is
S4
(a) 1.6 (b) 1.2
D
D (c) 2.6 (d) 3.65
8. The distance of the first black line from C is
(a) 120 µm (b) 105 µm
4. The ratio of intensity at S3 and S4 will be
(c) 210 µm (d) 180 µm
1
(a)
2
PASSAGE- 4
(b) 2
In the figure shown light of wavelength = 5000 Å is incident
(c) 1
on the slits (in a horizontal fixed plane) S1 and S2 separated
(d) .
by distance d = 1 mm. A horizontal screen S is released from
5. The ratio of maximum and minimum intensity on screen 2 is
rest from initial distance D0 = 1m from the plane of the slits.
2 Taking origin at O and positive x and y axis as shown, (Use
2 +1 2 +1
(a) ÷ (b) ÷ g = 10 m/s2)
2 1 2 1
= 5000Å
S1 d=1mm S2
4 2 2 Fixed
(c) (d) ÷. x
1 2+ 2 O
6. The intensity of light at a point eqidistant from S3 and S4 on y
D0=1m
screen 2 is
(a) 3I0 (b) 9I0
(c) (3 + 6)I 0 (d) zero. S Screen

MARK YOUR 3. 4. 5. 6. 7.
RESPONSE 8.
WAVE OPTICS 655

9. Velocity of central maxima in vector form at t = 2 sec. is I. Refraction at an interface never changes the phase of
(a) 20jˆ m / s (b) 5jˆ m / s the wave.
(c) 10 ˆj m / s (d) 25 ˆj m / s II. For reflection at the interface between two media 1
10. Acceleration of central maxima in vector form at t = 2 sec. is and 2. If n1 < n2 the reflected wave will change phase.
(a) 5 ˆj m / s 2 (b) 15 ˆj m / s 2 If n1 > n2 the reflected wave will not undergo a phase
(c) 10 ˆj m / s 2 (d) 20 ˆj m / s2 change.
11. Relative acceleration of second maxima with respect to first
minima, on the same side of central maxima is Medium 1, n1
(a) 2.5 10 3 m / s 2 ˆi (b) 7.5 10 3 m / s 2 ˆj
3 2 ˆ
(c) 5.5 10 m / s i (d) 7.5 10 3 m / s 2 ˆi
Medium 2, n2

For reference. nair =1.00


PASSAGE- 5
III. If the waves are in phase after reflection at all
Thin films, including soap bubbles and oil sticks, show patterns interfaces, then the effects of path length in the film
of alternating dark and bright regions resulting from interference are :
among the reflected light waves. If two waves are in phase their Constructive interference occurs when 2t = m /n m
crest and troughs will coincide. The interference will be = 0, 1, 2, 3, .......
constructive and the amplitude of either constituent wave . If the Destructive interference occurs when
two waves are out of phase by 1/2 a wavelength (180°), the crests
of one wave will coincide with the troughs of the other wave. The 1
2t = (m + ) / n m = 0, 1, 2, 3,...
interference will be destructive and the amplitude of the resultant 2
wave will be less than that of either constituent wave. If the wave are 180° out of phase after reflection at all
At the interface between two transparent media, some light is interfaces then the effects of path length in the film
reflected and some light is refracted. are :
1. When incident light, I, reaches the surface at point a, some Constructive interference occurs when
of the light is reflected as ray Ra and some is refracted
following the path ab to the back of the film. 2t = (m + 1 / 2) / n m = 0, 1, 2, 3 ......
2. At point b some of the light is refracted out of the film and Destructive interference occurs when
part is reflected back through the film along path bc. At 2t = m /n m = 0, 1, 2, 3, ......
point c some of the light is reflected back into the film and 12. A thin film with index of refraction 1.50 coats a glass lens
part is reflected out of the film as ray Rc with index of refraction 1.80. What is the minimum thickness
of the thin film that will strongly reflect light with
I R
a wavelength 600 nm?
R
c
(a) 150 nm (b) 200 nm
(c) 300 nm (d) 450 nm
13. A thin film with index of refraction 1.33 coats a glass lens
a c
with index of refraction 1.50. Which of the following choices
is the smallest film thicknesses that will not reflect light
film with wavelength 640 nm?
thickness, t
(a) 160 nm (b) 240 nm
(c) 360 nm (d) 480 nm
b
14. A soap film of thickness t is surrounded by air and is
Ra and Rc are parallel . However, Rc has traveled the extra illuminated at near normal incidence by monochromatic
distance within the film of abc. If the angle of incidence is
light with wavelength in the film. With respect to the
small, then abc is approximately twice the film’s thickness.
wavelength of the monochromatic light in the film, what
If Ra and Rc are in phase, they will undergo constructive
film thickness will produce maximum constructive
interference and the region ac will be bright . If Ra and Rc
interference?
are out of phase, they will undergo destructive interference
and the region ac will be dark. 1 1
The thickness of the film and the refractive indices of the (a) (b)
4 2
media at each interface determine the final phase
(c) 1 (d) 2
relationship between Ra and Rc

MARK YOUR 9. 10. 11. 12. 13.


RESPONSE 14.
656 IIT-JEE PHYSICS Challenger
15. The average human eye sees colors with wavelengths (a) The waves are close to destructive interference
between - 430 nm to 680 nm. For what visible wavelength(s) (b) The waves are close to constructive interference
will a 350-nm thick n = 1.35 soap film produce maximum (c) The waves show complete destructive interference
destructive interference? (d) The waves show complete constructive interference
(a) 945 nm (b) 473 nm 17. A thin film of liquid polymer n = 1.25 coats a slab of Pyrex,
(c) 315 nm (d) None of these n = 1.50. White light is incident perpendicularly to the film.
16. A 600 nm light is perpendicularly incident on a soap film In the reflections full destructive interference occurs for
600 nm and full constructive interference occurs for =
suspended in air. The film is 1.00 m thick with n = 1.35.
700 nm. What is the thickness of the polymer film?
Which statement most accurately describes the interfer-
(a) 120 nm (b) 280 nm
ence of the light reflected by the two surfaces of the film?
(c) 460 nm (d) 840 nm

MARK YOUR
15. 16. 17.
RESPONSE

difference is zero appears bright.


INSTRUCTIONS : In the following questions an Statement –1 is Statement - 2 : In YDSE set up magnitude of
given followed by a Statement – 2. Mark your responses from the electromagnetic field at central bright fringe
following options. is not varying with time.
3. Statement - 1 : Radio waves can be polarised.
(a) Both Statement - 1 and Statement - 2 are true and
Statement - 2 : Sound waves in air are longitudinal in
Statement - 2 is the correct explanation of 'Statement - 1'
nature.
(b) Both Statement - 1 and Statement - 2 are true and Statement
- 2 is not the correct explanation of 'Statement - 1' 4. Statement - 1 : Newton’s rings are formed in the reflected
(c) Statement - 1 is true but Statement - 2 is false system when the space between the lens
(d) Statement - 1 is false but Statement - 2 is true and the glass plate is filled with a liquid of
refracitve index greater than that of glass,
1. Statement - 1 : In YDSE, as shown in figure, central bright the central spot of the pattern is bright.
fringe is formed at O. If a liquid is filled Statement - 2 : This is because the reflection in these cases
between plane of slits and screen, the will be from a denser to rarer medium and
central bright fringe is shifted in upward the two interfering rays are reflected under
direction. similar conditions.
S1 5. Statement - 1 : Corpuscular theory fails in explaining the
velocities of light in air and water.
Statement - 2 : According to corpuscular theory, light
O should travel faster in denser medium than
in rarer medium.
6. Statement - 1 : In Young’s experiment, the fringe width for
S2 dark fringes is equal to that for white
Statement - 2 : If path difference at O increases y- fringes.
coordinate of central bright fringe will Statement - 2 : In Young’s double slit experiment the
change. fringes are performed with a source of white
2. Statement - 1 : In standard YDSE set up with visible light, light, then only black and bright fringes
the position on screen where phase are observed.

MARK YOUR 1. 2. 3. 4. 5.
RESPONSE 6.
WAVE OPTICS 657

1. Two point monochromatic and coherent sources of light of 4. White light is used to illuminate the two slits in a Young’s
wavelength are placed on the dotted line in front of an double slit experiment. The separation between the slits is b
infinite screen. The source emit waves in phase with each and the screen is at a distance d (> b) from the slits. At a
other. The distance between S1 and S2 is d while their point on the screen directly in front of one of the slits, certain
distance from the screen is much larger. wavelengths are missing. Some of these missing
wavelengths are

b2 2b 2
S1 S2 (a) = (b) =
O d d
d
b2 2b 2
(c) = (d) =
3d 3d
3 5. Two beams of light having intensities I and 4I interfere to
(a) if d is , at O minima will be observed
2 produce a fringe pattern on a screen. The phase difference
between the beam is /2 at a point A and at point B. Then
11 3 the difference between resultant intensities at A and B is.
(b) if d is , then intensity at O will be of maximum
6 4 (a) 2I (b) 4I
(c) 5I (d) 7I
intensity
6. In Young’s double slit experiment, if distance between the
(c) If d is 3 , O will be a maxima slits is doubled, then to keep fringe width unchanged
(a) wavelength should be double
7 3
(d) if d is , the intensity at O will be of maximum (b) distance between screen and slits should be doubled
6 4 (c) both should remain constant
intensity (d) none of the above
2. In the Young’ s double slit experiment, the interference 7. In figure, Young’s double slit experiment., Q is the position
pattern is found to have an intensity ratio between the bright of the first bright fringe on the right side of O. P is the 11th
and dark fringes as 9. This implies that fringe on the other side, as measured from Q. If = 6000
Å, then S1B will be equal to
(a) the intensities at the screen due to the two slits are 5
units and 4 units respectively Q
S1
(b) the intensities at the screen due to the two slits are 4 B
units and 1 units respectively
(c) the amplitude ratio is 3 S2 O
(d) the amplitude ratio is 2
3. Young’s double slit experiment is conducted with a slit P
separation of 0.5 mm, with the distance between the slit and (Fig. 15.2)
screen being 1 m. The wavelength of light used is 5000 Å. (a) 6 × 10–6 m (b) 6.6 × 10–6 m
The size of fringes formed on the screen if a parallel beam of –7
(c) 3.318 × 10 m (d) 3.144 × 10–7 m
light is incident, making an angle of 1/100 radian with the x 8. Which of the following are essential for producing
normal to the plane of the double slit is interference in Young’s double slit experiment ?
(a) Constant phase (b) Same wavelength
(a) 1 m (b) 1 mm
(c) Same amplitude (d) Same intensity
(c) < 1 mm (d) > 1 mm

MARK YOUR 1. 2. 3. 4. 5.
RESPONSE 6. 7. 8.
658 IIT-JEE PHYSICS Challenger

1. Column I Column II
(A) Young’s double slit experiment uses (p) Incoherent sources
(B) Sources of variable phase difference (q) Coherent source
(C) A point on a wavefront behaves as a light source (r) Superposition principle
(D) Net displacement is vector sum of individual (s) Huygen’s principle
displacements
2. Column - I Column - II
(A) If Young’s double slit experiment is performed in water instead of air (p) equal
then the fringe pattern will
(B) A plane wave front is incident normally on a circular operature and (q) more
diffraction pattern is obtained on the screen on another side of aperture.
On displacing the screen towards operture, the number of HPZ exposed
through the aperture is
(C) If the wavelength of a wave is large than the degree of diffraction (r) increase
observed
(D) For best contrast between maxima and minima in the interference (s) shrink
pattern of Young’s double slit experiment the intensity of light
emerging out of the two slits should
3. S1 and S2 in column I represent coherent point sources, S represents a point source.
= wavelength of light emitted by the sources.
Column I Column II
S1
2
(A) (p) Number of maxima = 2
/////////////////////////////////////////////////
S2 Infinite screen

1. 2.

MARK YOUR
RESPONSE
WAVE OPTICS 659

(B) (q) Number of minima = 2


S1 S2

Screen
S (cylindrical)

(C) (r) Number of maxima = 4

/////////////////////////////////////////////////
Mirror
Screen

Screen
60°
(D) S1 (s) Number of minima = 4
2 S2

(t) Number of maxima = 7


4. Column - I Column - II
(A) The fringe width for red colour as compared to that for violet (p) half
colour is approximately
(B) In a Young’s double slit experiment, the distance between the slits (q) double
is halved and the distance between slit and screen is doubled then
the fringe width will become
(C) If the distance of a surface from light source is doubled then the (r) quarter times
illuminance will become
(D) Light from a lamp is falling normally on a small (s) four times
surface. If the surface is tilted to 60° from this position, then the
illuminance of the surface will become

3. p q r s t 4.
A p q r s t
MARK YOUR B p q r s t
RESPONSE C p q r s t
D p q r s t
660 IIT-JEE PHYSICS Challenger

1. A vessel ABCD of 10 cm width has two small slits S1 and S2 P


Screen
sealed with identical glass plates of equal thickness. The
distance between the slits is 0.8 mm. POQ is the line
perpendicular to the plane AB and passing through O, the
middle point of S1 and S2. A monochromatic light source is
kept at S, 40 cm below P and 2 m from the vessel, to illuminate
D
the slits as shown in the figure below. Calculate the position
of the central bright fringe (in cm) on the other wall CD with
respect to the line OQ.
A
D
h S
A B
S1 Calculate the ratio of the minimum to the maximum
P intensities in the interference fringes formed near the point
Q
O P (shown in the figure).
40 cm S2 4. The Young’s double slit experiment is done in a medium of
refractive index 4/3. A light of 600 nm wavelength is falling
S 2m 10 cm on the slits having 0.45 mm separation. The lower slit S2 is
B C covered by a thin glass sheet of thickness 10.4 m and
refractive index 1.5. The interference pattern is observed on
2. In YDSE a light containing two wavelengths 500 nm and a screen placed 1.5 m from the slits as shown in Figure.
700 nm are used. Find the minimum distance (in mm) where Y
maxima of two wavelengths coincide. Given D/d = 103, where
D is the distance between the slits and the screen and d is
the distance between the slits.

3. A point source S emitting light of wavelength 600 nm is S1


placed at a very small height h above a flat reflecting surface
AB (see figure). The intensity of the reflected light is 36% S* O
of the incident intensity. Interference fringes are observed
on a screen placed parallel to the reflecting surface at a S2
very large distance D from it.

Find the location (inµm) of the central maximum (bright fringe

1. 2. 3. 4.

MARK
YOUR
RESPONSE
WAVE OPTICS 661

with zero path difference) on the y-axis. thickness (in µm) of the glass plate. (Absorption of light by
[All wavelengths in this problem are for the given medium glass plate may be neglected.)
of refractive index 4/3. Ignore dispersion] 8. In a modified Young’s double slit experiment, a
5. A coherent parallel beam of microwaves of wavelength = monochromatic uniform and parallel beam of light of
0.5 mm falls on a Young’s double slit apparatus. The wavelength 6000 Å and intensity (10/ ) W m–2 is incident
separation between the slits is 1.0 mm. The intensity of normally on two circular apertures A and B of radii 0.001 m
microwaves is measured on a screen placed parallel to the and 0.002 m respectively. A perfectly transparent film of
plane of the slits at a distance of 1.0 m from it as shown in thickness 2000 Å and refractive index 1.5 for the wavelength
Fig. of 6000 Å is placed in front of aperture A, see fig.

A
d = 1.0 mm F
x
B

D = 1.0m

Screen Calculate the power (in micro-watt) received at the focal


spot F of the lens.
If the incident beam falls normally on the double slit The lens is symmetrically placed with respect to the
apparatus, find the sum of the magnitudes (in m) of the apertures. Assume that 10% of the power received by each
aperture goes in the original direction and is brought to the
y-coordinates of all the interference minima on the screen.
focal spot.
6. In Young’s experiment, the source is red light of wavelength 9. Screen S is illuminated by two point sources A and B.
7 × 10–7 m. When a thin glass plate of refractive index 1.5 at Another source C sends a parallel beam of light towards
this wavelength is put in the path of one of the interfering point P on the screen (see figure). Line AP is normal to the
beams, the central bright fringe shifts by 10–3 m to the screen and the lines AP, BP and CP are in one plane. The
distance AP, BP and CP are in one plane. The distance AP,
position previously occupied by the 5th bright fringe. Find
BP and CP are 3 m, 1.5 m and 1.5 m respectively. The radiant
the thickness (in µm) of the plate. powers of sources A and B are 90 watts and 180 watts
7. In Young’s experiment, the upper slit is covered by a thin respectively. The beam from C is of intensity 20 watt/m2.
glass plate of refractive index 1.4 while the lower slit is Calculate the intensity (in W/m2)at P on the screen.
covered by another glass plate, having the same thickness C
as the first one but having refractive index 1.7. Interference
pattern is observed using light of wavelength 5400 Å. It is
found that the point P on the screen where the central 60°
maximum (n = 0) fell before the glass plates were inserted × P
A 60°
now has 3/4 the original intensity. It is further observed that
what used to be the fifth maximum earlier, lies below the
point P while the sixth minimum lies above P. Calculate the
B S

5. 6. 7. 8. 9.

MARK
YOUR
RESPONSE
662 IIT-JEE PHYSICS Challenger

1 (c) 11 (d) 21 (a) 31 (d) 41 (a) 51 (c)


2 (d) 12 (c) 22 (a) 32 (d) 42 (a) 52 (b)
3 (c) 13 (c) 23 (b) 33 (d) 43 (a) 53 (c)
4 (c) 14 (b) 24 (b) 34 (a) 44 (b) 54 (c)
5 (c) 15 (d) 25 (b) 35 (c) 45 (a) 55 (a)
6 (a) 16 (c) 26 (c) 36 (a) 46 (a) 56 (a)
7 (d) 17 (d) 27 (b) 37 (a) 47 (d) 57 (c)
8 (a) 18 (a) 28 (d) 38 (c) 48 (d)
9 (a) 19 (b) 29 (d) 39 (c) 49 (c)
10 (c) 20 (a) 30 (a) 40 (b) 50 (d)

1 (a) 5 (b) 9 (a) 13 (c) 17 (d)


2 (b) 6 (c) 10 (c) 14 (a)
3 (c) 7 (a) 11 (d) 15 (b)
4 (a) 8 (c) 12 (b) 16 (d)

1 (d) 3 (b) 5 (a)


2 (c) 4 (a) 6 (c )

1 (a, b, c, d) 3 (b) 5 (b) 7 (a)


2 (b, d) 4 (a, c) 6 (a, b) 8 (a, b)

1. A-q, r, s; B-p; C-s; D-r 3. A-p, q; B-r, s; C-s, t; D-p


2. A-s; B-r; C-q; D-p 4. A-q; B-s; C-r; D-p

1 2 2 3.5 3 0.06 4 4330


5 2.78 6 7 7 9.3 8 7
9 13.9
WAVE OPTICS 663

1. (c) Referring to the figure, the path difference between the D


two waves starting from S1 and S2 turns out to be 6. (a) d= ........(1)
(2 cos ) = n where n is taken as 1 to get the point of
According to question, = 5100 × 10–10 m
maximum intensity which is the same as a point O.
Therefore, the above relation gives cos = 1/2 so that 2
10 –2 m ........(2)
= 60° 10
D = 2m, d = ?
and tan = PO/D = 3 , giving PO = D 3 .
From eqs. (1) and (2)
path difference 2 51 10 8
P d= = 5.1 × 10–4 m
2 10 3
O 7. (d) The distance between the first dark fringe on either
S1 S2 side of the central maximum = width of central maximum
2 screen
9
D 2D 2 2 600 10
= = = 2.4 × 10 – 3 m = 2.4 mm
2. (d) The fringe width in Young's double slit experiment is
a 10 3
(0.45)t
D
= 8. (a) X0 ( 1)t 5 = 10
d 5890 10
where is the wavelength of light used. D is the 5 5890 10 10
distance between slit and screen. d is the distance t= = 6.544 × 10–4 cm
0.45
between the slit.
9. (a) a1 = 6 units, a2 = 8 units
(2 D ) D
'= =4 =4 2 2
d /2 d a1 6
+1 +1
3. (c) There can be three minima from central point to I max a2 8 I max 49
= = =
I min 2 2 I min 1
3 5 a1 6
corresponding to , ,path differences. 1 1
2 2 2 a2 8
total number of minima = 2nmax = 6.
4. (c) In absence of film or for m = 0 intensity is maximum at 10. (c) We know that
screen. As the value of m is increased, intensity shall 2 dy 2 d tan
decrease and then increase alternately. Hence the I( ) I 0 cos 2 where =
2 D
correct variation is.
d tan 150 tan
I ( ) = I 0 cos 2 ÷ = I 0 cos
2
÷
3 108 /106
I0

I 0 cos 2 tan ÷
2

5. (c) Let I1 = I and I2 = 4I

I max = ( I1 + I 2 )2 = ( I + 4I )2 = (3 I )2 = 9I S1
y
d/2
I min ( I1 I2 ) (
2
I 4I ) 2
=I d/2 D
S2
664 IIT-JEE PHYSICS Challenger

3.d
For = 30°; I( ) = Io cos2 ÷ So, d
2 3 9 2
(a + b) 1–
For = 90° ; I( ) = Io cos2 ( ) (a + b)2
For = 0°
I( ) = I0 Here, = 400 × 10–9, d = 200 × 10–9.
I( ) is not constant. 1 10 –2
(a + b) = = 1 10 –5 m / line
tD 1000
11. (d) Shift of fringe pattern = ( 1)
d
3 200 10 –9
10 So, d 6°
30 D (4800 10 ) D
= (0.6)t –5 9 4 4 10 –14
d d 1 10 1–
1 10 –10
30 4800 10 10
= 0.6t 18. (a) Locus of equal path difference are lines running parallel
30 4800 10 10
1.44 10 5 to axis of the cylinder. Hence straight fringes will be
t= = = 24 10 6
observed.
0.6 0.6
19. (b) Constructive interference happens when
12. (c) n1 1 = n2 2 for bright fringe
n (7.5 × 10–5) = (n + 1) (5 × 10–5) 2t (m 1/ 2) . The minimum value for m = is m = 1,
the maximum value is the integer portion of
5.0 10 –5
n= 5 = 2. 2d 1 2 0.034 10 3 1
2.5 10 + = + = 100.5
2 680 10 9 2
13. (c) Path difference between the opposite edges is .
For a phase difference of 2 we get a path diff. of . mmax = 100
20. (a) Optical path difference between the waves
= (n3 – n2) t
14. (b) Angular width = = 10 (given) 3
d ( n3 n2 ) t ( n3 n2 ) t
No. of fringes within 0.12° will be Phase difference = 2 2
(Vacuum ) n1 1
0.12 2 21. (a) When slits are of equal width.
n [2.09]
360 10 3 Imax µ (a + a)2 ( = 4a2)
The number of bright spots will be three Imin µ (a – a)2 ( = 0)
15. (d) For diffraction pattern to be observed, the dimension When one slit's width is twice that of other
of slit should be comparable to the wave length of
I1 W1 a 2 W a2
light. = = = 2 b = 2a
16. (c) Given d = 1mm, = 6.5 × 10–7m, D = 1m. I 2 W2 b 2 2W b

2n 1 D Imax µ (a + 2a )2 = (5.8 a2)


For n th dark fringe yn= ÷
2 d Imin µ ( 2a – a)2 = (= 0.17 a2)
For third fringe, 22. (a) Suppose the mth bright fringe of 6500 Å coincides with
7 the nth bright fringe of 5200Aº.
2 3 1 1 6.5 10
y3 = ÷ = 1.625 mm. m 1D n 2 D
2 1 10 3 Xn = =
d d
d n
17. (d) We have, = m 6500 D n 5200 D
d (a b) cos =
d d
The angular separation for IIIrd order is given by
m 5200 4
3.d = =
d = n 6500 5
(a b) 1 – sin 2 m 1D
distance y is y = y = 0.156 cm.
We also have for IIIrd order d
(a + b) sin = 3 23. (b) When a mica sheet of thickness t and refractive index
3 is introduced in the path of one of the interfering beams
sin = in a biprism arrangement, the shift in interference pattern
( a + b)
is given by
WAVE OPTICS 665

where, 2 = 400 nm
W
y = ( 1)t From (i) and (ii), we get

According to question,
Given y = W x2
= 6×10–7m Segment of
t = 12 × 10–7 m screen
W x1
So, y´ = W = ( – 1)t

= ( – 1) t

= +1 O
t (Centre
of screen)
6 10 –7
= + 1 = 0.5 + 1 = 1.5
12 10 –7 12 1 D k 2 D
=
24. (b) I µ I1 + I2 2 I1 I 2 cos ...(1) d d
Applying eq. (1) when phase difference is /2 12 600
k= = 18
I /2 µ I + 4I I /2 µ 5I 400
Again applying eq. (1) when d phase difference is 28. (d) The path difference produced by a sheet
I µ I + 4I + 2 I 4 I cos = (µ – 1)t
According to the given condition (for minimum
µ thickness)
I /2 – I µ 4I
25. (b) Equation of path difference form maxima in transmission (µ – 1) t =
(or weak reflection) 2
3 6600 10 10
Popt = 2n2 L = vacuum
, vacuum
....... t= = = 5500 Å.
2 2 2(µ 1) 2(1.6 1)

n2 3 L
2 L , ,..... L= 29. (d) E1= I0 = , without mirror
n1 ÷ 2 2 4n2 d2
(notice that = wavelength in medium is related to s' d d s d
vacuum as, vacuum n1 )
( 1)tD D D L L
26. (c) n= but = and E2 = +
d d d d 2
(3d )2
n ( 1) t /
20 = ( – 1) 2.5 × 10–3 ( /5000 ×10–8) L1 10 I0
= 2
= 1+ with mirror..
8 d 9 9
20 5000 10
1= 3 = 1.4 30. (a) Path difference = (µ – 1) t = n ;
2.5 10 For minimum t, n = 1;
D t=2
27. (b) In Young's double slit experiment, fringe width =
d 31. (d) D/d
Given that, twelve fringes of wavelength 1 occupy a The position of the central maxima is independent of
segment of screen wavelength used.

1D
n D
x2 x1 = 12 ...(i) yn = yn
d d
Where 1 = 600 nm 32. (d) At the area of total darkness minima will occur for both
Also, k fringes of wavelength 2 occupy the same the wavelengths.
segment of screen.
(2n + 1) (2m + 1)
D 1 2
x2 x1 = k 2 2 2
...(ii)
d (2n + 1) 1 = ( 2m +1) 2
666 IIT-JEE PHYSICS Challenger
we are concentrating at a point where the intensity is
(2n + 1) 560 7
or = = or 10n = 14m + 2 one fourth of the maximum intensity.
(2m + 1) 400 5 I = I + I + 2I cos
by inspection for m = 2, n =3 and for m = 7, n = 10, the
1 2
distance between them will be the distance between cos =
such points. 2 3
We take the least value of the angle as the point is in
D 1 (2n2 1) (2n1 + 1) central maxima]
i.e., s=
d 2 For a phase difference of 2 , the path difference is
put n2 = 10, n1 = 3 2
For a phase difference of , the path difference is
On solving we get, s = 28mm. 3
33. (d) For maxima d sin n 2
=
n 8 4 4 2 3 3
sin = = sin = tan = But the path difference (in terms of P and Q) is dsin
d 10 5 3
as shown in figure.
y
Also tan =
D d sin =
3
4D
y=
3 sin =
3d
D
34. (a) Using relation, y0 = (µ 1) t
d = sin 1 ÷
We have, 3d
36. (a) µ1S1O – µ1S2O + µ1t – µ2t = 0
z (1.5 1)
= 2
=
1
3 (µ 1) 3 2 (µ 1)
z µ1
2
S1
1 4 3
= µ–1=
4 0
µ 1 3 t

7 S2 µ2
µ== 1.75
4
35. (c) Let P be the point on the central maxima whose intensity
is one fourth of the maximum intensity .
µ1 (S1O – S2O) = (µ2 – µ1) t
For interference we know that
I = I1 + I 2 + 2 I1 I 2 cos µ2
(S1O – S2O) = 1÷ t
µ1
where I is the intensity at P and I1, I2 are the intensities
of light originating from A and B respectively and is 37. (a) Let x = distance between lenses, D is distance between
the phase difference at P. the source and first screen. As images of the source
due to both pieces of lens are observed on the screen,
from displacement method.
P
x D (D 4 f ) 4.9(4.9 2.4) = 3.5m

A Screen Screen
O
d 36mm
L1 1mm
in
B ds 6mm
P
S
L2

In YDSE, I1 = I2 = I and Imax = 4I 4.9m 0.6m


Let m1 and m2 are the magnification by the lenses L1
WAVE OPTICS 667

and L2, Hence from eq. (1), we have


D+x 1 3
m1 6, m2 = 0.3°
D x 6 0.4° 4
Let S1 is image formed by L1 of S and h1 is height of S1 43. (a) As A and B are point sources,
from P'
L Cos Cos
h1 = m1h = 36mm I= = (as = 4 L)
2
Let S2 is image formed by L2 of S and h2 is height of S2 r 4 r2
from P'
h2 = m2h = 1.0 mm 90 cos 0 º 10 W
IA = =
Distance between S1 and S2 (d) 4 (3 ) 2 4 m2 ;
= h1 – h2 = 36.0 – 1.0 = 35.0 mm
Position of central maximum O from P is 180 cos 60 º 10 W
IB = =
2
35 4 (1.5) m2
6+1+ = 24.5mm.
2
38. (c) The reflections from the boundaries will cause a net
180º phase shift.
The condition for bright fringes is 2t = (m + ½) film
Now, m = 124 since there is a bright fringe for m = 0 and

film =
n
1 1
m film m ÷
2 2
t= =
2 2n
1 9 As source C gives a parallel beam of light,

124 (552 10 m)
= 5
3.44 10 m W
2 (1.00) IC = I0 cos = 20 cos 60º = 10
39. (c) The sky is blue because tiny particles in the air are m2
more efficient at scattering short wavelength light than
10 10
they are at scattering long wavelength light. I = I A + I B + IC = + + 10
40. (b) Photons per area per second at a distance r are 4
5.00 × 1018/4 r2. Photons per second entering the eye,
5+4 W
radius R is then this times R2. Set this product equal = 10 14
to 500 per second and solve for r. 4 m2
41. (a) There is air on both sides of the soap film. 44. (b) Equivalent optical path length
the reflections of the light produce a net 180º phase
1 1
shift. x2 8.2
The condition for bright fringes is
= dx = 1.2 + ÷ dx = = 1.367m
2 6
0 0
2t = ( m + ½ ) film
45. (a) n th minimum has a distance from the centre =
(m ½ ) film (m ½ ) 1 D
t= = x = (2n + 1)
2 2n
2 d
(½)(650 10 9 m ) For a point on the screen directly in front of one of the
= 1.2 10 7 m
2(1.41) slits, x = d/2
42. (a) Let be the angular width in water. We know angular for minimum intensity in front of one of the slits =
d D
width = = ( 2n + 1)
d 2 2 d
Angular width µ
d2
=
= w
...... (1) (2n + 1)D
0.4 a
D
4 46. (a) Fringe width . =
Now, a w a a
= d
w w 3 When the apparatus is immersed in a liquid, then will
668 IIT-JEE PHYSICS Challenger
decrease µ times and hence is reduced µ (refractive 51. (c) If S, l and be the specific rotation of sugar solution,
index) times. polarimeter tube length and optical rotation
10 ' = (5.5) respectively, then the strength of solution is given by

D D 10
or 10 ÷ = (5.5) or = = c=
l S
d d 5.5
Here, = 19º
or µ = 1.8
l = 20 cm = 0.20 m
47. (d) 350 nm < < 750 nm
S = 0.5 deg m2/kg
48. (d) Due to reflection virtual source will be formed at distance
D from mirror. 19
c= = 190 kgm–3
0.20 0.5
The effective distance of the screen = 2D + 2D = 4D
Thus, 200 kgm–3 impure sugar solution contains 190
4D kg-m–3 pure sugar.
Fringe width =
d
190
49. (c) Treating the anti-reflectance coating like a camera-lens Purity of sugar = 100 ÷ % = 95%.
200
coating, one can obtain for its thickness t:
52. (b) When a ray of light is incident on a plano convex lens
1 placed on a glass plate, it reflects at two surfaces. These
2t = m + ÷ , m = 0, ± 1, ± 2,.... ......... (1)
2 n two reflected rays interfere and produce dark and bright
rings which are known as Newton’s rings.
Condition (1) is the condition for the destructive
interference for the normally incident and reflected
electromagnetic wave.
The minimum thickness of the coat refers to m = 0. This
gives for the thickness: The ray reflected at the upper surface of the air-film
suffers no phase change but the ray reflected internally
3.00
t= = cm = 0.500 cm. at the lower surface suffers a phase change of 180º.
4n 4 (1.5)
53. (c) According to Malus’ law
This anti-reflectance coating could be easily countered I = I0 cos2
by changing the wavelength of the radar to 1.50 cm = I0 (cos2 60º)
now creating maximum reflection. 2
1
50. (d) The fringe-width ( ) is given by =I0 × ÷
2
D I0
= =.
d
4
When the screen is displaced by distance D then the
54. (c) The width of the diffraction band is given by
fringe-width is changed by.
D
( D) =
= d
d
µ
( )d
= µ D
D
Here, = 10–3 m 1
and µ .
d
d = 0.03 mm
55. (a) When unpolarised light beam is incident from air onto
= 3 × 10–5 m
glass at the polarising angle, reflected beam is
D = 5 × 10–2 m completely polarised.
10 3
3 10 5
56. (a) In the phenomenon of interference the light energy is
=
5 10 2 redistributed and the distribution does not vary with
= 6 × 10–7 m time.
= (6 × 10–7 × 1010) Å 57. (c) Longitudinal waves cannot be polarised.
= 6000 Å.
WAVE OPTICS 669

dy 6. (c) = 3 – 4 = 30°
1. (a) S1P – S2P =
D IR = I3 + I4 + 2 I3 I 4 cos( ) = (3 + 6) I0 .
dy 7. (a), 8. (c).
x = (n0 + kt) – d sin = 0 Total phase difference at C,
D
For central maxima. = kd sin – kt (µ' – 1) for centre maxima at C, =0
d sin 2 10 3
sin 30°
P t= 5 10 3
( 1) ( 1)
µ' = 1.2
y µ = 1.2 × (4/3) = 1.6
2 Hence refractive index of mica slab = 1.6
n = (n0 + kt)
A black line is formed at the position where dark fringe
are formed for both the wavelength.
d O
The distance of first black line from centre bright line
S1 (2n 1) D
y= ........... (1)
2d
D (2n1 1) 1D (2n2 1) 2D
For black line, =
2d 2d
D sin (2n1 1)
y= ( y -coordinates of central maximum). = 2
, where 1
1
and 2 = 2
n0 + kt (2n2 1) 1 w w
dy – kD sin
2. (b) = = velocity of central maximum. (2n1 1) 7
dt (n0 + kt )2 =
(2n2 1) 5
3. (c) For central maxima to be formed at O
For minimum value, n1 = 4 and n2 = 3
n Hence distance of first black line
n' –1÷ b d sin
n' 10 2
(2 4 1) 4000 10 40 10 3
Here n' = n0 + kt, n = refractive index of plate. y= 3
2 2 10 4
d sin
n = n0 + kt + = 2.1 × 10–4 m = 210 µm
b
4. (a) For screen 1 9. (a), 10. (c), 11. (d).
Velocity and acceleration of central maximum = velocity
y and acceleration of screen
x = d sin = d ÷
D [ it does not move to the left or right on the screen]
2 vscreen = 0 + gt = 20 m/s
= ( x) vs = 20 ˆj m/s (in vector form) and as = 10 ˆj m/s 2
1
D = D0 + gt 2 [where D in distance of screen at a
I = 4I0 cos2 ÷ 2
2 time t along y-axis]
Solving Position vector of 2nd maximum is given by
For S3

2 rx = ± 2 i + Djˆ
3 = and I3 = I0 d
3 Position vector of 1st minimum is given by
For S4 1 Dˆ
rn = ± i + Djˆ
2 d
4 = and I4 = 2I0 Relative position vector of 2nd maximum with respect
2
I3 1 3 Dˆ
= . to 1st minimum is given by r = ± i
I4 2 2 d
Differentiating with respect to time,
Imax I3 + I 4 (1 + 2)2 d 2r 3 gˆ
5. (b) = ÷= 2 a= =± i = ± 7.5 10 3 m / s 2 iˆ
Imin I3 I4 (1 2) 2 2 d
dt
670 IIT-JEE PHYSICS Challenger

12. (b) n air < n film < n glass . Reflection with phase change Either ray a or ray b will undergo a phase change during
of /2 occurs for ray a at the air-film interface and for reflection. Therefore the two rays will be out of phase.
ray b at the film-glass interface. Therefore, reflections
alone keep both rays in phase. For constructive interference to occur, the optical path
difference must provide a 180º phase change for ray b.
This happens if 2t is an odd multiple of /2. Since 2t =
/2, t = /4.
15. (b) Choices (a) and (c) are eliminated immediately since they
are outside the visible range. For soap nair > nsoap. The
reflected wave, raya undergoes a phase change at the
air-soapy water interface. Ray Rb doesn’t change phase
at the soapy water-air interface. Based on interface
Constructive interference then depends on making the reflections, the two rays are out of phase. To maintain
path length difference, 2t, within the film a multiple of this, the optical path difference, 2t, must not produce a
. phase change in Rb. Therefore the path must be an integer
2t = m / n t = m(600 nm) / 2(1.50) = m 200 nm multiple of .
For m = 1,t = 200 nm 2t = m / n
13. (c) The ratio of the refractive indices is
n air < n film < n glass . Reflections at the interfaces = 2tn / m = 2(350 nm)(1.35) / m = 945 nm / m

do not produce a net phase difference between rays a For m = 1, = 945 nm


and b. (See diagram from Answer above.) Destructive For m = 2, = 473 nm [this is choice (b)]
interference requires that the optical path length For m = 3, = 315nm
through the film, 2t, be an odd multiple of /2: Choice (b), = 473 nm, is the only choice in the visible
2t = (m + 1/ 2) / n range.
16. (d) Again, ray Ra is reflected with phase change and ray Rb
t = (m + 1/ 2)(640nm) / 2(1.33)
isn’t. For the two reflected waves to interact:
t = (m + 1/ 2)240 nm 2t = M /n where M is either m or m + 1/2
For m = 0, t = 120 nm which is not one of the choices. M = 2tn/ = 2(1.00 × 10–6 m)(1.35)/600 x 10–9 nm = 4.5
For m = 1,t = 360 nm. M = m + 1/2 for m = 4. This describes the complete
14. (a) It isn’t necessary to know if
constructive interaction.
n air < n film < n glass 17. (d) Both rays Ra and Rb are reflected with a change of
phase. Therefore the net change of phase with
air reflection is zero. For constructive interference, 2t =
m /n and for destructive interference 2t = (m + 1/2) /n.
t = (m + 1/2)(600 nm)/2(1.25) = m700/2 (1.25)
600m + 300 = 700m, 300 = 100 m, m = 3
soap
Solve either equation for t :
t = 3(700 nm)/2( 1.25) = 840 nm
air

1. (d) Filling liquid will increase the optical path length by the refractive index of the first medium is more than the
same amount. second medium, there is no reversal of phase in reflected
CBF will not shift. ray so, central fringe remains bright.
2. (c) When x = 0 5. (a) Corpuscular theory fails to explain the velocityof light
for any the interference will be constructive. in air and water because it predicted light to have more
3. (b) Radio waves are electromagnetic so it is transverse in velocity in denser medium where as the fact is just the
nature. It can be polarised. Sound wave is longitudinal 6. (c) In Young’s experiments, fringe width of dark and white
in nature which can never be polarised. These two fringes are equal. If white light is used as source,
statements are uncorrelated. coloured fringes are observed representing bright
4. (a) Newton’s rings are formed in reflected system and if band of different colours.
WAVE OPTICS 671

1. (a, b, c, d)
b2
For maxima d = n . = when n = 1, 2
(2n 1)d
For minima d = (n + 1/2)
3 1 b2 b2
For intensity th of maximum d = n ± ÷ = , ,...
4 3 2 d 3d

( )2 = 9
5. (b) I ( ) = I1 + I2 + 2 I1I2 cos
I I1 + I 2
2. (b, d) max = Here, I1 = 1 and I2 = 4l
I min
( I1 I2 )
2 1
At point A, = /2
IA = I + 4I = 5I
I1 + I 2 3 At point B, = /2
= 4 I 2 = 2 I1
I1 I2 1 IB = I + 4I – 4I = I
IA – IB = 4I
I1 4 I1 a2
= =4= 2 D D
I2 2 I2 b 6. (a, b) As = =
d d
a If d' = 2d, we get
=2
b D
=
3. (b) The fringe width does not depend on the angle 2d
made by beam, for small angles. To keep ' and equal either is to be made
D d double or D is to made double.
4. (a, c)Here y (2n 1) (2n 1) 7. (a) Path difference between first and 11th bright fringe
2 d 2b
( d = b and D = d) = S1B (10 bright fringes) = 10 = 10 × (6000 × 10–7)m =
6 × 10–6 m
b 8. (a, b) If two waves have a constant phase difference
But y =
2 and have same wavelengths, then they produce
coherence.
b d
(2n 1)
2 2b

1. A-q, r, s; B-p; C-s; D-r (B) For maxima, x = n


2. A-s; B-r; C-q; D-p cos = n,
3. A-p, q; B-r, s; C-s, t; D-p Possible values of n = 0, 1
cos = 0, = 90°, 270° S1 S2
(A) D2 (2 )2 D x cos = 1, = 0°, 360°
For maximas x = n Number of maximas = 4
D2 + (2 )2 = (D + n )2
Similarly for minimas, x = (2n – 1)
4 2 n 2 2 2 Dn 2
Only two possible values of n, n = 1, (C) Virtual image of S will act as another source
x = d sin , d = 2
3 ; n = 2, D = 0
D= n
2 For maximas, n = 2 sin sin =
2
Similarly, for minimas, x (2n 1) n = 0, 1, 2,
2 = 0, 30°, 90°, 150°
672 IIT-JEE PHYSICS Challenger
Total maximas possible = 7 (centre + 3 up + 3 down)
2n 1
(D) x = 2 cos ; 60° cos =
4
n
For maximas, x = n cos = ; n = 0, 1, 2, 90°, 1
2 n = 1, cos ; > 60° ; n = 2,
4
= 60°, = 0°
3
Total maximas two, For minima, x = (2n 1) ; cos ; < 60° ; No. of possible minimas = 1.
2 4
4. A-q; B-s; C-r; D-p

1. 2 3. 0.06
The phase difference between two waves arriving at a 2
common point consists of two parts 2
I min I 0.36 I 0.4 1
(i) initial phase difference 0 = ÷ = ÷ = = 0.06
I max I + 0.36 I 1.6 16
(ii) phase difference ( ') due to path difference
2 [ If intensity of light falling on P directly from S is I, then
= p where p = path difference the intensity of light falling at P after reflection from AB is
2 0.36 I ]
= 0+ '= 0 + p 4. 4330
Let the central maxima is obtained at a distance x below O.
A [This is because a glass sheet is present in front of S2 which
D increases its path length to the screen. Therefore the path
P length of ray from S1 to the screen should also increase].
x
S1
X Q
O
S1
S2
40cm d/2
O x
P
d/2
S 10cm S2
2m B C t

Here S to P, since the source is same, 0= 0


2
= p . If p = 0 then = 0 then there will be central For central maxima
maxima. S1P = S2P
40 1 2 1/ 2 2 1/ 2 µg
d d
In OXS, tan = = ... (i) D +
2
+x = D +
2
= x÷ + 1÷ t
200 5 2 2 µm
x
In POQ, tan = ... (ii) µg
10 where 1÷ t = path ifference due to glass sheet.
µm
1 x
From (i) and (ii), =
5 10
x = 2 cm 2 1/ 2 2 1/ 2
d d
x x
2. 3.5 ÷ µg
At the place where maxima for both the wavelengths D 1+ 2 = D 1+ 2 + 1÷ t
D D µm
coincide, y will be same for both the maxima, i.e., ÷

n1 1 D n2 2 D n1 700 7
= = 2 = = d
2 1/ 2
d
2 1/ 2
d d n2 500 5 x x
1
2 ÷ µg
Minimum distance of maxima of the two wavelengths D 1 D 1+ 2 = 1÷ t
D D µm
from central fringe is ÷
= 5 × 700 × 10–9 × 103 = 3.5 mm.
WAVE OPTICS 673

d2 d2 1
+ x 2 + xd + x2 xd 1
1
1 4 1 4
D 1 1 4 y= = = 0.26 m
2
2 D 2 D2 1
2 15
1 ÷
4
µg 3
= 1÷ t m = – 1, sin
µm 4

xd µg 3
1÷ t 1 ÷ 3
D µm 4
y= = = – 1.13 m
3
2 7
1 ÷
µg D 1.5 (10.4 10 6 ) (1.5) 4
x= 1÷ t 1÷ × 3
µm d 4/3 0.45 10 3 m = + 2, sin =
4
= 4.33 × 10–3 m = 4330 µm y = + 1.13 m
5. 2.78
The path difference from the ray starting from S1 and S2 and 1
m = 0, sin y = – 0.26 m
reaching a point P will be 4
x = S2P – S1P S2M Required sum = | 0.26 | + | –1.13| + | 1.13 | + |–0.26 | = 2.78
From S1S2M 6. 7
For Red Light
S2 M
sin = S2M = d sin
S1S2 Dt (µ 1)
The shifts of fringes due to glass plate =
d
x = d sin P where t is the thickness of the plate.
This shift is equal to 5 where is the fringe width
y
Dt (µ 1) Dt (µ 1) 5 R D
S1 =5 =
d d d

d Q 5 5 7 10 7
M O
t= = = 7× 10–6 m = 7µm
(µ 1) 1.5 1
S2
7. 9.3
The time taken by the ray to reach P' from S1
D
(=1m)
d air d plate S1P ' t t S1 P ' t + tµ1
= + = + =
We know that the path difference for minimum intensity is Vair v plate c c / µ1 c

(2m – 1) where m = 1, 2, 3... Effective path travelled = S1P' – t + tµ1


2 where c is the speed of light in air.
P'
d sin = (2m – 1)
2
t x
(2m 1) (2m 1)0.5 2m 1
sin = = = S1 µ1
2d 2 1.0 4
Also 1 sin d P
1 . Therefore possible values of m are ± 1,
+ 2, 0
S2 µ2
From POQ
D sin
y = D tan = ... (i) D
2
1 sin Similarly, the time taken by the ray to reach P' from S2
Positions of minima
1 S2 P ' t + tµ2
For m = + 1, sin = =
c
4
674 IIT-JEE PHYSICS Challenger
Effective path travelled = S2P' – t + tµ2 8. 7
Path difference = S2P' – t + tµ2 – S1P' + t – tµ1 The power transmitted through
Also when there were no plates infront of the slits.
10
xd A = 10% of ÷ (0.001) 2
= S2P' – S1P' =
D
xd 10 10
S2P' – S1P' =
D = (0.001)2 = 10–6 W
100
xd
Path difference = + t (µ2 – µ1) The power transmitted through
D
For the point P, x = 0 10
B = 10% of ÷ (0.002) 2
Path difference = t (µ2 – µ1) = t (1.7 – 1.4) = 0.3 t... (i)
But the point P lies between the 5th maximum and 6th minimum
(given). 10 10
= (0.002)2 = 4 × 10–6 W
Therefore the path difference = 5 + D ...(ii) 100
Equating equations (i) and (ii), we get Let be the phase difference introduced by film
0.3t = 5 + ...(iii) 2
The path difference can be determined from the given = (path difference introduced by the film)
I 3
intensity at P, which is = . 2 2
I0 4 = (µ 1) t = 10 [1.5 –1]×2000×10–10
The expression I / I0 in terms of is 6000 10

I
= cos 2 ÷ =
3
radian
I0
The power received at F
3
For I / I0 = 3/4, we get cos ÷= P = P1 + P2 + 2 P1 P2 cos
2

= 10–6 + 4 × 10–6 + 2 10 6 4 10 6
cos
or = or = 3
6 6
= 7 × 10–6 W = 7 µW
Hence, the thickness of the glass plates (Eq. 3) is
9. 13.9
0.3 t = 5 + /6
The total intensity at point P will be
1 31 1 31 = IA + IB + IC
or t= ÷ = 0.3 54000Å÷
0.3 6 6 (Illuminating power) cos 90 cos 0
IA = =
= 9.3 × 104 Å = 9.3 × 10–6 m = 9.3 µm 4 r 2
4 32
Alternatively :
10
x = (µ2 – µ1) t = (1.7 – 1.4) t = 0.3 t = watt / m 2
Given that 5th maxima lies below P and 6th minima lies above 4
P, therefore the path difference should lie between 5 and C
(20W/m2)
5 + /2. Let Dx = 5 + where < /2.
2 2
The phase difference = x (5 ) 60O
2 P
A (90W/m ) O
60
We know that I ( ) = Imax cos2 ÷
2

3 B(180W/m2)
I max = I max cos 2 = 30° =
4 2 2 6 180 cos 60° 10
IB = 2
= watt / m 2
2 2 4 (1.5)
(5 ) x = = 0.3 t
6 6 IC = 20 cos 60° = 10
–6
t = 9.3 × 10 m = 9.3 µm 10 10
Ip = + + 10 = 13.9W/m 2
4
1. The ratio of frequencies of the shortest wave lengths of 7. Find the number of photon emitted per second by a 25 watt
Balmer and Lyman series of hydrogen atom is source of monochromatic light of wavelength 6600 Å. What
(a) 4 : 1 (b) 1 : 4 is the photoelectric current assuming 3% efficiency for
(c) 27 : 5 (d) 5 : 27 photoelectric effect ?
2. The first member of Balmer series of hydrogen has a 25 25
wavelength of 6563 Å the wavelength of its second member (a) 1019 J, 0.4amp (b) 1019 J, 6.2 amp
3 4
will be
25
(a) 4861 Å (b) 6563 Å (c) 1019 J, 0.8amp (d) none of these
2
(c) 3561 Å (d) 1215 Å 8. Electrons are bombarded to excite hydrogen atoms and six
3. An atom emits a photon of wavelength = 600 nm by spectral lines are observed. If Eg is the ground state energy
transition from an excited state of life time 8 × 10–9 s. If v of hydrogen, the minimum energy the bombarding electrons
represents the minimum uncertainty in the frequency of the should posses is
v
photon, the fractional width of the spectral line is of 8Eg 15 Eg
v (a) (b)
the order of 9 16
(a) 10–4 (b) 10–6
–8 35Eg 48Eg
(c) 10 (d) 10–10 (c) (d)
4. The half life of radioactive Radon is 3.8 days. The time at the 36 49
9. In Bohr theory of hydrogen atom, let r, v and E be the radius
1
end of which th of the radon sample will remain of orbit, speed of electron and the total energy of the electron
20 respectively. Which of the following quantities is
undecayed is (given log10 e = 0.4343 ) proportional to the quantum number n?
(a) 3.8 days (b) 16.5 days (a) vr (b) r E
(c) 33 days (d) 76 days. (c) r/E (d) r/v
5. The wavelength of Ka X-rays produced by an X-ray tube is 10. If the nuclear radius of 27Al is 3.6 Fermi, the approximate
0.76 Å. Find the atomic number of the anode material of the nuclear radius of 64Cu in Fermi is
tube ?
(a) 4.8 (b) 3.6
(a) 40 (b) 30
(c) 2.4 (d) 1.2
(c) 20 (d) 10
6. The element which has a K x-rays line of wavelength 1.8 Å 11. If the wavelength of the first line of the Balmer series in the
hydrogen spectrum is , then the wavelength of the first
is (R = 1.1 × 10–7 m–1, b = 1 and 5 / 33 = 0.39 ) line of the Lyman series is
(a) Co, Z = 27 (b) Iron, Z = 26 (a) (27/5) (b) (5/27)
(c) Mn, Z = 25 (d) Ni, Z = 28 (c) (32/27) (d) (27/32)

1. 2. 3. 4. 5.
MARK YOUR
6. 7. 8. 9. 10.
RESPONSE
11.
676 IIT-JEE PHYSICS Challenger
12. The difference between the longest wavelength line of the
Balmer series and shortest wavelength line of the Lyman R R
series for a hydrogenic atom (atomic number Z) equal to
. The value of the Rydberg constant for the given atom is :
(c) (d)
5 1 5 Z2
(a) (b) t
31 .Z 2 36 t

31 1 19. Calculate binding energy of 92U238.


(c) (d) none of these
5 .Z 2 Given M (U238) = 238.050783 amu, mn = 1.008665 amu and
13. A radioactive material decays by simultaneous emission of mP = 1.007825 amu
two particles with respective half-lives 1620 and 810 years. (a) 801.7 MeV (b) 18.7 MeV
The time, in years, after which one-fourth of the material (c) 0.7 MeV (d) 1801.7 MeV
remains is) 20. At radioactive equilibrium, the ratio between the atoms of
(a) 1080 (b) 2430 two radioactive elements (X) and (Y) was found to be
(c) 3240 (d) 4860
3.2 × 109 : 1 respectively. If half-life of the element (X) is
14. The energy difference between the first two levels of
hydrogen atom is 10.2 eV for another element of atomic 1.6 × 1010 years, then half-life of the element (Y) would be
number 10 and mass number 20, this will be (a) 3.2 × 109 years (b) 5 × 109 years
(a) 2040 eV (b) 0.201 eV (c) 1.6 × 1010 years (d) 5 years
(c) 510 eV (d) 1020 eV 21. A stationary Pb200
nucleus emits an alpha -particle with
15. Light of wavelength 200 Å fall on aluminium surface. Work kinetic energy T . The fraction of recoil energy of the
function of aluminium is 4.2 eV. What is the kinetic energy daughter nucleus to the total energy liberated is :
of the fastest emitted photoelectrons? (a) 1/196 (b) 4/196
(a) 2 eV (b) 1 eV
(c) 1/20 (d) 1/50
(c) 4 eV (d) 0.2 eV
16. In an a-decay the kinetic energy of a-particle is 48 MeV and 22. Half life of a radioactive substance is 20 minute. Difference
Q-value of the reaction is 50 MeV. The mass number of the between points of time when it is 33% disintegrated and
mother nucleus is (Assume that daughter nucleus is in 67% disintegrated is approximately
ground state) (a) 40 minute (b) 10 minute
(a) 96 (b) 100 (c) 15 minute (d) 20 minute
(c) 104 (d) 110 23. In the photoelectric experiment, if we use a monochromatic
17. The equation light, the I-V curve is as shown. If work function of the metal
4 11 H + 4
2 He
2+
+ 2e – + 26 MeV represents is 2eV, estimate the power of light used. (Assume efficiency
of photo emission = 10–3%, i.e., number of photoelectrons
(a) -decay (b) -decay emitted are 10–3% of number of photons incident on metal)
(c) fusion (d) fission
18. A radioactive nucleus X decays to a stable nucleus ‘Y’. I
Then the graph of rate of formation of ‘Y’ against time ‘t’ will 10µA
be

R R

–5 volt V
(a) (b)
(a) 2W (b) 5W
t t
(c) 7W (d) 10W

12. 13. 14. 15. 16.


MARK YOUR
17. 18. 19. 20. 21.
RESPONSE
22. 23.
MODERN PHYSICS 677

24. The ionization energy of a hydrogen-like Bohr atom is 4 31. A star initially has 1040 deutrons. It produces energy via the
Rydbergs. Find the wavelength of radiation emitted when processes 1 H 2 +1 H 2 H3 + p and
1
the electron jumps from the first excited state to the ground
2
state: 1H +1 H3 4
2 He + n . If the average power radiated by
[1 Rydberg = 2.2 × 10–18 joule the star is 10 16 W, the
deuteron supply of the star is
(h = 6.6 × 10–34 Js, c = 3 × 108 m/s.) exhausted in a time of the order of
Bohr radius of hydrogen atom = 5 × 10–11m] (The masses of nuclei are:
(a) 400 Å (b) 300 Å m(H 2 ) = 2.014amu, m(p) = 1.007amu ,
(c) 500 Å (d) 600 Å
m(n) = 1.0084amu, m(He 4 ) = 4.001amu )
25. Suppose potential energy between electron and proton at
separation r is given by U = K ln (r), where K is a constant. (a) 106 s (b) 108 s
(c) 10 s12 (d) 1016 s
For such a hypothetical hydrogen atom, the ratio of energy
difference between energy levels (n = 1 and n = 2) and (n = 32. A radioactive source in the form of metal sphere of diameter
2 and n = 4) is 10–3 m emits beta particle at a constant rate of 6.25 × 1010
(a) 1 (b) 2 particles per second. If the source is electrically insulated,
(c) 3 (d) 4 how long will it take for its potential to rise by 1.0 volt,
assuming that 80% of the emitted beta particles escape from
26. Consider particles, particles and - rays, each having
the source?
an energy of 0.5 MeV. In increasing order of penetrating
powers, the radiations are: (a) 6.95 µ sec (b) 0.95 µ sec
(c) 1.95 µ sec (d) 2.15 µ sec
(a) , , (b) , , 33. There are two radioactive substances A and B. Decay
(c) , , (d) , , constant of B is two times that of A. Initially, both have
equal number of nuclei. After n half lives of A, rate of
27. A hydrogen atom is in an excited state of principal quantum
disintegration of both are equal. The value of n is
number (n), it emits a photon of wavelength ( ), when it
(a) 4 (b) 2
returns to the ground state. The value of n is
(c) 1 (d) 5
R ( R 1) 2
(a) (b) 34. The binding energy of deuteron ( 1 H ) is 1.15 MeV per
R 1 R
4
R nucleon and an alpha particle ( 2He ) has a binding energy
(c) ( R 1) (d)
R 1 of 7.1 MeV per nucleon. Then in the reaction
28. An electron in a hydrogen atom makes a transition from n = 2 2 2
n1 to n = n2. The time period of electron in the initial state is 1H 1
H 2 He + Q
eight times that in the final state. Then which of the following the energy released Q is :
statement is true ? (a) 5.95 MeV (b) 26.1 MeV
(a) n1 = 3n2 (b) n1 = 4n2 (c) 23.8 MeV (d) 289.4 MeV
(c) n1 = 2n2 (d) n1 = 5n2 35. An energy of 24.6 eV is required to remove one of the
29. A neutron travelling with a velocity v and kinetic energy E electrons from a neutral helium atom. The energy (in eV)
has a perfectly elastic head-on collision with a nucleus of required to remove both the electrons from a neutral helium
an atom of mass number A at rest. The fraction of total energy atom is
retained by the neutron is approximately (a) 38.2 (b) 49.2
(a) [(A – 1)/(A + 1)]2 (b) [(A + 1)/(A – 1)]2 (c) 51.8 (d) 79.0
(c) [(A – 1)/A]2 (d) [(A + 1)/A]2 36. A horizontal beam of thermal neutrons with v = 2.2 ×
30. Determine the power output of a 92U235 reactor if it takes 30 10 3 meters/sec is directed to hit a target 1.1 meter away.
days to use 2kg of fuel. Energy released per fission is 200 If gravity is the only force, the beam would miss the
MeV and N = 6.023 × 1026 per kilomole. target approximately by :
(a) 63.28 MW (b) 3.28 MW (a) 1.01 × 10– 6 meter (b) 1.25 × 10– 6 meter
– 6
(c) 1.1 × 10 meter (d) 4.4 × 10– 6 meter
(c) 0.6 MW (d) 50.12 MW

24. 25. 26. 27. 28.


MARK YOUR
29. 30. 31. 32. 33.
RESPONSE
34. 35. 36.
678 IIT-JEE PHYSICS Challenger
37. X-rays are produced in an X-ray tube operating at a given 43. An particle passes rapidly through the exact centre of a
accelerating voltage. The wavelength of the continuous hydrogen molecule, moving on a line perpendicular to the
X-rays has values from internuclear axis. The distance between the nuclei is b.
(a) 0 to Where on its path does the particle experience the greatest
(b) min to where min >0 force? (Assume that the nuclei do not move much during
(c) 0 to where < the passage of the particle. Also neglect the electric field
max max
of the electrons in the molecule.)
(d) min to max where 0 < min < max <
38. The electric field of a light wave at a point is E = (100 N/c) +e
sin [(3 × 1015 s–1)t] sin [(6 × 1015s–1)t] where t is time in
seconds. This light falls on a metal surface having work
+2e b
function of 2 eV, then maximum possible kinetic energy of
photoelectrons is about -particle
+e
(a) 16 eV (b) 7 eV
(c) 6 eV (d) 4 eV H2 molecule
39. In the hydrogen atom spectrum 3–1 and 2–1 represent
wavelengths emitted due to transition from second and first b b
(a) (b)
excited states to the ground state respectively. The value of 2 2 2
3–1
is b
2–1 (c) (d) none
(a) 27/32 (b) 32/27 2
44. At t = 0, light of intensity 1012 photons/s–m2 of energy 6eV
(c) 4/9 (d) 9/4
per photon start falling on a plate with work function 2.5 eV.
40. A light of wavelength and intensity I is incident normally
If area of the plate is 2 ×10–4 m2 and for every 105 photons
on the surface. If reflection coefficient of surface is r, the
one photoelectron is emitted, charge on the plate at t = 25s
pressure exerted by light on the surface is equal to
is
2I I (a) 8 × 10 –15 C (b) 4 × 10 –14 C
(a) (b) r
c c (c) 12 × 10 –14 C (d) 16 × 10 –14 C
I I 45. When an electron in a hydrogen atom makes a transition
(c) (1 + r ) (d) (1 – r ) from 2nd excited stated to ground state it emits a photon of
c c
41. A regular hexagonal lamina of side a made up of perfectly frequency f. The frequency of photon emitted when an
absorbing material is kept in a region where a parallel beam electron of Li++ makes a transition from Ist excited state to
of light with intensity I having a large aperture falls on it. If ground state is
the area normal of the hexagon makes an angle of 30° with
243 141
the beam then the force experienced by the hexagon will be (a) f (b) f
32 32
5a 2 I 9a 2 I
(a) (b)
4c 4c 81 63
(b) f (d) f
32 32
a2 I 6a 2 I
(c) (d) 46. The activity of a radioactive sample is A1 at time t1 and A2 at
c c
time t2. If is average life of sample then the number of
42. The transition from the state n = 4 to n = 3 in a hydrogen-like nuclei decayed in time (t2 – t1) is
atom results in ultraviolet radiation. Infrared radiation will
be obtained in the transition ( A1 A2 )
(a) A1t1 – A2t2 (b)
(a) 2 1 (b) 3 2 2
(c) 4 2 (d) 5 4 (c) (A1 – A2) (t2 – t1) (d) (A1 – A2) .

MARK YOUR 37. 38. 39. 40. 41.


RESPONSE 42. 43. 44. 45. 46.
MODERN PHYSICS 679

47. The electric potential between a proton and an electron is 54. To decrease the cut-off wavelength of continuous X-rays
by 25%, the potential difference across X-ray tube
r
given by V = V0 ln , where r0 is a constant. Assuming
r0 100
(a) must be increased by %
Bohr’s model to be applicable, write variation of rn with n, n 3
being the principal quantum number ?
100
(a) rn µ n (b) rn µ 1/n (b) must be decreased by %
3
(c) rn µ n2 (d) rn µ 1/n2
48. Average life of a radioactive sample is 4 ms. Initially the (c) must be increased by 25%
total number of nuclei is N0. A charged capacitor of capacity (d) must be decreased by 25%
20 µF is connected across a resistor R. The value of R such 55. Electromagnetic radiation falls on a metallic body whose
that ratio of number of nuclei remaining to charge on work function is 2eV. For a particular radiation of frequency
capacitor remains constant with time is v, the maximum kinetic energy of the photoelectron is found
(a) 100 (b) 200 to be 4 eV. What would be the maximum kinetic energy of
(c) 300 (d) 400
49. If nuclei of a radioactive element is produced at constant 5v
photoelectron for the radiation of frequency ?
rate and they decays with decay constant . At t = 0, 3
number of nuclei is zero than the number of nuclei at time t
is 8
(a) eV (b) 8 eV
3
t t
(a) (1 e ) (b) e
10 20
(c) eV (d) eV
3 3
t t
(c) e (d) (1 e ) 56. A nucleus with mass number 220 initially at rest emits an
-particle. If the Q value of the reaction is 5.5 MeV, calculate
50. The kinetic energy of a non-relativistic electron increases
by 2% then the de-Broglie wavelength the kinetic energy of the -particle
(a) increases by 1% (b) decreases by 2% (a) 4.4 MeV (b) 5.4 MeV
(c) decreases by 1% (d) increases by 2% (c) 5.6 MeV (d) 6.5 MeV
57. A sample of hydrogen gas is excited by means of a
1
51. A radioactive element decays to th of its initial value in monochromatic radiation. In the subsequent emission
4
spectrum, 10 different wavelengths are obtained, all of which
time t. The ratio of its half life to mean life is
have energies greater than or equal to the energy of the
ln 2 ln 2 absorbed radiation. It follows that the initial quantum number
(a) 2 (b) of the state (before absorbing radiation) was
(c) ln 2 (d) ln 2 (a) 5 (b) 4
52. A sample of radioactive substance has 106 nuclei. If half life (c) 3 (d) 2.
is 20 seconds, the number of nuclei left in the sample after 58. The ratio of the min in a Coolidge tube to deBroglie of the
10 second is electrons striking the target depends on accelerating
(a) 104 (b) 2 × 105 potential V as
(c) 7 × 10 5 (d) 11 × 105
53. If the series limit wavelength of Lyman series for the (a) min
µ V (b) min
µV
hydrogen atom is 912 Å, then the series limit wavelength for de Broglie de Broglie
Balmer series of hydrogen atoms is
(a) 912 Å (b) 912 × 2 Å 1 1
(c) min
µ (d) min
µ .
912 de Broglie V de Broglie V
(c) 912 × 4 Å (d) Å
2

47. 48. 49. 50. 51.


MARK YOUR
52. 53. 54. 55. 56.
RESPONSE
57. 58.
680 IIT-JEE PHYSICS Challenger
59. Suppose that a material emits X–rays of wavelengths 63. Given that mass of proton is 1.00813 amu, mass of neutrons
, , is 1.00894 amu and mass of -particle is 4.00388 amu, the
K K L , when it is excited by fast moving electrons;
binding energy of -particle is
the wavelengths corresponding to K , K , L X– rays of (a) 28.172 MeV (b) 27.172 MeV
the material respectively. Then we can write (c) 13.52 MeV (d) 56.321 MeV
64. If 1, 2 and 3 are the wavelengths of K X-rays emitted by
(a) K K L 112Sn, 114Sn and 116Sn tin isotopes, then
(a) 1 = 2 = 3 (b) 1 > 2 > 3
(b) K K L
1 1 2
(c) 1< 2< 3 (d) + =
1 3 2
1 1 1 65. An element A decays into an element C by a two step
(c) = +
K K L process: A B + He24 and B C + 2e0 1 . Then
(a) A and C are isotopes (b) A and C are isobars
1 1 1
= + (c) B and C are isotopes (d) A and B are isobars
(d) 66. The diagram shows the energy levels for an electron in a
K K L
certain atom. Which transition shown represents the
60. A proton has kinetic energy E = 100 keV which is equal to emission of a photon with the most energy ?
that of a photon. The wavelength of photon is 2 and that n=4
of proton is 1. The ration of 2/ 1 is proportional to n=3
(a) E 2 (b) E1/2
(c) E –1 (d) E–1/2
n=2
61. A photon of frequency f causes the emission of a
photoelectron of maximum kinetic energy Ek from a metal. If
a photon of frequency 3f is incident on the same metal, the
maximum kinetic energy of the emitted photoelectron n=1
I II IIIIV
(a) equals 3Ek (a) I (b) II
(b) is greater than 3Ek (c) III (d) IV
(c) is less than 3Ek 67. In a certain hypothetical radioactive decay process, species
(d) may be equal to, less than or, greater than 3Ek A decays into species B and species B decays into species
C according to the reactions :
62. A proton, within a nucleus decays into a neutron during a
A 2B + particles + energy
process called + decay which occurs in some unstable
B 3C + particles + energy
nuclei. This process is represented by the equation
The decay constant for species is 1 = 1 sec–1 and that for
A A + the species B is 2 = 100 sec–1. Initially 104 moles of species
ZX Z 1Y +e + e
of A were present while there was none of B and C. It was
A
( )
Let m Z X represent the mass of an atom of A
Z X , and c
found that species B reaches its maximum number at a time
t0 = 2 ln (10) sec. Calculate the value of maximum number of
represent the velocity of light in vacuum. Assume that moles of B.
e is
massless. The Q value of the process is given by (a) 1 (b) 2
(c) 5 (d) 4
(a) m ZX( ) m(
A A
Z 1Y ) 2m
e
+ c
2 68. Electrons with de-Broglie wavelength fall on the target in
an X-ray tube. The cut-off wavelength of the emitted

( ) m( )c
A A 2
X-rays is
(b) m ZX Z 1Y
2
2mc 2h
= =
m( X ) m( Y) m
A A 2
(a) 0 (b) 0
(c) c h mc
Z Z 1 e+

m( X ) m( Y)+ m
A A 2 2m 2 c 2 3
(d) Z Z 1 + c . (c) 0 = (d) 0
e h2

MARK YOUR 59. 60. 61. 62. 63.


RESPONSE 64. 65. 66. 67. 68.
MODERN PHYSICS 681

69. A hydrogen atom and a doubly ionized lithium atom are 75. Which of these nuclear reactions is possible
both in the second excited state. If LH and LLi respectively
10 Ne + 2 He
(a) 23 1 20 4
represent their electronic angular momenta and EH and ELi 11 Na 1H
their energies, then
7 N + 1H
(b) 10 4 13 1
(a) LH > LLi and | EH | > | ELi | 5 B 2 He
(b) LH = LLi and | EH | < | ELi |
(c 10 1 11
(c) LH = LLi and | EH | > | ELi | 5 B 0n 5 B
(d) LH < LLi and | EH | < | ELi |
(d) 14 1 12
70. An -particle having a de-Broglie wavelength i collides 7 N 1H 6 C
with a stationary carbon nucleus. The -particle moves off
235 , one needs
76. For fission of uranium 92
in a different direction as shown below. U
initial direction of - particle (a) very fast neutrons (a few electrons volts)
de Broglie wavelength f (b) low energy neutrons ( 0.04 eV at 300K)
initial direction of -particle (c) a moderator making neutrons slow
de Broglie wavelength i
(d) both (b) and (c)
final direction of carbon nucleus 77. X-rays create an image of bone on a sheet of photographic
de Broglie wavelength e
film because
After the collision, the de Broglie wavelengths of the - (a) X-rays penetrate much more effectively through bone
particle and the carbon nucleus are f and e respectively. than through tissue. They strike the photographic film,
Which of the following relations about de Broglie which then fluoresces white wherever bone is present
wavelengths is correct
(b) X-rays are reflected by metals such as the calcium in
(a) i < f (b) i > f
bone. X-rays reflected from the bones are imaged with
(c) f = e (d) i = e
71. One gram of a radioactive sample of half-life 10 min is sealed a lens to form a real image of the bonds on photographic
in a capsule at time t = 0. Amount of sample decayed upto 5 film.
min is (c) the calcium atoms in bone absorb X-rays much more
(a) 0.293g (b) 0.5g effectively than the lighter atoms in tissue. The bones
(c) 0.25g (d) 0.707g cast shadows on the film.
72. A radioactive substance with decay constant of 0.5s–1 is (d) the hydrogen atoms in tissue absorb X-rays much more
being produced at a constant rate of 50 nuclei per second.
effectively than do the heavier atoms in bone. The
If there are no nuclei present initially, the time (in second)
tissue casts dark shadows on the film.
after which 25 nuclei will be present is
(a) 1 (b) ln 2 78. The carbon cycle of stars gives the following sequence of
(c) ln (4/3) (d) 2 ln (4/3) fuses and decays:
73. If a star can convert all the He nuclei completely into oxygen
nuclei, the energy released per oxygen nuclei is [Mass of p 12
C 13
N , 13 N 13
C + e+ +
He nucleus is 4.0026 amu and mass of Oxygen nucleus is
13 14 , p 14 15
15.9994 amu] p C N N O
(a) 7.6 MeV (b) 56.12 MeV
(c) 10.24 MeV (d) 23.9 MeV
15
O 15
N + e+ + , p 15
N 12
C + 4 He
74. The nuclear binding energies of the elements P and Q are
is a -quantum, is a neutron, e+ is a positron.
EP and EQ respectively. Three nuclei of elements Q fuse to
form one nucleus of element P. In this process the energy Determine the energy, which is released during the formation
released is ‘e’. The correct relation between EP, EQ and e of one mole of helium.
will be (a) 2.4181 GJ (b) 4.4181 TJ
(a) EQ = 3EP + e (b) EQ = 3EP – e (c) 1.2181 MJ (d) 2.4181 TJ
(c) EP = 3EQ + e (d) EP = 3EQ – e

MARK YOUR 69. 70. 71. 72. 73.


RESPONSE 74. 75. 76. 77. 78.
682 IIT-JEE PHYSICS Challenger
79. A radioactive nucleus can decay by two different processes. 82. A free particle with initial kinetic energy E, de-Broglie
The half life for the first process is t1 and that for the second wavelength , enters a region wherein it has a potential
process is t2. If effective half life is t, then energy V, what is the new de-Broglie wavelength ?
1 1 1 (a) (1 + E/V)
(a) t = t1 + t2 (b) = +
t t1 t 2 (b) (1 – V/E)
(c) (1 + V/E)0.5
2t1t 2 t1 + t 2 / (1 – V/E)0.5
(c) t= (d) t = (d)
t1 + t 2 2 83. The largest wavelength in the ultraviolet region of the
80. Two radioactive substances X and Y emit and particles hydrogen spectrum is 122 nm. The smallest wavelength in
respectively. Their disintegration constants are in the ratio the infrared region of the hydrogen spectrum (to the nearest
2 : 3. To have equal probabilities of getting emission of integer) is
and particles, the ratio of number of atoms X to that of Y at (a) 802 nm (b) 823 nm
any time instant is (c) 1882 nm (d) 1648 nm
(a) 2 : 3
84. The natural uranium ore now consists of 1 = 99.28% 238U
(b) 3 : 2
and 2 = 0.72% 235U. Half-life periods of 238U and 235U
(c) e : 1
(d) (e – 1) : 1 nuclei are correspondingly equal to T1 = 4.47 ×109 years
81. The beam of neutrons loses 15% of its initial number after and T2 = 0.70 × 109 years. Estimate the Earth’s age assuming
passing through the cadmium plate with thickness 1 mm. the amounts of two isotopes were equal at the moment of
The velocity of neutrons does not change. What fraction of birth of our planet.
neutrons passes through the cadmium plate with thickness (a) 5.90 × 109 years
10 mm ? (b) 2.90 × 109 years
(a) 0.5 (b) 1.0 (c) 5.10 × 102 years
(c) 0.2 (d) 0.8
(d) 3.12 × 106 years

MARK YOUR 79. 80. 81. 82. 83.


RESPONSE 84.

PASSAGE 1 2. Kinetic energy of the electron in the nth orbit will be


An electron is orbiting in a circular orbit of radius r under the nhBe nhBe
(a) (b)
influence of a constant magnetic field B. Assuming that Bohr’s 4 m 2 m
postulates regarding the quantization of angular momentum nhB nhB
holds good of this electron. [h = planck’s constant, e = charge (c) (d)
2 em 4 em
on electron and m = mass of electron)
3. The potential energy of interaction between the magnetic
1. Radius of nth orbit of the electron will be moment of the orbital current due to the electron moving in
2nh nh it swith orbit and the magnetic field is
(a) (b) nhB nhBe
Be 2 Be –
(a) (b)
2 me 4 m
neh 2neh
(c) (d) nhB nhBe
2 B B (c) – (d)
4 em 4 m

MARK YOUR
1. 2. 3.
RESPONSE
MODERN PHYSICS 683

PASSAGE 2
NB NB
A small particle of mass m moves in such a way that the potential
1
energy of particle is given as U m 2 r 2 where is constant
2
and r is the distance of particle from centre. If Bohr’s model of (c) t (d) t
quantization of angular momentum and circular orbit is valid for
the particle, answer the following questions (h = Planck’s constant) 8. The total number of nuclei of B at time t = 0
4. The radius of nth orbit of the particle is 2N 0 1 1
(a) (b) 2N 0 ÷
1 1
e e e2
nh 2 nh 2
(a) ÷ (b)
÷ 1
4 m 2 m (c) 2 N0 1 ÷ (d) 2N0 (e2 – e)
e
1
2 1 9. The number of nuclei of C at time t =
n h 2 0
nh 2
(c) ÷ (d) ÷
4 m 2 m 1 2 1 2
N0 1 +
e e2 ÷
(a) N0 1 2÷
(b)
e e
5. Kinetic energy of particle in nth orbit is
3 2
n2 h (c) N0 1 + (d) N0 (1 + e2 – e).
e e2 ÷
nh
(a) (b) 2
4 4
PASSAGE 4
nh n h
2 2 A sample contains two radioactive nuclei X and Y with half lives
(c) (d) . 2 hr and 1 hr respectively. The nucleus X-decays into the nucleus
2
Y and Y decays into a stable nucleus Z. At time t = 0 the activities
6. Total energy of particle in its orbits is of the components in the sample were equal and were each equal
nh nh to A0.
(a) (b)
4 2 tY2 = 2 hr tY2 = 1 hr
X Y Z
nh Suppose that NX, NY are the number of nuclei of X and Y,
(c) zero (d)
2 respectively, at time t. It is given that
.t
Y NY = NY) + C1 e Y . ......... (i)
X (NX +
PASSAGE 3
where X, Y are the decay constants of X and Y; C1 is an arbitrary
A radioactive nuclei ‘A’ decays into ‘B’ with mean life 1 = 0 and ‘B’ constant.
decays into ‘C’ with mean life 2 = 0. ‘C’ is final stable product of
10. The value of C1 is
the reaction. At t = 0 number of active nuclei of ‘A’ is N0 and number
of nuclei of ‘B’ and ‘C’ is zero. A0
(a) (b) – A0
7. Which of the following graphs between number of nuclei of 2
B versus time is correct ? (c) – 2 A0 (d) 0
11. Number of nuclei of Y at time t is equal to (t is in hour)
NB NB
A0 1 A0 t/2
(a) [2 t/2
2 t] (b) [2 2 t]
ln 2 ln 2

A0 t A0 2t
(a) (b) 2 t]
ln 2 ÷
t t (c) [2 (d) e
ln 2

MARK YOUR 4. 5. 6. 7. 8.
RESPONSE 9. 10. 11.
684 IIT-JEE PHYSICS Challenger
12. The ratio of active nuclei of Y at t = 4 h to that at t = 0 is PASSAGE 7
2 3 In a living organism, the quantity of C14 is the same as in the
(a) (b)
3 4 atmosphere. But in organisms which are dead, no exchange takes
place with the atmosphere and by measuring the decay rate of
8 7 14C in the old bones or wood, the time taken for the activity to
(c) (d) .
15 16 reduce to this level can be calculated. This gives the age of the
wood or bone.
PASSAGE 5 Given : T 1/2 for 14 C is 5370 years and the ratio of
The fission fragments in thermal fission of U235 are found to be 14 C/ 12 C is 1.3 × 10–12 .
98 136 18. The decay rate of 14C in 1g of carbon in a living organism is
42Mo and 54Xe . Estimate the energy liberated in the reaction.
The masses of U235, Xe136 and Mo98 are 235.044 U, 135.907 U (a) 25 Bq (b) 2.5 Bq
and 97.906 U respectively. (c) 0.25 Bq (d) 5 Bq
13. Number of electrons released is 19. If in an old sample of wood of 10g the decay rate is 30
(a) 1 (b) 2 decays per minute, the age of the wood is
(c) 3 (d) 4 (a) 50 years (b) 1000 years
14. Number of neutrons released is (c) 13310 years (d) 15300 years
(a) 1 (b) 2 20. The decay rate in another piece is found to be 0.30 Bq per
(c) 3 (d) 4 gm then we can conclude
15. Energy liberated in the fission process is (a) the sample is very recent
(a) 806 MeV (b) 506 MeV
(b) the observed decay is not that of 14C alone
(c) 206 MeV (d) 56 MeV
(c) there is a statistical error
(d) all of these
PASSAGE 6
A radionuclide with disintegration constant is produced in a PASSAGE 8
reactor at a constant rate (= 2 ) nuclei per second. At t = 0, there
are no nuclei present in the reactor. During each decay energy E0 The diagram shows the basic setup for the production of X-
is released. 20% of this energy is utilized in increasing the rays. A1 and A2 are two ammeters, which have readings 2.55 A
temperature of water. Assume that there is no loss of energy and 2.566A respectively. F is a filament which is also the cathode.
through water surface. (Given E0 = 100 times the energy required The potential difference applied between P and Q is 50000V.
to raise the temperature of mass m of water 0°C to 100°C). Assume that all X-ray photons have the maximum possible
16. Expression of energy released in time t is energy and that one X-ray photons is emitted for every 100
electrons incident on the target. You may assume that the kinetic
(a) E0 t energy of the other electrons reappear as heat in the tube.
t (1 e )
P Q

(b) 2E0 t
t (1 e )

E0 A2
(c) t (1 e t )
2
E0 F
(d) t
t (1 e ) A1
4
17. Find the increase in temperature of m mass of water in time 21. The number of X-ray photons produced per second is
t = T1/2 approximately
(a) 772 K (b) 512K (a) 1012 (b) 1015
(c) 630 K (d) 750 K (c) 10 18 (d) 1021

MARK YOUR 12. 13. 14. 15. 16.


RESPONSE 17. 18. 19. 20. 21.
MODERN PHYSICS 685

22. The momentum of each X-ray photon is approximately 26. Which transition will occur when a hydrogen atom is
(a) 3 × 10–17 kg ms–1 (b) 3 × 10–20 kg ms–1 radiated with radiation of wavelength 103 nm.
–23
(c) 3 × 10 kg ms –1 (d) 3 × 10–26 kg ms–1 (a) I (b) II
23. The rate at which heat is produced in the X-ray tube is (c) IV (d) V
approximately
(a) 79.2 W (b) 204W PASSAGE 10
(c) 2040 W (d) 792 W
A photoelectric effect is a process of emission of an electron from
PASSAGE 9 the surface of a metal after the absorption of a photon with energy
enough to make an electron free with some initial kinetic energy.
The figure shows an energy level diagram for the hydrogen atom. The minimum energy needed to release the electron with zero
Several transitions are marked as I, II, III......... The diagram is only
kinetic energy is called the work function of the material of the
indicative and not scale.
surface. Electrons in the photoelectric effect are called the
photoelectrons.
A light source emitting radiation at 7.00 × 1014 Hz is incapable of
ejecting photoelectrons from a certain metal. In an attempt to use
Principal quantum no.

this source to eject photoelectrons from the metal, the source is


III given a velocity toward the metal.

II IV V VI 1+ v / c
Doppler effect for light wave f = f
1 v/c
27. When the speed of the light source is equal to 0.280c,
I photoelectrons just begin to be ejected from the metal. What
is the work function of the metal ?
(a) 3.87 eV (b) 8.78 eV
24. In which transition is a Balmer series photon absorbed? (c) 2.17 eV (d) 1.78 eV
(a) II (b) III 28. When the speed of the light source is increased to 0.900c,
(c) IV (d) VI determine the maximum kinetic energy of the
25. The wavelength of the radiation involved in transition II is photoelectrons.
(a) 291 nm (b) 364 nm (a) 3.87 eV (b) 3.14 eV
(c) 487 nm (d) 652 nm (c) 8.78 eV (d) 1.78 eV

MARK YOUR 22. 23. 24. 25. 26.


RESPONSE 27. 28.

1. Statement - 1 : Photosensitivity of a metal is high if its 2. Statement - 1 : If the half life of a radioactive substance is
work function is small. 40 days then 25 % substance decay in 20
days.
Statement - 2 : Work function = hf 0 where f 0 is the
n
threshold frequency. 1
Statement - 2 : N = N0 ÷ where, n =
time elapsed
2 half life period

MARK YOUR
1. 2.
RESPONSE
686 IIT-JEE PHYSICS Challenger
3. Statement - 1 : Separation of isotope is possible because 10. Statement - 1 : Photoelectric effect demonstrates the
of the difference in electron numbers of paritcle nature of light.
isotope. Statement - 2 : The number of photoelectrons is
Statement - 2 : Isotope of an element can be separated by proportional to the frequency of light.
using a mass spectrometer. 11. Statement - 1 : The binding energy per nucleon, for nuclei
4. Statement - 1 : Kinetic energy of photo electrons emitted with atomic mass number A > 100,
by a photosensitive surface depends upon decreases with A.
the intensity of incident photon. Statement - 2 : The nuclear forces are weak for heavier
Statement - 2 : The ejection of electrons from metallic surface nuclei.
is possible with frequency of incident 12. Statement - 1 : If the accelerating potential in an X-ray
photon is more than the threshold tube is increased, the wavelengths of the
frequency. characteristic X-rays do not change.
5. Statement - 1 : Mass of moving photon varies inversely Statement - 2 : When an electron beam strikes the target
as the wavelength. in an X-ray tube, part of the kinetic energy
Statement - 2 : Energy of the particle = mass × (speed of is converted into X-ray energy.
light)2 13. Statement - 1 : If maximum frequency of Balmer and
6. A
Statement - 1 : z X undergoes 2 -decays, 2 -decays Paschen series is f1 and f2 respectively, then
frequency of Ist line of Balmer series is
and 2 -decays and the daughter product
(f1 – f2).
A 8
is Z 2 X . Statement - 2 : Difference of energy level between two
orbits is constant for a atom.
Statement - 2 : In -decays the mass number decreases
by 4 and atomic number decreases by 2. In 14. Statement - 1 : If the current in the filament of electron
2 -decays the mass number remains gun in a X–ray tube is increased, the
penetration power of X – rays is increased.
unchanged, but atomic number increases
by 1 only. Statement - 2 : Increasing current increases the number
of electrons emitted by the electron gun.
7. Statement - 1 : Bohr had to postulate that the electrons in
stationary orbits around the nucleus do 15. Statement - 1 : When cathode rays (accelerated with high
not radiate. voltage 10 kV) strikes a hard metallic
surface, highly penetrating radiation is
Statement - 2 : According to classical physics all moving
obtained.
electrons radiate.
Statement - 2 : Due to conversion of electron’s kinetic
8. Statement - 1 : Energy is released in nuclear fission.
energy into photon’s energy, the shortest
Statement - 2 : Total binding energy of the fission wavelength limit of X-rays produced is
fragments is larger than the total binding inversely proportional to the accelerating
energy of the parent nucleus. voltage.
9. Statement - 1 : Heavy water is preferred over ordinary 40
16. Statement - 1 : It is easy to remove a proton from 20 Ca
water as a moderator in reactors.
nucleus as compared to a neutron.
Statement - 2 : Heavy water, used for slowing down the
neutrons, has lesser absorption Statement - 2 : Inside nucleus neutrons are acted on only
probability of neutrons than ordinary by attractive forces but protons are also
water. acted on by repulsive forces.

3. 4. 5. 6. 7.
MARK YOUR
8. 9. 10. 11. 12.
RESPONSE
13. 14. 15. 16.
MODERN PHYSICS 687

1. Two photocathodes are illuminated by the light emitted by (b) number of electron reaching the anode will change
a single source. The dependence of photocurrent versus
(c) number of characteristics X-ray lines may increase
voltages between cathode and anode is shown by curves 1
and 2 as shown in the figure. (I/Imax represents ratio of (d) the difference between 0 (minimum wavelength) and
photocurrent to saturaion current) K (wavelength of K X-ray) will get changed.

I/Imax 6. The threshold wavelength for photoelectric emission from


a material is 5200 Å. Photoelectrons will be emitted when
this material is illuminated with monochromatic radiation
1 from a
2
(a) 50 watt infrared lamp (b) 1-watt infra-red lamp
V
(c) 50 watt ultraviolet lamp (d) 1-watt ultraviolet lamp
(a) Photocathode 1 has higher work function than 2
7. At a given instant there are 25% undecayed radioactive
(b) Photocathode 2 has higher work function than 1
nuclei in a sample. After 10 sec, the number of undecayed
(c) Saturation current may be different for 1 and 2
nuclei reduces to 12.5%. Then
(d) Saturation current must be same for 1 and 2
2. The initial nucleus of uranium series is 92U238 and the final (a) mean life of the nuclei is 14.43s
nucleus is 82 Pb 206 . When uranium decays to lead by (b) the time in which the number of original undecayed
emitting and particles then number of nuclei would reduce to 6.25% is 40 sec.
(a) -particles produced is 8
(c) the ratio of mean life to half-life is 0.1443
(b) -particles produced is 6
(c) -particles produced is 6 (d) suppose 12.5% of the radioactive nuclei remain
(d) -particles produced is 8 undecayed at say t = 0, then 100% would have remained
3. The shortest wavelength of X-rays emitted from an X-ray 40 seconds earlier
tube depends on 8. X-rays are emitted by a tube containing the element Niobium
(a) the current in the tube (Z = 41) as anti cathode, and the K X-rays are allowed to be
(b) the voltage applied to the tube incident on an unknown gas containing hydrogen like ions.
(c) the nature of the gas in tube It is found that the X-rays cause the emission of
(d) the atomic number of the target material photoelectrons with an energy of 2.7 keV from these ions.
4. 235 is -active. For large quantity of the element, choose
92U Choose the correct options
the correct options
(The intensity of X-rays is 100 mW/m2 and 1% of the X-rays
(a) the probability of a nucleus disintegrating during one cause ionization)
second interval remains constant for all time
(b) the probability of a nucleus disintegrating during one (a) the minimum voltage at which the x-ray tube should be
second is lower in the 1 half-life and greater in the 2 operated so that the momentum of the emitted
half-life. photoelectrons is doubled is 24.5 kV
(c) quite an appreciable quantity U235 will remain even (b) the approximate value of Z for the target of the
after the average lifetime anticathode after the momentum has been doubled is
(d) the energy of the emitted -particle is less than the 50
disintegration energy of U235 induces
5. In an X-ray tube, if the accelerating potential difference is (c) the approximate value of the atomic number of the
increased then atoms of the gas is 32
(a) the frequency of characteristic X-rays of a material will (d) no. of ions produced in 1 sec is 850 per m3
get changed

MARK YOUR 1. 2. 3. 4. 5.
RESPONSE 6. 7. 8.
688 IIT-JEE PHYSICS Challenger
9. C14 disintegrates by emission with an reaction energy (b) the saturation photocurrent depends on the
(Q-value) of 0.155 MeV. A particle with an energy of wavelength of the incident light
0.025MeV is emitted in a direction of 135 degrees to the (c) the stopping potential depends on the intensity of the
direction of motion of the recoil nucleus. Choose the correct incident light
options (c is speed of light in vacuum). (d) the saturation photocurrent depends on the intensity
(Me = 0.511 MeV/c2) of the incident light
12. When photons of energy 4.25 eV strike the surface of metal
(a) Momentum of the is 0.158 MeV/c A, the ejected photoelectrons have maximum kinetic energy,
(b) Momentum of the neutrino is 0.13 MeV/c TA eV and de Broglie wavelength A. The maximum kinetic
energy of photoelectrons liberated from another metal B by
(c) Momentum of the 14N is 0.179 MeV/c
photons of energy 4.70 eV is TB (TA 1.50) eV. If the be
(d) Momentum of the 14N is 0.158 MeV/c
Broglie wavelength of these photoelectrons is
10. A radioactive battery, consists of N0 atoms of a radioelement
emitting -rays with a disintegration constant , of which a B 2 A , then
fraction f is captured at the anode A and converted into a (a) The work function of A is 2.25 eV
current. A charged capacitor C initially charged as shown in (b) The work function of B is 4.20 eV.
the figure and a resistance R are connected across the (c) TA = 2.00 eV.
battery. Choose the correct options
(d) TB = 2.75 eV
(The battery contained N0 atoms at t = 0. )
13. The radioactive barium isotope 137Ba has a relatively short
half-life and can be easily extracted from a solution containing
its parent cesium (137Cs). This barium isotope is commonly
A used in an undergraduate laboratory exercise for
–q0
R demonstrating the radioactive decay law. Undergraduate
C
–q student using modest experimental equipment took the data,
presented in fig. Along the vertical axis, the student plotted
lnR that was a natural logarithm of a decay rate, or activity,
R. R is the rate of change of the number of nuclei of a parent
(a) The charge across the capacitor as a function of time t, isotope due to the decay (a conversion into the other
isotope). Choose the correct options
t
is t÷
q1e RC + q2 e ln R
(b) Time when the charge on the capacitor become zero is
1 q1
ln
1 q2 ÷ 8
RC
(c) Time when the charge on the capacitor becomes zero

1 q1
ln
q2 ÷
is
1
7
RC
(d) The charge across the capacitor as a function of time t
t
RC t÷
is q1e q2 e
6 t, min
11. For a photoelectric cell 0 1 2 3 4 5 6 7 8 9 10
(a) stopping potential depends on the wavelength of the
incident light

MARK YOUR
9. 10. 11. 12.
RESPONSE
MODERN PHYSICS 689

ln 2 V0
(a) The equation of the curve is ln R ln R0 t
T
Metal 1 Metal 2 Metal 3
(b) The half-life for the decay of 137Ba is 2.8 min.
ln 2 1 2 3
(c) The equation of the curve is ln R = ln R0 + t
T
(d) The half-life for the decay of 137Ba is 4.6 min. 1 (nm–1)
0.001 0.002 0.004
1
14. The graph between the stopping potential (V0) and ÷ is (a) 1: 2: 3 = 1: 2 : 4
(b) 1: 2: 3 = 4 : 2 :1
shown in the figure. 1 , 2 and 3 are work functions,
hc
which of the following is/are correct (c) tan µ
e
(d) ultravioletlight can be used to emit photoelectrons from
metal 2 and metal 3 only

MARK YOUR
13. 14.
RESPONSE

1. Match the following :


Column I Column I
(A) Nuclear fusion (p) Converts some matter into energy
(B) Nuclear fission (q) Generally possible for nuclei with low atomic number
(C) -decay (r) Generally possible for nuclei with higher atomic number
(D) Exothermic nuclear reaction (s) Essentially proceeds by weak nuclear forces
2. Some laws / processes are given in Column I. Match these with the physical phenomena given in Column II and indicate your
answer by darkening appropriate bubbles in the 4 × 4 matrix given in the ORS
Column I Column II
(A) Transition between two atomic energy levels (p) Characteristic X-rays
(B) Electron emission from a material (q) Photoelectric effect
(C) Mosley's law (r) Hydrogen spectrum
(D) Change of photon energy into kinetic energy of electrons (s) -decay

1. 2.

MARK YOUR
RESPONSE
690 IIT-JEE PHYSICS Challenger
3. In the photoelectric effect experiment, if v is frequency of radiation and I is intensity in terms of number of photons incident per
second per unit area, then match the following
Column-I Column -II
(A) If v is increased keeping I and work function constant (p) stopping potential increases
(B) If I is increased keeping v and work function constant. (q) saturation current increases
(C) If distance between cathode and anode is increased. (r) maximum kinetic energy of photoelectron increases
(D) Work function is decreased keeping v and I constant (s) stopping potential remains same

4. Match the columns :


Column I Column II
(A) Excited atom of small atomic number with a vacancy (p) X – rays
in M shell
(B) Excited atom of large atomic number with vacancy (q) –rays
in K shell
(C) Unstable nuclei (r) Infra red rays
(D) Electron proton annihilation (s) –decay

5. Column I Column II
(A) Wave nature of light (p) Young’s double slit experiment
(B) Particle nature of light (q) Photoelectric effect
(C) Wave nature of electrons (r) Electron microscope
(D) Particle nature of electrons (s) Cathode rays
6. Some quantities related to the photoelectric effect are mentioned under Column I and II. Match each quantity in Column I with
the corresponding quantities in Column II on which it depends.
Column I Column II
(A) Saturation current (p) Frequency of light
(B) Stopping potential (q) Work function
(C) de-Broglie wavelength of photoelectron (r) Area of photosensitive plate
(D) Force due to radiation falling on the photoplate (s) Intensity of light (at constant frequency
7. Match the columns :
Column I Column II
(A) Mass of products formed is less than that of reactants (p) – decay
(B) Binding energy per nucleon increases. (q) – – decay
(C) Emitted particles have variable kinetic energy (r) + – decay
(D) Two body decay process. (s) – decay

3. 4. 5. 6. 7.

MARK YOUR
RESPONSE
MODERN PHYSICS 691

8. Column I lists certain processes leading to the emission of electromagnetic radiation, while column II lists certain energies
typical of a quantum of electromagnetic radiation. Match the entries in Column I with those in Column II.
Column I Column II
(A) A photon is incident on calcium (solid) causing the (p) 10.2 eV
emission of a photoelectron of KE 1.6 eV
(B) An X- ray photon is emitted due to bremsstrahlung (q) 1.08 meV
process, occurring when a fast electronis stopped
in a heavy metal target
(C) An excited nucleus of 198Hg formed as a result of (r) 4.8 eV
– decay emits a – photon
(D) A photon is absorbed by a hydrogen atom in (s) 16keV
its ground state
9. Using Bohr’s model, match the following (where the letters n and Z have usual meaning).
Column I Column II
(A) Due to revolving electron, the magnetic field produced (p) n –5
at its centre is proportional to
(B) Magnetic moment of revolving electron is proportional to (q) n
(C) De-Broglie wavelength of revolving electron (r) Z3
is proportional to
(D) Area velocity of revolving electron about nucleus is (s) independent of Z
proportional to (t) inversely to Z
10. Q is energy released in the decay, mx is atomic mass of parent nucleus, my is atomic mass of daughter nucleus and me is mass
of electron, then match the following
Column I Column II
(A) k capture (p) Q = (mx – my) c2
(B) – decay (q) Q = (mx – my – me) c2
(C) + decay (r) Q = (mx – my – 2me) c2
(s) Q = (mx – my + 2me) c2
11. Match the following –
Column I Column II
(A) Characteristic X-rays (p) X-ray tube voltage
(B) Continuous X-rays (q) Knock out of e–
(C) Cut off wavelength (r) Bremsstrahlung radiation
(D) X-ray production (s) Moseley’s law

8. 9. p q r s t 10. 11.
A p q r s t
MARK YOUR B p q r s t
RESPONSE C p q r s t
D p q r s t
692 IIT-JEE PHYSICS Challenger

1. A star initially has 1040 deuterons. It produces energy via the transition to a higher energy level by absorbing
processes 1 H +1 H2 2
1
3 2
H +p, and 1 H +1H 3
2
4
He + n. monochromatic light of photon energy 2.7 eV. Subsequently,
If the average power radiated by the star is 1016 W, in what the atoms emit radiation of only six different photon
time (in tera-second) the deuteron supply of the star is energies. Some of the emitted photons have energy 2.7 eV,
exhausted ? some have energy more and some have less than 2.7 eV.
The masses of the nuclei are as follows : Find the ionization energy (in eV) for the gas atoms.
M (H 2 )=2.014 amu; 5. Light from a discharge tube containing hydrogen atoms falls
on the surface of a piece of sodium. The kinetic energy of the
M (p) = 1.007 amu; M(n) = 1.008 amu; M (He 4 )=4.001amu.
fastest photoelectrons emitted from sodium is 0.73 eV. The
2. A single electron orbits around a stationary nucleus of charge work function for sodium is 1.82 eV. Find the energy (in eV)of
+ Ze. Where Z is a constant and e is the magnitude of the the photons causing the photoelectric emission. (Ionization
electronic charge. It requires 47.2 eV to excite the electron
potential of hydrogen is 13.6 eV)
from the second Bohr orbit to the third Bohr orbit. Find the
energy (in eV) required to excite the electron from the third 6. A hydrogen like atom (atomic number Z) is ina higher excited
to the fourth Bohr orbit. state of quantum number n. The excited atom can make a
(The ionization energy of hydrogen atom = 13.6 eV, Bohr transition to the first excited state by successively emitting
radius = 5.3 × 10–11 metre, velocity of light = 3 × 108 m/sec. two photons of energy 10.2 and 17.0 eV respectively.
Planck’s constant = 6.6 × 10–34 joules - sec). Alternately, the atom from the same excited state can make
3. A doubly ionised lithium atom is hydrogen-like with atomic a transition to the second excited state by successively
number 3. Find the wavelength (in Å) of the radiation emitting two photons of energies 4.25 eV and 5.95 eV
required to excite the electron in Li++ from the first to the respectively. Determine the value of n. (Ionization energy
third Bohr orbit. (Ionisation energy of the hydrogen atom of H-atom = 13.6 eV)
equals 13.6 eV.)
7. At a given instant there are 25% undecayed radio-active nuclei
4. A gas of identical hydrogen-like atoms has some atoms in in a sample. After 10 seconds the number of undecayed nuclei
the lowest (ground) energy level A and some atoms in a reduces to 12.5%. Calculate the mean life (in sec) of the
particular upper (excited) energy level B and there are no radiactive nuclei.
atoms in any other energy level. The atoms of the gas make

1. 2. 3. 4. 5. 6. 7.

MARK
YOUR
RESPONSE
MODERN PHYSICS 693

8. A hydrogen-like atom of atomic number Z is in an excited 12. Frequency of a photon emitted due to transition of electron
state of quantum number 2n. It can emit a maximum energy of a certain element from L to K shell is found to be
photon of 204 eV. If it makes a transition to quantum state 4.2 × 1018 Hz. Using Moseley’s law, find the atomic number
n, a photon of energy 40.8 eV is emitted. Find Z. of the element, given that Rydberg’s constant
9. In a nuclear reaction 235U undergoes fission liberating 200 R = 1.1 × 107 m–1.
MeV of energy. The reactor has a 10% efficiency and 13. In hydrogen-like atom (Z = 11), nth line of Lyman series has
produces 1000 MW power. If the reactor is to function of
wavelength . The de-Broglie's wavelength of electron in
10 years, find the total mass (in mega gram) of uranium
the level from which it originated is also . Find the value of
required.
n?
10. A hydrogen-like atom (described by the Bohr model) is
14. Let neutron be a point mass and hydrogen atom a solid
observed to emit six wavelengths, originating from all
sphere. A neutron makes a head on collision with a hydrogen
possible transitions between a group of levels. These levels
atom in ground state kept at rest. The coefficient of restitution
have energies between -0.85 eV and -0.544 eV (including
for collision is e = 1/2. Find the minimum kinetic energy of
both these values). Find the atomic number of the atom. colliding neutron (in eV) so that hydrogen atom is excited to
(Take hc = 1240 eV-nm, ground state energy of hydrogen higher energy level such that magnitude of electrostatic
atom = –13.6 eV) potential energy in the excited state is one-eighth of K.E. of
11. Two metallic plates A and B, each of area 5 × 10–4 m2, are electron in ground state (mass of neutron = mass of
placed parallel to each other at a separation of 1 cm. Plate B hydrogen).
carries a positive charge of 33.7 × 10–12 C. A monochromatic 15. Consider the -decay
beam of light, with photons of energy 5 eV each, starts 198
Au 198
Hg + – +v –
falling on plate A at t = 0 so that 1016 photons fall on it per
square meter per second. Assume that one photoelectron where 198Hg represents a mercury nucleus in an excited
is emitted for every 106 incident photons. Also assume that state of energy 1.088 MeV above the ground state. What
can be the maximum kinetic energy of the electron emitted.
all the emitted photoelectrons are collected by plate B and
The atomic mass of 198Au is 197.968233 u and that of 198Hg
the work function of plate A remains constant at the value
is 197.966760 u and 1 u = 931 MeV/c2 (express your answer
2eV. If the number of photoelectrons emitted upto t = 10 s is
in MeV)
(x × 107), find x.
16. If an excited He+’ ion emits two photons in succession with
Neglect the time taken by the photoelectrons to reach plate wavelength 1026.7 and 304 Å while making a transition to
B. Take 0 = 8.85 × 10–12 C2/N-m2 its ground state. Taking value of R = 1.097 × 107/m, find the
quantum number of excited state of He+ ion.

8. 9. 10. 11. 12. MAR

MARK
YOUR
RESPONSE 13. 14. 15. 16.
694 IIT-JEE PHYSICS Challenger

1 (a) 15 (a) 29 (a) 43 (b) 57 (b) 71 (a)


2 (a) 16 (b) 30 (a) 44 (a) 58 (c) 72 (d)
3 (b) 17 (c) 31 (c) 45 (a) 59 (c) 73 (c)
4 (b) 18 (c) 32 (a) 46 (b) 60 (d) 74 (c)
5 (a) 19 (d) 33 (c) 47 (a) 61 (b) 75 (a)
6 (a) 20 (d) 34 (c) 48 (b) 62 (b) 76 (d)
7 (a) 21 (d) 35 (d) 49 (a) 63 (a) 77 (c)
8 (b) 22 (d) 36 (b) 50 (c) 64 (a) 78 (d)
9 (a) 23 (c) 37 (b) 51 (d) 65 (a) 79 (b)
10 (a) 24 (b) 38 (d) 52 (c) 66 (c) 80 (b)
11 (b) 25 (a) 39 (a) 53 (c) 67 (b) 81 (c)
12 (c) 26 (a) 40 (c) 54 (a) 68 (a) 82 (d)
13 (a) 27 (d) 41 (b) 55 (b) 69 (b) 83 (b)
14 (d) 28 (c) 42 (d) 56 (b) 70 (a) 84 (a)

1 (b) 6 (c) 11 (a) 16 (a) 21 (b) 26 (d)


2 (a) 7 (d) 12 (d) 17 (a) 22 (c) 27 (a)
3 (d) 8 (b) 13 (d) 18 (c) 23 (d) 28 (c)
4 (b) 9 (c) 14 (a) 19 (c) 24 (d)
5 (a) 10 (a) 15 (c) 20 (d) 25 (c)

1 (b) 5 (b) 9 (a) 13 (a)


2 (d) 6 (a) 10 (c) 14 (d)
3 (d) 7 (b) 11 (a) 15 (a)
4 (d) 8 (a) 12 (b) 16 (a)

1 (b, c) 5 (c, d) 9 (a, b, c) 13 (a, b)


2 (a, b) 6 (c, d) 10 (a, b) 14 (a, c)
3 (b, d) 7 (a, b) 11 (ad)
4 (a, c, d) 8 (a, b, c, d) 12 (a, b, c)

1. A-p, q; B-p, r; C-p, s; D-p, q, r 7. A-p, q, r, s; B-p, q, r, s; C-q, r; D-p


8. A-r; B-s; C-q; D-p
2. A-p, r; B-q, s; C-p; D-q 9. A-p, r; B-q, s; C-q, t; D-q, s
3. A-p, r; B-q, s; C-s; D-p, r 10. A-p; B-p; C-r
4. A-r; B-p; C-q, s; D-q 11. A-q, s; B-r; C-p; D-p, q, r, s
5. A-p; B-q; C-r; D-s
6. A-s; B-p, q; C-p, q; D-p, r, s

1 1.29 4 14.5 7 14.4 10 3 13 24 16 6


2 16.5 5 2.55 8 4 11 5 14 34
3 114 6 6 9 38.4 12 42 15 0.28
MODERN PHYSICS 695

1. (a) The shortest wavelength of Balmer series is for n2 =


=–
1 7 1 1
1.097 10
B 22 2
ch
t~h
7
1.097 10 4
= B = 1.097
10 7
4 h 600 10 –9
For Lyman series, shortest wavelength is for n2 = ~ . ~ =
t hc tc 8 10 –9 3 108
1 7 1 1
= 1.097 10 1.097 107
L 1 2 2 ~ 10 –6

10 7 4. (b) t1/2 = 3.8 day


L= 1.097 0.693 0.693
= = = 0.182
7 t1/ 2 3.8
B 4 /1.097 10 4
The ratio is = 7 =
1/1.097 10 1 If the initial number of atom is a = A0 then after time t
L
the number of atoms is a/20 = A. We have to find t.
2. (a) Wave length of spectral line of Balmer series is
2.303 A0 2.303 a
1 1 1 t = log = log
R A 0.182 a / 20
22 n22
2.303
For 1st line, n2 = 3 = log 20 = 16.46 days
0.182
1 1 1 5 5. (a) The wavelength of X-ray lines is given by
R = R
1 4 9 36
1 1 1
n22 ÷ , Where R is the Rydbergg
For second line n2 = 4 = RZ 2
n12
1 1 1 3
R R constant.
2 4 16 16
The Ka line characteristic of an element is produced
1 due to transition from the L-shell (n2 = 2) to the K-shell
5R 16 (n1 = 1). Thus
1
1
1 FG 1 1 IJ = 3 RZ
2
36 3R = RZ2 H12
2 2 K 4 2

4
2 = 1 × 0.746 = 6563 × 0.746 or Z2 =
3R
2 = 4861 Å
3. (b) E. t ~ h 4
= = 1599.25
hc 3 x (1097
. x 10 7 ) x (0.76 x 10 10
)
E=
or Z2 = 1600 Z = 40.
hc 1 3R
E= 6. (a) ( Z 1) 2
4
hc 4 4
. t~h ( Z 1) = = 7 10
3R 3 1.1 10 1.8 10
Now, c = v
200 5 78
+ =0 = = = 26
3 33 3
D
Z = 26 + 1 = 27
696 IIT-JEE PHYSICS Challenger
7. (a) Pin = 25W, = 6600 Å = 6600 × 10–10 m when No is initial number of atoms
nhv = P
0.693 0.693
Number of photons emitted/sec, Here 1 = and 2 = ;
1620 810
10
P P 25 6600 10 N 1 1 0.693 0.693
n= 34 = log e =– + ÷t
hc hc 6.64 10 3 108 No 4 4 1620 810
25
= 8.28 × 1019 = 1019 810 + 1620
3 2.303 [ – 2 × (.3010)] = –0.693 t
3% of emitted photons are producing current 1620 810

3 2 1620 810
I= ne = t = 1080 years
100 2430
3 25 14. (d) For H atom
= × 1019 × 1.6 × 10–19 = 0.4 A E2 – E1 = 10.2 eV
100 3
8. (b) To obtain 6 spectral line, as electron must be excited to 13.6
E= 2
Z 2E
Eg n
fourth orbit with energy , so that the difference is For Z = 10,
16
15 Eg 13.6 10 2
. E1 = = – 1360 eV
16 12
9. (a) Note that v is inversely proportional to n whereas r is
directly to n2. 13.6 102
E2 = = – 3.4 × 100 = –340 eV
10. (a) Nuclear radius, r µ A1/ 3 22
where A is mass number E2 – E1 = – 340 – (– 1360) = 1020 eV
1/ 3
15. (a) By Einstein’s equation of photo-electric effect, the
1/ 3
rCu A 64 maximum kinetic energy of emiited photo-electrons is
= Cu ÷ = ÷
rAl AAl 27 given by
hc
4
rCu = 3.6 Ek = h – W or Ek = –W
÷ = 4.8 Fermi
3
Where, h = Planck’s constant
11. (b) For first line of Balmer series
v = frequency of incident light
1 1 1
=R – ÷ W = work function of metal
4 9 = wavelength of incident light
36
R= 6.6 10 –34 3 108
5 Ek = eV – 4.2eV
Wavelength of the first line , of the Lyman 2000 10 –10 1.6 10 –19
L
series is given by So, Ek = 2eV
1 1 my
= R 1 – ÷ = 36 3 27
= 16. (b) We have K = .Q
L 4 5 4 5 my + m
5
L = K
A 4
.Q 48
A 4
.50 A = 100
27 A 4
12. (c) = 1– 2
17. (c) 4 11H + 4
2 He
2+
+ 2e – + 26 MeV
1 1
represent a fusion reaction.
1 1 R [1 0] Z 2
R Z2 18. (c) N N0 e t
Ny N 0 (1 e t
)
(2) 2 (3)2
dN
31 1 Rate of formation of Y = = + N0e t
On solving , R = dt
5 . Z2
X Y
–dN
13. (a) 1N 2N At t = 0, R N0 t=0 N00
dt
t , R=0 t=t N
–( )t
N
log e 1 2 Ny = N0 (1 – e– t)
No
MODERN PHYSICS 697

19. (d) Mass defect


67 33
m = [ZmP + (A – Z)mn] – M (U238) N2 = N 0 N0 = N0
100 100
= [92 × 1.007825 (238 – 92) × 1.008665] – 238.050783
33 t2
m = 1.93421 amu =e ......(2)
100
BE = 1.93421 × 931.5MeV = 1801.7 MeV
t2
1 2
or 0.33 = e
20. (d) X Y Z
At radioactive equilibrium, 2.303
or t2 = log (0.33) = 32.14 min.
( X) × (NX) = ( Y) × (NY)
Difference between time = t2 t1 = 20 min.
X NY
= N 23. (c) The energy of incident photons is given by
h = eVs + = 2 + 5 = 7eV
Y X
0
(t1 / 2 )Y NY (t1/ 2 )Y 1
(Vs is stopping potential and is work function)
(t1 / 2 ) X = N X or 1.6 1010 = 3.2 10 9 0

P 10 5 P
(t1/2)Y = 5 year Saturation current = 10–5 A = e= e
h 7 e
21. (d) Pb 200 X196 + 2 H 4 ( is photon emission efficiency)
0 = 4V1 + 196 V2 P = 7W
24. (b) The energy in the ground state
196 E1 = –4 Rydberg
V1 V2
4 E1 = –4 × 2.2 × 10–18J
1 1 The energy of the first excited state ( n = 2)
KT 4 V12 196V22
2 2 E1
E2 = = –2.2 10 –18 J
V1 49V2 4
The energy difference
1 E = E2 – E1 = 3 × 2.2 × 10–18J
KD 196 V22
2 Now, the wavelength of radiation emitted is
KD 196V22 49V22 hc
= = =
KT 4V12 + 196V22 V12 + 49V22 E
6.6 10 –34 3 108
49V22 49 = = 300Å
= 2 2 2 = 3 2.2 10 –18
V2 [(V1 / V2 ) + 49] (49) 2 + 49
dU
25. (a) = F (conservative force field)
1 1 dr
= = .
49 + 1 50 K
22. (d) Given : T1/2 = 20 min F= provides the centrifugal force for circular
r
0.693 0.693 motion of electron.
decay constant, = = = 0.0345/minute.
T1/ 2 20
mv 2 K nh
we have N = N0e– t = r=
r r 2 mK
Case-I: (When 33% disintegrated)
1 2 1
N1 = N 0
33
N0 =
67
N0 K.E. of electron = mv = K
100 100 2 2
P.E. of electron = K ln r
67 t1
N0 = N0e .......(1) E (n) = Total energy = K.E. + P.E.
100
67 1 K n2 h2
e t1
or 0.67 = e t1 = K + K ln r = 1 + log 3
100 2 2 4 mk

2.303 E (2) E (1)


t1 = (log 0.67) = 11.60 min. =1
Required ratio =
E (4) E (2)
Case-II: (when 67% disintegrated)
698 IIT-JEE PHYSICS Challenger
26. (a) The penetrating power is dependent on velocity. E 3.87 10 12 J
For a given energy, the velocity of radiation is highest As each reaction involves 3 deuterons, so total number
and -particle is least.
1040
of reactions involved in the process . If each
1 1 1 3
27. (d) As R ÷ reaction produces an energy E, then
n12 n22
1 1 1 1040
÷
R Etotal E 1.29 1028 J
12 n 2 3
Multiply both sides by Etotal Pt
1 1 1 Time of exhaustion of the star
1 R 1 2 ÷ or
1 2
n R n 1.29 10 28
1 1 R 1 t t 1.29 1012 s
= R 1016
or 1 or n =
n2 R R R 1 32. (a) Let t = time for the potential of metal sphere to rise by
28. (c) In the nth orbit, let rn be the radius and vn be the speed one volt.
of electron. Now particles emitted in this time
2 rn r
Time period, Tn = µ n (6.25 1011 ) t
vn vn
rn Number of -particles escaped in this time
1
Now rn µ n2; vn µ v n µ n or Tn µ n
3 3
n (80 /100) (6.25 1010 )t 5 1010 t
3
n1 Charge acquired by the sphere in t sec.
n
n2 ÷
Here 8 = or 1 = 2 i.e., n1 = 2n2
n2 Q (5 1010 t ) (1.6 10 19
)
m2 )v1 + 2 m2 v2
( m1 8 10 19 t coulomb …(i)
29. (a) v1 =
m1 + m2 ( emission of -particle lends to a charge e on metal
As v2 is zero, m2 > m1, v '1 is in the opposite direction. sphere)
m1 = 1, m2 = A. The capacitance C of a metal sphere is given by
( A 1) C 4 r
| v 1 |= v1 0
( A + 1) 1 10 3 10 12
The fraction of total energy retained is
9 109 2 18 farad …(ii)
1/ 2mv 12 ( A 1) 2
= we know that Q = C × V {Here V = 1 volt}
1/ 2v12 ( A + 1) 2
30. (a) Number of atoms in 2kg fuel 910 12
(8 10 )t ÷ 1
2 18
= 6.023 1026 5.12 1024 Solving it for t , we get t = 6.95 µsec.
235
number of fission per second 33. (c) Let A B 2
24
5.12 10 If N0 is total no. of atoms in A and B at t = 0, then initial
= 1.978 1018
30 24 60 60 rate of disintegration of A N 0 , and initial rate of
Energy released per fission
= 200 MeV = 200 × 1.6 × 10–13 = 3.2 × 10–11 J disintegration of B 2 N0
Power output = 3.2 × 10–11 × 1.978 × 1018 1
= 63.28 × 106 W = 63.28 MW As B= 2 A
TBTA
2
2 2
31. (c) 1H 1H 1 H3 p i.e. half life of B is half the half life of A.
2 3
After one half life of A.
1H 1H 2 He 4 n
dN N0
31 H 2 2 He 4
p n dt A 2
m m( 2 He 4 ) m(p) m(n) 3m(1 H 2 ) Equivalently, after two half lives of B
m [4.001 1.007 1.008 3(2.014)]amu dN 2 N0 N0
m 0.026amu dt B 4 2
2
| E| c | m| dN dN
Clearly,
E (9 1016 )(0.026 1.67 10 27
) dt A dt B
E (931.5)(0.026) MeV after n = 1, i.e., one half life of A.
MODERN PHYSICS 699

34. (c) Given, 1 H2 + H2 4


2H + Q
(b) Radiation pressure = P = I
1
The total binding energy of the deutrons 41.
= 4 × 1.15 = 4.60 MeV c
The total binding energy of alpha particle 1 a 3
Area of Hexagon a. ÷ 6
2 ÷
= 4 × 7.1 = 28.4 MeV
2
The energy released in the process
= 28.4 – 4.60 = 23.8 MeV.
I 3a 2 3 3 9a 2 I
35. (d) When one e– is removed from neutral helium atom, it = .
F = PA cos . =
becomes a one e– species. c 2 2 4c
For one e– species we know 42. (d) In Lyman series energy is released in U.V. region.
In Balmer series energy is released in Visible region.
–13.6Z 2 In Paschen/Bracket/P-fund series energy is released in
En = eV/atom
n2 I.R. region.
Alternatively
For helium ion, Z = 2 and for first orbit n = 1.
–13.6 1 1 1
E1 22 = – 54.4 eV We know that =R – ÷ for hydrogen atom
(1) 2 n12 n22
Energy required to remove this e– = + 54.4 eV for e– transition from n2 n1.
Total energy required = 54.4 + 24.6 = 79 eV
1 1 1 7
x 1.1 For n = 4 to n = 3, E µ =R – ÷= R
36. (b) t= = = 5 10 4 9 16 9 16
v 2.2 103
x 1
For 2 to 1, 3 to 2 and 4 to 2 we get more value of w.r.t,
y n = 4 to n = 3,
i.e., more energy than 4 3.
I.R. radiation has less energy than U.V. radiation.
1 2 1
y= gt = 10 (5 10 4 )2 = 1.25 × 10 – 6m 2 K ex
2 2 43. (b) Ep =
32
37. (b) The continuous X-ray spectrum is shown in figure. b2
E x + ÷
2
4

P
min
All wavelengths > min are found, x
12400
where = Å
min V (in volts) A B
+e b/2 0 b/2 +e
Here V is the applied voltage.
38. (d) 15 For maximum EP
max = 3 × 3 × 10 ;
dEP b
3 3 1015 =0 x=
vmax = Hz dx 2 2
2
K.Emax = hvmax – W = 4 eV 44. (a) Number of emitted electrons

1 1 1 8R 1012 2 10 4 25
39. (a) R = = = 5 × 104 electrons
2 2÷ 9 10 5
3 1 1 3
q = + ne = 8 × 10–15 C
1 1 1 3R 27
R 2 2÷
3 1
=
2 1 1 2 4 2 1
32 1 1
45. (a) E = hf = 13.6 Z2 ÷
F I n 22 n12
dP IA h
40. (c) F= = ÷ (1 + r ) P = = (1 + r )
A c
dt hc /
700 IIT-JEE PHYSICS Challenger

dN
1 1 49. (a) N
32 2÷ dt
f2
= 12 2
N dN t
f1 1 1 =
12 2 ÷
dt
1 32 0 N 0
Solving
243 t
f2 = f. N= (1 e ).
32
46. (d) Let N0 be the initial number of nuclei, then
h h
50. (c) = =
N1 = N 0 e t1 mv 2 km
and N2 = N0 e t2 d 1 dk 1
100 100 = 2 = 1%
Number of nuclei decayed = N1 – N2 2 k 2
Hence, A decreases by 1%.
= N 0 (e t1 e t2
)
ln 2
A0 A1 A2 51. (d) Half life, t1/ 2 =
t1 t2
= (e e )= ( A1 A2 ) .
1
47. (a) Given potential energy between electron Mean life, =
r
= eV0 log [ U = eV ] t1/ 2
r0 = ln 2.

d r eV 1 ln 2
10
|F| = eV0 log e = 0 6 20
dr r0 r0 r 52. (c) N = N0 e– /t = 10 e

v 106
= 106 e ln 2 = = 7.07 × 105
2
em
1 1 1
+ 53. (c) R
n12 n22
For limiting wavelength of Lyman series
n1 = 1, n2 =
But this force acts as centripetal force
1
mv 2 eV0 eV =R
mv = 0 2
...(i) L
r rr0 r0
For limiting wavelength of Balmer series
nh n1 = 2, n2 =
By Bohr’s postulate, mvr = ....(ii)
2 1 1 4
=R ÷ B =
From (i) and (ii), B 4 R

n 2 h2 r0 B = 4 L = 4 × 912 Å.
m2 v 2 r 2
= hc
mv 2 4 2
V0 e 54. (a) min = ev0
n2 h2 r0 hc hc
r2 = rµn
4 V0 me 1= ev1
; 2= ev2
48. (b) N = N0 e– t v1
2
= 3 v1
q = q0e–t/RC v2 =
1 4 v2
N t / RC
= constant when e– t = e v2 4 v2 v1 100
q =
v1 3 v1 ÷ 100 = 3
1
= 100
RC Hence, P.D. must be increased by %.
R = 200 . 3
MODERN PHYSICS 701

55. (b) hv = + Kmax hc


hv = 2 + 4 = 6 eV 58. (c) min =
eV
5
Now, h v÷ = 2 + K max h h
3 and de-Broglie = . =
2meV
5 59. (c) The energy level diagram of the atom is shown in the
Kmax = 6 2 Kmax = 8 eV..
3 figure. It is clear that
56. (b) By conservation of linear momentum,
v K K
MV = mv 216 V = 4v V= ...(i)
54
By energy conservation, L
1 2 1
mv + MV 2 = 5.5 × 1.6 × 10–13J
2 2 EK = E K + EL

or, vK = vK + vL
v
V m 1 1 1
M
or, = +
K K L
4 × 1.67 × 10–27 v2 + 216 × 1.67 × 10–27
60. (d) For photon,
v2 E=h
2 5.5 1.6 10 –13
54 54 hc
E =
216
1.67 × 10–27 v 4 +
2
= 2 × 5.5 × 1.6 × 10–13
54 54 hc
2
= ...(i)
E
2 5.5 1.6 54 54 10 –13
v =
2
= 2.586 10 14
1
(4 54 54 216)1.67 10 –27 For proton, E = m v 2p
2
K.E of - particle
2 2
1 m vp p2 p = 2mE
1
= mv 2
1
4 1.67 10 –27 2.586 1014 E= =
2 2 2 m 2m
From de Broglie Eqn.
8.637 10 –13
= 8.637 × 10–13 J = MeV = 5.4 MeV p=
h
1.6 10 –13 1
Alternatively
By conservation of momentum, p1 = p2 h h
1 = = ...(ii)
p 2mE
2 K1m1 = 2 K 2 m2
hc
2 K1 (216) = 2 K 2 (4) 2
= µ E 1/ 2
1 h
K2 = 54K1 ...(i) E
2mE
Also, K1 + K2 = 5.5 MeV ...(ii)
Solve equation (i) and (ii) 61. (b) We write Einstein's equation for the photoelectric effect,
57. (b) 10 emission lines hf – W = EK ... (i)
3hf – W = E'K ... (ii)
final state n = 5
If the initial state were not n = 4, in the emission From (i) and (ii), we get,
E 'K – 3EK = 2W > 0.
spectrum some lines with energies less than that of
absorbed radiation would have been observed.
initial state n = 4.
62. (b) Q = c2 mnuc Z X ( )
A
mnuc ( A
Z 1Y ) m
e+
n=5 By definition.
n=4 Putting,
n=3 mnuc ( X ) m( X )
A
Z
A
Z Zm +
e
n=2
A
n=1 and similarly for Z 1 Y ,
we get the final result.
702 IIT-JEE PHYSICS Challenger
63. (a) Mass defect m = [(1.00813 + 1.00894) ×
dN N
2 – 4.00388] amu 72. (d) 50
m = 0.03026 amu dt 0.5
Binding energy of alpha-particle N t
= (0.03026 × 931) MeV = 28.172 MeV. dN
= dt
64. (a) K depend on Z, hence all wavelength will be same. 0
50 2 N 0
65. (a) Isotopes : A & C have same number of protons.
N = (100 (1 – e–t/2)) = 25
1
66. (c) Energy level µ t = 2 ln (4/3)
n2 16
I transition does not represent emission. 73. (c) 4 42 He 8O

dN A dN B B.E. = m × 931.5 MeV


67. (b) 1N A , 2 1N A 2 NB , = (4 × 4.0026 – 15.9994) × 931.5 = 10.24 MeV
dt dt
74. (c) 3Q P + energy (e)
dN B e = EP – 3EQ
NB = maximum =0
dt 75. (a) Charge conservation is violated in [b, c, d], nucleon
2 conservation is violated in (d), only (a) works.
2 1N A 2 N Bmax N Bmax = 1
NA
2 76. (d) Fission of 235
92 U is possible by only slow neutrons. To
o

2 produce slow neutrons, one has to use moderators so


N Bmax = 1
N0e 1t =2 that fission is possible.
2 77. (c) X-rays create an image of bone on a sheet of
68. (a) The cut off wavelength is given by photographic film because the calcium atoms in bone
hc absorb X-rays much more effectively than the lighter
0 = ...(i) atoms in tissue. The bones cast shadows on the film.
eV
78. (d) Situation of our problem describes so-called Bethe
According to de Broglie equation cycle, first proposed by Hans Bethe in 1939 for stars
h h with the core temperature T > 15 × 106 K. Such
= = temperatures are sufficient to accelerate the main
p 2meV
constituents of stars - hydrogen nuclei, or protons,
which strike the carbon nuclei. The total release of
h2 h2
2
V= ..(ii) energy at the end of one cycle can be calculated as the
2meV 2me 2 total change in rest energy of all particles in the
From (i) and (ii), reactions. Attentive glance at the reactions of this cycle
clarifies that protons involved in the cycle are never
hc 2me 2 2mc 2 the products of reaction. Carbon isotope 12C is input
0 = =
eh2 h nucleus and also the output, and that is why we should
69. (b) For both the atoms the second excited state not take it into account. Other isotopes undergo
corresponds to n = 3. Therefore, the angular momentum transmutations and are not the products of the cycle.
for each of them is 3(h/2 ). The energy, however, is Thus, it is convenient to replace the real series of
proportional to Z2 where Z is the atomic number and reactions by one, as follows:
hence numerical value of energy for hydrogen is less
4p 4
He + 2e+ + 3 2
than that for lithium.
70. (a) Final speed of -particle will be less than the initial Energy released during the formation of one 4 He
speed therefore, nucleus is given by:
E1 = (4mp – mHe –2me+) c2
h h
÷< ÷ or f i
For one mole of helium this energy must be multiplied
f i by the Avogadro’s constant:
71. (a) Use the relation N = N 0 e – t where the decay
E N A c 2 (4 m p m4He 2m + )
constant = (0.693 / Thalf). The half life period e
T half is given to be 10min. The amount N of
23
radioactive sample remaining at the end of 5 min = 6.0221 10 8.9874 1016 (4 1.6726 10 27
27
comes out to be about 0.707g, so that the amount 6.6439 10 2 9.1094 10 31 )
of sample decayed is (1 – 0.707) = 0.293g. = 2.4181 × 1012 J = 2.4181 TJ
MODERN PHYSICS 703

The highest frequency and smallest wavelength for


0.693
79. (b) = infrared region will be for transition of electron from
t to 3rd orbit.
dN
N( 1 2) N 1 1 1
dt =R – ÷
n12 n22
1 2

1 1 1 1 4 1 1
= + = –9 2
– ÷
t t1 t2 3 122 10 3

80. (b) N1 1 N2 2 3 122 9 10 –9


= = 823.5nm
2 N2 2 4
1
= 84. (a) The radioactive decay law may be expressed in different
2 3 N1 3
ways. The simplest formula for it is:
ratio of no. of nuclei of X to that of Y = 3/2
81. (c) The fraction of lost neutrons after 1mm thickness layer N N0 2 t / T
of cadmium is : a = NL/N = 0.15 N0 – number of nuclei of some isotope at the moment
The number of transmitted neutrons is of t = 0;
correspondingly: N – number of nuclei that did not decay through the
N1 = (1 – a) N time interval t;
After the second 1mm-layer N2 neutrons pass: T – half-life time of the isotope. When t = T, N = N0/2.
N2 = (1 – a) N1 = (1 – a)2 N According to our problem we have the same values of
The fraction of neutrons passed through the 10 mm N0 for both isotopes. Assuming that the age of the
cadmium plate (so-called cadmium filter) is: Earth equals the time of decay t, we can calculate the
N10/N = (1 – a)10 = 0.8510 = 0.2 present-day number of nuclei of both isotopes:
82. (d) K.E. of the particle is E. If it goes to a region where it
has now a potential energy V, V has been taken out of N1 N0 2t / T1 .......... (1)
its kinetic energy similar to a body going to a height h
N 2 N 0 2t / T2 .......... (2)
losing kinetic energy and gaining potential energy at
the cost of kinetic energy. At the same time,
N1 1 ( N1 + N 2 ) .......... (1')
h h
de Broglie wavelength, = =
mv 2 Em N2 2 ( N1 + N 2 ) .......... (2')
The new kinetic energy, E' = E – V, First, we should equate (1) to (1') and (2) to (2')

V N0 2t / T1 1 ( N1 + N 2 ) .......... (3)
E E 1 ÷
E
N 0 2t / T2 2 ( N1 + N 2 ) .......... (4)
h This is a system with two unknowns: t and N0. Left
= ;
and right parts of both equations have the same
2mE (1 V / E )
factors. In such case division of (3) by (4) gives the
shortest solution. Division results in
=
(1 V / E )0.5 1 1
t
T2 T1 ÷
83. (b) The smallest frequency and largest wavelength in 2 = 1 ;
ultraviolet region will be for transition of electron from 2
orbit 2 to orbit 1.
1 1
t = log 2 1
÷
1 1 1 T2 T1
=R – ÷
2
n12 n22 Now it is possible to write the solution for unknown
age of the Earth t:
1 1 1 1 3R
=R 2
– 2
= R 1– = 1
–9 4 4 T1T2 log
122 10 m 1 2 ÷
2
t= ;
4 T1 T2
R= m –1
3 122 10 –9 Calculations give t = 5.90 × 109 years.
704 IIT-JEE PHYSICS Challenger

1. (b); 2. (a); 3. (d)


1 1
r = mv/Be N2 = 2N 0 ÷
e e2
nh
mvr = and N3 = N0 – (number of nuclei of A and B at time t = 0)
2
Solving these two equations, we get N0 1 1
= N0 2 N0 ÷
e e e2
nh nhBe
r= and v =
2 Be 2 m2 3 2
N3 = N 0 1 +
e e2 ÷
1 2 nhBe
k= mv =
2 4 m 10. (a) X Y Z

M = iA =
e
T
(r) 2 e
2 r/v
÷ ( r ) = evr2 = 4nehm
2 A0 = X NX (0) = YNY (0)

ln 2 ln 2
where X = , Y =
nBeh 2 1
Now, U = – MB cos 180° =
4 m Putting these values in the given equation for NY, we
4. (b); 5. (a); 6. (c) get, (at t = 0).
A0
dU mvn2 C1 =
F = 2
dr rn
11. (a) From equation (i) given in the passage.
mvn2
m 2 rn = ... (i) 1 A0
rn A0 e Xt e YT
NY =
Y X 2
nh
and mvnrn = ... (ii)
2 A0 2 1
Solving equations (i) and (ii) NY (t = 4) ln 2 4 16
12. (d) =
NY (t = 0) A0
nh
1/ 2
[2 1]
rn = ÷ ln 2
2 m
13. (d), 14. (a), 15. (c)
1 2 nh 235
= 98
+ 136
+ n1 ( 1 e0 ) + n2 (0 n1 ) + E
and KE = mvn = 92 U 42 Mo 54 Xe
2 4
So, n1 = 4 and n2 = 1 1 neutron + 4 electrons
1 1 Now energy liberated in fission process is
Total energy, E = U + KE = m 2 2
rn + mvn2 = 0.
2 2 E = (235.044 – 135.907 – 97.906 – 1.008665– 0.0006)
7. (d), 8. (b), 9. (c) × 931
Rate of disintegration of B at time t is = 206 MeV
16. (a), 17. (a)
dN 2 Rate of formation of nuclei at time t is
1 N1 2 N2
dt
dN
N,
dN2 + 1t dt
2N2 dt = 1 N 0 e
where N = number of nuclei at that time.
Multiplying on both side by e 2t and solving, we get
t
N (1 e )
1 N0 t/ 1 t/ 2
N2 = (e e )
2 1
Number of nuclei disintegrated in time t,

t
2 N0 t/ 1 t/ 2 Nd t (1 e )
N2 = (e e )
1 2
Energy released till time t,
at t = t0
MODERN PHYSICS 705

hc h
E = E0Nd = E0 t (1 e t ) 22. (c) E E = pc p=

20% of this energy is used for heating water, 19


50000eV 50000 1.6 10
p= =
0.2 E0 t (1 e t
) ms 3 108 3 108
23 1
= 2.66 × 10–23 kg-m/s 3 10 kg ms
0.2 E0 t (1 e t ) 23. (d) Power available from X-ray tube is :
= P = VI = 50000 × 0.016
ms
P = 800 Watt
2 1 Since efficiency of X-rays production is 1% therefore
0.2 E0 2 T1/ 2 1 ÷
2 99% of available power is converted into heat
At t = T1/2, =
ms Pheat = 0.99 × 800 W = 99 × 8 W
Pheat = 792 W
0.2 E0 [0.386]
= = 772 K (E0 = 100 ms × 100) 24. (d) For Balmer series, n1 = 2, n2 = 3, 4
ms 25. (c) In transition II, E2 = – 3.4 eV, E4 = – 0.85 eV,
18. (c) The number of 12C atoms in 1g of carbon, E = 2.55 eV
NA 6.022 1023 hc hc
N m N 1 E = = 487 nm.
12 12 E
= 5.02 × 1022 atoms. 26. (d) = 103 nm = 1030 Å
The ratio of 14C/12C atoms = 1.3 × 10–12 (Given) 12400
Number of 14C atoms = 5.02 × 1022 × 1.3 × 10–12 E= 12.0 eV
1030
= 6.5 × 1010
So difference of energy should be 12 eV.
0.693 Hence n1 = 1, 13.6 eV ; n2 = 3, –1.51 eV
Rate of decay R0 = N0 = T N0
1/ 2 27. (a), 28. (c)
The Einstein’s formula for the photoelectric effect gives
0.693 6.5 1010 the following relationship among the frequency of a
R0 = photon f, the work function of the metal , and the
5730 365 24 3600
maximum kinetic energy of the ejected electron Kmax:
= 0.25 Bq = 0.25 (decays/s) hf = + Kmax ............. (1)
where h is the Planck’s constant (h = 6.63 × 10–34 J·s).
19. (c) For 10g sample, number of decays = 0.5 per second. If f < fcut, where fcut is called the cutoff frequency, the
i.e. R = 0.05 and R0 = 0.25 for each gram of 14C photoelectric effect is impossible. So, fcut = / h.
When the photon source moves toward the metal, the
R 1 ln ( R0 / R) ln ( R0 / R)
e t
t= = incident photons are Doppler shifted to higher
R0 1 (0.693 / T1/ 2 ) frequencies that exceed fcut and make the photoelectric
effect possible.
5730 years 0.25
t ln ÷ = 13310 years If v = 0.280c, the frequency f '= fcut .
0.693 0.05
1+ v / c
20. (d) If there are no other radioactive ingredients, the sample f = f
is very recent. But the error of measurement must be 1 v/c
high unless the statistical error itself is large. In any = (7.00 × 1014) (1.33) = 9.33 × 1014Hz
case, for an old sample, the activity will not be higher Therefore, = (6.63 × 10–34 Js) × (9.33 × 1014 Hz)
than that of a recent one. = 6.18 × 10–19 J = 3.87 eV.
21. (b) The current = 2.566 – 2.55 = 0.016 At v = 0.900 c, f ' = 3.05 × 1015 Hz; and
0.016 Kmax = hf ' –
No. of e– = 19
= 1017 e
1.6 10
(6.63 10 34 Js) (3.05 1015 Hz)
for every 100 e–(s) 1 photon is produced = 3.87 eV
(1.60 × 10 19 J/eV)
1017 = 8.78eV
No. of X-ray photons = = 1015
100
706 IIT-JEE PHYSICS Challenger

1. (b) The photosensitivity of a metal is high when its work particle it should be made to collide with a body of
function is small. Work function of a metal depends almost equal mass. Ordinary water consisting of
not on the threshold frequency but on the nature of protons should be the best moderator but the proton
the metal. neutron collision results in the production of deutron
2. (d) Half life of radioactive substance is 40 days. It means with the emission of -particle as follows.
50% substance decays in 40 days. During this period 1 1 2
rate of decay is on decrease. So, 25% decay must have 0n 1p 1H
taken place is less than 20 days. This is undesirable reaction as it results in loss of
n neutron. Hence heavy water is preferred.
1 time elapsed
N = N0 ÷ , where n = half life period
So reason assigned is correct.
2 10. (c) Photoelectric effect demonstrate the particle nature of
3. (d) Isotope of an element can be separated by using a light.
mass spectrometer because isotopes have different The number of photoelectrons is proportional to the
atomic mass. intensity of light.
4. (d) Intensity of incident photon decides the number of 11. (a) Binding energy per nucleon decreases with A for nuclei
electrons ejected and not the kinetic energy. with atomic mass number A > 100 due to weak nuclear
Ejection of electron from metallic surface is possible forces. It can be explain as :
only when frequency of incident photon is more than At short distances, the nuclear force is stronger than
threshold frequency. the Coulomb force; it can overcome the Coulomb
h repulsion of protons inside the nucleus. At typical
5. (b) We know that for photon , mv = nucleon separation (1.3 fm) it is a very strong attractive
mass varies inversely as the wavelength. For particle force (104 newtons). Beyond about 1.3 fm separation,
E = mc 2, E is energy of particle if its mass is the force exponentially dies off to zero. However, the
converted into energy. Both are uncorrelated. Coulomb force between protons has a much larger
6. (a) The reason given is true. If we test the authenticity of range and becomes the only significant force between
assertion, protons when their separation exceeds about (2.5 fm,
A decay A 4 rays A > 100).
ZX Z 2X
12. (b) The wavelength of characteristic X-rays depends on
A 8 (2 rays) A 8
Z 4X Z 2X the type of atoms of which the target material is made.
7. (b) According to classical physics all moving electrons It does not depend on the accelerating potential.
around the nucleus will radiate because an accelerated Therefore, statement I is true.
electron in electric field will radiate e.m. wave. So reason When an electric beam strikes the target in an X-ray
is true. But reason does not explain the assertion. tube, part of the kinetic energy is converted into X-ray
Bohr's postulates that the electron in stationary orbits energy. This statement is true. But statement 2 does
do not radiate. His postulate was based not out of not explain statement 1.
compulsion but on the quantum theory. So assertion is 13. (a) hf1 = 0 – E2 and hf2 = 0 – E3
also true but reason does not explain assertion. h (f1 – f2) = E3 – E2
8. (a) Total binding energy of fragment nucleus is more than Statement – 1 is true and Statement – 2 is true.
total binding energy of parent nucleus. Since binding and Statement – 2 correctly explains Statement – 1.
energy results in decrease of total energy. Hence there 14. (d) Statement – 1 is false, the penetration power depends
is great decrease in energy fragment nucleus because upon accelerating potential.
energy is released in nuclear fission. Statement – 2 is true, increasing current increases the
A B + E temperature of filament causing it to emit more electrons.
( Parent ) ( Fragment ) ( Energy )
Energy of B is decrease but the binding energy of B is hc
increased due to release of energy from it. So reason 15. (a) KE = eV = h max =
min
supports the assertion.
9. (a) Heavy water is used to slow down the neutron. Fast hc 12400
neutrons are incapable of transforming their energy to min =
= = 1.24 Å (X-rays)
eV 10 103
the parent nuclei. When two bodies of the same mass
X-rays are highly penetrating radiation.
make collision exchange of velocity is most and when
16. (a) Net force on proton is less than neutron hence it is
two bodies of different masses collide there is least
easy to remove a proton.
exchange of velocities. So, to reduce the velocity of a
MODERN PHYSICS 707

1. (b, c) = hv – kmax
EK = 16.3 keV 2
Z gas = 1000, Z gas = 32
and kmax = |eVs|
2. (a, b) The mass number decreases by 32 hence If the moment of the electrons is doubled, then
8 -particles and 6 -particles are produced. the K.E. is increased by 4 times
The minimum energy of the X-rays is
92 U
238
82 Pb
206
+ 8 2 He 4 + 6 1
0
(16.3 + 3 × 2.7) keV = 24.5 keV
3. (b, d) Shortest wavelength means highest frequency.
This means highest energy. The energy of X-rays Since, EK ( Z 1)2
depends on the accelerating voltage provided in
the X-ray tube. 2
Z 1 24.5
= ÷ Z = 50
Also according to Moseley’s law v = a ( Z – b) . 40 16.3
Thus the frequency also depends on the atomic The no. of X-ray photons incident
number.
4. (a, c, d) 100 10 3 1019
= 100 mW/m2 = 3
per m 2
dN dN 1.6 24.5 10
(a) N0 e t µe t
No. of ions produced in 1 sec
dt dt
For an interval of one second 100 10 3 10 2 1019
= per m3 850 per m3
dN 1.6 24.5 103 3 108
µ e which is a constant
dt 9. (a, b, c) We know that the Q value of the reaction
(c) Since the quantity is large. E0 = Ev + E
(d) The disintegration energy of uranium is of the Ev = E0 – E = 0.155 – 0.025 = 0.130 MeV
order of MeV but the energy of the emitted -
particle is of the order of eV.
1 1 1 3
5. (c, d) RZ 2 2 2÷
= RZ 2
K 1 2 4
135°
1 eVA
=
0 hc Recoil nucleus
6. (c, d) The threshold wavelength is 5200Å. For ejection
of electrons, the wavelength of the light should Hence the momentum of the neutrino
be less than 5200 Å so that frequency increases pv = Ev/c = 0.13 MeV/c
and hence the energy of incident photon increase. Momentum of a -particle can be obtained from
U.V light has less wavelength than 5200 Å. the relation
7. (a, b) It is clear that half-life of nuclei is 10 seconds.
Mean life of the nuclei is p = (2 M e E ) = 2 0.511 0.025 MeV/c
T1/2 10 = 0.158 Me V/Vc
Tmean = = = 14.43 s
0.693 0.693 Using conservation of momentum of the system,
we have
1
6.25% of original number means of the original p sin 45° = pv sin
16
1 p
number = 4
of original number.. or sin = sin 45° = 0.859
(2) pv
Hence time taken is 4 × T1/2 = 40 sec.
and pR = p cos 45° + pv cos
8. (a, b, c, d) The wavelength of the K radiation from Nb is
4hc 1 4hc 1 2
= = = 0.158 (1/ 2) 0.13 [1 (0.859) ]
3R ( Z 1) 2 3R 402 = 0.179 MeV/c
708 IIT-JEE PHYSICS Challenger
10. (a, b) Let q be the charge on the capacitor after a time t. Dividing Equation (iv) and (ii)
2 2
TB h2 2m A
= A
2
A TA 2m B h2 2
B
R
C 2 2
TA – 1.5 1
= A
= A
=
TA 2 2 4
2 A 4 A

Then, we can write [ B 2 A given]


4TA – 6 =TA TA = 2eV
dq 1 q
+ ÷ = i (t ) ...... (1) From (i) WA = 2.25 eV
dt R C
From (iii) WB = 4.2 eV
where i (t)
Also TB = TA – 1.5 TB = 0.5eV
= ( N0 e t ) 2e. f (2e. f N0 ) e t = i0e t .. (2) 13. (a, b) The number N of nuclei with the half-life T
Solving eq. (1), we get remaining in time interval t is
t 1 t
ie
q= 0 + C1e RC , N = N0 2 T ......... (1)
1
RC where N0 is the number of nuclei at an instant
where C1 is an arbitrary constant. t = 0. Equation 1 is called the law of radioactive
Using the initial condition, we get, decay. The logarithmic form of eq.1 is following:
t
ln N ln N 0 ln 2 .......... (2)
1 T
i0 ÷ i0
q(t ) + q0 ÷ e RC + e t
To find a relationship between the number N and
1 1
÷ activity R let us write the definition of activity:
RC RC
t dN
= q1e RC + q2 e t R=
dt
The total charge on the capacitor becomes zero at Applying the implicit differentiation to equation
a time 2, we obtain
t = , where q ( ) = 0
dN ln 2
dt
1 q N T
0 q1e RC + q2 e t
or = ln 1 ÷
1 q2
RC dN ln 2
N
dt T
hc 1 1
11. (a, d) Vs ÷ , saturation current depends t t
e ln 2
0
= N0 2 T = R0 2 T ......... (3)
only on intensity of light. T
12. (a, b, c) For metal A, 4.25 = WA + TA ...(i) where R0 = N0 ln2/T is the activity at an instant of
time t = 0. Taking the logarithm of equation 3 we
1
Also T A = mv A2 obtain
2
ln 2
ln R ln R0 t
1 m 2 v 2A p 2 h2 T
= = A ...(ii)
2 m 2 m 2 m 2A which is the equation of a straight line with slope –
ln 2/T.
h
= The logarithmic plot shown in Figure is fitted
p by ln R = 8.4 – 0.25t.
For metal B, 4.7 = (TA – 1.5) + WB ...(iii) If t is measured in minutes, then slope is 0.25 per
minute. The half–life is
h2
Also TB = ...(iv) [as Eq. (ii)] ln 2
2
2m B T= min = 2.8 min
0.25
MODERN PHYSICS 709

14. (a, c) 1 : 2 : 3
= eV01 : eV02 : eV03 Comparing equation (i) by y = mx + c, we get the
hc
V0 slope of the line m = = tan
e
Metal 1 Metal 2 Metal 3 Option (c) is correct.
From the graph it is clear than,
1 2 3
1
–1
1/ (nm ) = 0.001nm –1 =
1
= 1000mn
01
0.001 0.002 0.004 01 0.001

= V = V01 : V02 : V03 = 0.001: 0.002 : 0.004 1


Also = 0.002nm –1 02 = 500nm
= 1: 2 : 4 02
Therefore option (a) is correct and 02 = 250nm
hc Violet colour light will have wavelength less than
By Einstein’s photoelectric equation. – = eV 400 nm.
Therefore this light will be unable to show photo
hc electric effect on plate, 3 Option (d) is wrong.
V= – ...(i)
e e

1. A-p, q; B-p, r; C-p, s; D-p, q, r 4. A-r; B-p; C-q, s; D-q


A. In a nuclear fusion reaction matter is converted into 5. A-p; B-q; C-r; D-s
energy and nuclei of low atomic number general give 6. A-s; B-p, q; C-p, q; D-p, r, s
this reaction.
7. A-p, q, r, s; B-p, q, r, s; C-q, r; D-p
B. In a nuclear fission reaction matter is converted into
8. A-r; B-s; C-q; D-p
energy and nuclei of high atomic number generally
given this reaction. 9. A-p, r; B-q, s; C-q, t; D-q, s
C. Self explanatory. 0i q e
D. Self explanatory. (A) B = where ieq = =
2 r T 2 r /v
2. A-p, r; B-q, s; C-p; D-q
3
A-p, r 0e v v ( Z / n) Z
B= µ µ µ
2 2 2 2 2 5
Reason : Characteristic X-ray are produced due to transition 4 r r (n / Z ) n
of electrons from one energy level to another.
(B) Magnetic moment,
Similarly the lines in the hydrogen spectrum is obtained due
to transition of electrons from one energy level to another. q e n2 Z
M = iA = ( r2 ) r 2 µ rv µ ÷ ÷
B-q, s T 2 r /v Z n
Reason : In photoelectric effect electrons from the metal
surface are emitted out upon the incidence of light of h 1 n
appropriate frequency. (C) = µ µ
mv Z / n Z
In b-decay, electrons are emitted from the nucleus of an
atom. L nh / 2
(D) Areal velocity = = µn
C-p 2m 2
Moseley gave a law which related frequency of emitted X- 10. A-p; B-p; C-r
ray with the atomic number of the target material For K capture : Q = (mx – my) c2 ,
v = a(Z – b) For – decay : Q = (mx – my) c2 ,
D-q For + decay : Q = (mx – my – 2me) c2
In photoelectric effect, energy of photons of incident ray 11. A-q, s; B-r; C-p; D-p, q, r, s
gets converted into kinetic energy of emitted electrons.
1
3. A-p, r; B-q, s; C-s; D-p, r Moseley’s law is for characteristic X-ray. min µ
V
710 IIT-JEE PHYSICS Challenger

1. 1.29
hc
2
1H + 1H
2 3
1H + p But E = hv =
2 3 4
1H + 1H 2He + n
Net Reaction 31H2 2He4 + p + n
hc 6.6 10 –34 3 108
m = 3 (2.014) – [4.001 + 1.007 + 1.008] = 0.026 = = = 0.1142 × 10–7
3 deuterons release 3.87 × 10–12 J E 108.8 1.6 10 –19
3.87 10 –12 10 40 = 1.142 × 10–8m = 114 Å
10th deuterons release = Alternatively
3
= 1.29 × 1028J
12400 12400
E 1.29 1028 = Å = = 114Å
E E (in eV) 108.8
P= t= = = 1.29 1012 = 1.29 Ts
t P 1016
2. 16.5 4. 14.5
For one electron species, the energy of nth orbit is given by The transition state of six different photon energies are
shown.
13.6
En = Z2 eV/atom.
n2
n = 4 [– 0.85 eV]
Increasing Energy
[0.65 eV]
13.6 2 13.6 2 n = 3 [– 1.5 eV]
E2 = – Z , E3 = – Z
4 9 [+ 1.9 eV]
n = 2 [– 3.4 eV]
1 1
E3 – E2 = –13.6 Z2 – ÷
9 4 [10.2 eV]
n = 1 [– 1.36 eV]
4 – 9 = + 13.6 5 Z2
= –13.6 Z2
36 36 Since after absorbing monochromatic light, some of the
But E3 – E2 = 47.2eV = (Given) emitted photons have energy more and some have less than
2.7 eV, this indicates that the excited level B is n = 2. (This is
13.6 5 2
Z = 47.2 because if n = 3 is the excited level then energy less than 2.7
36 eV is not possible).
47.2 36 For hydrogen like atoms we have
Z= =5
13.6 5 –13.6
En = Z 2 eV/atom
–13.6 2 n2
E4 = Z
16 –13.6 2 –13.6 2
E4 – E2 = Z – ÷ Z = 2.7
1 1 9 – 16 16 4
E4 – E3 = –13.6 Z = –13.6 Z2
2

16 9 9 16
1 1
= 16.5 eV Z 2 13.6 – = 2.7
3. 114 4 16

13.6 2 2.7 4 16
En = – Z eV/atom Z2
n2 13.6 12

–13.6 9 1 1
For Li2+, Z = 3 En = eV/atom I.E. = 13.6Z
2

2 2

n 1
13.6 9
E1 = – 13.6
2.7 4 16
= 14.5 eV
1 13.6 12
13.6 9 5. 2.55
and E3 = – = –13.6
9 The energy of photon causing photoelectric emission
E = E3 – E1 = – 13.6 – (–13.6 × 9) = Work function of sodium metal + KE of the fastest
= 13.6 × 8 = 108.8 eV/atom photoelectron
= 108.8 × 1.6 × 10–19 J/atom = 1.82 + 0.73 = 2.55 eV
MODERN PHYSICS 711

6. 6
For hydrogen like atoms Z2
40.8 = 40.8 4n2 = Z2 or 2n = Z ....(ii)
13.6 4n 2
En = – 2
Z 2 eV/atom From (i) and (ii)
n
Given En – E2 = 10.2 + 17 = 27.2eV ..(i) 1
204 = 13.6Z 2 1– ÷ = 13.6 Z –13.6
2
En – E3 = 4.24 + 5.95 = 10.2 eV Z2
E3 – E2 = 17 13.6Z2 = 204 + 13.6 = 217.6
13.6 2 13.6 2
But E3 – E2 = – Z – – Z ÷ Z2 =
217.6
= 16, Z = 4.
9 4 13.6
1 1 9. 38.4
= –13.6Z 2 –
9 4 The formula for of power will be

4 – 9 13.6 5 2 Pour
= –13.6Z 2 = Z =
36 36 Pin

13.6 5 2
Z = 17 Z=3 P 1000 106
36 Pin = out = = 1010 W
0.1
13.6 2 13.6
En – E2 – 3 – – 2 32 Energy required for this power is given by
n2 2
E
P=
9 9 4 – n2 t
= –13.6 – = –13.6 9 ...(ii)
n2 4 4n 2 E = P × t = 1010 × 86,400 × 365 × 10 = 3.1536 × 1018 J
From eq. (i) and (ii) 200 × 1.6 × 10–13 J of energy is released by 1 fission
= 3.1536 × 1018 J of energy is released by
4 – n2
–13.6 9 = 27.2 .1536 1018
4n 2 = 0.9855 × 1029 fission
– 122.4 (4 – n2) = 108.8n2 200 1.6 10 –13
= 0.985 × 1029 U235 atoms.
489.6
n2= = 36 0.9855 × 1029 atoms of Uranium has
13.6
n=6 235 0.9855 1029
7. 14.4 = 38.451 × 106g
From the given information, it is clear that half life of the 6.023 1023
radio active nuclei is 10 sec (since half the amount is = 38.451 × 103 kg = 38451 kg = 38.451 Mg 38.4 Mg
consumed in 10 second. 12.5% is half of 25% pl. note). 10. 3
Mean life If x is the difference in quantum number of the states than
1 1 t 10 x+1C = 6 x=3
= = = 1/ 2 = = 14.4 sec. 2
0.693/ t1/ 2 0.693 0.693
n+3
8. 4
13.6Z 2
Energy for an orbit of hydrogen like atoms is E n = –
n2
For transition from 2n to 1 n
Smallest
1 1
Maximum energy = 13.6Z
2
– ÷
1 (2n)2
– Z 2 (13.6eV)
Now, we have = – 0.85eV ...(i)
1 1 n2
204 = 13.6Z 2
1 4n 2 ÷
– ...(i)

Also for transition 2n n. – Z 2 (13.6eV)


and = –0.544 eV ...(ii)
(n + 3)2
1 1 3
40.8 = 13.6 Z 2 – ÷ 40.8 = 13.6 Z 2 ÷ Solving (i) and (ii), we get Z = 3.
n2 4n 2 4n 2
712 IIT-JEE PHYSICS Challenger
11. 5 Z
Number of electrons falling on the metal plate A
= 1016 × (5 × 10–4) 2 (0.529 10 –10 )( n + 1)

A B 1
1.09 107 Z 2 1 –
(n + 1)2
on solving, we get n = 24
Alternative

14. 34
Let e– in hydrogen atom is excited to nth level.
EKE (n –1) = 8 | EP.E.(n) |
d = 1 cm
13.6
13.6 eV 8| 2 eV | n=4
Number of photoelectrons emitted from metal plate A upto n2
10 seconds is 1
E = 13.6 1 – 15
4 16 ÷ 13.6eV = 12.75eV
(5 10 ) 10 16 16
ne 6
10 = 5×107
10 Using conservation of linear momentum
mv = mv1 + mv2 ...(1)
But ne = x ´107 Þ x = 5
( v2 – v1 ) = 2 v
1
12. 42 ...(2)
According to Bohr’s model, the energy released during
transition from n2 to n1 is given by v 3v
v1 = , v2 = ....(3)
4 4
1 1
E = hv = Rhc ( Z – b)2 – H-atom
n12 n22 neutron v
0 v1
For transition from L-shell to K-shell v2
m m
b = 1, n2 = 2, n1 = 1 After collision
Before collision
1 1
– = hv
(Z –1)2 Rhc By energy conservation
1 4
1 2 1 2 1 2
On putting the value of R = 1.1 × 10 7 m–1 (given), mv = mv1 + mv2 + E
c = 3 × 108 m/s 2 2 2
Z = 42 1 2 1 9
13. 24 mv 1 – – ÷ E
2 16 16
nth line of lyman series means electron jumping
from (n + 1)th orbit to 1st orbit for an electron to 1 2 16 8
mv E 12.75eV = 34 eV
move in (n + 1)th orbit. 2 6 3
2 r = (n + 1) K.E. of neutron = 34 eV.
2 2 15. 0.283
–10 (n + 1)
2
r= 0.529 10 If product nucleus 198Hg would had been in ground state,
(n + 1) (n + 1) Z the K.E. available to e– and antineutrino must have been
1
=
Z ( n + 1)2 a = (mAu – mHg) × 931 MeV = 1.3714 MeV
..(i) Since 198Hg is in excited state, actual K.E. available to
2 0.529 10 –10 Z
e – and antineutrino
Also we know that when electron jumps from (n + 1)th orbit K = (1.3714 – 1.088) MeV = 0.283 MeV
to 1st orbit. 16. 6
According to conservation of energy
1 1 1
= RZ 2 – hc hc 1 1
2
1 (n + 1)2 + = RhcZ2 2

1 2 1 n2
1
1.09 107 Z 2 1 – ...(ii)
1
+
1
= RZ 2 1 –
1
(n + 1)2 1 2 n2
From (i) and (ii)
1 1 2 1
2
1–
n 1 2 RZ 2
1/n2 = 0.0284 n = 5.93 = 6

You might also like